You are on page 1of 658

PREFACE

This book is designed for the aspirants of KVPY. As there is no prescribed syllabus for

KVPY, so complete care has been taken to cover the topics which are asked in previous

year test papers of KVPY. It is structured in a way to prepare an aspirant of KVPY


through detailed theory of topics followed by exercise covering the pattern of questions

asked in exam & thereby giving good exposure to the students. The complete solution

of exercise is provided in book itself. Each topic is followed by an exercise of questions

which are asked from that topic in previous years with answer key.

Complete care has been taken in designing this book to provide the material which

meets aspirant's expectation & requirement for target exam. But, your feedback is

valuable for us to improve this book even more. You can mail your feedback of this
book at dlpd@resonance.ac.in

Wishing you an interesting & satisfying experience of this book & All The Best for

Target Exam.

© Copyright reserved.
All rights reserved. Any photocopying, publishing or reproduction of full or any part of this study material is strictly prohibited. This material belongs to
enrolled student of RESONANCE only any sale/resale of this material is punishable under law, subject to Kota Jurisdiction only.
19RDLP
PHYSICS (KVPY)
KVPY (CLASS-XI # SA STREAM)

INDEX
Topic Name Page No.

RECTILINEAR MOTION 001 - 028

PROJECTILE MOTION 029 - 046

RELATIVE MOTION 047 - 069

NEWTON’S LAWS OF MOTION 070 - 101

FRICTION 102 - 120

WORK, POWER & ENERGY 121 - 160

CENTER OF MASS 161 - 206

RIGID BODY DYNAMICS 207 - 249

SIMPLE HARMONIC MOTION 250 - 274

FLUID MECHANICS 275 - 302

KTG & THERMODYNAMICS 303 - 340

CALORIMETRY & THERMAL EXPANSION 341 - 363

ELECTROSTATICS 364 - 411

CURRENT ELECTRICITY 412 - 455

GEOMETRICAL OPTICS 456 - 503

EMF & EMI 504 - 531

NUCLEAR PHYSICS 532 - 554

CIRCULAR MOTION 555 - 571

STRING WAVE 572 - 585

SOUND WAVE 586 - 604


ELASTICITY & VISCOSITY 605 - 617

SURFACE TENSION 618 - 628

UNIT & DIMENSION 629 - 650

© Copyright reserved.
All rights reserved. Any photocopying, publishing or reproduction of full or any part of this study material is strictly prohibited. This material belongs to enrolled student of
RESONANCE only. Any sale/resale of this material is punishable under law, subject to Kota Jurisdiction only.

(i)
PHYSICS (KVPY)
KVPY (CLASS-XI # SA STREAM)

CONTENTS

Topic Name Page No.

RECTILINEAR MOTION
Theory 001 - 018
Exercise 019 - 025
Answer key 026 - 026
Solution 026 - 028

PROJECTILE MOTION
Theory 029 - 038
Exercise 039 - 042
Answer key 042 - 042
Solution 043 - 046

RELATIVE MOTION
Theory 047 - 061
Exercise 062 - 065
Answer key 065 - 065
Solution 066 - 069

NEWTON’S LAWS OF MOTION


Theory 070 - 091
Exercise 092 - 097
Answer key 098 - 098
Solution 099 - 101

FRICTION
Theory 102 - 112
Exercise 113 - 117
Answer key 117 - 117
Solution 117 - 120

WORK, POWER & ENERGY


Theory 121 - 149
Exercise 150 - 155
Answer key 155 - 155
Solution 156 - 160

CENTER OF MASS
Theory 161 - 190
Exercise 191 - 198
Answer key 199 - 199
Solution 199 - 206

(ii)
CONTENTS

Topic Name Page No.

RIGID BODY DYNAMICS


Theory 207 - 243
Exercise 244 - 249
Answer key 249 - 249

SIMPLE HARMONIC MOTION


Theory 250 - 264
Exercise 265 - 270
Answer key 270 - 270
Solution 271 - 275

FLUID MECHANICS
Theory 276 - 291
Exercise 292 - 299
Answer key 299 - 299
Solution 300 - 303

KTG & THERMODYNAMICS


Theory 304 - 327
Exercise 328 - 337
Answer key 337 - 337
Solution 337 - 341

CALORIMETRY & THERMAL EXPANSION


Theory 342 - 356
Exercise 357 - 362
Answer key 363 - 363
Solution 363 - 365

ELECTROSTATICS
Theory 366 - 397
Exercise 398 - 407
Answer key 407 - 407
Solution 407 - 413
CURRENT ELECTRICITY
Theory 414 - 442
Exercise 443 - 452
Answer key 452 - 452
Solution 452 - 457

(iii)
CONTENTS

Topic Name Page No.

GEOMETRICAL OPTICS
Theory 458 - 488
Exercise 489 - 500
Answer key 501 - 501
Solution 501 - 506

EMF & EMI


Theory 507 - 520
Exercise 521 - 529
Answer key 530 - 530
Solution 530 - 534

NUCLEAR PHYSICS
Theory 535 - 549
Exercise 550 - 554
Answer key 555 - 555
Solution 555 - 557

CIRCULAR MOTION
Theory 558 - 573
Exercise 574 - 574
Answer key 574 - 574

STRING WAVE
Theory 575 - 588

SOUND WAVE
Theory 589 - 606
Exercise 607 - 607
Answer key 607 - 607

ELASTICITY & VISCOSITY


Theory 608 - 619
Exercise 620 - 620
Answer key 620 - 620

SURFACE TENSION
Theory 621 - 631

UNIT & DIMENSION


Theory 632 - 651
Exercise 652 - 653
Answer key 653 - 653

(iv)
RECT I LI N EAR M OT I ON
M ECH A N I CS
Mechanics is the branch of physics which deals with the cause and effects of motion of a
particle, rigid objects and deformable bodies etc. Mechanics is classified under two streams
namely Statics and Dynamics. Dynamics is further divided into Kinematics and Kinetics.

1. M OTI ON AN D R EST
Motion is a combined property of the object and the observer. There is no meaning of rest or
motion without the observer. Nothing is in absolute rest or in absolute motion.
An object is said to be in motion with respect to a observer, if its position changes with
respect to that observer. It may happen by both ways either observer moves or object moves.

2. R ECTI LI N EAR M OTI ON


Rectilinear motion is motion, along a straight line or in one dimension. It deals with the
kinematics of a particle in one dimension.

2 .1 P osi t ion
The position of a particle refers to its location in the space at a certain moment of time.
It is concerned with the question  “where is the particle at a particular moment of time?”

2 .2 D i sp l a c e m e n t
The change in the position of a moving object is known as

displacement. It is the vector joining the initial position ( r1 ) of r2

the particle to its final position ( r2 ) during an interval of time.
Displacement can be negative positive or zero.

2 .3 D i st a n ce
The length of the actual path travelled by a particle during a given time interval is called
as distance. The distance travelled is a scalar quantity which is quite different from
displacement. In general, the distance travelled between two points may not be equal to
the magnitude of the displacement between the same points.

KVPY_ SA STREAM # 1
Example 1.
Ram takes path 1 (straight line) to go from P to Q and 2

Shyam takes path 2 (semicircle).


P Q
(a) Find the distance travelled by Ram and Shyam? 1
100 m

(b) Find the displacement of Ram and Shyam?


Solution :
(a) Distance travelled by Ram = 100 m
Distance travelled by Shyam = (50 m) = 50 m
(b) Displacement of Ram = 100 m
Displacement of Shyam = 100 m


2 .4 Ave ra ge Ve locit y (in an interval) :
The average velocity of a moving particle over a certain time interval is defined as the
displacement divided by the lapsed time.

displacement
Average Velocity =
time int erval
for straight line motion, along xaxis, we have

x x f  xi
v av = v = <v> = =
t t f  ti
The dimension of velocity is [LT -1] and its SI unit is m/s.
The average velocity is a vector in the direction of displacement. For motion in a straight
line, directional aspect of a vector can be taken care of by +ve and -ve sign of the
quantity.
2 .5 In st a nt a n e ou s Ve locit y (at an instant) :
The velocity at a particular instant of time is known as instantaneous velocity. The term
“velocity” usually means instantaneous velocity.

 x  dx
V inst. = lim   =
t  0  t  dt

In other words, the instantaneous velocity at a given moment (say , t) is the limiting
value of the average velocity as we let t approach zero. The limit as t  0 is written
in calculus notation as dx/dt and is called the derivative of x with respect to t.

N ot e :
 The magnitude of instantaneous velocity and instantaneous speed are equal.

 The determination of instantaneous velocity by using the definition usually involves


dx
calculation of derivative. W e can find v = by using the standard results from
dt
differential calculus.
 Instantaneous velocity is always tangential to the path.

KVPY_ SA STREAM # 2
Example 2. A particle starts from a point A and travels along the solid curve shown in figure. Find approximately
the position B of the particle such that the average velocity between the positions A and B has the
same direction as the instantaneous velocity at B.

Answer : x = 5m, y = 3m
Solution : The given curve shows the path of the particle starting at y = 4 m.
displacement
Average velocity = time taken ; where displacement is straight line distance between points
Instantaneous velocity at any point is the tangent drawn to the curve at that point.

Now, as shown in the graph, line AB shows displacement as well as the tangent to the given curve.
Hence, point B is the point at which direction of AB shows average as well as instantaneous velocity.

2 .6 Ave r a g e Sp e e d ( i n a n i n t e r va l)
Average speed is defined as the total path length travelled divided by the total time
interval during which the motion has taken place. It helps in describing the motion along
the actual path.
distance travelled
Average Speed =
time interval
The dimension of velocity is [LT -1] and its SI unit is m/s.

N ot e :
 Average speed is always positive in contrast to average velocity which being a vector,
can be positive or negative.
 If the motion of a particle is along a straight line and in same direction then,
average velocity = average speed.
 Average speed is, in general, greater than the magnitude of average velocity.

KVPY_ SA STREAM # 3
Example 3. In the example 1, if Ram takes 4 seconds and Shyam takes 5 seconds to go from P to Q, find
(a) Average speed of Ram and Shyam?
(b) Average velocity of Ram and Shyam?
100
Solution : (a) Average speed of Ram = m/s = 25 m/s
4
50
Average speed of Shyam =
5 m/s = 10 m/s
100
(b) Average velocity of Ram = m/s = 25 m/s (From P to Q)
4
100
Average velocity of Shyam = m/s = 20 m/s (From P to Q)
5
Example 4. A particle travels half of total distance with speed v 1 and next half with speed v 2 along a
straight line. Find out the average speed of the particle?
Solution : Let total distance travelled by the particle be 2s.
s
Time taken to travel first half =
v1
s
Time taken to travel next half =
v2

Total distance covered 2s 2v1v2


Average speed = = = (harmonic progression)
Total time taken s s v1  v 2

v1 v 2

Example 5. A person travelling on a straight line moves with a uniform velocity v 1 for some time and with
uniform velocity v 2 for the next equal time. The average velocity v is given by
v1 v 2
Answer : v (Arithmetic progression)
2

Solution :
As shown, the person travels from A to B through a distance S, where first part S 1 is travelled
in time t/2 and next S 2 also in time t/2.
S1 S2
So, according to the condition : v 1 = and v 2 =
t/2 t/2

v1 t v 2 t
S1  S 2 
Average velocity =
Total displaceme nt
= = 2 2 = v1  v 2
Total time taken t t 2


2 .6 Ave r a g e a cce le r a t i on ( i n a n i n t e rva l) :
The average acceleration for a finite time interval is defined as :
change in velocity
Average acceleration =
time interval
Average acceleration is a vector quantity whose direction is same as that of the change
in velocity.
  
 v v f  vi
a av = =
t t

KVPY_ SA STREAM # 4
Since for a straight line motion the velocities are along a line, therefore

v v f  vi
a av = =
t tf  ti
(where one has to substitute v f and v i with proper signs in one dimensional motion)

2 .8 I n st a n t a n e o u s Acce l e ra t i o n ( a t a n in st a n t ) :
The instantaneous acceleration of a particle is its acceleration at a particular instant of
time. It is defined as the derivative (rate of change) of velocity with respect to time. W e
usually mean instantaneous acceleration when we say “ acceleration”. For straight motion
we define instantaneous acceleration as :

 
dv  v   dv  v 
a = = lim   and in general a = dt = t 0  t 
lim
dt t 0  t   
The dimension of acceleration is [LT -2] and its SI unit is m/s 2.

3. GR AP H I CAL I N TER P R ETATI ON OF S OM E QUAN TI TI ES


3 .1 Ave r a ge Ve l o cit y
If a particle passes a point P (x i) at time t = t i and reaches Q (x f ) at a later time instant

x xf  xi
t = t f , its average velocity in the interval PQ is V av = =
t tf  ti

This expression suggests that the average velocity is equal xf


Q
t o t he sl ope of t he l i ne (chord) j oi ni ng t he poi nt s
xi
corresponding to P and Q on the xt graph. P

O ti tf t
3 .2 I n st a n t a n e o u s Ve l o ci t y

Consider the motion of the particle between the two points P and Q on the xt graph
shown. As the point Q is brought closer and closer to the point P, the time interval
between PQ (t, t , t,......) get progressively smaller. The average velocity for each
time interval is the slope of the appropriate dotted line (PQ, PQ, PQ......).

As the point Q approaches P, the time interval approaches


zero, but at the same time the slope of the dotted line
approaches that of the tangent to the curve at the point P.
As t  0, V av (=x/t)  V inst.
Geometrically, as t  0, chord PQ  tangent at P.

Hence the instantaneous velocity at P is the slope of the tangent


at P in the x  t graph. W hen the slope of the x  t graph is
positive, v is positive (as at the point A in figure). At C, v is negative
because the tangent has negati v e slope. The inst antaneous
velocity at point B (turning point) is zero as the slope is zero.

KVPY_ SA STREAM # 5
3 .3 I n st a n t a n e o u s Acce l e ra t i on : v

The derivative of velocity with respect to time is the slope of the


tangent in velocity time (v t) graph. a=0
a>0 a<0
a=0 t
0
a=0

Example 6. Position of a particle as a function of time is given as x = 5t 2 + 4t + 3. Find the velocity


and acceleration of the particle at t = 2 s?
dx
Solution : Velocity; v = = 10t + 4
dt
At t = 2 s
v = 10(2) + 4
v = 24 m/s
d2x
Acceleration; a = = 10
dt 2
Acceleration is constant, so at t = 2 s a = 10 m/s2


4. M OTI ON W I TH UN I FOR M VELOCI T Y
Consider a particle moving along xaxis with uniform velocity u starting from the point x
= x i at t = 0.
Equations of x, v, a are : x (t) = x i + ut ; v (t) = u ; a (t) = 0
 x t graph is a straight line of slope u through x i.
 as velocity is constant, v  t graph is a horizontal line.
 at graph coincides with time axis because a = 0 at all time instants.

x
xi slo
pe
= u

u is negative
t
O

v v

positive velocity
u t
O

negative velocity
t u
O

5. UN I FOR M LY ACCELER ATED M OTI ON :


If a particle is accelerated with constant acceleration in an interval of time, then the motion is
termed as uniformly accelerated motion in that interval of time.
For uniformly accelerated motion along a straight line (xaxis) during a time interval of t seconds,
the following important results can be used.

KVPY_ SA STREAM # 6
v u
(a) a
t

v u
(b) Vav 
2
(c) S = (v av)t

v u
(d) S t
 2 
(e) v = u + at

(f) s = ut + 1/2 at 2
s = vt  1/2 at 2
x f = x i + ut + 1/2 at 2
(g) v 2 = u 2 + 2as
(h) sn = u + a/2 (2n  1)
u = initial velocity (at the beginning of interval)
a = acceleration
v = final velocity (at the end of interval)
s = displacement (x f  x i)
x f = final coordinate (position)
x i = initial coordinate (position)
sn = displacement during the n th sec

6. D I R ECTI ON S OF VECTOR S I N STR AI GH T LI N E M OTI ON


In straight line motion, all the vectors (position, displacement, velocity & acceleration) will have
only one component (along the line of motion) and there will be only two possible directions for
each vector.
 For example, if a particle is moving in a horizontal line (x axis), the two directions are
right and left. Any vector directed towards right can be represented by a positive number
and towards left can be represented by a negative number.

- +

line of motion
 For vertical or inclined motion, upward direction can
be taken +ve and downward as ve
+

line of motion

+
on
oti
fm
o
-
line
-

 For objects moving vertically near the surface of the earth, the only force acting on the
particle is its weight (mg) i.e. the gravitational pull of the earth. Hence acceleration for
this type of motion will always be a = g i.e. a =  9.8 m/s2 (ve sign, because the
force and acceleration are directed downwards, If we select upward direction as positive).

N ot e :
 If acceleration is in same direction as velocity, then speed of the particle
increases.
 If acceleration is in opposite direction to the velocity then speed decreases i.e. the
particle slows down. This situation is known as retardation.

KVPY_ SA STREAM # 7
Example 7. A particle moving rectilinearly with constant acceleration is having initial velocity of
10 m/s. After some time, its velocity becomes 30 m/s. Find out velocity of the particle
at the mid point of its path?
Solution : Let the total distance be 2x.
 distance upto midpoint = x
Let the velocity at the mid point be v
and acceleration be a.
From equations of motion
v 2 = 10 2 + 2ax ____ (1)
30 2 = v 2 + 2ax ____ (2)
(2) - (1) gives
v 2 - 30 2 = 10 2 - v 2
 v 2 = 500  v = 10 5 m/s

Example 8. Mr. Sharma brakes his car with constant acceleration from a velocity of 25 m/s to
15 m/s over a distance of 200 m.
(a) How much time elapses during this interval?
(b) W hat is the acceleration?
(c) If he has to continue braking with the same constant acceleration, how much
longer would it take for him to stop and how much additional distance would he
cover?
Solution : (a) W e select positive direction for our coordinate system to be the direction of the
velocity and choose the origin so that x i = 0 when the braking begins. Then the
initial velocity is u x = +25 m/s at t = 0, and the final velocity and position are
v x = +15 m/s and x = 200 m at time t.
Since the acceleration is constant, the average velocity in the interval can be
found from the average of the initial and final velocities.

1 1
 v av, x = (u x + v x) = (15 + 25) = 20 m/s.
2 2
The average velocity can also be expressed as

x
v av, x = . W ith x = 200 m
t
and t = t  0, we can solve for t:
Δx 200
t = v = = 10 s.
av, x 20
(b) W e can now find the acceleration using v x = u x + a xt

vx  u x 15  25
ax = = =  1 m/s2.
t 10
The acceleration is negative, which means that the positive velocity is becoming
smaller as brakes are applied (as expected).
(c) Now with known acceleration, we can find the total time for the car to go from
velocity u x = 25 m/s to v x = 0. Solving for t, we find

vx  u x 0  25
t = = = 25 s.
ax 1
KVPY_ SA STREAM # 8
The total distance covered is

1 2
x = x i + u xt + at
2 x

1
= 0 + (25)(25) + (1)(25) 2
2
= 625 – 312.5
= 312.5 m.
Additional distance covered
= 312.5 – 200
= 112.5 m.

Example 9. A police inspector in a jeep is chasing a pickpocket an a straight road. The jeep is
going at its maximum speed v (assumed uniform). The pickpocket rides on the motor-
cycle of a waiting friend when the jeep is at a distance d away, and the motorcycle
starts with a constant acceleration a. Show that the pick pocket will be caught if

v  2ad .
Solution : Suppose the pickpocket is caught at a time t after motorcycle starts. The distance
travelled by the motorcycle during this interval is

1
s  at2 ____ (1)
2
During this interval the jeep travels a distance
s  d  vt ____ (2)
By (1) and (2),

1 2
at  d  vt
2

v  v 2  2ad
or, t
a
The pickpocket will be caught if t is real and positive.
This will be possible if

v 2  2ad or, v 2ad

Example 10. A man is standing 40 m behind the bus. Bus starts with 1 m/sec 2 constant acceleration
and also at the same instant the man starts moving with constant speed 9 m/s. Find
the time taken by man to catch the bus.

1m/sec2

x=0 40 m t=0
t=0 x = 40

Solution : Let af ter time ‘t’ man will catch the bus
For bus
1 1
x = x 0 + ut + at 2 , x = 40 + 0(t) + (1) t 2
2 2

KVPY_ SA STREAM # 9
t2
x = 40 + ............. (i)
2
For man, x = 9t ............. (ii)
From (i) & (ii)

t2
40 + = 9t or t = 8 s or t = 10s.
2

Example 11. A particle is dropped from height 100 m and another particle is projected vertically up
with v elocity 50 m/s from the ground along the same line. Find out the position where
two particle will meet? (take g = 10 m/s 2)
Solution : Let the upward direction is positive. y=100m A u=0 m/s
Let the particles meet at a distance y from the ground.
For particle A,
y 0 = + 100 m
u = 0 m/s
a =  10 m/s2
y=0m u=50 m/s
1 1 2 B
y = 100 + 0(t)   10  t 2 [y = y 0 + ut + at ]
2 2
= 100 - 5t 2 ---- (1)
For particle B,
y0 = 0 m
u = + 50 m/s
a =  10 m/s2
1
y = 50(t) 
2
 10  t 2
= 50t  5t 2 ---- (2)
According to the problem;
50t  5t 2 = 100  5t 2
t = 2 sec
Putting t = 2 sec in eqn. (1),
y = 100  20 = 80 m
Hence, the particles will meet at a height 80 m above the ground.

Example 12. A particle is dropped from a tower. It is found that it travels 45 m in the last second of
its journey. Find out the height of the tower? (take g = 10 m/s 2)
Solution :
Let the total time of journey be n seconds.

a 10
Using; sn  u  ( 2n  1)  45 = 0 + ( 2n  1)
2 2
n = 5 sec

1 2 1
Height of tower; h = gt =  10  5 2 = 125 m
2 2


7. R EACTI ON TI M E
W hen a situation demands our immediate action. It takes some time before we really respond.
Reaction time is the time a person takes to observe, think and act.

KVPY_ SA STREAM # 10
Example 13. A stone is dropped from a balloon going up with a uniform velocity of 5 m/s. If the
balloon was 60 m high when the stone was dropped, find its height when the stone hits
the ground. Take g = 10 m/s 2.
1
Solution : S = ut + at 2
2

1
– 60 = 5(t) + (–10) t 2
2
+ve
– 60 = 5t – 5t 2
5t 2 – 5t – 60 = 0 –ve
60m
t 2– t – 12 = 0
t 2 – 4t + 3t – 12 = 0
(t – 4) (t + 3) = 0
 t = 4
Height of balloon from ground at this instant
= 60 + 4 × 5 = 80 m

Example 14. A balloon is rising with constant acceleration 2 m/sec 2. Two stones are released from
the balloon at the interval of 2 sec. Find out the distance between the two stones 1 sec.
after the release of second stone.
Solution : Acceleration of balloon = 2 m/sec 2
Let at t = 0, y = 0 when the first stone is released.
1
By the question, y 1 = 0 t 1 + gt 12
2
(taking vertical upward as – ve and downward as + ve)
9
 Position of  st stone = g
2
(1 second after release of second stone will be the 3 rd second for the 1 st stone)
1
For second stone y 2 = ut 2 + gt 22
2
u = 0 + at = – 2 × 2 = – 4m/s (taking vertical upward as – ve and downward as + ve)
1
 y2 = – 4 × 1 + g × (1) 2 (t 2 = 1 second)
2
The 2 nd stone is released after 2 second
1 1
 y = – at 2 = – × 2 × 4 = – 4
2 2

1
So, Position of second stone f rom the origin = – 4 + g – 4
2

1 1
Distance between two stones = g × 9 – g × 1 + 8 = 48 m.
2 2

N ot e :
 As the particle is detached from the balloon it is having the same velocity as that of
balloon, but its acceleration is only due to gravity and is equal to g.

KVPY_ SA STREAM # 11

8. STR AI GH T LI N E-EQUATI ON , GR AP H , SLOP E ( + ve , – ve , ze ro sl op e ) .
If  is the angle at which a straight line is inclined to the positive direction of x axis, &
0°  < 180°,  90°, then the slope of the line, denoted by m, is defined by m = tan . If  is
90°, m does not exist, but the line is parallel to the yaxis. If = 0, then m = 0 & the line is
parallel to the x-axis.
Slope  intercept form : y = mx + c is the equation of a straight line whose slope is m &
which makes an intercept c on the yaxis.
dy
m = slope = tan =
dx

y y y

–ve slope
+ve slope C
C slope = 0 
 C
x x x

9. PARABOLIC CURVE-EQUATION, GRAPH (VARIOUS SITUATIONS UP, DOW N, LEFT,


RIGHT W ITH CONDITIONS)

y y

y2 = kx x y2 = –kx x

y y

x2 = ky x x2 = –ky x

W here k is a positive constant.

Eq u a t i o n o f p a r a b o l a : y
Case (i) : y = ax 2 + bx + c
For a > 0
The nature of the parabola will be like that of the of nature x 2 = ky
Minimum value of y exists at the vertex of the parabola. x
D
y min = where D = b 2 – 4ac
4a

b D 
Coordinates of vertex =  , 
 2a 4a 

KVPY_ SA STREAM # 12
Case (ii) : a < 0 y
The nature of the parabola will be like that of the nature of x 2 = –ky
Maximum value of y exists at the vertex of parabola.
D x
y max = where D = b 2 – 4ac
4a

10. GR AP H S IN UN IFOR M LY ACCELERATED M OTI ON (a  0 )


 x is a quadratic polynomial in terms of t. Hence x  t graph is a parabola.

x
x
a<0
xi xi
a>0

t t
0 0

x-t graph
 v is a linear polynomial in terms of t. Hence v t graph is a straight line of slope a.

v v
a
= slo
pe u pe
slo = a
u
a is positive a is negative
t t
0 0

v-t graph
 at graph is a horizontal line because a is constant.

a a
positive
acceleration
a
0
negative
acceleration
t a
0

a-t graph

11. I N TER P R ETATI ON OF S OM E M OR E GR AP H S


1 1 . 1 P o sit i on vs Tim e gra p h
(i) Z e r o Ve lo cit y
As position of particle is fixed at all the time,
so the body is at rest.
dx
Slope; = tan = tan 0º = 0
dt
Velocity of particle is zero

(ii) Un i f o r m Ve l oci t y
dx
Here tanis constant tan =
dt
dx
 is constant.
dt
 velocity of particle is constant.

KVPY_ SA STREAM # 13
(iii) N on unif orm ve locit y (in crea sing wit h t i m e)
In this case;
As time is increasing,  is also increasing.

dx
 = tan is also increasing
dt
Hence, velocity of particle is increasing.

( i v) N on u n if orm ve locit y ( d e cre a sin g w it h t im e )


In this case;
As time increases,  decreases.

dx
 = tan also decreases.
dt
Hence, velocity of particle is decreasing.

1 1 . 2 Ve l ocit y vs t i m e gr a p h
(i) Z e ro a cce l e ra t ion
Velocity is constant.
tan = 0

dv
 = 0
dt
Hence, acceleration is zero.

(ii) Un i f o r m a cce l e r a t i o n
tan is constant.

dv
 = constant
dt
Hence, it shows constant acceleration.

(iii) Un i f o r m r e t a r d a t i o n
Since  > 90º
 tan is constant and negative.

dv
 = negative constant
dt
Hence, it shows constant retardation.

1 1 .3 Accelera tion vs tim e gra ph


(i) Con st a n t a cce le ra t i on
tan = 0

da
 = 0
dt
Hence, acceleration is constant.

KVPY_ SA STREAM # 14
(ii) Un i f o r m l y i n cr e a si n g a cce l e r a t i o n
 is constant.
0º <  < 90º  tan > 0

da
 = tan = constant > 0
dt
Hence, acceleration is uniformly increasing with time.

(iii) Un if o r m l y d e cr e a si n g a cce le r a t i o n
Since  > 90º
 tan is constant and negative.

da
 = negative constant
dt
Hence, acceleration is uniformly decreasing with time

Example 15.
The displacement vs time graph of a particle moving along
a straight line is shown in the figure. Draw velocity vs time
and acceleration vs time graph.

Solution :
x = 4t 2

dx
v = = 8t
dt
Hence, velocity-time graph is a straight line
having slope i.e. tan = 8.

dv
a = = 8
dt

Hence, acceleration is constant throughout and is equal to 8.


12. DISPLACEM ENT FROM v - t GRAPH & CHANGE IN VELOCITY FROM a -t GRAPH
Displacement = x = area under v t graph.
Since a negative v elocity causes a negative displacement, areas below
the time axis are taken negative. In similar way, can see that  v = a t
leads to the conclusion that area under a  t graph gives the change
in velocity v during that interval.

KVPY_ SA STREAM # 15
Example 16. Describe the motion shown by the following velocity-time graphs.

(a) (b)

Solution : (a) During interval AB: velocity is +v e so the particle is moving in +ve direction,
but it is slowing down as acceleration (slope of v-t curve) is negative. During
interval BC: particle remains at rest as velocity is zero. Acceleration is also
zero. During interval CD: velocity is -ve so the particle is moving in -ve direc-
tion and is speeding up as acceleration is also negative.
(b) During interval AB: particle is moving in +v e direction with constant velocity
and acceleration is zero. During interval BC: particle is moving in +ve direction
as velocity is +ve, but it slows down until it comes to rest as acceleration is
negative. During interval CD: velocity is -ve so the particle is moving in -ve
direction and is speeding up as acceleration is also negative.

I m po rt a n t P oi n t s t o R e m e m b e r

 For uniformly accelerated motion (a  0), xt graph is a parabola (opening upwards if
a > 0 and opening downwards if a < 0). The slope of tangent at any point of the parabola
gives the velocity at that instant.

 For uniformly accelerated motion (a  0), vt graph is a straight line whose slope gives
the acceleration of the particle.

 In general, the slope of tangent in xt graph is velocity and the slope of tangent in v t
graph is the acceleration.

 The area under at graph gives the change in velocity.

 The area between the v t graph gives the distance travelled by the particle, if we take
all areas as positive.

 Area under vt graph gives displacement, if areas below the taxis are taken negative.

Example 17. For a particle moving along x-axis, velocity-time graph is


as shown in f igure. Find t he distance trav elled and
displacement of the particle?

Solution :
Distance travelled = Area under v-t graph (taking all areas as +ve.)
Distance travelled = Area of trapezium + Area of triangle

=
1
2  6 8 + 1  4  5
2 2
= 32 + 10
= 42 m
KVPY_ SA STREAM # 16
Displacement = Area under v-t graph (taking areas below time axis as –ve.)
Displacement = Area of trapezium  Area of triangle

=
1
2  6 8  1  4  5
2 2
= 32  10 = 22 m
Hence, distance travelled = 42 m and displacement = 22 m.


13. M OTI ON W I TH N ON -UN I FOR M ACCELER ATI ON
( US E OF D EFI N I TE I N TEGR ALS )
tf
x =  v ( t )dt (displacement in time interval t = t i to t f )
ti

The expression on the right hand side is called t he def init e int egral of v (t) bet ween
t = t i and t = t f . . Similarly change in velocity
tf
v = v f  v i =  a( t)dt
ti

1 3 . 1 So lvin g P ro b le m s w h ich I n vol ve s N on u n if or m Acce le ra t i on


(i) Acce le ra t ion d e p e n d in g on ve locit y v or t im e t
dv
By def inition of acceleration, we have a = . If a is in terms of t,
dt
v v t
t dv
 dv =  a ( t ) dt . If a is in terms of v, 
v0
a (v)
  dt .
0
v0 0
On integrating, we get a relation between v and t, and then
x t
using  dx =  v (t ) dt , x and t can also be related.
x0 0
(ii) Acce le ra t i on d e p e n d in g on ve locit y v or p osit i on x
dv dv dx dx dv
a =  a =  a =
dt dx dt dt dx
dv
 a=v
dx
This is another important expression for acceleration.
If a is in terms of x,
v x
 v dv =  a ( x ) dx .
v0 x0

v x
v dv
If a is in terms of v, 
v0
a( v )
  dx
x0
On integrating, we get a relation between x and v.
x t
dx
Using 
x0
v (x) =  dt , we can relate x and t.
0

KVPY_ SA STREAM # 17
Example 18. An object starts from rest at t = 0 and accelerates at a rate given by a = 6t. W hat is
(a) its velocity and
(b) its displacement at any time t?
Solution : As acceleration is given as a function of time,
v(t) t

  dv   a(t)dt
v(t 0 ) t0

Here t 0 = 0 and v(t 0) = 0


t
 t2  t t2
 v(t) =  6tdt
0
= 6 
 2 0
= 6 (
2
- 0) = 3t 2

So, v(t) = 3t 2
t

As x   v(t)dt
t0

t
 t3  t  t3 
 x   3t 2dt =  
3 =  0 = t 3
3
0  3 0 3 
Hence, velocity v(t) = 3t 2 and displacement x  t 3 .

Example 19. For a particle moving alongv + x-axis, acceleration is given as a = x. Find the position
as a function of time? Given that at t = 0 , x = 1 v = 1.

vdv v2 x2
Solution : a = x  = x  = + C
dx 2 2
 t = 0, x = 1 and v = 1
 C = 0  v 2 = x2
v = ±x but given that x = 1 when v = 1
dx dx
 v = x  = x  = dt
dt x
 nx = t + C  0 = 0 + C   nx = t
x = et

Example 20. For a particle moving along x-axis, acceleration is given as a = v. Find the position as a
function of time ?
Given that at t = 0 , x = 0 v = 1.
dv dv
Solution : a=v 
dt
=v   v
=  dt
 nv = t + c  0 = 0 + c
dx
 dx =  e dt
t
v = et  = et 
dt
 x = et + c  0 = 1 + c
x = et – 1

KVPY_ SA STREAM # 18
1. A particle is moving along a straight line. Its velocity varies as v = 6 – 2t where v is in m/s and t in seconds.
Find the difference between distance covered and magnitude of displacement in first 4 seconds.
(A) 4m (B) 2m (C) 1m (D) 0 m

2. For a particle moving along x-axis, which of the velocity versus position graphs given in options below
is possible : (position is represented by x-coordinate of the particle)
v v v

x (B) x x
(A) (C) (D) None of these

3. Two particles 1 and 2 start from origin and move along same straight line in same direction. The acceleration
versus time graph of both particles is shown in figure. Choose correct statement about the motion of 1 and 2:

(A) During motion from O to T somewhere one particle will overtake the other.
(B) During motion from O to T average speed of both the particles is same.
(C) During motion from O to T average speed of both the particles are different.
(D) Displacement covered by the two particles from O to T are same.

4. A particle moves in x-y plane according to equations, x = 4 t 2 + 5 t + 16 & y = 5 t. The acceleration of


the particle will be:
(A) 8 m/sec2 (B) 13 m/sec2 (C) 14 m/sec2 (D) none of these

5. At t = 0 second a particle is at point A and moves along the shown path (ABCDE) with uniform speed
 
v =  1   m/s. Both the straight segments AB and DE are along the diameter BD of semicircle BCD of
 3 
radius R = 1 metre. Then the instant of time at which the instantaneous velocity of the particle is along the
direction of average velocity from t = 0 second to that instant is also

1
(A) sec. (B) 1 sec (C) sec (D) 1.5 sec.
2  
1  
 6
6. For a particle moving along x-axis, the acceleration a of the particle in terms of its x-coordinate and it
is given by a = – 9x, where x is in meters and a is in m/s2. Take acceleration, velocity and displacement
in positive x-direction as positive. The initial velocity of particle at x = 0 is u = + 6 m/s. The velocity of
particle at x = 2 m will be :
(A) + 6 2 m/s (B) – 6 2 m/s (C) 72 m/s (D) 0

KVPY_ SA STREAM # 19
7. A car starts from rest & again comes to rest after travelling 200 m in a straight line. If its acceleration and
deacceleration are limited to 10 m/s2 & 20 m/s2 respectively then minimum time the car will take to travel the
distance is -
20
(A) 20 s (B) 10 s (C) 2 15 s (D) s
3
8. Figure shows the position(x) versus time(t) graph for a particle moving in a straight line. At which point, the
instantaneous velocity is greater than the average velocity from 'O' to that point ?

(A) A (B) B (C) C (D) None of these

1
9. A particle moves in a straight line with an acceleration a ms –2 at time ‘t’ seconds where a = – .
t2
When t = 1 the particle has a velocity of 3ms then find the velocity when t = 4
–1

(A) 7 m/s (B) 2 m/s (C) 2.25 m/s (D) 9/2 m/s

10. The displacement of a body from a reference point is given by x = 2 t  3, where x is in metres and
t in seconds. This shows that the body:
(A) is at rest at t = 3/2 (B) is decelerated for t > 3/2
(C) is decelerated for t < 3/2 (D) is in uniform motion

11. Two particles at a distance 5m apart, are thrown towards each other on an inclined smooth plane with
equal speeds ‘v’ . It is known that both particle move along the same straight line. Find the value of v if
they collide at the point from where the lower particle is thrown. Inclined plane is inclined at an angle of
300 with the horizontal. [take g = 10m/s2 ]
(A) 2.5 m/sec (B) 5 m/sec (C) 7.5 m/sec (D) 10 m/sec

12. A stone is dropped from the top of a tower. When it has fallen by 5m from the top, another stone is dropped
from a point 25m below the top. If both stones reach the ground at the same moment, then height of the tower
from ground is : (take g = 10m/s2)
(A) 45 m (B) 50m (C) 60m (D) 65m

13. A stone is released at rest from a height of 45 m above the horizontal level ground. There is horizontal
wind blowing due to which stone acquires an additional (in addition to acceleration due to gravity)
horizontal acceleration of magnitude 10 m/s 2 . Then the net distance travelled by stone before reaching
ground is: ( Take g = 10m/s2 and neglect air resistance)
(A) 45 m. (B) 45 2 m (C) 45 3 m (D) cannot be determined

14. The uniform motion of a runner is synchronised with the tossing of a coin upwards. Speed of runner is
constant and he covers a distance ‘S1’ during upward journey of coin, while distance S2 is covered by runner
during downward journey of coin. Provided that motion of coin is perpendicular to earth (g = constant) and air
offers uniform resistance against motion of coin, the relation between ‘S1’ and ‘S2’ will be :
(A) S1 = S2 (B) S1 > S2 (C) S1 < S2
(D) Nothing can be predicted from given condition.

15. A particle is moving in a straight line with initial velocity u and uniform acceleration f. If the sum of the
distances travelled in t th and (t + 1)th seconds is 100cm, then its velocity after t seconds, in cm/s, is
(A) 20 (B) 30 (C) 50 (D) 80

KVPY_ SA STREAM # 20
16. Displacement-time graph of motion of a particle moving along a straight line and starting from A is shown. If
its velocities at points A and B are -15m/s and zero respectively then average velocity between these two
points is:

(A) -5 m/s (B) -3 m/s (C) -4 m/s (D) 5 m/s

17. The displacement-time graph of a moving object is shown in figure. The velocity-time graph
representing the motion of the same body is :

(a)

(A) (B) (C) (D)

x
18. A particle is moving in x-y plane along curve y = and ux = 4 – 2t. The displacement verses time graph of
2
the particle would be (where all parameters are in S.I. units)

(A) (B) (C) (D)

19. The acceleration-displacement graph of a particle moving in a straight line is as shown in figure, initial
velocity of particle is zero. Find velocity (in m/s) of the particle when displacement of the particle is s = 16m.

(A) 60 m/s (B) 64 m/s (C) 20 m/s (D) 1 m/s

KVPY_ SA STREAM # 21
20. A stone is projected at an angle 45° with horizontal above horizontal x-axis as shown in figure-1 (and y-axis
is vertical). Four graphs representing the magnitude of horizontal (vx) or magnitude of vertical (vy) component
of velocity of stone w.r.t. time t are as shown. Then which of the following magnitude of velocity versus time
graph best represents v x versus t and v y versus t, respectively for the stone.

(D)
t
(IV)
Vx versus t Vy versus t
(A) I IV
(B) II III
(C) I II
(D) II IV

21. A particle is moving along a straight line whose velocity displacement graph is shown. What is the acceleration
when displacement is 2m.

(A) 3 m/s2 (B) – 1 m/s2 (C) – 3 m/sec2 (D) 1 m/sec2

22. The graph shows the variation of velocity of a rocket with time. The maximum height attained by the rocket is :

(A) 11 km (B) 50 km (C) 55 km (D) 60 km

23. The position time graph of a moving particle is shown. Then which of the below best represents its velocity
time graph : (1 = 2 = 120º)

(A) (B) (C) (D)

KVPY_ SA STREAM # 22
24. The initial velocity of a particle is given by u (at t = 0) and the acceleration by f, where f = at (here t is
time and a is constant). Which of the following relation is valid?
at 2
(A) v = u + at 2 (B) v = u + (C) v = u + at (D) v = u
2
25. A student determined to test the law of gravity for himself walks off a sky scraper 320 m high with a
stopwatch in hand and starts his free fall (zero initial velocity). 5 second later, superman arrives at the
scene and dives off the roof to save the student. What must be superman's initial velocity in order that
he catches the student just before reaching the ground ?
[Assume that the superman's acceleration is that of any freely falling body.] (g = 10 m/s2)
275 187
(A) 98 m/s (B) m/s (C) m/s (D) It is not possible
3 2

KVPY PROBLEMS (PREVIOUS YEARS)


1. A body is moving with constant acceleration from A to B in a straight line. C is the mid-point of AB. If u and
v are the speeds at A and B respectively. The speed at C is [KVPY_2007]

uv v–u u2  v 2 v 2 – u2
(A) (B) (C) (D)
2 2 2 2

2. A juggler tosses a ball up in the air with initial speed u. At the instant it reaches its maximum height H, he
tosses up a second ball with the same initial speed. The two balls will collide at a height. [KVPY_2011]
H H 3H 3
(A) (B) (C) (D) H
4 2 4 4

3. A ball falls vertically downward and bounces off a horizontal floor. The speed of the ball just before reaching
the floor (u1) is equal to the speed just after leaving contact with the floor (u2) ; u1 = u2 . The corresponding
magnitudes of accelerations are denoted respectively by a1 and a2. The air resistance during motion is
proportional to speed and is not negligible. If g is acceleration due to gravity, then : [KVPY_2012]
(A) a1 < a2 (B) a1 = a2  g (C) a1 > a2 (D) a1 = a2 = g

4. The accompanying graph of position x versus time t represents the motion of a particle. If p and q are both
positive constants, the expression that best describes the acceleration  of the particle is. [KVPY_2013]

(A) a = – p – qt (B) a = –p + qt (C) a = p + qt (D) a = p – qt

5. In the follwing displacement (x) vs time (t) graph, at which among the points P,Q, and R is the object's speed
increasing? [KVPY_2014_SA]
x

P
Q
(0,0) t
R

(A) R only (B) P only (C) Q and R only (D) P,Q,R

KVPY_ SA STREAM # 23
6. An object at rest at the origin begins to move in the +x direction with a uniform acceleration of 1 m/s2
for 4 s and then it continues moving with a uniform velocity of 4 m/s in the same direction. The x – t
graph for object’s motion will be [KVPY_2015_SA_1 Mark]

x x

(A) (B)

4s t 4s t

x x

(C) (D)

4s t 4s t

7. A parachutist with total weight 75 kg drops vertically onto a sandy ground with a speed of 2 ms -1 and
comes to a halt over a distance of 0.25m. The average force from the ground on her is close to.
[KVPY_2015_SA_1Mark]
(A) 600 N (B) 1200 N (C) 1350 N (D) 1950 N

8. A person walks 25.0° north of east for 3.18 km. How far would she have to walk due north and then due east
to arrive at the same location ? [KVPY_2016_SA]
(A) towards north 2.88 km and towards east 1.34 km.
(B) towards north 2.11 km and towards east 2.11 km
(C) towards north 1.25 km and towards east 1.93 km
(D) towards north 1.34 km and towards east 2.88 km.
9. A stone thrown down with a speed u takes a time t1 to reach the ground, while another stone, thrown upwards
from the same point with the same speed, takes time t2. The maximum height the second stone reaches
from the ground is [KVPY_2016_SA]
(A) ½ gt1t2 (B) g/8 (t1 + t2)2 (C) g/8 (t1 – t2)2 (D) ½ gt 22

10. A student performs an experiment to determine the acceleration due to gravity g. The student throws a steel
ball up with initial velocity u and measures the height h traveled by it at different times t. The graph the
student should plot on a graph paper to readily obtain the value of g is. [KVPY_2017_SA_1Mark]
(A) h versus t (B) h versus t2 (C) h versus t (D) h/t versus t

11. A person goes from point P to point Q covering 1/3 of the distance with speed 10 km/hr, the next 1/3 of the
distance at 20 km/hr and the last 1/3 of the distance at 60 km/hr. The average speed of the person is
[KVPY_2017_SA_1Mark]
(A) 30 km/hr (B) 24km/hr (C) 18 km/hr (D) 12 km/hr

KVPY_ SA STREAM # 24
12. A ball is dropped vertically from height h and is bouncing elastically on the floor (see figure). Which of the
following plots best depicts the acceleration of the ball as a function of time. [KVPY_2017_SA_1Mark]

t
acceleration

acceleration
(A) t (B) t
acceleration

acceleration

(C) t (D) t

13. A particle starts moving along a line from zero initial velocity and comes to rest after moving distance d.
During its motion it had a constant acceleration f over 2/3 of the distance, and covered the rest of the
distance with constant retardation. The time taken to cover the distance is [KVPY_2017_SA_1Mark]
(A) 2d/ 3f (B) 2 d/ 3f (C) 3d / f (D) 3d/ 2f

14. If a ball is thrown at a velocity of 45 m/s in vertical upward direction, then what would be the velocity profile as
function of height ? Assume g = 10 m/s2. [KVPY-SA_2018 1 Mark]

45
V(m/s)
(A) (B)
0
0 Height (m) 101

45 45
V(m/s) V(m/s)
(C) (D)
0 0
0 Height (m) 101 0 Height (m) 101

KVPY_ SA STREAM # 25
EXERCISE-1
1. (B) 2. (D) 3. (C) 4. (A) 5. (B) 6. (D) 7. (C)
8. (C) 9. (C) 10. (A) 11. (A) 12. (A) 13. (B) 14. (C)
15. (C) 16. (C) 17. (C) 18. (C) 19. (B) 20. (B) 21. (C)
22. (C) 23. (B) 24. (B) 25. (B)

EXERCISE-2
1. (C) 2. (C) 3. (A) 4. (D) 5. (A) 6. (B) 7. (C)
8. (D) 9. (C) 10. (D) 11. (C) 12. (B) 13. (C) 14. (A)

t 4

1. Displacement =  vdt =  (6  2t )dt


0 0
= 2m

4 3 4

Distance = 
0
| v | dt = 
0

(6  2t )dt  (2t  6)dt = 10 m
3

 Distance – displacements = 10 – 8 = – 2

2. It is clearly visible from all graphs that as x-increases. Velocity changes sign. Since this is not possible,
no graph represents the possible motion.

3. Since acceleration of 1 is always greater or equal to 2, 2 will never overtake 1. Also average speed of 1 is
greater than that of 2. The displacement curve of the two will be different as they have different accelerations.

d2 x
4. ax = =8
dt 2

d2 y
ay = = 0 So, anet = ax
dt 2

5. (Moderate) The required condition is achieved when the particle is at P such that AP is tangent to
semicircle.
R
R
Hence required time = 3 = 1 second.
v

v 2
dv
6. v
dx
= – 9x   v dv    9x dx
6 0

v2  62
 = – 18 or v=0
2

KVPY_ SA STREAM # 26
Vmax
7. = 10 v
2t
 Vmax = 20t vmax
Area of graph
1
 V × 3t = 200 t
2 max
2t t
3 20
 t × 20 t = 200  t=
2 3

total time = 3t = 2 15 sec

8. At point 'C', the slope of the curve is greater than the slope of line joining from 'O' to 'C'.
v 4
dv 1 1
9. a=
dt
= 2
t
  dv =   t
3 1
2
dt

4
1 1 3 9
 v – 3 =   = 1  v = 3 = = 2.25 m/s.
t
 1 4 4 4

10. x =2t3
dx
x = 2t  3 2  = 8t – 12 = 0 when t = 3/2
dt
d2 x
=8
dt 2
So A & B are correct

1
11. Down the plane 5 = v . t + (g sin ) t2 ....(1)
2
v
at the plane 0 = v – g sinq t1  t1 =
g sin 
2v
t = 2t1 = [time taken by B complaining back to initial position]
g sin 
2
2.v 2 1 g sin .4v
5= + 2 2
g sin  2 g sin 

 1
10  10   
2 = 100 10
10 g sin = 8v 2 v= = = 2.5 m/sec.
8 16 4

12. Vel. of Ist stone when passing at A 


V2 = 0 + 2.10.5
V = 10 m/s 5m
S1 – S2 = 20 m. A
20 m
 1 2 1 2
 10.t  2 10.t  -  2 .10.t   20 B
S1
   
t = 2s S2
1 ////////////////////////////
S2 = . 10 . 4 = 20 m
2
Ht = 25 + 20 = 45 m.

KVPY_ SA STREAM # 27
13. AO = 45 m, AB = 45 2 A
anet= 2g
net acceleration makes 45° with horizontal. Since u = 0,
The stone moves along AB. 45m
 Net distance travelled before
O 45° B
reaching ground is= 45 2 m.

AO = 45 m, AB = 45 2

14. Due to air resistance, the time of ascent is less than time of descent.
 Distance S2 is more than 'S1'.

15. Sum of distance travelled in t th & (t + 1)th


Seconds = 100 cm
a a
 u+ (2t – 1) + u + (2(t + 1)–1) = 100
2 2
 2u + 2ft = 100  u + ft = 100/2  v = 50 cm/s.

x f  xi 6  18
16. Van = = = -4m/s
t 4 1
x
18. y= implies that particle moving in a straight line passing through origin.
2
ux = 4 – 2t  ax = – 2 and 4x = 4
1 1
x = 4x.t + a . t2 = 4t + × (– 2) × t2
2 x 2
x = 4t – t2

5 5
S = x 2  42  S = x  S = (4t – t2)
2 2
19. v dv = ads
v 12m

  v dv =  a ds
0
0

2
v
 = area under a-s graph from s = 0 to s = 16m.
2
= 2 + 12 + 6 + 12 = 32 m /s
or v= 64 m/s = 8 m/s
20. Magnitude of v x remains constant. Magnitude of v y decreases linearly with time to zero and then increases.
Therefore the correct graphs are

dv
21. a= v
ds
a = (3 m/s) (–tan 45º) (1 sec.)
= – 3 m/sec2

22. Height is maximum when


1
maximum height = (110) (1000) = 55 × 103 m.
2

23. When displacement time curve is upward parabola then acceleration is constant and +ve. Therefore
v–t graph for upward parabola is straight line of positive slope. Similarly, opposite for downward parabola.

KVPY_ SA STREAM # 28
PROJ ECT I LE M OT I ON

1. BASIC CONCEPT :
1 .1 Proje ct ile
Any object that is given an initial velocity obliquely, and that subsequently follows a path determined
by the net constant force, (In this chapter constant force is gravitational force) acting on it is called
a projectile.
Examples of projectile motion :
 A cricket ball hit by the batsman for a six
 A bullet fired from a gun.
 A packet dropped from a plane; but the motion of the aeroplane itself is not projectile motion
because there are forces other than gravity acting on it due to the thrust of its engine.

1 .2 Assum ptions of Projectile M otion :


 We shall consider only trajectories that are of sufficiently short range so that the gravitational
force can be considered constant in both magnitude and direction.

 All effects of air resistance will be ignored.

 Earth is assumed to be flat.

1 .3 Projectile M otion :
 The motion of projectile is known as projectile motion.

 It is an example of two dimensional motion with constant acceleration.

 Projectile motion is considered as combination of two simultaneous motions in mutually


perpendicular directions which are completely independent from each other i.e. horizontal
motion and vertical motion.

= +

Parabolic path = vertical motion + horizontal motion.

Ga lileo’s Sta tem ent :


Two perpendicular directions of motion are independent from each other. In other words any
vector quantity directed along a direction remains unaffected by a vector perpendicular to it.

KVPY_ SA STREAM # 29
2. PROJECTILE THROW N AT AN ANGLE W ITH HORIZONTAL

2 2
(ucos) +(usin – gt)

 Consider a projectile thrown with a velocity u making an angle  with the horizontal.

 Initial velocity u is resolved in components in a coordinate system in which horizontal direction is


taken as x-axis, vertical direction as y-axis and point of projection as origin.
ux = u cos  uy = u sin 

 Again this projectile motion can be considered as the combination of horizontal and vertical motion.
Therefore,
` Horizontal direction Vertical direction
(a) Initial velocity ux = u cos  Initial velocity uy = u sin 
(b) Acceleration ax = 0 Acceleration ay = g
(c) Velocity after time t, v x = u cos  Velocity after time t,v y = u sin  – gt

2 .1 Tim e of flight :
The displacement along vertical direction is zero for the complete flight.
Hence, along vertical direction net displacement = 0

1 2u sin 
 (u sin ) T – gT2 = 0  T=
2 g
2 .2 Horizonta l ra nge :
2u sin  u 2 sin 2
R = ux .T  R = u cos .  R=
g g

2 .3 M a xim um height :
At the highest point of its trajectory, particle moves horizontally, and hence vertical component of
velocity is zero.
Using 3rd equation of motion i.e.
v 2 = u2 + 2as
we have for vertical direction

u 2 sin 2 
0 = u2 sin2  – 2gH  H=
2g

2 .4 Re sulta nt velocity :

v  v x î  v y ĵ = u cos  î + (u sin  – gt) ĵ

Where, | v | = u 2 cos 2   (u sin   gt )2 and tan  = v y / v x .

u cos 
Also, vcos = ucos  v =
cos 

KVPY_ SA STREAM # 30
N ot e :
 Results of article 2.2, and 2.3 are valid only if projectile lands at same horizontal level from
which it was projected.
 Vertical component of velocity is positive when particle is moving up and vertical component of
velocity is negative when particle is coming down if vertical upwards direction is taken as positive.

2 .5 Genera l result :
 For maximum range  = 45º

u2 R max
Rmax =  Hmax =
g 2

 We get the same range for two angle of projections  and (90 – ) but in both cases,
maximum heights attained by the particles are different.

u 2 sin 2
This is because, R = , and sin 2 (90 – ) = sin 180 – 2 = sin 2
g

 f R=H
u 2 sin 2 u 2 sin 2 
i.e. =  tan  = 4
g 2g

 Range can also be expressed as

u 2 sin 2 2u sin .u cos  2u x u y


R= = =
g g g

Example 1. A body is projected with a speed of 30 ms–1 at an angle of 30º with the vertical. Find the maximum
height, time of flight and the horizontal range of the motion. [ Take g = 10 m/s2 ]
Solution : Here u = 30 ms–1, Angle of projection,  = 90 – 30 = 60º
Maximum height,

u 2 sin 2  30 2 sin2 60º 900 3 135


H= = =  = m
2g 2  10 20 4 4

2u sin  2  30  sin 60 0
Time of flight, T= = = 3 3 sec.
g 10

u 2 sin 2 30  30  2 sin 60 º cos 60º


Horizontal range = R = = = 45 3 m
g 10

Example 2. A projectile is thrown with a speed of 100 m/s making an angle of 600 with the horizontal. Find the
time after which its inclination with the horizontal is 450 ?
Solution : ux = 100 × cos600 = 50
uy = 100 × sin600 = 50 3

v y = uy + ay t = 50 3 – gt and v x = ux = 50
When angle is 450 ,
vy
tan 450 =  vy = vx  50 3 – gt = 50
vx

 50 ( 3  1) = gt  t=5  3  1 s
KVPY_ SA STREAM # 31
Example 3. A large number of bullets are fired in all directions with the same speed v. What is the maximum area
on the ground on which these bullets will spread ?
Solution : Maximum distance up to which a bullet can be fired is its maximum range, therefore

v2
Rmax =
g

v 4
Maximum area = (Rmax)2 = .
g2


Example 4. The velocity of projection of a projectile is given by : u  5 î  10 ĵ . Find
(a) Time of flight,
(b) Maximum height,
(c) Range
Solution : We have ux = 5 uy = 10

2u sin  2u y 2  10
(a) Time of flight = = = =2s
g g 10

u 2 sin 2 
2
uy 10  10
(b) Maximum height = = = =5m
2g 2g 2  10

2u sin .u cos  2u x u y 2  10  5


(c) Range = = = = 10 m
g g 10

Example 5. A particle is projected at an angle of 300 w.r.t. horizontal with speed 20 m/s :
(i) Find the position vector of the particle after 1s.
(ii) Find the angle between velocity vector and position vector at t = 1s.
Solution : (i) x = u cos  t

3
= 20 × ×t = 10 3 m
2
1
y = u sin t – × 10 × t2
2
1
= 20 × × (1) – 5 (1)2 = 5m
2


Position vector, r = 10 3 î  5 ĵ ,


| r |  10 3  5
2 2

(ii) v x = 10 3 î
v y = uy + ayt = 10 – g t = 0

 v = 10 3 î , | v | = 10 3
 
v  r  (10 3 î )  (10 3 î  5 ĵ ) = 300
   
v . r = | v | | r | cos 
 
vr 300
 cos  =   =
| v || r | 10 3 325

 3 
 = cos  2
1
 
 13 

KVPY_ SA STREAM # 32

3. EQUATION OF TRAJECTORY
The path followed by a particle (here projectile) during its motion is called its Trajectory. Equation of trajectory
is the relation between instantaneous coordinates (Here x & y coordinate) of the particle.
If we consider the horizontal direction,
x = ux.t
x = u cos . t ...(1)
For vertical direction :
y = uy . t – 1/2 gt2
= u sin  . t – 1/2 gt2 ...(2)
Eliminating ‘t’ from equation (1) & (2)
2
x 1  x 
y = u sin  . – g  
u cos  2  u cos  

gx 2
 y = x tan  –
2u2 cos 2 
This is an equation of parabola called as trajectory equation of projectile motion.

Other forms of trajectory equation :

gx 2 (1  tan 2 )
 y = x tan  –
2u2

gx 2
 y = x tan  –
2u 2 cos 2 

 gx 
 y = x tan  1 – 2 2 
 2u cos  tan  

 gx 
 y = x tan  1  2 
 2u sin  cos  

 x
 y = x tan  1  
 R

Example 6. Find the value of  in the diagram given


below so that the projectile can hit the target.

gx 2 (1  tan 2 )
Solution y = x tan  –
2u2

5  (20)2
 10 = 20 tan – (1 + tan2)
(20)2
2 = 4 tan  – (1 + tan2)  tan2 – 4 tan  + 3 = 0
 (tan  – 3) (tan  – 1) = 0  tan  = 3, 1  = 45° , tan–1(3)

KVPY_ SA STREAM # 33
Example 7. A ball is thrown from ground level so as to just clear a wall 4 m high at a distance of 4 m and falls at
a distance of 14 m from the wall. Find the magnitude and direction of initial velocity of the ball. Figure
is given below.

Solution P

The ball passes through the point P(4, 4). Also range = 4 + 14 = 18 m.
The trajectory of the the ball is,
x
y = x tan  ( 1 – )
R
 4 7
Now x = 4m, y = 4m and R = 18 m  4 = 4 tan  1   = 4 tan  .
 18  9

9 9 2u 2 sin  cos 
or tan =   = tan–1 And R=
7 7 g

2 9 7
or 18 = × u2 × ×  u= 182
9. 8 130 130


4. PROJECTILE THROW N PARALLEL TO THE HORIZONTAL FROM SOM E HEIGHT
Consider a projectile thrown from point O at some height h from the ground
with a velocity u. Now we shall study the characteristics of projectile motion
by resolving the motion along horizontal and vertical directions.
Horizontal direction Vertical direction
(i) Initial velocity ux = u Initial velocity uy = 0
(ii) Acceleration ax = 0 Acceleration ay = g (downward)
4 .1 Tim e of flight :
This is equal to the time taken by the projectile to return to ground. From equation of motion
1 2
S = ut + at , along vertical direction, we get
2

1 1 2 2h
– h = uyt + (–g)t2  h= gt  t=
2 2 g
4 .2 Horizonta l ra nge :
Distance covered by the projectile along the horizontal direction between the point of projection to
the point on the ground.
2h
R = ux . t  R =u g
4 .3 Velocity a t a genera l point P(x, y) :

v= u2x  u 2y
Here horizontal velocity of the projectile after time t
vx = u
velocity of projectile in vertical direction after time t
v y = 0 + (–g)t = –gt = gt (downward)

 v= u2  g2 t 2 and tan  = vy/vx

KVPY_ SA STREAM # 34
4 .4 Velocity with which the projectile hits the ground :
Vx = u
Vy2 = 02 – 2g(–h)
Vy = 2gh

V= Vx2  Vy2  V= 2  2gh


u

4 .5 Tra jectory equa tion :


The path traced by projectile is called the trajectory.
After time t,
x = ut ....(1)
1 2
y= gt ....(2)
2
From equation (1)
t = x/u
Put the value of t in equation (2)

1 2
y= g. x
2
2 u
This is trajectory equation of the particle projected horizontally from some height.

Exa m ples ba sed on horizonta l projection from som e height :


Example 8. A projectile is fired horizontally with a speed of 98 ms–1 from the top of a hill 490 m high. Find (i) the
time taken to reach the ground (ii) the distance of the target from the hill and (iii) the velocity with
which the projectile hits the ground. (take g = 9.8 m/s2)
Solution : (i) The projectile is fired from the top O of a hill with speed u = 98 ms–1 along
the horizontal as shown as OX. It reaches the target P at vertical depth
OA, in the coordinate system as shown, OA = y = 490 m
1 2
As, y= gt
2
1
 490 = × 9.8 t2
2
or t= 100 = 10 s.

(ii) Distance of the target from the hill is given by,


AP = x = Horizontal velocity × time = 98 × 10 = 980 m.

(iii) The horizontal and vertical components of velocity v of the projectile at point P are
v x = u = 98 ms–1
v y = uy + gt = 0 + 9.8 × 10 = 98 ms–1

V= v 2x  v 2y = 98 2  98 2 = 98 2 ms–1

Now if the resultant velocity v makes an angle  with the horizontal, then
vy 98
tan  = = =1   = 45º
vx 98

Example 9. A motorcycle stunt rider rides off the edge of a cliff. Just at the edge his velocity is horizontal, with
magnitude 9.0 m/s. Find the motorcycle’s position, distance from the edge of the cliff and velocity
after 0.5 s.

KVPY_ SA STREAM # 35
Solution : At t = 0.50 s, the x and y-coordinates are

v0
x

y
vx = v0
x
a
®
vy = –gt v

x = v 0t = (9.0 m/s) (0.50 s) = 4.5 m


1 2 1 5
y=– gt = – (10 m/s2) (0.50 s)2 = – m
2 2 4
The negative value of y shows that this time
the motorcycle is below its starting point.
The motorcycle’s distance from the origin at this time
2 2
9 5 349
r= x2  y2 =     = m.
2
  4 4
The components of velocity at this time are
v x = v 0 = 9.0 m/s
v y = – gt = (–10 m/s2) (0.50 s) = –5 m/s.
The speed (magnitude of the velocity) at this time is

v= v 2x  v 2y = (9 . 0 m / s )2  ( 5 m / s ) 2 = 106 m/s

Example 10. An object is thrown between two tall buildings. 180 m from each other. The object is thrown horizon-
tally from a window 55 m above ground from one building through a window 10.9 m above ground in
the other building. Find out the speed of projection. (use g = 9.8 m/s2)

2h 2  44.1
Solution : t= g = 9. 8

t = 3 sec. 44.1
55 m
R = uT
180 m
180 10.9 m
= u ; u = 60 m/s
3 180 m

6. PROJECTION FROM A M OVING PLATFORM

CASE (1) : When a ball is thrown upward from a truck moving with uniform speed, then observer A standing in
the truck, will see the ball moving in straight vertical line (upward & downward).
The observer B sitting on road, will see the ball moving in a parabolic path. The horizontal speed of
the ball is equal to the speed of the truck.

KVPY_ SA STREAM # 36
CASE (2) : When a ball is thrown at some angle ‘’ in the direction of motion of the truck, horizontal &
vertical component of ball’s velocity w.r.t. observer A standing on the truck, is ucos, and usin
respectively.
Horizontal & vertical component of ball’s velocity w.r.t. observer B sitting on the ground, is
ux = ucos + v and uy=usin respectively.

usin
u

ucos
A

B
v

CASE (3) : When a ball is thrown at some angle ‘’ in the opposite direction of motion of the truck, horizontal &
vertical component of ball’s velocity w.r.t. observer A standing on the truck, is ucos, and usin
respectively.
Horizontal & vertical component of ball’s velocity w.r.t. observer B sitting on the ground, is
ux = ucos – v and uy=usin respectively.

usin
u


ucos
A

B v

CASE (4) : When a ball is thrown at some angle ‘’ from a platform moving with speed v upwards, horizontal &
vertical component of ball’s velocity w.r.t. observer A standing on the moving platform, is ucos and
usin respectively.
Horizontal & vertical component of ball’s velocity w.r.t. observer B sitting on the ground, is
ux = ucos and uy = usin + v respectively.

usin
u

A 
ucos
v

CASE (5) : When a ball is thrown at some angle ‘’ from a platform moving with speed v downwards, horizontal
& vertical component of ball’s velocity w.r.t. observer A standing on the moving platform, is ucos
and usin respectively.
Horizontal & vertical component of ball’s velocity w.r.t. observer B sitting on the ground, is
ux = ucos and uy=usin – v respectively.

usin
u


ucos
A

KVPY_ SA STREAM # 37
Example 11. A boy standing on a long railroad car throws a ball straight upwards. The car is moving on the
horizontal road with an acceleration of 1 m/s2 and the projection speed in the vertical direction is 9.8
m/s. How far behind the boy will the ball fall on the car ?
Solution : Let the initial velocity of car be ‘u’.
time of flight,

2u y
t= =2
g

where uy = component of velocity in vertical direction


Distance travelled by car

1
xc = u × 2 + × 1 × 22
2
= 2u + 2
distance travelled by ball
xb = u × 2
xc – xb = 2u + 2 – 2u
= 2m Ans.

Example 12. A fighter plane moving with a speed of 50 2 m/s upward at an angle of 45º with the vertical,
releases a bomb. Find
(a) Time of flight
(b) Maximum height of the bomb above ground

1
Solution : (a) y = uy t + ayt2
2

1
–1000 = 50t –  10  t 2
2
t2 – 10t – 200 = 0
(t – 20) (t + 10) = 0
t = 20 sec

u2y 50 2 50  50
(b) H = = = = 125 m
2g 2g 20

Hence maximum height above ground


H = 1000 + 125 = 1125 m

KVPY_ SA STREAM # 38
1. A particle moves along the parabolic path y = ax 2 in such a way that the y-component of the velocity
remains constant, say c. The x and y coordinates are in meters. Then acceleration of the particle at
x = 1 m is

c2 c
(A) ac k̂ (B) 2ac2 ĵ (C)  î (D)  î
4a 2 2a

2.* A particle is projected with a velocity ‘u’ in horizontal direction as shown in the figure. Find ‘u’ so that
the particle collides orthogonally with the inclined plane of the fixed wedge.

2gh 2gh 2gh 2gh


(A) 2 (B) 2 (C) 2 (D)
2  cos  2  sin  1  cos  1  sin2 

3. A particle is projected from the horizontal x-z plane, in vertical x-y plane where x-axis is horizontal and
positive y-axis vertically upwards. The graph of ‘y’ coordinate of the particle v/s time is as shown. The range
of the particle is 3 m . Then the speed of the projected particle is :

403
(A) 3 m/ s (B) m/s (C) 2 5 m/s (D) 28 m / s
4

4. Velocity of a stone projected, 2 second before it reaches the maximum height, makes angle 53° with the
horizontal then the velocity at highest point will be
(A) 20 m/s (B) 15 m/s (C) 25 m/s (D) 80/3 m/s

5. A pebble is thrown horizontally from the top of a 20 m high tower with an initial velocity of 10 m/s. The air drag
is negligible. The speed of the pebble when it is at the same distance from top as well as base of the tower
(g = 10 m/s2)
(A) 10 2 m/s (B) 10 3 m/s (C) 20 m/s (D) 25 m/s

6. A projectile A is projected from ground. An observer B running on ground with uniform velocity of magnitude
‘v’ observes A to move along a straight line. The time of flight of A as measured by B is T. Then the range R
of projectile on ground is
(A) R = vT (B) R < vT (C) R > vT
(D) information insufficient to draw inference

7. A ball is projected with velocity u at right angle to the slope which is inclined at an angle  with the horizon-
tal. The distance ‘x’ along the inclined plane that it will travel before again striking the slope is -

2u2 2u2 2u2 tan  2u 2 tan 


(A) cos  (B) tan  (C) (D)
g g g cos  g sin 

KVPY_ SA STREAM # 39
8. Two guns are mounted (fixed) on two vertical cliffs that are very high from the ground as shown in figure.
The muzzle velocity of the shell from G 1 is u1 and that from G 2 is u2. The guns aim exactly towards
each other The ratio u1 : u2 such that the shells collide with each other in air is (Assume that there is
no resistance of air)

(A) 1 : 2 (B) 1 : 4
(C) will not collide for any ratio (D) will collide for any ratio
9. A ball is thrown upward at an angle of 30° with the horizontal and lands on the top edge of a building
that is 20 m away. The top edge is 5m above the throwing point. The initial speed of the ball in metre/
second is (take g = 10 m/s2) :
(A) 10 m/s (B) 20 m/s (C) 25 m/s (D) 30 m/s

10. A ball is thrown from bottom of an incline plane at an angle  from the inclined surface up the plane.
Another ball is thrown from a point on the inclined plane with same speed and at same angle from the
inclined surface down the plane. If in the two cases, maximum height attained by the balls with respect
to the inclined surface during projectile motion are h 1 and h2 then :
(A) h1 > h2 (B) h1 < h2
(C) h1 = h2 (D) All the three can be possible
11. Two projectiles, one fired from the surface of the earth with speed 5 m s–1 and the other fired from the
surface of a planet with initial speed 3 m s–1 trace identical trajectories. Neglecting friction, the value of
acceleration due to gravity on the planet is :
(A) 8.5 m s–2 (B) 5.9 m s–2 (C) 3.5 m s–2 (D) 16.3 m s–2
12. A stone is projected from level ground such that its horizontal and vertical components of initial velocity are
ux = 10 m/s and uy = 20 m/s respectively. Then the angle between velocity vector of stone one second before
and one second after it attains maximum height is :
(A) 30° (B) 45° (C) 60° (D) 90°
13. Stone is projected from level ground. At a height 0.4 m from the ground, the velocity of the stone in

vector form is   ( 6 î  2 ĵ ) m/s (the x-axis is horizontal and y-axis is vertically upwards). The angle of
projection with horizontal is (g = 10m/s2)
(A) 450 (B) 600 (C) 300 (D) tan–1(3/4)
14. Neglect air resistance in this problem. A cannon simultaneously fires two identical cannonballs at targets
1 and 2 as shown below. If the cannonballs have identical initial speeds, which of the following statement is
true ? Assume the point of projection and targets to be at same horizontal level.

2 1

(A) Target 1 is hit before target 2. (B) Target 2 is hit before target 1.
(C) Both are hit at the same time. (D) Data insufficient for inference
15. From an elevated point A, stone is projected vertically upwards. When the stone reaches a distance 'h'
below A its velocity is double of what it was at a height h above A. The greatest height attained by the
stone above A is
3h 5h
(A) 2 h (B) 3 h (C) (D)
5 3

KVPY_ SA STREAM # 40
16. A particle–1 is projected with speed v 1 from a point A making an angle of 30º with the vertical. At the same
instant, a second particle–2 is thrown vertically upwards from point B. Both the particles reach height H,
v1
simultaneously. The ratio of v is :
2

2 1 3
(A) (B) (C) (D) 3
3 3 2

17. Two persons X and Y are playing with two different balls of masses m and 2m. If X throws his ball vertically up
and Y at an angle  with vertical, both of them stay in air for the same time. The maximum heights attained
by the two are in the ratio :
(A) 2 : 1 (B) 1 : cos  (C) 1 : 1 (D) 1 : sec 

18. A gun is pointed towards a 100 m high target as shown in figure. Target is released at the same time
when the gun is shot. To hit the target in air, muzzle speed should not less than (g = 9.8 m/s 2)
Target

100 m

gun

100 m
(A) 7 2 m/s (B) 14 5 m/s (C) 10 m/s (D) 10 10 m/s

19. A fruit is falling with zero initial velocity from a 50 m high tree. A man, which is at 50 m horizontal distance
from the tree fire a bullet with a speed of 10 2 m/sec. at an angle 45º with horizontal. After how much time
will the bullet hit the fruit ?

(A) 2 sec. (B) 5 sec. (C) 4 sec. (D) None of these

20. A helicoptor is flying at an altitude of 200 m with 25 m/s at an angle of 37º above horizontal when a package
is dropped from it. After how much time (in sec) package hits the ground : (g = 10 m/s2)
(A) 7 (B) 8 (C) 9 (D) 10

KVPY_ SA STREAM # 41
KVPY PROBLEMS (PREVIOUS YEARS)

1. A boy standing on the foothpath tosses a ball straight up and catch it. The driver of a car passing by moving
with uniform velocity sees this. [KVPY_2009]

The trajectory of the ball as seen by the driver will be -

(A) (B) (C) (D)

2. According to the quantum theory, a photon of electromagnetic radiation of frequency  has energy
E = h where h is known as planck’s constant. According to the theory of relativity, a particle of mass m has
equivalent energy E = mc2, where c is speed of light. Thus a photon can be treated as a particle having
h
effective mass m = . If a flash of light is sent horizonatally in earth’s gravitational field, then photons while
C2
traveling a horizontal distance d would fall through a distance given by - [KVPY_2009 2 Marks]

gd2 h mcd2
(A) (B) (C) (D) zero
2c 2 mc h

3. A firecracker is thrown with velocity of 30 ms–1 in a direction which makes an angle of 75º with the vertical
axis. At some point on its trajectory, the firecracker split into two identical pieces in such a way that one
piece falls 27 m far from the shooting point. Assuming that all trajectories are contained in the same plane,
how far will the other piece fall from the shooting point ? (Take g = 10 ms–2 and neglect air resistance)
[KVPY_2012]
(A) 63 m or 144 m (B) 28 m or 72 m (C) 72 m or 99 m (D) 63 m or 117 m

EXERCISE-1
1. (C) 2.* (A) 3. (D) 4. (B) 5. (B) 6. (A) 7. (C)

8. (D) 9. (B) 10. (C) 11. (C) 12. (D) 13. (C) 14. (A)

15. (D) 16. (A) 17. (C) 18. (B) 19. (D) 20. (A)

EXERCISE-2
1. (C) 2. (A) 3. (D)

KVPY_ SA STREAM # 42
1. y = ax 2
dy dx
= c = 2ax
dt dt
2
d2 y  dx  d2 x
= 0 = 2a   + 2ax
dt 2  dt  dt 2
2 2
d2 x  dx  1  c  1 c2 c2
=–      =  2 3 = 
dt 2  dt  x  2ax  x 4a x 4a 2

2.* When particle collides with the plane, its component of velocity parallel to inclined plane should be zero.

u
0 = u cos – g sin t (along the plane equation) t = g cot 
 u = gt tan
2gh
u=
2  cos 2 

3. From graph
uy = tan 60° = 3 m/s
2 u x uy 2  ux  3
Range R = or 3 =
g g
or ux = 5 m/s

 u= u 2x  u2y = 28 m/s

4. Two second before maximum height v y = g × 2 = 20 cm/s


20
tan53° = v v x = 15 m/s
x
at maximum height v = v x = 15 m/s

5. The pebble is at same distance from top and base of the tower after it falls down by a distance h = 10 m.

v 2 = u2 + 2gh = 100 + 2 × 10 × 10 = 300


 v = 10 3 m/s.

KVPY_ SA STREAM # 43
6. The horizontal and vertical components of initial velocity of projectile are as shown in figure. Since the
observer moving with uniform velocity v sees the projectile moving in straight line
Hence v = u cos 

usin usin

A ucos A
B v B
g g

velocity of A and B given velocity of A given


from frame of ground from frame of B

The time of flight as measured by observer B is


2 u sin 
T=
g
Hence horizontal range of projectile on ground is
R = (u cos)T = vT

1 2u
7. Equation for motion perpendicular to the incline  u.t - g cos . t2  t = g cos 
2

2
1  2u 
Along incline x= 0+ × (g sin   
2  g cos  

8. As the relative acceleration is zero, the particles will meet for all values of u1 and u2 (Assuming they do
not collide with the ground. It is given that the cliffs are very high)

gx 2
9. y = x tan –
2u 2 cos 2 
10  400
5 = 20 tan30° –
2u  cos 2 30
2

 u  20 m/s

10. For both the particles

uy = usin  and ay = g cos 


So y motion will be similar for both the particles.
 Maximum height and time of flight will be same for the both.  h1 = h2

KVPY_ SA STREAM # 44
11. If trajectories are identical,
HE = HP
2 2
uE sin2  up sin2 
 =
2g 2a
2
up 9
 a= 2 g = × 9.8 = 3.5 m/s2 .
uE 25

12. (Easy) It is obvious from the figure shown that the angle between the velocity vectors 1 sec before and 1 sec
after it attains maximum height is 90°.


13. Let u  ( u  î  u y ĵ ) m/s be velocity of projection.

At height h = 0.4 m, v  (6 î  2 ĵ) m/s
As horizontal component of velocity remains constant.
ux = 6 m/s
Taking motion in y-direction.
v 2y  u 2y  2gh
2
(2)2 = u y – 2 × 10 × 0.4

u2y = 12

uy = 12
uy 12 1
hence, tan  =  
ux 6 3
 = 30°
14. (Easy) Time of flight and maximum height depends on vertical component of initial velocity. Time of flight is
less in case I so it hits target first.
15. Let v be the speed at a height h above A, the speed at height h below A is 2v.
If u is initial speed at A then
v 2 = u2 – 2gh and (2v)2 = u2 + 2gh
10
solving u2 = gh .... (1)
3
If H is max height above A, then
u2 = 2gH .... (2)
from 1 and 2
5
H= h.
3
16. Both reach the highest point same time, hence their initial vertical component must be same.
v 1cos30º = v2
v1 2
 v2 = 3 .

KVPY_ SA STREAM # 45
2 u1
17. For X : T1 =
g

2 u1 cos  2 u1
For Y : T2 = =  u1 = u2 cos 
g g

u12
Again, for X : H1 =
2g

u 22 sin 2 (90  ) u 22 cos 2 


for Y : H2 = =
2g 2g

H1 u12
 = 2 = 1:1
H2 u 2 cos 2 

18. In the limiting condition gun will hit the target just before it hits the ground

u2 sin 2  45º
 Range = 100 m = 100 m
g

 u
u= 100  9.8 = 14 5 m/s
45º
100 m

19. If we observe relative to the fruit,


Time to strike the bullet,

Srel 50 2
t= V = = 5 sec.
rel 10 2

But actual time of flight of the bullet is

1
2  10 2 
2u sin  2
g
= = 2 sec.
10

So, after 2 sec., the bullet will hit the ground, so it cannot hit the fruit.

1
20. – 200 = 25 sin 37º t – (10) t2\
2
 t = 8 sec.

KVPY_ SA STREAM # 46
R E L AT I V E M O T I O N


1 RELATIVE M OTION
Motion is a combined property of the object under study as well as the observer. It is always relative ; there
is no such thing like absolute motion or absolute rest. Motion is always defined with respect to an observer
or reference frame.
Reference frame :
Reference frame is an axis system from which motion is observed along with a clock attached to the axis, to
measure time. Reference frame can be stationary or moving.

 Suppose there are two persons A and B sitting in a car moving at constant speed. Two stationary persons C
and D observe them from the ground.

B A
D C

Here B appears to be moving for C and D, but at rest for A. Similarly C appears to be at rest for D but moving
backward for A and B.

2 RELATIVE M OTION IN ONE DIM ENSION :


2 .1 Re la tive Position :
It is the position of a particle w.r.t. observer.
In general if position of A w.r.t. to origin is xA and that of B w.r.t. origin is xB then “Position of A w.r.t.
B” xAB is
xB xAB
xAB = xA – xB xA

Origin B A

Example 1. See the figure (take +ve direction towards right and –ve towards left)
Find xBA , xCA , xCB , xAB and xAC .

A B C

4m 6m

10m
origin

Here, Position of B w.r.t. A is 4 m towards right . (xBA = +4m)


Position of C w.r.t. A is 10 m towards right . (xCA = +10m)
Position of C w.r.t. B is 6 m towards right (xCB = +6m)
Position of A w.r.t. B is 4 m towards left. (xAB = –4 m)
Position of A w.r.t. C is 10 m towards left. (xAC = –10m)

KVPY_ SA STREAM # 47
2 .2 Rela t ive Ve locit y
Definition : Relative velocity of a particle A with respect to B is defined as the velocity with which A
appears to move if B is considered to be at rest. In other words, it is the velocity with which A
appears to move as seen by B considering itself to be at rest.

NOTE 1 : All velocities are relative & have no significance unless observer is specified. However, when we say
“velocity of A”, what we mean is , velocity of A w.r.t. ground which is assumed to be at rest.
Relative velocity in one dimension -
If xA is the position of A w.r.t. ground, xB is position of B w.r.t. ground and xAB is position of A w.r.t. B then we
dx A
can say v A = velocity of A w.r.t. ground =
dt
dx B
v B = velocity of B w.r.t. ground =
dt
dx AB d
and v AB = velocity of A w.r.t. B = = ( x A  xB )
dt dt

dx A dx B
= –
dt dt
Thus

v AB = v A – v B

NOTE 2. : Velocity of an object w.r.t. itself is always zero.

Example 2. An object A is moving with 5 m/s and B is moving with 20 m/s in the same direction. (Positive x-axis)
(i) Find velocity of B with respect to A.

(ii) Find velocity of A with respect to B

Solution : (i) v B = +20 m/s v A = +5 m/s vBA = v B – v A = +15 m/s


(ii) v B = +20 m/s, v A = +15 m/s ; v AB = v A – v B = – 15 m/s

Note : vBA = – vAB

Example 3. Two objects A and B are moving towards each other with velocities 10 m/s and 12 m/s respectively
as shown.

(i) Find the velocity of A with respect to B.

(ii) Find the velocity of B with respect to A

Solution : v A = +10 , v B = – 12
(i) v AB = v A – v B = (10) – (–12) = 22 m/s.

(ii) v BA = v B – v A = (–12) – (10) = –22 m/s.

KVPY_ SA STREAM # 48

2 .3 Rela tive Acce lera t ion

It is the rate at which relative velocity is changing.

d v AB dv A dv B
aAB = = – = aA – aB
dt dt dt

Equations of motion when relative acceleration is constant.

v rel = urel + arel t

1
srel = urel t + a t2
2 rel

v 2rel = u2rel + 2arel srel

2 .4 Ve locity of Approa ch / Sepa ra tion

It is the component of relative velocity of one particle w.r.t. another, along the line joining them.

If the separation is decreasing, we say it is velocity of approach and if separation is increasing, then
we say it is velocity of separation.

In one dimension, since relative velocity is along the line joining A and B, hence velocity of approach
/ separation is simply equal to magnitude of relative velocity of A w.r.t. B.

Example 4. A particle A is moving with a speed of 10 m/s towards right and another particle B is moving at speed
of 12 m/s towards left. Find their velocity of approach.

Solution : VA = +10 , VB = – 12  VAB = VA – VB  10 – (–12) = 22 m/s


since separation is decreasing hence Vapp = |VAB| = 22 m/s

Ans. : 22 m/s

Example 5 A particle A is moving with a speed of 20 m/s towards right and another particle B is moving at a
speed of 5 m/s towards right. Find their velocity of approach.

Solution : VA = +20 , VB = +5

VAB = VA – VB

20 – (+5) = 15 m/s
since separation is decreasing hence Vapp = |VAB| = 15 m/s

Ans. : 15 m/s

KVPY_ SA STREAM # 49
Example 6. A particle A is moving with a speed of 10 m/s towards right, particle B is moving at a speed of
10 m/s towards right and another particle C is moving at speed of 10 m/s towards left. The separa-
tion between A and B is 100 m. Find the time interval between C meeting B and C meeting A.

100m
10m/s 10m/s

A B
10m/s

seperation between A and C 100


Solution : t= = = 5 sec.
Vapp of A and C 10 – ( 10 )
Ans. : 5 sec.

d d
Note : aapp =  v app , asep = v sep
 dt  dt

v app =  aapp dt , v sep =  asep dt


Example 7. A and B are thrown vertically upward with velocity, 5 m/s and 10 m/s respectively (g = 10 m/s2. Find
separation between them after one second
1 2
Solution : SA = ut – gt
2

1
= 5t – × 10 × t2
2

= 5 × 1 – 5 × 12
=5–5 =0

1 2 1
SB = ut – gt . = 10 × 1 – × 10 × 12 = 10 – 5 = 5
2 2

 SB – SA = separation = 5m.
Aliter :
  
aBA = a B – a A = (–10) – (–10) = 0
  
Also v BA = v B – v A = 10 – 5 = 5 m/s
 
 sBA (in 1 sec) = v BA × t

=5×1
=5m
 Distance between A and B after 1 sec = 5 m.

Example 8. A ball is thrown downwards with a speed of 20 m/s from the top of a
building 150 m high and simultaneously another ball is thrown vertically
upwards with a speed of 30 m/s from the foot of the building. Find the time
after which both the balls will meet. (g = 10 m/s2)

KVPY_ SA STREAM # 50
Solution : S1 = 20 t + 5 t2
S2 = 30 t – 5 t2
S1 + S2 = 150  150 = 50 t  t=3s
Aliter :
Relative acceleration of both is zero since both have same acceleration in downward direction
  
a AB = a A – aB =g–g=0

v BA = 30 – (–20) = 50

sBA = v BA × t

sBA 150
t= v = =3s
BA 50

Example 9. Two cars C1 and C2 moving in the same direction on a straight single lane road with velocities 12 m/
s and 10 m/s respectively. When the separation between the two was 200 m C2 started accelerating
to avoid collision. What is the minimum acceleration of car C2 so that they don’t collide.

Solution : Acceleration of car 1 w.r.t. car 2


    
a1 2  a1  a 2 = a C1 – a C2 = 0 – a = (–a)

  
u12 = u1 – u2 = 12 – 10 = 2 m/s.

The collision is just avoided if relative velocity becomes zero just at the moment the two cars meet
each other.
i.e. v 12 = 0 When s12 = 200
 
Now v 12 = 0, u12 = 2 , a1 2 = – a and s12 = 200

2 2
 v 12 – u12 = 2a12s12

1
 0 – 22 = – 2 × a × 200  a= m/s2 = 0.1 m/s2 = 1 cm/s2.
100

 Minimum acceleration needed by car C2 = 1 cm/s2


3. RELATIVE M OTION IN TW O DIM ENSION

rA = position of A with respect to O


rB = position of B with respect to O

rAB = position of A with respect to B.

KVPY_ SA STREAM # 51
    
rAB  rA  rB (The vector sum rA  rB can be done by  law of addition or resolution method)

  
d ( rAB ) d ( rA ) d ( rB )   
 = – .  v AB  v A  v B
dt dt dt

  
d (v AB ) d(v A ) d (v B )   
= –  a AB  a A  aB
dt dt dt

Example 10. Object A and B both have speed of 10 m/s. A is moving towards East while B is moving towards

North starting from the same point as shown. Find velocity of A relative to B ( v AB )

Solution : Method 1
  
v AB = v A – v B


 v AB = 102

Method 2
 
v A = 10 î , v B = 10 ĵ

  
v AB = v A – v B = 10 î – 10 ĵ


 v AB = 102

   
Note : v A  v B = v 2A  v B2  2v A v B cos  , where  is angle between v A and v B

Example 11. Two particles A and B are projected in air. A is thrown with
a speed of 30 m/sec and B with a speed of 40 m/sec as 40
shown in the figure. What is the separation between them vB
after 1 sec.
30
vA
     B
Solution : a AB = a A – aB = g  g = 0 A
53° 37°

 v AB = 30 2  40 2 = 50

 sAB = v ABt = 50 t = 50 m

KVPY_ SA STREAM # 52
Example 12. An old man and a boy are walking towards each other and a bird is flying over them as shown in the
figure.

s
12 m/s m/
20
37º
bird
16 m/s


2m/s
Tree
Boy 16 m/s old person

(1) Find the velocity of tree, bird and old man as seen by boy.
(2) Find the velocity of tree, bird and boy as seen by old man
(3) Find the velocity of tree, boy and old man as seen by bird.
Solution : (1) With respect to boy :
v tree = 16 m/s () or – 16 î

v bird = 12 m/s () or 12 ĵ

v old man = 18 m/s () or – 18 î


(2) With respect to old man :
v Boy = 18 m/s () or 18 î

v Tree = 2 m/s () or 2 î

v Bird = 18 m/s () and 12 m/s () or 18 î + 12 ĵ


(3) With respect to Bird :

v Tree = 12 m/s () and 16 m/s () or – 12 ĵ – 16 î

v old man = 18 m/s () and 12 m/s () or – 18 î – 12 ĵ

v Boy= 12 m/s (). or – 12 ĵ



3 .1 Rela tive M ot ion in Lift

Projectile m otion in a lift m oving with a ccele ra tion a upwa rds


(1) In the reference frame of lift, acceleration of a freely falling object is g + a
(2) Velocity at maximum height = u cos 

2u sin 
(3) T= ga

u 2 sin 2 
(4) Maximum height (H) =
2(g  a)

u 2 sin 2
(5) Range =
ga

KVPY_ SA STREAM # 53
Example 13 A lift is moving up with acceleration a. A person inside the lift throws the
ball upwards with a velocity u relative to hand.
(a) What is the time of flight of the ball?
(b) What is the maximum height reached by the ball in the lift?
  
Solution : (a) aBL = a B – aL =g+a

  1 
s = u t + aBL t2
2

1 2u
0 = uT – (g + a)T2  T = ( g  a)
2
(b) v 2 – u2 = 2 as

u2
0 – u2 = –2(g + a) H H=
2(g  a)


4. RELATIVE M OTION IN RIVER FLOW
 
If a man can swim relative to water with velocity v mR and water is flowing relative to ground with velocity v R ,

velocity of man relative to ground v m will be given by :
  
v mR = v m – v R
  
or v m = v mR + v R
  
If v R = 0, then v m = v mR
in words, velocity of man in still water = velocity of man w.r.t. river
4 .1 River Problem in One Dim ension :
 Velocity of river is u & velocity of man in still water is v.
Case - 1
Man swimming downstream (along the direction of river flow)
In this case velocity of river v R = + u
velocity of man w.r.t. river v mR = +v
   (u + v)
now v m = v mR + v R = u + v u

Case - 2
Man swimming upstream (opposite to the direction of river flow)

In this case velocity of river v R = – u

velocity of man w.r.t. river v mR = +v (v - u)
   u
now v m = v mR + v R = (v – u)

Example 14 A swimmer capable of swimming with velocity ‘v’ relative to water jumps in a flowing river having
velocity ‘u’ . The man swims a distance d down stream and returns back to the original position. Find
out the time taken in complete motion.
Solution : Total time = time of swimming downstream + time of swimming upstream
d d 2dv
t = tdown + tup = + = 2 Ans.
vu v u v  u2

KVPY_ SA STREAM # 54

4 .2 M otion of M a n Swim m ing in a River

Consider a man swimming in a river with a velocity of v MR relative to river at an angle of  with the
river flow

The velocity of river is VR .
Let there be two observers  and  , observer  is on ground and observer  is on a raft floating along
with the river and hence moving with the same velocity as that of river. Hence motion w.r.t. observer
 is same as motion w.r.t. river. i.e. the man will appear to swim at an angle  with the river flow for
observer  .
  
For observer  the velocity of swimmer will be v M = v MR + v R ,
Hence the swimmer will appear to move at an angle ’ with the river flow.

Observer  vMR
d
-- -- -- -- -- -- -- -- -- -- -- -- -- --
-- -- -- -- -- -- -- -- -- -- -- -- -- --
--------------

Observer 

 : Motion of swimmer for observer 


 : Motion of swimmer for observer 

4 .3 River problem in two dim ension (crossing river) :



Consider a man swimming in a river with a velocity of v MR relative to river at an angle of  with the
river flow
The velocity of river is VR and the width of the river is d
x
  
v M = v MR + v R
y

 v M = (v MRcos î + v MRsin ĵ ) +v R î d VMR VM
x
 
 v M = (v MRcos + v R) î + v MRsin ĵ
VR
Here v MRsinis the component of velocity of man in the direction perpendicular to the river flow.
This component of velocity is responsible for the man crossing the river. Hence if the time to cross
the river is t , then
d d
t= = v sin 
vy MR

D R I FT
It is defined as the displacement of man in the direction of river flow. (see the figure).
It is simply the displacement along x-axis, during the period the man crosses the river. (v MRcos + vR)is the
component of velocity of man in the direction of river flow and this component of velocity is responsible for drift
along the river flow. If drift is x then,
Drift = v x × t
d
x = (v MRcos + v R)×
v MR sin 

KVPY_ SA STREAM # 55
4 .4 Crossing the river in shortest tim e
d
As we know that t = . Clearly t will be minimum when  = 90° i.e. time to cross the river
v MR sin 

d
will be minimum if man swims perpendicular to the river flow. Which is equal to .
v MR
4 .5 Crossing the river in shortest pa th, M inim um Drift
The minimum possible drift is zero. In this case the man swims in the direction perpendicular to the
river flow as seen from the ground. This path is known as shortest path
here xmin = 0  (v MRcos + v R) = 0
vR
or cos = –
v MR
 since cos  is – ve,   > 90° , i.e. for minimum drift the man must swim at some angle  with the
perpendicular in backward direction.
vR
Where sin  =
v MR

1  v R  vR
  = cos  v  
 v MR < 1 i.e. v R < v MR
 MR 
i.e. minimum drift is zero if and only if velocity of man in still water is greater than or equal to the velocity of
river.

 Time to cross the river along the shortest path


VR
d d
t= =
VMR
v MR sin  2
v MR  VR2
VM

Note :
 If vR > vMR then it is not possible to have zero drift. In this case the minimum drift
(corresponding to shortest possible path is non zero and the condition for minimum drift
v MR v
can be proved to be cos = – or sin  = MR for minimum but non zero drift.
vR vR

Example 15. A 400 m wide river is flowing at a rate of 2.0 m/s.A boat is sailing with a velocity of 10 m/s with
respect to the water, in a direction perpendicular to the river.
(a) Find the time taken by the boat to reach the opposite bank.
(b) How far from the point directly opposite to the starting point does the boat reach the opposite
bank.
(c) In what direction does the boat actually move, with river flow (downstream).

Solution :

KVPY_ SA STREAM # 56
(a) time taken to cross the river
d 400 m
t= v = = 40 s Ans.
y 10 m/ s
(b) drift (x) = (v x)(t) = (2m/s) (40s) = 80 m Ans.
(c) Actual direction of boat,
 10 
 = tan–1   = tan–1 5, (downstream) with the river flow..
 2 

Example 16. A man can swim at the rate of 5 km/h in still water. A 1 km wide river flows at the rate of 3 km/h. The
man wishes to swim across the river directly opposite to the starting point.
(a) Along what direction must the man swim?
(b) What should be his resultant velocity?
(c) How much time will he take to cross the river?
Solution : The velocity of man with respect to river v mR= 5 km/hr, this is greater than the river flow velocity,
therefore, he can cross the river directly (along the shortest path). The angle of swim must be

  vr   vr  3
= + sin–1  v  = 90º + sin-1  v  = 90º + sin–1   = 90º + 37º
2  mR   mR  5
= 127º w.r.t. the river flow or 37° w.r.t. perpendicular in backward direction Ans.
2
(b) Resultant velocity will be v m = v mR  v R2 = 5 2  3 2 = 4 km/hr
along the direction perpendicular to the river flow.
(c) time taken to cross the
d 1 km 1
t= = = h = 15 min
2
v mR  v R2 4 km / hr 4

Example 17. A man wishing to cross a river flowing with velocity u jumps at an angle with the river flow.
(i) Find the net velocity of the man with respect to ground if he can swim with speed v in still water.
(ii) In what direction does the man actually move.
(iii) Find how far from the point directly opposite to the starting point does the man reach the
opposite bank, if the width of the river is d. (i.e. drift)
Solution : (i) v MR = v , v R = u
  
v M = v MR + v R

 Velocity of man , v M = u 2  v 2  2v u cos  Ans.

v sin 
(ii) tan  = Ans.
u  v cos 
(iii) (v sin) t = d
v M  v MR  v R
d
 t=
v sin 
x = (u + v cos ) t
d
= (u + v cos) Ans.
v sin 
Example 18. A boat moves relative to water with a velocity v which is n times less than the river flow velocity u. At
what angle to the stream direction must the boat move to minimize drifting?
Solution : (In this problem, one thing should be carefully noted that the velocity of boat is less than the river flow
velocity. Hence boat cannot reach the point directly opposite to its starting point. i.e. drift can never
be zero.)
Suppose boat starts at an angle  from the normal direction up stream as shown.
Component of velocity of boat along the river, v x = u – v sin 
and velocity perpendicular to the river, v y = v cos .
d d
time taken to cross the river is t = v = .
y v cos 
Drift x = (v x)t

KVPY_ SA STREAM # 57
d B drift = x C
= (u – v sin )
v cos 
y
ud d u
= sec  – d tan  x
v v cos
v 
dx
The drift x is minimum, when = 0, A u-v sin
d

 ud 
or   (sec  . tan ) – d sec2  = 0
 v 
u
or sin  = 1
v
v
or sin  =
u

This is the result we stated without proof as a note in section 4.5


v
so, for minimum drift, the boat must move at an angle  = sin–1   = sin–1 1 from normal direction.
u n

5. W IND AIRPLANE PROBLEM S
This is very similar to boat river flow problems. The only difference is that boat is replaced by aeroplane and
river is replaced by wind.
Thus,
velocity of aeroplane with respect to wind
  
v aw  v a  v w
  
or v a  v aw  v w

where, v a = velocity of aeroplane w.r.t. ground

and, v w = velocity of wind.

Example 19 An aeroplane flies along a straight path A to B and returns back again. The distance between A and
B is  and the aeroplane maintains the constant speed v w.r.t. wind. There is a steady wind with
a speed u at an angle  with line AB. Determine the expression for the total time of the trip.
Solution : Suppose plane is oriented at an angle  w.r.t. line AB while the plane is moving from A to B :

Velocity of plane along AB = v cos – ucos ,

and for no-drift from line AB


u sin 
v sin = usin  sin =
v

time taken from A to B : tAB =
v cos   u cos 
Suppose plane is oriented at an angle ’ w.r.t. line AB while the plane is moving from B to A :

'

velocity of plane along BA = vcos + u cos and for no drift from line AB
vsin = usin

KVPY_ SA STREAM # 58
u sin 
 sin =
v
  = ’

time taken from B to A : tBA =
v cos   u cos 
 
total time taken = tAB + tBA = +
v cos   u cos  v cos   u cos 

u 2 sin2 
2 v 1 
2v cos  v2
= = .
v 2 cos 2  – u2 cos 2  v 2  u2

Example 20. Find the time an aeroplane having velocity v, takes to fly around a square with side a if the wind is
blowing at a velocity u along one side of the square.
2a  v  v 2  u2 
Answer : 2 2  
v u  

D C
Solution :
vwind= u

A B

Velocity of aeroplane while flying through AB vA=v+u


vA = v + u
a
tAB =
vu

Velocity of aeroplane while flying through BC u


v A  v 2  u2
vA = v
v 2  u2
a
tBC =
v 2  u2
Velocity of aeroplane while flying through CD v u
vA = v – u vA=v – u
a
tCD =
v–u
Velocity of aeroplane while flying through DA
v
v A  v 2  u2
vA = 2
v u 2

u
a
tDA =
v 2  u2

Total time = tAB + tBC + tCD + tDA


a a a a 2a  v  v 2  u2 
=    = 2  
vu 2
v u 2 v  u 2
v u 2 v  u2  

KVPY_ SA STREAM # 59

6. RAIN PROBLEM
 
If rain is falling vertically with a velocity v R and an observer is moving horizontally with velocity v m , the
velocity of rain relative to observer will be :
  
v Rm = v R – v m or v Rm = v R2  v m
2

 vm 
and direction  = tan–1  v  with the vertical as shown in figure.
 R

–vm


vRm
vR

Example 21 Rain is falling vertically at speed of 10 m/s and a man is moving with velocity 6 m/s. Find the angle
at which the man should hold his umbrella to avoid getting wet.

Solution :

  
v rain = –10 ĵ v man = 6 ĵ


v r.w.r.t. man = –10 ĵ – 6 î

6 3
tan  =   = tan–1  
10 5
Where  is angle with vertical

Example 22 A man moving with 5m/s observes rain falling vertically at the rate of 10 m/s. Find the speed and
direction of the rain with respect to ground.

KVPY_ SA STREAM # 60
Solution : v RM = 10 m/s, v M = 5 m/s
  
v RM = v R – v M
   
 v R = v RM + v M  vR = 5 5

1 1
tan  = ,  = tan–1 .
2 2

Example 23. A standing man, observes rain falling with velocity of 20 m/s at an angle of 30º with the vertical.
(i) Find the velocity with which the man should move so that rain appears to fall vertically to him.
(ii) Now if he further increases his speed, rain again appears to fall at 30º with the vertical. Find his
new velocity.

Solution : (i) v m = –v î (let)


v R = –10 î – 103 ĵ


v RM = –(10 – v) î – 103 ĵ

 – (10 – v) = 0 (for vertical fall, horizontal component must be zero)


or v = 10 m/s Ans.

(ii) v R = –10 î – 103 ĵ

v m = –v x î


v RM = – (10 – v x) î – 103 ĵ
Angle with the vertical = 30º

10  v x
 tan 30º =  v x = 20 m/s
 10 3

KVPY_ SA STREAM # 61
1. A flag on a bus is fluttering in north direction & wind is blowing in east direction. Then which of the following
will be true -
(A) bus is moving in south direction.
(B) bus is moving in north east direction.
(C) bus may be moving in any direction between south & east.
(D) bus may be moving in any direction between south & west.

2. Two aeroplanes fly from their respective position 'A' and 'B' starting at
the same time and reach the point 'C' (along straight line) simultaneously
when wind was not blowing. On a windy day they head towards 'C' but
both reach the point 'D' simultaneously in the same time which they
took to reach 'C'. Then the wind is blowing in
(A) North-East direction
(B) North-West direction
(C) Direction making an angle 0 <  < 90 with North towards West.
(D) North direction

3. A man starts running along a straight road with uniform velocity u î observes that the rain is falling vertically
downward. If he doubles his speed, he finds that the rain is coming at an angle  to the vertical. The velocity
of rain with respect to the ground is :
u
(A) u î  u tan  ĵ (B) u î  u cot  ĵ (C) u î  u cot  ĵ (D) î  u ĵ
tan 
4. For four particles A, B, C & D, the velocities of one with respect to other are given as V DC is 20 m/s towards
north, V BC is 20 m/s towards east and V BA is 20 m/s towards south. Then V DA is
(A) 20 m/s towards north (B) 20 m/s towards south
(C) 20 m/s towards east (D) 20 m/s towards west

5. Two towns A and B are connected by a regular bus service with a bus leaving in either direction every
T minutes with same speed. A man cycling with speed of 20km/h in the direction A to B, notices that a bus
goes past him every t1 = 18minutes in the direction of motion, and every t2 = 6 minutes in the opposite
direction. What is the period T of the bus service? Assume that velocity of cyclist is less than velocity of bus:
(A) 4.5 minutes (B) 24 minutes (C) 9 minutes (D) 12 minutes

6. Car A and car B move on a straight road and their velocity versus time graphs are as shown in figure.
Comparing the motion of car A in between t = 0 to t = 8 sec. and motion of car B in between t = 0 to
t = 7 sec., pick up the correct statement.

v (m/s) v (m/s)

10 m/s 10 m/s

t(s) t(s)
t=2s t=8s t=3s t=7s
Car A Car B

(A) Distance travelled by car A is less than distance travelled by car B.


(B) Distance travelled by car A is greater than distance travelled by car B.
(C) Average speed of both cars are equal.
(D) Average speed of car A is less than average speed of car B.

KVPY_ SA STREAM # 62
7. It is raining with a velocity of 5 2 m/s at an angle of 45° with vertical. To a man driving with constant velocity
‘v’ down the inclined plane having angle  rain appears to move vertically upwards with a velocity of 5 m/s.
Then v and  are :
1
(A) 5 m/s , tan–1 5 (B) 5 5 m/s, tan–1 5 (C) 5 5 m/s, tan–1 (D) 5 5 m/s, tan–1 2
2

8. Two balls are projected from points A and B in a vertical plane as shown in
V1
figure. AB is a straight vertical line. The balls will collide in mid–air if is
V2
equal to :
sin 2 sin 1
(A) sin  (B)
sin  2
1

cos  2 cos 1
(C) (D)
cos 1 cos  2

9. In given figure, a smooth square platform ABCD is moving towards right with a uniform speed 10 3 m/s. At
what angle  must a particle be projected from B with speed 20 m/s so that it strikes the point A?

 2   2 
(A) 60º (B) cos–1  
 (C) 30º (D) sin–1  

 3  3

10.* In the figure shown a particle is thrown vertically upwards with a speed 35 2
10m/s
m/s from the bottom most point of the wedge. At the same time the wedge
is started to move horizontally right wards with a constant acceleration 10
35 m/s
m/s2 , then the time after which the particle strikes the wedge is (Taking g
= 10 m/s2)
37° A
(A) 4s (B) 28/3 s
(C) 28/5 s (D) 7 s

11. In a river of 20 m width. Half part of river flows with speed 10 m/sec
and remaining half part flows with 20 m/sec. as shown in figure. A
man starts to swim from A and reaches to point B in 10 sec. Man
swims with speed Vm with respect to river at an angle  with line
AB. Then, angle  will be : (man swims with constant speed and
in same direction with respect to river throughout the motion) :
 15 
(A) tan–1(5) (B) tan–1  
 2 

 2   1
(C) cos–1   (D) tan–1  
 15  5

12. A ship is selling due north with speed 9 m/s in sea. Wind is blowing with speed 2 m/s from S–W direction.
A person holding a flag in his hand running on the deck of ship with speed 3 m/s due east relative to ship.
Direction in which flag will flutter :
(A) tan–1(2) W of N (B) tan–1(2) S of E (C) tan–1(2) N of W (D) none of these

KVPY_ SA STREAM # 63
13. Two particle A & B are moving with velocity ( î  ĵ  2 3k̂ )m/s and (  2 î  ĵ  3k̂ ) m/s respectively. At an

instant A is crossing the point (1,3, 3 ) m & at the same instant B is crossing the point (–2,1, 2 3 ) then the
rate at which separation between them decreases at the given instant is :
(A) –2.5 m/s (B) 2.5 m/s (C) –4.5 m/s (D) 4.5 m/s

14. A man can swim certain distance in still water up and down in time t1. If he swims to some distance down
stream the river and returns back to same point he takes time t2. Then :
(A) t1 = t2 (B) t1 < t2 (C) t1 < t2 (D) t1 & t2 can't be compared

15.* A person is standing on a plank which starts slipping over a fixed


smooth wedge from rest position hence person also moves with
1
plank. After second of motion begins he observed that rain is
2

7
falling vertically down ward. After sec. he again observed that
3
rains is coming horizontally. From the given information find the
speed of rain w.r.t. ground. (use g = 10 m/s2)
(A) 4 m/s (B) 6 m/s
(C) 12 m/s (D) 10 m/s
16. An airplane flies between two cities separated by a distance D. Assume the wind blows directly from
one city to the other at a speed VA (as shown) and the speed of the airplane is Vo relative to the air. Find
the time taken by the airplane to make a round trip between the two cities (that is, to fly from city A to
city B and then back to City A) ?

2DVo DVo 2DVo DVo


(A) (B) (C) (D)
V02  VA2 V02  VA2 V02  VA2 V02  VA2

17. An aeroplane is flying in vertical plane at an angle of 300 with the horizontal (north) and wind is blowing from
west. A package is dropped from an aeroplane. The velocity of the wind if package hits a kite flying in the

space with a position vector R  ( 400 3 î  80 ĵ  200k̂ ) m with respect to the point of dropping.

(Here î and ĵ are the unit vectors along north and vertically up respectively and k̂ be the unit vector due
east. Assume that the bag is light enough to get carried away by the wind) :
(A) 50 m/sec (B) 25 m/sec (C) 20 m/sec (D) 10 m/sec
18. Person A observes B moving in east direction with speed 10 m/s, B observes C moving in south direction with
speed 20 m/s, C observes D moving in west direction with speed 30 m/s & D observes a tree moving with
speed 40 m/s in north direction. Then the actual direction of motion of person ‘A’ will be -
(A) north - west (B) north - east (C) south - east (D) none of these
19. A ship is travelling due east at 10 km/h. A ship heading 30° east of north is always due north from the first
ship. The speed of the second ship in km/h is -
(A) 20 2 (B) 20 3 / 2 (C) 20 (D) 20/ 2

20. A body is thrown up in a lift with a velocity u relative to the lift and the time of flight is found to be ‘ t ‘.
The acceleration with which the lift is moving up is :
u  gt 2u  g t u  gt 2u  g t
(A) (B) (C) (D)
t t t t

KVPY_ SA STREAM # 64
KVPY PROBLEMS (PREVIOUS YEARS)
1. A ball is thrown vertically upwards with a certain initial velocity. Assume that there is no resistance due to air.
Among the graphs below, the graph that is not an appropriate representation of the motion of the ball is :
[KVPY_2008]

Potential Energy
Potential Energy
Kinetic Energy

Kinetic Energy
(i) (ii) ) (iii) (iv)

Time Time Height of the ball Speed of the ball

(A) A (B) B (C) C (D) D

2.. A girl standing at point P on a beach wishes to reach a point Q in the sea as quickly as possible. She can
run at 6 kmh–1 on the beach and swim at 4 km-h–1 in the sea. She should take the path [KVPY_2011]

(A) PAQ (B) PBQ (C) PCQ (D) PDQ

3. A boat crossing a river moves with a velocity v relative to still water. The river is flowing with a velocity v/2 with
respect to the bank. The angle with respect to the flow direction with which the boat should move to minimize
the drift is : [KVPY_2012]
(A) 30º (B) 60º (C) 150º (D) 120º

4. Two stones of mass m1 and m2 (such that m1 > m2) are dropped t time apart from the same height towards
the ground. At a later time t the difference in their speed is V and their mutual separation is S. While both
stones are in flight [KVPY_2013]
(A) V decreases with time and S increases with time
(B) Both V and S increase with time
(C) V remains constant with time and S decreases with time
(D) V remains constant with time and S increases with time

EXERCISE - 1
1. (C) 2. (B) 3. (C) 4. (D) 5. (C) 6. (D) 7. (D)
8. (A) 9. (A) 10.* (A) 11. (B) 12. (D) 13. (C) 14. (B)
15.* (B) 16. (A) 17. (B) 18. (C) 19. (C) 20. (B)

EXERCISE - 2
1. (D) 2. (C) 3. (D) 4. (D)

KVPY_ SA STREAM # 65
1. (moderate) Flag will flutter in the direction of wind with respect to bus.

and V WB = V W – V B = V W + (– V B )
(Addition of two vector always lies between them)

2. Without wind A reaches to C and with wind it reaches to D in same


time so wind must deflect from C to D so wind blow in the direction of
CD
  
VAG  VAW  VWG
  
 VAG t  VAW t  VWG t

AC = VAW t

CD = V WG t

3. v RM = v R  v m

v RM = v R  u î = (VRX î + VRY ĵ ) – u î

Since , v RM has only y component with respect to the man.


so, VRX – u = 0
 VRX = u
After doubling the speed -
v Rm´ = (u î + v Ry ĵ ) – 2u î

= – u î +v Ry ĵ

u
given tan  = v
Ry

 v Ry = u cot 
so v R = u î + u cot  ( ĵ )

4. Let î and ĵ be unit vectors in direction of east and north respectively..

 V DC  20 ĵ , V BC  20 î and V BA  20 ĵ

 V DA  V DC  V CB  V BA = 20 ĵ  20 î  20 ĵ =  20 î  V DA   20 î

5. Let velocity of bus is V. Then seperation between two nearest buses is VT


A bus moving in same direction travels the distance VT in 18 minutes to meet the cyclist.
srel
so t=
v rel

18 VT
 hrs  ...(1) (Vrel = V – 20)
60 ( V  20)
A bus moving in opposite direction travels the distance VT in 6 minutes tomeet the cyclist.

KVPY_ SA STREAM # 66
Srel
So t= V
rel

6 VT
  ...(2) (here Vrel = V + 20)
60 V  20
from (1) & (2)
V  20
3=  V = 40 Km/hr.
V  20
from (1)
18 40T

60 (40  20 )
18 40
  T
60 20
9 9
 T= hrs =  60 minutes = 9 minutes
60 60

1
6. Distance travelled by A = s = 10 × 2 + × 10 × 8 = 60
2
1
Distance travelled by B = s = 10 × 3 + × 10 × 4 = 50
2
60
Average speed of A = = 7.5
8
50
Average speed of B = = 7.1
7

  
7. v rm = v r  v m
  
v rm = v r  (  v m )
 
v r = 5 2 m/s v r m = 5 m/s
Let v m = x
x= BC 2  AC 2 = 5 5 m/s
BC
tan  = =2
AC
  = tan–1 2

8. V2sin2 = V1sin1 (horizontal component should be same)


V1 sin  2
 V2
= sin 1
.

9. For collision,
10 3 = 20 sin
 = 60º.

2u  2  35 sin 53
10.* (a) T = = = 4 sec
a 10 cos 53  10 cos 37

KVPY_ SA STREAM # 67
11. 10(Vmcos) = 20
Vmcos = 2m/sec
Vmsin = 15 m/sec.
15
tan = .
2

12. Vs = 9 ĵ

 Î  ĵ 
Vw = 2  = Î  ĵ Vms = 3Î = Vm –Vs
 2 

Vm = 3Î +Vs = 3 Î  9 ĵ Vwm = Vw– Vm

= ( î  ĵ ) – [3 î  9 ĵ]

=  2 î – 8 ĵ

13. VA = Î  Ĵ  K̂

VB  –2 Î  Ĵ  3 K̂

rA = (1,3, 3) rB = (–2,1, 2 3 )

BA  rAB =  3 Î  2Ĵ – 3 K̂ | AB | = 943 = 16 = 4

VA .AB 11
VA cos  A  = –
| AB | 4

VB .BA
VB cos B 
| BA |

7
VBcosB = –
4
11 7 18
VAcosA + VBcosB = – – – = – 4.5m/s
4 4 4

2d d d 2vd 2d
14. t1 = t2 =  = 2 2 =
v v u v u v u u2
v2 
v
2d
t2 =  t2 > t1 .
 u2 
v 1  2 
 v 

15.* Vrx = V1 cos 37º = (g sin 37º t1) cos 37º

Vry = V sin 37º = g sin2 37º t


2 2

Vr = Vr2x  Vr2y

10  3
=
25 16 t 12  9 t 22

6
= 4  21  6m / s
5

KVPY_ SA STREAM # 68
16. The speed of the plane as it goes from city A to city B is Vo + VA and the speed of the plane as it goes from
city B to city A is VoVA . Therefore the time taken by the plane to go once round the trip is

D D 2DVo
t=  = 2
Vo  VA Vo  VA V0  VA2


17. Let velocity of the aeroplane be v P  u cos 30 0 î  u sin 30 0 ĵ and velocity of the wind be v, then

3 u 
u t î   t  5t 2  ĵ  vtk̂ = 400 3 î  80 ĵ  200k̂
2 2 

3 u
 u t = 400 3 , t – 5t 2 = 80, vt = 200
2 2

u
ut = 800 and t – 5t 2 = 80
2
 400 – 5t 2 = 80
 t 2 = 64  t = 8 sec.

200
 v= = 25 m/s
8

18. All the velocities are marked in diagram where G represents ground

adding we get

Then V GD  V DC  V CB  V BA = V GA   V AG

Hence velocity of A is towards south east.

KVPY_ SA STREAM # 69
N EWT ON ’S LA W OF M OT I ON

1. FOR CE
A pull or push which changes or tends to change the state of rest or of uniform motion or direction of motion
of any object is called force. Force is the interaction between the object and the source (providing the pull
or push). It is a vector quantity.

Effect of resultant force :


(1) may change only speed
(2) may change only direction of motion.
(3) may change both the speed and direction of motion.
(4) may change size and shape of a body
kg m
Unit of force : Newton and (MKS System)
s2
g  cm
dyne and (CGS System)
s2
1 Newton = 105 dyne

Kilogram force (kgf)


The force with which earth attracts a 1kg body towards its centre is called kilogram force, thus
Forece in newton
kgf =
g

Dimensional Formula of force : [M L T–2 ]


1 .1 Funda m e nta l Force s
All the forces observed in nature such as muscular force, tension, reaction, friction, elastic, weight,
electric, magnetic, nuclear, etc., can be explained in terms of only following four basic interactions:

[A] Gr a vi t a t i o n a l Fo r c e
The force of interaction which exists between two particles of masses m1 and m2, due to their
masses is called gravitational force.

 mm 
F  G 13 2 r
r
 = position vector of test particle ‘T’ with respect to source particle ‘S’. and G = univ ersal
r
gravitational constant
= 6.67 × 10–11 Nm2/kg2.
(i) It is the weakest force and is always attractive.
(ii) It is a long range force as it acts between any two particles situated at any distance in the
universe.
(iii) It is independent of the nature of medium between the particles.

An apple is freely falling as shown in figure, When it is at a height h, force between earth and apple
is given by

KVPY_ SA STREAM # 70
GM em
F= where Me  mass of earth, R e  radius of earth
(R e  h) 2
It acts towards earth’s centre. Now rearranging above result,

GMe  Re 
2
 Re 
2  GM e 
F=m. 2 .  
  F = mg  
 g  2 
Re  Re  h   Re  h   R e 

Re
Here h << Re, so 1
Re  h
 F = mg
This is the force exerted by earth on any particle of mass m near the earth surface. The value of
g = 9.81 m/s2 ~ 10 m/s ~   m/s ~  32 ft/s . It is also called acceleration due to gravity near the
2 2 2 2

surface of earth.

[B] El e c t r o m a g n e t i c Fo r c e

Force exerted by one particle on the other because of the electric charge on the particles is called
electromagnetic force.
Following are the main characteristics of electromagnetic force
(a) These can be attractive or repulsive.
(b) These are long range forces
(c) These depend on the nature of medium between the charged particles.
(d) All macroscopic forces (except gravitational) which we experience as push or pull or by
contact are electromagnetic, i.e., tension in a rope, the force of friction, normal reaction,
muscular force, and force experienced by a deformed spring are electromagnetic forces.
These are manifestations of the electromagnetic attractions and repulsions between atoms/
molecules.

[ C] N u c l e a r Fo r c e

It is the strongest force. It keeps nucleons (neutrons and protons) together inside the nucleus
inspite of large electric repulsion between protons. Radioactivity, fission, and fusion, etc. result
because of unbalancing of nuclear forces. It acts within the nucleus that too upto a very small
distance.

[D ] W e a k Fo r c e

It acts between any two elementary particles. Under its action a neutron can change into a proton
emitting an electron and a particle called antineutrino. The range of weak force is very small, in fact
much smaller than the size of a proton or a neutron.
It has been found that for two protons at a distance of 1 Fermi :
F N:F EM:F W :F G ::1:10 -2:10 -7:10 -38

1 .2 Cla ssif ica t ion of f orce s on t h e b a sis of con t a ct :


(A) Field Force:
Force which acts on an object at a distance by the interaction of the object with the field produced
by other object is called field force. Examples
(a) Gravitation force
(b) Electromagnetic force
(B) Cont a ct Force :

Forces which are transmitted between bodies by short range atomic molecular interactions are
called contact forces. When two objects come in contact they exert contact forces on each other.

KVPY_ SA STREAM # 71
Examples:
(a) Normal force (N):
It is the component of contact force perpendicular to the surface. It measures how strongly the
surfaces in contact are pressed against each other. It is the electromagnetic force.
A table is placed on Earth as shown in figure

Here table presses the earth normal force exerted by four legs of table on earth are as shown in
figure.

Now a boy pushes a block kept on a frictionless surface.

Here, force exerted by boy on block is electromagnetic interaction which arises due to similar
charges appearing on finger and contact surface of block, it is normal force.

A block is kept on inclined surface. Component of its weight presses the surface perpendicularly
due to which contact force acts between surface and block.

Normal force exerted by block on the surface of inclined plane is shown in figure.

Force acts perpendicular to the surface

Example 1. Two blocks are kept in contact on a smooth surface as shown in figure. Draw normal force exerted by
A on B.

Solution : In above problem, block A does not push block B, so there is no molecular interaction between A and
B. Hence normal force exerted by A on B is zero.

KVPY_ SA STREAM # 72
Note :
 Normal is a dependent force, it comes in role when one surface presses the other.

(b) Tension :

Tension in a string is a electromagnetic force. It arises when a string is pulled. If a


massless string is not pulled, tension in it is zero. A string suspended by rigid support
is pulled by a force ‘F’ as shown in figure, for calculating the tension at point ‘A’ we
draw F.B.D. of marked portion of the string; Here string is massless.
F.B.D. of marked portion
T

 T=F
String is considered to be made of a number of small segments
which attracts each other due to electromagnetic nature as shown
in figure. The attraction force between two segments is equal and
opposite due to Newton’s third law.

For calculating tension at any segment, we consider two or more than two parts as a system.

Here interaction between segments are considered as internal forces, so they are not shown in
F.B.D.
( C) Frictiona l force :
It is the component of contact force tangential to the surface. It opposes the relative motion (or
attempted relative motion) of the two surfaces in contact.
2. THIRD LAW OF M OTION :
To every action, there is always an equal and opposite reaction. Newton’s law from an 1803 translation from
Latin as Newton wrote
“To every action there is always opposed an equal and opposite reaction: to the mutual actions of two bodies upon
each other are always equal, and directed to contrary parts.”
2 .1 Im porta nt points a bout the Third La w

(a) The terms ‘action’ and ‘reaction’ in the Third Law mean nothing else but ‘force’. A simple and
clear way of stating the Third Law is as follows : Forces always occur in pairs. Force on a
body A by B is equal and opposite to the force on the body B by A.

KVPY_ SA STREAM # 73
(b) The terms ‘action’ and ‘reaction’ in the Third Law may give a wrong impression that action
comes before reaction i.e. action is the cause and reaction the effect. There is no such
cause-effect relation implied in the Third Law. The force on A by B and the force on B by A
act at the same instant. Any one of them may be called action and the other reaction.
(c) Action and reaction forces act on different bodies, not on the same body. Thus if we are
considering the motion of any one body (A or B), only one of the two forces is relevant. It is
an error to add up the two forces and claim that the net force is zero.
However, if you are considering the system of two bodies as a whole, FAB (force on A due to
B) and FBA (force on B due to A) are internal forces of the system ( A + B). They add up to
give a null force. Internal forces in a body or a system of particles thus cancel away in pairs.
This is an important fact that enables the Second Law to be applicable to a body or a
system of particles.

3. SYSTEM :
Two or more than two objects which interact with each other form a system.

3 .1 Cla ssifica tion of force s on the ba sis of bounda ry of syste m :

(A) Int e rna l Force s:

Forces acting each with in a system among its constituents.

(B) Ext erna l Force s:

Forces exerted on the constituents of a system by the outside surroundings are called as external
forces.

( C) Re a l Force :

Force which acts on an object due to other object is called as real force. An isolated object (far away
from all objects) does not experience any real force.

4. FREE BODY DIAGRAM


A free body diagram consists of a diagrammatic representations of a single body or a subsystem of bodies
isolated from its surroundings showing all the forces acting on it.

4 .1 Ste ps for F.B.D .

Step 1: Identify the object or system and isolate it from other objects clearly specify its boundary.

Step 2: First draw non-contact external force in the diagram. Generally it is weight.
Step 3: Draw contact forces which acts at the boundary of the object or system. Contact forces are
normal, friction, tension and applied force.
In F.B.D, internal forces are not drawn, only external are drawn.

Example 2. A block of mass ‘m’ is kept on the ground as shown in figure.


(i) Draw F.B.D. of block.

(ii) Are forces acting on block action–reaction pair.

(iii) If answer is no, draw action reaction pair.

KVPY_ SA STREAM # 74
Solution : (i) F.B.D. of block

(ii) ‘N’ and mg are not action-reaction pair. Since pair act on different bodies, and they are of same
nature.
(iii) Pair of ‘mg’ of block acts on earth in opposite direction.

and pair of ‘N’ acts on surface as shown in figure.

Example 3. Two sphere A and B are placed between two vertical walls as shown in figure. Draw the free body
diagrams of both the spheres.

Solution : F.B.D. of sphere ‘A’ :

F.B.D. of sphere ‘B’ :

(exerted by A)

Note : Here NAB and NBA are the action–reaction pair (Newton’s third law).

5. NEW TON’S LAW S OF M OTION :
5 .1 First La w of M otion
Each body continues to be in its state of rest or of uniform motion in a straight line unless compelled
by some external force to act otherwise.

KVPY_ SA STREAM # 75
Newton’s first law is really a statement about reference frames in that it defines the types of reference
frames in which the laws of Newtonian mechanics hold. From this point of view the first law is
expressed as:
If the net force acting on a body is zero, it is possible to find a set of reference frames in which that
body has no acceleration.
Newton’s first law is sometimes called the law of inertia and the reference frames that it defines are
called inertial reference frames.
Newton’s law from an 1803 translation from Latin as Newton wrote
“ Every body preserves in its state of rest, or of uniform motion in a right line, unless it is compelled to
change that state by forces impressed thereon.”
Examples of this law :
(a) A bullet fired on a glass window makes a clean hole through it while a stone breaks the
whole of it. The speed of bullet is very high. Due to its large inertia of motion, it cuts a clean
hole through the glass. When a stone is thrown, it inertia is much lower so it cannot cut
through the glass.
(b) A passenger sitting in a bus gets a jerk when the bus starts or stops suddenly.

5 .2 Second Law of M otion :


The rate of change of momentum of a body is proportional to the applied force and takes place in the
direction in which the force acts.
Newton’s law from an 1803 translation from Latin as Newton wrote
“ The alteration of motion is ever proportional to the motive force impressed; and is made in the direction
of the right line in which that force is impressed.”
Mathematically
 dp 
F 
or F  ma
dt
  
where p  m v , p = Linear momentum.
 
In case of two particles having linear momentum P1 and P2 and moving towards each other under
mutual forces, from Newton’s second law ;
 
d dp1 dp 2
dt
p 1  p 2  = F = 0 
dt
+
dt
=0

   
F1  F2 = 0  F2  F1
which is Newton’s third law.

5 .3 Im port a nt points a bout second la w


(a) The Second Law is obviously consistent with the First Law as F = 0 Implies a = 0.
(b) The Second Law of motion is a vector law. It is actually a combination of three equations,
one for each component of the vectors :
dp x dp y dp z
Fx   ma x Fy   ma y Fz   ma z
dt dt dt
This means that if a force is not parallel to the velocity of the body, but makes some angle
with it, it changes only the component of velocity along the direction of force. The component
of velocity normal to the force remains unchanged.

KVPY_ SA STREAM # 76
(b) The Second Law of motion given above is strictly applicable to a single point mass. The
force F in the law stand for the net external force on the particle and a stands for the
acceleration of the particle. Any internal forces in the system are not to be included in F.
(c) The Second Law of motion is a local relation. What this means is that the force F at a point
in space (location of the particle) at a certain instant of time is related to a at the same point
at the same instant. That is acceleration here and now is determined by the force here and
now not by any history of the motion of the particle.

5 .4 Applica t ions of Ne wt on’s La ws

(a ) W hen object s a re in equilibrium

To solve problems involving objects in equilibrium:


Step 1: Make a sketch of the problem.
Step 2: Isolate a single object and then draw the free-body diagram for the object. Label all
external forces acting on it.
Step 3: Choose a convenient coordinate system and resolve all forces into rectangular components
along x and y direction.

Step 4: Apply the equations  Fx  0 and  Fy  0 .

Step 5: Step 4 will give you two equations with several unknown quantities. If you have only two
unknown quantities at this point, you can solve the two equations for those unknown
quantities.
Step 6: If step 5 produces two equations with more than two unknowns, go back to step 2 and
select another object and repeat these steps.
Eventually at step 5 you will have enough equations to solve for all unknown quantities.

Example 4. A ‘block’ of mass 10 kg is suspended with string as shown in figure.


Find tension in the string. (g = 10 m/s2)

Solution : F.B.D. of block


Fy = 0
T – 10 g = 0
 T = 100 N
Example 5. The system shown in figure is in equilibrium. Find the magnitude of
tension in each string ; T1,T2,T3 and T4. (g = 10 m/s–2)

Solution : F.B.D. of block 10 kg

T 0 = 10 g
T0 = 100 N

KVPY_ SA STREAM # 77
F.B.D. of point ‘A’
Fy  O
T2 cos 30º = T0 = 100 N

200
T2 = N
3

Fx  O
T1 = T2 sin 30º

200 1 100
= . = N.
3 2 3
F.B.D. of point of ‘B’
Fy  O  T4 cos600 = T2cos300

and Fx  O  T3 + T2 sin300 = T4sin600

200
 T3 = N, T4 = 200 N
3

50N
Example 6. Two blocks are kept in contact as shown in figure. Find 0
30
(a) forces exerted by surfaces (floor and wall) on blocks.

///////////////
(b) contact force between two blocks. 100 N 10kg 20kg

Solution : F.B.D. of 10 kg block

N1 = 10 g = 100 N .............(1)

N2 = 100 N .............(2) N4
50 N
0
F.B.D. of 20 kg block 30

N2 = 50 sin 30º + N3
N2 N3
 N3 = 100 – 25 = 75 N ............(3)
and N4 = 50 cos 300 + 20 g
N4 = 243.30 N 20 g

Example 7. Find magnitude of force exerted by string on pulley.

Solution : F.B.D. of 10 kg block :

T = 10 g = 100 N
F.B.D. of pulley :

KVPY_ SA STREAM # 78
Since string is massless, so tension in both sides of string is same.
Force exerted by string

= (100 )2  (100 )2 = 100 2 N

Note : Since pulley is in equilibrium position, so net forces on it is zero.


Hence force exerted by hinge on it is 100 2 N.

(b) Acce le ra t ing Obje ct s


To solve problems involving objects that are in accelerated motion :

Step 1: Make a sketch of the problem.


Step 2: Isolate a single object and then draw the free-body diagram for that object. Label all
external forces acting on it. Be sure to include all the forces acting on the chosen body, but
be equally carefully not include any force exerted by the body on some other body. Some of
the forces may be unknown; label them with algebraic symbols.
Step 3: Choose a convenient coordinate system, show location of coordinate axes explicitly in the
free-body diagram, and then determine components of forces with reference to these axes
and resolve all forces into x and y components.

Step 4: Apply the equations  Fx  m a x and  Fy  m a y .

Step 5: Step 4 will give two equations with several unknown quantities. If you have only two unknown
quantities at this point, you can solve the two equations for those unknown quantities.
Step 6: If step 5 produces two equations with more than two unknowns, go back to step 2 and
select another object and repeat these steps. Eventually at step 5 you will have enough
equations to solve for all unknown quantities.

Example 8. A force F is applied horizontally on mass m1 as shown in figure.


Find the contact force between m1 and m2.
Solution : Considering both blocks as a system to find
the common acceleration.
Common acceleration
F
a = (m  m ) .....(1)
1 2

To find the contact force between ‘A’ and ‘B’ we draw F.B.D. of mass m2.
F.B.D. of mass m2

Fx = max

N = m2 .a
m 2F
N = (m  m )
1 2

KVPY_ SA STREAM # 79

Example 9. The velocity of a particle of mass 2 kg is given by v  at î  bt 2 ĵ . Find the force acting on the particle.
Solution : From second law of motion :

 dP d  d 
F = (m v ) = 2. ( at î  bt 2 ĵ )  F  2a î  4bt ĵ
dt dt dt

Example 10. A 5 kg block has a rope of mass 2 kg attached to its underside and a 3 kg
block is suspended from the other end of the rope. The whole system is
accelerated upward at 2 m/s2 by an external force F0.
(a) What is F0 ?
(b) What is the net force on rope ?
(c) What is the tension at middle point of the rope ? (g = 10 m/s2)
Solution : For calculating the value of F0, consider two blocks with the rope as a system.
F.B.D. of whole system

(a)

F0 – 100 = 10 × 2
F = 120 N .....(1)
(b) According to Newton’s second law, net force on rope.
F = ma = (2) (2)
=4N .....(2)
(c) For calculating tension at the middle point we draw F.B.D. of 3 kg block with half of the rope
(mass 1 kg) as shown.

T – 4g = 4.(2) ; T = 48 N

Example 11. A block of mass 50 kg is kept on another block of mass 1 kg as shown in


figure. A horizontal force of 10 N is applied on the 1kg block. (All surface
are smooth). Find (g = 10 m/s2)
(a) Acceleration of block A and B.
(b) Force exerted by B on A.
Solution : (a) F.B.D. of 50 kg

N2 = 50 g = 500 N
along horizontal direction, there is no force aB = 0
(b) F.B.D. of 1 kg block :
N1 N2

10N
//////////////////////////////////////////

1g
along horizontal direction
10 = 1 aA.
aA = 10 m/s2
along vertical direction
 N1 = N2 + 1g
= 500 + 10 = 510 N

KVPY_ SA STREAM # 80
Example 12. A horizontal force is applied on a uniform rod of length L kept on a frictionless surface. Find the
tension in rod at a distance ‘x’ from the end where force is applied.

Solution : Considering rod as a system, we find acceleration of rod


F
a=
M
now draw F.B.D. of rod having length ‘x’ as shown in figure.

Using Newton’s second law


M M F x
F – T =   x.a  T= F x.  T = F(1  ) .
L L M L

Example 13. One end of string which passes through pulley and connected to 10 kg
mass at other end is pulled by 100 N force. Find out the acceleration of 10
kg mass. (g = 9.8 m/s2)
Solution : Since string is pulled by 100 N force. So tension in the string is 100 N.
F.B.D. of 10 kg block

100 – 10 g = 10 a
100 – 10 × 9.8 = 10 a
a = 0.2 m/s2.

Example 14. Two blocks m1 and m2 are placed on a smooth inclined plane as shown in figure.
If they are released from rest. Find :
(i) acceleration of mass m1 and m2
(ii) tension in the string
(iii) net force on pulley exerted by string

Solution : F.B.D. of m1 :
m1gsin – T = m1a
3
g–T = 3 a ...........(i)
2

F.B.D. of m2 :
T – m2gsin = m2a

3
T – 1. g = 1.a ..........(ii)
2
Adding eq.(i) and (ii) we get a = 0
Putting this value in eq.(i) we get

3g
T= ,
2
F.B.D. of pulley
FR = 2T

3
FR = g
2

KVPY_ SA STREAM # 81
Example 15. A 60 kg painter stands on a 15 kg platform. A rope attached to the
platform and passing over an overhead pulley allows the painter to
raise himself along with the platform.
(i) To get started, he pulls the rope down with a force of 400
N. Find the acceleration of the platform as well as that of
the painter.
(ii) What force must he exert on the rope so as to attain an
upward speed of 1 m/s in 1s?
(iii) What force should he apply now to maintain the constant
speed of 1 m/s?
Solution : The free body diagram of the painter and the platform as a system can be
drawn as shown in the
figure. Note that the tension in the string is equal to the force by which he
pulls the rope.
(i) Applying Newton’s Second Law

2T  (M  m)g
2T – (M + m)g = (M + m)a or a=
Mm

Here M = 60 kg; m = 15 kg; T = 400 N


g = 10 m/s2

2( 400 )  ( 60  15)(10)
a= = 0.67 m/s2
60  15

(ii) To attain a speed of 1 m/s in one second, the acceleration a must be 1 m/s2.
Thus, the applied force is

1 1
F= (M + m) (g + a) = (60 + 15) (10 + 1) = 412.5 N
2 2

(iii) When the painter and the platform move (upward) together with a constant speed, it is in a
state of dynamic equilibrium.
Thus, 2F – (M + m) g = 0

(M  m)g (60  15)(10)


or F= = = 375 N
2 2


6. W EIGHING M ACHINE :
A weighing machine does not measure the weight but measures the force exerted by object on its upper
surface.

Example 16. A man of mass 60 Kg is standing on a weighing machine placed on ground. Calculate the reading of
machine (g = 10 m/s2).

KVPY_ SA STREAM # 82
Solution : For calculating the reading of weighing machine, we draw F.B.D. of man and machine separately.

F.B.D. of man F.B.D. of weighing machine

N = Mg

Here force exerted by object on upper surface is N

Reading of weighing machine

N = Mg = 60 × 10

N = 600 N.


7. SPRING FORCE :
Every spring resists any attempt to change its length; when it is compressed or extended, it exerts force at
its ends. The force exerted by a spring is given by F = -kx, where x is the change in length and k is the
stiffness constant or spring constant (unit Nm-1).

When spring is in its natural length, spring force is zero.

F=0

F = – kx

Example 17. Two blocks are connected by a spring of natural length 2 m.


The force constant of spring is 200 N/m.
Find spring force in following situations :
(a) If block ‘A’ and ‘B’ both are displaced by 0.5 m in same direction.
(b) If block ‘A’ and ‘B’ both are displaced by 0.5 m in opposite direction.
Solution : (a) Since both blocks are displaced by 0.5 m in same direction, so change in length of spring is
zero. Hence, spring force is zero.
(b) In this case, change in length of spring is 1 m. In case of extension or compression of spring,
spring force is F = Kx = (200) . (1)
F = 200 N

KVPY_ SA STREAM # 83
Example 18. Force constant of a spring is 100 N/m. If a 10 kg block attached with
the spring is at rest, then find extension in the spring. (g = 10 m/s2)
Solution : In this situation, spring is in extended state so spring force acts in upward direction.
Let x be the extension in the spring.
F.B.D. of 10 kg block :
Fs = 10g
 Kx = 100
 (100)x = (100)
 x = 1m

Example 19. Two blocks ‘A’ and ‘B’ of same mass ‘m’ attached with a light
spring are suspended by a string as shown in figure. Find the
acceleration of block ‘A’ and ‘B’ just after the string is cut.
Solution : When block A and B are in equilibrium position

F.B.D of ‘B’ ......... (i)

F.B.D of ‘A’ .......(ii)

when string is cut, tension T becomes zero. But spring does not change its shape just after cutting.
So spring force acts on mass B, again draw F.B.D. of blocks A and B as shown in figure

F.B.D. of ‘B’

F.B.D. of ‘A’


7 .1 Spring Ba la nce :

It does not measure the weight. t measures the force exerted by

the object at the hook.


Symbolically, it is represented as shown in figure.

A block of mass ‘m’ is suspended at hook.

When spring balance is in equilibrium, we draw the F.B.D. of mass m for calculating the reading of
balance.

KVPY_ SA STREAM # 84
F.B.D. of ‘m’.

mg – T = 0

T = mg

Magnitude of T gives the reading of spring balance.

Example 20. A block of mass 20 kg is suspended through two light spring balances as

shown in figure. Calculate the :


(1) reading of spring balance (1).

(2) reading of spring balance (2).

Solution: For calculating the reading, first we draw F.B.D. of 20 kg block.


F.B.D of 20 kg.
mg – T = 0
T = 20 g = 200 N

Since both balances are light so, both the scales will read 20 kg.


8. CONSTRAINED M OTION:
8 .1 St ring Constra int :
When two objects are connected through a string and if the string have the following properties :
(a) The length of the string remains constant i.e. inextensible string.
(b) Always remains tight, does not slacks.
Then the parameters of the motion of the objects along the length of the string and in the direction of
extension have a definite relation between them.
St eps for St ring Constra int

Step 1. Identify all the objects and number of strings in the problem.
Step 2. Assume variable to represent the parameters of motion such as displacement, velocity
acceleration etc.
(i) Object which moves along a line can be specified by one variable.
(ii) Object moving in a plane are specified by two variables.
(iii) Objects moving in 3-D requires three variables to represent the motion.

Step 3. Identify a single string and divide it into different linear sections and write in the equation
format.  1+  2+  3+  4+  5+  6 = 

Step 4. Differentiate with respect to time

d 1 d 2 d 3
   ....  0
dt dt dt

KVPY_ SA STREAM # 85
d 1
= represents the rate of increment of the portion 1, end points are always in contact with some
dt
d 1
object so take the velocity of the object along the length of the string  V1  V2
dt

Take positive sign if it tends to increase the length and negative sign if it tends to decrease the
length. Here +V1 represents that upper end is tending to increase the length at rate V1 and lower end
is tending to increase the length at rate V2.
Step 5. Repeat all above steps for different-different strings.
Let us consider a problem given below
Here 1 + 2 = constant
VP VP
d 1 d 2 1
+ 0 2
dt dt V1
V2
(V1 – VP) + (VP – V2) = 0

V1  V2
Vp =
2

Similarly,

a1  a2
aP = Remember this result
2

Example 21. Two blocks of masses m1 and m2 are attached at the ends of an inextensible
string which passes over a smooth massless pulley. If m1 > m2, find :
(i) the acceleration of each block m2

(ii) the tension in the string. m1

Solution : The block m1 is assumed to be moving downward and the block m2 is assumed to be moving
upward. It is merely an assumption and it does not imply the real direction. If the values of a1 and a2
come out to be positive then only the assumed directions are correct; otherwise the body moves in
the opposite direction. Since the pulley is smooth and massless, therefore, the tension on each
side of the pulley is same.
The free body diagram of each block is shown in the figure.
F.B.D. of m2 F.B.D. of m1

Applying Newton’s second Law on blocks m1 and m2


Block m1 m1g – T = m1a1 .............(1)
Block m2 –m2g + T= m2a2 .............(2)

KVPY_ SA STREAM # 86
Number of unknowns : T, a1 and a2 (three)
Number of equations: only two
Obviously, we require one more equation to solve the problem. Note that whenever one finds the
number of equations less than the number of unknowns, one must think about the constraint relation.
Now we are going to explain the mathematical procedure for this.
How to determine Constraint Relation ?
(1) Assume the direction of acceleration of each block, e.g. a1 (downward) and a2 (upward) in this case.
(2) Locate the position of each block from a fixed point (depending on convenience), e.g. centre of the
pulley in this case.
(3) Identify the constraint and write down the equation of constraint in terms of the distance assumed.
For example, in the chosen problem, the length of string remains constant is the constraint or
restriction.
Thus, x1 + x2 = constant

dx 1 dx 2
Differentiating both the sides w.r.t. time we get + =0
dt dt

Each term on the left side represents the velocity of the blocks.
Since we have to find a relation between accelerations,
therefore we differentiate it once again w.r.t. time.
d2 x1 d2 x 2
Thus + =0
dt 2 dt 2

Since, the block m1 is assumed to be moving downward (x1 is increasing with time)

d2 x 1
 = + a1
dt 2

and block m2 is assumed to be moving upward (x2 is decreasing with time)

d2 x 2
 = – a2
dt 2

Thus a1 – a2 = 0
or a1 = a2 = a (say) is the required constraint relation.
Substituting a1 = a2 = a in equations (1) and (2) and solving them, we get

 m1  m 2   2m1m 2 
(i) a = m  m  g (ii) T = m  m  g
 1 2  1 2

Example 22. A system of three masses m1, m2 and m3 are shown in the figure.
The pulleys are smooth and massless; the strings are massless and inextensible.
(i) Find the tensions in the strings.
(ii) Find the acceleration of each mass.
Solution : All the blocks are assumed to be moving downward and the free
body diagram of each block is shown in figure.

KVPY_ SA STREAM # 87
F.B.D. m3 F.B.D. m2 F.B.D. m1 F.B.D. of pulley

Applying Newton’s Second Law to


Block m1 : m1g – T1 = m1a1 ....(1)
Block m2 : m2g – T1 = m2a2 ....(2)
Block m3 : m3g – T2 = m3a3 ....(3)
Pulley : T2 = 2T1 ....(4)
Number of unknowns a1, a2, a3, T1 and T2 (Five)
Number of equations : Four
The constraint relation among accelerations can be obtained
as follows
For upper string x 3 + x 0 = c1
For lower string x2 – x0) + (x1 – x0) = c2
x2 + x1 – 2x0 = c2
Eliminating x0 from the above two relations,
we get x1 + x1 + 2x3 = 2c1 + c2 = constant.
Differentiating twice with respect to time,

d2 x1 d2 x 2 d2 x 3
we get + +2 =0
dt 2 dt 2 dt 2

or a1 + a2 + 2a3 = 0 ......(5)
Solving equations (1) to (5), we get

 4m1m2m3 
(i) T1 =  4m m  m (m  m )  g T2 = 2T1
 1 2 3 1 2 

 4m1m2  m1m3  3m2m3   3m1m3  m2m3  4m1m2 


(ii) a1 =  4m m  m (m  m )  g a2 =  4m m  m (m  m )  g
 1 2 3 1 2   1 2 3 1 2 

 4m1m 2  m3 (m1  m2 ) 
a3 =  4m m  m (m  m )  g
 1 2 3 1 2 

KVPY_ SA STREAM # 88
Example 23. The figure shows one end of a string being pulled down at constant
velocity v. Find the velocity of mass ‘m’ as a function of ‘x’.

Solution : Using constraint equation

2 x 2  b 2 + y = length of string = constant


Differentiating w.r.t. time :
2  dx   dy 
. 2x      = 0
2
2 x b 2  dt   dt 

 dy   dx  v
   =v    =  x2  b2
 dt   dt  2x


8 .2 W edge Constra int :
Conditions :
(i) There is a regular contact between two objects.
(ii) Objects are rigid.
The relative velocity perpendicular to the contact plane of the two rigid objects is always zero if there
is a regular contact between the objects. Wedge constraint is applied for each contact.

In other words,
Components of velocity along perpendicular direction to the contact plane of the two objects is
always equal if there is no deformations and they remain in contact.

Example 24. A rod of mass 2m moves vertically downward on the surface of wedge of mass as shown in figure.
Find the relation between velocity of rod and that of the wedge at any instant.

Solution : Using wedge constraint.


Component of velocity of rod along perpendicular to
inclined surface is equal to velocity of wedge along
that direction.
u cos  = v sin 

u
= tan 
v
u = v tan 

KVPY_ SA STREAM # 89

10. NEW TON’S LAW FOR NON INERTIAL FRAM E :
  
FRe al  FPseudo  ma
Net sum of real and pseudo force is taken in the resultant force.

a = Acceleration of the particle in the non inertial frame

FPseudo =  m a
Frame
Pseudo force is always directed opposite to the direction of the acceleration of the frame.
Pseudo force is an imaginary force and there is no action-reaction for it. So it has nothing to do with Newton’s
Third Law.

1 0 .1 Re fere nce Fra m e:


A frame of reference is basically a coordinate system in which motion of object is analyzed. There are two
types of reference frames.
(a) Inertial reference frame: Frame of reference either stationary or moving with constant velocity.
(b) Non-inertial reference frame: A frame of reference moving with non-zero acceleration.

Example 25. A lift having a simple pendulum attached with its ceiling is moving upward with constant acceleration
‘a’. What will be the tension in the string of pendulum with respect to a boy inside the lift and a boy
standing on earth, mass of bob of simple pendulum is m.
Solution : F.B.D . of bob (with respect to ground)
T – mg = ma
T = mg + ma ........(i)
With respect to boy inside the lift, the acceleration of bob is zero.
So he will write above equation in this manner.
T – mg = m. (0) .  T = mg
He will tell the value of tension in string is mg. But this is ‘wrong’ . To correct his result, he makes a
free body diagram in this manner, and uses Newton’s second law.

T = mg + ma ..................(ii)
By using this extra force, equations (i) and (ii) give the same result . This extra force is called
pseudo force. This pseudo force is used when a problem is solved with a accelerating frame (Non-
inertial)
Note : Magnitude of Pseudo force = mass of system × acceleration of frame of reference .
Direction of force:
Opposite to the direction of acceleration of frame of reference, (not in the direction of motion of frame of
reference)

KVPY_ SA STREAM # 90
Example 26. A box is moving upward with retardation ‘a’ <g, find the direction and magnitude of “ pseudo force”
acting on block of mass ‘m’ placed inside the box. Also calculate normal force exerted by surface on
block

Solution : Pseudo force acts opposite to the direction of acceleration of reference frame.
pseudo force = ma in upward direction
F.B.D of ‘m’ w.r.t. box (non–inertial)

Example 27. All surfaces are smooth in the adjoining figure. Find F such that block remains stationary with
respect to wedge.

F
Acceleration of (block + wedge) is a =
Solution :
M  m
Let us solve the problem by using both frames.
From inertial frame of reference (Ground)
F.B.D. of block w.r.t. ground (Apply real forces) :
with respect to ground block is moving with an acceleration ‘a’ .

  Fy = 0 N cos  = mg ...........(i)

and   Fx = ma N sin  = ma .....(ii)


From Eqs. (i) and (ii)
a = g tan  F = (M + m) a = (M + m) g tan 
From non–inertial frame of reference (Wedge) :
F.B.D. of block w.r.t . wedge (real forces + pseudo force)
w.r.t. wedge, block is stationary

  Fy = 0 N cos  = mg ..............(iii)

  Fx = 0 N sin  = ma ..................(iv)

From Eqs. (iii) and (iv) , we will get the same result
i.e. F = (M + m) g tan .

KVPY_ SA STREAM # 91
1. Mass m shown in figure is in equilibrium. If it is displaced further by x and released find its acceleration just
after it is released. Take pulleys to be light & smooth and strings light.

4kx 2kx 4kx


(A) (B) (C) (D) none of these
5m 5m m

2. Same spring is attached with 2 kg, 3 kg and 1 kg blocks in three different cases as shown in figure. If x1, x2
and x3 be the extensions in the spring in these cases then (Assume all the blocks to move with uniform
acceleration)

(A) x1 = 0, x3 > x2 (B) x2 > x1 > x3 (C) x3 > x1 > x2 (D) x1 > x2 > x3
3. Figure shows a 5 kg ladder hanging from a string that is connected with a ceiling and is having a spring
balance connected in between. A boy of mass 25 kg is climbing up the ladder at acceleration 1 m/s2.
Assuming the spring balance and the string to be massless and the spring to show a constant reading, the
reading of the spring balance is : (Take g = 10 m/s2)

(A) 30 kg (B) 32.5 kg (C) 35 kg (D) 37.5 kg


4. A light spring is compressed and placed horizontally between a vertical
fixed wall and a block free to slide over a smooth horizontal table top as
shown in the figure. The system is released from rest. The graph which
represents the relation between the magnitude of acceleration ‘ a ‘ of the
block and the distance ‘ x ‘ travelled by it (as long as the spring is
compressed) is

(A) (B) (C) (D)

KVPY_ SA STREAM # 92
5. A block of mass m resting in contact with a support S connected to a spring on a smooth horizontal floor as
shown. If the block is pushed toward right by a distance x and released, which of the following graph may
represent the speed of block as a function of distance from the wall w :

x
(A) (B) (C) (D)

6. In the figure shown acceleration of monkey relative to the rope if it exerts a force of 80 N on string will
be:

(A) 2 m/s2 downwards (B) 4 m/s2 upwards


(C) 4 m/s2 downwards (D) 8 m/s2 downwards

7. In the given figure inclined surface and pulleys are smooth. Strings and pulleys are massless. Acceleration
of mass m is :

m
2m
30°

7g 7g 7g 7g
(A) (B) (C) (D)
9 11 18 20

8. A balloon is tied to a block. The mass of the block is 2kg. The tension of the string between the balloon
and the block is 30N. Due to the wind, the string has an angle  relative to the vertical direction.
cos = 4/5 and sin = 3/5. Assume the acceleration of gravity is g = 10 m/s2. Also assume the block is
small so the force on the block from the wind can be ignored. Then the x-component and the y-
component of the acceleration a of the block.

wind
y
Balloon

T=30N
2kg x

(A) 9 m/s2, 2 m/s2 (B) 9 m/s2, 12 m/s2 (C) 18 m/s2, 2 m/s2 (D) 18 m/s2, 12 m/s2

KVPY_ SA STREAM # 93
9.* A block of mass m is pulled along a horizontal friction less surface by a rod of mass m as shown in figure. A
horizontal force F is applied to one end of the rod.

Find the force exerted by the block on the rod.


2
F
   mg
2
(A) F (B) F 2  mg2 (C) (D) F/2
2

10. As shown to the right, two blocks with masses m and M (M > m) are pushed by a force F in both Case I and
Case II. The surface on which blocks lie, is horizontal and frictionless. Let RI be the force that m exerts on
M in case I and RII be the force that m exerts on M in case II. Which of the following statement is true?

(A) RI = RII and is not equal to zero or F.


(B) RI = RII = F
(C) RI < RII
(D) RI > RII

11. A lift is falling with an acceleration 2 m/s2. A ball of mass 100 gm is attached at one end of the string and the
other end is fixed to the ceiling of the lift. The ball remains at rest relative to lift. The tension in the string is:
(g = 10 m/s2)
(A) 1.2 N (B) 0.8 N (C) 10 N (D) 0.2 N

12. Block A of mass m is placed on a plank B. A light support S is fixed on


plank B and is attached with the block A with a spring of spring constant K.
Consider that initially spring is in its natural length. Find the maximum
compression in the spring, if the plank B is given an acceleration a. All the
surfaces are smooth :
ma 2ma
(A) (B)
2k k
ma 4ma
(C) (D)
k k

13. In figure, two identical particles each of mass m are tied together with an
inextensible light string. This is pulled at its centre with a constant force F. If
the whole system lies on a smooth horizontal plane, then the acceleration of
approach of particles towards each other at the instant shown in figure is :

3F F
(A) (B)
2m 2 3m

2F F
(C) (D)
3m 3m

14. A block of mass m lies on wedge of mass M, which lies on fixed


horizontal surface. The wedge is free to move on the horizontal m F
surface. A horizontal force of magnitude F is applied on block as
shown, neglecting friction at all surfaces, the value of force F such M
that block has no relative motion w.r.t. wedge will be : (where g is
acceleration due to gravity)
(A) (M + m) g tan (B) (M + m) g cot Fixed horizontal surface
m m
(C) (M + m) g tan (D) (M + m) g cot
M M

KVPY_ SA STREAM # 94
15. Acceleration (w.r.t.ground) of pulleys and blocks are as shown in the figure. All pulleys & strings are mass-
less & frictionless then the magnitude of aA and aB are :

(A) aA = 0, aB = 7 (B) aA = 0, aB = 5 (C) aA = 0, aB = 12 (D) aA = 5, aB = 7

16. A wedge is moving with an acceleration a = 4m/s2 vertically up as shown


in figure. What is acceleration of block of mass 1 kg w.r.t. wedge &
normal reaction by wedge on block respectively. All surfaces are smooth.
Choose correct pair : (g = 10 m/s2)
(A) 7 m/s2 , 7 3 N (B) 5 m/s2, 5 3 N

(C) 4.9 m/s2 , 4.9 3 N (D) 10 m/s2 , 10 3 N

17. Two smooth rings A and B are connected by a string. A another string connects Ring A and block, if strings
are tight then at the given moment :

(A) v B = 15 m/s (B) v B = 24 m/s (C) v B = 20 m/s (D) v B = 12 m/s

18. Two blocks of mass 2 kg and 3 kg are placed in a cart. A variable


force F is acting on the Cart. At an instant acceleration of different
blocks and cart are as shown. Find value of F at this instant (mass
of cart is 10 kg) All accelerations are horizontal.
(A) 27 N (B) 81 N
(C) 30 N (D) 90 N

19. A 2 kg toy car can move along x axis. Graph shows resultant force F x, acting on the car which begins
at rest at time t = 0. The velocity of the particle at t = 10 s is :
Fx(N)
4

t(s)
0
4 8 9 10 11

-2

(A) – i m/s (B) – 1.5 i m/s (C) 6.5 i m/s (D) 13 i m/s

20. A bird of mass 400 gm is flying horizontally at a certain height with constant acceleration of 5 m/s2. The force
exerted by surrounding air on the bird has magnitude (g = 10 m/s2)
(A) 4 N (B) 2 N (C) 4 2 N (D) 2 5 N

KVPY_ SA STREAM # 95
21. A mass of 1 kg is suspended by a string A. Another string C is connected to its lower end (see figure). If a
sudden jerk is given to C, then
A

1kg B

C
(A) The portion AB of the string will break (B) The portion BC of the string will break
(C) None of the strings will break (D) The mass will start rotating
22. A diwali rocket is ejecting 0.05 kg of gases per second at a velocity of 400 m/sec. The accelerating force
on the rocket is
(A) 20 dynes (B) 20 N (C) 22 dynes (D) 1000 N

23. A body of mass 2 kg moving on a horizontal surface with an initial velocity of 4 m/sec comes to rest after
2 sec. If one wants to keep this body moving on the same surface with a velocity of 4 m/sec, the force
required is
(A) 8 N (B) 4 N (C) Zero (D) 2 N
24. If the tension in the cable of 1000 kg elevator is 1000 kg weight, the elevator
(A) Is accelerating upwards (B) Is accelerating downwards
(C) May be at rest or accelerating (D) May be at rest or in uniform motion
25. A cork is submerged in water by a spring attached to the bottom of a pail. When the pail is kept in a
elevator moving with an acceleration downwards, the spring length
(A) Increases (B) Decreases (C) Remains unchanged (D) Data insufficient

KVPY PROBLEMS (PREVIOUS YEARS)


1. A fully loaded elevator has a mass of 6000 kg. The tension in the cable as the elevator is accelerated
downward with an acceleration of 2ms–2 is (Take g = I0 ms–2) [KVPY_2007]
(A) 7·2 × 104 N (B) 4.8 × 104 N (C) 6 × 104 N (D) 1.2 × 104 N

2. A light string goes over a frictionless polley. At its one end hangs a mass of 2 kg and at the other end hangs
a mass of 6 kg. Both the masses are supported by hands to keep them at rest. When the masses are
released, they being to move and the string gets taut. [KVPY_2008]
(Take g = 10 ms–2) The tension in the string during the motion of the masses is :
(A) 60 N (B) 30 N (C) 20 N (D) 40 N
3. A block of mass M is at rest on a plane surface inclined at an angle  to the horizontal The magnitude of force
exerted by the plane on the block is : [KVPY_2009]
(A) Mg cos (B) Mg sin  (C) Mg tan (D) Mg

4. A spring balance A reads 2 kg when a block of mass m suspended


from it. Another balance B reads 3 kg when a beaker with a liquid A
is put on its pan. The two balanes are now so arranged that the
hanging mass m is fully immersed inside the liquid in the beaker
as shown in the figure. In this situation. [KVPY_2009]

(A) the balance A will read 2 kg and B will read 5 kg.


(B) the balance A will read 2 kg and B will read 3 kg.
(C) the balance A will read less than 2 kg and B will read between 3 kg and 5 kg.
B
(D) the balance A will read less than 2 kg and B will read 3 kg.

KVPY_ SA STREAM # 96
5. A small child tries to move a large rubber toy placed on the ground. The toy does not move but gets deformed

under her pushing force (F) which is obliquely upward as shown . Then [KVPY_2011]


(A) The resultant of the pushing force (F) , weight of the toy, normal force by the ground on the toy and the
frictional force is zero.
(B) The normal force by the ground is equal and opposite to the weight of the toy.

(C) The pushing force (F) of the child is balanced by the equal and opposite frictional force

(D) The pushing force (F) of the child is balanced by the total internal force in the toy generated due to
deformation
6. A box, when hung from a spring balance shows a reading of 50 kg. If the same box is hung from the same
spring balance inside an evacuated chamber, the reading on the scale will be [KVPY_2014_SA]
(A) 50 kg because the mass of the box remains unchanged
(B) 50 kg because the effect of the absence of the atmosphere will be indentical on the box and the spring
balance
(C) Less than 50 kg because the weight of the column of air on the box will be absent
(D) More than 50 kg because the atmospheric buoyancy force will be absent
7. In an experiment, setup A consists of two parallel wires which carry currents in opposite directions as shown
in the figure. A second setup B is identical to setup A, except that there is a metal plate between the wires.
[KVPY_2016_SA] [ 1 Marks]

Setup A Setup B

Let FA and FB be the magnitude of the force between the two wires in setup A and setup B, respectively.
(A) FA > FB 0 (B) FA > FB (C) FA = FB 0 (D) FA > FB = 0
8. Consider two masses with m1 > m2 connected by a light inextensible string that passes over a pulley of
radius R and moment of inertia  about its axis of rotation. The string does not slip on the pulley and the pulley
turns without friction. The two masses are released from rest separated by a vertical distance 2h. When the
two masses pass each other, the speed of the masses is proportional to. [KVPY_2016_SB] [ 2 Marks]

m1  m2 (m1  m2 )(m1  m2 )  
m1  m2 
(A)  (B)  (C) R2 (D) R2
m1  m2  m1  m2  2
R2 R m1  m2 m1  m2

9. In Guericke’s experiment to show the effect of atmospheric pressure, two copper hemispheres were tightly
fitted to each other to form a hollow sphere and the air from the sphere was pumped out to create vacuum
inside. If the radius of each hemisphere is R and the atmospheric pressure is P, then the minimum force
required (when the two hemispheres are pulled apart by the same force) to separate the hemispheres is
[KVPY_2017_SA] [ 1 Marks]
(A) 2R2P (B) 4R2P (C) R2P (D) R2P/2

KVPY_ SA STREAM # 97
10. A box filled with water has a small hole on its side near the bottom. It is dropped from the top of a tower. As
it falls, a camera attached on the side of the box records the shape of the water stream coming out of the
hole. The resulting video will show [KVPY-SA_2018 1 Mark]
(A) the water coming down forming a parabolic stream.
(B) the water going up forming a parabolic stream.
(C) the water coming out in a straight line.
(D) no water coming out.

EXERCISE - 1

1. (C) 2. (B) 3. (B) 4. (C) 5. (A) 6. (B) 7. (C)

8. (A) 9.* (C) 10. (C) 11. (B) 12. (B) 13. (D) 14. (C)

15. (C) 16. (A) 17. (C) 18. (A) 19. (C) 20. (D) 21. (B)

22. (B) 23. (B) 24. (D) 25. (B)

EXERCISE - 2
1. (B) 2. (B) 3. (D) 4. (C) 5. (A) 6. (D) 7. (C)

8. (C) 9. (C) 10. (D)

KVPY_ SA STREAM # 98
1. Initially the block is at rest under action of force 2T upward and mg
downwards. When the block is pulled downwards by x, the spring
extends by 2x. Hence tension T increases by 2kx. Thus the net
unbalanced force on block of mass m is 4kx.
4kx
 acceleration of the block is =
m

2. If m1 and m2 are masses of blocks then tension T in the string as well as spring are
2 m1 m 2
T= g
m1  m 2
T1 = 2g T2 = 2.4 g T3 = 1.33 g
 T2 > T1 > T3 or x2 > x1 > x3

3. If reading of spring balance is T, then applying NLM on (man + ladder) system


T – (25 + 5)g = 25 a
T – 30g = 25 a  T – 300 = 25(1)  T = 325 N = 32.5 kg.

4. Let the initial compression of spring be . Then the acceleration after the block travels a distance x is
k
a= ( – x)
m
 The graph of a vs x is

5. The slope of v – x graph is


dv dv dt a
slope  . =
dx dt dx v
At the beginning ‘a’ is maximum (as kx is maximum).
hence the slope is nonzero positive.
At the instant spring attain its natural length, ‘a’ becomes zero (as x becomes zero) , after which no force
acts on the block along horizontal hence acceleration continues to be zero.
Now (A) and (B) options satisfy our need.
Applying work – energy theorem from initial compressed position to a lesser compressed position:
1 1 1 1
k[x02 – (x)2] = k[x02] – k ( – s)2 = mv 2 – 0, where s is the distance from the wall.
2 2 2 2
 V – s graph is not a straight line.
v

s
Hence the option : is correct.

7. 2mg – T = 2m × 2a  (i)
2T – mg sin 30° = m × a  (ii)
m
from equation (i) and (ii)s
2m
7g 30°
a= .
18

KVPY_ SA STREAM # 99
8. The magnitude of the force (from the string) is T = 30N.
The x-component = T sin = 30 × 3/5 = 18N.
The y-component = T cos = 30 × 4/5 = 24N.
The total force on the block is:
the x-component = 18N.
the y-component = 24 – mg = 24 – 20 = 4N.
The x-component of the acceleration = 18N/2kg = 9m/s2.
The y-component of the acceleration = 4N/2kg = 2m/s2

9.* FBD of rod :


R

F
 F 
F– R cos = m  2m  ..........(i)
 
R sin = mg ..........(ii)
by eq.(i) and (ii)
R= (F / 2)2  (mg )2

F
10. Acceleration of all blocks in both cases is same and is a =
mM
In case I : required normal reaction RI is equal to net force on block of mass m
 RI = ma
In case II : required normal reaction RII is equal to net force on block of mass M.
 RII = Ma
Hence RII > RI

11. T = m (g – a)
= 0.1 (10 – 2) = 0.8 N

dv
12. mv = ma – kx
dx
0 x


0

mvdv  (ma  kx)dx
0

2ma
x= .
k
13. F = 2T cos 30°
T sin 30
a=
m
F
a=
2 3m
F
aapproach = 2a = .
3m

14. For no relative motion between wedge and block, let the
acceleration of both block and wedge be 'a' towards left.
N
From FBD of block
F
N cos = mg .... (1) a
and F– N sin = ma .... (2)
a Nsin
From FBD of wedge mg
N sin = Ma .... (3)
m
from equation (1), (2) and (3) solving we get F = (M + m) g tan
M
KVPY_ SA STREAM # 100
2  aA
15. 1=  aA = 0 take upward direction positive
2
a of pulley at right hand side
1 a
3=  a = 5 m/s2
2
2  a B
5=  aB = 12 m/s2
2

16. In the frame of wedge acceleration = (g + a) sin  = 7 m/s2


Normal reaction m (g + a) cos  = 7 3 N

17.

v A cos37º = 12
v A = 15 m/s
and v A cos37º = v B cos53º
 v B = 20 m/s.

   
18. Fnet = m1a1  m 2 a 2  m 3 a3
 F = 10 × 2 + 3 × 3 – 2 × 1 = 27 N.

19.  dp = pf – pi =  F dt = Area under the curve.


pi = 0
Net Area = 16 – 2 – 1 = 13 N-s
13
= 6.5 i m/s
= Vf =
2
[As momentum is positive, particle is moving along positive x axis.]

20.

Fx = ma = 0.4 × 5 = 2N
Fy = mg = 0.4 × 10 = 4 N
2 2
F= f x  Fy = 4  16 = 20 = 2 5 N.

KVPY_ SA STREAM # 101


FRI CT I ON

1. FRI CT I ON
When two bodies are kept in contact, electromagnetic forces act between the charged particles (molecules)
at the surfaces of the bodies. Thus, each body exerts a contact force on the other. The magnitudes of the
contact forces acting on the two bodies are equal but their directions are opposite and therefore the contact
forces obey Newton’s third law.

The direction of the contact force acting on a particular body is not necessarily perpendicular to the contact
surface. We can resolve this contact force into two components, one perpendicular to the contact surface
and the other parallel to it (figure. The perpendicular component is called the normal contact force or normal
force ( generally written as N) and the parallel component is called friction (generally written as f).
Therefore if R is contact force then
R= f 2  N2

2. REASON S FOR FRI CT I ON


(i) nter-locking of extended parts of one object into the extended parts of the other object.
(ii) Bonding between the molecules of the two surfaces or objects in contact.

3. FRI CT I ON FORCE I S OF T WO T Y PES.


a. Kinetic b. Static
(a) Kinetic Friction Force
Kinetic friction exists between two contact surfaces only when there is relative motion between
the two contact surfaces. It stops acting when relative motion between two surfaces ceases.
DIRECTION OF KINECTIC FRICTION ON AN OBJECT
t is opposite to the relative velocity of the object with respect to the other object in contact considered.
Note that its direction is not opposite to the force applied it is opposite to the relative motion of the
body considered which is in contact with the other surface.

MAGNITUDE OF KINETIC FRICTION


The magnitude of the kinetic friction is proportional to the normal force acting between the two bodies. We
can write
f k = k N
where N is the normal force. The proportionality constant k is called the coefficient of kinetic friction
and its value depends on the nature of the two surfaces in contact.

KVPY_ SA STREAM # 102


Example 1. Find the direction of kinetic friction force
(a) on the block, exerted by the ground.
(b) on the ground, exerted by the block.

Solution : (a)

(b)

where f1 and f2 are the friction forces on the block and ground respectively.

Example 2. In above example correct relation between magnitude of f1 and f2 is


(A) f 1 > f2 (B) f 2 > f1
(C) f 1 = f 2 (D) not possible to decide due to insufficient data.
Solution : By Newton‘s third law the above friction forces are action-reaction pair and equal but opposite to
each other in direction. Hence (C).
Also note that the direction of kinetic friction has nothing to do with applied force F.

Example 3. All surfaces as shown in the figure are rough. Draw the friction force on A & B
A 10m/s

B 20m/s
////////////////////////////////////////////////////////

Solution :

Kinetic friction acts in such a way so as to reduce relative motion.

Example 4. Find out the distance travelled by the blocks shown in the figure before it stops.
10 m/s
10 kg
/////////////////////////////////////////
µ k=0.5

Solution :
N
N – 10 g = 0
N = 100 N
f x = µkN fk
µ = µs = µk when not mentioned
10g
f x = 0.5 × 100 = 50 N
Fx = ma
50 = 10 a  a = 5  v 2 = u2 + 2as
02 = 102 + 2 (–5) (S)  S = 10 m

Example 5. Find out the distance travelled by the block on incline before it stops. Initial velocity of the block is
10 m/s and coefficient of friction between the block and incline is  = 0.5.

s
m/
10

37° fixed
µ

KVPY_ SA STREAM # 103


Solution : N = mg cos37°
 mg sin 37° + µN = ma
a = 10 m/s2 down the incline
Now v 2 = u2 + 2as
0 = 102 + 2(–10) S
S=5m

Example 6. Find the time taken in the above example by the block to reach the initial position.
Solution : a = g sin 37° – µg cos 37°
 a = 2 m/s2 down the incline

1 2 1
 S = ut + at  S= × 2 × t2 m
2 2 5

t= 5 sec.

Example 7. A block is given a velocity of 10 m/s and a force of 100 N in addition to friction force is also acting on
the block. Find the retardation of the block?

Solution : As there is relative motion


 kinetic friction will act to reduce this relative motion.
f k = µN = 0.1 × 10 × 10 = 10 N
100 + 10 = 10a

110
a= = 11 m/s2
10


(b) STATIC FRICTION
t exists between the two surfaces when there is tendency of relative motion but no relative motion
along the two contact surface.
For example consider a bed inside a room ; when we gently push the bed with a finger, the bed does
not move. This means that the bed has a tendency to move in the direction of applied force but does
not move as there exists static friction force acting in the opposite direction of the applied force.

Example 8. What is value of static friction force on the block?

Solution : In horizontal direction as acceleration is zero.


Therefore  F = 0 .
 = 0

KVPY_ SA STREAM # 104



Direction of st a tic friction force :
The static friction force on an object is opposite to its impending motion relative to the surface.
Following steps should be followed in determining the direction of static friction force on an object.
(i) Draw the free body diagram with respect to the other object on which it is kept.
(ii) Include pseudo force also if contact surface is accelerating.
(iii) Decide the resultant force and the component parallel to the surface of this resultant force.
(iv) The direction of static friction is opposite to the above component of resultant force.

Note : Here once again the static friction is involved when there is no relative motion between two
surfaces.

Example 9. In the following figure an object of mass M is kept on a rough table as seen
from above. Forces are applied on it as shown. Find the direction of static
friction if the object does not move.

Solution : In the above problem we first draw the free body diagram of find the resultant force.

As the object doe not move this is not a case of limiting friction. The direction of static friction is opposite to
the direction of the resultant force FR as shown in figure by fs . Its magnitude is equal to 25 N.


4. M AGN I T U DE OF K I N ET I C AN D STAT I C FRI CT I ON
Kinetic friction :
The magnitude of the kinetic friction is proportional to the normal force acting between the two bodies. We
can write
f k = k N
where N is the normal force. The proportionality constant k is called the coefficient of kinetic friction
and its value depends on the nature of the two surfaces in contact. If the surfaces are smooth k will be
small, if the surfaces are rough k will be large. It also depends on the materials of the two bodies in contact.
Static friction :
The magnitude of static friction is equal and opposite to the external force exerted, till the object at which
force is exerted is at rest. This means it is a variable and self adjusting force. However it has a
maximum value called limiting friction.
f max = sN
The actual force of static friction may be smaller than sN and its value depends on other forces acting on the
body. The magnitude of frictional force is equal to that required to keep the body at relative rest.
0  f s  f smax
Here s and k are proportionality constants. s is called coefficient of static friction and k is called coefficient
of kinetic friction. They are dimensionless quantities independent of shape and area of contact . It is a
property of the two contact

KVPY_ SA STREAM # 105


surfaces. s > k for a given pair of surfaces. If not mentioned then s = k can be
taken. Value of  can be from 0 to .

Following table gives a rough estimate of the values of coefficient of static friction between certain pairs of
materials. The actual value depends on the degree of smoothness and other environmental factors. For
example, wood may be prepared at various degrees of smoothness and the friction coefficient will vary.

Material s Material s
Steel and steel 0.58 Copper and copper 1.60
Steel and brass 0.35 Teflon and teflon 0.04
Glass and glass 1.00 Rubber tyre on dry
1.0
Wood and wood 0.35 concrete road
Rubber tyre on wet
Wood and metal 0.40 0.7
concrete road

Example 10. Find acceleration of block. Initially the block is at rest.

50 N

10 Kg

µ=0.5

Solution : zero

Example 11. Find out acceleration of the block. Initially the block is at rest.

Solution : N + 24 – 100 = 0 for vertical direction


 N = 76 N
Now 0  f s  µs N
0  f s  76 × 0.5
0  f s  38 N
 32 < 38 Hence f = 32
 acceleration of block is zero.

KVPY_ SA STREAM # 106


Example 12. Find out acceleration of the block for different ranges of F.
F
m

µ
Solution : 0  f  µSN
0  f  µSmg
a=0 if F  µSmg
F  µMg
a= if F > µMg
M

Example 13. Find out acceleration of the block. Initially the block is at rest.

51N
10 kg

µS = 0.5
µk = 0.3

Solution : 0  f s  µSN
0  f s  50
Now 51 > 50
 Block will move but if the block starts moving then kinetic friction is involved.
KF = µk N = 0.3 × 100 = 30 N
51
10  51 – 30 = 10 a
30
 a = 2.1 m/s2
Example 14. Find out the minimum force that must be applied on the block vertically 100N
10kg
downwards so that the block doesn’t move.
N
µ=0.5
F
Solution : 10 kg
100N
f
10 g

100 – fs = 0
 f s = 100 ........ (1)
F + 10 g = N  N = 100 + F ........... (2)
Now 0  f S  N
100  0.5 N
100  0.5 [100 + F]
200  100 + F
F  100 N
 Minimum F = 100 N

Example 15. The angle of inclination is slowly increased. Find out the angle at which the block starts moving.

KVPY_ SA STREAM # 107


Solution : 0  f  µS N
N f
mg sin > f smax
s
 g co
mg sin  > µN sin m
g
mg sin > µ mg cos  m 
 tan  > µ
 = tan–1 µ
for tan  µ no sliding on inclined plane.
This method is used for finding out the value of µ practically.

Example 16. Find out the acceleration of the block. If the block is initially at rest.

Solution : (FBD of the block excluding friction)


N = 10 g cos 37° = 80 N N
Now 0  f s  µN 75
kg
0  f s  0.5 × 80 10
°
 f s  40 N 37
° c os
n 37 Mg
si
N Mg
75

60 80
We will put value of f in the last i.e. in the direction opposite to resultant of other forces. f acts down
the incline and its value is of = 75 – 60 = 15 N
So acceleration is zero
Example 17. In the above problem how much force should be added to 75 N force so that block starts to move
up the incline.
Solution :  60 + 40 = 75 + f extra
 f s = 25 N
Example 18. In the above problem what is the minimum force by which 75 N force should be replaced with so that
the block does not move.
Solution : In this case the block has a tendency to move downwards. F
Hence friction acts upwards. 40
 F + 40 = 60
 F = 20 N 60
Example 19. Top view of a block on a table is shown (g = 10 m/s2) .

Find out the acceleration of the block.

KVPY_ SA STREAM # 108


Solution :
40N

40 g FR = 40 2  56N

Now fs  µN
 f s  50
FR > f smax
Hence the block will move.

40 2  50
a=
10
 
= 4 2  5 m/s2

Example 20. Find minimum µ so that the blocks remain stationary.

Solution : T = 100 g = 1000 N 1000


 f = 1000 to keep the block stationary f
Now fmax = 1000
µN = 1000
µ=2
Can µ be greater than 1 ?
Yes 0 < µ  

Example 21. Find out minimum acceleration of block A so that the 10 kg block doesn’t fall.

Solution :
µN
Applying NL in horizontal direction
N = 10 a ......... (1) a
N 10
Applying NL in vertical direction
10 g = µ N .......(2) 10 g
10 g = µ 10 a from (1) & (2)

g
 a = = 20 m/s2
µ
Example 22. Find the tension in the string in situation as shown in the figure below. Forces 120 N and 100 N start
acting when the system is at rest and the maximum value of static friction on 10 kg is 90 N and that on
20 kg is 60N?

KVPY_ SA STREAM # 109


Solution : (i) Let us assume that system moves towards left then as it is clear from FBD, net force in horizontal
direction is towards right. Therefore the assumption is not valid.

Above assumption is not possible as net force on system comes towards right. Hence system is
not moving towards left.
(ii) Similarly let us assume that system moves towards right.

Above assumption is also not possible as net force on the system is towards left in this situation.
Hence assumption is again not valid.
Therefore it can be concluded that the system is stationary.

Assuming that the 10 kg block reaches limiting friction first then using FBD’s.

120 = T + 90  T = 30 N
Also T + f = 100
 30 + f = 100 
f = 70 N which is not possible as the limiting value is 60 N for this surface of block.
 Our assumption is wrong and now taking the 20 kg surface to be limiting we have

T + 60 = 100 N  T = 40 N
Also f + T = 120 N  f = 80 N
This is acceptable as static friction at this surface should be less than 90 N.
Hence the tension in the string is
T = 40 N.
Example 23. In the following figure force F is gradually increased from zero. Draw
the graph between applied force F and tension T in the string. The
coefficient of static friction between the block and the ground is s .

Solution : As the external force F is gradually increased from zero it is


compensated by the friction and the string bears no tension. When
limiting friction is achieved by increasing force F to a value till smg,
the further increase in F is transferred to the string.

Example 24. Force F is gradually increased from zero. Determine whether the block will first slide or lift up?

KVPY_ SA STREAM # 110


Solution : There are minimum magnitude of forces required both in horizontal and vertical direction either to
slide on lift up the block. The block will first slide on lift up will depend upon which minimum magni-
tude of force is lesser.
For vertical direction to start lifting up
F sin 37º + N – Mg 0.
N becomes zero just lifting condition.
10g
Flift 
3/5

500
 Flift  N
3
For horizontal direction to start sliding
F cos 37  sN
F cos 37º > 0.5 [ 10g – F sin 37º] ( N = 10 g – F sin 37º)
50
Hence Fslide >
cos 37 º 0.5 sin 37 º

500
Fslide > N
11

500
Flift > N.
3
 Fslide < Flift
Therefore the block will begin to slide before lifting.

T WO BLO CK PRO BLEM S

µ=0.5 10 A F = 50 N
Example 25. Find the acceleration of the two blocks. The system is initially
Smooth
at rest and the friction coefficient are as shown in the figure? 10 B
//////////////////////////////////////////////////
Solution : Method of solving
Step 1 : Make force diagram.
Step 2 : Show static friction force by f because value of friction is not known.
Step 3 : Calculate separately for two cases.
Case 1 : Move together
Step 4 : Calculate acceleration.
Step 5 : Check value of friction for above case.
Step 6 : If required friction is less than available it
means they will move together else
move separately.
Step 7: (a) above acceleration will be common
acceleration for both
Case 2 : Move separately
Step 7(b) If they move separately then kinetic friction is involved. whose value is µN.
Step 8 : Calculate acceleration for above case.
f
A 50
B
f
f max = µN
 f  50 N (available friction)

KVPY_ SA STREAM # 111


Move together Move separately

50
(i) a= = 2.5 m/s2 No need to calculate
10  10

(ii) Check friction for B :


f = 10 × 2.5 = 25
25 N is required which is less than available friction
hence they will move together.
and aA = aB = 2.5 m/s2

Example 26. Find the acceleration of the two blocks. The system is initially at rest and the friction coefficient are
as shown in the figure?

µ=0.5 10 A 101 N
smooth
10 B
//////////////////////////////////////////////////

Solution : f max = 50 N
 f  50 N
f
A 101 B
f

101
(i) If they move together a= = 5.05 m/s2
20
(ii) Check friction on B
f
10 f = 10 × 5.05 = 50.5 (required)

50.5 > 50 (therefore required > available)


Hence they will not move together.
(iii) Hence they move separately so kinetic friction is involved. 101
A
fk=50
fk = µN=50
B

101 50 50
 for aA = = 5.1 m/s2  aB = = 5 m/s2
10 10
Also aA > aB as force is applied on A.

KVPY_ SA STREAM # 112


1. Two blocks each of mass m are placed on a rough horizontal surface
and connected by a massless inelastic string as shown. The coefficient
of friction between each block and horizontal surface is . The string
connecting both the blocks initially has zero tension. The minimum Block B Block A
force to be applied on block A to just move the two block system m m
horizontally (without the string getting slack) is :
2mg
(A) 2mg (B)
2  1

2mg mg
(C) (D)
4 2  1 2  1

2. A mass m is supported as shown in the figure by ideal strings connected


to a rigid wall and to a mass 3m at rest on a fixed horizontal surface.
The string connected to larger mass is horizontal, that connected to
smaller mass is vertical and the one connected to wall makes an angle 60°
60° with horizontal.Then the minimum coefficient of static friction 3m
between the larger mass and the horizontal surface that permits the
m
system to remain in equilibrium in the situation shown is:
1 1
(A) (B)
3 3 3

3 3
(C) (D)
2 2

3. The coefficient of friction between the block A of mass m and block B of


mass 2m is . There is no friction between block B and the inclined plane.
If the system of blocks A and B are released from rest and there is no
slipping between A and B, then
(A) 2  cos–1 (2) (B) 2  sin–1 (2)
(C)  tan–1 () (D) 2 tan–1 (/2)

4.* A 200 gm block is pressed with a minimum force against a rough vertical wall so that the block does not fall.
If coefficient of static friction between the wall & the block is 0.4. Find the minimum force (in Newton) (g =
10 m/s2)
(A) 2 (B) 0.8 (C)  (D) 4 N

 1
5. Value of  is increased gradually from  = 0. At  = tan–1   both the blocks
2
just start slipping. Than value of 2 is: (g = 10 m/s2)
(A) 0.5 (B) 0.4
(C) 0.6 (D) 0.3

6. A block of mass 2 kg is given a push for a moment horizontally and


then the block starts sliding over a horizontal plane. The graph shows
the velocity-time graph of the motion. The co-efficient of sliding friction
between the plane and the block is:
(A) 0.02 (B) 0.20
(C) 0.04 (D) 0.40

KVPY_ SA STREAM # 113


7. Four blocks are arranged on a smooth fixed horizontal surface as
shown. The masses of upper blocks are 'm' kg and lower blocks
are 'M' kg as shown in figure. The coefficient of friction between
upper and lower blocks is . Both the upper blocks are connected
by inextensible light string having initial zero tension. For all four
blocks to move with same acceleration, the maximum value of
horizontal force F applied to right bottom block (as shown) is
2 mg(m  M) 2 mg (m  M)
(A) 2m  M
(B) 2g (m + M) (C) m  2M
(D) g (m + 2M)

8. Consider the following statements :


(1) Static friction may do positive work on an object.
(2) It may be possible that on a body kinetic friction act in the direction of motion of body
(3) A lift going down with retardation increases the weight of an object measured by a spring balance
The correct order of True/False of above statement is :
(A) T T T (B) T F T (C) T T F (D) F F T

9. A block is placed on a rough horizontal plane attached with an elastic spring as shown in figure.

Initially spring is unstretched. If the plane is now slowly lifted from  = 0° to  = 90°, then the graph
showing extension in the spring (x) versus angle () is

(A) (B) (C) (D)

10. A block of mass m rests on a rough horrizontal surface, having friction


co-efficient  = 0.8. A force F is applied on the block at an angle 53º to
the horizontal. What should be the minimum value of F, so that the
block starts sliding ?
(A) 20 mg (B) 10 mg
(C) 2 mg (D) None of these

11.* For the arrangement shown in figure a force F is applied on block A in


horizontal direction as shown in figure so that it's acceleration is 8 m/
sec2. What is the acceleration of block B with respect to block A.
3
(Friction coefficient is 0.5 at all surfaces and take g = 10m/sec 2 , tan 37° = )
4
22
(A) 10 m/sec2 (B) 8 3 m/sec2 (C) 2 41 m/sec2 (D) m/sec2
5

12. A small block is projected up from the bottom of a fixed incline, inclined at 60º from the horizontal. Coefficient
3
of friction between the block & the incline is . If the ratio of descending time to ascending time is  then
2
the value of 2 -
(A) 2 (B) 3 (C) 4/3 (D) 3/2

KVPY_ SA STREAM # 114


13. O is a point at the bottom of a rough plane inclined at an angle  to the
tan
horizontal. Coefficient of friction between AB is and between BO
2
3 tan 
is . B is the mid–point of AO. A block is released from rest at A.
2
Then identify which graphs are correct during motion of block from
point A to O taking direction down the incline plane as positive :

(A) (B) (C) (D)

14. In given diagram what is the minimum value of a horizontal external force F on Block 'A' so that block
'B' will slide on ground is:

(A) 30 N (B) 20 N (C) 10 N (D) Not possible

15. In the previous question the minimum force 'F' required, so that block 'A' will slip on block 'B' is :
(A) 30 N (B) 20 N (C) 10 N (D) Not possible

16. With reference to the figure shown, if the coefficient of friction at the surfaces is 0.42, then the force
required to pull out the 6.0 kg block with an acceleration of 1.50 m/s2 will be:

(A) 36 N (B) 24 N (C) 84 N (D) 51 N

17. Two blocks A and B are as shown in figure. The minimum horizontal force F applied on block ‘B’ for
which slipping begins at ‘B’ and ground is :

(A) 100 N (B) 120 N (C) 50 N (D) 140 N


 1
18. A block of mass m is lying on a wedge having inclination angle  = tan–1   .
5
2
Wedge is moving with a constant acceleration a = 2 m/s horizontally. The
minimum value of coefficient of friction , so that m remains stationary with
respect to wedge is :
5 5
(A) (B)
6 12
1 2
(C) (D)
5 5

KVPY_ SA STREAM # 115


19. Find the maximum horizontal force F which can be applied such that sliding does not occur between A and B.
A
µ = 0.5
1kg
F
2kg
B
µ = 0.1

(A) 20 N (B) 24 N (C) 30 N (D) 332 N

20. Two bodies placed on a wedge kept fixed on a horizontal surface


as shown in figure are separated by a spring. They are in equilib-
rium by a force of 600 N applied on A. There is no friction between
B and inclined plane and µ is coefficient of friction between A and
inclined plane then (g = 10 m/s2)-
3 3
(A) µ = (B) µ >
4 4
5 5
(C) µ = (D) µ >
6 6

KVPY PROBLEMS (PREVIOUS YEARS)

1. A block of mass M rests on a rough horizontal table. A steadily increasing horizontal force is applied such
that the block starts to slide on the table without toppling. The force is continued even after sliding has
started. Assume the coefficients of static and kinetic friction between the table and the block to be equal.
The correct representation of the variation of the frictional forces, ƒ, exerted by the table on the block with
time t is given by : [KVPY-2010]

(A) (B) (C) (D)

2. A girl holds a book of mass m against a vertical wall with a horizontal force F using her finger so that the book
does not move. The frictional force on the book by the wall is : [KVPY_2012]
(A) F and along the finger but pointing towards the girl
(B) F upwards where  is the coefficient of static friction
(C) mg and upwards
(D) equal and opposite to the resultant of F and mg

3. A box when dropped from a certain height reaches the ground with a speed v. When it skides from rest from
the same height down a rough inclined plane inclined at in angle 45° to the horizontal, it reaches the ground
with a speed v/3. The coefficient of sliding friction between the box and the plane is (acceleration due to
gravity is 10 ms-2) [KVPY_2014_SA]
8 1 2 1
(A) (B) (C) (D)
9 9 3 3

KVPY_ SA STREAM # 116


4. Consider the system shown below

F
X Y

A horizontal force F is applied to a block X of mass 8 kg such that the block Y of mass 2kg adjacent to it
does not slip downwards under gravity. There is no friction between the horizontal plane and the base of the
block X. The coefficient of friction between the surfaces of blocks X and Y is 0.5. Take acceleration due to
gravity to be 10ms-2. The minimum value of F is [KVPY_2014_SB]
(A) 200N (B) 160N (C) 40N (D) 240N

5. A small box resting on one edge of the table is struck in such a way that it slides off the other edge,
1 m away, after 2 seconds. The coefficient of kinetic friction between the box and the table.
[KVPY_2015_SB] [1 Mark]
(A) must be less than 0.05 (B) must be exactly zero
(C) must be more than 0.05 (D) must be exactly 0.05

EXERCISE - 1

1. (B) 2. (B) 3. (C) 4.* (C) 5. (C) 6. (B) 7. (A)

8. (A) 9. (A) 10. (D) 11.* (C) 12. (B) 13. (B) 14. (D)

15. (B) 16. (D) 17. (A) 18. (B) 19. (B) 20. (D)

EXERCISE - 2
1. (A) 2. (C) 3. (A) 4. (A) 5. (A)

1. FBD of B
N

f= N T
Fsin
mg N F
T = N1 = mg .... (i)
FBD of A T Fcos
F sin + N2 = mg .... (ii) f= N
F cos = T + N2 .... (iii) mg
From equations (i), (ii) and (iii)
F cos = mg + (mg – F sin)
F (cos + sin) = 2 mg
2mg
F=
cos    sin 

2mg
hence, F min =
1  2

KVPY_ SA STREAM # 117


2. At the instant 3m is about to slip, tension T
in all the strings are as shown
 3 mg = T cos 60° .... (1)
3 mg 60°
and mg = T sin 60° .... (2)
1
 = mg
3 3

3. For no relative motion between A and B, both move down the incline with
acceleration g sin .
From free body diagram of A
mg – N = mg sin2  ................. (1)
f = mg sin  cos   N ................. (2)
Solving 1 and 2
  tan  or  tan–1 ()

4.* F = mg (F = minimum force)


mg
F=

0.2kg  10m / s 2
= = 5N Ans.
0.4
5. At limiting case (10 + 20) g sin = [(10) (0.3) + 20 2] g cos
 2 = 0.6.

6. The magnitude of deceleration from graph is


80
a= = 2m/s2
40
The deceleration of block is
a
a = µg  µ= = 0.2
g
7. As the value of F is increased from zero, the frictional force between right upper and lower block is more in
comparision to that between left upper and lower block. Hence friction will reach its maximum value between
the right two blocks first. Based on the maximum value of friction f max and the mass that needs to be
accelerated by this force (see figure), the maximum acceleration of these blocks is
fmax  mg
amax = (m  m  M)  2 m  M

Now, looking at the entire system as whole

2 mg (m  M)
F = (2m + 2M) amax =
2m  M

8. Statement (1) : It is possible in a two block system moving together .


Statement (2) : It is possible in case of two block system sliding on each other.
Statement (3) : The acceleration of the lift is upward, hence N = m(g + a)

9. Till block starts moving x =0.


Afterwords
Kx = mg sin – mg cos
mg sin    mg cos 
x=
K
x is maximum at  = 90°.

KVPY_ SA STREAM # 118


10. To Slide

F cos 53º > s (mg + F sin 53º)

3F 3.2
> 0.8 mg + F
5 5
Which is impossible, so block cannot slide. This situation is called self-locking.

11.* aB,I = g sin 53° – gcos53°

4 1 3
=g    
5 2 5

g 5 
 = 5 ms2
5  2 

4 3
64  25  2  5  8  
3 4

1 1
12. S= (g sin  + µ g cos ) t 2a = (g sin  – µ g cos ) t 2d
2 2

td sin   µ cos 
 = = 3 =
ta sin   µ cos 

3
 2 = 3 ( = 60º & µ = )
2

13. For motion between AB


tan 
ma = mg sin  – mg cos 
2
g sin 
a= (downward)
2
For motion between BO
3 tan 
ma = mg cos  – mg sin 
2
g sin 
a= (upward)
2
The velocity increases from zero to maximum value at B and then starts decreasing with same rate and
finally becomes zero at O.

14.

F 1(max) = 0.2 × 10 × 10 = 20 N
F 2(max) = 0.1 × 30 × 10 = 30 N
F 1(max) < F 2(max)
 'B' can never move.

KVPY_ SA STREAM # 119


15. F 1(max) = 20 N

 F  20 N, to make 'A' slip on 'B'.

16. F – 8(0.42) (10) – 2(0.42) (10) = 6 (1.5)


F – 42 = 9
F = 51 N

17. The sliding shall start at lower surface first


it F > 0.5 [10 + 10] g
or F > 100 N

18.

ma cos  + m g sin  = µN
N + ma sin = mg cos 
N = [mg cos  – ma sin ]
ma cos  + mg sin  = µ [mgcos  – masin]
 a cos   g sin  
  =µ
 g cos  – a sin  
 a  g tan   5
µ =  g – a tan   = .
  12

19. No sliding between A and B. Hence both move together


F – 0.1 × 3 × g = 3a ...(i)
F = 2T ...(ii)
T – 0.5 × 1 . g = 1 . a ...(iii)
On solving
F  0.3g F 
= .   0. 5g 
3 2 
 F = 2 × (1.5 – 0.3) × 10 = 24 N

20.

Spring force
f s = mg sin  = 60 × 10 × sin 53° = 480 N
4
f = f s + 40 g sin 53° – F = 480 + 40 × 10 × – 600 = 200 M N
5

200 200 5
  = =
40  g  cos 53 3 6
40  10 
5

KVPY_ SA STREAM # 120


WORK , POWER & EN ERGY

INTRODUCTION :
The term ‘work’ as understood in everyday life has a different meaning in scientific sense. If a coolie is
carrying a load on his head and waiting for the arrival of the train, he is not performing any work in the
scientific sense. In the present study, we shall have a look into the scientific aspect of this most commonly
used term i.e., work.

W ORK DONE BY CONSTANT FORCE :


The physical meaning of the term work is entirely different from the meaning attached to it in everyday life. In
everyday life, the term ‘work’ is considered to be synonym of ‘labour’, ‘toil’, effort’ etc. In physics, there is a
specific way of defining work.
Work is said to be done by a force when the force produces a displacement in the body on which
it acts in any direction except perpendicular to the direction of the force.
For work to be done, following two conditions must be fulfilled.
(i) A force must be applied.
(ii) The applied force must produce a displacement in any direction except perpendicular to the direction
of the force.
 
Suppose a force F is applied on a body in such a way that the body suffers a displacement S in the
direction of the force. Then the work done is given by
W = FS

However, the displacement does not always take place in the direction of the force. Suppose a constant
   
force F , applied on a body, produces a displacement S in the body in such a way that S is inclined to F at
an angle . Now the work done will be given by the dot product of force and displacement.
 
W= F . S

Since work is the dot product of two vectors therefore it is a scalar quantity.
W = FS cos 
or W = (F cos )S
 W = component of force in the direction of displacement × magnitudes of displacement.

So work is the product of the component of force in the direction of displacement and the magnitude of the
displacement.
Also, W = F (S cos )
or work is product of the component of displacement in the direction of the force and the magnitude of the
displacement.

KVPY_ SA STREAM # 121


Spe cia l Ca se s :
Ca se (i)
When  = 90º, then W = FS cos 90º = 0
So, work done by a force is zero if the body is displaced in a direction perpendicular to the
direction of the force.
Examples :
1. Consider a body sliding over a horizontal surface. The work done
by the force of gravity and the reaction of the surface will be zero.
This is because both the force of gravity and the reaction act
normally to the displacement.

The same argument can be applied to a man carrying a load on his head and walking on a railway
platform.

2. Consider a body moving in a circle with constant speed. At every point of the circular path, the
centripetal force and the displacement are mutually perpendicular (Figure). So, the work done by the
centripetal force is zero. The same argument can be applied to a satellite moving in a circular orbit.
In this case, the gravitational force is always perpendicular to displacement. So, work done by
gravitational force is zero.

3. The tension in the string of a simple pendulum is always perpendicular to displacement. (Figure).
So, work done by the tension is zero.

Ca se (ii) :
When S = 0, then W = 0.
So, work done by a force is zero if the body suffers no displacement on the application of a force.

Example :
A person carrying a load on his head and standing at a given place does no work.

Ca se (iii) :
When 0º   < 90º [Figure], then cos  is positive. Therefore.
W (= FS cos ) is positive.
Work done by a force is said to be positive if the applied force has a
component in the direction of the displacement.

Examples :
1. When a horse pulls a cart, the applied force and the displacement are in the same direction. So,
work done by the horse is positive.
2. When a load is lifted, the lifting force and the displacement act in the same direction. So, work done
by the lifting force is positive.
3. When a spring is stretched, both the stretching force and the displacement act in the same direction.
So, work done by the stretching force is positive.

KVPY_ SA STREAM # 122


Ca se (iv) :
When 90º <   180º (Figure), then cos is negative. Therefore
W (= FS cos ) is negative.
Work done by a force is said to be negative if the applied
force has component in a direction opposite to that of the
displacement.

Examples :
1. When brakes are applied to a moving vehicle, the work done by the braking force is negative. This is
because the braking force and the displacement act in opposite directions.

2. When a body is dragged along a rough surface, the work done by the frictional force is negative. This
is because the frictional force acts in a direction opposite to that of the displacement.

3. When a body is lifted, the work done by the gravitational force is negative. This is because the
gravitational force acts vertically downwards while the displacement is in the vertically upwards
direction.

Example 1. Figure shows four situations in which a force acts on a box while the box slides rightward a distance
d across a frictionless floor. The magnitudes of the forces are identical, their orientations are as
shown. Rank the situations according to the work done on the box during the displacement, from
most positive to most negative.

(A) (B) (C) (D)

Answer : D, C, B, A

Explanation : In (D)  = 0º, cos  = 1 (maximum value). So, work done is maximum.
In (C)  < 90º, cos  is positive. Therefore, W is positive.
In (B)  = 90º, cos  is zero. W is zero.
In (A)  is obtuse, cos  is negative. W is negative.

W ORK DONE BY M ULTIPLE FORCES:
   
If several forces act on a particle, then we can replace F in equation W = F . S by the net force F where
   
 F = F 1 + F 2 + F 3 + .....
 
 W = [ F ] . S ...(i)

This gives the work done by the net force during a displacement S of the particle.
We can rewrite equation (i) as :
     
W = F 1 . S + F 2 . S + F 3 . S + .....
or W = W 1 + W 2 + W 3 + ..........
So, the work done on the particle is the sum of the individual works done by all the forces acting on the
particle.

KVPY_ SA STREAM # 123


Im porta nt points a bout work :
1. Work is defined for an interval or displacement. There is no term like instantaneous work similar to
instantaneous velocity.
2. For a particular displacement, work done by a force is independent of type of motion i.e. whether it
moves with constant velocity, constant acceleration or retardation etc.
3. For a particular displacement work is independent of time. Work will be same for same displacement
whether the time taken is small or large.
4. When several forces act, work done by a force for a particular displacement is independent of other
forces.
5. A force is independent from reference frame. Its displacement depends on frame so work done by a
force is frame dependent therefore work done by a force can be different in different reference frame.
6. Effect of work is change in kinetic energy of the particle or system.
7. Work is done by the source or agent that applies the force.
Units of work :
1. Unit of work :
I. In cgs system, the unit of work is erg.
One erg of work is said to be done when a force of one dyne displaces a body through one centimetre
in its own direction.
 1 erg = 1 dyne × 1 cm = 1g cm s–2 × 1 cm = 1 g cm2 s–2
Note, Erg is also called dyne centimetre.
II. In SI i.e., International System of units, the unit of work is joule (abbreviated as J). It is named after
the famous British physicist James Personal Joule (1818 – 1869).
One joule of work is said to be done when a force of one Newton displaces a body through one metre
in its own direction.
1 joule = 1 Newton × 1 metre = 1 kg ×1 m/s2 × 1 m = 1kg m2 s–2
Note Another name for joule is Newton metre.
Relation between joule and erg
1 joule = 1 Newton × 1 metre
1 joule = 105 dyne × 102 cm = 107 dyne cm
1 joule = 107 erg
1 erg = 10–7 joule

DIM ENSIONS OF W ORK :


[Work] = [Force] [Distance] = [MLT–2] [L] = [ML2T–2]
Work has one dimension in mass, two dimensions in length and ‘–2’ dimensions in time,
On the basis of dimensional formula, the unit of work is kg m2 s–2.
Note that 1 kg m2 s–2 = (1 kg m s–2) m = 1 N m = 1 J.

Example 2. There is an elastic ball and a rigid wall. Ball is thrown towards the wall. The work done by the
normal reaction exerted by the wall on the ball is -
(A) +ve (B) – ve (C*) zero (D) None of these
Solution : As the point of application of force does not move, the w.d by normal reaction is zero.

Example 3. Work done by the normal reaction when a person climbs up the stairs is -
(A) +ve (B) – ve (C*) zero (D) None of these
Solution : As the point of application of force does not move, the w.d by normal reaction is zero.

Example 4. Work done by static friction force when a person starts running is ________ .
Solution : As the point of application of force does not move, the w.d by static friction is zero.

KVPY_ SA STREAM # 124



WORK DONE BY VARIOUS REAL FORCES

Work done by gravity Force.

(1)
Example 5. 10 m/s (2)
10 m/s
(3)
(4)

100 m

The mass of the particle is 2 kg. It is projected as shown in four different ways with same speed of
10 m/s. Find out the work done by gravity by the time the stone falls on ground.

 
Solution : W = F S cos  = 2000 J in each case.


Work done by normal reaction.

100m
Example 6. F=120N
20kg 10kg A B

(i) Find work done by force F on A during 100 m displacement.


(ii) Find work done by force F on B during 100 m displacement.
(iii) Find work done by normal reaction on B and A during the given displacement.
(iv) Find out the kinetic energy of block A & B finally.

Solution : (i) (W F)on A = FS cos 


= 120 × 100 × cos 0°
= 12000 J
(ii) (W F)on B =0
 F does not act on B

4 m/s2

(iii) N 10 kg N = 10 × 4 = 40 N 120 20 kg 40
B A

(W N)on B = 40 × 100 × cos 0°


= 4000 J
(W N)on A = 40 × 100 × cos 180° = – 4000 J

KVPY_ SA STREAM # 125


(iv) v 2 = u2 + 2as u=0
 v 2 = 2 × 4 × 100  v = 20 2 m/s

1
 KEA = × 20 × 800 = 8000 J
2
1
KEB = × 10 × 800 = 4000 J
2
W.D. by normal reaction on system of A & B is zero. i.e. w.d. by internal reaction on a rigid system
is zero.

Example 7. A particle is displaced from point A (1, 2) to B(3, 4) by applying force F = 2 î + 3 ĵ . Find the

work done by F to move the particle from point A to B.
 
Solution : W = F . S
A B
S = (3 – 1) î + (4 – 2) ĵ >
(1,2) (3,4)
= (2 î +3 ĵ ) . (2 î + 2 ĵ ) = 2 × 2 + 3 × 2 = 10 units


ENERGY :
Definition: Energy is defined as internal capacity of doing work. When we say that a body has energy we
mean that it can do work.
Energy appears in many forms such as mechanical, electrical, chemical, thermal (heat), optical (light),
acoustical (sound), molecular, atomic, nuclear etc., and can change from one form to the other.

KINETIC ENERGY :
Definition:
Kinetic energy is the internal capacity of doing work of the object by virtue of its motion.
Kinetic energy is a scalar property that is associated with state of motion of an object. An aero-plane in
straight and level flight has kinetic energy of translation and a rotating wheel on a machine has kinetic energy
of rotation. If a particle of mass m is moving with speed ‘v’ much less than the speed of the light than the
kinetic energy ‘K’ is given by
1
K mv 2
2
Im porta nt Point s for K.E.
 
1. As mass m and v 2 ( v . v ) are always positive, kinetic energy is always positive scalar i.e, kinetic
energy can never be negative.
2. The kinetic energy depends on the frame of reference,
p2
K= and P= 2 m K ; P = linear momentum
2m
The speed v may be acquired by the body in any manner. The kinetic energy of a group of particles
or bodies is the sum of the kinetic energies of the individual particles. Consider a system consisting
of n particles of masses m1, m2, ......, mn. Let v1 , v 2 , ..... , v n be their respective velocities. Then,
the total kinetic energy Ek of the system is given by
1 1 1
Ek = m1v 12 + m2v 22 + ......... + mnv n2
2 2 2
If m is measured in gram and v in cm s–1, then the kinetic energy is measured in erg. If m is
measured in kilogram and v in m s–1, then the kinetic energy is measured in joule. It may be noted
that the units of kinetic energy are the same as those of work. Infect, this is true of all forms of
energy since they are inter-convertible.

KVPY_ SA STREAM # 126


Typica l k inet ic e nergies (K) :

S.No. Object Mass (kg) Speed (m s–1) K(J)


1 Air molecule –26 500 –21
10  10
2 Rain drop at terminal speed 3.5 × 10–5 9 1.4 × 10–3
3 Stone dropped from 10 m 1 14 10 2
4 Bullet 5 × 10–5 200 10 3
5 Running athlete 70 10 3.5 × 103
6 Car 2000 25 6.3 × 105

RELATION BETW EEN M OM ENTUM AND KINETIC ENERGY :


Consider a body of mass m moving with velocity v. Linear momentum of the body, p = mv

1
Kinetic energy of the body, Ek = mv 2
2

1 p2
Ek = (m2v 2) or Ek = or p= 2mE k
2m 2m

Example 8. The kinetic energy of a body is increased by 21%. What is the percentage increase in the magnitude
of linear momentum of the body?
121 1 121 1 11
Solution : Ek2 = E or m v 22 = mv 12 or v2 = v
100 k1 2 100 2 10 1

11 11
or mv 2 = mv1 or p2 = p
10 10 1

p2 11 1
or –1= –1=
p1 10 10

p 2  p1 1
or p1
× 100 = × 100 = 10
10
So, the percentage increase in the magnitude of linear momentum is 10%.

Example 9. smooth F = 10N


10kg

Force shown acts for 2 seconds. Find out w.d. by force F on 10 kg in 3 seconds.
    
Solution : w = F  S  w = F  S cos 0°  w = 10 S

1 2 1
Now 10 = 10 a  a = 1 m/s2  S= at = × 1 × 22 = 2 m
2 2
w = 10 × 2 = 20 J

Example 10. Find Kinetic energy after 2 seconds.


Solution : V = 0 + at  V = 1 × 2 = 2m/s
1
 KE × 10 × 22 = 20 J.
2

KVPY_ SA STREAM # 127



W ORK DONE BY A VARIABLE FORCE :
When the magnitude and direction of a force vary in three dimensions, it can be expressed as a function of
the position. For a variable force work is calculated for infinitely small displacement and for this displacement
force is assumed to be constant
 
dW = F.ds
The total work done will be sum of infinitely small work
B B
 
WA  B = 
A
F  ds =
 (F cos ) ds
A
In terms of rectangular components,
 
F = Fx î + Fy ĵ + Fz k̂  ds = dx î + dy ĵ + dz k̂
x y z
B B B
W A B  F
x
x dx   F dy   F dz
y
y
z
z
A A A

 
Example 11. An object is displaced from position vector r1  (2 î  3 ĵ ) m to r2  ( 4 î  6 ĵ ) m under the action of a

force F  (3x 2 î  2yĵ )N . Find the work done by this force.
 
rf   r2
Solution : W  
ri
F  dr =  
r1
(3x 2 î  2y ĵ)  (dx î  dy ĵ  dzk̂ )

r2
=  
r1
(3x 2 dx  2ydy ) = [ x 3  y 2 ] (( 24,, 36)) = 83 J Ans.


AREA UNDER FORCE DISPLACEM ENT CURVE :
Graphically area under the force-displacement is the work done

The work done can be positive or negative as per the area above the x-axis or below the x-axis respectively.

Example 12. A force F = 0.5x + 10 acts on a particle. Here F is in Newton and x is in metre. Calculate the work
done by the force during the displacement of the particle from x = 0 to x = 2 metre.

Solution : Small amount of work done dW in giving a small displacement dx is given by
 
dW = F . dx
or dW = Fdx cos 0º
or dW = Fdx [ cos 0º = 1]
x2 x 2
Total work done, W =

x 0
Fdx  
x 0
(0.5 x + 10)dx

x 2 x2
x 2 x 2 x2
= x 0
0.5x dx  x 0
10 dx = 0.5
2
x 0
+ 10 x
x 0

0.5 2
= [2 – 02] + 10[2 – 0] = (1 + 20) = 21 J
2

KVPY_ SA STREAM # 128




Work done by Variable Force W =  dW   F  ds


Example 13. An object is displaced from point A(1, 2) to B(0, 1) by applying force F = x î + 2y ĵ .

Find out work done by F to move the object from point A to B.
  A B
Solution : dW = F . ds >
(1,2) (0,1)
dW = (x î + 2y ĵ ) (dx î + dy î )

0 1

dW =  x dx +  2y dy
1 2

 W = – 3.5 J

Example 14. The linear momentum of a body is increased by 10%. What is the percentage change in its kinetic
energy?
Solution : Percentage increase in kinetic energy = 21%]

 110 11 E2  11 
2
121
 H int .mv 2  mv1, v 2  v1,   
 100 10 E1  10  100

E2  E1 
Percentage increase in kinetic energy   100  = 21%
E1 

Example 15. A time dependent force F = 10 t is applied on 10 kg block as shown in figure.


t=0
F = 10 t
10kg

Find out the work done by F in 2 seconds.


 
Solution : dW = F . ds
dW = 10 t. dx
dW = 10 t v dt .............. (1)  dx = vdt
dv
also 10 × = 10 t
dt

v t t2
  dv =  t dt
0 0
 v=
2
................. (2)

 t2 
from (1) & (2) dW = 10 t .  2  dt dW = 5t3dt
 

W=
5 4
4
t  
2
0
= 20 J

1
Aliter : K.E. = × 10 (22 – 0) = 0
2

KVPY_ SA STREAM # 129



WORK DONE BY SPRING FORCE

Example 16.

Initially spring is relaxed. A person starts pulling the spring by applying a variable force F. Find out
the work done by F to stretch it slowly to a distance by x.
x
 Kx 2  2
 = Kx
x
Solution :  dW =  F  ds = 0 K x dx  W = 
 2

0 2
Example 17. In the above example
(i) Where has the work gone ?
(ii) Work done by spring on wall is zero. Why?
(iii) Work done by spring force on man is _______ .
Solution : (i) It is stored in the form of potential energy in spring.
(ii) Zero, as displacement is zero.
1 2
(iii) – Kx
2

Example 18. Find out work done by applied force to slowly extend the spring from x to 2x.
Solution : Initially the spring is extended by x
LN
 
W = F . ds
F
2x
W=  K x . dx
x

2x x=0 x=x x = 2x
 Kx 2  3
W=  2  = Kx2
  x 2
2x
It can also found by difference of PE.
1 1 2 3 2
i.e. Uf = K (2x)2 = 2kx2  Ui = kx  Uf – Ui = kx
2 2 2


WORK DONE BY OTHER CONSTANT FORCES

Example 19. A block of mass m is released from top of a smooth fixed inclined plane of inclination .

Find out work done by normal reaction & gravity during the time block comes to bottom.
Solution : W N = 0 as F  S
 
W g = F . S = mg . S . cos (90 – ) = mg S sin  = mgh

KVPY_ SA STREAM # 130


Example 20. Find out the speed of the block at the bottom and its kinetic energy.
Solution : V2 = u2 + 2as
h
V2 = 0 + 2(g sin )  V2 = 2gh  V= 2gh
sin 

1
KE = mv 2 = mgh
2


WORK DONE BY TENSION

Example 21. A bob of pendulum is released at rest from extreme position as


shown in figure. Find work done by tension from A to B , B to C
and C to A.

Solution : Zero because FT  dS at all time.

Example 22. In the above question find out work done by gravity from A to B and B to C.
 
Solution : W g = F . S
= mg S cos 
W g = mg( –  cos ) for A to B
W g = – mg ( –  cos ) for B to C

Example 23.

The system is released from rest. When 10 kg block reaches at ground then find :
(i) Work done by gravity on 10 kg (ii) Work done by gravity on 5 kg
(iii) Work done by tension on 10 kg (iv) Work done by tension on 5 kg.

KVPY_ SA STREAM # 131


Solution : (i) (W g)10 kg = 10 g x 2
= 200 J
(ii) (W g) 5kg = 5 g × 2 × cos 180°
= – 100 J
200
(iii) (W T) 10 kg = × 2 × cos 180°
3

400
= J
3

200 400
(iv) (W T) 5 kg = × 2 × cos 0° = J
3 3
Net w.d. by tension is zero. Work done by internal tension i.e. (tension acting within system) on the
system is always zero if the length remains constant.

Example 24. A block of mass m is released from top of an incline plane of inclination . The coefficient of friction
between the block and incline surface is  ( < tan ). Find work done by normal reaction, gravity &
friction, when block moves from top to the bottom.

Solution : WN = 0  FN  S
W g = mg  sin 
W f = – µmg cos  . 

Example 25. What is kinetic energy of block of mass m at bottom in above problem.
Solution : V2 = u2 + 2as
V2 = 2(gsin  – µ g cos ) ()

1
 KE = m 2 (g sin  – µg cos ) 
2
= mg  (sin  – µ cos )

WORK DONE BY FRICTION

Example 26. In the given figure

(i) Find work done by applied force during displacement 2m.


(ii) Find work done by frictional force on B by A during the displacement.

KVPY_ SA STREAM # 132


Solution : (i) 100 × 2 × cos 0° = 200 J
(ii) f smax = µmg = 0.5 × 10 g = 50 N
Assuming they move together.
100 = 20 a  a = 5 m/s2
Check Friction on A
f = 10 × 5 = 50 N
f reqd = favailable  They move together
Hence (W f)on B = – 100 J
(W f) on A = 100 J
}
Net zero
i.e. w.d. by internal static friction is zero.

Example 27. A block of mass 10 kg is projected with speed 10 m/s on the surface of the plank of mass 10 kg,
kept on smooth ground as shown in figure.

(i) Find out the velocity of two blocks when frictional force stops acting.
(ii) Find out displacement of A & B till velocity becomes equal.
2
A 1m/s 2
1m/s
B 10N
Solution : (i) 10 10
10N
VA = 10 – 1t  VB = 1t
Frictional force stops acting when
VA = VB  10 – t = t
10 = 2t  t = 5 sec.
VB = VA = 5m/s
5m/s
Situation becomes
5 m/s

1
(ii) SA = 10 x 5 – × 1 × 52 = 37.5 m
2


SB = × 1 × 52 = 12.5 m
2

Example 28. In the above question find work done by kinetic friction on A & B.

Solution : (W KF)on A = 10 × 37.5 cos 180° = – 375 J

(W KF)on B = 10 × 12.5 cos 0


= 125 J
work done by KF on system of A & B
= – 375 + 125 = – 250 J
Work done by KF on a system is always negative.
Heat generated = – (W KF) on system
(W KF) on system = – (f K × Srelative)
= – 10 × 25 = – 250 J

KVPY_ SA STREAM # 133


True / False :
Example 29. Work done by kinetic friction on a body is never zero.
Answer : False

Example 30. Work done by kinetic friction on a system is always negative.


Answer : True


WORK DONE BY PSEUDO FORCE
Kinetic Energy of a body frame dependent as velocity is a frame dependent quantity. Therefore
pseudo force work has to be considered.

Example 31. A block of mass 10 kg is pulled by force F = 100 N. It covers a distance 500 m in 10 sec. From initial
point. This motion is observed by three observers A, B and C as shown in figure.

a=0 u=0
a=0 10m/s a=10 m/s2
A v=0 B C

Find out work done by the force F in 10 seconds as observed by A, B & C.


Solution : (W F)on block w.r.t A = 100 × 500 J = 50,000 J
(W F)on block w.r.t B = 100 [500 – 10 × 10] = 40,000 J
(W F)on block w.r.t C = 100 [500 – 500] = 0


WORK DONE BY INTERNAL FORCE
FAB = – FBA i.e. sum of internal forces is zero.

But it is not necessary that work done by internal force is zero. There must be some deformation or
reformation between the system to do internal work. In case of a rigid body work done by internal force is
zero.
W ork -Energy The orem :
According to work-energy theorem, the work done by all the forces on a particle is equal to the change in its
kinetic energy.
W C + W NC + W PS = K
Where, W C is the work done by all the conservative forces.
W NC is the work done by all non-conservative forces.
W PS is the work done by all psuedo forces.

M odified Form of W ork -Energy Theorem :


We know that conservative forces are associated with the concept of potential energy, that is
W C = U
So, Work-Energy theorem may be modified as
W NC + W PS = K + U
W NC + W PS = E

KVPY_ SA STREAM # 134


Example 32. A body of mass m when released from rest from a height h, hits the ground with speed gh .
Find work done by resistive force.
Solution : Identify initial and final state and identify all forces.
W g + W air res. + W int force = K

mgh + W air res + 0 =


1
2
m  gh 
2
–0

mgh
 W air res. = 
2

Example 33. The bob of a simple pendulum of length l is released when the string is horizontal. Find its speed at
the bottom. A
Solution : W g + W T = K O

1
mg + 0 = mu2 – 0 
2

u= 2g 
u
B
Example 34. A block is given a speed u up the inclined plane as shown.

u x

µ

Using work energy theorem find out x when the block stops moving.
Solution : W g + W f + W N = K

1 u2
– mg x sin  – µ mgx cos  + 0 = 0 – mu2  x=
2 2g(sin   cos )

Example 35. The masses M1 and M2 (M2 > M1) are released from rest.

M1 M2

Using work energy theorem find out velocity of the blocks when they move a distance x.
Solution : (W all F)system = (K)system
(W g)sys + (W T)sys = (K)sys as (W T)sys = 0

1
M2gx – M1gx = (M + M2)V2 – 0 ........... (1)
2 1

2(M2 – M1 )gx
V= M1  M2

KVPY_ SA STREAM # 135


Example 36. In the above question find out acceleration of blocks.
1 dv
Solution : (M2g – M1g) = (M1 + M2) 2v [Differentiating equation (1) above]
2 dx

 M2  M1  dv
  M  M g = v =a
 1 2  dx

Example 37. A stone is projected with initial velocity u from a building of height h. After some time the stone
falls on ground. Find out speed with it strikes the ground.
Solution : W all forces = K
W g = K u >
1 1
mgh = mv 2 – mu2
2 2

v= u 2  2gh


Power :
Power is defined as the time rate of doing work.
When the time taken to complete a given amount of work is important, we measure the power of the agent of
doing work.
The average power ( P or pav) delivered by an agent is given by
W
P or pav = where W is the amount of work done in time t.
t
Power is the ratio of two scalars- work and time. So, power is a scalar quantity. If time taken to complete a
given amount of work is more, then power is less. For a short duration dt, if P is the power delivered during
this duration, then
  
F  dS  dS  
P = F  = F . v
dt dt
This is instantaneous power. It may be +ve, –ve or zero.
By definition of dot product,
P = Fv cos 
 
where  is the smaller angle between F and v .
This P is called as instantaneous power if dt is very small.

Example 38. A block moves in uniform circular motion because a cord tied to the block is anchored at the centre
of a circle. Is the power of the force exerted on the block by the cord is positive, negative or zero?
Answer : Zero
 
Explanation. F and v are perpendicular..
 
 Power = F . v = Fv cos 90º = Zero.


Unit of Power :
A unit power is the power of an agent which does unit work in unit time.
The power of an agent is said to be one watt if it does one joule of work in one second.
1 watt = 1 joule/second = 107 erg/second

1 newton  1 metre
Also, 1 watt = = 1 N m s–1 .
1 second

KVPY_ SA STREAM # 136


Dimensional formula of power

[ Work] [ ML2 T 2 ]
[Power] = = = [ML2 T–3]
[Time] [T]
Power has 1 dimension in mass, 2 dimensions in length and – 3 dimensions in time.

S.No. Human Activity Power (W)


1 Heart beat 1.2
2 Sleeping 83
3 Sitting 120
4 Riding in a car 140
–1
5 Walking (4.8 km h ) 265
–1
6 Cycling (15 km h ) 410
7 Playing Tennis 440
–1
8 Swimming (breaststroke, 1.6 km h ) 475
9 Skating 535
–1
10 Climbing Stairs (116 steps min ) 685
–1
11 Cycling (21.3 km h ) 700
12 Playing Basketball 800

13 Tube light 40
14 Fan 60

Example 39 A block moves with constant velocity 1 m/s under the action of horizontal force 50 N on a horizontal
surface. What is the power of external force and friction?

Solution : Since a = 0 i.e. f k = 50 N


Pext = 50 × 1 = 50 W
Pf = –50 × 1= –50 W

Power is also the rate at which energy is supplied.
Net power = P1 + P2 + P3 .............

dW1 dW2  dW1  dW2  .......... 


Pnet = + ......  Pnet =  dt

dt dt  

dK
Pnet =  W all = K
dt
 Rate of change of kinetic energy is also power.

Example 40 A stone is projected with velocity at an angle  with horizontal. Find out
(i) Average power of the gravity during time t.
(ii) Instantaneous power due to gravitational force at time t where t is time of flight.
(iii) When is average power zero ?
(iv) When is Pinst zero ?
(v) When is Pinst negative ?
(vi) When is Pinst positive ?

KVPY_ SA STREAM # 137


 1 
mg u sin t  gt 2 
w mgh  2 
Solution : (i) <P> = =– =–
T t t
v
 gt  u
<P> = mg   u sin 
2  h

(ii) Instantaneous power
 
P = Fv  
= (–mg ĵ ) [ucos î + (usin – gt) ĵ ) ]
= –mg(usin  – gt)
gt 2u sin 
(iii)  u sin   t= , i.e. time of flight.
2 g

  u sin 
(iv) When F & V are  i.e. at t = which is at the highest point.
g
(v) From base to highest point.
(vi) From highest point to base.

POTEN TIAL EN ERGY
Energy :- It is the internal capacity to do work.

Kinetic Energy :- It is internal capacity to do work by virtue of relative motion.

Potential Energy :- It is the internal capacity to do work by virtue of relative position.


Example :- Gravitational Potential Energy , Spring PE etc.

Potential Energy
Definition:
Potential energy is the internal capacity of doing work of a system by virtue of its configuration.
In case of conservative force (field) potential energy is equal to negative of work done by
the conservative force in shifting the body from some reference position to given position.
Therefore, in case of conservative force
U2 r2   r2  
U1 
dU   F  d r
r1
i.e., U 2  U1  
r 
F  dr  W
1

Whenever and wherever possible, we take the reference point at  and assume potential energy to be zero
there, i.e., If we take r1 =  and U1 = 0 then
r 

U   F  d r  W

(a) Gravitation Potential Energy :


U = mgh for a particle at a height h above reference level.

Example 41 Calculate potential energy of a uniform vertical rod of mass M and length .

KVPY_ SA STREAM # 138


Solution : M dU = (dm) gx
U 
M 
 dU     dx gx
dx 0 0

x Mg
U=
2

(b) Spring potential energy :
1 2
U= Kx
2
Where x is change in length from its natural length.

Note : Gravitational potential energy can be +ve , –ve or zero but spring potential energy will always be +ve.

Example 42 In the given figure, a uniform rod of mass m and length l is hinged
at one end. Find the work done by applied force in slowly bringing  = 1m
=60°
the rod to the inclined position.
m=5kg
Solution : W ALL = K by work energy theorem Fapp
Ny
  1 Nx 
x= – cos 60° = m = 0.25 m
2 2 4
WNx  WNy  Wg  WFapp  K ( W  W  0 ) x
Nx Ny

 0 – mg(0.25) + W Fapp = 0 – 0
 K = 0 as slowly brought Fapp
 W Fapp = 5 × 9.8 × 0.25 mg

It can be seen that W Fapp = mgh = 5 × 9.8 × 0.25 J.


M, 
Example 43 A uniform rod of mass M and length  is held in position
shown in the figure. Find the potential energy of the rod.

U=O

Solution : dU = dm.g.h
M, 

M dx
 dU = 0  dx . g . x sin  x
xsin 

U=O
Mg sin  
U=
2

 
Note that centre of mass of the rod is at height  2 sin   from the ground.
 

Example 44 A uniform solid sphere of mass M and radius R is kept on the horizontal surface.

M,R

Find potential energy of the solid sphere.

KVPY_ SA STREAM # 139


Solution : U = Mghcm
for symmetrical body COM is the geometrical centre of the body.
 U = MgR

Example 45 Find work done by gravity in moving the block


(i) from A to B M
B
(ii) from B to A
(iii) Calculate UA – UB
(iv) Calculate UB – UA h

Solution : (W g)A to B = – mgh


M
(W g)B to A = mgh A U=0
UA – UB = – mgh
UB – UA = mgh


It can be said
Wg = – U
Work done by gravity is the -ve of the change in PE .
i.e. W g = – [Uf – Ui]  W g = Ui – Uf

Important Points for P.E. :


1. Potential energy can be defined only for conservative forces. It has no relevance for non-conservative
forces.
2. Potential energy can be positive or negative, depending upon choice of frame of reference.
3. Potential energy depends on frame of reference but change in potential energy is independent of
reference frame.
4. Potential energy should be considered to be a property of the entire system, rather than assigning
it to any specific particle.
5. It is a function of position and does not depend on the path.


Work done by spring force
As above W SP = – U
2
U=0 U = 1/2 kx
U=0
k

x=0 x=x

 W SPF = – U
W SPF = Ui – UF

1 2 1
W SPF = 0 – kx = – kx2
2 2

KVPY_ SA STREAM # 140


Example 46

Peg


M,L

Find out work done by external agent to slowly hang the lower end of the chain to the peg.

Peg

L
Solution : M,L Initially Ui = – Mg
2

L
Uf = – Mg
4

L L L
 W g = Ui – Uf = (–Mg ) – (–Mg ) = – Mg
2 4 4

L
Using work energy theorem, W g + W ext = K = 0  W ext = – W g = Mg
4

Example 47 K = 100 N/m

C B A
x=0 x=0.1 x=0.2

Find out the work done by spring force from A to B and from B to C. x = 0 is position of natural length.

1 1
Solution : (W spring)AB = Ui – Uf = K (0.2)2 – K(0.1)2
2 2

3
 (W spring)AB = J
2
Similarly

1
(W spring)BC = J
2

KVPY_ SA STREAM # 141


Example 48 (a) The mass m is moved from A to C along three different paths
(i) ABC
(ii) ADC
(iii) AC
Find out work done by gravity in the three cases.
(b) The block is moved from A to C along three different paths.
Applied force is horizontal. Find work done by friction force
in path
(i) ABC
(ii) ADC
Rough
(iii) AC horizontal
Solution : (a) (i) –mgh plane
(ii) –mgh
(iii) –mgh
(b) (i) W ABC = –mg (a+b)
(ii) W ADC = –mg (a+b)

(iii) W AC = –mg ( a 2  b 2 )


CONSERVATIVE FORCES
A force is said to be conservative if work done by or against the force in moving a body depends only on the
initial and final positions of the body and not on the nature of path followed between the initial and final
positions.

Consider a body of mass m being raised to a height h vertically upwards as show in above figure. The work
done is mgh. Suppose we take the body along the path as in (b). The work done during horizontal motion is
zero. Adding up the works done in the two vertical parts of the paths, we get the result mgh once again. Any
arbitrary path like the one shown in (c) can be broken into elementary horizontal and vertical portions. Work
done along the horizontal parts is zero. The work done along the vertical parts add up to mgh. Thus we
conclude that the work done in raising a body against gravity is independent of the path taken. It only
depends upon the initial and final positions of the body. We conclude from this discussion that the force of
gravity is a conservative force.

Examples of Conservative forces.

(i) Gravitational force, not only due to the Earth but in its general form as given by the universal law of
gravitation, is a conservative force.

(ii) Elastic force in a stretched or compressed spring is a conservative force.

(iii) Electrostatic force between two electric charges is a conservative force.

(iv) Magnetic force between two magnetic poles is a conservative forces.

In fact, all fundamental forces of nature are conservative in nature.

KVPY_ SA STREAM # 142


Forces acting along the line joining the centres of two bodies are called central forces. Gravitational force
and Electrostatic forces are two important examples of central forces. Central forces are conservative
forces.

PROPERTIES OF CONSERVATIVE FORCES

(i) Work done by or against a conservative force depends only on the initial and final positions of the
body.

(ii) Work done by or against a conservative force does no depend upon the nature of the path between initial and
final positions of the body.

If the work done a by a force in moving a body from an initial location to a final location is independent of the
path taken between the two points, then the force is conservative.

(iii) Work done by or against a conservative force in a round trip is zero.

If a body moves under the action of a force that does no total work during any round trip, then the force is
conservative; otherwise it is non-conservative.

The concept of potential energy exists only in the case of conservative forces.

(iv) The work done by a conservative force is completely recoverable.

Complete recoverability is an important aspect of the work of a conservative force.

NON-CONSERVATIVE FORCES

A force is said to be non-conservative if work done by or against the force in moving a body depends upon the
path between the initial and final positions.

The frictional forces are non-conservative forces. This is because the work done against friction depends on
the length of the path along which a body is moved. It does not depend only on the initial and final positions.
Note that the work done by frictional force in a round trip is not zero.

The velocity-dependent forces such as air resistance, viscous force etc., are non conservative forces.

S.No. Conse rva tive force s Non-Conse rva tive force s

1 W ork done does not depend upon path W ork done depends on path.

2 W ork done in round trip is z ero. W ork done in a round trip is not z ero.

Forces are veloc ity -dependent and


3 Central in nature.
retarding in nature.

W hen only a c onservative forc e ac ts within a


sys trem, the k inetic enrgy and potential energy W ork done against a non-c onservative
4
can change. However their sum, the m echanic al forc e may be dis sipated as heat energy.
energy of the s ystem, does not change.

W ork done in not c ompletely


5 W ork done is completely recoverable.
recoverable.

KVPY_ SA STREAM # 143


Example 49 The figure shows three paths connecting points a and b.
A single force F does the indicated work on a particle
moving along each path in the indicated direction. On the
basis of this information, is force F conservative?

Answer : No

Explanation : For a conservative force, the work done in a round trip should be zero.


Example 50 Find the work done by a force F = x î + y ĵ acting on a particle to displace it from point A (0, 0) to
B(2, 3).

Solution :

.
dW = F .ds = (x î + y ĵ ) (dx î + dy ĵ )

2 3
2 x2 y2 313
0 0   0

W = xdx + ydy =  2  +  2  =
  0 2
units

True or False
Example 51. In case of a non conservative force work done along two different paths will always be different.
Answer : False
Example 52. In case of non conservative force work done along two different paths may be different.
Answer : True
Example 53. In case of non conservative force work done along all possible paths cannot be same.
Answer : True

Example 54. Find work done by a force F = x î + xy ĵ acting on a particle to displace it from point O(0, 0) to C(2, 2).

2 2

Solution :  dW =  0
xdx +  xydy
0
can be found cannot be found
until x is known in
terms of y i.e. until
equation of path is known.

Example 55. Find the work done by F from O to C for above example if paths are given as below..

Solution : OAC  OA + AC
for OA y = 0  dy = 0
2

  dWOA =
 xdx +0  W OA = 2 J
0

KVPY_ SA STREAM # 144


for AC x=2 dx = 0
2
 dWAC = 0 + 2
 ydy
0
 W AC = 4J

 W OAC = W OA + W AC = 2 + 4 = 6J
(ii) OBC  OB + BC
for OB x = 0 dx = 0  W OB = O
for BC y = 2 dy = 0
2
x2
  dW =  xdx  W = 2  = 2 J
  0
 W OAC  W OBC
Hence the force is non-conservative.
(iii) For W OC dW = xdy + xydx
for OC x=y dx = dy
2 2 14
 xdx +  y dy
2
dW = W= unit
0 0 3


POTENTIAL ENERGY AND CONSERVATIVE FORCE :
FS = –  U/  s,
i.e. the projection of the field force, the vector F, at a given point in the direction of the displacement dr equals
the derivative of the potential energy U with respect to a given direction, taken with the opposite sign. The
designation of a partial derivative  /  s emphasizes the fact of deriving with respect to a definite direction.

So, having reversed the sign of the partial derivatives of the function U with respect to x, y, z, we obtain the
projection Fx, Fy and Fz of the vector F on the unit vectors i, j and k. Hence, one can readily find the vector
itself : F = Fxi + Fyj + Fzk, or
When conservative force does positive work then PE decreases
dU = – dw
dU = – F.ds
dU = – (Fx î + Fy ĵ + Fz k̂ ) . (dx î + dy ĵ + dz k̂ )
dU = – Fxdx – Fydy – Fzdz
if y & z are constants then dy = 0 dz = 0
dU = – Fxdx
dU
 Fx = – if y & z are constant
dx

U
 Fx =
x

u u
Similarly Fy = y ; Fz =
z

 U U U 
F = –  x i  y j  z k  .
 
The quantity in parentheses is referred to as the scalar gradient of the function U and is denoted by grad U
or U. We shall use the second, more convenient, designation where  (“nabla”) signifies the symbolic
vector or operator
  
= i  j k
x y z

KVPY_ SA STREAM # 145


Potential Energy curve :
 A graph plotted between the PE a particle and its
displacement from the centre of force field is called
PE curve.
 Using graph, we can predict the rate of motion of
a particle at various positions.
dU
 Force on the particle is F(x) = –
dx

Case : I On increasing x, if U increases, force is in (–) ve x direction i.e. attraction force.


Case : II On increasing x, if U decreases, force is in (+) ve x-direction i.e. repulsion force.

1 2
Example 56. The potential energy of spring is given by U = kx , where x is extension spring
2
Find the force associated with this potential energy.
 u
Solution : Fx = = – kx Fy = 0 Fz = 0.
x

Example 57. The potential energy of a particle in a space is given by U = x2 + y2.


Find the force associated with this potential energy.
 u
Solution : Fx = = – [2x + 0] = –2x
x

 u
Fy = = – (2y + 0) = –2y
y

F = – 2x î – 2y ĵ

Example 58. Find out the potential energy of given force F = – 2x î – 2y ĵ .
Solution : dU = –dW

 dU =   (2x î  2yĵ)  (dx î  dyĵ)


 dU =  2xdx   2ydy
 U = x2 + y2 + C

Example 59 Find out the potential energy of the force F = y î + x ĵ .


Solution : dU = – dW
dU = – (y î + x ĵ ) . (dx î + dy ĵ )

 dU =   ydx    xdy
 dU =  dxy  U = –xy + c

Example 60 Find out the force for which potential energy U = – xy.

  U U    ( xy) (  xy) 


î  ĵ  î  ĵ 
Solution : F = –  x y   F = –  x y 

F = y î + x ĵ Hence verifying the previous example.

KVPY_ SA STREAM # 146



EQ U I LI BRI U M O F A PART I CLE
Different positions of a particle :-
Position of equilibrium :
dU
If net force acting on a body is zero, it is said to be in equilibrium. For equilibrium = 0. Points P, Q & R
dx
are the states of equilibrium positions.
Types of equilibrium :
 Stable equilibrium :
When a particle is displaced slightly from a position and a force acting on it brings it back to the
initial position, it is said to be in stable equilibrium position.

dU d2U
Necessary conditions : – = 0, and = +ve
dx dx 2

 Unstable Equilibrium :
When a particle is displaced slightly from a position and force acting on it tries to displace the
particle further away from the equilibrium position, it is said to be in unstable equilibrium.
dU d2U
Condition : – = 0 potential energy is maximum i.e. = = – ve
dx dx 2
 Neutral equilibrium :
In the neutral equilibrium potential energy is constant. When a particle is displaced from its position
it does not experience any force acting on it and continues to be in equilibrium in the displaced
position. This is said to be neutral equilibrium.
A particle is in equilibrium if the acceleration of the particle is zero. As acceleration is frame dependent
quantity therefore equilibrium depends on motion of observer also.

a b
Example 61 The potential energy between two atoms in a molecule is given by, U(x) = 12 – , where a and b
x x6
are positive constants and x is the distance between the atoms. The system is in stable equilibrium
when -
1/ 6
a  2a   11a 
(A) x = 0 (B) x = (C) x =   (D) x =  
2b  b   5b 

a b
Solution : (C) Given that, U(x) = 12 –
x x6

du
We, know F=– = (–12) a x–13 – (–6b) x–7 = 0
dx

1/ 6
6b 12a  2a 
or = or x = 12a/6b = 2a/b
6
or x=  
x7 x13  b 

1/ 6
d2U  2a 
= +ve at x =  
dx 2  b 

KVPY_ SA STREAM # 147


u=0
Example 62. (A) (B) (C) (D)

E Moon
(E) (F)
Earth

of the cases above which is not a case of equilibrium.


Solution : (F) as moon is always accelerated. It has centripetal acceleration or it is changing its velocity all the
time.

Example 63.

Find out positions of equilibrium and determine whether they are stable, unstable or neutral.
Solution : Equilibrium is at B,D,F as force is zero here.
For checking type of equilibrium displace slightly.
We have B as stable equilibrium
D as unstable equilibrium and F as neutral equilibrium

Example 64. U

Identify the points of equilibrium and discuss their nature.


U
Solution : C,E,F are points of equilibrium because F =
x
 When slope of U - x curve is zero then F is zero.
Check stability through slopes at near by points.
If we move right then slope should be positive for stable equilibrium and vice versa. In short it is
like a hill and plateau.


MECHANICAL ENERGY :
Definition: Mechanical energy ‘E’ of an object or a system is defined as the sum of kinetic energy ‘K’ and
potential energy ‘U’, i.e., E = K + U

KVPY_ SA STREAM # 148


Important Points for M.E.:
1. It is a scalar quantity having dimensions [ML2T-2] and SI units joule.
2. It depends on frame of reference.
3. A body can have mechanical energy without having either kinetic energy or potential energy. However,
if both kinetic and potential energies are zero, mechanical energy will be zero. The converse may or
may not be true, i.e., if E = 0 either both PE and KE are zero or PE may be negative and KE may be
positive such that KE + PE = 0.
4. As mechanical energy E = K + U, i.e., E - U = K. Now as K is always positive, E - U  0,i.e., for
existence of a particle in the field, E  U.
5. As mechanical energy E = K + U and K is always positive, so, if ‘U’ is positive ‘E’ will be positive.
However, if potential energy U is negative, ‘E’ will be positive if K > |U| and E will be negative if K < |U|
i.e., mechanical energy of a body or system can be negative, and negative mechanical energy
means that potential energy is negative and in magnitude it is more than kinetic energy. Such a
state is called bound state, e.g., electron in an atom or a satellite moving around a planet are in
bound state.

Example 65 As shown in figure there is a spring block system. Block of mass 500 g is pressed against a
horizontal spring fixed at one end to compress the spring through 5.0 cm. The spring constant is
500 N/m. When released, the block moves horizontally till it leaves the spring. Calculate the distance
where it will hit the ground 4 m below the spring?

Solution : When block released, the block moves horizontally with speed V till it leaves the spring.

1 2 1
By energy conservation kx = mv 2
2 2

kx 2 kx 2
V2 =  V=
m m

2H
Time of flight t = g
So. horizontal distance travelled from the free end of the spring is V × t

kx 2 2H
= × g
m

500  (0.05 )2 2 4
= × =2m
0.5 10

So, At a horizontal distance of 2 m from the free end of the spring.

KVPY_ SA STREAM # 149


1. A body moves along a straight line and the variation of its kinetic energy with time is linear as depicted in the
figure below :

Then force acting on the body is :


(A) directly proportional to velocity (B) inversely proportional to velocity
(C) zero (D) constant

2. A particle of mass m moving along a straight line experiences force F


which varies with the distance x travelled as shown in the figure. If the
2 F0 x 0
velocity of the particle at x 0 is , then velocity at 4 x 0 is:
m
2 F0 x 0 F0 x 0
(A) 2 (B) 2
m m

F0 x 0
(C) (D) none of these
m

3. Graph shows the acceleration of a 3 kg particle as an applied force moves it from rest along x axis. The
total work done by the force on the particle by the time the particle reaches x = 6 m, is equal to

(A) 20 J (B) 30 J (C) 40 J (D) 60 J

4. A particle is displaced from x = +3 m to x = –3 m. Variation of force with position is given in graph. Work done
by this force during this above displacement is :

(A) –30 J (B) 90 J (C) 30 J (D) –90 J

5. When a conservative force does positive work, the potential energy of the system associated with that
force:
(A) decreases (B) increases (C) remains constant
(D) depends on whether other non conservative force is working or not.

KVPY_ SA STREAM # 150


6. Which of the following statement is not true?
(A) Work done by conservative force on an object depends only on the initial and final states and not on
the path taken.
(B) The change in the potential energy of a system corresponding to conservative internal forces is equal to
negative of the work done by these forces.
(C) If some of the internal forces within a system are non-conservative, then the mechanical energy of the
system is not constant.
(D) If the internal forces are conservative, the work done by the internal forces is equal to the change in
mechanical energy.

7. A particle is projected vertically upwards with a speed of 16 m/s, after some time, when it again passes
through the point of projection, its speed is found to be 8 m/s. It is known that the work done by air
resistance is same during upward and downward motion. Then the maximum height attained by the particle
is (Take g = 10 m/s2 ) :
(A) 8 m (B) 4.8 m (C) 17.6 m (D) 12.8 m

8. Two identical blocks A and B are placed on two inclined planes as shown in diagram. Neglect air
resistance and other friction
A L B
N

h h
Fixed Fixed
J K M 2 O
Read the following statements and choose the correct options.
Statements  : Kinetic energy of 'A' on sliding to J will be greater than the kinetic energy of B on falling
to M.
Statements  : Acceleration of 'A' will be greater than acceleration of 'B' when both are released to
slide on inclined plane
Statements  : Work done by external agent to move block slowly from position B to O is negative
(A) only statement  is true (B) only statement  is true
(C) only  and  are true (D) only  and  are true
A
9. A particle is projected along a horizontal field whose coefficient of friction varies as  = 2 where r is
r
the distance from the origin in meters and A is a positive constant. The initial distance of the particle is
1 m from the origin and its velocity is radially outwards. The minimum initial velocity at this point so
that particle never stops is :
(A)  (B) 2 gA (C) 2g A (D) 4 gA

10. Two blocks m1 = 5kg and m2 = 2kg are connected at the two ends of a spring of force constant k = 100 Nm-
1
. Friction coefficient between m1 and ground is 0.4, and between m2 and ground is 0.2. The minimum
horizontal velocity v that must be imparted to the m2 towards right in order to just move m1 over surface is :
(g = 10m/s2)
-1
k = 100 Nm
m1 m2
5kg 2kg

µ 1=.4 µ 2=.2
(A) 1.4 m / s (B) 1.8 m / s (C) 2.2 m/s (D) 2.8 m/s

11. A body has kinetic energy E when projected at an angle of projection for maximum range. Its K.E. at
the highest point of its path will be :
E E
(A) E (B) (C) (D) zero
2 2
12. Two objects are initially at rest on a frictionless surface. Object 1 has a greater mass than object 2.
The same constant force starts to act on each object. The force is removed from each object after it
accelerates over a distance d. After the force is removed from both objects, which statement is correct
(p : momentum; K: kinetic energy) ?
(A) p1 < p2 (B) p1 > p2 (C) K1 > K2 (D) K1 < K2

KVPY_ SA STREAM # 151


13. One end of a light spring of force constant k is fixed to the ceiling. The other end is fixed to a block of
mass m. Initially the spring is relaxed. The work done by an external agent to lower the hanging body
of mass m slowly till it comes to equilibrium is:
m 2 g2 m 2 g2 m 2 g2 m 2 g2
(A) 3 (B) (C)  3 (D) 
2k 2k 2k 2k
14. Two blocks each of mass m are joined together using an ideal spring of force constant K and natural length
0. The blocks are touching each other when the system is released from rest on a rough horizontal surface.
0
Both the blocks come to rest simultaneously when the extension in the spring is . The coefficient of
4
friction between each block and the surface assuming it to be same for both blocks & surface is :
K 0 K 0 3K 0 17 K 0
(A) (B) 8mg (C) 8mg (D) 20 mg
40mg
15. A bead is connected at one end of an inextensible massless string whose
other end is fixed to a fixed cylinder. Cylinder does not rotate. The bead can
move on a horizontal smooth surface. The bead is given a velocity v 0 perpen-
dicular to the string. The bead moves on a curve and consequently collides on
the cylinder after sometime. Then, before it collides :
(A) Work done by string on the bead is positive (B) The average speed of the bead is v 0.
(C) The tension in the string remains constant. (D) Kinetic energy of bead increases gradually.
16. The potential energy of a 0.64 kg particle moving along the x-axis is given by U(x) = 8x 2, where, U is in
joules and 'x' is in meters. When the particle is at x = 1.0 m, it is traveling in the positive x-direction
with a speed 5.0 m/s. It next stops momentarily to turn around at 'x', where 'x'.
1
(A) is equal to 2 (B) is equal to 2 (C) is equal to (D) None of these
2
17. A block is suspended by a spring initially in natural length o.
Take intial position of block as zero reference level for /////////////////////////
gravitational potential energy of block. The block is released
from rest at its initial position. Now as the block comes down 0

till the maximum downward displacement of the block for the


first time, the sum of the elastic potential energy of the spring Initial position zero gravitational
of block P.E. level
and the gravitational potential energy of the block: x
(A) increases
(B) decreases
(C) can either increase or decrease depending on the spring constant
(D) decreases first then increases
18. The blocks A and B shown in the figure have masses MA = 5 kg
and MB = 4 kg. The system is released from rest. The speed of B
3 after A has travelled a distance 1 m along the incline is
3 3
(A) g (B) g
2 4
g g
(C) (D)
2 3 2
t2
19. A body of mass 6 kg is acted upon by a force which causes a displacement in it given by x = metre where
4
t is the time in second. The work done by the force is 2 seconds is:
(a) 12 J (b) 9 J (c) 6 J (d) 3 J
20. Find velocity of A in the figure after it has started from rest at t = 0
in the position shown and travelled 9m along the frictionless surface.
MA & MB are 300 kg & 200 Kg respectively. (Take g = 10 m/s2)
7000
(A) m/s (B) 56 m/s
93

3000
(C) 72 m/s (D) m/s
31
KVPY_ SA STREAM # 152
21. The block is released from rest on rough inclined as shown. Maximum compression in spring is [g = 10 m/s2] :

(A) 1.2 m (B) 0.2 m (C) 2.2 m (D) 5.2 m


22. A block of mass 5 kg is released from rest when compression in
spring is 2m. Block is not attached with the spring and natural
length of the spring is 4m. Maximum height of block from ground
is : (g = 10 m/s2)
(A) 5.5 m (B) 4.5 m
(C) 6 m (D) 7.5 m

23. Kinetic energy of a particle moving in a straight line varies with time as K = 4t 2. The force acting on the
particle.
(A) is constant (B) is increasing
(C) is decreasing (D) first increases and then decreases

24. A small block is attached to a vertical unstretched spring and is released from rest . The other end of the
spring is fixed. The ratio of maximum potential energy in the spring to that of total energy of oscillation :
(A) 1 : 1 (B) 2 : 1 (C) 1 : 2 (D) 4 : 1

25. A horse drinks water from a cubical container of side 1 m. The level of
the stomach of horse is at 2 m from the ground. Assume that all the
water drunk by the horse is at a level of 2 m from the ground. Then
minimum work done by the horse in drinking the entire water of the
container is (Take water = 1000 kg/m3 and g = 10 m/s2 ) :
(A) 10 kJ (B) 15 kJ (C) 20 kJ (D) zero
26. One end of an unstretched vertical spring is attached to the ceiling and an object attached to the other
end is slowly lowered to its equilibrium position. If S be gain in spring energy & G be loss in gravitational
potential energy in the process, then
(A) S = G (B) S = 2G (C) G = 2S (D) None of these
27. A simple pendulum of length L is suspended from the top of a flat beam
of thickness L/2. The bob is pulled away from the beam so that it
L/2
makes an angle  with the vertical as shown in the figure. It is then L
released from rest. If  is the maximum angular deflection to the right,
then
(A) 2cos=1+ 2cos  (B) 2cos=1– cos 
(C) 2cos=1+ cos  (D) none of these
28. The force acting on the system is represented against distance r as shown.
The potential energy of the system will be best represented by. (At x = 0,
potential energy = 0)

(A) (B)

(C) (D)

KVPY_ SA STREAM # 153


29. In the figure shown the potential energy U of a particle is plotted against its position ' x ' from origin. Then
which of the following statement is correct. A particle at:

(A) x1 is in stable equilibrium (B) x2 is in stable equilibrium


(C) x3 is in stable equilibrium (D) none of these
30. A block of mass m slides along the horizontal track with kinetic friction
. A man pulls the block through a light rope which makes an angle 
with the horizontal as shown in the figure . The tension in the light rope
is T as shown. The block moves with constant speed V. Power delivered
by the man to the block is:
(A) TV (B) TV cos 
(C) (T cos  mg) V (D) zero
31. An engine pumps up 1000 kg of coal from a mine 100 m deep in 0.5 sec. The pump is running with diesel and
efficiency of diesel engine is 25%. Then its power consumption will be (g = 10m/sec2):
(A) 200 k W (B) 8000 kW (C) 1000 kW (D) 500 kW
32. A self propelled vehicle of mass m whose engine delivers constant power P has an acceleration
a = P/mv (assume that there is no friction). In order to increase its velocity from v 1 to v 2 the distance it has to
travel will be:
3P m m m
(A) (v 22 – v12) (B) 3P (v23 - v13) (C) 3P (v22 – v12) (D) 3P (v2 – v1)
m
33. Water is flowing in a river at rate 2.0 m/sec. The river is 50m wide and has an average depth 5.0m. The power
from the river flow current is (density of water = 1000 kg/m3) : (Assume no velocity gradient with depth)
(A) 0.5 MW (B) 1.0 MW (C) 1.5 MW (D) 2.0 MW

34.* The kinetic energy ‘K’ of a particle moving along a circle of radius r depends on distance covered as K = As2
where A is a constant. Find force acting on the particle as a function of s.
(A) As (B) 2As (C) 3As (D) 4As

35. The work done by the frictional force on a surface in drawing a circle of radius r on the surface by a
pencil of negligible mass with a normal pressing force N (coefficient of friction µ k) is :
(A) 4r2 K N (B) –2r2 K N (C) –2r K N (D) zero

KVPY PROBLEMS (PREVIOUS YEARS)


1. There is a smooth fixed concave surface. A particle is released from p. Find :
[KVPY-2010]

(i) PE as function of  (ii) KE as a function 


(iii) time taken from P to Q (iv) the reaction force at 

2. A 750 W motor drives a pump which lifts 300 litres of water per minute to a height of 6 meters. The efficiency
of the motor is nearly (take acceleration due to gravity to be 10 m/s2) [KVPY_2011]
(A) 30% (B) 40% (C) 50% (D) 20%

KVPY_ SA STREAM # 154


3. A body of 0.5 kg moves along the positive x - axis under the influence of a varying force F (in Newtons) as
shown below : [KVPY_2011]

If the speed of the object at x = 4m is 3.16 ms–1 then its speed at x = 8 m is :


(A) 3.16 ms–1 (B) 9.3 ms–1 (C) 8 ms–1 (D) 6.8 ms–1

4. A block of mass m is sliding down an inclined plane with constant speed. At a certain instant t0, its height
above the ground is h. The coefficient of kinetic friction between the block and the the plane is . If the block
reaches the ground at a later instant tg , then the energy dissipated by friction in the time interval (tg – t0) is:
[KVPY_2012]

(A) mgh (B) mgh (C) mgh/sin (D) mgh/cos

5. A particle with total mechanical energy, which is small and negative, is under the influence of a one dimen-
sional potential U(x) = x4/4 – x2/2 J where x is in meters. At time t = 0 s, it is at x = – 0.5 m. Then at a later
time it can be found. [KVPY_2016_SB] [1 Marks]
(A) anywhere on the x axis (B) between x = – 1.0 m to x = 1.0 m
(C) between x = – 1.0 m to x = 0.0 m (D) between x = 0.0 m to x = 1.0 m.

EXERCISE - 1
1. (B) 2. (D) 3. (D) 4. (C) 5. (A) 6. (D) 7. (A)

8. (D) 9. (C) 10. (D) 11. (C) 12. (B) 13. (D) 14. (C)

15. (B) 16. (B) 17. (D) 18. (C) 19. (D) 20. (A) 21. (B)

22. (A) 23. (A) 24. (D) 25. (B) 26. (C) 27. (C) 28. (A)

29. (D) 30. (B) 31. (B) 32. (B) 33. (B) 34.* (B) 35. (D)

EXERCISE - 2

2. (B) 3. (D) 4. 5. (C)

KVPY_ SA STREAM # 155


1
1. mv 2 = ct
2
m dv dv c
2v =c  m =
2 dt dt v
1
F
v

2. Increase in KE = work done

1 1  2Fo x o  1 11F0 x0
m v 22 – mx   = (2F0 + F0) 3x0  v2 = .
2 2  m  2 m

3. Acceleration as shown in the graph can be converted into force by multiplying with m = 3 kg. Therefore
area under the curve (F – x curve) is
1
[ × 2 × 12] + [4 × 12] = 60 J.
2

4.

W = W 1 + W 2 = 10(0 – 3) + (–20) (–3 – 0) = –30 + 60 = +30J.

7. From work energy theorem


for upward motion
1
m (16)2 = mgh + W (work by air resistance)
2
for downward motion
1
m (8)2 = mgh – W
2
1
[(16)2 + (8)2] = 2 gh or h=8m
2

8. Statement  : Work done by gravity is same for motion from A to J and B to M for equal mass. So K.E.
will be equal.
Statement  : Acceleration = g sin 
sin A > sin B
h h

 2
Statement III :
W g + W ext = 0 (Because moved slowly)
W ext = – W g
from B to O : W g is positive so W ext < 0

9. Work done against friction must equal the initial kinetic energy.
 
1 v2 1 v2


2
mv 2 = 
1
 mg dx ;
2

= A g 2 dx
1
x
;
2
 1
= Ag  
 x 1

v 2 = 2gA  v= 2g A

KVPY_ SA STREAM # 156


10. At the moment m2 stops, extension in the spring must be able to produce enough force to move m1 or
.4  5  10
kx = µ1m1g  x = = 20cm.
100
As it is equal to displacement of m2 also, applying work-energy theorem on m2
1 1
0– mv 2 = W s + W f = – kx2 – µ2m2g. x
2 2
1 1
 m2v 2 = × 100 × 0.2 × 0.2 + 0.2 × 2 × 10 × 0.2
2 2
v 2 = 2 + 0.8  v = 2.8 m / s

12. From work energy theorem


Fd = Kf – Ki = Kf
So, K1 = K2
p12 p2
 2
2 m1 2 m 2
 m1 > m2
So, p1 > p2

13. Initial velocity = 0, final velocity = 0 (for slowly motion)


mg
for equilibrium mg = kx  x=
k
wF + wg + wsp = 0 – 0
1 2
wF + mgx – kx = 0
2

 m 2 g2
wF = .
2k

14.

Work energy theorem,


2
   1   1
 mg  0  0  = K  0   K 0 2
 4  2  4  2

3K 0
 = 8mg Ans.

15. Work done by all the forces acting on the bead zero. This means speed will remain constant.

16. KE + PE = constant
1
× (0.64) × 52 + (8 × 1.02 ) = 0 + (8x 2 )
2
 x= 2

KVPY_ SA STREAM # 157


17. /////////////////////////

0

zero gravitational potential U


x energy level

1 mg
Potential energy U = kx2 – mgx 0 x=
K
2 x
2mg
K
From graph between U versus elongation x,
1
18. If A moves down the incline by 1 metre, B shall move up by
2
metre. If the speed of B is v then the speed of A is 2v.
From conservation of energy:
Gain in K.E. = loss in P.E.
1 1 3 1
m A (2v)2 + mB v 2 = mA g × – mB g ×
2 2 5 2
Solving we get
1 g
v= Ans.
2 3

19. The velocity of the body a time t is given by


dx d  t 2  t
 V 
dt dt  4  2
 At t = 0 , V = u = 0 and t = 2 s, V = 1ms-1, Now, work done = increase in KE
1 1 1 1 1
 mV 2  mu 2  mV 2  0  mV 2   6  (1)2 = 3J, Hence the correct choice is (d).
2 2 2 2 2
20. 17 – 10 = 7m downwards
1 1
mgh = 200 × 10 × 7 = × 300 × v A2 + × 200 × v B2
2 2
6
cos =
10
2
1 1 3
v A cos = vB 200 × 10 × 7 = × 300 × v A2 + × 200 × v A2   .
2 2 5

21. Let maximum compression is x

W net = k.e
mg(4.8 + x)sin37º – mg(4.8 + x)cos37º
1 2
 kx = 0  x = 0.2 m
2
22. By energy conservation,
1 2 1
kx = mgh + mv 2
2 2
1 1
= × 300 × (2)2 = 5 × 10 × 2 + × 5v 2  v 2 = 200
2 2
v 2 sin2 30º
Also, H= = 2.5 m
2g
So, total height from ground = 3 + 2.5 = 5.5 m.

KVPY_ SA STREAM # 158


1 1 dv
23. mv 2  4 . t 2  m 2v = 4 × 2t
2 2 dt

dv 8t 8t
 m. = =  8m
dt v 8t 2
m

1
24. Maximum spring potential energy = K (2A)2
2

1
K ( 2A ) 2
1 2
Total energy of oscillation = KA2  required ratio = =4:1
2 1
KA 2
2

25. The mass of water is

m = 1 × 103 kg h=1.5m

 The increase in potential energy of water is


= mgh = (1 × 103) (10) 1.5 = 15 kJ

mg
26. At equilibrium positon x =
k

1 2 1  mg  mgx 1
Uspring = kx  k   .x  = (loss in G.P.E.)
2 2  k  2 2

 G = 2S

27. From Conservation of energy mgL cos= mg (L/2) + mg (L/2) cos . Solving we get 2cos=1+ cos 

du
28. By F =  . Graph A is correct.
dr

du
29. x = x 1 and x = x 3 are not equilibrium positions because  0 at these points.
dx
x = x 2 is unstable, as U is maximum at this point.

30. (Easy) The power delivered = F.v = TV cos

KVPY_ SA STREAM # 159


d
31. P= (mgh)
dt

1000  10  100 2000


Pact = Pact = 2000 kW Pconsumption = kW = 8000 kW.
0.5 0.25

P
32. acceleration a=
mv

dv P
v =
dx mv

s v2
m
  v dv
2
dx 
P
0 v1

m 3
S= ( v 2  v13 ) .
3P
33. Volume flow rate = AV
Mass flow rate = AV

1
Per second K.E. flow = Power = (AV)V2
2

1
= AV
V3
2

1
= (1000) (50 × 5) (2)3
2
= 1 MW.

34.* K = As2
dk d dS
P= = (As2) = 2As .
dt dt dt
P = 2AV . S
Now, P = F.V = 2AV.s
F = 2As

KVPY_ SA STREAM # 160


CEN T ER OF M ASS

CEN TER OF M ASS
Every physical system has associated with it a certain point whose motion characterises the motion of
the whole system. When the system moves under some external forces, then this point moves as if the
entire mass of the system is concentrated at this point and also the external force is applied at this
point for translational motion. This point is called the center of mass of the system.

CENTER OF M ASS OF A SYSTEM OF 'N' DISCRETE PARTICLES


Consider a system of N point masses m1, m2, m3, ................ m n whose
  
position vectors from origin O are given by r1 , r2 , r3 ,...............

rn respectively. Then the position vector of the center of mass C of the
n
system is given by. 
  
m1r1  m 2 r2  ........  m n rn
m r
i 1
i i
 
rcm = m1  m 2  ........  mn
; rcm = n

m
i 1
i
n


rcm =
1
M
m r
i 1
i i


where, mi ri is called the moment of mass of the particle w.r.t O.

 n 

M = m i

 is the total mass of the system.
 i1 

n

Note: If the origin is taken at the center of mass then  m r =0. hence, the COM is the point about which
i 1
i i

the sum of “mass moments” of the system is zero.

P OSI TI ON OF COM OF TW O P AR TI CLES


Center of mass of two particles of masses m 1 and m 2 separated by a distance r lies in between the
two particles. The distance of center of mass from any of the particle (r) is inversely proportional
to the mass of the particle (m)
i.e. r  1/m
r1 m2
or =
r2 m1
or m 1r 1 = m 2r 2

 m2   m1 
or r 1 =   r and r = 
 m m
r

 m2  m1 
2
 1 2 
Here, r 1 = distance of COM from m 1
and r 2 = distance of COM from m 2
From the above discussion, we see that
r 1 = r 2 = 1/2 if m 1 = m 2, i.e., COM lies midway between the two particles of equal masses.
Similarly, r 1 > r 2 if m 1 < m 2 and r 1 < r 2 if m 2 < m 1, i.e., COM is nearer to the particle having larger
mass.

KVPY_ SA STREAM # 161


Example 1. Two particles of mass 1 kg and 2 kg are located at x = 0 and x = 3 m. Find the position of their
center of mass.
Solution :
Since, both the particles lies on x-axis, the COM will also lie on x-axis. Let the COM is
located at x = x, then
r 1 = distance of COM from the particle of mass 1 kg = x

and r 2 = distance of COM from the particle of mass 2 kg = (3 – x)


r1 m2 x 2
Using r2
= m1
or = or x = 2 m
3x 1
Thus, the COM of the two particles is located at x = 2 m. Ans.
Example 2. The position vector of three particles of masses m 1 = 1 kg, m 2 = 2 kg and m 3 = 3 kg are
  
r1  ( î  4 ĵ  k̂ ) m , r2  ( î  ĵ  k̂ ) m and r3  ( 2 î  ĵ  2k̂ ) m respectively. Find the position vector
of their center of mass.
Solution :
The position vector of COM of the three particles will be given by
  
 m1r1  m 2 r2  m 3 r3
rCOM 
m1  m 2  m 3
Substituting the values, we get

 (1)( î  4 ĵ  k̂ )  (2)( î  ĵ  k̂ )  (3)(2î – ĵ – 2k̂ ) 1


rCOM   ( 3 î  ˆj  k̂ ) m Ans.
1 2  3 2


CENTER OF M ASS OF A CONTINUOUS M ASS DISTRIBUTION
For continuous mass distribution the center of mass can be located by replacing summation sign with an
integral sign. Proper limits for the integral are chosen according to the situation

x cm =
 x dm , y =
 y dm , z =
 z dm
 dm  dm  dm
cm cm

 dm = M (mass of the body)

 1 
rcm =
M  rdm .
Note: If an object has symmetric mass distribution about x axis then y coordinate of COM is zero and
vice-versa

CEN TER OF M ASS OF A UN IFOR M R OD


Suppose a rod of mass M and length L is lying along the x-axis with its one end at x = 0 and the
M
other at x = L. Mass per unit length of the rod =
L

KVPY_ SA STREAM # 162


M
Hence, dm, (the mass of the element dx situated at x = x is) = dx
L
The coordinates of the element dx are (x, 0, 0). Therefore, x-coordinate of COM of the rod will be
L
0
x dm dx
x COM =
 dm x=0 x=x x=L

L M 
 ( x )  dx 
L  = 1
L L

0
= x dx 
M L 0 2
The y-coordinate of COM is

y COM =
 y dm = 0
 dm
Similarly, z COM = 0

L 
i.e., the coordinates of COM of the rod are  , 0, 0  , i.e. it lies at the center of the rod.
 2 

Example 3. A rod of length L is placed along the x-axis between x = 0 and x = L. The linear density (mass/length)
 of the rod varies with the distance x from the origin as  = Rx. Here, R is a positive constant. Find
the position of center of mass of this rod.
Solution :
Mass of element dx situated at x = x is y
dm =  dx = Rx dx
The COM of the element has coordinates (x, 0, 0).
Therefore, x-coordinate of COM of the rod will be dx
L x=0 x=x x=L x
0
x dm
x COM =
 dm
L L
 x3 

L 2
 0
( x)(Rx) dx R x dx
0
 
 3  0

2L
= L = = L
L 3
 x 
2
(Rx) dx
0 
R x dx
0
 
 2  0

 y dm
The y-coordinate of COM of the rod is y COM = =0 (as y = 0)
 dm
Similarly, z COM = 0

 2L 
Hence, the center of mass of the rod lies at  ,0, 0 Ans.
3 

KVPY_ SA STREAM # 163



CEN TER OF M ASS OF A SEM I CI R CULAR R I N G
Figure shows the object (semi circular ring). By observation we can say that the x-coordinate of
the center of mass of the ring is zero as the half ring is symmetrical about y-axis on both sides of
the origin. Only we are required to find the y-coordinate of the center of mass.
Y

Rd

y=Rsin
y cm d
 X

To find y cm we use y cm =
1
M
 dm y ...(i)

Here for dm we consider an elemental arc of the ring at an angle  from the x-direction of angular
width d. If radius of the ring is R then its y coordinate will be R sin, here dm is giv en as
M
dm = × R d
R
So from equation ---(i), we have


R
y cm =
1
M 
M
R
Rd (R sin) =

 sin  d
0
0

2R
y cm = ...(ii)

CEN TER OF M AS S OF SEM I CI R CULAR D I SC


Figure shows the half disc of mass M and radius R. Here, we are only required to find the y-
coordinate of the center of mass of this disc as center of mass will be located on its half vertical
diameter. Here to find y cm, we consider a small elemental ring of mass dm of radius x on the disc
(disc can be considered to be made up such thin rings of increasing radii) which will be integrated
from 0 to R. Here dm is given as
2M
dm = ( x)dx
R 2
Y
ycm

dx

x
X
R

2x
Now the y-coordinate of the element is taken as , as in previous section, we have derived that

2R
the center of mass of a semi circular ring is concentrated at

R R
4M
Here y cm is given as y cm =
1
M 
0
dm
2x

=
1
M
 R
0
2
x 2 dx

4R
y cm =
3

KVPY_ SA STREAM # 164


CEN TER OF M ASS OF A H OLLOW H EM I SP H ER E
A hollow hemisphere of mass M and radius R. Now we consider an elemental circular strip of
angular width d at an angular distance  from the base of the hemisphere. This strip will have an
area.
dS = 2R cos  Rd

Its mass dm is given as


M
dm = 2R cos  Rd
2R 2
Here y-coordinate of this strip of mass dm can be taken as R sin. Now we can obtain the center
of mass of the system as.
  
2 2 2
1 1  M  R
y cm =
M  dm R sin 
0
=
M   2R
0
2
 
2 R 2 cos  d  R sin = R sin  cos  d  y cm =
0
2

Example 4. Find out the center of mass of an isosceles triangle of base length a and altitude b. Assume that the
mass of the triangle is uniformly distributed over its area.
Solution :
To locate the center of mass of the triangle, we take a strip of width dx at a distance x from the vertex
of the triangle. Length of this strip can be evaluated by similar triangles as
 = x. (a/b)
2M
Mass of the strip is dm =  dx
ab
Distance of center of mass from the vertex of the triangle is
1
x CM =
M  x dm

b
2x 2
= b
0
2
dx

2
= b
3
Proceeding in the similar manner, we can find the COM of certain rigid bodies. Center of
mass of some well known rigid bodies are given below :
1. Center of mass of a uniform rectangular, square or circular plate lies at its center. Axis of symme-
try plane of symmetry.

KVPY_ SA STREAM # 165


2. For a laminar type (2-dimensional) body with uniform negligible thickness the formulae for finding
the position of center of mass are as follows :
   
 m r  m 2 r2  .... A 1t r1  A 2 t r2  ....
rCOM  1 1  (  m = At)
m1  m 2  .... A 1t  A 2 t  ....
 
 A 1r1  A 2 r2  ....
or rCOM  Here, A stands for the area,
A1  A 2  ....

3. If some mass of area is removed from a rigid body, then the position of center of mass of the
remaining portion is obtained from the following formulae:
   
 m1r1  m 2 r2  A1r1  A 2 r2
(i) rCOM  or rCOM  A  A
m1  m 2 1 2

m1x1  m 2 x 2 A1x1  A 2 x 2
(ii) x COM  or x COM  A  A
m1  m 2 1 2

m1y1  m 2 y 2 A y  A 2y2
y COM  or y COM  1 1
m1  m 2 A1  A 2

m1z1  m 2 z 2 A z  A 2z2
and z COM  or z COM  1 1
m1  m 2 A1  A 2
  
Here, m1, A1, r1 , x1, y1 and z1 are the values for the whole mass while m2, A2, r2 , x 2 , y2 and z2 are the values
for the mass which has been removed. Let us see two examples in support of the above theory.
Example 5. Find the position of center of mass of the uniform lamina shown in figure.
Y

a X
O

Solution : Here,
A 1 = area of complete circle = a 2
2
a a 2
A 2 = area of small circle =    =
2 4
(x 1, y 1) = coordinates of center of mass of large circle = (0, 0)

a 
and (x 2, y 2) = coordinates of center of mass of small circle =  , 0 
2 

A1x1  A 2 x 2
Using x COM =
A1  A 2

a 2  a   1
    
4  2 8 a
we get x COM = 2
= a= –
 a  3  6
a 2   
4 4

and y COM = 0 as y 1 and y 2 both are zero.


 a 
Therefore, coordinates of COM of the lamina shown in figure are   , 0  Ans.
 6 

KVPY_ SA STREAM # 166



CEN TER OF M ASS OF SOM E COM M ON SYSTEM S

 A system of two point masses m1 r1 = m2 r2


The center of mass lies closer to the heavier mass.

 Rectangular plate (By symmetry)

b L
xc = yc =
2 2

 A triangular plate (By qualitative argument)

h
at the centroid : yc =
3

 A semi-circular ring

2R
yc = xc = 0

 A semi-circular disc

4R
yc = xc = 0
3

 A hemispherical shell

R
yc = xc = 0
2

 A solid hemisphere

3R
yc = xc = 0
8

 A circular cone (solid)

h
yc =
4

KVPY_ SA STREAM # 167


 A circular cone (hollow)

h
yc =
3

M OTI ON OF CEN TER OF M ASS AN D CON SER VATI ON OF M OM EN TUM :


Velocity of center of m a ss of system

dr1 dr dr dr
m1  m 2 2  m 3 3 ..............  m n n    
 dt dt dt dt m1 v 1  m 2 v 2  m 3 v 3 ..........  mn v n
v cm = =
M M
Here numerator of the right hand side term is the total momentum of the system i.e., summation
of momentum of the individual component (particle) of the system
Hence velocity of center of mass of the system is the ratio of momentum of the system to the mass of the
system.
 
 PSyste= M v cm

Acce le ra t ion of ce n t e r of m a ss of syst e m

dv dv dv dv    
 m1 1  m 2 2  m 3 3 ..............  m n n m1a1  m 2a 2  m 3 a 3 ..........  mn an
a cm = dt dt dt dt =
M
M

Net force on system Net External Force  Net int ernal Force Net External Force
= = =
M M M
(action and reaction both of an internal force must be within the system. Vector summation will
cancel all internal forces and hence net internal force on system is zero)
 
 Fext = M a cm

where Fext is the sum of the 'external' forces acting on the system. The internal forces which the
particles exert on one another play absolutely no role in the motion of the center of mass.
If no external force is acting on a system of particles, the acceleration of center of mass of the
system will be zero. If a c = 0, it implies that v c must be a constant and if v cm is a constant, it
implies that the total momentum of the system must remain constant. It leads to the principal of
conservation of momentum in absence of external forces.
 
If Fext  0 then v cm = constant
“If resultant external force is zero on the system, then the net momentum of the system
must remain constant”.
M ot ion of COM in a m ovin g syst e m of p a rt icle s:
(1) COM at rest :
If F ext = 0 and V cm = 0, then COM remains at rest. Individual components of the system may
move and have non-zero momentum due to mutual forces (internal), but the net momentum of
the system remains zero.
(i) All the particles of the system are at rest.
(ii) Particles are moving such that their net momentum is zero.
example:

KVPY_ SA STREAM # 168


(iii) A bomb at rest suddenly explodes into various smaller fragments, all moving in different directions
then, since the explosive forces are internal & there is no external force on the system for explo-
sion therefore, the COM of the bomb will remain at the original position and the fragment fly such
that their net momentum remains zero.
(iv) Two men standing on a frictionless platform, push each other, then also their net momentum
remains zero because the push forces are internal for the two men system.
(v) A boat floating in a lake, also has net momentum zero if the people on it changes their position,
because the friction force required to move the people is internal of the boat system.
(vi) Objects initially at rest, if moving under mutual forces (electrostatic or gravitation)also have net
momentum zero.
(vii) A light spring of spring constant k kept compressed between two blocks of masses m 1 and
m 2 on a smooth horizontal surface. W hen released, the blocks acquire velocities in oppo-
site directions, such that the net momentum is zero.

(viii) In a fan, all particles are moving but COM is at rest

(2) COM moving with uniform velocity :


If F ext = 0, then V cm remains constant theref ore, net momentum of the system also remains
conserved. Individual components of the system may have variable velocity and momentum due to
mutual forces (internal), but the net momentum of the system remains constant and COM contin-
ues to move with the initial velocity.
(i) All the particles of the system are moving with same velocity.
e.g.: A car moving with uniform speed on a straight road, has its COM moving with a
constant velocity.

(ii) Internal explosions / breaking does not change the motion of COM and net momentum
remains conserv ed. A bomb moving in a straight line suddenly explodes into various smaller
fragments, all moving in different directions then, since the explosive forces are internal & there is
no external force on the system for explosion therefore, the COM of the bomb will continue the
original motion and the fragment fly such that their net momentum remains conserved.
(iii) Man jumping from cart or buggy also exert internal forces therefore net momentum of the
system and hence, Motion of COM remains conserved.
(iv) Two moving blocks connected by a light spring on a smooth horizontal surface. If the
acting forces is only due to spring then COM will remain in its motion and momentum will
remain conserved.
(v) Particles colliding in absence of external impulsive forces also have their momentum con-
served.

(3) COM moving with acceleration :


If an external force is present then COM continues its original
motion as if the external f orce is acting on it, irrespective of
internal forces.
Example:
Projectile motion : An axe thrown in air at an angle  with the
h o r i z o n t a l wi l l p e r f o r m a c o m p l i c a t e d m o t i o n o f
rotation as well as parabolic motion under the ef f ect of
gravitation

u 2 sin 2  u 2 sin 2 2u sin 


H com = R com = T= g
2g g

KVPY_ SA STREAM # 169


Example: Rcom
Circular Motion : A rod hinged at an end, rotates, than its
Fc
COM performs circular motion. The centripetal force (F c)
required in the circular motion is assumed to be acting on
 Fc
mg
Fc Fc
the COM.
mg
Fc  mω 2R COM mg
mg

Example 6. A projectile is fired at a speed of 100 m/s at an angle of 37º above the horizontal. At the highest point,
the projectile breaks into two parts of mass ratio 1 : 3, the lighter piece coming to rest. Find the
distance from the launching point to the point where the heavier piece lands.
Solution :
Internal force do not eff ect the motion of the center of mass, the center of mass hits the
ground at the position where the original projectile would have landed. The range of the
original projectile is,

3 4
2  10 4  
2u 2 sin  cos  5 5
x COM = = m
g 10

= 960 m
The center of mass will hit the ground at this position. As the smaller block comes to rest
after breaking, it falls down vertically and hits the ground at half of the range, i.e., at x =
480 m. If the heavier block hits the ground at x 2, then
m1x1  m 2 x 2
x COM = m1  m 2

(m)(480 )  (3m)( x 2 )
960 =
(m  3m)
x 2 = 1120 m Ans.

M om e ntum Conse rva tion :


The total linear momentum of a system of particles is equal to the product of the total mass of the
 
system and the velocity of its center of mass. P = M v cm


 dP
Fext =
dt

 dP 
If Fext = 0  =0 ; P = constant
dt
When the vector sum of the external forces acting on a system is zero, the total linear momentum of
the system remains constant.
   
P + P + P + ...............+ P = constant.
1 2 3 n

Example 7. A shell is fired from a cannon with a speed of 100 m/s at an angle 60º with the horizontal (positive x-
direction). At the highest point of its trajectory, the shell explodes into two equal fragments. One of
the fragments moves along the negative x-direction with a speed of 50 m/s. What is the speed of the
other fragment at the time of explosion.

KVPY_ SA STREAM # 170


Solution : As we know in absence of external force the motion of center of mass of a body remains
unaffected. Thus, here the center of mass of the two fragments will continue to follow the
original projectile path. The velocity of the shell at the highest point of trajectory is
v M = ucos = 100 ×cos60º = 50 m/s.
Let v 1 be the speed of the fragment which moves along the negative x-direction and the
other fragment has speed v 2,. which must be along positive x-direction. Now from momen-
tum conservation, we have
m m
mv = v1 + v or 2v = v 2 – v 1
2 2 2
or v 2 = 2v + v 1 = (2 × 50) + 50 = 150 m/s

Example 8. A man of mass m is standing on a platform of mass M kept on smooth ice. If the man starts moving
on the platform with a speed v relative to the platform, with what velocity relative to the ice does the
platform recoil ?
Solution :
Consider the situation shown in figure. Suppose the man moves at a speed w towards right and
the platform recoils at a speed V towards left, both relative to the ice. Hence, the speed of the
man relative to the platform is V + w. By the question,
V + w = v, or w = v – V .............(i) w
Taking the platform and the man to be the system, there is no V
external horizontal force on the system. The linear momentum ice
of the system remains constant. Initially both the man and the /////////////////////////////////////////////////
platform were at rest. Thus,
0 = MV - mw or, MV = m (v – V) [Using (i)]
mv
or, V= .
M m

Example 9. A flat car of mass M is at rest on a frictionless floor with a child of mass m standing at its edge. If
child jumps off from the car towards right with an initial velocity u, with respect to the car, find the
velocity of the car after its jump.
Solution :
Let car attains a velocity v, and the net velocity of the child with respect to earth will be
u – v, as u is its velocity with respect to car.

m m
u
v
M M

/////////////////////////////////////////////////////////////////// ///////////////////////////////////////////////////////////

Initially, the system was at rest, thus according to momentum conservation, momentum
after jump must be zero, as
m (u – v) = M v
mu
v=
mM

Example 10. Each of the blocks shown in figure has mass 1 kg. The rear block moves with a speed of
2 m/s towards the front block kept at rest. The spring attached to the front block is light and
has a spring constant 50 N/m. Find the maximum compression of the spring. Assume, on a
friction less surface

k=50N/m
1kg 1kg
/////////////////////////////////////////////////////////////////

KVPY_ SA STREAM # 171


Solution : Maximum compression will take place when the blocks move with equal velocity. As no net
external horizontal force acts on the system of the two blocks, the total linear momentum will
remain constant. If V is the common speed at maximum compression, we have,
(1 kg) (2 m/s) = (1 kg)V + (1 kg)V
or, V = 1 m/s.
1
Initial kinetic energy = (1 kg) (2 m/s)2 = 2 J.
2
Final kinetic energy
1 1
(1 kg) (1m/s) 2 +
= (1 kg) (1 m/s) 2 = 1 J
2 2
The kinetic energy lost is stored as the elastic energy in the spring.
1
Hence, (50 N/m) x 2 = 2J – 1J = 1 J or, x = 0.2 m.
2

Example 11. A light spring of spring constant k is kept compressed between two blocks of masses m and M
on a smooth horizontal surface. When released, the blocks acquire velocities in opposite di-
rections.
The spring loses contact with the blocks when it acquires natural length. If the spring was
initially compressed through a distance x, find the final speeds of the two blocks.
Solution : Consider the two blocks plus the spring to be the system. No external force acts on this
system in horizontal direction. Hence, the linear momentum will remain constant. Sup-
pose, the block of mass M moves with a speed v 1 and the other block with a speed v after
losing contact with the spring. From conservation of linear momentum in horizontal direc-
tion we have
m
Mv 1 – mv 2 = 0 or v 1 = v , ....(i)
M 2

1 2
Initially, the energy of the system = kx
2

1 1
Finally, the energy of the system = mv 22 + Mv 12
2 2
As there is no friction, mechanical energy will remain conserved.
1 1 1
Therefore, mv 22 + Mv 12 = kx 2 ...(ii)
2 2 2
Solving Eqs. (i) and (ii), we get
1/ 2 1/ 2
 kM   km 
or, v2 =   x and v1 =   x Ans.
 m(M  m)   M(M  m) 


I M P ULS E

Impulse of a force F acting on a body for the time interval t = t 1 to t = t 2is defined as :-
t2   
  dv 
 =
t1 F dt   = F dt = m
dt dt = m dv  
    
 = m( v 2 - v 1 ) = Δ P = change in momentum due to force F
 t2  
Also, Res =

t1
FRes dt = Δ P

(impulse - momentum theorem)

KVPY_ SA STREAM # 172


Note: Impulse applied to an object in a given time interv al can also be
calculated from the area under force time (F-t) graph in the same
time interval.

Instantaneous Impulse :
There are many cases when a force acts for such a short time that the effect is instantaneous,
e.g., a bat striking a ball. In such cases, although the magnitude of the force and the time for
which it acts may each be unknown but the value of their product (i.e., impulse) can be known by
measuring the initial and final momenta. Thus, we can write.
    

  F dt  P  Pf  Pi

Important Points :
(1) It is a vector quantity.
(2) Dimensions = [MLT –1]
(3) S unit = kg m/s
(4) Direction is along change in momentum.
(5) Magnitude is equal to area under the F-t. graph.
  
 

(6)  = Fdt = Fav dt = Fav  t
(7) It is not a property of a particle, but it is a measure of the degree to which an external force
changes the momentum of the particle.

Example 12. The hero of a stunt film fires 50 g bullets from a machine gun, each at a speed of 1.0 km/s. If
he fires 20 bullets in 4 seconds, what average force does he exert against the machine gun
during this period.
Solution :
The momentum of each bullet
= (0.050 kg) (1000 m/s) = 50 kg-m/s.
The gun has been imparted this much amount of momentum by each bullet fired. Thus, the rate
of change of momentum of the gun
(50 kg  m / s)  20
= = 250 N.
4s
In order to hold the gun, the hero must exert a force of 250 N against the gun.


Im pulsive f orce :
A force, of relatively higher magnitude and acting for relatively shorter time, is called impulsive force.
An impulsive force can change the momentum of a body in a finite magnitude in a very short time
interval. Impulsive force is a relative term. There is no clear boundary between an impulsive and Non-
Impulsive force.
Note: Usually colliding forces are impulsive in nature.
Since, the application time is very small, hence, very little motion of the particle takes place.
Important points :
1. Gravitational force and spring force are always non-Impulsive.
2. Normal, tension and friction are case dependent.
3. An impulsive force can only be balanced by another impulsive force.

KVPY_ SA STREAM # 173


1. Impulsive Normal : In case of collision, normal forces at the surface of collision are always impulsive

eg. Ni = Impulsive; Ng = Non-impulsive

N1

Both normals are Impulsive


N2
/////////////////////////////

N1
N1
N1, N3 = Impulsive; N2 = non-impulsive
//////////////////////////////////////////////////////////////////
N3
N2

Both normals are Impulsive

2. Impulsive Friction : If the normal between the two objects is impulsive, then the friction between the
two will also be impulsive.

Friction at both surfaces is impulsive

Friction due to N2 is non-impulsive and due to N3 and N1 are impulsive

KVPY_ SA STREAM # 174


3. Impulsive Tensions : W hen a string jerks, equal and opposite tension act suddenly at each
end. Consequently equal and opposite impulses act on the bodies attached with the string in the
direction of the string. There are two cases to be considered.
(a) One end of the string is fixed :
The impulse which acts at the fixed end of the string cannot change the momentum of the
fixed object there. The object attached to the free end however will undergo a change in
momentum in the direction of the string. The momentum remains unchanged in a direction
perpendicular to the string where no impulsive forces act.
///////////////////
(b) Both ends of the string attached to movable objects :
In this case equal and opposite impulses act on
T
the two objects, producing equal and opposite T is Impulsive
A
changes in momentum. The total momentum of the
system therefore remains constant, although the
B
momentum of each individual object is changed in T is non-impulsive
the direction of the string. Perpendicular to the string
C
however, no impulse acts and the momentum of each T is non-impulsive
particle in this direction is unchanged.
All normal are impulsive but tension
T is impulsive only for the ball A
For this example:
In case of rod, Tension is always impulsive and in case of spring, Tension is always non-impulsive.

Example 13. Two identical block A and B, connected by a massless string are placed on a frictionless horizontal
plane. A bullet having same mass, moving with speed u strikes block B from behind as shown. If the
bullet gets embedded into the block B then find :
m m
m u
A C B

/////////////////////////////////////////////////////
(a) The velocity of A,B,C after collision.
(b) Impulse on A due to tension in the string
(c) Impulse on C due to normal force of collision.
(d) Impulse on B due to normal force of collision.
Solution :
u
(a) By Conservation of linear momentum v =
3

mu
(b)  T dt  3

u  2mu
(c)  N dt  m 3  u  =
3

mu
(d)  (N  T) dt =  Ndt   T dt  3

2mu
 N dt  3


COLLISION OR IM PACT
Collision is an event in which an impulsive force acts between two or more bodies for a short time,
which results in change of their velocities.

KVPY_ SA STREAM # 175


Note : (a) In a collision, particles may or may not come in physical contact.

(b) The duration of collision, t is negligible as compared to the usual time intervals of observation
of motion.
(c) In a collision the effect of external non impulsive forces such as gravity are not taken into a
account as due to small duration of collision (t) average impulsive force responsible for colli-
sion is much larger than external forces acting on the system.
The collision is infact a redistribution of total momentum of the particles. Thus, law of
conservation of linear momentum is indispensable in dealing with the phenomenon of collision
between particles.
Line of Im pa ct
The line passing through the common normal to the surfaces in contact during impact is called line of
impact. The force during collision acts along this line on both the bodies.
Direction of Line of impact can be determined by:
(a) Geometry of colliding objects like spheres, discs, wedge etc.
(b) Direction of change of momentum.
If one particle is stationary before the collision then the line of impact will be along its motion after
collision.
Cla ssifica tion of collisions
(a) On the basis of line of impact
(i) Head-on collision : If the velocities of the colliding particles are along the same line before
and after the collision.
(ii) Oblique collision : If the velocities of the colliding particles are along different lines before
and after the collision.
(b) On the basis of energy :
(i) Elastic collision : In an elastic collision, the colliding particles regain their shape and size
completely after collision. i.e., no fraction of mechanical energy remains stored as defor-
mation potential energy in the bodies. Thus, kinetic energy of system after collision is equal
to kinetic energy of system before collision. Thus in addition to the linear momentum, ki-
netic energy also remains conserved before and after collision.
(ii) Inelastic collision : In an inelastic collision, the colliding particles do not regain their shape
and size completely after collision. Some fraction of mechanical energy is retained by the
colliding particles in the form of deformation potential energy. Thus, the kinetic energy of
the particles after collision is not equal to that of before collision. However, in the absence
of external forces, law of conservation of linear momentum still holds good.
(iii) Perfectly inelastic : If velocity of separation along the line of impact just after collision be-
comes zero then the collision is perfectly inelastic. Collision is said to be perfectly
inelastic if both the particles stick together after collision and move with same velocity,

Note : Actually collision between all real objects are neither perfectly elastic nor perfectly inelastic, its inelas-
tic in nature.

Exa m p le s of lin e of i m p a ct a n d co llisi on s b a se d on lin e of im p a ct


(i) Two balls A and B are approaching each other such that their centers are moving along line CD.

Head on Collision

KVPY_ SA STREAM # 176


(ii) Two balls A and B are approaching each other such that their center are moving along dotted lines as shown
in figure.

Oblique Collision
(iii) Ball is falling on a stationary wedge.

Oblique Collision
COEFFICIENT OF RESTITUTION (e )
The coeff icient of restitution is def ined as the ratio of the impulses of ref ormation and
deformation of either body.

Im pulse of reformation  F dt
r
e= =
Im pulse of deformation
 F dt
d

Velocity of seperation along line of impact


= Velocity of approach along line of impact
The most general expression for coefficient of restitution is
velocity of separation of points of contact along line of impact
e=
velocity of approach of point of contact along line of impact

Example for calculation of e


Two smooth balls A and B approaching each other such that their centers are moving along line CD in
absence of external impulsive force. The velocities of A and B just before collision be u1 and u2 respectively.
The velocities of A and B just after collision be v 1 and v 2 respectively.
Just Before collision
Just After collision
u1 u2 v1 v2

Line of impact Line of impact


C D C D
A B A B

u1 > u2 v1 < v2
u2 v v2
u1 v v1
ND ND NR
NR
m1 m1 m1
m2 m2 m2

 Deformation Reformation

KVPY_ SA STREAM # 177


 F ext = 0 momentum is conserved for the system.
m1u1  m 2u2 m1v1  m 2 v 2
 m 1u1 + m 2 u2 = (m 1 + m 2)v = m 1v 1 + m 2v 2  v= = .......(1)
m1  m 2 m1  m 2
Impulse of Deformation :
JD = change in momentum of any one body during deformation.
= m 2 (v – u2) for m2
= m 1 (–v + u1) for m1

Impulse of Reformation :
JR = change in momentum of any one body during Reformation.
= m 2 (v 2 – v) for m2
= m 1 (v – v 1) for m1

Impulse of Reformation (JR ) v 2  v1 Velocity of separation along line of impact
e=  = =
Impulse of Deformation ( JD ) u1  u 2 Velocity of approach along line of impact

Note : e is independent of shape and mass of object but depends on the material.
The coefficient of restitution is constant for a pair of materials.
(a) e=1  Impulse of Reformation = Impulse of Deformation
 Velocity of separation = Velocity of approach
 Kinetic energy of particles after collision may be equal to that of before collision.
 Collision is elastic.
(b) e=0  Impulse of Reformation = 0
 Velocity of separation = 0
 Kinetic energy of particles after collision is not equal to that of before collision.
 Collision is perfectly inelastic .
(c) 0<e<1  Impulse of Reformation < Impulse of Deformation
 Velocity of separation < Velocity of approach
 Kinetic energy of particles after collision is not equal to that of before collision.
 Collision is Inelastic.

Note : In case of contact collisions e is always less than unity.


 0e1

Important Point :
In case of elastic collision, if rough surface is present then
kf < ki (because friction is impulsive)
Where, k is Kinetic Energy.

Rough
/////////////////////////////////////

A particle ‘B’ moving along the dotted line collides with a rod also in state of motion as shown in the figure.
The particle B comes in contact with point C on the rod.
To write down the expression for coefficient of restitution e, we first draw the line of impact. Then we resolve
the components of velocities of points of contact of both the bodies along line of impact just before and just
after collision.

v 2 x  v1x
Then e = u u
1x 2x

KVPY_ SA STREAM # 178


Co ll ision i n on e d im e n sio n ( H e a d o n )

m1 m2 m1 m2
u1 u2 v1 v2

(a) (b)
Before Collision After Collision

u1 > u2 v2 > v1
v 2  v1
e=  (u1 – u2)e = (v 2 – v 1)
u1  u 2
By momentum conservation,
m 1u1 + m2u2 = m1v 1 + m2v 2  v 2 = v 1 + e(u1 – u2)

m1u1  m 2u 2  m 2 e(u1  u 2 ) m1u1  m 2u 2  m1e(u1  u 2 )


and v1 =  v2 =
m1  m 2 m1  m 2

Special Case :
(1) e=0
 v1 = v2
 for perfectly inelastic collision, both the bodies, move with same vel. after collision.
(2) e=1
and m 1 = m 2 = m,
we get v 1 = u2 and v 2 = u1
i.e., when two particles of equal mass collide elastically and the collision is head on, they ex-
change their velocities., e.g.

v1=0
2m/s m
m

Before Collision

(3) m 1 >> m 2
m2
m 1 + m 2  m 1 and 0
m1
 v 1 = u1 No change
and v 2 = u1 + e(u1 – u2)

Example 14. Two identical balls are approaching towards each other on a straight line with velocity 2 m/s
and 4 m/s respectively. Find the final velocities, after elastic collision between them.

m 2m/s 4m/s m

Solution : The two velocities will be exchanged and the final motion is reverse of initial motion for both.

4m/s m m 2m/s

KVPY_ SA STREAM # 179


Example 15. Three balls A, B and C of same mass ‘m’ are placed on a frictionless horizontal plane in a
straight line as shown. Ball A is moved with velocity u towards the middle ball B. If all the
collisions are elastic then, find the final velocities of all the balls.
m m m
u
A B C
//////////////////////////////////////////
Solution : A collides elastically with B and comes to rest but B starts moving with velocity u
m m m
u
A B C
//////////////////////////////////////////
After a while B collides elastically with C and comes to rest but C starts moving with velocity u
m m m
u
A B C
//////////////////////////////////////////
 Final velocities
VA = 0;
VB = 0 and VC = u Ans.

Example 16. Two particles of mass m and 2m moving in opposite directions on a frictionless surface collide
elastically with velocity v and 2v respectively. Find their velocities after collision, also find the
fraction of kinetic energy lost by the colliding particles.

2v v
m 2m

Solution : Let the final velocities of m and 2m be v 1 and v 2 respectively as shown in the figure:
v2
m 2m

By conservation of momentum:
m(2v) + 2m(–v) = m(v 1) + 2m (v 2)
or 0 = mv 1 + 2mv 2
or v 1 + 2v 2 = 0 .........(1)
and since the collision is elastic:
v 2 – v 1 = 2v –(–v)
or v 2 – v 1 = 3v .........(2)
Solving the above two equations, we get,
v 2 = v and v 1 = –2v Ans.

i.e., the mass 2m returns with velocity v while the mass m returns with velocity 2v in the
direction shown in figure:
v
m 2m

The collision was elastic therefore, no kinetic energy is lost, KE loss = KE i - KE f

1 1  1 1 
or,  m( 2v ) 2  ( 2m)(  v ) 2    m( 2v ) 2  ( 2m )v 2  = 0
 2 2   2 2 

Example 17. On a frictionless surface, a ball of mass m moving at a speed v makes a head on collision with
an identical ball at rest. The kinetic energy of the balls after the collision is 3/4th of the original.
Find the coefficient of restitution.
Solution : As we have seen in the above discussion, that under the given conditions :

KVPY_ SA STREAM # 180


By using conservation of linear momentum and equation of e, we get,

 1 e   1 e 
v 1'    v and v2 '   v
 2   2 

3 1 1 3  1 mv 2 
Given that Kf  Ki or mv 1’ 2 + mv 2’ 2 =  
4 2 2 4 2 
Substituting the value, we get
2 2
 1 e   1 e  3 1
  +   = or e= Ans.
 2   2  4 2

Example 18. A ball is moving with velocity 2 m/s towards a heavy wall moving towards the ball with speed
1m/s as shown in fig. Assuming collision to be elastic, find the velocity of the ball immediately
after the collision.

Solution : The speed of wall will not change af ter the collision. So, let v be the velocity of the ball
after collision in the direction shown in figure. Since collision is elastic (e = 1),

separation speed = approach speed


or v–1=2+1
or v = 4 m/s Ans.


Col l isi on in t w o d im e n si on ( o b li q u e )
1. A pair of equal and opposite impulses act along common normal direction. Hence, linear momen-
tum of individual particles do change along common normal direction. If mass of the colliding
particles remain constant during collision, then we can say that linear v elocity of the indiv idual
particles change during collision in this direction.
2. No component of impulse act along common tangent direction. Hence, linear momentum or linear
velocity of individual particles (if mass is constant) remain unchanged along this direction.
3. Net impulse on both the particles is zero during collision. Hence, net momentum of both the
particles remain conserved before and after collision in any direction.
4. Definition of coefficient of restitution can be applied along common normal direction, i.e., along
common normal direction we can apply
Relative speed of separation = e (relative speed of approach)

KVPY_ SA STREAM # 181


Example 19. A ball of mass m hits a floor with a speed v 0 making an angle of incidence a with the normal.
The coefficient of restitution is e. Find the speed of the reflected ball and the angle of reflection
of the ball.
Solution :
The component of velocity v 0 along common tangential direction v 0 sin  will remain un-
changed. Let v be the component along common normal direction after collision. Applying,
Relative speed of separation = e (Relative speed of approach)
along common normal direction, we get
v = ev 0 cos 

v (= ev0cos)

v'

v0sin

Thus, after collision components of velocity v’ are v 0 sin  and ev 0 cos 

 v '  ( v 0 sin  )2  (ev 0 cos  )2 Ans.

v 0 sin 
and tan  =
ev 0 cos 

tan 
or tan  = Ans.
e

Note : For elastic collision, e = 1


 v’ = v 0 and  = 

Example 20. A ball of mass m makes an elastic collision with another identical ball at rest. Show that if the
collision is oblique, the bodies go at right angles to each other after collision.
Solution :
In head on elastic collision between two particles, they exchange their velocities. In this
case, the component of ball 1 along common normal direction, v cos 

KVPY_ SA STREAM # 182


becomes zero after collision, while that of 2 becomes v cos . W hile the components
along common tangent direction of both the particles remain unchanged. Thus, the com-
ponents along common tangent and common normal direction of both the balls in tabular
form are given below.

Component along common Component along common normal


Ball
tangent direction direction
Before collision After collision Before collision After collision
1 v sin  v sin  v cos  
2    v cos 

From the above table and figure, we see that both the balls move at right angle after
collision with velocities v sin  and v cos .
Note : When two identical bodies have an oblique elastic collision, with one body at rest before collision, then
the two bodies will go in  directions.

Example 21. Two spheres are moving towards each other. Both
2kg
have same radius but their masses are 2 kg and 4 4m/s
kg. If the velocities are 4 m/s and 2 m/s respec- A
tively and coefficient of restitution is e = 1/3, find. R
2m/s R

(a) The common velocity along the line of impact. B


4kg
(b) Final velocities along line of impact.
(c) Impulse of deformation.
(d) impulse of reformation.
(e) Maximum potential energy of deformation.
(f) Loss in kinetic energy due to collision.

Solution :

A 4m/s C Line of motion


2kg 
R R
R 4kg
Line of motion 2m/s B
Line of impact

BC R 1
In ABC sin  == = or  = 30º
AB 2R 2
(a) By conservation of momentum along line of impact.

4sin30º

2kg 4m/s
30º 4sin30º
4cos30º
2cos30º
30º
v
2m/s B 4kg
2sin30º
2sin30º
Maximum Deformed State
Just Before Collision Along LOI

2(4 cos 30º) – 4(2cos 30º) = (2 + 4)v


or v = 0 (common velocity along LO)

KVPY_ SA STREAM # 183


4sin30º
(b) A v
2kg 1
4kg B
v2
2sin30º

Just After Collision Along LOI

Let v 1 and v 2 be the final velocity of A and B respectively then, by conservation of momentum along line
of impact,
2(4cos 30º) – 4(2cos 30º) = 2(v 1) + 4(v 2) or 0 = v 1 + 2v 2 ......... (1)
velocity of separation along LO 
By coefficient of restitution, e
velocity of approach along LO 

1 v 2  v1
or = or v2 – v1 = 3 ........ (2)
3 4 cos 30º 2 cos 30º

2 1
from the above two equations, v1 = m / s and v 2 = m/s.
3 3

(c) JD = m1(v – u1) = 2(0 – 4 cos 30º) = – 4 3 N-s

1 4
(d) JR = eJD = (–4 3 ) = – N-s
3 3
(e) Maximum potential energy of deformation is equal to loss in kinetic energy during deformation upto
maximum deformed state,
1 1 1
U= m (u cos )2 + m2(u2 cos )2 – (m1 + m2)v 2
2 1 1 2 2
1 1 1
= 2(4 cos 30º)2 + 4(–2cos 30º)2 – (2 + 4) (0)2 or U = 18 Joule.
2 2 2
(f) Loss in kinetic energy,

1 1 1 2 1 2
KE = m1(u1 cos )2 + m2(u2 cos )2 –  m1v 1  m 2 v 2 
2 2 2 2 

 1  2 2 1  1 2 
1 1     4  
= 2(4 cos 30º)2 + 4(–2 cos 30º)2 –  2 2  
2  3  
3
2 2   
KE = 16 Joule
Example 22. Two point particles A and B are placed in line on a frictionless horizontal plane. If particle A
(mass 1 kg) is moved with velocity 10 m/s towards stationary particle B (mass 2 kg) and after
collision the two move at an angle of 45º with the initial direction of motion, then find :
1kg 10m/s 2kg
A B
(a) Velocities of A and B just after collision.
(b) Coefficient of restitution.
Solution : The very first step to solve such problems is to find the line of impact which is along the direction of
force applied by A on B, resulting the stationary B to move. Thus, by watching the direction of
motion of B, line of impact can be determined. In this case line of impact is along the direction of
motion of B. i.e. 45º with the initial direction of motion of A.
vA
45º y
45º
vB
x
Line of impact

KVPY_ SA STREAM # 184


(a) By conservation of momentum, along x direction: mA uA = mAv A cos 45º + mBv B cos 45º
or 1(10) = 1(v A cos 45º) + 2(v B (cos 45º)
or v A + 2v B = 10 2 ............ (1)
along y direction
0 = mAv A sin 45º + mBv B sin 45º
or 0 = 1(v A sin 45º) – 2(v B sin 45º)
or vA = 2vB ............ (2)
solving the two equations,

10 5
vA = m/s and vB = m/s.
2 2

velocity of separation along line of impect


(b) e
veloctiy of approach along line of impect

vA

uA A
A 45º B 90º
uA cos45º vA cos 90º vB

LO LO

Before collision After collision

5
0
v B  v A cos 90º 2 1
or e= = 10 = Ans.
u A cos 45 º 2
2

Example 23. A smooth sphere of mass m is moving on a horizontal plane with a velocity 3 î  ĵ when it

collides with a vertical wall which is parallel to the vector ĵ . If the coefficient of restitution

1
between the sphere and the wall is , find
2
(a) the velocity of the sphere after impact,
(b) the loss in kinetic energy caused by the impact.

(c) the impulse J that acts on the sphere.
Solution :

Let v be the velocity of the sphere after impact.

To find v we must separate the velocity components parallel and perpendicular to the
wall.
Using the law of restitution the component of velocity paral-
lel to the wall remains unchanged while component perpen-
dicular to the wall becomes e times in opposite direction.
 3
Thus, v   î  ĵ
2

KVPY_ SA STREAM # 185


3
(a) Therefore, the velocity of the sphere after impact is =  î  ĵ Ans.
2
  3 2  27
1 1 1 1  2
(b) The loss in K.E. = mv 2 – mu 2 = m(3 2 + 1 2) – m   2   1   8 m Ans.
2 2 2 2   

    3 
(c)
  
  9
J = P  Pf  Pi  m( v )  m(u) = m   2 î  ĵ  – m 3 î  ĵ =  2 m î Ans.


VAR IABLE M ASS SYSTEM :

If a mass is added or ejected from a system, at rate  kg/s and relative velocity v rel (w.r.t. the system),

then the force exerted by this mass on the system has magnitude  v rel .

Thrust Force ( Ft )
   dm 
Ft  v rel  
 dt 

Suppose at some moment t = t mass of a body is m and its velocity is v . After some time at t =
 
t + dt its mass becomes (m – dm) and v elocity becomes v  dv . The mass dm is ejected with
  
relative velocity v r . Absolute velocity of mass ‘dm’ is therefore ( v + v r ). If no external forces are
acting on the system, the linear momentum of the system will remain conserved, or
 
Pi  Pf
    
or m v = (m – dm) ( v + d v ) + dm ( v + v r )
      
or m v = m v + md v – (dm) v – (dm) (d v ) + (dm) v + v r dm

The term (dm) (d v ) is too small and can be neglected.
 
 md v = – v r dm

 dv    dm 
or m   = vr   
 dt   dt 

 dv 
Here, m 


dt 
 
 = thrust force F

t

dm
and – = rate at which mass is ejecting
dt
   dm 
or Ft  v r  
 dt 
Problems related to variable mass can be solved in following four steps
1. Make a list of all the forces acting on the main mass and apply them on it.

   dm 
2. Apply an additional thrust force Ft on the mass, the magnitude of which is v r   dt  and direc-
 

tion is given by the direction of v r in case the mass is increasing and otherwise the direction of

– v r if it is decreasing.
3. Find net force on the mass and apply
 
dv
Fnet  m (m = mass at the particular instant)
dt
4. Integrate it with proper limits to find velocity at any time t.

KVPY_ SA STREAM # 186


Note : Problems of one-dimensional motion (which are mostly asked in JEE) can be solved in easier
manner just by assigning positive and negative signs to all vector quantities. Here are f ew ex-
ample in support of the above theory.

Example 24. A flat car of mass m 0 starts moving to the right due to a
constant horizontal force F. Sand spills on the flat car from
a stationary hopper. The rate of loading is constant and
eq ual t o  k g/ s. F i nd t h e t i m e dep en den ce of t he 
velocity and the acceleration of the flat car in the process m0 F
of loading. The friction is negligibly small.

Solution : Initial velocity of the flat car is zero. Let v be its velocity at time t and m its mass at that
instant. Then

At t = 0, v = 0 and m = m 0 at t = t, v = v and m = m 0 + t
Here, v r = v (backwards)
dm
= 
dt

dm
 Ft = v r = v (backwards)
dt
Net force on the flat car at time t is F net = F – F t
dv
or m = F – v ....(i)
dt

dv v dv t dt
or (m 0 + t)
dt
=F – v or  0 F  v
=  0 m 0  t

1 1
 – [  n (F – v)] 0v = [  n (m 0 + t)] 0t
 

 F   m0  t 
  n   =  n  
 m 
 F  v   0 

F m  t Ft
 = 0 or v= Ans.
F  v m0 m 0  t

dv
From Eq. (i), = acceleration of flat car at time t
dt

 Ft 
F 
F  v  m 0  t  Fm 0
or = a=   or a= Ans.
m m0  t (m 0  t ) 2
 
 

KVPY_ SA STREAM # 187


Example 25. A cart loaded with sand moves along a horizontal floor due to a constant force F coinciding in
direction with the cart’s velocity vector. In the process sand spills through a hole in the bottom
with a constant rate kg/s. Find the acceleration and velocity of the cart at the moment t, if at
the initial moment t = 0 the cart with loaded sand had the mass m 0 and its velocity was equal
to zero. Friction is to be neglected.
Solution : In this problem the sand spills through a hole in the bottom of the cart. Hence, the relative
velocity of the sand v r will be zero because it will acquire the same velocity as that of the
cart at the moment.
vr = 0
v
m
 dm  F
Thus, F t = 0  as Ft  v r 
 dt 
v
and the net force will be F only.
 F net = F

 dv 
or m  = F ....(i)
 dt 

dv
But here m = m 0 – t  (m 0 – t) =F
dt

F
or 
v
dv = 
t F dt
 v=  n ( m0   t )  0t
0 0 m 0  t 

F  m 0 
or v= ln
  m0  t 
Ans.

From eq. (i), acceleration of the cart


dv F
a= =
dt m
F
or a = m  t Ans.
0


R ock e t p r o p u lsio n :
Let m 0 be the mass of the rocket at time t = 0. m its mass at any time t and v its velocity at that
moment. Initially, let us suppose that the velocity of the rocket is u.

  dm 
Further, let   be the mass of the gas ejected per unit time and v r the exhaust velocity of the
 dt 

  dm 
gases with respect to rocket. Usually   and v r are kept constant throughout the journey of
 dt 
the rocket. Now, let us write few equations which can be used in the problems of rocket propul-
sion. At time t = t,

KVPY_ SA STREAM # 188


  dm 
1. Thrust force on the rocket Ft = v r   (upwards)
 dt 
2. W eight of the rocket W = mg (downwards)
3. Net force on the rocket F net = F t – W (upwards)
  dm 
or F net = v r   –mg
 dt 

F
4. Net acceleration of the rocket a=
m

dv vr   dm  vr
or
dt
=
m

 dt 
 –g or dv =
m
 dm – g dt

v m  dm t
or  u
dv  v r  m0 m
–g  0
dt

 m0 
Thus, v = u – gt + v r  n   ...(i)
 m 

 dm  dm
Note : 1. F t = v r    is upwards, as v r is downwards and is negative.
 dt  dt

 m0 
2. If gravity is ignored and initial velocity of the rocket u = 0, Eq. (i) reduces to v = v r ln  .
 m 

Example 26. A rocket, with an initial mass of 1000 kg, is launched vertically upwards from rest under gravity.
The rocket burns fuel at the rate of 10 kg per second. The burnt matter is ejected vertically
downwards with a speed of 2000 ms–1 relative to the rocket. If burning stops after one minute.
Find the maximum velocity of the rocket. (Take g as at 10 ms–2)
Solution :
Using the velocity equation

m 
v = u – gt + v r ln  0 
 m 
Here u = 0, t = 60s, g = 10 m/s 2, v r = 2000 m/s, m 0 = 1000 kg
and m = 1000 – 10 × 60 = 400 kg

 1000 
W e get v = 0 – 600 + 2000 ln  
 400 
or v = 2000 ln 2.5 – 600
The maximum velocity of the rocket is 200(10 ln 2.5 – 3) = 1232.6 ms –1 Ans.


LIN EAR M OM ENTUM CONSERVATION IN PRESENCE OF EXTERNAL FORCE.

 dP    
Fext =  Fext dt = dP  dP = Fext )impulsive dt
dt
  
 If Fext )impulsive = 0  dP = 0 or P is constant

KVPY_ SA STREAM # 189


Note: Momentum is conserved if the external force present is non-impulsive. eg. gravitation or spring force

Example 27. Two balls are moving towards each other on a vertical line collides with each other as shown.
Find their velocities just after collision.

2kg

3m/s

4m/s

4kg

Solution : Let the final velocity of 4 kg ball just after collision be v. Since, 4m/s
external force is gravitational which is non - impulsive, hence,
linear momentum will be conserved.
Applying linear momentum conservation: 2kg
2(–3) + 4(4) = 2(4) + 4(v)
1 v
or v= m/s
2
4kg

/////////////////////
Example 28. A bullet of mass 50g is fired from below into the bob of mass
450g of a long simple pendulum as shown in figure. The bullet
remains inside the bob and the bob rises through a height of
1.8 m. Find the speed of the bullet. Take g = 10 m/s2.
v

Solution : Let the speed of the bullet be v. Let the common velocity of the bullet and the bob, after the
bullet is embedded into the bob, is V. By the principle of conservation of the linear momentum,
(0.05 kg) v v
V= =
0.45 kg  0.05 kg 10
The string becomes loose and the bob will go up with a deceleration of g = 10 m/s2. As it comes
to rest at a height of 1.8 m, using the equation v 2 = u2 + 2ax,
( v / 10)2
1.8 m =
2  10 m / s 2
or, v = 60 m/s.

KVPY_ SA STREAM # 190


OBJECTIVE QUESTIONS
1. In the figure shown a hole of radius 2 cm is made in a
semicircular disc of radius 6  cm at a distance 8 cm from the
centre C of the disc. The distance of the centre of mass of this
system from point C is:

(A) 4 cm (B) 8 cm (C) 6 cm (D) 12 cm

2. The ‘ y ’ co-ordinate of the centre of mass of the system of three


rods of length ‘2a‘ and two rods of length ‘a’ as shown in figure is
: (Assume all rods to be of uniform and equal density)
9a 9a
(A) (B)
8 3 16 3

8a
(C) zero (D)
3

3. A segment of angle  is cut from a half disc of symmetrically as shown.


If the centre of mass of the remaining part is at a distance ‘a’ from O
and the centre of mass of the original disc was at distance d then it
can be definitely said that :
(A) a = d (B) a > d (C) a < d
(D) A,B,C depends on the angle of segment cut from disc.

4. A solid ball of mass m and radius R is released from the posi-


tion shown in a large hollow fixed shell of same mass m and
radius 3R as shown in figure. The displacement of the centre
of mass of the system from its initial position when the solid
ball touches the lower surface of the hollow shell is : (centres
of both the spheres coincide initially) :

(A) 0 (B) 3R (C) 2R (D) R

5. Which of the following figures best represents the stable equilibrium of a uniform semicircular rod in vertical
plane. It is hinged at point A.

AX AX

A
(A) (B) (C) (D)

6. In the figure shown a semicircular area is removed from a uniform square plate
of side ‘  ’ and mass (before removing) ‘m’. The x-coordinate of centre of mass y
of remaining portion is (The origin is at the centre of square)
 (  2)  (  2)
(A) – 2 (8  ) (B)
2 (8   ) x

 (  2)
(C) – (D) None of these
8

KVPY_ SA STREAM # 191


7. With respect to the centre of mass of a two body system.
(A) the two bodies may move in same direction
(B) the heavier body may move faster than the lighter body
(C) one body may be at rest while the second body is moving
(D) none of these

8. A system consists of two point masses, A and B of masses 1 kg and 2 kg respectively. At an instant the
kinetic energy of A with respect to the centre of mass is 2 Joules and the velocity of centre of mass is 2 m/
s. The kinetic energy of the system at this instant is :
(A) 9 J (B) 11 J (C) 13 J (D) none of these

9. In the figure shown the system is at rest initially. Two persons


'A' and 'B' of masses 40 kg each move with speeds v 1 and v 2
respectively towards each other on a plank lying on a smooth
horizontal surface as shown in figure. Plank travels a distance
of 20 m towards right direction in 5 sec. (Here v 1 and v 2 are
given with respect to the plank). Then :
(i) v 1 = 10 m/s, v 2 = 0 m/s (ii) v 1 = 15 m/s, v 2 = 5 m/s
(iii) v 1 = 20 m/s, v 2 = 10 m/s
(A) only (i) and (iii) are possible (B) All (i), (ii) and (iii) are not possible
(C) only (ii) and (iii) are possible (D) All (i), (ii) and (iii) are possible

rough
10. When a block is placed on a wedge as shown in figure, the block starts
sliding down and the wedge also start sliding on ground. All surfaces are Block
rough. The centre of mass of (wedge + block) system will move Wedge
(A) leftward and downward. (B) right ward and downward.
rough
(C) leftward and upwards. (D) only downward.

11. A long plank of mass 2 kg is placed on a smooth horizontal surface. Two blocks each of mass 1 kg are
placed in contact on its rough surface. The coefficient of friction between first block and plank is 0.2 and that
between second block and plank is 0.4. The blocks are projected in the opposite direction on the plank with
velocity 2 m/s and 6 m/s respectively. The velocity of plank when both the blocks come to relative rest with
respect to the plank and total work done by friction on the system (blocks + plank) will be :
(A) 2 m/s, –18 J (B) 1 m/s, zero (C) 1 m/s, – 18 J (D) 2 m/s, zero

12. A small block of mass 'm' is placed on bigger block of mass M, which is placed on a frictionless horizontal
surface. The two blocks are given equal speed u, but opposite directions, as shown in the figure. After
sometime, it is observed that both the blocks are moving in the direction of motion of the lower block, with a
u
speed greater than . It can be concluded that -
2
u
friction m
Horizontal
M u frictionless surface
/////////////////////////////////////////////////////////////
(A) M > 3m (B) 3M < m
(C) m > 2M (D) M, m can have any value such that M > m

13. A bullet of mass m penetrates a thickness t of a fixed plate of mass M. If M is free to move and resistance
supposed to be uniform and same in both cases, the thickness penetrated is :

Mt mt Mm M2
(A) (B) (C) t (D) t
Mm Mm (M  m)2 (M  m)2

KVPY_ SA STREAM # 192


14. A particle of constant mass m moves in straight line under a time varying force. The change in momentum of
the particle in a time interval t1 to t2 can be given as
t2

p=  F dt .
t1

The force F acting on the particle as a function of time t is shown in the graph. If velocity of the particle is zero
at t = 0 then

(A) The particle is moving under constant acceleration.


(B) The particle is moving with constant velocity
(C) The particle is moving with decreasing speed.
(D) The change in momentum of the particle from time t1 = 0 to t2 = 5 s in 50 Ns

15. Two blocks initially at rest having masses m 1 and m 2 are connected by spring of spring constant k (as
shown in the figure). The block of mass m 1 is pulled by a constant force F 1 and the other block is pulled
by a constant force F 2. Find the maximum elongation of the spring.

m1F2  m 2F1 2m1F2  m 2F1  m 2F2  m1F1 2m 2F2  m1F1 


(A) (B) (C) (D)
k m1  m 2  km1  m 2  k m1  m 2  k m1  m 2 

16. A hemisphere of mass 3 m and radius R is free to slide with its base on
a smooth horizontal table. A particle of mass m is placed on the top of
the hemisphere. If particle is displaced with a negligible velocity, then
find the angular velocity of the particle relative to the centre of the
hemisphere at an angular displacement , when velocity of hemisphere
is v.
4v 3v 5v 2v
(A) (B) (C) (D)
R cos  R cos  R cos  R cos 

17. Two blocks of masses m1 and m2 are connected by spring of constant K. The
spring is initially compressed and the system is released from rest at t = 0
second. The work done by spring on the blocks m1 and m2 be W 1 and W 2
respectively by time t. The speeds of both the blocks at time ‘t’ are non zero.
W1
Then the value of W equals to
2
2 2
m1 m2 m  m 
 1   2 
(A) (B) (C)   (D)  
m2 m1
 m2   m1 

18. A projectile of range R bursts at its highest point in two fragments. The heavier is double the mass of the
R
lighter. Lighter fragment falls at horizontal distance from the point of projection in the opposite side of
2
projection. The distance, where other part falls, from point of projection is.
7R 5R 8R 6R
(A) (B) (C) (D)
4 4 4 4

KVPY_ SA STREAM # 193


19. Ball A of mass m after sliding from an inclined plane, strikes elastically
another ball B of same mass at rest. Find the minimum height H so
that ball B just completes the circular motion.
5R
(A) H = 3R (B) H = 2R (C) H = (D) H = 4R
2

20. A particle ‘A’ of mass m collides head on with another stationary particle ‘B’ of the same mass ‘m’.The
kinetic energy lost by the colliding particle 'A' will be maximum if the coefficient of the restituition is
(A) 1 (B) 0 (C) 0.5 (D) none
21. Two masses ‘ m ’ and ‘ 2 m ’ are placed in fixed horizontal circular smooth
hollow tube as shown. The mass ‘ m ’ is moving with speed ‘u’ and the
mass ‘ 2 m ’ is stationary. After their first collision, the time elapsed for next
collision. (coefficient of restitution e = 1/2)
2r 4r 3r 12r
(A) (B) (C) (D)
u u u u
22. A particle of mass m1 moves with speed u1 and collide head on with a stationary particle of mass m2. After
the collision, the velocities of the particles are v 1 and v 2, and e is coefficient of restitution for the collision. If v 1
is to be positive, i.e., for the first particle to continue moving in same direction then :
m2 m1 m1  m 2 m1  m 2
(A) >e (B) >e (C) >e (D) >e
m1 m2 m2 m2

23. Two balls approach each other with same speed and undergo head on elastic collision after which one
of them of mass 3 kg comes to rest. The mass of the other ball is
(A) 1 kg (B) 3 kg (C) 6 kg (D) 9 kg
100
24. During the head on collision of two masses 1 kg and 2 kg the maximum energy of deformation is J.
3
If before collision the masses are moving in the same direction, then their velocity of approach before
the collision is :
(A) 10 m/sec. (B) 5 m/sec. (C) 20 m/sec. (D) 10 2 m/sec.
25. In an elastic collision :
(A) kinetic energy of each particle is conserved
(B) momentum of the system must be conserved
(C) velocity of approach must be equal to velocity of separation along line of impact
(D) momentum of the system must be conserved along the line of impact.
26. A heavy elevator is moving upwards with constant velocity 5 m/sec. At time t = 0 a ball situated at a
distance of 100 m from the elevator is dropped. Find the velocity of ball just after the collision (elastic).
(use g = 10 m/s2)
(A) 10 m/s (B) 20 m/s (C) 40 m/s (D) 50 m/s
27. Two particles A and B of masses 10 kg and 38 kg respectively are moving along the same straight line
with velocities 15 m/s & 3 m/s respectively in the same direction. After elastic collision the velocities of
A and B are v A and v B in the direction of initial motion. Then :

(A) v A = 20, v B = 8 (B) v A =  4, v B = 8


(C) v A = 16, v B = 28 (D) v A =  5, v B = 10
28. A block of mass M is moving on a smooth horizontal surface with constant speed u. Bullets are fired
horizontally against the block to reduce the velocity of the block to half its initial value. Bullets get embedded
in the block. Mass of each bullet is ‘m’ and speed ‘u’. Then the number of bullets required is :
M M M
(A) (B) (C) (D) None of these
3m m 2m

KVPY_ SA STREAM # 194


29. A train is running at a speed of 108 km/hr. An inspection cart is also moving with a speed 10m/s in the same

 1
direction as train. The train hits the cart  e   . The velocity of the cart after the collision is (in m/s) nearly..
 2
(A) 20 (B) 40 (C) 50
(D) cannot determine as the masses of train and cart are not given.
30. Two rubber balls, each of radius R, collide elastically. During the small time of collision, which of the
following curves shows the correct variation of the elastic potential energy (U(r)) with the separation
between the centers (r) :

(A) (B) (C) (D)


31. A particle moving with a velocity v = 2 î + 3 ĵ – 4k̂ strikes a smooth fixed wall placed in x-y plane. Assuming
the collision to be inelastic, the velocity of the particle just after the collision may be
(A) 2 î + 3 ĵ + 3k̂ (B) 2 î + 3 ĵ + 4k̂ (C) 2 î + 3 ĵ + 5k̂ (D) 2 î – 3 ĵ – 4k̂

1
32. In the figure shown, coefficient of restitution between A and B is e = , then :
2

v 3
(A) velocity of B after collision is (B) impulse between two during collision is mv
2 4
3 1
(C) loss of K.E. during collision is mv 2 (D) loss of K.E. during collision is mv 2
8 4

33. As shown in the figure a body of mass m moving horizontally with


speed 3 m/s hits a fixed smooth wedge and goes up with a velocity
v f in the vertical direction. If angle of wedge is 30º, the velocity v f will be:
(A) 3 m/s (B) 3 m/s
1
(C) m/s (D) this is not possible
3
34. A small smooth disc of mass m and radius r moving with an initial velocity ‘v’ along the positive x-axis collided
with a big disc of mass 2m and radius 2r which was initially at rest with its centre at origin as shown in figure.

r v
2r
x
O

If the coefficient of restitution is 0 then velocity of larger disc after collision is


8v 2 2 8v 2 2 v 2 2 8v
(A) î  v ĵ (B) î  v ĵ (C) î (D) v î  ĵ
27 27 27 27 3 27 27

KVPY_ SA STREAM # 195


35. A ball ‘A’ of mass M collides elastically with another identical ball ‘B’ at rest as shown in figure. Initially
velocity of ball ‘A’ is u m/s. After collision :

(A) speed of ball A is u cos. (B) speed of ball B is u sin.


(C) velocity of ball A is usin2 i + u sincos j (D) None of these
36. Two balls of the same mass are dropped from the same height onto the floor. The first ball bounces
upwards from the floor elastically. The second ball sticks to the floor. The first applies an impulse to the
floor of I 1 the second applies an impulse 2 (for the duration of collision). Then the relation between both
the impulses is,
1 1
(A) 2 = 21 (B) 2 = (C) 2 = 41 (D) 2 =
2 4

37. The given figure shows a plot of the time dependent force F x
acting on a particle in motion along the x-axis. W hat is the
total impulse delivered by this force to the particle from time t
= 0 to t = 2second?

(A) 0 (B) 1 kg-m/s


(C) 2 kg-m/s (D) 3 kg-m/s
38. A very small cube is released from edge of a fixed hemispherical frictionless
bowl whose radius is 0.45 m. Which of the following is correct. (mass of
bowl = 4kg, mass of cube is 40 gm)
(A) workdone by ground on bowl is non zero.
(B) Impulse by ground on bowl is zero. ////////////////////////////////

(C) Impulse by ground on system is non zero & equal to 0.12 kg/ms
(D) work done by normal reaction of bowl on cube is zero.
39. A ball is thrown onto a rough floor with speed u at angle  = 45º. If
it rebounds with speed v at the same angle  = 45º. Find the
coefficient of kinetic friction between the floor and the ball. The
coefficient of restitution is e = 0.6 :
(A) 0.5 (B) 0.6
(C) 0.1 (D) 0.25

40. Sand is falling on a flat car being pulled with constant speed. The rate
of mass falling on the cart is constant. Then the horizontal component
of force exerted by the falling sand on the cart
(A) increases (B) decreases
(C) remains constant (D) increases and then decreases

KVPY PROBLEMS (PREVIOUS YEARS)


1. On a horizontal frictional frozen lake, a girl (36 kg) and a box (9kg) are connected to each other by means of
a rope. Initially they are 20 m apart. The girl exerts a horizontal force on the box, pulling it towards her. How
far has the girl travelled when she meets the box ? [KVPY_2011]
(A) 10 m (B) Since there is no friction, the girl will not move
(C) 16 m (D) 4m
2. A clay ball of mass m and speed v strikes another metal ball of same mass m, which is at rest. They stick
together after collision. The kinetic energy of the system after collision is : [KVPY_2012]
(A) mv2/2 (B) mv2/4 (C) 2 mv2 (D) mv2

KVPY_ SA STREAM # 196


3. A man inside a freely falling box throws a heavy ball towards a side wall. The ball keeps on bouncing between
the opposite walls of the box. We neglect air resistance and friciton. Which of the following figures depicts
the motion of the centre of mass of the entire system (man, the ball and the box)? [KVPY_2013]

(A) (B) (C) (D)

4. A ball is thrown horizontally from a height with a certain initial


velocity at time t = 0. The ball bounces repeatedly from the ground
with the coefficient of restitution less than 1 as shown.

Neglect air resistance and taking the upward direction as positive,


which figure qualitatively depicts the vertical component of the ball’s
velocity (Vy) as a function of time (t)? [KVPY_2013]

(A) (B)

(C) (D)

5. A uniform square wooden sheet of side a has its center of mass located at point O as shown in the figure on
the left. A square portion of side b of this sheet is cut out to produce and L-shaped sheet as shown in the
figure on the right. [KVPY_2014_SA]
a b

O
P

The center of mass of the L-shaped sheet lies at the point P (in the diagram) when
(A) a/b = ( 5 -1)/2 (B) a/b = ( 5 +1)/2 (C) a/b = ( 3 -1)/1 (D) a/b = ( 3 +1)/2

6. A uniform thin rod of length 2L and mass m lies on a horizontal table. A horizontal inpulse J is given to the rod
at one end. There is no friction. The total K.E. of the rod just after the impulse will be : [KVPY_2014_SB]
(A) J2/2m (B) J2/m (C) 2J2/m (D) 6J2/m

KVPY_ SA STREAM # 197


7. Two uniform plates of the same thickness and area but of different materials, one shaped like an
isosceles triangle and the other shaped like a rectangle are joined together to form a composite body
as shown in the figure. If the centre of mass of the composite body is located at the midpoint of their
common side, the ratio between masses of the triangle to that of the rectangle is
[KVPY_2015_SB] [1 Mark]

(A) 1 : 1 (B) 4 : 3 (C) 3 : 4 (D) 2 : 1

8. Two identical objects each of radii R and masses m 1 and m 2 are suspended using two strings of equal
length L as shown in the figure (R<<L). The angle θ which mass m2 makes with the vertical is approximately
[KVPY_2015_SB] [1 Mark]

m1 m2

m1 R 2m1 R 2m2 R m2 R
(A) (B) (C) (D)
(m1  m2 ) L (m1  m2 ) L (m1  m2 ) L (m1  m2 ) L

9. The distance between the vertex and the center of mass of a uniform solid planar circular segment of
angular size  and radius R is given by.. [KVPY_2015_SB] [2 Marks]

4 sin( / 2) sin( / 2) 4   2
(A) R (B) R (C) R cos  (D) R cos( )
3   3 2 3
10. A smaller with side b (depicted by dashed lines) is excised from a bigger uniform cube with side  as shown
below such that both cubes have a common vertex P. Let X = a/b. If the centre of mass of the remaining solid
is at the vertex O of smaller cube then X satisfies. [KVPY_2016_SB] [2 Marks]
P
b

O

(A) x3 – x2 – X – 1 = 0 (B) x2 – x –1 = 0
(C) x3 – X2 – X – 1 = 0 (D) X3 – X2 – X + 1 = 0

KVPY_ SA STREAM # 198


EXERCISE - 1
1. (B) 2. (B) 3. (C) 4. (D) 5. (B) 6. (D) 7. (D)
8. (A) 9. (B) 10. (B) 11. (C) 12. (A) 13. (A) 14. (D)
15. (B) 16. (A) 17. (B) 18. (A) 19. (C) 20. (A) 21. (B)
22. (B) 23. (A) 24. (A) 25. (C) 26. (D) 27. (B) 28. (A)
29. (B) 30. (C) 31. (A) 32. (B) 33. (D) 34. (A) 35. (C)
36. (B) 37. (C) 38. (D) 39. (D) 40. (C)

EXERCISE - 2
1. (D) 2. (B) 3. (A) 4. (B) 5. (B) 6. (C) 7. (C)
8. (B) 9. (A) 10. (A)

1. Taking C as origin and x & y–axes as shown in figure.


Due to symmetry about y–axis
x cm = 0

m1y1  m2y 2 
ycm =  m  m 
 1 2 

 ( 6 )2   4(6) 
   – [ (2)2 (8)]
 2   3 
 
= ( 6 )2 (m  Area)
 (2)2
2

8(18 2  4 )
= = 8 cm.
(18 2  4)

2. The y - coordinate of centre of mass is


m1y1  m 2 y 2  m 3 y 3  m 4 y 4  m 5 y 5
y =
m1  m 2  m 3  m 4  m 5

 3   3   3 
2m  a   2m(0)  2m(0)  m  a  m  a
 2   4   4 
     
= 8m
9a
=
16 3

3. Removed part has more mass near its circumference (away from O). So after its removal centre of mass
should move nearer to O.
i.e. a < d

4. Initially coordinate of centre of mass is (0, 0)

When solid ball touches the lower surface of hollow sphere


m(2R)  mx0
yCm = =R
mm
So finally coordinate of COM (0, –R)
displacement is (0 – (–R)) = R

KVPY_ SA STREAM # 199


5. Stable equilibrium will be when c.m. is below the point of suspension.
m1x1  m 2 x 2
6. xcm = y
m1  m 2
m1 = mass of square plate
=m x
x1 = c. m. of square plate = 0 2
,0

m2 = mass of removed part

 2 
 
m  4  
= – 2  2  =– m
  8
 
x2 = c.m. of removed part
 4   1  4 
=    =  
2 3  2  2  3 

m  4   4
 .1      
8 2 3  3
 xcm = xcm =– 
 2 (8   )
m m
8
7. With respect to centre of mass of two body system, both the bodies move in opposite direction or both
remain at rest.

If ri denotes the position vector of ith particle (of mass mi) with respect to centre of mass, then
 
 mi ri  0 or  mi v i  0
 
For two body system m1v 1  m 2 v 2  0
Where v denotes velocity of particle with respect to centre of mass
 m 
v1   2 v 2
m1
So these two bodies can not move in the same direction with respect to centre of mass
|m2v2| = |m1v1|
If heavier body moves faster, then
|m2v2| > |m1v1|  m2 > m1 and v 2 > v 1
But by conservation of linear momentum this is not possible.
 
 m1v 1  m 2 v 2  0 .... (1)
 
If v 1  0 and v 2  0
From (i) this is not possible.

1
8. KEA/CM = .1.(v A/CM)2 = 2 Joules
2
 VA/CM = 2 m/s.
Let ; COM move towards +ve x-direction.

Then, v A / CM = 2 î
 


v B/CM =  î ( Use ; v CM =
m1v1  m 2 v 2
m1  m 2
) KESystem =
1
2
 
2 1
 
.1. 2 î  v CM + .2  î  v CM
2

2

=
1
2
 
 1

4  v 2CM  2.2 î.v CM  .2. 1  v CM
2
2 
 2.î.v CM 
 
= (2  2  2 î.v CM )  (1  4  2 î.v CM )
= 9J Ans.

KVPY_ SA STREAM # 200


20
9. vp = = 4 m/s
5
(x cm ) = 0
40(v 1 + 4) – 40(v 2 – 4) + 20. (4) = 0
40(v 1 – v 2 + 10) = 0
v 1 – v 2 = – 10
If v 1 = 10 m/s, v 2 = zero
v 1 = 20 m/s, v 2 = 10 m/s
v 1 = 15 m/s, v 2 = 5 m/s

10. Friction force between wedge and block is internal i.e. will not change motion of COM. Friction force on
the wedge by ground is external and causes COM to move towards right. Gravitational force (mg) on
block brings it downward hence COM comes down.

11. LMC +
1 × 6 – 1 × 2 = (1 + 1 + 2)v
v = 1 m/s
W f = k f – ki
1 1 1
= × 4 × 12 – × 1 × 22 – × 1 × 62
2 2 2
= 2 – 2 – 18 = – 18 J.

12. Conserving linear momentum in horizontal direction w.r.t. ground


Mu – mu = (M + m) v
(M  m)
v= u
(M  m)

u (M  m) u (M  m ) 1
It v >  u>  > ,  M > 3m
2 (M  m) 2 (M  m ) 2

1
13. mu2 = Rt
2
mu = (M + m)v
1 1
mu2 – (M + m)v 2 = Rt'
2 2

M
t' = t.
Mm

14. Since F is increasing, hence acceleration is also increasing.


5


p = F dt
0
= Area under F – t graph

1
= × 5 × 20 N – s = 50 N s.
2
15.

F2  F1
Acceleration of centre of mass = m  m
1 2
With respect to center of mass.
Net force acting on m 1 and m 2

KVPY_ SA STREAM # 201


Applying work energy the same on system.
1
Fx 1 + Fx 2 – K(x 1+x 2)2 = 0 – 0
2

2F 2(m1F2  m 2F1 )
x max = x 1 + x 2 = =
k k(m1  m 2 )

16. Let u be velocity of the particle relative to centre of the hemisphere at an angular displacement ()
tangentially.

As the external force on the system is zero. m


From conservation of linear momentum,
3m
Pf = Pi
u v
 3mv + m(v – u cos) = 0
4v
 u= .
cos 
Hence, angular velocity of the particle relative to the centre of hemisphere
u 4v
= = .
R R cos 

17. (Moderate) Work done by spring on the block of mass m1 = W 1 = change in kinetic energy
1 1 1
= m v 2– m (0)2 = m1 v 12
2 1 1 2 1 2
1
Similarly W 2 = mv2
2 2 2
1
W1 m1v12
2
= 1
W2
m2 v 2 2
2
from conservation of momentum
m1v1 = m v
2 2
W1 v1 m2
 = = .
W2 v2 m1
18. At highest point
2m m
mu = v – 2u
3 3
2u 2 5u
mu + = v  v=
3 3 2
5R R 7R
total horizontal distance = + =
2 2 2 4

19. For the just completing the circular motion, minimum velocity at bottom in
vB = 5gR
Energy conservation b/w point A and B
1
MgH + 0 = 0 + mv B2
2
1 5R
MgH = m (5gR)  H=
2 2

KVPY_ SA STREAM # 202


20. All energy is transfered to other particles.

21. Let the speeds of balls of mass m and 2m after collision be v 1 and v 2 as shown in figure.
Applying conservation of momentum
u
mv 1 + 2mv 2 = mu and –v 1 + v 2 =
2
u
solving we get v 1 = 0 and v2 =
2
u
Hence the ball of mass m comes to rest and ball of mass 2m moves with speed .
2
2 r 4r
t= =
u/ 2 u
22. v1 > 0
m1u1  m 2u2  m 2 e(u1  u2 ) m1
 m1  m 2
>0  m1 – m2e > 0  m2
> e.

23.

(m1  m 2 ) u 2m 2 (u)
v1 =  0
m1  m 2 m1  m 2
 m 1 = 3m 2
Given m 1 = 3kg
 m 2 = 1 kg.

1 m1 m 2 100
24.  U = (V1 – V2)2 =
2 (m1  m 2 ) 3
2m .m 100
(V1 – V2)2 × =
2 (m  2m) 3
putting m = 1 kg :
(V1 – V2) = 10 m/sec.

AlternateSolution:
When deformation is maximum both the particles are moving with same velocity . So applying momentum
conservation.
m1v 1 + m2v2 = m1v 1’ + m2v1’
m1v1  m 2 v 2
v 1’ =
m1  m 2
Applying energy conservation:
1 1 1
m v 2 + m v 2 = (m 1 + m 2) (v 1´ )2 + U deformation
2 1 1 2 2 2 2
1 m1m 2 100
 U deformation= m1  m2  × (v 1 – v 2) = 3 v 1 – v 2 = 10m/sec.
2
2

26. With respect to elevator,


initial velocity of ball = 5m/s, downwards
Let velocity of ball just before striking the elevator be v,
then v 2 = 52 + 2 × 10 × 100
= 25 + 2000 = 2025
v = 45 m/s
Since the collision is elastic, velocity of approach = velocity of separation
 velocity of separation = 45 m/s
 velocity of ball after collision = 50 m/s (w.r.t ground)

KVPY_ SA STREAM # 203


 m1  m 2  2m 2u1
27. VA =  m  m  u1 

 1 2  m 1  m2

 10  38  2  38  3
=   15  = – 4 m/s.
 10  38  10  38

28. Let 'n' be the required number of bullets.

Momentum conservation :
u
Mu – n(mu) = (M + nm)
2
M  nm M
 M – nm =  3nm = M  n= Ans.
2 3m

5
29. 108 km/h = 108 × = 30 m/s
18

30m/s 10m/s v1 v2
Train Cart Train Cart
M m M m

From momentum conservation


M × 30 + m × 10 = Mv 1 + mv 2 ...(i)
e × velocity of approach = velocity of separation
1
 × (30 – 10) = v 2 – v 1
2
 10 = v 2 – v 1 ...(ii)
On solving ,

10
40  m .
M  40  m  10 M
v2 = m/s = m  40 m/s
Mm 1
M

30. When separation r = 2R, there is no compression so ellastic P.E. = 0. At this position, pressing force
is also zero so slope of the graph also must be zero.
When separation is minimum, elastic P.E. is maximum
So, correct Ans. is (C).

31. As a result of collision of the particle with the wall, the component of velocity (z-component) normal to the
wall is changed.
The z-component of velocity reverses in direction with lesser magnitude due to inelastic nature of collision.
Hence 2 î + 3 ĵ + 3k̂ is the only possible answer..

32.
(After collision)
v = vA + vB .......(1)
1 vB – v A
=
2 v
v
v B – vA = .......(2)
2

KVPY_ SA STREAM # 204


3v
vB =
4
v
vA =
4
 3v 
IB = m  – 0  = 3mv
 4  4
 v  2  3 v  2 
1 1 3
loss in K.E. = mv – m  4    4   =
2
mv 2.
2 2      16

33. Velocity along the plane does not change


So 3 Sin 60º = Vf sin 30º
 Vf = 3 m/s > 3 m/s
Which is impossible
34. The larger sphere will move along line of impact. y
AB e = 0, velocity of larger sphere m 2m
mv cos  v cos 
v = = r 3r
m  2m 3 x
velocity of larger sphere O

= v cos î – v sin  ĵ
v v 8 2 2
= cos2  î – sin  cos  ĵ = î – ĵ
3 3 27 27

35.

v B = ucos2 i – u cos sin j

VA = usin2i +usin cos j.


36. For the duration of collision
Impulse = change in momentum
Speeds of both the balls are same just before the collision with ground. (  v 2 = 2gh)
For elastic collision of first ball with ground velocity of first ball is reversed
  
1  mv  mu
  
= m (  u)  m u = 2 m u
For the second ball

   1
2  0  mu = 1  I2 =
2 2

KVPY_ SA STREAM # 205


37. Impulse =  F dt

1
= Area under curve = (2) (2) = 2 kg-m/sec.
2

38. Let speed of cube at bottom = v


applying work energy theorum

1 1
mv 2 – m(o)2 = mg (R)
2 2
v 2 = 2 gR = 2 × 10 × .45 = 9
v=3
Impulse by gravitation system = P
= 40×10–3 × 3
= 120 × 10–3 = .12 kg/ms

39. Apply conservation of momentum,

mu cos45º = mv cos45º + Nt


mu sin45º = –mv sin45º + Nt

v sin 45 º
e=
u sin 45 º

 = 0.25.

40. The horizontal component of velocity of sand just before falling on the
cart is v s = 0.
The horizontal speed of cart = v C (constant).
The rate of mass falling on cart = µ (constant).
Horizontal force exerted by falling sand on cart = µ v rel = µ (v c – v s) = µ v c
 µ and v c are constant, the horizontal force is constant.

KVPY_ SA STREAM # 206


RI GI D BODY DY N AM I CS

1. R I G I D B O DY :
Rigid body is defined as a system of particles in which distance between each pair of particles
remains constant (with respect to time). Remember, rigid body is a mathematical concept and any
system which satisfies the above condition is said to be rigid as long as it satisfies it.

 A & B are beads which move on a circular fixed ring

 If a system is rigid, since there is no change in the distance between any pair of particles of the
system, shape and size of system remains constant. Hence we intuitively feel that while a
stone or cricket ball are rigid bodies, a balloon or elastic string is non rigid.
But any of the above system is rigid as long as relative distance does not change, whether it is
a cricket ball or a balloon. But at the moment when the bat hits the cricket ball or if the balloon
is squeezed, relative distance changes and now the system behaves like a non-rigid system.

 For every pair of particles in a rigid body, there is no velocity of separation or approach between
the particles. i.e. any relative motion of a point B on a rigid body with respect to another point
A on the rigid body will be perpendicular to line joining A to B, hence with respect to any
particle A of a rigid body the motion of any other particle B of that rigid body is circular motion.
 
Let velocities of A and B with respect ground be VA and VB respectively in the figure below..

A VA
1
B

VB 2

KVPY_ SA STREAM # 207


If the above body is rigid V A cos 1 = VB cos 2 (velocity of approach / separation is zero)
VBA = relative velocity of B with respect to A.
VBA = VA sin 1 + VB sin 2 (which is perpendicular to line AB)
B will appear to move in a circle to an observer fixed at A.

 W.r.t. any point of the rigid body the angular velocity of all other points of the that rigid body is same.

 Suppose A, B, C is a rigid system hence during any motion sides AB, BC and CA must rotate through
the same angle. Hence all the sides rotate by the same rate.

From figure (i) angular velocity of A and B w.r.t. C is ,


From figure (ii) angular velocity of A and C w.r.t. B is ,

I. Pure Translational M otion :


A body is said to be in pure translational motion, if the displacement of each particle of the system is

same during any time interval. During such a motion, all the particles have same displacement ( s) ,
 
velocity ( v ) and acceleration (a) at an instant.
Consider a system of n particle of mass m1, m2, m3, ...... mn under going pure translation.
then from above definition of translational motion

    
a1  a2  a3  ...... an = a (say)
    
and v1  v 2  v 3  ...... v n = v (say)
From Newton's laws for a system.
   
Fext = m1a1 + m2a2 + m3a3 + ......................

 
Fext = M a

KVPY_ SA STREAM # 208


Where M = Total mass of the body
   
P = m1v1 + m2 v 2 + m3 v 3 + ......................
 
P = Mv
1 1 1
Total Kinetic Energy of body = m1v 12 + m2v 22 + ................. = Mv 2
2 2 2

II. Pure Rota tiona l M ot ion :

Figure shows a rigid body of arbitrary shape in rotation about a fixed axis, called the axis of rotation.
Every point of the body moves in a circle whose center lies on the axis of rotation, and every point
moves through the same angle during a particular time interval. Such a motion is called pure rotation.
We know that each particle has same angular velocity (since the body is rigid.)
so, v 1 = r1, v 2 = r2, v 3 = r3 ...... v n = rn
1 1
Total Kinetic Energy = m v 2+ m v 2 + ..........................
2 1 1 2 2 2
1
= [m 1r12 + m 2r22 + ..........................] 2
2
1
= 2 Where  = m 1r12 + m 2r22 + ............. (is called moment of inertia)
2
 = angular speed of body.

III. Com bine d Tra nsla t iona l a nd Rota tiona l M ot ion :


A body is said to be in combined translation and rotational motion if all point in the body rotates about
an axis of rotation and the axis of rotation moves with respect to the ground. Any general motion of a
rigid body can be viewed as a combined translational and rotational motion.

Example 1.
A body is moving down into a well through a rope passing over a fixed pulley of radius 10 cm. Assume
that there is no slipping between rope & pulley. Calculate the angular velocity and angular acceleration
of the pulley at an instant when the body is going down at a speed of 20 cm/s and has an acceleration
of 4.0 m/s2.
Solution :
Since the rope does not slip on the pulley, the linear speed v of the rim of the pulley is same as the
speed of the body.
The angular velocity of the pulley is then
20 cm / s
 = v/r = = 2 rad/s
10 cm

4.0 m / s 2
and the angular acceleration of the pulley is  = a/r = = 40 rad/s2.
10 cm

KVPY_ SA STREAM # 209


Example 2.
A disc rotates with a uniform angular acceleration of 2.0 rad/s 2 about its axis. If the disc starts from
rest, how many revolutions will it make in the first 10 seconds?
Solution :
The angular displacement in the first 10 seconds is given by
1 1
 = 0t +  t2 = (2.0 rad/s2) (10 s)2 = 100 rad.
2 2
As the wheel turns by 2 radian in each revolution, the number of revolutions in 10 s is
100
n= = 16.
2
Example 3.
The wheel of a motor, accelerated uniformly from rest, rotates through 5 radian during the first second.
Calculate the angle rotated during the next second.
Solution :
As the angular acceleration is constant, we have
1 1
 = 0t +  t 2 =  t 2.
2 2
1
Thus, 5 rad =  (1s)2
2
 = 10 rad/s2 or  = 10 rad/s2
The angle rotated during the first two seconds is
1
= × (10 rad/s2) (2s)2 = 20 rad.
2
Thus, the angle rotated during the 2nd second is
20 rad – 5 rad = 15 rad.
Example 4.
Starting from rest, a fan takes four seconds to attain the maximum speed of 400 rpm (revolution per
minute). Assuming uniform acceleration, calculate the time taken by the fan in attaining half the maximum
speed.
Solution :
Let the angular acceleration be . According to the question,
400 rev/min = 0 +  4 ...........(i)
Let t be the time taken in attaining the speed of 200 rev/min which is half the maximum.
Then, 200 rev/min = 0 + t ...........(ii)
Dividing (i) by (ii), we get,
t = 2 sec.

Example 5.
The motor of an engine is rotating about its axis with an angular velocity of 120 rev/minute. It comes to
rest in 10 s, after being switched off the engine. Assuming uniform angular deceleration, find the
number of revolutions made by it before coming to rest.
Solution :
The initial angular velocity = 120 rev/minute = (4) rad/s.
Final angular velocity = 0.
Time interval = 10 s.
Let the angular acceleration be . Using the equation  = 0 + t, we obtain
 = ( – 4/10) rad/s2
The angle rotated by the motor during this motion is

1 2  rad  1  4 rad 
 =0t + t =  4   (10s) –  2
 (10s)2
2  s  2  10 s 
= 20 rad = 10 revolutions.
Hence the motor rotates through 10 revolutions before coming to rest.

KVPY_ SA STREAM # 210



2. M OM EN T OF I N ERT I A ( I ) ABOU T AN AX I S :
(i) Moment of inertia of a system of n particles about an axis is defined as 
m 1 r12 + m 2 r22 + ..................+ m n rn 2
n

i.e. =  mr
i1
i i
2

where, ri = It is perpendicular distance of mass m i from axis of rotation


S units of Moment of nertia is Kgm 2.
Moment of inertia is a scalar positive quantity.

(ii) For a continuous system :

r
2
= (dm)

where dm = mass of a small element


r = perpendicular distance of the mass element dm from the axis

Moment of Inertia depends on :


(i) density of the material of body
(ii) shape & size of body
(iii) axis of rotation
In totality we can say that it depends upon distribution of mass relative to axis of rotation.

N ot e :  Moment of inertia does not change if the mass :


(i) is shifted parallel to the axis of the rotation because r i does not change.
(ii) is rotated about axis of rotation in a circular path because ri does not change.

Example 6.
Two particles having masses m 1 & m 2 are situated in a plane
perpendicular to line AB at a distance of r 1 and r2 respectively
as shown.
(i) Find the moment of inertia of the system about
axis AB ?
(ii) Find the moment of inertia of the system about
an axis passing through m 1 and perpendicular to
the line joining m 1 and m 2 ?
(iii) Find the the moment of inertia of the system about an
axis passing through m 1 and m 2 ?
(iv) Find moment of inertia about axis passing through centre of mass and perpendicular to line joining
m 1 and m 2
Solution
(i) Moment of inertia of particle on left is 1 = m 1r12.
Moment of Inertia of particle on right is 2 = m 2r22.
Moment of Inertia of the system about AB is  = 1+ 2 = m 1r22 + m 2r22
(ii) Moment of inertia of particle on left is 1 = 0
Moment of Inertia of particle on right is 2 = m 2(r1 + r2)2.
Moment of Inertia of the system about AB is  = 1+ 2 = 0 + m 2(r1 + r2)2
(iii) Moment of inertia of particle on left is 1 = 0
Moment of Inertia of particle on right is 2 = 0
Moment of Inertia of the system about AB is  = 1+ 2 = 0 + 0

KVPY_ SA STREAM # 211


 r1  r2 
(iv) Centre of mass of system r CM = m2  m  m  = Distance of centre mass from mass m
 1
 1 2 

 r1  r2 
Distance of centre of mass from mass m 2 = m1 m  m 

 1 2 
2 2
 r1  r2   r1  r2 
So moment of inertia about centre of mass = cm = m1 m 2  + m 2  m1 
m1  m 2   m m 
   1 2 

m1m 2
CM = (r + r )2
m1  m 2 1 2
Example 7. Four particles each of mass m are kept at the four corners of a
square of edge a. Find the moment of inertia of the system
about a line perpendicular to the plane of the square and
passing through the centre of the square.
Solution : The perpendicular distance of every particle from the given line is a/ 2 .

1
The moment of inertia of one particle is, therefore, m(a/ 2 )2 = ma2. The moment of inertia
2
of the system is, therefore,
1
4× ma2 = 2ma2.
2

Example 8. Three particles, each of mass m, are situated at the vertices of


an equilateral triangle PQR of side a as shown in figure.
Calculate the moment of inertia of
the system about (i) the line PX perpendicular to PQ in the plane of PQR.
(ii) One of the sides of the triangle PQR
(iii) About an axis passing through
the centroid and perpendicular to plane of the triangle PQR.
Solution (i) Perpendicular distance of P from PX = 0
Perpendicular distance of Q from PX = a
Perpendicular distance of R from PX = a/2
Thus, the moment of inertia of the particle at P = 0, of the particle at Q - ma2, and of the particle
at R = m(a/2)2. The moment of inertia of the three-particle system about PX is
5 ma2
0 + ma2 + m(a/2)2 =
4
Note that the particles on the axis do not contribute to the moment of inertia.
(ii) Moment of inertia about the side PR = mass of particle Q × square of perpendicular distance
2
 3  2
of Q from side PR, PR = m  a  = 3ma

 2  4

a
(iii) Distance of centroid from all the particle is , so moment of inertia about an axis and
3

2
 a 
passing through the centroic perpendicular plane of triangle PQR = R = 3m   = ma2

 3

Example 9. Calculate the moment of inertia of a ring having mass M, radius R and having uniform mass
distribution about an axis passing through the centre of ring and perpendicular to the plane of
ring ?

KVPY_ SA STREAM # 212


 (dm)r
2
Solution =
Because each element is equally distanced from the axis so r = R


2 2
= R dm  MR
 = MR2

(Note : Answer will remain same even if the mass is nonuniformly distributed because  dm  M always.)
Example 10. Calculate the moment of inertia of a uniform rod of mass M and length  about an axis 1,2,3 and 4.
1 2

com
3
d
4


M  M 2
 (dm)r    dx  x
2 2
Solution (1)= = =
0 3

/2
 M  2 M 2
(2)=  (dm) r = 2
  dx  x =
 / 2  
 12
(3)= 0 (axis 3 passing through the axis of rod)

(4)= d  (dm )  Md
2 2

Example 11. Determined the moment of inertia of a uniform rectangular plate


of mass, side 'b' and ''' about an axis passing through the
edge 'b' and in the plane of plate.
Solution : Each section of dm mass rod in the rectangular plate
has moment of inertia about an axis passing through
dm  2
edge 'b' d =
3 dm

2 M 2
So  =  d 
3 
dm =
3

Example 12. Find out the moment of Inertia of figures shown each having mass M, radius R and having
uniform mass distribution about an axis passing through the centre and perpendicular to the
plane ?

(a) R (b) R (c)


x x  R
x

Solution : MR2 (infact M.I. of any part of mass M of a ring of radius R about axis passing through geometrical
centre and perpendicular to the plane of the ring is = MR 2 )


(iii) Moment of inertia of a large object can be calculated by integrating M..of an element of the object:

=  d element

where d = moment of inertia of a small element


Element chosen should be such that : either perpendicular distance of axis from each point of the
element is same or the moment of inertia of the element about the axis of rotation is known.

KVPY_ SA STREAM # 213


Example 13. Determine the moment of Inertia of a uniform disc having mass M, radius R about an axis
passing through centre & perpendicular to the plane of disc ?

Solution : =  d ring

element - ring d = dmr2


M
dm = 2rdr (here we have used the
R 2
uniform mass distribution)
R
M
 =  R
0
2 . (2rdr).r2

MR 2
 =
2

Example 14. Calculate the moment of inertia of a uniform hollow cylinder of mass
M, radius R and length  about its axis.
Solution : Moment of inertia of a uniform hollow cylinder is mass = dm


  (dm)R 2
= mR2


3. T WO I M PO RTAN T T H E OR EM S O N M O M E N T OF I N E RT I A
(i) P e rp e n d icu la r Axis Th e ore m [Only applicable to plane
laminar bodies (i.e. for 2-dimensional objects only)].

If axis 1 & 2 are in the plane of the body and perpendicular to each other.
Axis 3 in perpendicular to plane of 1 & 2 .
Then, 3 = 1 + 2

 The point of intersection of the three axis need not be center of mass, it can be any point in the plane of body
which lies on the body or even outside it.

Example 15. Calculate the moment of inertia of a uniform disc of mass M


and radius R about a diameter.

Solution : Let AB and CD be two mutually perpendicular diameters of the disc.


Take them as X and Y-axes and the line perpendicular to the plane of
the disc through the
1
centre as the Z-axis. The moment of inertia of the ring about the Z-axis is  = MR2. As the
2
disc is uniform, all of its diameters are equivalent and so x = y, From perpendicular axes
theorem,

z MR 2
z = x + y. Hence x = = .
2 4

KVPY_ SA STREAM # 214


Example 16. Two uniform identical rods each of mass M and length  are
joined to form a cross as shown in figure. Find the moment of
inertia of the
cross about a bisector as shown dotted in the figure.
Solution Consider the line perpendicular to the plane of the figure through the centre of the cross. The

M 2
moment of inertia of each rod about this line is and hence the moment of inertia of the
12

M 2
cross is . The moment of inertia of the cross about the two bisector are equal by symmetry
6
and according to the theorem of perpendicular axes, the moment of inertia of the cross about

M 2
the bisector is .
12
Example 17. In the figure shown find moment of inertia of a plate having 4 2
mass M, length  and width b about axis 1,2,3 and 4. Assume 1
that mass is uniformly distributed.
Solution Moment of inertia of the plate about axis 1 C
(by taking rods perpendicular to axis 1) I 1 = Mb2 / 3 3 b
Moment of inertia of the plate about axis 2
(by taking rods perpendicular to axis 2) I 2 = M2 / 12 
Moment of inertia of the plate about axis 3
(by taking rods perpendicular to axis 3) I 3 = Mb2 / 12
Moment of inertia of the plate about axis 4
(by taking rods perpendicular to axis 4) I 4 = M2 / 3

Example 18. Find the moment of Inertia of a uniform disc of mass M and radius R about a diameter.
Solution : Consider x & y two mutually perpendicular diameters of the ring.
x + y = z
x = y (due to symmetry)

MR 2
z =
2

MR 2
x = y =
4


( ii) Pa ra llel Axis Theorem (Applicable to planer as well as 3 dimensional objects):

If  = Moment of Inertia of the object about axis AB


cm = Moment of Inertia of the object about an axis
passing through centre of mass and parallel to axis 

M = Total mass of object


d = perpendicular distance between axis AB about which
moment of nertia is to be calculated & the one passing
through the centre of mass and parallel to it.
 = cm + Md2

KVPY_ SA STREAM # 215


Example 19. Find out relation between I1 and I2 .
I1 and I2 moment of inertia of a rigid body mass m about an axis as shown in figure.

a b
COM

I1 I2

Solution : Using parallel axis theorem I1 = IC + ma2 .............. (1)


I2 = IC + mb2 ............... (2)
From (1) and (2) I1 – I2 = m(a2 – b2)

Example 20. Find the moment of inertia of a uniform sphere of mass m and radius R about a tangent if the
spheres (i) solid (ii) hollow
Solution (i) Using parallel axis theorem
I = ICM + md2
for solid sphere
2
ICM = mR2 , d = R
5 R R
7
I= mR2
5
(ii) Using parallel axis theorem
I = ICM + md2
for hollow sphere
2 solid sphere
ICM = mR2 , d = R hollow sphere
3

5
I= mR2
3
Example 21 Calculate the moment of inertia of a hollow cylinder of mass M and radius R about a line
parallel to the axis of the cylinder and on the surface of the cylinder.

Solution The moment of inertia of the cylinder about its axis = MR2.

Using parallel axes theorem,  = 0 + MR2 = MR2 + MR2 = 2 MR2.


Similarly, the moment of inertia of a hollow sphere about a tangent is
2 5
MR2 + MR2 = MR2.
3 3

Example 22. Find out the moment of inertia of a semi circular disc about an axis passing through its centre
of mass and perpendicular to the plane?

KVPY_ SA STREAM # 216


Solution : Moment of inertia of a semi circular disc about an axis passing through centre and perpendicular
MR 2
to plane of disc, I =
2
2
Using parallel axis theorem I  I CM  Md , d is the perpendicular distance between two parallel
axis passing through centre C and COM.
2
MR 2 4R MR 2  4R 
I= , d=  = ICM +M  
2 3 2  3 

 MR 2  4R  
2

ICM =   M  
 2  3  
 P
Example 23. Find the moment of inertia of the two uniform joint rods having ×
mass m each about point P as shown in figure. Using parallel 
axis theorem.
m 2
Solution : Moment of inertia of rod 1 about axis P , I1 =
3
1 P
2 2
m  
Moment of inertia of rod 2 about axis P, I2 = + m 5 
12  2 
So moment of inertia of a system about axis P , COM 5
2
m 2 m 2   m 2 2
I = I1 + I2 = + + m 5   I=
3 12  2  3


List of som e use ful f orm ule :

Object Moment of Inertia

Solid Sphere

2
MR 2 (Uniform)
5

Hollow Sphere

2
MR 2 (Uniform)
3

Ring.

MR2 (Uniform or Non Uniform)

KVPY_ SA STREAM # 217


Disc

MR 2
(Uniform)
2

Hollow cylinder

MR2 (Uniform or Non Uniform)

Solid cylinder

MR 2
(Uniform)
2

Thin rod

ML2
(Uniform)
3

ML2
(Uniform)
12

Two thin rod

2m  2
(Uniform)
3

Rectangular Plate

M(a 2  b 2 )
= (Uniform)
12

KVPY_ SA STREAM # 218


Square Plate

Ma2
AB = CD = DF = (Uniform)
12

Square Plate

Ma2
(Uniform)
6

Cuboid

M(a 2  b 2 )
(Uniform)
12

4. R A D I U S O F G Y R AT I O N :
As a measure of the way in which the mass of rigid body is distributed with respect to the axis of rotation, we
define a new parameter, the radius of gyration (K). It is related to the moment of inertia and total mass of the body.
 = MK2
where  = Moment of Inertia of a body
M = Mass of a body
K = Radius of gyration

K=
M
 Length K is the geometrical property of the body and axis of rotation.
S.. Unit of K is meter.

Example 24. Find the radius of gyration of a solid uniform sphere of radius R about its tangent.
2 7 7
Solution = mR 2  mR 2 = mR 2 = mK2  K= R
5 5 5
Example 25. Find the radius of gyration of a hollow uniform sphere of radius R about its tangent.
5
Solution Moment of inertia of a hollow sphere about a tangent, I = MR2
3

5 5
MK2 = MR2  K= R
3 3

KVPY_ SA STREAM # 219



5. M OM EN T OF I N ERT I A OF BODI ES WI T H CU T :

Example 26
A uniform disc of radius R has a round disc of radius R/3 cut as shown in
Fig. .The mass of the remaining (shaded) portion of the disc equals M.
Find the moment of inertia of such a disc relative to the axis passing
through geometrical centre of original disc and perpendicular to the plane
of the disc.
Solution
Let the mass per unit area of the material of disc be . Now the empty space can be considered as having
density – and .
Now 0 =  + –
 = ( R2)R2/2 = M.I. of  about o
 (R / 3)2 (R / 3)2
– = + [–(R/3)2] (2R/3)2 = M.I. of – about o
2
4
 0 = MR2 Ans.
9

Example 27.
Find the moment of inertia of a uniform disc of radius R1 having an empty symmetric annular region of
radius R2 in between, about an axis passing through geometrical centre and perpendicular to the disc.

M  R14  R 24 
Sol. =   =  ×  

 (R12  R 22 )  2 

=
 2
M R1  R 2
2
 Ans.
2


6. T ORQU E :
Torque represents the capability of a force to produce change
in the rotational motion of the body.
6 .1 Torque a bou t a point :
   
Torque of force F about a point   rF

Where F = force applied
P = point of application of force
Q = Point about which we want to calculate the torque.

r = position vector of the point of application of force w.r.t. the point about
which we want to determine the torque.

 = r F sin = rF = rF
Where  = angle between the direction of force and the position vector of P wrt. Q.
r = r sin  = perpendicular distance of line of action of force from point Q ,it is
also called force arm.
 
F  = F sin  = component of F perpendicular to r
S unit of torque is Nm
 Torque is a vector quantity and its direction is determined using right hand thumb rule and its always
perpendicular to the plane of rotation of the body.

KVPY_ SA STREAM # 220


Example 28. A particle of mass M is released in vertical plane from a point P at x = x 0 on the x-axis it falls
vertically along the y-axis. Find the torque  acting on the particle at a time t about origin ?
Solution : Torque is produced by the force of gravity.

 = r F sin  k̂
or  = rF  x 0mg
x0
= r mg = mgx 0 k̂
r
Example 29. A particle having mass m is projected with a velocity v 0 from a
point P on a horizontal ground making an angle  with
horizontal. Find out the
torque about the point of projection acting on the particle V0
when it is at its maximum height ? 
2 P Q
R v sin 2
Solution :  = rFsin = mg = 0 mg
2 2g
2
mv 0 sin 2
=
2
y
Example 30. Find the torque about point O and A.
(1,1) 30°

F  5 3 î  5 ĵ 60°
    
Solution : Torque about point O,   r0  F , r0  î  ĵ , F  5 3 î  5 ĵ

  ( î  ĵ ) × ( 5 3 î  5 ĵ ) = 5(1  3 ) k̂ x
O A B
    
Torque about point A ,   ra  F , ra  ĵ , F  5 3 î  5 ĵ

  ĵ × ( 5 3 î  5 ĵ ) = 5( 3 ) k̂


6 .2 Torque a bout a n a xis :

 
The torque of a force F about an axis AB is defined as the component of torque of F about any
point O on the axis AB, along the axis AB.
   
In the given figure torque of F about O is 0  r  F
 
The torque of F about AB, AB is component of 0 along line AB.

There are four cases of torque of a force about an axis.:

KVPY_ SA STREAM # 221



Case I : Force is parallel to the axis of rotation, F || AB
AB is the axis of rotation about which torque is required
     
r  F is perpendicular to F , but F || AB , hence r  F is perpendicular to AB .
 
The component of r  F along AB is, therefore, zero.
Case II : The line of force intersects the axis of rotation (F intersect AB)

   
F intersects AB along r then F and r are along the same line. The torque about O is
 
r  F = 0.
Hence component this torque along line AB is also zero.
 
Case III : F perpendicular to AB but F and AB do not intersect.

In the three dimensions, two lines may be perpendicular without intersecting each other.
Two nonparallel and nonintersecting lines are called skew lines.
Figure shows the plane through the point of application of force P that is perpendicular to the
axis of rotation AB. Suppose the plane intersects the axis at the point O. The force F is in this
plane (since F is perpendicular to AB). Taking the origin at O,
  
Torque = r  F = OP × F .
Thus, torque = rF sin  = F(OS)

where OS is the perpendicular from O to the line of action of the force F . The line OS is also
perpendicular to the axis of rotation. It is thus the length of the common perpendicular to the
force and the axis of rotation.
  
The direction of  = OP × F is along the axis AB because AB  OP and AB  F . The

torque about AB is, therefore, equal to the magnitude of  that is F.(OS).

 Thus, the torque of F about AB = magnitude of the force F × length of the common perpendicular to the
force and the axis. The common perpendicular OS is called the lever arm or moment arm of this torque.

Case IV : F and AB are skew but not perpendicular..

Here we resolve F into two components, one is parallel to axis and other is perpendicular to
axis. Torque of the parallel part is zero and that of the perpendicular part may be found, by
using the result of case (III).

KVPY_ SA STREAM # 222


Example 31. Find the torque of weight about the axis passing through point P.

    
Solution   r  F , r  R , F  mg sin 
r and F both are at perpendicular so torque about point P = mgRsin

Example 32. A bob of mass m is suspended at point O by string of length  . Bob is moving in a
orizontal circle find out (i) torque of gravity and tension about point O and O'. (ii) Net
torque about axis OO' .

Solution (i) Torque about point O


Torque of tension (T), ten = 0 (tension is passing through point O)
Torque of gravity mg = mgsin 
Torque about point O'
Torque of gravity mg = mgr r = sin 

Torque of tension mg = mgsin  (along negative ĵ )

Torque of tension ten = Trsin(90+ ) (Tcos = mg)


ten = Trcos
mg
ten = (sin ) cos  = mg sin (along positive ĵ )
cos 
(ii) Torque about axis OO'
Torque of gravity about axis OO' mg = 0 (force mg parallel to axis OO')
Torque of tension about axis OO' ten = 0 (force T is passing through the axis OO')
Net torque about axis OO' net = 0

6 .3 Force Couple :
A pair of forces each of same magnitude and acting in
opposite direction is called a force couple.
Torque due to couple = Magnitude of one force ×
distance between their lines of action.
Magnitude of torque = = F (2d)

KVPY_ SA STREAM # 223


 A couple does not exert a net force on an object even though it exerts a torque.
 Net torque due to a force couple is same about any point.

Torque about A = x 1F + x 2F
= F(x 1 + x 2) = Fd
Torque about B = y 1F – y2F
= F(y1 – y2) = Fd
 If net force acting on a system is zero, torque is same about any point.
 A consequence is that, if F net = 0 and net = 0 about one point, then net = 0 about any point.


6 .4 Point of Applica t ion of Force :
Point of Application of force is the point at which, if net force is assumed to be acting, then it
will produce same translational as well as rotational effect, as was produced earlier.
We can also define point of application of force as a point about which torque of all the forces is zero.

  
Consider three forces F1,F2 ,F3 acting on a body if D is point of application of force then torque of
  
F1  F2  F3 acting at a point D about O is same as the original torque about O

      
    
r1  F1  r2  F2  r3  F3 = r  (F1  F2  F3 )

Example 33. Determine the point of application of force, when forces of 20 N


& 30 N are acting on the rod as shown in figure.

Solution : Nett force acting on the rod F rel = 10N


Nett torque acting on the rod about point C
c = (20 × 0) + ( 30 × 20) = 600 clockwise
Let the point of application be at a distance x from point C
600 = 10 x  x = 60 cm
 70 cm from A is point of Application

KVPY_ SA STREAM # 224


Note : (i) Point of application of gravitational force is known as the centre of gravity.

(ii) Centre of gravity coincides with the centre of mass if value of g is assumed to be constant.
(iii) Concept of point of application of force is imaginary, as in some cases it can lie outside the body.


6 .5 Rot a t ion a bout a f ixe d a xis :
If Hinge = moment of inertia about the axis of rotation (since this axis passes through the hinge,
hence the name Hinge ).


 ext  = resultant external torque acting on the body about axis of rotation
 = angular acceleration of the body.

 ext Hinge = Hinge 
1 2 Hinge x
Rotational Kinetic Energy = .  . 
 2

P  M v CM
  Fixed axis of
Fexternal  M a CM Rotation

Net external force acting on the body has two component tangential and centripetal.
v2
 F C = maC = m = m2 rCM  F t = mat = m rCM
rCM

Example 34. A pulley having radius r and moment of inertia  about its axis
is fixed at the top of an inclined plane of inclination  as shown
in figure. A string is wrapped round the pulley and its free end
supports a block of mass m which can slide on the plane.
Initially, the pulley is rotating at a speed 0 in a direction such
that the block slides up the plane. Calculate the distance moved
by the block before stopping ?
Solution : Suppose the deceleration of the block is a. The linear deceleration of the rim of the pulley is
also a. The angular deceleration of the pulley is  = a/r. If the tension in the string is T, the
equations of motion are as follows:
mg sin  – T = ma and Tr =  = a/r.
Eliminating T from these equations,
a
mg sin –  2 = ma
r

mgr 2 sin
giving, a =
  mr2
The initial velocity of the block up the incline is v = 0 r. Thus, the distance moved by the block
before stopping is
2 2
v2 0 r 2 (  mr 2 ) (  mr2 )0
x= = =
2a 2m r 2 sin  2m g sin

Example 35. The pulley shown in figure has a moment of inertia  about its
axis and its radius is r. Calculate the magnitude of the
acceleration of the two blocks. Assume that the string is light
and does not slip on the pulley.

KVPY_ SA STREAM # 225


Solution : Suppose the tension in the left string is T 1 and that in the right string in T 2. Suppose the block
of mass m 1 goes down with an acceleration  and the other block moves up with the same
acceleration. This is also the tangential acceleration of the rim of the wheel as the string does
not slip over the rim. The angular acceleration of the wheel is, therefore,  = a/r. The equations
of motion for the mass m 1, the mass m 2 and the pulley are as follows :
m 1g – T 1 = m 1a .........(i)
T 2 – m 2g = m 2a .........(ii)
T 1r – T 2r = I = I /r .........(iii)
Putting T 1 and T 2 from (i) and (ii) into (iii),
a
[(m 1g – a) – m 2(g + a)] r = 
r

(m1  m2 )gr 3
which gives a = .
  (m1  m2 )r 2


7. EQU I LI BRI U M
A system is in mechanical equilibrium if it is in translational as well as rotational equilibrium.
For this : Fnet  0

net  0 (about every point)

Y
F1

F2
F5
X

F4 F3

From (6.3), if Fnet  0 then net is same about every point

Hence necessary and sufficient condition for equilibrium is Fnet  0 , net  0 about any one point,

which we can choose as per our convenience. ( net will automatically be zero about every point)

unstable stable Neutral


equilibrium equilibrium equilibrium

The equilibrium of a body is called stable if the body tries to regain its equilibrium position after being
slightly displaced and released. It is called unstable if it gets further displaced after being slightly
displaced and released. If it can stay in equilibrium even after being slightly displaced and released, it
is said to be in neutral equilibrium.

KVPY_ SA STREAM # 226


Example 36. Two boys weighing 20 kg and 25 kg are trying to balance a
seesaw of total length 4 m, with the fulcrum at the centre. If
one of the boys is sitting at an end, where should the other sit ?
Solution : It is clear that the 20 kg kid should sit at the end and the 25 kg kid should sit closer to the
centre. Suppose his distance from the centre is x. As the boys are in equilibrium, the normal
force between a boy and the seesaw equals the weight of that boy. Considering the rotational
equilibrium of the seesaw, the torque of the forces acting on it should add to zero. The forces
are
(a) (25 kg) g downward by the 25 kg boy,
(b) (20 kg) g downward by the 20 kg boy,
(c) weight of the seesaw and
(d) the normal force by the fulcrum.
Taking torques about the fulcrum,
(25 kg)g x = (20 kg)g (2 m) or x = 1.6 m.
Example 37. A uniform rod of mass m = 15 kg leans against a smooth vertical wall making an angle  = 37°
with horizontal. The other ends rests on a rough horizontal floor. Calculate the normal force
2
and the friction force that the floor exerts on the rod. [Take g = 10 m/s ]
S olution : The forces acting on the rod are shown in figure. They are
a) Its weight W,
( ( ( (

b) normal force N1 by the vertical wall,


c) normal force N2 by the floor and
d) frictional force f by the floor.
Taking horizontal and vertical components,
N1 = f ..........( i)
and N2 = mg ..........( ii)
Taking torque about B,
N1 AO) = mg( CB)
(

AB 3 W 4
or, N1 AB) cos = mg 2 sin  or N1 5 = 2 5
(

2
or, N1 = 3 W ..........( iii)
2
The normal force by the floor is N2 = W = 15 kg) 10 m/s = 150 N.
(

2
The frictional force is f = N1 = 3 W = 100 N.


8. AN GU LAR M OM EN T U M ( L )
8 . 1 . Angula r m om e nt um of a p a rt icle a bout a p oint .
  
L = r P  L = rpsin

or L = r × P

or L = P× r

Where P = momentum of particle

r = position of vector of particle with respect to point O about which angular momentum
is to be calculated .
 
= angle between vectors r & P
r= perpendicular distance of line of motion of particle from point O.

P= component of momentum perpendicular to r .
SI unit of angular momentum is kgm 2/sec.

KVPY_ SA STREAM # 227


Example 38. A particle of mass m is projected at time t = 0 from a point O with a speed u at an angle of 45°
to the horizontal. Calculate the magnitude and the direction of the angular momentum of the
particle about the point O at time t = u/g.
Solution : Let us take the origin at P, X-axis along the horizontal and Y-axis along the vertically upward
direction as shown in figure.
For horizontal motion during the time 0 to t,
v x = u cos 45° = u/ 2

u u u2
and x = v xt = . = .
2 g 2g
For vertical motion,

u (1  2 )
v y = u sin 45° = –u= u
2 2

1 2
and y = (u sin 45°) t – gt
2

u2 u2 u2
= – = ( 2 – 1).
2g 2g 2g
The angular momentum of the particle at time t about the origin is
   
L= r × p = mr × v
= m( î x + ĵ y) × ( î v x + ĵ v y) = m ( k̂ xv y – k̂ yv x)

 u2  u u2 u2 
   (1  2 )  ( 2  1)  = mu3
= m k̂  – k̂ .
 2 g  2 2g 2  2 2g

mu3
Thus, the angular momentum of the particle is in the negative Z-direction i.e.,
2 2g
perpendicular to the plane of motion, going into the plane.

Example 39. A particle of mass 'm' starts moving from point (o,d)
with a constant velocity u î . Find out its angular
momentum about origin at this moment what will be
the answer at the later time?


Solution : L = – m d u k̂ .

Example 40. A particle of mass 'm' is projected on horizontal ground with an initial velocity of u making an
angle with horizontal . Find out the angular momentum of particle about the point of projection
when .
(i) it just starts its motion
(ii) it is at highest point of path.
(iii) it just strikes the ground.

u 2 sin 2  u 2 sin 2
Ans. (i) O ; (ii) mu cos ; (iii) mu sin 
2g g

KVPY_ SA STREAM # 228


Solution : (i) Angular momentum about point O is zero.
(ii) Angular momentum about point A.
   A ucos
L  r p
H
L = H × mu cos 
O R B
u 2 sin 2 
L = mu cos Ans. usin
2g
(iii) Angular momentum about point B.
u 2 sin 2
L = R × mu sin mu sin  Ans.
g

8 .2 Angula r m om e nt um of a rigid body rot a t ing a bout f ixe d a xis :
A

r1

r2

rn

Angular momentum of a rigid body about the fixed axis AB is L AB = L1 + L2 + L3 +....... +Ln
L1 = m 1 r1r1 , L2 = m 2 r2r2 , L3 = m 3 r3r3 , Ln = m n rn rn

LAB = m 1 r1r1 + m 2 r2r2 + m 3 r3r3 ........ + m n rn rn

nn n  n 
LAB =  mn (rn )2 ×  
 mn (rn )2   H 
n1  n1 
LAB = H  LH = I H 
LH = Angular momentum of object about axis of rotation.
IH = Moment of Inertia of rigid , body about axis of rotation.
 = angular velocity of the object.

Example 41. Two particles balls A and B, each of mass m, are attached rigidly to the ends of a light rod of
length . The system rotates about the perpendicular bisector of the rod at an angular speed .
Calculate the angular momentum of the individual particles and of the system about the axis of
rotation.
Solution : Consider the situation shown in figure. The velocity of the particle A with respect to the centre

O is v = . The angular momentum of the particle with respect to the axis is
2

  1
L1 = mvr = m     = m2. The same the angular momentum L 2 of the second
 2  2 4
particle. The angular momentum of the system is equal to sum of these two angular momentum
i.e., L = 1/2 m2.
KVPY_ SA STREAM # 229
Example 42. Two small balls of mass m each are attached to a light rod of length , one at its centre and the
other at a free end. The rod is fixed at the other end and is rotated in horizontal plane at an
angular speed . Calculate the angular momentum of the ball at the end with respect to the ball
at the centre.
Solution : The situation is shown in figure. The velocity of the ball A with respect to the fixed end O is
v A =   / 2  and that of B with respect to O is v B = . Hence the velocity of B with respect to
A is v B – v A =   / 2  . The angular momentum of B with respect to A is, therefore,

  1
L = mvr = m   = m2
2 2 4
along the direction perpendicular to the plane of rotation.


8 .3 Conserva t ion of Angula r M om e nt um

 dL
Newton's 2nd law in rotation :  
dt
 
where  and L are about the same axis.
 Angular momentum of a particle or a system remains constant if  ext = 0 about the axis of rotation.
Even if net angular momentum is not constant, one of its component of an angular momentum about an
axis remains constant if component of torque about that axis is zero

Impulse of Torque :  dt  J


J  Charge in angular momentum.

Example 43. A uniform rod of mass m and length can rotate freely on a
smooth horizontal plane about a vertical axis hinged at point
H. A point mass having same mass m coming with an initial
speed u perpendicular to the rod, strikes the rod in-elastically
at its free end. Find out the angular velocity of the rod just after
collision ?
Solution : Angular momentum is conserved about H because no external force is present in horizontal
plane which is producing torque about H.
 m 2  3a
mul =  3  m  
2
 =
  4

Example 44. A uniform rod of mass m 1 and length  lies on a frictionless horizontal plane. A particle of mass
m 2 moving at a speed v 0 perpendicular to the length of the rod strikes it at a distance /3 from
the centre and stops after the collision. Calculate (a) the velocity of the centre of the rod and
(b) the angular velocity of the rod about its centre just after the collision.
Solution : The situation is shown in figure. Consider the rod and
w
the particle together as the system. As there
is no external resultant f orce, the linear
®
r
m om ent um of the syst em wil l rem ains A
0
V
constant. Also there is no resultant external
torque on the system and so the angular
momentum of the system about the any line
(b)
will remain constant. Suppose the velocity of
the centre of the rod is V and the angular
velocity about the centre is .
KVPY_ SA STREAM # 230
(a) The linear momentum before the collision is mv and that after the collision is MV.

 m2 
Thus, m2v 0 = m1V, or V =  m  v 0
 1
(b) Let A be the centre of the rod when it is at rest. Let AB be the line perpendicular to the plane
of the figure. Consider the angular momentum of “the rod plus the particle” system about AB.
Initially the rod is at rest. The angular momentum of the particle about AB is L = m 2v 0 (/3)
After the collision, the particle comes to rest. The angular momentum of the rod
about A is
         
L = L cm + m 1 r0 × V As r0 || V , r0 × V = 0 Thus, L = L cm
Hence the angular momentum of the rod about AB is

m12 m2v m 2 4m 2 v 0
L = = . Thus, = 1  or, =
12 3 12 m1

9. COM BI N ED T RAN SLAT I ON AL AN D ROTAT I ON AL M OT I ON


OF A RI GI D BODY
The general motion of a rigid body can be thought of as a sum of two independent motions. A transla-
tion of some point of the body plus a rotation about this point . A most convenient choice of the point is
the centre of mass of the body as it greatly simplifies the calculations.

 Consider a fan inside a train, and an observer A on the platform.


It the fan is switched off while the train moves, the motion of fan is pure translation as each point on the
fan undergoes same translation in any time interval.
It fan is switched on while the train is at rest the motion of fan is pure rotation about is axle ; as each
point on the axle is at rest, while other points revolve about it with equal angular velocity.
if the fan is switched on while the train is moving, the motion of fan to the observer on the platform is
neither pure translation nor pure rotation. This motion is an example of general motion of a rigid body.
Now if there is an observer B inside the train, the motion of fan will appear to him as pure rotation.
Hence we can see that the general motion of fan w.r.t. observer A can be resolved into pure rotation of
fan as observed by observer B plus pure translation of observer B (w.r.t. observer A)
Such a resolution of general motion of a rigid body into pure rotation & pure translation is not restricted
to just the fan inside the train, but is possible for motion of any rigid system.
9 .1 Kine m a t ics of ge ne ra l m otion of a rigid body :
For a rigid body as earlier stated value of angular displacement () , angular velocity (), angu-
lar acceleration () is same for all points on the rigid body about any other point on the rigid
body.
Hence if we know velocity of any one point (say A) on the rigid body and angular velocity of any
point on the rigid body about any other point on the rigid body (say ), velocity of each point on
the rigid body can be calculated.

since distance AB is fixed VA


  VB = VA + VBA
VBA  AB
B
VBA 
we know that  = r VBA=   rAB
BA
VA
VBA = VBA = rBA
  
in vector form VBA =   rBA A
   rigid body
Now from relative velocity : VBA  VB  VA
      
VB  VA  VBA  VB  VA    r BA
   
similarly aB  a A    rBA [for any rigid system]

KVPY_ SA STREAM # 231


Example 45. Consider the general motion of a wheel (radius r) which can be view on pure translation of its
center O (with the velocity v) and pure rotation about O (with angular velocity )

C  ĵ
  (k̂ )

V î
D B
O

A
       
Find out v AO , v BO , v CO , v DO and v A , v B , v C , v D
  
Solution : v AO =   rAO 


v AO =  (–k̂ ) OA




v AO =  (–k̂ ) r( ĵ ) 

v AO =   r î

similarly v BO = r ( ĵ )

v CO = r ( î )

v DO =r ( ĵ )
  
v A = v O  v AO  v î  r î
  
similarly v B = v O  v BO  v î  r ĵ
  
v C = v O  v CO  v î  r î
  
v D = v O  v DO  v î  r ĵ

9 .2 Pure R olling (or rolling w it hout slid ing) :


Pure rolling is a special case of general rotation of a rigid body with circular cross section (e.g.
wheel, disc, ring, sphere) moving on some surface. Here, there is no relative motion between
the rolling body and the surface of contact, at the point of contact

 A

Here contact point is A & contact surface is horizontal ground. For pure rolling velocity of A
w.r.t. ground = 0  VA = 0 .

 A
V

KVPY_ SA STREAM # 232


From above figure, for pure rolling, velocity of A w.r.t. to plank is zero  VA =V.

r
v

A

From above figure for, pure rolling, velocity of A w.r.t. ground is zero.
v –r = 0
v = r
Similarly a = r

Example 46. A wheel of radius r rolls (rolling without sleeping) on a level road as shown in figure.
B 
r
v

Find out velocity of point A and B


Solution : Contact surface is in rest for pure rolling velocity of point A is zero.
so v = r
velocity of point B = v + r = 2v

9 .3 D yna m ics of ge ne ra l m ot ion of a rigid body :


This motion can be viewed as translation of centre of mass and rotation about an axis passing
through centre of mass
If CM = Moment of inertia about this axis passing through COM
cm = Net torque about this axis passing through COM

a CM = Acceleration of COM

v CM = Velocity of COM

Fext = Net external force acting on the system.

Psystem = Linear momentum of system.

L CM = Angular momentum about centre of mass.

rCM = Position vector of COM w.r.t. point A.
 
then (i)  cm   cm 
 
(ii) Fext  Ma cm
 
(iii) Psystem  Mv cm

1 1
(vi) Total K.E.= Mv cm2 +  cm  2
2 2
 
(v) L CM   CM 
 
(vi) Angular momentum about point A = L about C.M. + L of C.M. about A
   
L A   cm   rcm  Mv cm

dL A d 
    d
 (  cm   rcm  Mv cm )   A . Notice that torque equation can be applied to a rigid body in
dt dt dt
a general motion only and only about an axis through centre of mass.

KVPY_ SA STREAM # 233


Example 47. A uniform sphere of mass 200 g rolls without slipping on a plane surface so that its centre
moves at a speed of 2.00 cm/s. Find its kinetic energy.
Solution : As the sphere rolls without slipping on the plane surface, its angular speed about the centre is
v cm
= . The kinetic energy is
r

1 1 1 2 1
K=  2 + Mv cm2 = . Mr22 + Mv cm2
2 cm 2 2 5 2

1 1 7 7
= Mv cm2 + Mv cm2 = Mv cm2 = (0.200 kg) (0.02 m/s) 2 = 5.6 × 10–5 J.
5 2 10 10
B F
Example 48. A constant force F acts tangentially at the highest point of a
r
uniform disc of mass m kept on a rough horizontal surface as C

shown in figure. If the disc rolls without slipping, calculate the


acceleration of the centre (C) and point A and B of the disc. A rough surface

Solution : The situation is shown in figure. As the force F rotates the disc, the point of contact has a
tendency to slip towards left so that the static friction on the disc will act towards right. Let r be
the radius of the disc and a be the linear acceleration of the centre of the disc. The angular
acceleration about the centre of the disc is  = a/r, as there is no slipping.
For the linear motion of the centre,
F + f = ma ..........(i)
and for the rotational motion about the centre,

1 2 a 1
Fr – f r =  =  mr    or, F–f= ma, ...........(iii)
2  r 2
From (i) and (ii),

3 4F
2F = ma or a = 3m .
2
Acceleration of point A is zero.

 4F   8F 
Acceleration of point B is 2a = 2   =   Ans.
 3 m   3m 

Example 49. A circular rigid body of mass m, radius R and radius of gyration (k) rolls without slipping on an
inclined plane of a inclination . Find the linear acceleration of the rigid body and force of friction on
it. What must be the minimum value of coefficient of friction so that rigid body may roll without
sliding?
Solution :

If a is the acceleration of the centre of mass of the rigid body and f the force of friction between
sphere and the plane, the equation of translatory and rotatory motion of the rigid body will be.

KVPY_ SA STREAM # 234


mg sin  – f = ma (Translatory motion)
fR =   (Rotatory motion)


f=
R

 = mk2 , due to pure rolling a = R mg sin  – = mR
R

 mk 2 
mg sin  = m R + mg sin  = m R +
R R

mk 2  R 2  k 2 
mg sin  = ma + a
mg sin  =  2 
R  R 

g sin  g sin   mk 2 a mg k 2 sin 


a= a= f= f= 
 R2  k 2   2  R R2 R2  k 2
  1  k 
 R
2
  R2 
 

mk 2
f N a    mg cos 
R2

k2 g sin  tan  tan 


R2 ×  µg cos  µ  µmin =
R 2 (k 2  R 2 )  R2   R2 
1  2  1  2 
 k   k 

Note : From above example if rigid bodies are solid cylinder, hollow cylinder, solid sphere and hollow sphere.
(1) Increasing order of acceleration.
asolid sphere > ahollow sphere > asolid cylinder > ahollow cylinder
(2) Increasing order of required friction force for pure rolling.
f hollow cylinder > f hollow sphere > f solid cylinder > fsolid sphere
(3) Increasing order of required minimum friction coefficient for pure rolling.
µhollow cylinder > µhollow sphere > µsolid cylinder > µsolid sphere


9 .4 Insta nta neous a xis of rota tion :
It is the axis about which the combined translational and rotational motion appears as pure rotational
motion.
The combined effect of translation of centre of mass and rotation about an axis through the centre of
mass is equivalent to a pure rotation with the same angular speed about a stationary axis ; this axis
is called instantaneous axis of rotation. It is defined for an instant and its position changes with time.
eg. In pure rolling the point of contact with the surface is the instantaneous axis of rotation.
Geometrical construction of instantaneous axis of rotation (I.A.R). Draw velocity vector at any
two points on the rigid body. The I.A.R. is the point of intersection of the perpendicular drawn on
them.

KVPY_ SA STREAM # 235


 In case of pure rolling the lower point is instantaneously axis of rotation.
The motion of body in pure rolling can therefore by analysed as pure rotation about this axis.
Consequently
P = P
P = P
K.E. = 1/2 P 2
Where P is moment of inertial instantaneous axis of rotation passing through P.

Example 50.

Prove that kinetic energy = 1/2 P 2


1 1 1 1
Solution : K.E. =  2 + Mv cm2 = I 2 + M2R2
2 cm 2 2 cm 2
1
( + MR2) 2
2 cm
1
2
 
 contact po int 2

 Notice that pure rolling of uniform object equation of torque can also be applied about the contact point.

Example 51. A uniform bar of length  and mass m stands vertically touching a vertical wall (y-axis). When slightly
displaced, its lower end begins to slide along the floor (x-axis). Obtain an expression for the angular
velocity () of the bar as a function of . Neglect friction everywhere.

Solution : The position of instantaneous axis of rotation (IAOR) is shown in figure.

KVPY_ SA STREAM # 236


   
C   cos , sin   r  = half of the diagonal
 2 2  2
All surfaces are smooth. Therefore, mechanical energy will remain conserved.
 Decrease in gravitational potential energy of bar = increase in rotational kinetic energy of bar
about IAOR.
 1
 mg (1 – sin ) = 2 ....(1)
2 2

m 2
Here, = + mr2 (about IAOR)
12

m 2 m 2 m 2
or = + =
12 4 3
Substituting in Eq. (1), we have

1  m  2
2

mg (1 – sin ) = 
2 2  3 

3g (1  sin )
or  Ans.

The nature of friction in the following cases assume body is perfectly rigid


v
(i) v = R
smooth surface
No friction and pure rolling.


v

(ii) v = R rough surface


No friction and pure rolling (If the body is not perfectly rigid, then there is a small friction acting in this
case which is called rolling friction)


v

(iii) v > R or v < R smooth surface


No friction force but not pure rolling.


v
(iv) v > R v > R
fk
rough surface
There is Relative Motion at point of contact so Kinetic Friction, fk = µN will act in backward direction.
This kinetic friction decrease v and increase  , so after some time v = R and pure rolling will
resume like in case (ii).


v
(v) v < R v < R
rough surface fk

KVPY_ SA STREAM # 237


There is Relative Motion at point of contact so Kinetic Friction, fk = µN will act in forward direction.
This kinetic friction increase v and decrease  , so after some time v = R and pure rolling will
resume like in case (ii).


v F
(vi) v = R (initial)
smooth surface
No friction and no pure rolling.


v F
(vii) v = R (initial) fs
rough surface
Static friction whose value can be lie between zero and µsN will act in backward direction. If coeffi-
cient of friction is sufficiently high, then fs compensates for increasing v due to F by increasing  and
body may continue in pure rolling with increases v as well as  .

Example 52. A rigid body of mass m and radius r rolls without slipping on a rough surface. A force is acting on a
rigid body x distance from the centre as shown in figure. Find the value of x so that static friction is
zero.

P F
x
O
a
a = R
f
rough surface
Solution : Torque about centre of mass Fx = cm  ............ (1)
F = ma ........... (2)
From eqn. (1) & (2)
max = cm  (a = R)
 cm
x=
mR

Note :- For pure rolling if any friction is required then friction force will be statics friction. It may be zero,
backward direction or forward direction depending on value of x. If F below the point P then friction
force will act in backward direction or above the point P friction force will act in forward direction.

Example 53. A hollow sphere is projected horizontally along a rough surface with speed v and angular velocity 0
v
find out the ratio . So that the sphere stope moving after some time.
0


a=0
a v v=0
fk v
 0R
Solution : Torque about lowest point of sphere.
f k × R = 
2
mg × R = mR 2 
3
3g
= angular acceleration in opposition direction of angular velocity.
2R

KVPY_ SA STREAM # 238


 = 0 – t (final angular velocity  = 0)
3g
0 = t
2R
0  2R
t=
3g
acceleration 'a = g'
v f = v – at (final velocity vf = 0)
v = g × t
v
t=
g
To stop the sphere time at which v &  are zero, should be same.
v 20R v 2R
 =   3
g 3g 0


9 .5 Rolling on m oving surfa ce
Sufficiently Rough
m
a + R
 m F
/////////////////////////////////////////////

smooth
Friction on the plate backward or on cylinder friction forward so cylinder move forward.

m a
f f
m F
Because of pure rolling static friction f.
2f b
mR 2
fR =  =
2 mR
f = ma F – f = mb
R
F = m(a + b) a=
2
At contact point
3R
b = a + R b=
2
b = 3a F = 4ma
F 3F
a= b=
4m 4m
b a
w.r.t. plate distance is covered = 
and acceleration w.r.t. plate (b – a)
1 
= (b – a) t2
2

1 a m
= ×2at2 = t = = 2
2 F F

KVPY_ SA STREAM # 239


Example 54. There are two cylinders of radii R 1 and R2 having moments of inertia 1 and 2 about their
respective axes as shown in figure . Initially, the cylinders rotate about their axes with angular
speed 1 and 2 as shown in the figure. The cylinders are moved closed to touch each other
keeping the axes parallel. The cylinders first slip over each other at the contact but the slipping
finally ceases due to the friction between them. Calculate the angular speeds of the cylinders
after the slipping ceases.

Solution : When slipping ceases, the linear speeds of the points of contact of the two cylinders will be
equal. If 1 and 2 be the respective angular speeds, we have
1 R1 and 2 R2 ................(i)
The change in the angular speed is brought about by the frictional force which acts as long as the
slipping exists. If this force f acts for a time t, the torque on the first cylinder is fR1 and that on the
second is fR2. Assuming 1 > 2, the corresponding angular impulses are – fR1t and fR2t, We,
there fore, have
– f R1 t = 1 (1 –1) and fR2 t = 2 (2 –2)
1 2
or, – R (1 –1) = R (2 –2) ................(ii)
1 2

 1 ω1 R 2   2 ω 2 R 1  1 ω1 R 2   2 ω 2 R 1
Solving (i) and (ii) 1 = R2 and 2 = R1.
 2 R12  1 R 22  2 R12  1 R 22

Example 55. A hollow cylinder of mass m is suspended through two light


strings rapped around it as shown in figure. Calculate (a) the
tension T in the string and (b) the speed of the cylinder as it
falls through a distance .

Solution : The portion of the strings between the ceiling and the cylinder is at rest. Hence the points of
the cylinder where the strings leave it are at rest. The cylinder is thus rolling without slipping on
the strings. Suppose the centre of the cylinder falls with an acceleration a. The angular
acceleration of the cylinder about its axis is  = a/R, as the cylinder does not slip over the
strings.
The equation of motion for the centre of mass of the cylinder is
Mg – 2T = Ma .............(i)
and for the motion about the centre of mass, it is
 
2 Tr = Mr 2 α = Mra or 2T = Ma.
From (i) and (ii),
g Mg
a= and T = .
2 4
As the centre of the cylinder starts moving from rest, the velocity after it has fallen through a
distance  is given by

g
v2 = 2    or v= g .
2

KVPY_ SA STREAM # 240


Example 56. A hollow sphere of mass M and radius R as shown in figure
slips on a rough horizontal plane. At some instant it has linear
v0
velocity v 0 and angular velocity about the centre 2 R as shown

in figure. Calculate the linear velocity after the sphere starts


pure rolling.
v0
Solution : Velocity of the centre = v 0 and the angular velocity about the centre =
2R
. Thus v 0 > 0R. The

sphere slips forward and thus the friction by the plane on the sphere will act backward. As the
friction is kinetic, its value is µN = µMg and the sphere will be decelerated by acm = f/M. Hence,
f
v(t) = v 0 – t. .............(i)
M
This friction will also have a torque  = fr about the centre. This torque is clockwise and in the
direction of 0. Hence the angular acceleration about the centre will be
R 3f
=f 2 =
2 MR
(2 / 3 )MR

3f v0 3f
and the clockwise angular velocity at time t will be (t) = 0 + 2 MR t = 2 R + 2 MR t.

v0 3f
Pure rolling starts when v(t) = R(t) i.e., v(t) = 2 + 2 M t. ............(ii)

3 3 v0 2 4
Eliminating t from (i) and (ii), v(t) + v(t) = v 0 + 2 or, v(t) = × 2v 0 = v 0.
2 2 5 5
Thus, the sphere rolls with linear velocity 4v 0/5 in the forward direction.

1 0 . T OPPLI N G :
In many situations an external force is applied to a body to cause it to slide along a surface. In certain
cases, the body may tip over before sliding ensues. This is known as topping.
(1) There is a no horizontal force so pressure at bottom is uniform and normal is colinear with mg.

(2) If a force is applied at COM, pressure is not uniform Normal shifts right so that torque of N can
counter balance torque of friction.

KVPY_ SA STREAM # 241


(3) If F is continuously increased N keeps shifting towards right until it reaches the right most point D.
Here we have assumed that the surface is sufficiently rough so that there is as sliding on F is increase
to F max.
If force is increased any further, then torque of N can not counter balance torque of friction f r & body will
topple.
The value of force now is the max value for which toppling will not occure F max.

F max = f r
N = mg
f r . b/2 = N . a/2  f r = Na/b = mg a/b, F max = mg a/b

(4) If surface is not sufficiently rough and the body slides before F is increased to F max = mg a/b then
body will slide before toppling. Once body starts sliding friciton becomes constant and hence no
topping. This is the case if
F max > f limit
 mg a/b > mg
 < a/b

Condition for toppling when   a/b in this case body will topple if F > mg a/b
but if  < a/b, body will not topple any value of F applied a COM

Example 57.

Find out minimum value of F for toppling


Solution : Never topple

KVPY_ SA STREAM # 242


Example 58. A uniform cube of side 'a' and mass m rests on a rough horizontal table. A horizontal force F is
a
applied normal to one of the faces at a point directly below the centre of the face, at a height
4
above the base.
(i) What is the minimum value of F for which the cube begins to tip about an edge?
(ii) What is the minimum value of s so that toppling occures.
(iii) If  = min , find minimum force for topping.
(iv) Minimum s so that F min can cause toppling.
Solution : (i) In the limiting case normal reaction will pass through O. The cube will tip about O if torque
of Fabout O exceeds the torque of mg.

a a
Hence, F    mg   or F > 2 mg
4
  2
therefore, minimum value of F is 2 mg

a/2 F
fr a/4
O
mg

(ii) In this case since it is not acting at COM, toppling can occur even after body started slinding
because increasing the the torque of F about COM.hence min = 0,
(iii) Now body is sliding before toppling, O is not I.A.R., torque equation can not be applied across
it. It can now be applied about COM.
a a
F× =N× ................ (1)
4 2
N = mg .......................... (2)
from (1) and (2)
F = 2 mg

(iv) F > 2 mg ................... (1) (from sol. (i))


N = mg .......................(2)
F = µ sN = µ smg ........... (3) from (1) and (2)
µs = 2

KVPY_ SA STREAM # 243


1. A uniform disc of radius R lies in the x-y plane, with its centre at origin. its moment of inertia about z-
axis is equal to its moment of inertia about line y = x + c. The value of c will be
R R R
(A)  (B) ± (C) (D) –R
2 2 4

2. The moment of inertia of a uniform thin rod of mass m and length L about
two axis PQ and RS passing through centre of rod C and in the plane of the
rod are PQ and RS respectively. Then PQ + RS is equal to

m 2 m 2
(A) (B)
3 2

m 2 m 2
(C) (D)
4 12

3. A uniform rod of length 4L and mass M is suspended from a horizontal roof


by two light strings of length L and 2L as shown. Then the tension in the
left string of length L is
Mg Mg
(A) (B)
2 3

3 Mg
(C) Mg (D)
5 4

4. A rod is horizontal and hinged at point O at the moment shown. Speed of


mid–point of rod at the given moment is :
(A) 10 m/s (B) 5 2 m/s
(C) 20 2 m/s (D) 5 m/s

5. A rod of mass m is supported by string AB and friction due to wall. Then


friction force on rod due to wall is : (g = acceleration due to gravity).
(A) mg upward (B) mg downward
mg
(C) upward (D) Data insufficient
2

6. A uniform disc of mass M and radius R is released from rest in the shown position. PQ is a string, OP is a
horizontal line, O is the centre of the disc and distance OP is R/2. Then tension in the string just after the
disc is released will be :

Mg Mg 2Mg
(A) (B) (C) (D) none of these
2 3 3

KVPY_ SA STREAM # 244


7. At shown instant a thin uniform rod AB of length L = 1m and mass m = 1 kg is coincident with y-axis
such that centre of rod is at origin. The velocity of end A and centre O of rod at shown instant are
 
VA  2 î m/s and Vo  10 î m/s respectively. Then the kinetic energy of rod at the shown instant is :

y
A

x
O

(A) 42 J (B) 56 J (C) 74 J (D) None of these

8. A uniform rod of mass m is hinged at a point L/4 from one end of the rod and is at rest. An impulse is
imparted on the other end of the rod perpendicular to it. The angular speed of the rod just after the
application of the impulse is :
36 I 36 I 23 I I
(A) (B) (C) (D)
mL 7 mL 7 mL 7 mL

9. In the above question, the impulse due to the hinge is :


 
(A) in the direction of  (B) opposite to 
7 7
2 2
(C) in the direction of  (D) opposite to 
7 7

10. A uniform disc of mass M and radius R is rotating about its centre of mass (the centre of mass is at
rest) with an angular speed . The angular momentum of disc about a point A (as shown) will be

1 1
(A) MR2 + MhR  (B) MR2 (C) MR2 + MhR  (D) None of these
2 2

11. A uniform disc of mass m and radius R is undergoing fixed axis rotation about its own axis and centre
O of disc remains stationary. The angular speed of disc is . Then the magnitude of angular momentum
of disc about shown point Q is : (OP = x and PQ = R)

m,R

P x O

R
Q

( x 2  2R 2 ) ( x2  R2 ) mx 2 mR 2
(A) m  (B) m  (C)  (D) 
2 2 2 2

KVPY_ SA STREAM # 245


12. A uniform circular disc placed on a horizontal rough surface has initially a
velocity v 0 and an angular velocity 0 as shown in the figure. The disc comes to
rest after moving some distance in the direction of motion. Then v 0/ 0 is:
(A) r/2 (B) r (C) 3 r/2 (D) 2

13. An impulsive force F acts horizontally on a solid sphere of radius R placed on


a horizontal surface. The line of action of the impulsive force is at a height h
above the centre of the sphere. If the rotational and transational kinetic energies
of the sphere just after the impulse are equal, then h is equal to :
R 2 R 2
(A) (B) R (C) (D) R
2 5 2 5

14. A solid sphere of mass m and radius r is gently placed on a conveyer belt moving with constant velocity
2
V. If the coefficient of friction between the belt and sphere is , the distance travelled by the centre of
7
the sphere before it starts pure rolling is

V2 2V 2 2V 2 2V 2
(A) (B) (C) (D)
7g 49 g 5g 7g

15. A nonuniform sphere at rest on a rough horizontal surface is acted upon by a force F as shown. The friction
force acting on it is

(1) towards left (2) towards right (3) zero


Then possible option is -
(A) 1 or 2 (B) 1 or 3 (C) 2 or 3 (D) 1or 2 or 3
16. A horizontal force F is applied to the lower end of a uniform thin rod of mass 4kg and
length L = 50 cm as shown in the figure. The rod undergoes only translational motion
along the smooth horizontal surface. If F = 60 N determine the angle  for translation
motion of the rod. (g = 10 m/s2)
1  2  1  1  1  1  1  1 
(A) tan   (B) tan   (C) sin   (D) sin  
3 2 5 4

17. A solid cylinder and a solid sphere, both having the same mass and radius, are released from an incline of angle
 one by one. They roll on the incline without slipping. The statement that holds good in this motion is that
(A) the force of friction that acts on the two is the same
(B) the force of friction is greater in case of a sphere than for a cylinder.
(C) the force of friction is greater in case of cylinder than for sphere.
(D) the force of friction will depend on the nature of the surface of the body that is moving and that of the
inclined surface, and is independent of the shape and size of the moving body.

18. A uniform solid cylinder is given an angular speed  and placed on a rough plate of negligible thickness.
The horizontal surface below the plate is smooth. Then the angular speed of the cylinder when it starts
pure rolling on the plate will be: [ Assume sufficient length of plate ]

m

m'
///////////////////////////////////////////////////////////////

  2
(A) (B) (C) (D) none of these
2 3 3

KVPY_ SA STREAM # 246


19. A uniform solid ball of mass ‘m’ rolls without sliding on a fixed horizontal surface. The velocity of the
lowest point of the ball with respect to the centre of the ball is V. The total kinetic energy of the ball is:
7 1 4
(A) mv 2 (B) mv 2 (C) mv 2 (D) mv 2
10 5 3

20. A solid homogeneous cylinder of height h and base radius r is kept vertically on a conveyer belt moving
horizontally with an increasing velocity v = a + bt 2. If the cylinder is not allowed to slip then the time
when the cylinder is about to topple, will be equal to
rg 2 rg 2 bg rg
(A) (B) (C) (D) 2 bh
bh bh rh

KVPY PROBLEMS (PREVIOUS YEARS)


1. The rear wheels of a car are turning at an angular speed of 60 rad/s. The brakes are applied for 5s, causing
a uniform angular retardation of 8 rads–2. The number of revolutions turned by the rear wheels during the
braking period is about [KVPY_2007]
(A) 48 (B) 96 (C) 32 (D) 12
2. A solid square plate is spun around different axes with the same angular speed. In which of the following
choice of axis of rotation will the kinetic energy of the plate be the largest ? [KVPY_2009]
(A) through the central normal to the plate. (B) along one of the diagonals of the plate.
(C) along one of the edges of the plate. (D) through one corner normal to the plate.
3. A solid cylinder P rolls without slipping from rest down an inclined plane attaining a speed v p at the bottom.
Another smooth solid cyclinder Q of same mass and dimensions slides without friction from rest down the
 vp 
inclined plane attaining a speed v q at the bottom. The ratio of the speeds  v  is :
 q
[KVPY_2014_SB]
 3  3  2  4
(A)  4  (B)  2  (C)  3  (D)  3 
       
4. A solid sphere spinning about a horizontal axis with an angular velocity  is placed on a horizontal surface.
Subsequently it rolls without slipping with an angular velocity of
[KVPY_2014_SB]
(A) 2  /5 (B) 7  /5 (C) 2  /7 (D) 

5. A hollow tilted cylindrical vessel of negligible mass rests on a horizontal plane as known. The diameter
of the base is a and the side of the cylinder makes an angle  with the horizontal. Water is then slowly
poured into the cylinder. The cylinder topples over when the water reaches a certain height h, given by.
[KVPY_2015_SA_1 Mark]

a
(A) h  2a tan  (B) h  a tan 2  (C) h  a tan  (D) h  tan 
2

KVPY_ SA STREAM # 247


6. A rigid body in the shape of a “V” has two equal arms made of uniform rods. What must the angle
between the two rods be so that when the body is suspended from one end, the other arm is horizontal?
[KVPY_2015_SA_2 Mark]

1 1 1 1 1 1 1


(A) cos 1   (B) cos   (C) cos   (D) cos  
3 2 4 6

7. A horizontal disk of moment of inertia 4.25 kg-m 2 with respect to its axis of symmetry is spinning
counter clockwise at 15 revolutions per second about its axis, as viewed from above. A second disk of
moment of inertia 1.80kg-m2 with respect to its axis of symmetry is spinning clockwise at 25 revolutions
per second as viewed from above about the same axis and is dropped on top of the first disk. The two
disks stick together and rotate as one about their axis of symmetry. The new angular velocity of the
system as viewed from above is close to. [KVPY_2015_SB] [1 Mark]
(A) 18 revolutions/second and clockwise (B) 18 revolutions/second and counter clockwise
(C) 3 revolutions/second and clockwise (D) 3 revolutions/second and counter clockwise

8. The moments of inertia of a non-uniform circular disc (of mass M and radius R) about four mutually
perpendicular tangents AB,BC CD, DA are I1 , I 2 , I 3 and I 4 respectively (the square ABCD circumscribes
the circle.) The distance of the center of mass of the disc from its geometrical center is given by.
[KVPY_2015_SB] [2 Mark]
1 1
(A) ( I 3  I 3 ) 2  ( I 2  I 4 )2 (B) (I3  I3 )2  ( I2  I4 )2
4MR 12 MR
1 1
(C) ( I1  I 2 ) 2  ( I 3  I 4 ) 2 (D) ( I1  I 3 ) 2  ( I 2  I 4 ) 2
3MR 2MR

9. A rigid ball rolls without slipping on a surface shown below. [KVPY_2016_SA] [1 Mark]

Which on the following is the most likely representation of the distance travelled by the ball vs time graph ?

distance distance distance distance

(A) (B) (C) (D)


time time time time
A. B. C. D.

10. Which one of the following four graphs best depict the variation with x of the moment of inertia I of a uniform
triangular lamina about an axis parallel to its base at a distance x from it : [KVPY_2016_SA] [2 Mark]

h
x

I I I I

(A) (B) (C) (D)

h x h x h x h x

KVPY_ SA STREAM # 248


11. A uniform metal plate shaped like a triangle ABC has a mass of 540 gm. The length of the sides AB, BC, and
CA are 3 cm, 5 cm and 4 cm, respectively. The plate is pivoted freely about the point A. What mass must be
added to a vertex, so that the plate can hang with the long edge horizontal ? [KVPY_2016_SA] [2 Mark]
(A) 140 gm at C (B) 540 gm at C (C) 140 gm at B (D) 540 gm at B

12. A “V” shaped rigid body has two identical uniform arms. What must be the angle between the two arms so
that when the body is hung from one end, the other arm is horizontal ? [KVPY_2016_SA] [2 Mark]
(A) cos–1 (1/3) (B) cos–1 (1/2) (C) cos–1 (1/4) (D) cos–1 (1/6)

13. A uniform ring of radius R is moving on a horizontal surface with speed v and then climbs up a ramp of
inclination 30º to a height h. There is no slipping in the entire motion. Then h is : [KVPY_2016_SB] [1 Mark]
(A) v 2/ 2g (B) v 2/ g (C) 3v2/ 2g (D) 2v2/ g

1
14. One end of a rod of length L = 1 m is fixed to a point on the circumference of a wheel of radius R  m . The
3
other end is sliding freely along a straight channel passing through the center O of the wheel as shown in the
figure below. The wheel is rotating with a constant angular velocity  about O. [KVPY_2017_SB] [2 Mark]

R L

O
P

The speed of the sliding end P when  = 60º is


2  2 
(A) (B) (C) (D)
3 3 3 3

15. A solid cube of wood of side 2a and mass M is resting on a horizontal surface as shown in the figure. The
cube is free to rotate about a fixed axis AB. A bullet of mass m (<< M) and speed v is shot horizontally at the
face opposite to ABCD at a height of 4a/3 from the surface to impart the cube and angular speed . It strikes
the face and embeds in the cube. Then  is close to (note : the moment of inertia of the cube about an axis
2Ma 2
perpendicular to the face and passing through the center of mass is [KVPY_2017_SB] [2 Mark]
3
C
D

Mv Mv mv mv
(A) (B) (C) (D)
ma 2m a Ma 2Ma

EXERCISE - 1
1. (B) 2. (D) 3. (A) 4. (D) 5. (C) 6. (C) 7. (C)

8. (B) 9. (C) 10. (B) 11. (D) 12. (A) 13. (D) 14. (A)

15. (D) 16. (A) 17. (C) 18. (D) 19. (A) 20. (A)

EXERCISE - 2
1. (C) 2. (D) 3. (C) 4. (C) 5. (C) 6. (A) 7. (C)

8. (A) 9. (D) 10. (A) 11. (A) 12. (A) 13. (B) 14. (A)
15. (D)

KVPY_ SA STREAM # 249


SI M PLE H ARM ON I C M OT I ON
~~~~~~
1. PERIODIC MOTION
When a body or a moving particle repeats its motion along a definite path after regular interval of time, its
motion is said to be Periodic Motion and interval of time is called time period or harmonic motion period
(T). The path of periodic motion may be linear, circular, elliptical or any other curve. For example, rotation of
earth about the sun.
2. OSCILLATORY MOTION
‘To and Fro‘ type of motion is called an Oscillatory Motion. It need not be periodic and need not have fixed
extreme positions. For example, motion of pendulum of a wall clock.
The oscillatory motions in which energy is conserved are also periodic.
The force / torque (directed towards equilibrium point) acting in oscillatory motion is called restoring
force / torque.
Damped oscillations are those in which energy is consumed due to some resistive forces and hence total
mechanical energy decreases.

3. SIMPLE HARMONIC MOTION


If the restoring force/ torque acting on the body in oscillatory motion is directly proportional to the displace-
ment of body/particle and is always directed towards equilibrium position then the motion is called simple
Harmonic Motion (SHM). It is the simplest (easy to analyze) form of oscillatory motion.

3.1 TYPES OF SHM


(a) Linear SHM : When a particle undergoes to and fro motion about an equilibrium position, along a straight
line. A and B are extreme positions. M is mean position. AM = MB = Amplitude

(b) Angular SHM : When a body/particle is free to rotate oscillate about a given axis on a curved path.

3.2 EQUATION OF SIMPLE HARMONIC MOTION (SHM) :


The necessary and sufficient condition for SHM is
F = – kx
where k = positive constant for a SHM = Force constant
x = displacement from mean position.
d2 x
or m = – kx
dt 2

d2 x k
 2 + x=0 [differential equation of SHM]
dt m

d2 x k
 2 + 2x = 0 where  =
dt m
It’s solution is x = A sin (t + )

3.3 CHARACTERISTICS OF SHM


Note : In the figure shown, path of the particle is on a straight line.
(a) Displacement - It is defined as the distance of the particle from
the mean position at that instant. Displacement in SHM at
time t is given by x = A sin (t + )
(b) Amplitude - It is the maximum value of displacement of the
particle from its equilibrium position.
1
Amplitude = [distance between extreme points or positions]
2
It depends on energy of the system.

KVPY_ SA STREAM # 250


2
(c) Angular Frequency () :  = = 2f and its unit is rad/sec.
T

(d) Frequency (f) : Number of oscillations completed in unit time interval is called frequency of
1 
oscillations, f = = , its units is sec–1 or Hz.
T 2 
(e) Time period (T) : Smallest time interval after which the oscillatory motion gets repeated is called time
2 m
period, T = = 2
 k

d2 x
Example 1. For a particle performing SHM, equation of motion is given as + 4x = 0. Find the time period.
dt 2
d2 x
Solution : =  4x 2 = 4 = 2
dt 2

2
Time period; T= =


(f) Phase : The physical quantity which represents the state of motion of particle (eg. its position and
direction of motion at any instant).
The argument (t + ) of sinusoidal function is called instantaneous phase of the motion.

(g) Phase constant () : Constant  in equation of SHM is called phase constant or initial phase.
It depends on initial position and direction of velocity.

(h) Velocity(v) : Velocity at an instant is the rate of change of particle’s position w.r.t time at that instant.
Let the displacement from mean position is given by
x = A sin t + 
dx d
Velocity, v= = Asin(  t   ) 
dt dt

v = A cos t +  or, v =  A2  x2


At mean position (x = 0), velocity is maximum.
v max = A
At extreme position (x = A), velocity is minimum.
v min = zero
Speed (v)
GRAPH OF SPEED (v) VS DISPLACEMENT (x):

v   A 2  x2 
v 2  2 A 2  x 2 
v2 x2
v 2 + 2x2 = 2A2  1
2 A 2 A2 –A A x
GRAPH WOULD BE AN ELLIPSE
(i) Acceleration : Acceleration at an instant is the rate of change of particle’s velocity w.r.t. time at that
instant.
dv d
Acceleration, a = = [ A  cos(  t   )]
dt dt
a = 2A sin (t + 
a = 2x

KVPY_ SA STREAM # 251


Note  Negative sign shows that acceleration is always directed towards the mean position.
At mean position (x = 0), acceleration is minimum.
amin = zero
At extreme position (x = A), acceleration is maximum.
amax = 2A
GRAPH OF ACCELERATION (A) VS DISPLACEMENT (x)

A
a =  x
2 x
–A

 1 
Example 2. The equation of particle executing simple harmonic motion is x = (5 m) sin (  s )t   . Write
 3 
down the amplitude, time period and maximum speed. Also find the velocity at t = 1 s.
Solution : Comparing with equation x = A sin (t + ),
we see that the amplitude = 5 m,
2 2
and time period = = = 2s.
  s 1
The maximum speed = A  = 5 m ×  s–1 = 5 m/s.
dx
The velocity at time t = = A  cos (t + )
dt
At t = 1 s,
  5
v = (5 m) ( s–1) cos     = – m/s.
 3  2

Example 3. A particle executing simple harmonic motion has angular frequency 6.28 s–1 and amplitude 10 cm.
Find (a) the time period, (b) the maximum speed, (c) the maximum acceleration, (d) the speed when
the displacement is 6 cm from the mean position, (e) the speed at t = 1/6 s assuming that the
motion starts from rest at t = 0.
2 2
Solution : (a) Time period = = s = 1 s.
 6.28
(b) Maximum speed = A = (0.1 m) (6.28 s–1)
= 0.628 m/s.
(c) Maximum acceleration = A2
= (0.1 m) (6.28 s–1)2
= 4 m/s2.
(d) v= 2 2
A 2  x 2 = (6.28 s ) (10 cm)  ( 6 cm) = 50.2 cm/s.
–1

(e) At t = 0, the velocity is zero i.e., the particle is at an extreme. The equation for displacement
may be written as
x = A cost.
The velocity is v = – A  sin t.
1  6.28 
At t= s, v = – (0.1 m) (6.28 s–1) sin  
6  6 

= ( – 0.628 m/s) sin = 54.4 cm/s.
3

KVPY_ SA STREAM # 252


Example 4. A particle starts from mean position and moves towards positive
extreme as shown. Find the equation of the SHM. Amplitude of t=0
SHM is A. -A 0 A
Solution : General equation of SHM can be written as x = A sin (t + )
At t = 0, x = 0
 0 = A sin
  = 0,    [0,2 )
Also; at t = 0, v = ve
 A cos = ve
or, = 0
Hence, if the particle is at mean position at t = 0 and is moving towards +ve extreme, then the
equation of SHM is given by x = A sint
Similarly t=0
for -A 0 A
= 
 equation of SHM is x = A sin(t + )
or, x = A sint

Note :
 If mean position is not at the origin, then we can replace x by x  x0 and the eqn.
becomes x  x0 = A sin t + , where x0 is the position co-ordinate of the mean position.

Example 5. A particle is performing SHM of amplitude “A” and time period “T”. Find the time taken by the particle
to go from 0 to A/2.
Solution : Let equation of SHM be x = A sin t
when x = 0 , t = 0
when x = A/2 ; A/2 = A sin t
or sin t = 1/2 t = π /6
2
t = π /6 t = T/12
T
Hence , time taken is T/12, where T is time period of SHM.

Example 6. A particle of mass 2 kg is moving on a straight line under the action of force F = (8 – 2x) N. It is
released at rest from x = 6 m.
(a) Is the particle moving simple harmonically.
(b) Find the equilibrium position of the particle.
(c) Write the equation of motion of the particle.
(d) Find the time period of SHM.
Solution : F = 8 – 2x
or F = –2(x – 4)
at equilibrium position F = 0
 x = 4 is equilibrium position
Hence the motion of particle is SHM with force constant 2 and equilibrium position x = 4.
(a) Yes, motion is SHM.
(b) Equilibrium position is x = 4 v=0
(c) At x = 6 m, particle is at rest i.e. it is one of the extreme position
Hence amplitude is A = 2 m and initially particle is at the extreme position. 0 x=4 x=6
 Equation of SHM can be written as
k 2
x – 4 = 2 cos t , where  = = =1
m 2
i.e. x = 4 + 2 cos t
2
(d) Time period, T = = 2sec.

KVPY_ SA STREAM # 253



4. SHM AS A PROJECTION OF UNIFORM CIRCULAR MOTION
P0
Consider a particle moving on a circle of radius A with a a x(t) P vx (t)
constant angular speed  as shown in figure.

Suppose the particle is on the top of the circle (Y-axis) at


O Q x
t = 0. The radius OP makes an angle  = t with the Y- x(t)
axis at time t. Drop a perpendicular PQ on X-axis. The
components of position vector, velocity vector and accel-
eration vector at time t on the X-axis are
x(t) = A sin t v x(t)

v x(t) = A cos t
ax(t) =  2A sin t -A 0 +A
x(t)

Above equations show that the foot of perpendicular Q executes a simple harmonic motion on the X-axis.
The amplitude is A and angular frequency is . Similarly the foot of perpendicular on Y-axis will also execute
SHM of amplitude A and angular frequency [y(t) = A cos t. The phases of the two simple harmonic
motions differ by /2.

5. GRAPHICAL REPRESENTATION OF DISPLACEMENT, VELOCITY & ACCELERATION IN SHM


Displacement, x = A sin t
π
Velocity, v = A cos t = Asin t + ) or v= A 2  x2
2
Acceleration, a =  A sint =  A sin t + π )
2 2
or a = – 2x

Note :  v= A 2  x2
a=–x 2

These relations are true for any equation of x.

time, t 0 T/4 T/2 3T/4 T


displacement, x 0 A 0 A 0
Velocity, v A 0 A 0 A
acceleration, a 0 2A 0 2A 0

x
T/4 T/2 3T/4 T 5T/4 3T/2
A

–A
v

KVPY_ SA STREAM # 254


1. All the three quantities displacement, velocity and acceleration vary harmonically with time, having
same period.
2. The velocity amplitude is times the displacement amplitude (v max = A).

3. The acceleration amplitude is 2 times the displacement amplitude (amax = A).

π
4. In SHM, the velocity is ahead of displacement by a phase angle of .
2

π
5. In SHM, the acceleration is ahead of velocity by a phase angle of .
2

6. ENERGY OF SHM
6.1 Kinetic Energy (KE)
1 1 1
mv 2 = m2 (A2 – x2) = k (A2 – x2) (as a function of x)
2 2 2
1 1
= m A22 cos2 (t + ) = KA2 cos2 (t + ) (as a function of t)
2 2
1 2 1 2 1 2
KEmax = kA ; KE 0T = kA ; KE 0A = kA
2 4 3

Frequency of KE = 2 × (frequency of SHM)

6.2 Potential Energy (PE)


1 1
Kx2 (as a function of x) = kA2 sin2 (t + ) (as a function of time)
2 2
6.3 Total Mechanical Energy (TME)
Total mechanical energy = Kinetic energy + Potential energy
1 1 1
=
k (A2 – x2) + Kx2 = KA2
2 2 2
Hence total mechanical energy is constant in SHM.
6.4 Graphical Variation of energy of particle in SHM.

E KE
TE
PE

–A/ 2 O A/ 2 A x
–A

Example 7. A particle of mass 0.50 kg executes a simple harmonic motion under a force F = – (50 N/m)x. If it
crosses the centre of oscillation with a speed of 10 m/s, find the amplitude of the motion.

Solution : The kinetic energy of the particle when it is at the centre of oscillation is
1 1
E= mv 2 = (0.50 kg) (10 m/s)2 = 25 J.
2 2
The potential energy is zero here. At the maximum displacement x = A, the speed is zero and hence
1 2
the kinetic energy is zero. The potential energy here is kA . As there is no loss of energy,,
2

KVPY_ SA STREAM # 255


1
kA2 = 25 J .............(i)
2
The force on the particle is given by
F = – (50 N/m)x.
Thus, the spring constant is k = 50 N/m.
Equation (i) gives
1
(50 N/m) A2 = 25 J or, A = 1 m.
2


7. SPRING-MASS SYSTEM

m
(1) T = 2
k
smooth surface

k m
(2) T = 2
k

(3) If spring has mass ms then

ms
T = 2
m
3 [Not in JEE, for other exams]
k

Example 8. A particle of mass 200 g executes a simple harmonic motion. The restoring force is provided by a
spring of spring constant 80 N/m. Find the time period.
Solution : The time period is

m 200  10 3 kg
T = 2 = 2 = 2× 0.05 s = 0.31 s.
k 80 N / m

Example 9. The friction coefficient between the two blocks shown in figure is µ and the horizontal plane is
smooth. (a) If the system is slightly displaced and released, find the time period. (b) Find the
magnitude of the frictional force between the blocks when the displacement from the mean position
is x. (c) What can be the maximum amplitude if the upper block does not slip relative to the lower
block ?

KVPY_ SA STREAM # 256


Solution : (a) For small amplitude, the two blocks oscillate together. The angular frequency is

k Mm
= and so the time period T = 2 .
Mm k
(b) The acceleration of the blocks at displacement x from the mean position is
  kx 
a = – 2x =  
Mm

  mkx 
The resultant force on the upper block is, therefore, ma =  M  m 
 
This force is provided by the friction of the lower block. Hence, the magnitude of the frictional
 mk | x | 
force is  
 Mm 

mk A
(c) Maximum force of friction required for simple harmonic motion of the upper block is
Mm
at the extreme positions. But the maximum frictional force can only be µ mg. Hence
mk A µ(M  m)g
= µ mg or, A=
M m k

Example 10. A block of mass m is suspended from the ceiling of a stationary elevator through a spring of spring
constant k and suddenly, the cable breaks and the elevator starts falling freely. Show that block now
executes a simple harmonic motion of amplitude mg/k in the elevator.

Solution : When the elevator is stationary, the spring is stretched to support


the block. If the extension is x, the tension is kx which should
balance the weight of the block.

Thus, x = mg/k. As the cable breaks, the elevator starts falling with acceleration ‘g’. We shall work
in the frame of reference of the elevator. Then we have to use a pseudo force mg upward on the block.
This force will ‘balance’ the weight. Thus, the block is subjected to a net force kx by the spring when
it is at a distance x from the position of unstretched spring. Hence, its motion in the elevator is
simple harmonic with its mean position corresponding to the unstretched spring. Initially, the spring
is stretched by x = mg/k, where the velocity of the block (with respect to the elevator) is zero. Thus,
the amplitude of the resulting simple harmonic motion is mg/k.

Example 11. The left block in figure collides inelastically with the right block
and sticks to it. Find the amplitude of the resulting simple harmonic
motion.

Solution : Assuming the collision to last for a small interval only, we can apply the principle of conservation of
2
v 1 v
momentum. The common velocity after the collision is . The kinetic energy = (2m)  
2 2 2
1
= mv 2. This is also the total energy of vibration as the spring is unstretched at this moment. If the
4
1
amplitude is A, the total energy can also be written as kA2. Thus,
2
1 2 1 m
kA = mv 2, giving A = 2 k v..
2 4

KVPY_ SA STREAM # 257


Example 12. Two blocks of mass m and m are connected with a spring of natural length l and spring constant k.
1 2
The system is lying on a smooth horizontal surface. Initially spring is compressed by x as shown in
0
figure.
 – x0

Show that the two blocks will perform SHM about their equilibrium position. Also (a) find the time
period, (b) find amplitude of each block and (c) length of spring as a function of time.

Solution : (a) Here both the blocks will be in equilibrium at the same time when spring is in its natural length. Let
EP1 and EP2 be equilibrium positions of block A and B as shown in figure.

EP1  EP2

EP1 EP2
x1 x2

Let at any time during oscillations, blocks are at a distance of x1 and x2 from their equilibrium
positions.
As no external force is acting on the spring block system
 (m1 + m2)xcm = m1x1  m2x2 = 0 or m1x1 = m2x2
For 1st particle, force equation can be written as

d2 x1 m1
k(x + x ) =  m or, k(x + x )=m a
1 2 1
dt 2 1 m2 1 1 1

k(m1  m 2 ) 2
k(m1  m 2 )
or, a1 =  x1   =
m1m 2 m1m 2

m1m 2  m1m 2
Hence, T = 2 k(m  m )  2 where  = (m  m ) which is known as reduced mass
1 2 K 1 2

Ans (a)
Similarly time period of 2nd particle can be found. Both will be having the same time period.

(b) Let the amplitude of blocks be A1 and A2.


m1A1 = m2A2
By energy conservation;

1 1
k(A + A )2 = k x 2 or, A +A =x
2 1 2 2 0 1 2 0

m1
or, A +A =x or, A + A =x
1 2 0 1 m2 1 0

m2 x 0 m1x 0
or, A = Similarly, A = m m
1 m1  m 2 2 1 2

KVPY_ SA STREAM # 258


(c) Consider equilibrium position of 1st particle as origin, i.e. x = 0 .
EP1 l
EP2
x co-ordinate of particles can be written as

x = A cost and x =   A cost


1 1 2 2

Hence, length of spring at time t can be written as; x=0

length = x2  x1

=   (A1 + A2)cost

Example 13. The system is in equilibrium and at rest. Now mass m1 is removed from m2. Find //////////////////////////

the time period and amplitude of resultant motion. Spring constant is K.

Solution : Initial extension in the spring


(m1  m2 ) g
x= m1
K m2

m2 g
Now, if we remove m1, equilibrium position(E.P.) of m2 will be below natural length of spring.
K
////////////////////////// //////////////////////////

N.L
m2g
(m1+m2)g K
K E.P
m1g
K

At the initial position, since velocity is zero i.e. it is the extreme position.

m1 g
Hence Amplitude =
K

m2
Time period = 2
K

Since only block of mass m is oscillating


2


8. COMBINATION OF SPRINGS
8.1 Series Combination :
Total displacement x = x1 + x2
Tension in both springs = k1 x1 = k2 x2
 Equivalent spring constant in series combination Keq is given by :

m
1/keq = 1/k1 + 1/k2  T = 2 k
eq

KVPY_ SA STREAM # 259


Note :
 In series combination, tension is same in all the springs & extension will be different. (If k is same
then deformation is also same )
 In series combination , extension of springs will be reciprocal of its spring constant.
 Spring constant of spring is reciprocal of its natural length
 k  1/
 k1 1 = k2 2 = k3 3
 If a spring is cut in ‘n’ pieces then spring constant of one piece will be nk.
8.2 Parallel combination :
Extension is same for both springs but force acting will be different.
Force acting on the system = F

 F = – (k1 x + k2 x)  F = – (k1 + k2 ) x  F = – keqx


m
 keq = k1 + k2  T = 2 k eq
9. METHOD’S TO DETERMINE TIME PERIOD, ANGULAR FREQUENCY IN S.H.M.
(a) Force / torque method
(b) Energy method

/////////////////////////
Example 14. The string, the spring and the pulley shown in figure are light.
Find the time period of the mass m.
Solution (a) Force Method
m
Let in equilibrium position of the block, extension in spring is x0. k
 kx0 = mg -- (1) ////////////////////////////
Now if we displace the block by x in the downward /////////////////////////
direction, net force on the block towards mean
position is
F = k(x + x0)  mg = kx using (1) Natural length
Hence the net force is acting towards mean x0
m Equilibrium position
position and is also proportional to x.So, the k x
m
particle will perform S.H.M. and its time period
would be ////////////////////////////

m
T = 2
k
(b) Energy Method
Let gravitational potential energy is to be zero at the level of the block when spring is in its natural
length.
Now at a distance x below that level, let speed of the block be v.
Since total mechanical energy is conserved in S.H.M.
1 2 1 2
  mgx + kx + mv = constant
2 2
Differentiating w.r.t. time, we get
 mgv + kxv + mva = 0
where a is acceleration.
mg
 F = ma =  kx + mg or F =  k(x  )
k
mg
This shows that for the motion, force constant is k and equilibrium position is x = .
k
m
So, the particle will perform S.H.M. and its time period would be T = 2
k

KVPY_ SA STREAM # 260



10. SIMPLE PENDULUM
If a heavy point mass is suspended by a weightless, inextensible and
perfectly flexible string from a rigid support, then this arrangement is called
a simple pendulum


Time period of a simple pendulum T = 2 g
(some times we can take g = 2 for making calculation simple)

Note :
 If angular amplitude of simple pendulum is more, then time period

  20 
1  
T = 2 g  16  (Not in JEE, For other exams)
 
where 0 is in radians.
 General formula for time period of simple pendulum when  is comparable to radius of Earth R.

1
T = 2  1 1 where, R = Radius of the earth
g  
R 

R
 Time period of simple pendulum of infinite length is maximum and is given by: T = 2 = 84.6 min
g
(Where R is radius of earth)
 Time period of seconds pendulum is 2 sec and  = 0.993 m.
 Simple pendulum performs angular S.H.M. but due to small angular displacement, it is considered
as linear S.H.M.
 If time period of clock based on simple pendulum increases then clock will be slow but if time period
decrease then clock will be fast.

T 1 
 If g remains constant &  is change in length, then  100   100
T 2 

T 1 g
 If  remain constant & g is change in acceleration then, T  100   2 g  100

 If  is change in length & g is change in acceleration due to gravity then,


T  1  1 g 
 100      100
T 2  2 g 

Example 15 A simple pendulum of length 40 cm oscillates with an angular amplitude of 0.04 rad. Find (a) the time
period, (b) the linear amplitude of the bob, (c) the speed of the bob when the string makes 0.02 rad with
the vertical and (d) the angular acceleration when the bob is in momentary rest. Take g = 10 m/s2.

10 m / s 2
Solution : (a) The angular frequency is = g/ = = 5 s–1
0.4m

2 2
the time period is = = 1.26 s.
 5 s 1

(b) Linear amplitude = 40 cm × 0.04 = 1.6 cm

KVPY_ SA STREAM # 261


(c) Angular speed at displacement 0.02 rad is

 = (5 s–1) (0.04 )2  (0.02)2 rad = 0.17 rad/s.

where speed of the bob at this instant


= (40 cm) × 0.175–1 = 6.8 cm/s.
(d) At momentary rest, the bob is in extreme position.
Thus, the angular acceleration
 = (0.04 rad) (25 s–2) = 1 rad/s2.


10.1 Time Period of Simple Pendulum in accelerating Reference Frame :


T = 2 g where
eff .

 
geff. = Effective acceleration in accelerating reference system = g  a , at mean position

a = acceleration of the point of suspension w.r.t. ground.
 
Condition for applying this formula: g  a = constant

Net tension in string


Also geff = at mean position
mass of bob

Example 16. A simple pendulum is suspended from the ceiling of a car accelerating uniformly on a horizontal
road. If the acceleration is a0 and the length of the pendulum is , find the time period of small
oscillations about the mean position.
Solution : We shall work in the car frame. As it is accelerated with respect to the road, we shall have to apply
a pseudo force ma0 on the bob of mass m.
For mean position, the acceleration of the bob with respect to the car should be zero. If  be the
angle made by the string with the vertical, the tension, weight and the pseudo force will add to zero
in this position.

Hence, resultant of mg and ma0 (say F = m g2  a 20 ) has to be along the string.

ma0 a0
 tan = mg = g

Now, suppose the string is further deflected by an angle  as


shown in figure.
Now, restoring torque can be given by
(F sin )  = – m  2 
Substituting F and using sin for small 

( m g2  a 20 )   = – m  2 

g2  a 20 g2  a 20
or, -  so; 2 =
 
This is an equation of simple harmonic motion with time period

2 
T= = 2 2
 ( g  a 20 )1 / 4

KVPY_ SA STREAM # 262




10.2 If forces other then m g acts then :

  F
T = 2 g where geff. = g 
eff . m

F = constant force acting on ‘m’.

Example 17. A simple pendulum of length ‘’ and having bob of mass ‘m’ is doing angular SHM inside water. A
constant buoyant force equal to half the weight of the bob is acting on the ball. Find the time period
of oscillations?

mg / 2 2
Solution : Here geff. = g – = g/2. Hence T = 2 g
m

11. COMPOUND PENDULUM / PHYSICAL PENDULUM S S

When a rigid body is suspended from an axis and made to oscillate about that

then it is called compound pendulum. 

C = Position of center of mass C


C
S = Point of suspension
 = Distance between point of suspension and center of mass
(it remains constant during motion)
For small angular displacement “” from mean position
The restoring torque is given by
mgsin
mg  for small , sin ~ 
or,  mg where,  = Moment of inertia about point of suspension.
mg mg
or,   or, 2 =
 

Time period, T = 2 mg  = CM + m2
Where CM = moment of inertia relative to the axis which passes from the center of mass & parallel to the
axis of oscillation.

 CM  m  2
T = 2
mg
where CM = mk2
k = gyration radius (about axis passing from centre of mass)

mk 2  m 2 k 2  2 L eq
T = 2 T = 2 = 2
mg g g

k2
Leq =   = equivalent length of simple pendulum ;

T
T is minimum when  = k.

2k
Tmin = 2 g 2k
T  2
g
Graph of T vs  = k 

KVPY_ SA STREAM # 263


Example 18. A uniform rod of length 1.00 m is suspended through an end and is set into oscillation with small
amplitude under gravity. Find the time period of oscillation. (g = 10 m/s2)
Solution : For small amplitude the angular motion is nearly simple harmonic and the time period is given by


T = 2 mg( / 2)

(m  2 / 3 )
= 2
mg( / 2)

2
= 2 3g

2  1.00 m 2
= 2 = s.
3  10 m / s 2 15


12. TORSIONAL PENDULUM
In torsional pendulum, an extended object is suspended at the cen-
tre by a light torsion wire. A torsion wire is essentially inextensible,
but is free to twist about its axis. When the lower end of the wire is
rotated by a slight amount, the wire applies a restoring torque caus- A
ing the body to oscillate (rotate) about vertical wire, when released. 

The restoring torque produced is given by C A X C X


C where, C = Torsional constant
or,  C where,  = Moment of inertia about the vertical axis.

C
or,   θ


Time Period, T = 2
C

Example 19. A uniform disc of radius 5.0 cm and mass 200 g is fixed at its centre to a metal wire, the other end
of which is fixed to a ceiling. The hanging disc is rotated about the wire through an angle and is
released. If the disc makes torsional oscillations with time period 0.20 s, find the torsional constant
of the wire.
Solution : The situation is shown in figure. The moment of inertia of the disc about the wire is

mr 2 (0.200 kg)(5.0  10 2 m )2
= = = 2.5 × 10–4 kg - m2.
2 2
The time period is given by

 4 2  4 2 (2.5  10 4 kg  m 2 ) kg  m2
T = 2 or, C = = = 0.25 .
C T2 (0.20 s)2 s2

KVPY_ SA STREAM # 264


1. A man of mass 60 kg standing on a platform executing S.H.M. in the vertical plane. The displacement from
the mean position varies as y = 0.5 sin (2 ft). The value of f, for which the man will feel weightlessness at the
highest point is: (y is in metres)

g 2g
(A) (B) 4 g (C) (D) 2 2g
4 2
2. A street car moves rectilinearly from station A (here car stops) to the next station B (here also car stops) with
an acceleration varying according to the law f = a - bx, where a and b are positive constants and x is the
distance from station A. The distance between the two stations & the maximum velocity are
2a a b a a b a a
(A) x  b ; v max  (B) x  ; v max  (C) x  2b ; v max  (D) x  ; v max 
b 2a b a b b
3. A particle performs SHM with a time period T and amplitude a. The magnitude of average velocity of the
a
particle over the time interval during which it travels a distance from the extreme position is
2
a 2a 3a a
(A) (B) (C) (D)
T T T 2T
4. A particle undergoes SHM with a time period of 2 seconds. In how much time will it travel from its mean
position to a displacement equal to half of its amplitude
(A) 1/2 sec (B) 1/3 sec (C) 1/4 sec (D) 1/6 sec.
5. For a particle in S.H.M., if the amplitude of displacement is ‘a’ and the amplitude of velocity is ‘v’ the
amplitude of acceleration is
v2 v2 v
(A) va (B) (C) (D)
a 2a a
6. A particle performs S.H.M. on xaxis with amplitude A and time period T. The time taken by the particle to
travel a distance A/5 starting from rest is:
T T 4 T  1 T  1
(A) (B) cos1   (C) cos1   (D) sin1  
20 2  5 2  5 2  5

 
7. Two SHM are represented by equations, y1 = 6 cos  6  t   , y = 3 ( 3 sin 3  t  cos 3  t )
 6 2
(A) ratio of their amplitudes is 1 (B) ratio of their time periods is 1
(C) ratio of their maximum velocities is 1 (D) ratio of their maximum acceleration is 1.
x
8. The oscillations represented by curve 1 in the graph are 2
expressed by equation x = A sin t. The equation for the A1
oscillations represented by curve 2 is expressed as : t

(A) x = 2A sin(t – /2) (B) x = 2A sin(t + /2)


(C) x = – 2A sin(t – /2) (D) x = A sin(t – /2) -2A

9. The energy of a particle executing simple harmonic is given by E = ax2 + bv 2 where ‘x’ is the displacement
from mean position x = 0 and v is the velocity of the particle at x then choose the INCORRECT statement.
E E
(A) amplitude of S.H.M is (B) Maximum velocity of the particle during S.H.M is
a b
b
(C) Time peried of motion is 2
a
(D) displacement of the particle is proportional to the velocity of the particle.

KVPY_ SA STREAM # 265


10. A loaded vertical spring executes SHM with period of 4s. The difference between the kinetic energy and
the potential energy of this system oscillates with a period of
(A) 8 s (B) 1 s (C) 2s (D) 4s

11. The coefficient of friction between block of mass m and 2m is  = 2 tan .


There is no friction between block of mass 2m and inclined plane. The
maximum amplitude of two block system for which there is no relative
motion between both the blocks.
k mg sin 
(A) g sin  (B)
m k
3mg sin 
(C) (D) None of these
k

12. A horizontal spring–block system of mass 2kg executes S.H.M. When the block is passing through its
equilibrium position, an object of mass 1kg is put gently on it and the two move together. The new amplitude
of vibration is (A being its initial amplitude):

2 3 A
(A) A (B) A (C) 2A (D)
3 2 2

13. A constant force produces maximum velocity V on the block connected to the spring of force constant
K as shown in the fig. When the force constant of spring becomes 4K, the maximum velocity of the
block is (block is at rest when spring is relaxed) :

(A) V/4 (B) 2 V (C) V/2 (D) V

14. A system is shown in the figure. The time period for small oscillations of the two blocks will be.

3m 3m 3m 3m
(A) 2  (B) 2  (C) 2  (D) 2 
k 2k 4k 8k

15. A block of mass ‘ m ‘ is attached with two springs of spring constant ‘ k


‘ is performing SHM on a smooth horizontal surface. One of the spring
is cut when the block is at extreme position. Find the ratio of the
amplitude of new SHM & old SHM.
1 1
(A) (B) (C) 1 (D) 2
2 2

16. A block of mass ‘m’ is suspended from a spring and executes vertical SHM of
time period T as shown in figure. The amplitude of the SHM is A and spring is
never in compressed state during the oscillation. The magnitude of minimum
force exerted by spring on the block is

4 2 4 2
(A) mg  mA (B) mg  mA
T2 T2

2 2
(C) mg  mA (D) mg  mA
T2 T2

KVPY_ SA STREAM # 266


17. A horizontal rod of mass m and length L is pivoted smoothly at one end. The rod’s other end is supported
by a spring of force constant k. The rod is rotated (in vertical plane) by a small angle  from its
horizontal equilibrium position and released.

The angular frequency of the subsequent simple harmonic motion is :

3k k 3 k 3g k
(A) (B) (C)  (D)
m 3m m 2L m

18. A simple pendulum 50 cm long is suspended from the roof of a cart


accelerating in the horizontal direction with constant acceleration
3 g m/s2. The period of small oscillations of the pendulum about
its equilibrium position is (g = 2 m/s2) :
(A) 1.0 sec (B) 2 sec
(C) 1.53 sec (D) 1.68 sec

19. A metre stick swinging in vertical plane about an fixed horizontal axis
passing through its one end undergoes small oscillation of frequency f 0.
If the bottom half of the stick is cut off, then its new frequency of small
oscillation would become:
(A) f 0 (B) 2 f0 (C) 2f 0 (D) 2 2 f 0

20. Two particle of mass m each are fixed to a massless rod of length 2 . The
rod is smoothly hinged at one end to a ceiling. It performs oscillation of
small angle in vertical plane. The length of the equivalent simple pendulum
m
is
3 10
(A) (B) m
2 3

5
(C) (D) None of these
3

21. A uniform disc of mass m & radius R is pivoted at its centre O with its
R/2
R
plane vertical as shown in figure. A circular portion of disc of radius is
2 O
removed from it. The time period of small oscillations of remaining portion
R
about O is -

R 13R 39R 7R
(A) 3 (B)  g
(C) 2 (D) 2 6g
g 16g

22. A disc is hinged such that it can freely rotate in a vertical plane about a point on its radius. If radius of disc is
'R', then what will be minimum time period of its simple harmonic motion?

R 3R 2R R
(A) 2 g (B) 2 2g (C) 2 (D) 2 2g
g

23. x = x1 + x2 (where x1 = 4 cos t and x2 = 3 sin t) is the equation of motion of a particle along x-axis. The
phase difference between x1 and x is :
(A) 37º (B) 53º (C) 90º (D) none of these

KVPY_ SA STREAM # 267


24. The magnitude of average acceleration in half time period from equilibrium position in a simple harmonic
motion is

2 A 2 A 2 A2
(A) (B) (C) (D) Zero
 2 2

25. Acceleration a versus time t graph of a body in SHM is given by a curve shown below. T is the time period.
Then corresponding graph between kinetic energy KE and time t is correctly represented by

(A) (B) (C) (D)

KVPY PROBLEMS (PREVIOUS YEARS)

1. Figure shows an assembly consisting of a number of pendulums


String
of varying lengths. The driver pendulum is pulled aside and released
Driver
so that it oscillates in a plane perpendicular to that of the diagram. pendulum
It will be observed that [KVPY_2007]
(A) all pendulums oscillate with the frequency of the driver pendulum and have the same amplitude.
(B) pendulums oscillate with different frequencies but equal amplitude, the shortest pendulum oscillating
with the highest frequency.
(C) all pendulums oscillate with the frequency of the driver pendulum; the pendulum with length equal to that
of the driver has the greatest amplitude.
(D) pendulums oscillate with different frequencies and different amplitudes.

2. A hollow pendulum bob filled with water has a small hole at the bottom through which water escapes at a
constant rate. Which of the following statements describes the variation of the time period (T) of the pendulum
as the water flows out ? [KVPY-2010]
(A) T decreases first and then increases. (B) T increases first and then decreases.
(C) T increases throughout. (D) T does not change.

3. There is a smooth fixed concave surface. A particle is released from p. Find : [KVPY-2010]

(i) PE as function of 
(ii) KE as a function 
(iii) time taken from P to 
(iv) the reaction force at 

KVPY_ SA STREAM # 268


4. A simple pendulum oscillates freely between points A and B. [KVPY_2011]

We now put a peg (nail) at some point C as shown. As the pendulum moves from A to the right, the string will
bend at C and the pendulum will go to its extreme point D. Ignoring friction, the point D.
(A) will lie on the line AB (B) will lie above the line AB
(C) will lie below the line AB (D) will coincide with B

5. A large number of random snap shots using a camera are taken of a particle in simple harmonic motion
between x = -x0 and x = +x0 with origine x = 0 as the mean position. A histogram of the total number of times
the particle is recorded about a given position (Event no.) would most closely resemble : [KVPY_2014_SA]
Event no.

Event no.

Event no.

Event no.
(A) (B) (C) (D)
-x0 0 x0 x -x0 0 x0 x -x0 0 x0 x -x0 0 x0 x

6. A simple pendulum is attached to a block which slides without friction down


an inclined plane (ABC) having an angle of inclination  as shown
while the block is sliding down the pendulum oscillates in such a way that at
its mean position the direction of the string is : [KVPY_2014_SB]
(A) at angle  to the perpendicular to the inclined plane AC.
(B) parallel to the inclined plane AC.
(C) vertically downwards.
(D) perpendicular to the inclined plane AC.

7. A simple pendulum of length l is made to oscillate with amplitude of 45 degrees. The acceleration due

to gravity is g. Let T0  2 l / g . The time period of oscillation of this pendulum will be


(A) T0 irrespective of the amplitude [KVPY_2015_SB] [1 Mark]
(B) Slightly less than T0
(C) Slightly more then T0
(D) Dependent on whether it swings in a plane aligned with the north-south or east west directions.

8. A light balloon filled with helium of density He is tied to a long light string of length l and the string is attached
to the ground. If the balloon is displaced slightly in the horizontal direction from the equilibrium and released
then : [KVPY_2016_SB] [2 Mark]

 air l
(A) The ballon undergoes simple harmonic motion with period 2     
 air He g

 air  He l
(B) The ballon undergoes simple harmonic motion with period 2   
 air g

 He  l
(C) The ballon undergoes simple harmonic motion with period 2      g
 air He 

 air  He l
(D) The ballon undergoes conical oscillations with period 2     
 air He g

KVPY_ SA STREAM # 269


9. A potential is given by V(x) = (x + ) 2/2 for x < 0 and V (X) = k (X - )2/2 for x >0. The schematic varitation
of oscillation period (T) for a performating periodic motion in this potential as a function of its energy E is:
[KVPY-SA_2018 2 Mark]

(A) (B)
E

T T

(C) (D)
E E

EXERCISE - 1
1. (C) 2. (A) 3. (C) 4. (D) 5. (B) 6. (B) 7. (A)
8. (A) 9. (D) 10. (C) 11. (C) 12. (A) 13. (C) 14. (C)
15. (C) 16. (A) 17. (A) 18. (A) 19. (B) 20. (C) 21. (B)
22. (C) 23. (A) 24. (A) 25. (A)

EXERCISE - 2
1. (B) 2. (B) 4. (A) 5. (C) 6. (D) 7. (C) 8. (C)

9. (B)

KVPY_ SA STREAM # 270


1. If he feels weightlessness then at the highest point, acceleration must be g.
2g
 g = 2A  = 2f = 2g  f=
2

2. f = a – bx
For maximum velocity, acceleration should be zero.
a
i.e. a – bx = 0  x=
b
a
 At x = , the particle has its maximum velocity..
b

vdv v2 bx 2
f= = a – bx  = ax  c
dx 2 2
At x = 0 ; v = 0  c=0
a
Substituting ; x= ; gives
b
a
v max =
b
Also, the velocity of the car should become zero at station B.

bx 2  2a 
i.e. ax  0  x=0;x=  
2  b 

2a
 Distance between the cars is
b
Alternate : f = a – bx means particle will do SHM.
At mean position ; f = 0
a
 x=
b

In the figure shown, 'C' is the mean position and A & B are extreme positions
2a a a
 x max = & Vmax = A = b.  .
b b b

a
3. The magnitude of displacement in the given time interval =
2
a T
Time taken by the particle to cover a distance starting from rest =
2 6
a / 2 3a
Hence the magnitude of average velocity over given time interval is = =
T/6 T

2 A A 2
4. x = A sin t ; for x =  = A sin t
T 2 2 T
T
Solving t = .
12
5. Maximum velocity v =  a
v2
Maximum acceleration f = 2 a  f=
a
KVPY_ SA STREAM # 271
6. Particle is starting from rest, i.e. from one of its extreme position.
A
As particle moves a distance , we can represent it on a circle as shown. y
5
4A / 5 4 4
cos  =   = cos–1  
A 5 5
4A/5 A/5
P
4 1 4 x
  
t = cos–1 t= cos–1  
5  5 A

Q
T 4
= cos–1  
2 5
Method : As starts from rest i.e. from extreme position x = A sin (t +  )
 A
At t = 0 ; x = A  =  A– = A cos t
2 5
4 4
= cos t  t = cos–1
5 5
T 4
t= cos–1  
2 5

 
7. y1 = 6 cos  6  t  
 6
 
y2 = 6 sin  3  
 3
A1/A2 = 1 : 1
8. Oscillations represented by curve 2 lags in phase by /2 and the periods are same. Amplitude of curve
2 is double that of 1.
9. amplitude is obtained for v = 0
E
 A=
a
Maximum velocity is obtained for x = 0
E
Vmax =
b
Vmax = A 
E
b a

= E b
a
2 b
T=  2
 a

1
10. (Moderate) KE = m2 a2 cos2 t
2
1
PE = m2 a2 sin2 t
2
1
KE – PE = m2 a2 (cos2 t – sin2 t)
2
1
m2 a2 cos2 t
=
2
 Angular frequency = 2
T
 The time period = = 2 s
2
KVPY_ SA STREAM # 272
11. The maximum static frictional force is
f = µmg cos = 2 tan  mg cos  = 2 mg sin mean position (e k/ ; fLFkfr )
A
Applying Newton’s second law to block at lower extreme position f
extreme position
(p j e fLFkfr )
f – mgsin = m2A  f = m2A + mg sin  mgsinq

3 mg sin  q
or 2A = g sin  or A=
k

12. Conserving momentum : 2V = 3V


2
 V = V..
3
1 1 1
Ei = m V 2 = .2.V2 = V2  KA2 = V2.
2 1 1 2 2
1 1 2 2 2
Ef = .m 2 V22 = .3. . V2 = V2
2 2 3 3 3

1 2 2
 KA2 = V2 = Ei (  Ei = V2 from above)
2 3 3

1 2 1 2
 KA2 = ( KA2)  A = A Ans.
2 3 2 3
13. By work energy theorem;
1 1
Fx 1 – kx 2 = mv 2  (1)
2 1 2
1 1
and Fx 2 – k'x 22 = mv'2  (2)
2 2
; where x 1, x 2 are initial and final extensions and
v, v' are initial and final velocities.
In both cases : force applied is same, and velocity becomes maximum when F = kx.
(after which the mass will decelerate)
 F = kx 1 = (4k)x 2
x1
 x2 =
4
Substituting in (2) :
2
Fx1 1  x1 
– (4k)  
4 2  4 

1 1 1
 [Fx 1 – kx 12] = mv'2  (3)
4 2 2
Dividing (3)/(1) ; we get :
1 v'2 v
= 2  v'= .
4 v 2

14. Both the spring are in series


K(2K ) 2K
 Keq = =
K  2K 3
Time period
 m1m 2
T = 2 where  =
K eq m1  m 2

m m 3 3m
Here =  T = 2  = 2
2 2 2K 4K

KVPY_ SA STREAM # 273


Method II

k
m m
/////////////////////////////////////////////////////////////////////////////////////

k
m m
/////////////////////////////////////////////////////////////////////////////////////////////////////////////////////////////////////

x1 x2

 mx 1 = mx 2  x 1 = x 2
force equation for first block;
2k d2 x 1
(x 1 + x 2 ) = –m
3 dt 2

d2 x1 4k
Put x 1 = x 2  + × x1 = 0
dt 2 3m

4k 3m
 2 =  T = 2
3m 4K
15. Amplitude of the SHM is determined by the initial condition and is independent of the spring constant.

16. The spring is never compressed. Hence spring shall exert least force on the block when the block is at
topmost position.

17. Restoring torque :


 = ky L = KL2 (Since y  L from figure)
2
mL
 kL2 = .
3
3k
= ·
m
3k
 = (torque due to mg was already balanced so it is not taken in calculation)
m

18. With respect to the cart, equilibrium position of the pendulum is


shown.
If displaced by small angle  from this position, then it will execute SHM about this equilibrium position,
time period of which is given by :

L
T = 2 g
eff

 geff = g 2  ( 3g ) 2
 geff = 2g
 T = 1.0 second

KVPY_ SA STREAM # 274


1 mg
19. f0 =
2 
where,  is distance between point of suspension and centre of mass of the body.
Thus, for the stick of length L and mass m :

L
1 m.g. 1 6g
f0 = 2 =
2 2
(mL / 12) 2  L

when bottom half of the stick is cut off

m L
.g.
1 2 4 1 12g
f 0’ = =  = 2 f0
2 m (L / 2)2 2 L
2 12


T = 2 mg ,  = m + m( 2 )2 = 5m 2
2
20.

5m 2 5
= 2 = 2 3g
3
2mg
2

5
 Leq =
3


21. T = 2 mgy
cm

MR 2  1 M  R 2 M  R 2  13MR 2
= –        =
2 2 4 2 42  32
 

R 3M
 ycm = &m=
6 4

13 R
T = 2 4 g

22. For minimum time period

mR 2 mR 2
R  2R
x=  T = 2 2 2 = 2
2 mgR g
2

23. X1 = 4cost
X2 = 3sint
so X = 4cost + 3sint
4 3 
= 5  cos t  sin t 
 5 5 
= 5 [cos37º. cost + sin37º. sint]
= 5 cos(t – 37º)
so phase diffrence between X1 and X2 is 37º

KVPY_ SA STREAM # 275


F L U I D M ECH AN I CS

D EFI N I TI ON OF FLUI D
The term fluid refers to a substance that can flow and does not have a shape of its own. For
example liquid and gases.
Fluid includes property (A) Density (B) Viscosity (C) Bulk modulus of elasticity (D) pressure
(E) specific gravity

P R ESSUR E I N A FLUI D
The pressure p is defined as the magnitude of the normal force acting on a unit surface area.

F
P =
A

F = normal force on a surface area A.

The pressure is a scalar quantity. This is because hydrostatic pressure is transmitted equally in
all directions when force is applied, which shows that a definite direction is not associated with
pressure.
Thrust. The total force exerted by a liquid on any surface in contact with it is called thrust of the liquid.

CON S EQUEN CES OF P R ES S UR E


(i) Railway tracks are laid on large sized wooden or iron sleepers. This is because the weight (force)
of the train is spread over a large area of the sleeper. This reduces the pressure acting on the
ground and hence prevents the yielding of ground under the weight of the train.
(ii) A sharp knife is more effective in cutting the objects than a blunt knife.
The pressure exerted = Force/area. The sharp knif e transmits force over a small area as
compared to the blunt knif e. Hence the pressure exerted in case of sharp knife is more than in
case of blunt knife.
(iii) A camel walks easily on sand but a man cannot inspite of the fact that a camel is much heavier
than man.
This is because the area of camel’s feet is large as compared to man’s feet. So the pressure
exerted by camel on the sand is very small as compared to the pressure exerted by man. Due to
large pressure, sand under the feet of man yields and hence he cannot walk easily on sand.

VAR I ATI ON OF P R ESS UR E W I TH H EI GH T


Assumptions : (i) unaccelerated liquid (ii) uniform density of liquid (iii) uniform gravity
dp
W eight of the small element dh is balanced by the excess pressure. It means = g.
dh

P h

 dp  g
 dh
Pa 0

 P = P a + gh

KVPY_ SA STREAM # 276


P AS CAL’ S LAW
if the pressure in a liquid is changed at a particular, point the change is transmitted to the entire
liquid without being diminished in magnitude. In the above case if P a is increased by some amount
than P must increase to maintained the diff erence (P – P a) = hg. This is Pascal’s Law which
states that Hydraulic lift is common application of Pascal’s Law.

1. Hydraulic press.
f W W
P =  or f  a
a A A
as A >> a then f << W..
This can be used to lift a heavy load placed on the platform of larger piston or to press the things
placed between the piston and the heavy platform. The work done by applied f orce is equal to
change in potential energy of the weight in hydraulic press.

Example 1. The area of cross-section of the two arms of a hydraulic press are 1 cm 2 and 10 cm 2
respectively (figure). A force of 50 N is applied on the water in the thicker arm. What force
should be applied on the water in the thinner arm so that the water may remain in
equilibrium?

Solution : In equilibrium, the pressures at the two surfaces should be equal as they lie in the same
horizontal level. If the atmospheric pressure is P and a force F is applied to maintain the
equilibrium, the pressures are
50 N F
P0 + and P 0 + respectively..
10cm2 1 cm 2

This givens F = 5 N.


2. Hydraulic Brake.
Hydraulic brake system is used in auto-mobiles to retard the motion.

KVPY_ SA STREAM # 277


H YD R OS TATI C P AR AD OX
Pressure is directly proportional to depth and by applying pascal’s law it can be seen that pres-
sure is independent of the size and shape of the containing vessel. (In all the three cases the
heights are same).

A B C
PA = PB = PC

ATM OS P H ER I C P R ES S U R E
Definition.
The atmospheric pressure at any point is numerically equal to the weight of a column of air of unit
cross-sectional area extending from that point to the top of the atmosphere.
At 0ºC, density of mercury = 13.595 g cm –3, and at sea level, g = 980.66 cm s –2
Now P = hg.
Atmospheric pressure = 76 × 13.595 × 980.66 dyne cm –2 = 1.013 × 10 5 N-m 2 (p a)

Height of Atmosphere
The standard atmospheric pressure is 1.013 × 10 5 Pa (N m –2). If the atmosphere of earth has a
uniform density  = 1.30 kg m –3, then the height h of the air column which exerts the standard
atmospheric pressure is given by
 hg = 1.013 × 10 5

1.013  10 5 1.013  10 5
h= = m = 7.95 × 10 3 m  8 km.
g 1.13  9.8
In fact, density of air is not constant but decreases with height. The density becomes half at
1
about 6 km high, th at about 12 km and so on. Therefore, we can not draw a clear cut line above
4
which there is no atmosphere. Anyhow the atmosphere extends upto 1200 km. This limit is con-
sidered for all practical purposes.

M EAS UR EM EN T OF ATM OS P H ER I C P R ES S UR E
1. Mercury Barometer.
To measure the atmospheric pressure experimentally, torricelli invented a mercury barometer in
1643.

P a =hg

The pressure exerted by a mercury column of 1mm high is called 1 Torr.


1 Torr = 1 mm of mercury column
2. Open tube Manometer
Open-tube manometer is used to measure the pressure gauge. W hen equilibrium is reached, the
pressure at the bottom of left limb is equal to the pressure at the bottom of right limb.

Pa
y = y2–y1
P y2

P+y1 g y1
Pa+y2 g

KVPY_ SA STREAM # 278


i.e. P + y 1 g = P a + y 2 g
P – P a = g (y 2 – y 1) = gy
P – P a = g (y 2 – y 1) = gy
P = absolute pressure, P – P a = gauge pressure.
Thus, knowing y and  (density of liquid), we can measure the gauge pressure.

Example 2. The manom et er shown bel ow is used t o measure t he di f f erence i n water l ev el


between the two tanks. Calculate this difference for the conditions indicated.

Solution : P a + h 1 g – 401g + 40g = P a + h 2 g


h 2 g – h 1 g = 40 g – 40 1g

as 1 = 0.9
(h 2 – h 1) g = 40g – 36g
h 2 – h 1 = 4 cm


3. Water Barometer.
Let us suppose water is used in the barometer instead of mercury.

1.013  10 5
hg = 1.013 × 10 5 or h=
g
The height of the water column in the tube will be 10.3 m. Such a long tube cannot be managed
easily, thus water barometer is not feasible.

Example 3. I n a g i v e n U - t u b e ( o p e n a t o n e - e n d ) f i n d o u t r e l a t i o n b e t we e n P a n d P a .
Given d 2 = 2 × 13.6 gm/cm 3 d 1 = 13.6 gm/cm 3

y=25 cm d2 P

x=26cm

d1

KVPY_ SA STREAM # 279


Solution : Pressure in a liquid at same level is same i.e. at A – A–,
Pa  d2 yg  xd1g  P
Pa

y d2 P

x
A A

d
In C.G.S. 1

P a + 13.6 × 2 × 25 × g + 13.6 × 26 × g = P
P a + 13.6 × g [50 + 26] = P
2P a = P [P a = 13.6 × g × 76]

Example 4. Find out pressure at points A and B. Also find angle ‘’.

Solution : Pressure at A – P A = P atm – 1 gl sin 


Pressure at B P B = P atm + 2 gh 
But P B is also equal to P B = P A + 3 gl sin 
Hence - P atm + 2 gh = P A + 3 gl sin 
P atm + 2 gh = P atm – 1 gl sin  + 3 gl sin 
 2h
sin  = (   ) .
3 1

Example 5. In the given figure, the container slides down with acceleration
‘a’ on an incline of angle ‘’. Liquid is stationary with respect
to container. Find out -
(i) Angle made by surface of liquid with horizontal plane.
(ii) Angle if a = g sin .

Solution : Consider a fluid particle on surface. The forces acting on it are shown in figure.

Resultant force acting on liquid surface, will always normal to it


ma cos  a cos 
tan  = =
mg  ma sin  (g  a sin )

KVPY_ SA STREAM # 280


a cos 
Thus angle of liquid surface with the horizontal is equal to  = tan –1
(g  a sin )

 a cos   g sin  cos 


(ii) If a = g sin , then  = tan –1  2 
 = tan –1
 g  g sin   g cos 2 
= tan –1 (tan ) =

Example 6. W ater and liquid is filled up behind a square wall of side  . Find out

(a) Pressures at A, B and C (b) Forces in part AB and BC


(c) Total force and point of application of force. (neglect atmosphere pressure in every
calculation)
Solution : (a) As there is no liquid above ‘A’,
So pressure at A, P A = 0
Pressure at B, P B = gh 1
Pressure at C, P C = gh 1 + 2gh 2
(b) Force at A = 0
Take a strip of width ‘dx’ at a depth ‘x’ in part AB.
Pressure is equal to gx.
Force on strip = pressure × area
dF = gx  dx
Total force upto B
h1
gxh12 1000  10  10  5  5
F=  gxdx
0
=
2
=
2
= 1.25 × 10 6 N
In part BC for force take a elementary strip of width dx in portion BC. Pressure is equal to
=gh 1 + 2g(x – h 1)
Force on elementary strip = pressure × area
dF = [gh 1 + 2g(x – h 1)]  dx
Total force on part BC

F= 
h1
[ gh1  2 g ( x  h1 )]  dx


  x2 
=  gh1x  2 g  2  h1x  
    h1

  2  h12 
= gh 1h 2  + 2g   2  h1  h12 
 

2g
= gh 1h 2 + [  2 + h 12 – 2h 1 ] = gh 1h 2 +g  (  – h 1) 2
2
= gh 2 [h 1 + h 2] = gh 2 2 = 1000 × 10 × 5 × 10 × 10 = 5 × 10 6 N

KVPY_ SA STREAM # 281


(c) Total force = 5 × 10 6 + 1.25 × 10 6 = 6.25 × 10 6 N
Taking torque about A
h1

Total torque of force in AB =  dF  x =  gxdx.x


0

h
 gx 3  1 gh13 1000  10  10  125 1.25  10 7
=  3  = = = N-m
  0 3 3 3

Total torque of force in BC =  dF  x


h2 2h 2
On solving we get = gh 1h 2 [h 1 + ] + gh 22 [h 1 + ]
2 3

10
= 1000 × 10 × 5 × 5 × 10 [5 + 2.5] + 1000 × 10 × 25 × 10 [5 + ]
3

62.5 118.75
= 2.5 × 7.5 × 10 6 + × 10 6 = × 10 6
3 3

11.875  10 7 1.25  10 7 13.125  10 7


Total torque = + =
3 3 3
Total torque = total force × distance of point of application of force from top = F . x p

13.125  10 7
6.25 × 10 6 x p =
3
x p = 7m
Alternatively
W e can solve this problem by pressure diagram also.
Force on ‘AB’ part is area of triangle ‘ABC’

h1 gh12  2
F AB = gh 1 × × =
2 2
Torque of force of AB part about A -

gh12  2h1
 AB = 
2 3

gh13  g 4
= 
3 24
Force on ‘BC’ part is area of trapezium -
h2
F BC = gh 1h 2 + 2gh 2 ×  = gh 1h 2 + gh 22
2
Torque of force of ‘BC’ part about ‘A’ -
h2 2h 2
 BC = gh 1h 2 (h 1 + ) + gh 22 (h 1 + )
2 3

g 3   3      g 3    g 4


=  2  4  + g   =   3  4  = 19
4   4 2 3 4   48

KVPY_ SA STREAM # 282


gh12
Total force = + gh 1h 2 + gh 22
2

g 3 3 g 3 1  1  1 5g 3


= + g +  =
8 4 4  2  8

19g 4 g 4 21g 4


Total torque = + =
48 24 48

21g 4 5 g  3 21g 4
But F xp =  p =
48 8 48

21 21 10
xp = = = 7m
30 30
Thus total force is acting at 7m below A point.


AR CH I M ED ES’ P R I N CI P LE
According to this principle, when a body is immersed wholly or partially in a fluid, it loses its
weight which is equal to the weight of the fluid displaced by the body.
Up thrust = buoyancy = Vg V = volume submerged = density of liquid.
Relation between density of solid and liquid
weight of the floating solid = weight of the liquid displaced
1 V2
V 1 1 g = V 2 2 g  
2 V1

Density of solid Volume of the immeresed portion of the solid


or 
Density of liquid Total Volume of the solid
This relationship is valid in accelerating fluid also. Thus, the force acting on the body are :
(i) its weight Mg which acts downward and
(ii) net upward thrust on the body or the buoyant force (mg)
Hence the apparent weight of the body = Mg – mg = weight of the body – weight of the displaced liquid.
Or Actual Weight of body – Apparent weight of body = weight of the liquid displaced.
The point through which the upward thrust or the buoyant force acts when the body is immersed
in the liquid is called its centre of buoyancy. This will coincide with the centre of gravity if the solid
body is homogeneous. On the other hand if the body is not homogeneous, then the centre of
gravity may not lie on the line of the upward thrust and hence there may be a torque that causes
rotation in the body.
If the centre of gravity of the body and the centre of buoyancy lie on the same straight line, the
body is in equilibrium.
If the centre of gravity of the body does not coincide with the centre of buoyancy (i.e., the line of
upthrust), then torque acts on the body. This torque causes the rotational motion of the body.

Example 7. A metal piece of mass 10 g is suspended by a vertical spring. The spring elongates 10 cm
over its natural length to keep the piece in equilibrium. A beaker containing water is now
placed below the piece so as to immerse the piece completely in water. Find the elongation
of the spring. Density of metal = 9000 kg/m 3. Take g = 10 m/s2.
Solution : Let the spring constant be k. W hen the piece is hanging in air, the equilibrium condition
gives
k (10 cm) = (0.01 kg) (10 m/s 2)
or k (10 cm) = 0.1 N. .............(i)

KVPY_ SA STREAM # 283


The volume of the metal piece
0.01kg 1
= = × 10 –5 m 3.
9000 kg / m 3 9
This is also the volume of water displaced when the piece is immersed in water. The force
of buoyancy
= weight of the liquid displaced
1
= × 10 –5 m 3 × (1000 kg/m 3) × (10 m/s2)
9
= 0.011 N.
If the elongation of the spring is x when the piece is immersed in water, the equilibrium
condition of the piece gives,
kx = 0.1 N – 0.011 N = 0.089 N. ..............(ii)
0.089
By (i) and (ii), x = cm = 0.0089 cm.
10

Example 8. A cubical block of plastic of edge 3 cm floats in water. The lower


surface of the cube just touches the free end of a vertical spring
fixed at the bottom of the pot. Find the maximum weight that can be
put on the block without wetting it. Density of plastic = 800 kg/m 3
and spring constant of the spring = 100 N/m. Take g = 10 m/s 2.

Solution : The specific gravity of the block = 0.8. Hence the height inside water = 3 cm × 0.8 = 2.4
cm. The height outside water = 3 cm – 2.4 = 0.6 cm. Suppose the maximum weight that
can be put without wetting it is W. The block in this case is completely immersed in the
water. The volume of the displaced water
= volume of the block = 27 × 10 –6 m 3.
Hence, the force of buoyancy
= (27 × 10 –6 m 3) × 1(1000 kg/m3) × (10 m/s 2) = 0.27 N.
The spring is compressed by 0.6 cm and hence the upward force exerted by the spring
= 100 N/m × 0.6 cm = 0.6 N.
The force of buoyancy and the spring force taken together balance the weight of the block
plus the weight W put on the block. The weight of the block is
W = (27 × 10 –6 m) × (800 kg/m 3) × (10 m/s2) = 0.22 N.
Thus, W = 0.27 N + 0.6 N – 0.22 N = 0.65 N.

Example 9. A wooden plank of length 2  m and uniform cross-section is hinged at one end to the
bottom of a tank as shown in figure. The tank is filled with water up to a height of  m. The
specific gravity of the plank is 0.5. Find the angle  that the plank makes with the vertical
in the equilibrium position. (Exclude the case  = 0)

Solution : The forces acting on the plank are shown in the figure. The height of water level is
 . The length of the plank is 2  . The weight of the plank acts through the centre B of the
plank. We have OB =  . The buoyant force F acts through the point A which is the middle
point of the dipped part OC of the plank.

OC 
We have OA = =
2 cos 
.
2
Let the mass per unit length of the plank be .
Its weight mg = 2  g.
KVPY_ SA STREAM # 284
  
The mass of the part OC of the plank =   .
 cos  
1  2
The mass of water displaced = = .
0.5 cos  cos 
2g
The buoyant force F is, theref ore, F = .
cos 
Now, for equilibrium, the torque of mg about O should balance the torque of F about O.
So, mg (OB) sin = F(OA) sin
 2     1 1
or, (2  )  =     or, cos2= or, cos = ,
 cos    2 cos   2 2
or,  = 45°.
Example 10. A cylindrical block of wood of mass m, radius r & density  is floating in water with its axis
vertical. It is depressed a little and then released. If the motion of the block is simple
harmonic. Find its frequency.
Solution : Suppose a height h of the block is dipped in the water in equilibrium position. If r be the
radius of the cylindrical block, the volume of the water displaced = r 2h. For floating in
equilibrium,
 r 2 hg = W ..............(i)
where  is the density of water and W the weight of the block.
Now suppose during the vertical motion, the block is further dipped through a distance x
at some instant. The volume of the displaced water is  r 2 (h + x). The forces acting on the
block are, the weight W vertically downward and the buoyancy  r 2(h + x) g vertically
upward.
Net force on the block at displacement x from the equilibrium position is
F = W – r 2 (h + x)g = W – r 2 hg – r 2xg
Using (i) F = – r 2 gx = – kx,
where k = r 2 g.
Thus, the block executes SHM with frequency.

1 k 1 r 2g
v = = .
2 m 2 m


P R ES SUR E I N CASE OF ACCELER ATI N G FLUI D
(i) Liquid Placed in elevator :
When elevator accelerates upward with acceleration a 0 then pressure in the fluid, at depth ‘h’ may
be given by,
P = h [g + a 0]
and force of buoyancy, B = m (g + a 0)

(ii) Free surface of liquid in horizontal acceleration :

a0
tan  =
g

a 0
P 1 – P 2 =   a 0 where P 1 and P 2 are pressures at point 1 & 2. Then h 1 – h 2 =
g

KVPY_ SA STREAM # 285


Example 11. An open rectangular tank 1.5 m wide 2m deep and 2m long is half filled with water. It is
accelerated horizontally at 3.27 m/sec 2 in the direction of its length. Determine the depth
of water at each end of tank. [g = 9.81 m/sec 2]
a 1
Solution : tan  = =
g 3
Depth at corner ‘A’
= 1 – 1.5 tan
= 0.5 m Ans.
Depth at corner ‘B’
= 1 + 1.5 tan  = 1.5 m Ans.

(constant)
(iii) Free surface of liquid in case of rotating cylinder.
r B
2 2 2 A
v  r h
h= =
2g 2g C

S TR EAM LI N E FLOW
The path taken by a particle in flowing fluid is called its line of flow. In the case of steady flow all
the particles passing through a given point follow the same path and hence we have a unique line
of flow passing through a given point which is also called streamline.
CH AR ACTER I STI CS OF S TR EAM LI N E
1. A tangent at any point on the stream line gives the direction of the velocity of the fluid particle at
that point.
2. Two streamlines never intersect each other.
Laminar flow : If the liquid flows over a horizontal surface in the form of layers of different veloci-
ties, then the flow of liquid is called Laminar flow. The particle of one layer do not go to another
layer. In general, Laminar flow is a streamline flow.
Turbulent Flow : The flow of fluid in which velocity of all particles crossing a given point is not
same and the motion of the fluid becomes disorderly or irregular is called turbulent flow.
R EYN OLD ’ S N UM BER
According to Reynolds, the critical velocity (v c ) of a liquid flowing through a long narrow tube is
(i) directly proportional to the coefficient of viscosity () of the liquid.
(ii) inversely proportional to the density  of the liquid and
(iii) inversely proportional to the diameter (D) of the tube.
 R v c D
That is v c  or v c = D or = ...............(1)
D 
where R is the Reynold number.
If R < 2000, the flow of liquid is streamline or laminar. If R > 3000, the flow is turbulent. If R l ies
between 2000 and 3000, the flow is unstable and may change from streamline flow to turbulent flow.

EQUATI ON OF CON TI N UI T Y
The equation of continuity expresses the law of conservation of mass in fluid dynamics.

a1 v 1 = a2 v 2

KVPY_ SA STREAM # 286


In general av = constant . This is called equation of continuity and states that as the area of
cross section of the tube of flow becomes larger, the liquid’s (fluid) speed becomes smaller and
vice-versa.
Illustrations -
(i) Velocity of liquid is greater in the narrow tube as compared to the velocity of the liquid in
a broader tube.
(ii) Deep waters run slow can be explained from the equation of continuity i.e., av = constant.
W here water is deep the area of cross section increases hence velocity decreases.

EN ER GY OF A LI QUI D
A liquid can posses three types of energies :

(i) Kinetic energy :


The energy possessed by a liquid due to its motion is called kinetic energy. The kinetic energy of
1
a liquid of mass m moving with speed v is mv 2.
2

1
mv 2 1 2
 K.E. per unit mass = 2 = v .
m 2

(ii) Potential energy :


The potential energy of a liquid of mass m at a height h is m g h.
mgh
 P.E. per unit mass = = gh
m

(iii) Pressure energy :


The energy possessed by a liquid by v irtue of its pressure is called pressure energy.
Consider a vessel fitted with piston at one side (figure). Let this vessel is filled with a liquid. Let
‘A’ be the area of cross section of the piston and P be the pressure experienced by the liquid.
The force acting on the piston = PA
If dx be the distance moved by the piston, then work done by the force = PA dx = PdV
where dV = Adx, volume of the liquid swept.
This work done is equal to the pressure energy of the liquid.
 Pressure energy of liquid in volume dV = PdV.
The mass of the liquid having volume dV = dV,
 is the density of the liquid.
PdV P
 Pressure energy per unit mass of the liquid = = .
dV 

BER N OULLI ’ S TH EO R EM
It states that the sum of pressure energy, kinetic energy and potential energy per unit mass or per
unit volume or per unit weight is always constant f or an ideal (i.e. incompressible and
non-viscous) fluid having stream-line flow.
P 1 2
i.e. + v + gh = constant.
 2

KVPY_ SA STREAM # 287


Example 12. A circular cylinder of height h 0 = 10 cm and radius r 0 = 2cm is opened at the top and filled
with liquid. It is rotated about its vertical axis. Determine the speed of rotation so that half
the area of the bottom gets exposed. (g = 10 m/sec 2).
Solution : Area of bottom = r 02
If r is radius of the exposed bottom, then

1 r0
 r2 = r 02 r=
2 2
Applying Bernoulli’s equation between points (1) and (2) -
1 1
P atm + v 12 – gH = P atm + v 22 – g(H – h 0)
2 2
1
– gh 0 = (v 22 – v 12)  2gh 0 = [v 12 – v 22] = [w2r 02 – w2r 2]
2
r 0 = 2 × 10 –2 m  2gh 0 = w2 [r 02 – r 2]
2 2
w= r gh = 10  0.1 = 100 radian / sec.
0 2  10 2
Example 13. W ater flows in a horizontal tube as shown in figure. The pressure of water changes by 600
N/m 2 between x and y where the areas of cross-section are 3cm 2 and 1.5cm 2 respectively.
Find the rate of flow of water through the tube.

Solution : Let the velocity at x = v x and that at y = v y.


vy 3 cm2
By the equation of continuity, v = = 2.
x 1.5cm2
By Bernoulli’s equation,
1 1 1 1 3
Px +  v x2 = P y + v y2 or, Px – Py = (2v y)2 – v y2 = v y2
2 2 2 2 2

N3 1000 kg  2
or, 600 =
  v or, v x = 0.4 m 2 / s 2 = 0.63 m/s.
m 22  m3  x
The rate of flow = (3 cm 2) (0.63 m/s) = 189 cm 3/s.

AP P LI CATI ON OF BER N OULLI ’ S TH EOR EM
(i) Bunsen burner
(ii) Lift of an airfoil.
(iii) Spinning of a ball (Magnus effect)
(iv) The sprayer.
(v) A ping-pong ball in an air jet
(vi) Torricelli’s theorem (speed of efflux)
At point A, P 1 = P, v 1 = 0 and h 1 = h
At point B, P 2 = P, v 2 = v (speed of efflux) and h = 0
P1 1 2 P2 1 2
Using Bernoulli’s theorem + gh 1 + v1 = + gh 2 = v , we have
 2  2 2

P P 1 2 1 2
 + gh + 0 =  + 0 + 2 v  v = gh or v = 2gh
2

KVPY_ SA STREAM # 288


Example 14. A cylindrical container of cross-section area, A is filled up with water upto height ‘h’.
Water may exit through a tap of cross section area ‘a’ in the bottom of container. Find out

(a) Velocity of water just after opening of tap.


(b) The area of cross-section of water stream coming out of tape at depth h 0 below
tap in terms of ‘a’ just after opening of tap.
(c) Time in which container becomes empty.
1/ 2
a
(Given :    0.02 , h = 20 cm, h = 20 cm )
A 0

Solution : Applying Bernoulli’s equation between (1) and (2) -


1 1
P a + gh + v 12 = P a + v 22
2 2
Through continuity equation :
av 2 1 1
Av 1 = av 2, v 1 = gh + v 2 = v 22
a 2 1 2
2gh
on solving - v 2 = a 2 = 2m/sec. ....(1)
1
A2
(b) Applying Bernoulli’s equation between (2) and (3)
1 1
v 2 + gh 0 = v 32
2 2 2
Through continuity equation -
av 2
av 2 = a’ v 3  v3 =
a'

2
1 1  av 2 

 v 22 + gh 0 = 
2 2  a' 

2
1 1 a
× 2 × 2 + gh 0 =   ×2×2
2 2  a' 

2 2
a 9.8  .20 a a
  = 1+    = 1.98  a’ =
 a'  2  a'  1.98
(c) From (1) at any height ‘h’ of liquid level in container, the velocity through tap,

2gh
v = = 20 h
0.98
we know, volume of liquid coming out of tap = decrease in volume of liquid in
container.

KVPY_ SA STREAM # 289


For any small time interval ‘dt’
av 2dt = – A · dx
t 0
A dx
a 20 x dt = – A dx
0

 dt = –
a 
h
20 x

A A
t =
a 20 2 x  0
h
 t =
a 20 2 h

A h 2A 0.20 2A
= ×2 × = = × 0.1
a 20 a 20 a

1/ 2
a A 1
Given   = 0.02 or = = 2500
A a 0.0004
Thus t = 2 × 2500 × 0.1 = 500 second.

Example 15.

In a given arrangement
(a) Find out velocity of water coming out of ‘C’
(b) Find out pressure at A, B and C.
Solution : (a) Applying Bernoulli’s equation between liquid surface
B
and point ‘C’.
h2
1 2 1
Pa + v 1 = P a – gh 3 + v 22
2 2 v1 h1
h
through continuity equation A
h3
av 2
Av 1 = av 2 , v 1 =
A
C v2
2
1 a 2 1
  2 v 2 = –gh 3 + v 22
2 A 2

2gh3 2gh 3
v 22 = , v2 =
a2 a2
1 2 1
A A2
(b) Pressure at A just outside the tube , P A = P atm + gh 1
For pressure at B,
1
P A + 0 + 0 = P B + gh 2 + v B2
2

 
 
1 2gh 3
P B = P A – gh 2 –   
2  a2 
 1  
 A2 
Pressure at C, P C = P atm

KVPY_ SA STREAM # 290



(VII) Venturimeter.
It is a gauge put on a flow pipe to measure the flow of speed of a liquid (Fig). Let the liquid of
density  be flowing through a pipe of area of cross section A 1. Let A 2 be the area of cross section
at the throat and a manometer is attached as shown in the figure. Let v 1 and P 1 be the velocity of
the flow and pressure at point A, v 2 and P 2 be the corresponding quantities at point B.

Using Bernoulli’s theorem :


P1 1 2 P2 1 2
+ gh 1 + v = + gh 2 + v , we get
 2 1  2 2

P1 1 2 P2 1 2
+ gh + v1 = + gh + v (Since h 1 = h 2 = h)
 2  2 2

1
or (P 1 – P 2) = ( v 22 – v 12) ....(1)
2
According to continuity equation, A 1 v 1 = A 2v 2

A 
or v 2   1  v
 A2  1
Substituting the value of v 2 in equation (1) we have

 2   A  2 
1  A1  v 2  v 2  1  1   1
(P 1 – P 2) =    1 1
 2  v1
2
 A 2 
2  A 2   


Since A 1 > A 2, theref ore, P 1 > P 2

2(P1  P2 ) 2A 22 (P1  P2 )
or v12 = =
 A  2   ( A12  A 22 )
  1   1
 A 2  
 
where (P 1 – P 2) = m gh and h is the difference in heights of the liquid levels in the two tubes.

2m gh
 A  2 
v1 =   1   1
 A 2  
 

The flow rate (R) i.e., the volume of the liquid flowing per second is given by R = v 1 A 1.

(viii) During wind storm,


The velocity of air just above the roof is large so according to Bernoulli’s theorem, the pressure
just above the roof is less than pressure below the roof. Due to this pressure difference an upward
force acts on the roof which is blown of without damaging other parts of the house.

(ix) W hen a fast moving train cross a person standing near a railway track, the person has a tendency
to fall towards the train. This is because a fast moving train produces large velocity in air between
person and the train and hence pressure decreases according to Bernoulli’s theorem. Thus the
excess pressure on the other side pushes the person towards the train.

~~~~~~

KVPY_ SA STREAM # 291


1. A cubical block of copper of side 10 cm is floating in a vessel containing mercury. Water is poured into
the vessel so that the copper block just gets submerged. The height of water column is
(Hg = 13.6 g/cc , Cu= 7.3 g/cc, water =1 gm/cc)
(A) 1.25 cm (B) 2.5 cm (C) 5 cm (D) 7.5 cm

2. A long cyclindrical drum is filled with water. Two small holes are made on
the side of the drum as shown in the fig. Find the depth of the liquid in the
drum if the ranges of water from the holes are equal.
(A) 0.2 m (B) 0.4 m
(C) 0.3 m (D) 0.5 m

3. A block of silver of mass 4 kg hanging from a string is immersed in a liquid of relative density 0.72. If relative
density of silver is 10, then tension in the string will be:[ take g = 10 m/s2 ]
(A) 37.12 N (B) 42 N (C) 73 N (D) 21 N

4. Water (density ) is flowing through the uniform tube of cross-sectional


area A with a constant speed v as shown in the figure. The magnitude
of force exerted by the water on the curved corner of the tube is
(neglect viscous forces)
(A) 3 Av 2 (B) 2Av 2

Av 2
(C) 2Av 2 (D)
2
5. The centre of buoyancy of a floating object is
(A) at the centre of gravity of the object.
(B) at the centre of gravity of the submerged part of the object.
(C) at the centre of gravity of the remaining part outside the fluid of the object.
(D) at the centre of gravity of the fluid displaced by the submerged part of the object.

6. You are studying for an exam on the eight floor of your luxurious apartment building. You look out from the
window and notice that one of your neighbours is giving a party on the ground-floor terrace and has placed a
huge punch bowl full of an interesting looking beverage (specific gravity 1) directly below your window. You
quickly string together 80 drinking straws to form a giant straw that can reach the punch bowl 80 feet below.
You dip the straw into the punch and begin to suck. When you use a single drinking straw to drink something,
it takes you 0.1 seconds to raise the liquid to your lips. But when you use this giant drinking straw,
(A) you find that you can’t raise the liquid to your lips no matter how hard you try.
(B) it takes you 8 seconds (80 times 0.1 second) to raise the liquid to your lips.
(C) it takes you 800 seconds (80 divided by 0.1 scond) to raise the liquid to your lips.
(D) it takes you 640 seconds (80 times 80 times 0.1 second) to raise the liquid to your lips.

7. An open tank 10m long and 2m deep is filled up to 1.5 m height of oil of specific gravity 0.82. The tank
is uniformly accelerated along its length from rest to a speed of 20 m/sec horizontally. The shortest
time in which the speed may be attained without spilling any oil is : [g = 10 m/sec 2]
(A) 20 sec. (B) 18 sec. (C) 10 sec. (D) 5 sec.

8. Following are some statements about buoyant force: (Liquid is of uniform density)
(i) Buoyant force depends upon orientation of the concerned body inside the liquid.
(ii) Buoyant force depends upon the density of the body immersed.
(iii) Buoyant force depends on the fact whether the system is on moon or on the earth.
(iv) Buoyant force depends upon the depth at which the body (fully immersed in the liquid) is placed inside
the liquid.
Of these statements :
(A) Only (i), (ii) and (iv) are correct. (B) Only (ii) is correct.
(C) Only (iii) and (iv) are correct. (D) (i), (ii) and (iv) are incorrect.

KVPY_ SA STREAM # 292


9. A water tank stands on the roof of a building as shown. Then the value of h for which the distance 'x' covered
by the water is maximum
h
1m

3m

(A) 0.5 m (B) 0.67 m (C) 1 m (D) none of these


10. A boy carries a fish in one hand and a bucket (not full) of water in the other hand. If he places the
fish in the bucket , the weight now carried by him (assume that water does not spill):
(A) is less than before (B) is more than before
(C) is the same as before (D) depends upon his speed
11. An open water tanker moving on a horizontal straight road has a cubi-
cal block of cork floating over its surface. If the tanker has an
accelecration of a as shown, the acceleration of the cork w.r.t. con- a
tainer is (ignore viscosity) -
a2
(A) Zero (B)
g

a
(C) y g2  a 2 (D) a

12. In the figure shown, a light container is kept on a horizontal rough


Sh
surface of coefficient of friction  = . A very small hole of area S is
V
made at depth 'h'. Water of volume 'V' is filled in the container. The
friction is not sufficient to keep the container at rest. The acceleration
of the container initially is
V Sh
(A) g (B) g (C) zero (D) g
Sh V
13. An empty glass jar is submerged in tank of water with open
mouth of the jar downwards, so that air inside the jar is trapped
and cannot get out. As the jar is pushed down slowly, the
magnitude of net buoyant force on the system of volume of gas
trapped in the jar and the jar :
(A) increases (B) decreases
(C) remains same (D) Information is insufficient to draw inference.
14. One end of light inelastic string is tied to a helium filled balloon and
its other end is tied to bottom of a water filled container at point O.
The container lies on a fixed horizontal surface and is pulled
horizontally towards right with constant horizontal acceleration of
magnitude a. Assuming no relative motion of balloon and water with
respect to container, the string will be inclined with vertical line
passing through O by an angle. (g is acceleration due to gravity)
a
(A)  = tan–1 and string will be on right of vertical line passing through O.
g
g
(B)  = tan–1 and string will be on right of vertical line passing through O.
a
a
(C)  = tan–1 and string will be on left of vertical line passing through O.
g
g
(D)  = tan–1 and string will be on left of vertical line passing through O.
a

KVPY_ SA STREAM # 293


15. Figure shows a closed container completely filled with an ideal liquid
of density . In the liquid there is a spherical body of volume V and
density  attached to a string whose other end is attached to the roof
of the container. The container is accelerating with an acceleration 'a'
towards right. The magnitude of force exerted by the liquid on the
spherical body when it is in equilibrium with respect to the liquid, will
be:
(A) V a 2  g2 + Va (B) Vg+ Va (C) V(g+a) (D) V a 2  g2

16. A solid sphere of iron at 2°C is lying at the bottom of a bucket full of water at 2°C. If the temperature of
the water is increased to 3°C, the buoyant force on the sphere due to water will MADE RKV Sir 2008
(A) Increase (B) Be unchanged (C) Decrease
(D) Increase or decrease depends upon the numerical values of coefficient of expansion of water and iron.
17. A fixed container is filled with a liquid of density up to a height
4m. A horizontal slit of small width but of area = 0.5 m2 is made at
a height of 2m from bottom. The speed of top surface of the water
level is (area of top surface of container is 4 m2 and g = 10 m/s2) :

(A) 20 m/sec (B) 40 m/sec (C) 80 m/sec (D) None of these


63 63 63

18. Bantu slips into a large lake and he doesn't know swimming but he is a great fan of Superman, he
shouted for help remembering Superman. As usual Superman arrives on the top of a cliff and due to some
reason Superman lost his flying power immediately after arrival on cliff. Due to shortage of time somehow
Superman manages a strong and long straw and decided to drink whole water of lake to save Bantu.(Data:
Atmospheric pressure =1.2 × 105 Pa, g=10m/s2, density of water= 1000kg/m 3) Assume Superman has
infinite power and ability to drink whole water which of the following statements is/are true.
(1) Superman cannot save Bantu by this way.
(2) Superman can drink some water but not whole water.
(3) Superman will save Bantu by drinking whole water.

(A) Only (1) & (2) (B) Only (3)


(C) Only (1) (D) All (1), (2) & (3) are wrong

19. A water container is kept on a weighing balance. Water from a tap is falling vertically into the container
with a volume rate of Q. The velocity of water when it hits the water surface is v and comes to rest after
hitting. At a particular instant of time, the mass of container and water is m. The force registered by the
weighing balance at this instant of time is : [assume no splash]
Qv Qv 2
(A) mg + Qv (B) mg + (C) mg + (D) mg
2 2

20. Ethanol of density  = 700 kg/m 3 flows smoothly through a horizontal pipe that tappers in cross-
A1
sectional area from A 1 = 1.2 × 10–3 m 2 to A2 = . The pressure difference between the wide and the
2
narrow sections of pipe is 4200 Pa. What is the volume flow rate of ethanol in multiples of 10 –4 m 3/s.
(A) 12 (B) 16 (C) 24 (D) None of these

KVPY_ SA STREAM # 294


21. The height of liquid in the tube below the air and above the liquid level in the container is h. The temperature
of the air is now slightly increased. After the equilirbium state is achieved, the height h will (assume no
change in the temperature of the liquid in the container.)

(A) remain same. (B) decrease


(C) increase (D) decrease or increase, depending on the density of liquid above air.

22. A vessel has the shape shown in figure. Water, which has density 103 kg/m3, is filled in the vessel. The
pressure at the point A, ignoring the atmospheric pressure, is (g = 10 m/s2)

(A) 1.2 × 104 N/m2 (B) 1.0 × 104 N/m2 (C) 2.2 × 104 N/m2 (D) 2.4 × 104 N/m2

23. 1m height of water level is maintained in a container which is placed on a platform of height 2m. A hole is
punched at a height h from the ground. For which value of h given in options, the water falls at maximum
distance from the base ?

(A) 1.5 m (B) 2.01 m (C) 2.25 m (D) 2.50 m

24. A large open tank is filled with water upto a height H. A small hole is made at the base of the tank. It takes
H
T1 time to decrease the height of water to (n > 1) and it takes T2 time to take out the remaining water.
n
If T1 = T2, then the value of n is :
(A) 2 (B) 3 (C) 4 (D) 2 2

KVPY_ SA STREAM # 295


25. Figure shows five identical open–top container filled upto the brim with water. In figure(b), toy duck is
floating, in figure(c) a heavy toy duck sinks and touches the bottom, in figure (d) a heavy toy duck is
suspended from a string and in figure (e) a light toy duck is tied with a massless string. If all the five
containers are put on weighing machine, the correct order of their reading will be :

(A) (a) = (b) = (c) = (d) = (e) (B) (e) < (a) = (b) = (d) < (c)
(C) (e) < (d) < (a) = (b) < (c) (D) (a) = (b) = (d) = (e) < (c)

26. A non–viscous ideal fluid is flowing vertically downwards in a


pipe. The area of cross-section at section 'A' is double that at
section 'B'. Vertical distance between section A and section B
is h and the height of water column in tube (2) is more than
h
that in tube (1) by distance. Velocity of the fluid at section
2
A is :
gh
(A) gh (B)
2

gh gh
(C) (D)
3 5

27. Two immiscible liquids are poured in a U–tube having densities 1 = 1.0 × 103 kg/m3 and 2 = 3.0 × 103 kg/
h1
m3. Find the ratio of heights (of the liquids above their interface) h .
2

(A) 1.5 (B) 3 (C) 1/3 (D) 6

28. The given container is in contact with vertical wall. The


container has liquid of density . There is a small hole of
cross–section area of 'A'. At the given moment ground
can exert maximum friction force Agh on the container.
At given instant the normal force exerted by the vertical
wall on container is xAgh, find the value of 'x'.

(A) 7 (B) 0
(C) 1 (D) 5
KVPY_ SA STREAM # 296
29. A simple open U-tube contains mercury. Now water is poured slowly upto height 27.2
cm in the left arm. How high (in cm) does the mercury rise in the right arm from its
initial level in equilibrium state? (Take density of mercury = 13600 kg/m3 and that of
water = 1000kg/m3 and g = 10 m/s2)

(A) 2 cm (B) 4 cm (C) 1 cm (D) 1.5 cm

30. A liquid is kept in a cylindrical vessel which is rotated about its axis. The liquid rises at the sides. If the
radius of the vessel is 0.05 m and the speed of rotation is 2 rev/s, The difference in the height of the
liquid at the centre of the vessel and its sides will be (2 = 10) :
(A) 3 cm (B) 2 cm (C) 3/2 cm (D) 2/3 cm

KVPY PROBLEMS (PREVIOUS YEARS)


1. There is a steady water flow in a horizontal tube in which one part has cross sectional area A1 and the other
part has cross sectional area A2. Assume that water is incompressible. If A1/A2 = 16, the ratio of the speed
u1 in part 1 and the speed u2 in part 2, i.e. u1/u2 is : [KVPY_2008]

1 1
(A) (B) 4 (C) (D) 1
16 4

2. A block of wood is floating on oil with half of its volume submerged. If the density of oil 840 kg m–3, the relative
density of wood (relative to water) is : [KVPY_2008]
(A) 0.84 (B) 0.42 (C) 0.21 (D) 1.00

3. A rubber pipe with a diameter of 10 cm is connected to a nozzle 2 cm in diameter. Water flowing through the.
pipe at a speed of 0.6 ms–1, comes out like a jet through the nozzle. The backward force of the nozzle is
about : [KVPY_2008]
(A) 7.7 N (B) 67.9 N (C) zero (D) 2.8 N

4. Two immiscible liquids, A and B are kept in a U-tube. If the density of liquid A is smaller than the density of
liquid B, then the equilibrium situation is. [KVPY_2009]

A A
A

(A) (B) (C) (D) None of these

5. An object with uniform desity  is attached to a spring that is known to stretch linearly with applied force as
shown below [KVPY_2011]

KVPY_ SA STREAM # 297


When the spring object system is immersed in a liquid of density 1 as shown in the figure, the spring
stretches by an amount x1 ( > 1). When the experiment is repeated in a liquid of density 2 < 1 . the spring
strethces by an amound x2. Neglecting any buoyant force on the spring, the density of the object is :
1x1  2 x 2 1x 2  2 x1 1x 2  2 x1 1x1  2 x 2
(A)   x1  x 2 (B)   x 2  x1 (C)   x1  x 2 (D)   x1  x 2

6.* A solid cube and a solid sphere both made of same material are completely submerged in water but to
different depths. The sphere and the cube have same surface area. The buoyant force is : [KVPY_2012]

(A) greater for the cube than the sphere (B) greater for the sphere than the cube
(C) same for the sphere and the cube (D) greater for the object that is submerged deeper
7. A tall tank filled with water has an irregular shape as shown. The wall CD makes an angle of 45° with the
horizontal; the wall AB is normal to the base BC. The lengths AB and CD are much smaller than the height
h of water (figure not to scale). [KVPY_2013]

Let P1, P2 and P3 be the pressures exerted by the water on the wall AB, base BC and the wall CD respectively.
Density of water is  and g is acceleration due to gravity. Then, approximately

1 1
(A) P1 = P2 = P3 (B) P1 = 0, P3 = P2 (C) P = P = P2 (D) P1 = P3 = 0, P2 = hg
2 1 3
2

8. A machine is blowing spherical soap bubbles of different raddi filled with helium gas. It is found that if the
bubbles have a radius smaller than 1 cm, then they sink to the floor in still air. Larger bubbles float in the air.
Assume that the thickness of the soap film in all bubbles is uniform and equal. Assume that the density of
soap solution is same as that of water (= 1000 kgm-3). The density of helium inside the bubbles and air are
0.18 kg m-3 and 1.23 kg m-3, respectively. Then the thickness of the soap film of the bubbles is (note 1  m
= 10-6m) [KVPY_2014_SA]
(A) 0.50  m (B) 1.50  m (C) 7.00  m (D) 3.50  m

9. Water containing air bubbles flows without turbulence through a horizontal pipe which has a region of narrow
cross- section. In this region the bubbles : [KVPY_2014_SB]
(A) move with greater speed and are smaller than in the rest of the pipe
(B) move with greater speed and are larger in size than in the rest of the pipe
(C) move with lesser speed and are smaller than in the rest of the pipe.
(D) move with lesser speed and are of the same size as in the rest of the pipe
10. Physical processes are sometimes described visually by lines. Only the following can cross :
[KVPY_2016_SB] [1 Marks]
(A) Streamlines in fluid flow (B) Lines of forces in electrostatics
(C) Rays in geometrical optics (D) Lines of force in magnetism

KVPY_ SA STREAM # 298


11. A nurse measures the blood pressure of a seated patient to be 190 mm of Hg.[KVPY_2016_SB] [1 Marks]
(A) the blood pressure at the patient’s feet is less than 190 mm of Hg.
(B) the actual pressure is about 0.25 times the atmospheric pressure.
(C) the blood pressure at the patient’s neck is more than 190 mm of Hg.
(D) the actual pressure is about 1.25 times the atmospheric pressure.
12. A spherical marble of radius 1 cm is stuck in a circular hole of radius slightly smaller than its own radius (for
calculation purpose, both can be taken same) at the bottom of a bucket of height 40 cm and filled with water
up to 10 cm. If the mass of the marble is 20 g, the net force on the marble due to water is close to
[KVPY_2017_SB] [2 Marks]

(A) 0.02 N upwards (B) 0.02 N downwards (C) 0.04 N upwards (D) 0.04 N downwards

13. A very large block of ice of the size of a volleyball court and of uniform thickness of 8m is floating on water.
A person standing near its edge wishes to fetch a bucketful of water using a rope. The smallest length of rope
required for this is about [KVPY-SA_2018 1 Mark]
(A) 3.6m (B) 1.8m (C) 0.9m (D) 0.4m

EXERCISE - 1

1. (C) 2. (C) 3. (A) 4. (A) 5. (D) 6. (A) 7. (A)

8. (D) 9. (C) 10. (C) 11. (A) 12. (D) 13. (B) 14. (A)

15. (D) 16. (A) 17. (B) 18. (C) 19. (A) 20. (C) 21. (A)

22. (C) 23. (B) 24. (C) 25. (B) 26. (C) 27. (B) 28. (D)

29. (C) 30. (B)

EXERCISE - 2

1. (A) 2. (B) 3. (B) 4. (C) 5. (B) 6. 7. (A)


8. (D) 9. (B) 10. (C) 11. (D) 12. (D) 13. (C)

KVPY_ SA STREAM # 299


1. Let h = height to of water column
then wgh + Hg g(10–h) = Cu g(10)
 h + 13.6 (10 – h) = 73 63 = 12.6 h  h = 5 cm

2. (Range)1 = (Range)2
2  0.1 2  0.2
2g( – 0.1)  2g( – 0.2)
g g
 = 0.3

3. Let S, L be the density of silver and liquid. Also m and V be the mass and volume of silver block.
 Tension in string = mg – bouyant force
T = SVg – L Vg = (S – L) Vg
m
Also V = s

  S  L  3
 T =    mg = (10  0.72)  10 × 4 × 10

 S  10  10 3
= 37.12 N.

3
4. | P x | = mv sin 60° = mv
2

mv 3
| P y | = + mv = mv
2 2

9 3
 | Pnet |  Px2  Py2 =    mv
4 4

| P net |  3 mv

 Since, dm  A ( v dt )  
 dm   
 | F net |  3  . v = 3 A v 2
Ans.   dm  A  v 
 dt   dt 

5. Figure shows the points described in the question.

C.G. of submerged part of the object will be at the centre of buoyancy if the object is uniform,
Otherwise shifts from this point. From figure (D) is correct.

6. The liquid will rise up to 80 fect height only if the pressure at that point is less
than. p0 by an amount w.g.h. ie; Pressure at point A should be PA = P0 – gh
< 0 (As gh = 103 × 10 × 80 > 105 (P0)) and it is not possible for us to make
the pressure negative.

KVPY_ SA STREAM # 300


v
7. v = u + a xt , a x =
t
ax v 0.5
tan = g = tg = (in triangle ABC)
5

10  20
 t= = 20 sec.
10

8. F b = vligg
'g' is different on moon and on the earth.
Hence only (iii) is a correct statement.

9. Vefflux = 2gh

( 4  h)2
time of fall t = g

x = Vefflux t = 2 h( 4  h)
the roots of x are (0,4) and the maximum of x is at h = 2.
The permitted value of h is 0 to 1 clearly h = 1 will give the
maximum value of x is this interval.
Aliter : If the column of water itself were from ground upto a height of 4m, h = 2m would give the maximum
range x. Farther the hole is from this midpoint, lower the range. Here the nearest point possible to this
midpoint is the base of the container. Hence h = 1m.

10. In either case he carries same mass and hence same weight. (Buoyant force is internal force of
bucket and fish system)
 (C) is true N

arel
11. marel = mgsin – macos
but for water surface tan  = a/g ma
 arel = 0
mgsin-macos
mgcosmasin
mg

12. Let the density of water be , then the force by escaping liquid on
container =  S ( 2gh ) 2

2 Sgh   Vg  2Sh 


 acceleration of container a = =    g
V  V 
Sh Sh
Now  = a= g
V V
13. As the jar is pushed down, due to increase in hydrostatic pressure volume of gas trapped decreases.
Hence net bouyant force decreases.

14. Let the density of gas and water by g and  .


The volume enclosed by balloon is V.
Then the FBD of balloon is as shown

Fx a
 In equilibrium tan  = 
Fy g

KVPY_ SA STREAM # 301


15. From the frame of reference of liquid, effective gravity (resultant of

weight and pseudo force per unit mass) = a 2  g2

 Effective force due to liquid = V a 2  g2


Aliter :
The force exerted on spherical body by surrounding liquid
is equal to sum of contact forces on displaced liquid by
the surrounding liquid as shown in .

Hence the force exerted on spherical body


by surrounding liquid = V a 2  g2 .

16. As the temperature of water is increased from 2°C to 3°C the density of water increases (remember
anamolous behaviour of water), also the volume of sphere increases. Therefore bouyant force on sphere
due to water shall increase.

17. Patm + .g.2 + 1/2 . V12 = Patm + 1/2 . V22


4g + V12 = V22
A1 V1 = A2V2
4g + V12 = 64 V12
63V12 = 40

40
V1 = m/sec
63

18. Superman cannot drink water at all because at cliff height 12.5 m, pressure due to water in straw at the
lake is P = hg = 12.5 × 1000 × 10 = 1.25 × 105 Pa. Which is more than atmospheric pressure.

19. F = wt. of water + force by impact of water


dP
F = mg +
dt
F = mg + Q(v – 0)
F = mg + Qv

20. From equation of continuity


A1V1 = A2V2  V2 = 2V1 .... (1)
From Bernoulli's theorem
1 2 1 2
P1 +  V1 = P2 +  V2
2 2
1 2 2
  (V2  V1 ) = P1 – P2
2
1
 × 700 × 3V12 = 4200
2
V1 = 2 m/s
Volume flow rate A1V1 = 1.2 × 10–3 × 2 m 3/s = 24 × 10–4 m 3/s

21. (Easy) When the temperature of air is increased, the pressure due to air will still remain constant. Hence h
will remain constant.

22. (Moderate) Pressure at A will be due to water height (1.0 + 1.0 + 0.2) m
 PA = gh = 103 × 10 × 2.2 N/m2
1 2
23. (Moderate) The water will fall maximum distance if the hole is made at nearest to = 1.5 m. The nearest
2
point is at the bottom of the container.

KVPY_ SA STREAM # 302


dy
24. A = a 2gy
dt

2A  H 
H
a 2g  n  = T1

2A  H 
 0 = T
a 2g  n 

2

T1 = T2
n = 4.

25. (b) weight of the displaced water = weight of the duck


so, the weight of the system (b) will be equal to that of system (a)
(c) as the density of the duck is more that of water, the weight of system (c) is more than that of
system (a).
(d) The buoyancy force transmitted from the duck to the water is equal to the weight of the displaced water.
So, weight of system (d) = weight of system (a)
(e) As the density of the duck is less than that of water, so weight of system (e) is less that of system (a).

26. Applying Bernoulli's equation between section A and B


1 1
PA + v 2 + ghA = PB + v B2 + ghB
2 A 2
1 2 1
PA + v + gh = PB + (2v)2 + 0
2 2
3 2 h
PB – PA = gh – v and PB – PA = g
2 2

h 3 gh
Pg = gh – v 2  v= .
2 2 3

27. p0 + 1gh1 = p0 + 2gh2


h1 2
 h2 = 1 = 3.

28. Thrust force F1 = Av 2 = A[2g(3h)]


FBD of container,

N + F2 = F1
N = F1 – F2 = 6Agh – Agh
= 5Agh.

29. Pressure at A that is at interface of water and mercury and B is same as


both are at same horizontal level in same liquid. Thus

P0 + 27.2 × 10–2 × 1000 × 10 = P0 + x × 13600 × 10


x = 2cm.
x
Height to which mercury has risen = = 1 cm.
2

KVPY_ SA STREAM # 303


K I N ET I C T H EORY OF GASES AN D T H ERM ODY N AM I CS

~~~~ ~~~~
KI NETI C THEORY OF GASES :
Kinetic theory of gases is based on the following basic assumptions.
(a) A gas consists of very large number of molecules. These molecules are identical, perfectly
elastic and hard spheres. They are so small that the volume of molecules is negligible as
compared with the volume of the gas.
(b) Molecules do not have any preferred direction of motion, motion is completely random.
(c) These molecules travel in straight lines and in free motion most of the time. The time of the
collision between any two molecules is very small.
(d) The collision between molecules and the wall of the container is perfectly elastic. It means
kinetic energy is conserved in each collision.
(e) The path travelled by a molecule between two collisions is called free path and the mean of this
distance travelled by a molecule is called mean free path.
(f) The motion of molecules is governed by Newton's law of motion
(g) The effect of gravity on the motion of molecules is negligible.

EXPRESSI ON FOR THE PRESSURE OF A GAS:


Let us suppose that a gas is enclosed in a cubical box having length  . Let there are ' N ' identical
molecules, each having mass ' m '. Since the molecules are of same mass and perfectly elastic, so
their mutual collisions result in the interchange of velocities only. Only collisions with the walls of the
container contribute to the pressure by the gas molecules. Let us focus on a molecule having velocity
v 1 and components of velocity v x1 , v y1 , v z1 along x, y and z-axis as shown in figure.

v 12 = v 2 x1  v 2 y1  v 2 z1
The change in momentum of the molecule after one collision with wall BCGF
= m v x1  ( m v x1 ) = 2 m v x1 .

dis tan ce 2
The time taken between the successive impacts on the face BCGF = = v
velocity x1

change in momentum 2mv x1 mv 2 x1


Time rate of change of momentum due to collision = = 2 / v  
time taken x1

Hence the net force on the wall BCGF due to the impact of n molecules of the gas is :

Fx =
mv 2x

1

mv 2x

2

mv 2x 3

 ............ 
mv 2x

n

m 2


v x  v 2x  v 2x  ..............  v 2x
1 2 3 n
 = mN  v 2
x

where  v 2x  = mean square velocity in x-direction. Since molecules do not favour any particular

direction therefore  v 2x  =  v 2y  =  v 2z  . But < v 2 > =  v 2x  +  v 2y  +  v 2z 

 v2 
  v 2x  = . Pressure is equal to force divided by area.
3

KVPY_ SA STREAM # 304


Fx M M
P=   v2    v 2  . Pressure is independent of x, y, z directions.
2 3 3 3V

Where  3 = volume of the container = V


M = total mass of the gas, < v 2 > = mean square speed of molecules
1 2
 P=  v 
3
1 2 3 3
As PV = n RT , then total translational K.E. of gas = M v  = PV = n RT
2 2 2

3
Translational kinetic energy of 1 molecule = kT (it is independent of nature of gas)
2

3P 3P 3RT 3kT
 v2  = or vrms =  
 ρ Mmole m
Where v rms is root mean square speed of the gas.
1 2 1 2 3
Pressure exerted by the gas is P =  <v 2 > =   <v 2 > or P = E,E= P
3 3 2 3 2
Thus total translational kinetic energy per unit volume (it is called energy density) of the gas is numerically
3
equal to times the pressure exerted by the gas.
2

I M P ORTANT P OI NTS :
1
(a) v rms  T and v rms 
M m o le
(b) At absolute zero, the motion of all molecules of the gas stops.
(c) At higher temperature and low pressure or at higher temperature and low density, a real gas
behaves as an ideal gas.

M AXW ELL'S D I STRI BUTI ON LAW :


dN( v )
Distribution Curve – A plot of (number of molecules per unit speed interval) against v is known
dv
as Maxwell's distribution curv e. The total area under the curv e is giv en by the integral
 
dN(v )

0
dv 
dv  dN(v )  N .
0

dN( v )
[Note:- The actual formula of is not in JEE syllabus.]
dv
Figure shows the distribution curves for two different temperatures. At any temperature the number of
molecules in a given speed interval dv is given by the area under the curve in that interval (shown
shaded). This number increases, as the speed increases, upto a maximum and then decreases
asymptotically towards zero. Thus, maximum number of the molecules have speed lying within a small
range centered about the speed corresponding the peak (A) of the curve. This speed is called the 'most
probable speed' v p or v mp.

KVPY_ SA STREAM # 305


The distribution curve is asymmetrical about its peak (the most probable speed v p ) because the lowest
possible speed is zero, whereas there is no limit to the upper speed a molecule can attain. Therefore,
the average speed v is slightly larger than the most probable speed v p . The root-mean-square speed,
v rms, is still larger (v rms > v > v p ).
Average (or Mean) Speed :
8 kT
v =  m
= 1.59 kT / m . (derivation is not in the course)

RMS Speed :

3kT = 1.73 kT
v rms = v 2  = .
m m

Most Probable Speed :


The most probable speed v p or v mp is the speed possessed by the maximum number of molecules, and
corresponds to the maximum (peak) of the distribution curve. Mathematically, it is obtained by the
condition.
dN( v )
= 0 [by substitution of formula of dN(v) (which is not in the course)]
dv
Hence the most probable speed is
2kT
vp = = 1.41 kT / m .
m

From the above expression, we can see that


v rms > v > v p.

The laws which can be deduced with the help of kinetic theory of gases are below.
(a) Boyle's law
(b) Charle's law
(c) Avogadro's hypothesis
(d) Graham's law of diffusion of gases
(e) Regnault's or Gay Lussac's law
(f) Dalton's Law of Partial Pressure
(g) Ideal Gas Equation or Equation of state

D EGREE OF FREEDOM :
Total number of independent co-ordinates which must be known to completely specify the position and
configuration of dynamical system is known as "degree of freedom f". Maximum possible translational
1 2 1 2 1 2
degrees of freedom are three i.e.  mv x  mv y  mv z 
2 2 2 

1 2 1 2 1 2
Maximum possible rotational degrees of freedom are three i.e.   x  x   y  y   z  z 
2 2 2 
Vibrational degrees of freedom are two i.e. (Kinetic energy of vibration and Potential energy of vibration)

Mono atomic : (all inert gases Ex. He , Ar etc.) f=3 (translational)


Diatomic : (gases like H2 , N2 , O 2 etc.) f=5 (3 translational + 2 rotational)
If temp < 70 K for diatomic molecules, then f=3
If temp in between 250 K to 5000 K , then f =5
If temp > 5000 K f = 7 [ 3 translational.+ 2 rotational + 2 vibrational ]

KVPY_ SA STREAM # 306


M AXW ELL'S LAW OF EQUI PARTI TION OF ENERGY :
1
Energy associated with each degree of freedom = kT. If degree of freedom of a molecule is f , then
2
1
total kinetic energy of that molecule  fkT
2

IN TERNAL ENERGY :
The internal energy of a system is the sum of kinetic and potential energies of the molecules of the
system. It is denoted by U. Internal energy (U) of the system is the function of its absolute temperature
(T) and its volume (V). i.e. U = f (T, V)
In case of an ideal gas, intermolecular force is zero. Hence its potential energy is also zero. In this
case, the internal energy is only due to kinetic energy, which depends on the absolute temperature of
f
the gas. i.e. U = f (T). For an ideal gas internal energy U = nRT..
2

Example 1. A light container having a diatomic gas enclosed within is moving with velocity V. Mass of the gas is
M and number of moles is n.
(i) What is the kinetic energy of gas w.r.t. centre of mass of the system?
(ii) What is K.E. of gas w.r.t. ground?

5
Solution : (i) K.E. = nRT
2

(ii) Kinetic energy of gas w.r.t. ground = Kinetic energy of gas w.r.t. centre of mass + Kinetic
energy of centre of mass w.r.t. ground.

1 5
K.E. = MV2 + nRT
2 2

Example 2. Two non conducting containers having volume V1 and V2 contain


mono atomic and diatomic gases respectively. They are connected
as shown in figure. Pressure and temperature in the two contain-
ers are P1 , T1 and P2 , T2 respectively. Initially stop cock is closed,
if the stop cock is opened find the final pressure and temperature.

P1V1 P2 V2
Solution : n1 = RT n2 =
RT2
1

n = n1 + n2 (number of moles are conserved)


Finally pressure in both parts & temperature of the both the gases will become equal.

P( V1  V2 ) P1V1 P2 V2
= +
RT RT 1 RT2

From energy conservation

3 5 3 5
n1RT1 + n2RT2 = n1RT + n RT
2 2 2 2 2

(3P1V1  5P2 V2 ) T1T2  3P1V1  5P2 V2   P1V1T2  P2 V2 T1 


 T = 3P V T  5P V T  P=   
 3P V T  5P V T   V1  V2 
1 1 2 2 2 1  1 1 2 2 2 1  

KVPY_ SA STREAM # 307



I N DI CATOR D IAGRAM :
A graph representing the variation of pressure or variation of temperature or variation of volume with
each other is called indicator diagram.

(A) Every point of Indicator diagram represents a unique state (P, V, T) of gases.
(B) Every curve on Indicator diagram represents a unique process.

TH ER M ODYN AM I CS
Thermodynamics is mainly the study of exchange of heat energy between bodies and conversion of the
same into mechanical energy and vice-versa.

THERM ODYN AM I C SYSTEM


Collection of an extremely large number of atoms or molecules confined within certain boundaries such
that it has a certain value of pressure (P), volume (V) and temperature (T) is called a thermodynamic
system. Anything outside the thermodynamic system to which energy or matter is exchanged is called
its surroundings. Taking into consideration the interaction between a system and its surroundings
thermodynamic system is divided into three classes :
(a) Open system : A system is said to be an open system if it can exchange both energy and
matter with its surroundings.
(b) Closed system : A system is said to be closed system if it can exchange only energy (not
matter with its surroundings).
(c) Isolated system : A system is said to be isolated if it can neither exchange energy nor matter
with its surroundings.

ZEROTH LAW OF THERM ODYN AM ICS :


If two systems (B and C) are separately in thermal equilibrium with a third one (A), then they themselves
are in thermal equilibrium with each other.

EQUATI ON OF STATE ( FOR AN IDEAL GASES) :


The relation between the thermodynamic variables (P, V, T) of the system is called equation of state.
The equation of state for an ideal gas of n moles is given by
PV = nRT,

W ORK DONE BY A GAS :


Let P and V be the pressure and volume of the gas. If A be the area of the piston, then force exerted by
gas on the piston is, F = P  A.
Let the piston move through a small distance dx during the expansion of the gas. Work done for a small
displacement dx is dW = F dx = PA dx

KVPY_ SA STREAM # 308


Since A dx = dV, increase in volume of the gas is dV  dW = P dV

or W=  dW  P dV
Area enclosed under P-V curve gives work done during process.

D I FFEREN T TYP ES OF P ROCESSES :


(a) Isothermal Process :
T = constant [Boyle's law applicable] PV = constant

There is exchange of heat between system and surroundings. System should be compressed
or expanded very slowly so that there is sufficient time for exchange of heat to keep the
temperature constant.
Slope of PV curve in isothermal process:
dP P
PV = constant = C  
dV V
Work done in isothermal process:

Vf If Vf  Vi then W is positive


W = nRT  n  
Vi If Vf  Vi then W is negative 

 Vf 
W = 2.303 n R T log10 V 
 i 

Internal energy in isothermal process :

U = f (T) U = 0

(b) Iso- choric Process (Isometric Process) :


V = constant
 change in volume is zero
P
 is constant
T
P
 const. (Gay lussac’s law)
T
Work done in isochoric process :
Since change in volume is zero therefore dW = P dV = 0

KVPY_ SA STREAM # 309


Indicator diagram of isochoric process :

f
Change in internal energy in isochoric process : U = n R T
2

f
Heat given in isochoric process : Q = U = n R T
2

(c) Isobaric Process : Pressure remains constant in isobaric process


V
 P = constant  = constant
T
Indicator diagram of isobaric process :

Work done in isobaric process :


W = P V = P (Vfinal – Vinitial ) = nR (T final – T initial )

Change in internal energy in isobaric process : U = n CV T


Heat given in isobaric process :
Q = U + W
f f
Q = n R T + P [Vf  Vi] = n R T + nR T
2 2
Above expression gives an idea that to increase temperature by T in isobaric process heat
required is more than in isochoric process.

(d) Cyclic Process : In the cyclic process initial and final states are same therefore initial state =
final state
Work done = Area enclosed under P-V diagram.
Change in internal Energy U = 0
Q = U + W
 Q = W
If the process on P-V curve is clockwise , then net work done is (+ve) and vice-versa.
The graphs shown below explains when work is positive and when it is negative

(+) work
P

KVPY_ SA STREAM # 310


Example 3. The cylinder shown in the figure has conducting walls and temperature of the surrounding is T, the
piston is initially in equilibrium, the cylinder contains n moles of a gas. Now the piston is displaced
slowly by an external agent to make the volume double of its initial value. Find work done by external
agent in terms of n, R, T

Solution : 1st Method :


Work done by external agent is positive, because Fext and displacement are in the same direction.
Since walls are conducting therefore temperature remains constant.

Applying equilibrium condition when pressure of the gas is P


PA + Fext = Patm A
Fext = Patm A – PA

d d d d 2V
nRT
W ext = 
0
Fext dx =
P atm Adx –  PA dx = Patm A 
0
dx – 
V
V
dV
0 0

= Patm Ad – nRT ln2 = Patm . V0 – nRTln2 = nRT (1 – ln2)


2nd Method
Applying work energy theorem on the piston
As W all = K.E
K.E = 0 (given)

Vf
W gas + W atm + W ext = 0 nRT ln V – nRT + W ext = 0 W ext = nRT (1 – ln2)
i

Example 4. A non conducting piston of mass m and area of cross section A is placed on a non conducting
cylinder as shown in figure. Temperature, spring constant, height of the piston are given by T, K, h
respectively. Initially spring is relaxed and piston is at rest. Find
(i) Number of moles
(ii) Work done by gas to displace the piston by distance d when the gas is heated slowly.

(iii) Find the final temperature

KVPY_ SA STREAM # 311


 mg 
 Patm   Ah
 mg   A 
Solution : (i) PV = nRT   Patm   Ah = nRT
T  n=
 A  RT

(ii) 1st method


Applying Newton’s law on the piston
mg + Patm A + Kx = PgasA

d
d
W gas = P
0
gas A dx =
 (mg + Patm A + Kx) dx.
0

1 2
 W gas = mgd + PatmdA + Kd
2

2nd method
Applying work energy theorem on the piston
W all = KE
Since piston moves slowly therefore KE = 0
W gravity + W gas + W atm + W spring = 0
1 2 1 2
– mgd + W gas + (–Patm Ad) + [–( Kd – 0)] = 0  W gas = mgd + PatmdA + Kd
2 2

Example 5. Find out the work done in the given graph. Also draw the corresponding T-V curve and P-T curve.

Solution : Since in P-V curves area under the cycle is equal to work done therefore work done by the gas is
equal to P0 V0.
Line A B and CD are isochoric line, line BC and DA are isobaric line.
 the T-V curve and P-T curve are drawn as shown.

Example 6. T-V curve of cyclic process is shown below, number of moles of the gas are n find the total work done
during the cycle.

KVPY_ SA STREAM # 312


Solution : Since path AB and CD are isochoric therefore work done is zero during path AB and CD. Process
BC and DA are isothermal, therefore
VC
W BC = nR2T0 ln V = 2nRT0 ln 2
B

VA
W DA = nRT0 ln V = –nRT0 ln 2
D
Total work done = W BC + W DA = 2nRT0 ln 2 – nRT0 ln 2
= nRT0 ln 2
Example 7. P-T curve of a cyclic process is shown. Find out the work done by the gas in the given process if
number of moles of the gas are n.

Solution : Since path AB and CD are isochoric therefore work done during AB and CD is zero. Path BC and DA
are isobaric.
Hence W BC = nRT = nR(T3 – T2)
W DA = nR(T1 – T4) Total work done = W BC + W DA = nR(T1 + T3 –T4 – T2)

Example 8. In figure, a cyclic process ABCA of 3 moles of an ideal gas is given. The temperatures of the gas at
B and C are 500 K and 1000 K respectively. If the work done on the gas in process CA is 2500 J
then f ind the net heat absorbed or released by an ideal gas .
Take R = 25/3 J/mol–K.

Solution : The change in internal energy during the cyclic process is zero. Hence, the heat supplied to the gas
is equal to the work done by it. Hence,
Q = W AB + W BC + W CA .......(i)
The work done during the process AB is zero
W BC = PB (VC – VB)
= nR(TC – TB)
= (3 mol) (25/3 J/mol–K) (500 K)
= 12500 J
As W CA = – 2500 J (given)
 Q = 0 + 12500 – 2500 [from ......(i)]
Q = 10 kJ

FIRST LAW OF THERM ODYN AM ICS :
The first law of thermodynamics is the law of conservation of energy. It states that if a system absorbs
heat dQ and as a result the internal energy of the system changes by dU and the system does a work
dW, then dQ = dU + W.
But, W = P dV dQ = dU + P dV
which is the mathematical statement of first law of thermodynamics.
Heat gained by a system, work done by a system and increase in internal energy are taken as
positive.
Heat lost by a system, work done on a system and decrease in internal energy are taken as
negative.

KVPY_ SA STREAM # 313


Example 9. 1 gm water at 100ºC is heated to convert into steam at 100ºC at 1 atm. Find out change in internal
energy of water. It is given that volume of 1 gm water at 100ºC = 1 cc, volume of 1 gm steam at 100ºC
= 1671 cc. Latent heat of vaporization = 540 cal/g. (Mechanical equivalent of heat J = 4.2J/cal.)
Solution : From first law of thermodynamic Q = U + W
Q = mL = 1 × 540 cal. = 540 cal.

10 5 (1671  1)  10 6 10 5  (1670 )  10 6
W = PV = =  40 cal.
4 .2 4 .2
U = 540 – 40 = 500 cal.

Example 10. Two moles of a monoatomic gas at 300 K are kept in a non con-
ducting container enclosed by a piston. Gas is now compressed to
increase the temperature from 300 K to 400 K. Find work done by
25
the gas (R = J/mol–K)
3

Solution : Q = U + W
Since container is non conducting therefore
Q = 0 = U + W

f 3
 W = – U = –n R T = –2 × R (400 – 300)
2 2

25
=–3× × 100 J = – 2500 J
3

Example 11. In figure, a sample of an ideal gas is taken through the cyclic process abca. 800 J of work is done
by the gas during process ab. If gas absorb no heat in process ab, rejects 100 J of heat during bc
and absorb 500 J of heat during process ca. Then (a) find the internal energy of the gas at b and c
if it is 1000 J at a. (b) Also calculate the work done by the gas during the part bc.

Solution : (a) In process ab


Q = U + W
0 = UB – 100 + 800
UB = 200 J
for Cyclic process
Q = U + W
400 = 0 + 800 + W BC
W BC = – 400 J
for process bc
Q = U + W
– 100 = – 400 + UC – 200
 UC = 500 J

KVPY_ SA STREAM # 314


Example 12. Two moles of nitrogen gas is kept in a cylinder of cross-section area 10 cm2. The cylinder is closed
by a light frictionless piston. Now the gas is slowly heated such that the displacement of piston
during process is 50 cm, find the rise in temperature of gas when 200 J of heat is added in it.
(Atmospheric pressure = 100 kPa, R = 25/3 J/mol-K)
Solution : The change in internal energy of the gas is
U = 5/2 nR (T)
= 5/2 × 2R × (T) = 5R × T
The heat given to the gas = 200 J
The work done by the gas is
W = Q – U
= 200 J – 5R T .......(i)
As the distance moved by the piston is 50 cm,  the work done is
W = PV = PAx = 105 × 10 × 10–4 × 50 × 10–2 .......(ii)
From (i) and (ii)
T = 18/5 K = 3.6 K

Example 13. An ideal gas initially has pressure P volume V and temperature T. It is isothermally expanded to four
times of its original volume, then it is compressed at constant pressure to attain its original volume
V. Finally, the gas is heated at constant volume to get the original temperature T. (a) Draw V-T curve
(b) Calculate the total work done by the gas in the process. (given n2 = 0.693)
Solution :

(a) V–T curve for all process is shown in figure. The initial state is represented by the point A.
In the first step, it is isothermally expanded to a volume 4V. This is shown by AB. Then the
pressure is kept constant and the gas is compressed to the initial volume V. From the ideal
gas equation, V/T is constant at constant pressure (PV = nRT). Hence, the process is
shown by a line BC which passes through the origin. At point C, the volume is V. In the final
step, the gas is heated at constant volume to a temperature T. This is shown by CA. The
final state is the same as the initial state.
(b) Total work done by gas, W Total = W AB + W BC + W CA
4V
W AB = nRT In = 2nRT In 2 = 2PV In 2.
V
Also PA VA = PB VB (As AB is an isothermal process)
PA VA PV P
or, PB =   .
VB 4V 4
In the step BC, the pressure remains constant. Hence the work done is,
P 3PV
W BC = (V – 4V) = – .
4 4
In the step CA, the volume remains constant and so the work done is zero. The net work
done by the gas in the cyclic process is
W = W AB + W BC + W CA
3PV
= 2PV ln2 – +0
4
Hence, the work done by the gas 0.636 PV.

KVPY_ SA STREAM # 315


Example 14. A diatomic gas is heated at constant pressure. If 105 J of heat is given to the gas, find (a) the change
in internal energy of the gas (b) the work done by the gas.
Solution : Suppose the volume changes from V1 to V2 and the temperature changes from T1 to T2.
The heat supplied is

2U U  nfRT 


Q = U + PV = U + nRT = U + 
f  2 
(a) The change in internal energy is
 2
Q = U 1  
 f

 2
105 = U 1   , U = 75 J
 5
(b) The work done by the gas is
W = Q – U
= 105 J – 75 J = 30 J.


Ef ficie ncy of a cycle (  ) :
total Mechanical work done by the gas in the whole process

Heat absorbed by the gas (only  ve)

area under the cycle in P - V curve


=
Heat injected into the system

 Q   T 
  1  2  for Heat Engine,    1  2  for Carnot cycle
 Q1   T1 

Example 15. n moles of a diatomic gas has undergone a cyclic process ABCA as shown in figure. Temperature at
A is T0. Find
(i) Volume at C ?
(ii) Maximum temperature ?
(iii) Total heat given to gas ?
(iv) Is heat rejected by the gas, if yes how much heat is rejected ?
(v) Find out the efficiency
Solution : (i) For process AC, P  V

2P0 P0
=  Vc = 2V0
Vc V0

(ii) Since process AB is isochoric hence

PA PB
=  TB = 2T0
TA TB

TB TC
Since process BC is isobaric therefore =
VB VC

 TC = 2TB = 4 T0  Tmax = 4T0

KVPY_ SA STREAM # 316


(iii) Since process is cyclic therefore

1
Q = W = area under the cycle = PV .
2 0 0
(iv) Since U and W both are negative in process CA
 Q is negative in process CA and heat is rejected in process CA
QCA = W CA + UCA

1 5
=  [P0 + 2P0] V0 – nR (TC – TA )
2 2

1 5  4P0 V0 P0 V0 
=  [P0 + 2P0] V0 – nR   
2 2  nR nR 

= –9P0V0 (Heat rejected)

work done by the gas P0 V0 / 2


(v)  = efficiency of the cycle =  = × 100
heat injected Qinjected

Qinj = QAB + QBC

5  5  19
=  nR(2T0  T0 ) +  nR(2T0 )  2P0 (2V0  V0 ) = PV.
2  2  2 0 0

100
= %
19


SP ECI FI C HEAT :
The specific heat capacity of a substance is defined as the heat supplied per unit mass of the substance
per unit rise in the temperature. If an amount Q of heat is given to a mass m of the substance and its
temperature rises by T, the specific heat capacity s is given by equation
Q
s=
m T
The molar heat capacity of a gas is defined as the heat given per mole of the gas per unit rise in the
temperature. The molar heat capacity at constant volume, denoted by C V, is :
 Q  f
Cv =  n T  = R
  cons tan t volume 2
and the molar heat capacity at constant pressure, denoted by C p is,
 Q  f 
CP =  n T  =   1 R
  cons tan t pressure 2 
where n is the amount of the gas in number of moles and f is degree of freedom. Quite often, the term
specific heat capacity or specific heat is used for molar heat capacity. It is advised that the unit be
carefully noted to determine the actual meaning. The unit of specific heat capacity is J/kg-K whereas
that of molar heat capacity is J/mol–K.

M OLAR HEAT CAPACITY OF ID EAL GAS IN TERM S OF R :


(i) For a monoatomic gas f = 3
3 5 CP 5
CV = R , CP  R  = C  3  1.67
2 2 V

KVPY_ SA STREAM # 317


(ii) For a diatmoc gas f = 5
5 7 CP
CV  R, CP  R ,   1.4
2 2 CV

(iii) For a Triatomic gas f = 6


C V  3R, CP  4R
CP 4
   1.33 [Note for CO 2; f = 5, it is linear]
CV 3
In general if f is the degree of freedom of a molecule , then ,
f f  CP  2 
CV  R , CP    1R ,   1 
2 2  C V  f

Example 16. Two moles of a diatomic gas at 300 K are enclosed in a cylinder as shown in figure. Piston is light.
Find out the heat given if the gas is slowly heated to 400 K in the following three cases.
(i) Piston is free to move
(ii) If piston does not move
(iii) If piston is heavy and movable.
Solution : (i) Since pressure is constant

7
 Q = nCP T = 2 × × R × (400 – 300) = 700 R
2

(ii) Since volume is constant


 W = 0 and Q = U (from first law)

5
Q = U = nCVT = 2× × R × (400 – 300) = 500 R
2

(iii) Since pressure is constant

7
 Q = nCP T = 2 × × R × (400 – 300) = 700 R
2

Example 17. P-V curve of a diatomic gas is shown in the figure. Find the total heat given to the gas in the process
AB and BC

Solution : From first law of thermodynamics


QABC = UABC + W ABC
VC 2 V0
W ABC = W AB + W BC = 0 + nR TB ln = nR TB ln V = nRTB ln 2 = 2P0 V0 ln 2
VB 0

5 5
U = nCV T = (2P0V0 – P0V0)  QABC = P V + 2P0V0 ln 2.
2 2 0 0

KVPY_ SA STREAM # 318


Example 18. From given data, calculate the value of mechanical equivalent of heat. The specific heat capacity of
air at constant volume 170 cal/kg-K, =Cp/Cv = 1.4 and the density of air at STP is 1.29 kg/m3. Gas
constant R = 8.3 J/mol-K.
Solution : Using pV = nRT, the volume of 1 mole of air at STP is
nRT (1 mol)  (8.3 J / mol  K )  (273K )
V = = 0.0224m3.
p 1.01 10 5 N / m2
The mass of 1 mole is, therefore,
(1.29 kg/m3) × (0.0224 m3) = 0.029 kg.
1
The number of moles in 1 kg is . The molar heat capacity at constant volume is
0.029
170 cal
Cv  = 4.93 cal/mol-K.
(1/ 0.029 ) mol  K
Hence, Cp = Cv = 1.4 × 4.93 cal/mol-K
or, Cp – Cv = 0.4 × 4.93 cal/mol-K
= 1.97 cal/mol-K.
Also, Cp – Cv = R = 8.3 J/mol-K.
Thus, 8.3 J = 1.97 cal.
The mechanical equivalent of heat is
8.3 J
=4.2 J/cal.
1.97 cal


Avera ge M ola r Spe cific Hea t of M e ta ls :
[Dulong a nd Pe tit la w]
At room temperature average molar specific heat of all metals are same and is nearly equal to 3R
( 6 cal. mol -1 K-1 ).
[Note : Temp. above which the metals have constant CV is called Debye temp.]

M AYER'S EQUATION : CP  CV = R (for ideal gases only)

Adia ba t ic proce ss :
When no heat is supplied or extracted from the system the process is called adiabatic. Process is
sudden so that there is no time for exchange of heat. If walls of a container are thermally insulated no
heat can cross the boundary of the system and process is adiabatic.

Equation of adiabatic process is given by


PV = constant [Poisson Law]
T  P1– = constant
T V 1 = constant

Slope of PVcurve in adiabatic process : Since PV  is a constant

dP P
 =   
dV V

KVPY_ SA STREAM # 319


Slope of PTcurve in adiabatic process : Since T  P1– is a constant

dP  P ( ) P
  =
dT (1   ) T (   1) T

Slope of TVcurve :

dV 1 V
=
dT (   1) T

Work done in adiabatic Process :


PVi i  Pf Vf nR(Ti  Tf )
W  U  nCv (Ti  Tf )  
(   1)  1
work done by system is (+ve) , if T i > T f (For expansion)
work done on the system is (ve) if T i < T f (For compression)


Example 19. A container having slightly conducting walls contains air. The initial temperature and volume are
47ºC (equal to the temperature of the surrounding) and 400cm3 respectively. Find the rise in the
temperature if the gas is compressed to 200cm3 (a) in a short time (b) in a long time. Take  = 1.4.
[20.4 = 1.3]
Solution : (a) When the gas is compressed in a short time, the process is adiabatic. Thus,
T2V2 –1 = T1V1–1
 1 0.4
V   400 
or T2  T1 1  = (320 K) ×  200  = 416 K.
 V2   
Rise in temperature = T2 – T1 = 96 K.
(b) When the gas is compressed in a long time, the process is isothermal. Thus, the temperature
remains same that is 47ºC  The rise in temperature = 0.

Example 20. An ideal monoatomic gas is enclosed in a non conducting cylin-


der having a piston which can move freely. Suddenly gas is com-
pressed to 1/8 of its initial volume. Find the final pressure and
temperature if initial pressure and temperature are P0 and T0 re-
spectively.

Solution : Since process is adiabatic therefore

5/3
5 V  CP 5R 3R 5 
P0 V3 = Pfinal   .    /  
8  CV 2 2 3

Pfinal = 32 P0 .
Since process is adiabatic therefore

2/3
 V0 
T1 V1–1 = T2 V2–1  T0 V02/3 = Tfinal    T = 4T0
 8 

KVPY_ SA STREAM # 320


Example 21. A cylindrical container having non conducting walls is partitioned in two equal parts such that the
volume of each part is V0 . A movable non conducting piston is kept between the two parts. Gas on
V0
left is slowly heated so that the gas on right is compressed upto volume . Find pressure and
8
temperature on both sides if initial pressure and temperature, were P0 and T0 respectively. Also find
heat given by the heater to the gas. (number of moles in each part is n )

Solution : Since the process on right is adiabatic therefore


PV = constant
 P0 V0 = Pfinal (V0 / 8)  Pfinal = 32 P0
T0 V0–1 = Tfinal (V0/8)– 1  Tfinal = 4T0
Let volume of the left part is V1
V0 15 V0
 2V0 = V1 +  V1 = .
8 8
PV
Since number of moles on left part remains constant therefore for the left part = constant.
T
Final pressure on both sides will be same
P0 V0 Pfinal V1
 =  Tfinal = 60 T0
T0 Tfinal
Q = U + W
5R 3R 5nR 3nR
Q = n (60T0 – T0) + n (4T0 – T0) Q = × 59T0 + × 3T0 = 152 nRT0
2 2 2 2


FREE EXPAN SI ON
If a system, say a gas expands in such a way that no heat enters or leaves the system and also no
work is done by or on the system , then the expansion is called the "free expansion".
Q = 0 , U = 0 and W = 0. Temperature in the free expansion remains constant.

Example 22.
A non conducting cylinder having volume 2V0 is partitioned by a fixed non conducting wall in two equal parts.
Partition is attached with a valve. Right side of the partition is a vacuum and left part is filled with a gas having
pressure and temperature P0 and T0 respectively. If valve is opened find the final pressure and temperature of
the two parts.

Solution : From the first law thermodynamics Q = U + W


Since gas expands freely therefore W = 0 , since no heat is given to gas Q = 0
 U = 0 and temperature remains constant.
Tfinal = T0
Since the process is isothermal therefore P0 × V0 = Pfinal × 2V0  Pfinal = P0/2

KVPY_ SA STREAM # 321



Com pa rison of slope s of a n I so-t he rm a l a nd Adia ba t ic Curve

dP dP

dV adia dV isothermal

In compression up to same final volume: | W adia |> | W isothermal |


In Expansion up to same final volume: W isothermal > W adia

Lim ita tions of Ist La w of Therm odyna m ics :


The first law of thermodynamics tells us that heat and mechanical work are interconvertible. However,
this law fails to explain the following points :
(i) It does not tell us about the direction of transfer of heat.
(ii) It does not tell us about the conditions under which heat energy is converted into work.
(iii) It does not tell us whether some process is possible or not.

M ixture of non-rea cting ga se s:


n1 M1  n 2 M2
(a) Molecular weight = ,
n1  n2
M1 & M2 are molar masses.
n1 C V1  n 2 C V2
(b) Specific heat CV = ,
n1  n 2

n1 CP1  n2 CP2
CP =
n1  n2

Cpmix n1Cp1  n2 Cp2  .......


(c) for mixture,  = C 
vmix n1Cv1  n2 Cv 2  .......

2. COND UCTI ON
The process of transmission of heat energy in which heat is transferred from one particle of the medium to
the other, but each particle of the medium stays at its own position is called conduction, for example if you
hold an iron rod with one of its end on a fire for some time, the handle will get hot. The heat is transferred from
the fire to the handle by conduction along the length of iron rod. The vibrational amplitude of atoms and
electrons of the iron rod at the hot end takes on relatively higher values due to the higher temperature of their
environment. These increased vibrational amplitude are transferred along the rod, from atom to atom during
collision between adjacent atoms. In this way a region of rising temperature extends itself along the rod
to your hand. Consider a slab of face area A, Lateral thickness L, whose faces have temperatures TH and
TC(TH > TC). L
TC

TH
Q2

Q1
O A B
x dx
Now consider two cross sections in the slab at positions A and B separated by a lateral distance of dx. Let
temperature of face A be T and that of face B be T + T. Then experiments show that Q, the amount of heat
crossing the area A of the slab at position x in time t is given by

KVPY_ SA STREAM # 322


Q dT
= –KA .............(2.1)
t dx

Here K is a constant depending on the material of the slab and is named thermal conductivity of the material,

 dT 
and the quantity   is called temperature gradient. The (–) sign in equation (2.1) shows heat flows from
 dx 
high to low temperature (T is a –ve quantity)

3. STEADY STATE
If the temperature of a cross-section at any position x in the above slab remains constant with time (remem-
ber, it does vary with position x), the slab is said to be in steady state.
Remember steady-state is distinct from thermal equilibrium for which temperature at any position (x) in the
slab must be same.
For a conductor in steady state there is no absorption or emission of heat at any cross-section. (as tempera-
ture at each point remains constant with time). The left and right face are maintained at constant tempera-
tures TH and TC respectively, and all other faces must be covered with adiabatic walls so that no heat
escapes through them and same amount of heat flows through each cross-section in a given Interval of time.
Hence Q1 = Q = Q2. Consequently the temperature gradient is constant throughout the slab.
dT T Tf  Ti TC  TH
Hence, = = =
dx L L L

Q T Q  TH  TC 
and = –KA  = KA   .... (3.1)
t L t  L 
Here Q is the amount of heat flowing through a cross-section of slab at any position in a time interval of t.

Example 23. One face of an aluminium cube of edge 2 metre is maintained at 100ºC and the other end is main-
tained at 0ºC. All other surfaces are covered by adiabatic walls. Find the amount of heat flowing
through the cube in 5 seconds. (thermal conductivity of aluminium is 209 W/m–ºC)
Solution : Heat will flow from the end at 100ºC to the end at 0ºC.
Area of cross-section perpendicular to direction of heat flow,
A = 4m2
Q (TH  TC )
then = KA
t L

(209 W / mº C)(4m 2 )(100 º C  0º C)(5 sec)


Q= = 209 KJ Ans.
2m


4. THERM AL RESISTANCE TO CONDUCTION
If you are interested in insulating your house from cold weather or for that matter keeping the meal hot in your
tiffin-box, you are more interested in poor heat conductors, rather than good conductors. For this reason, the
concept of thermal resistance R has been introduced.
For a slab of cross-section A, Lateral thickness L and thermal conductivity K,
L
R ................ (4.1)
KA
In terms of R, the amount of heat flowing though a slab in steady-state (in time t)
Q (TH  TC )

t R

KVPY_ SA STREAM # 323


Q
If we name as thermal current iT
t
TH  TC
then, iT  (4.2)
R
This is mathematically equivalent to OHM’s law, with temperature playing the role of electric potential. Hence
results derived from OHM’s law are also valid for thermal conduction.
More over, for a slab in steady state we have seen earlier that the thermal current iL remains same at each
cross-section. This is analogous to kirchoff’s current law in electricity, which can now be very conveniently
applied to thermal conduction.

50ºC
Example 24. Three identical rods of length 1m each, having cross-section
area of 1cm2 each and made of Aluminium, copper and steel

l
ee
respectively are maintained at temperatures of 12ºC, 4ºC and

st
50ºC respectively at their separate ends. 12ºC
Aluminium
Find the temperature of their common junction.

co
[ KCu = 400 W/m-K , KAl = 200 W/m-K , Ksteel = 50 W/m-K ]

pp
e
r
L 1 10 4 4ºC
Solution : RAl = = 4 =
KA 10  200 200

10 4 10 4
Similarly Rsteel = and Rcopper =
50 400
Let temperature of common junction = T
then from Kirchoff;s current laws,
iAl + isteel + iCu = 0

T  12 T  50 T4
 + + =0
R Al Rsteel R Cu

 (T – 12) 200 + (T – 50) 50 + (T – 4) 400


 4(T – 12) + (T – 50) + 8 (T – 4) = 0
 13T = 48 + 50 + 32 = 130
 T = 10ºC Ans.


5. SLABS IN PARALLEL AND SERIES
5 .1 Sla bs in series (in stea dy sta te)
Consider a composite slab consisting of two materials having different thicknesses L1 and L2 differ-
ent cross-sectional areas A1 and A2 and different thermal conductivities K1 and K2. The temperature
at the outer surface of the slabs are maintained at TH and TC, and all lateral surfaces are covered by
an adiabatic coating.

KVPY_ SA STREAM # 324


Let temperature at the junction be T, since steady state has been achieved thermal current through each
slab will be equal. Then thermal current through the first slab.

Q TH  T
i= =
R1
or TH – T = iR1 ...............(5.1)
t
and that through the second slab,

Q T  TC
i= = or T – TC = iR2 ...............(5.2)
t R2

adding eqn. 5.1 and eqn 5.2

TH  TC
TH – TL = (R1 + R2) i or i = R R
1 2

Thus these two slabs are equivalent to a single slab of thermal resistance R1 + R2.
If more than two slabs are joined in series and are allowed to attain steady state, then equivalent thermal
resistance is given by
R = R1 + R2 + R3 + ....... ...(5.3)

Example 25 The figure shows the cross-section of the outer wall of a house built in a hill-resort to keep the house
insulated from the freezing temperature of outside. The wall consists of teak wood of thickness L1
and brick of thickness (L2 = 5L1), sandwitching two layers of an unknown material with identical
thermal conductivities and thickness. The thermal conductivity of teak wood is K1 and that of brick is
(K2 = 5K). Heat conduction through the wall has reached a steady state with the temperature of
three surfaces being known. (T1 = 25ºC, T2 = 20ºC and T5 = –20ºC). Find the interface temperature
T4 and T3.
T1 T2 T3 T4 T5

L1 L L L4
Solution : Let interface area be A. then thermal resistance of wood,

L1
R1 = K A
1

and that of brick wall

L2 5L1
R2 = K A = 5K A = R1
2 1

Let thermal resistance of the each sand witch layer = R. Then the above wall can be visualised as a
circuit
iT R1 R R R1 iT

25ºC 20ºC T3 T4 –20ºC

thermal current through each wall is same.

KVPY_ SA STREAM # 325


25  20 20  T3 T  T4 T4  20
Hence = = 3 =
R1 R R R1

 25 – 20 = T4 + 20  T4 = –15ºC Ans.

also, 20 – T3 = T3 – T4

20  T4
 T3 = = 2.5ºC Ans.
2

Example 26. In example 3, K1 = 0.125 W/m–ºC, K2 = 5K1 = 0.625 W/m–ºC and thermal conductivity of the
unknown material is K = 0.25 W/mºC. L1 = 4cm, L2 = 5L1 = 20cm. If the house consists of a single
room of total wall area of 100 m2, then find the power of the electric heater being used in the room.

(4  10 2 m)
Solution : Ist method R1 = R2 = = 32 × 10–4 ºC/w
(0.125 w / mº C)(100m2 )

25 – 20 20 – T3 17.5 K
 =  L= × L = 28 cm
R1 R 5 K1 1

L
R= = 112 × 10–4 ºC/W
KA
the equivalent thermal resistance of the entire wall = R1 + R2 + 2R = 288 × 10–4 ºC/W
TH  TC
 Net heat current, i.e. amount of heat flowing out of the house per second =
R
25º C  (20º C)
45  10 4
= 4 =
watt = 1.56 Kwatt
288  10 º C / w 288
Hence the heater must supply 1.56 kW to compensate for the outflow of heat. Ans.
nd
II method
T1 – T2 25 – 20
i= = = 1.56 Kwatt
R1 32  10 – 4


5 .2 Sla bs in pa ra llel :
L

SLAB 1 Q1
K1 A1

SLAB 2 Q2
K2 A2
Heat reservoir
at temperature TH adiabatic coating
Heat reservoir
at temperature TC

Consider two slabs held between the same heat reservoirs, their thermal conductivities K1 and K2 and cross-
sectional areas A1 and A2
L L
then R1 = , R2 =
K 1A1 K 2A 2
TH  TC
thermal current through slab 1 i1 
R1

KVPY_ SA STREAM # 326


TH  TC
and that through slab 2 i2 
R2
Net heat current from the hot to cold reservoir
 1 1 
i = i1  i2  (TH  TC )  
R
 1 R 2
TH  TC 1 1 1
Comparing with i = , we get, = 
R eq R eq R1 R 2
If more than two rods are joined in parallel, the equivalent thermal resistance is given by
1 1 1 1

R eq = R1 R 2 + R 3 + ..... ...............(5.4)

Example 27. Three copper rods and three steel rods each of length  = 10 cm and area of cross-section 1 cm2 are
connected as shown

C
steel copper

steel
copper
E (0ºC)
A B
(125ºC) copper
steel

If ends A and E are maintained at temperatures 125ºC and 0ºC respectively, calculate the amount of
heat flowing per second from the hot to cold function. [ KCu = 400 W/m-K , Ksteel = 50 W/m-K ]

L 10 1m 1000
Solution : Rsteel = = 4 2 = ºC/W..
KA 50( W / mº C)  10 m 50

1000
Similarly RCu = ºC/W
400
Junction C and D are identical in every respect and both will have same temperature. Consequently,
the rod CD is in thermal equilibrium and no heat will flow through it. Hence it can be neglected in
further analysis.
Now rod BC and CE are in series their equivalent resistance is R1 = RS + RCu similarly rods BD and
DE are in series with same equivalent resistance R1 = RS + RCu these two are in parallel giving an
equivalent resistance of

R1 R  R Cu
= S
2 2
This resistance is connected in series with rod AB. Hence the net equivalent of the combination is

R1 3R steel  R Cu  3 1 
R = Rsteel + = = 500   C / W
2 2  50 400 

TH  TC 125 º C
Now i= = = 4 watt. Ans.
R  3 1 
500  º C / W
 50 400 

KVPY_ SA STREAM # 327


1. The ratio of r.m.s. speed to the r.ms. angular speed of a diatomic gas at certain temperature is:
(assume m = mass of one molecule, M = molecular mass,  = moment of inertia of the molecules)

3 3I 3I
(A) (B) (C) (D) 1
2 2M 2m

2. A gas mixture consists of 2 moles of oxygen and 4 moles of argon at temperature T. Neglecting all
vibrational modes, the total internal energy of the system is:
(A) 4 R T (B) 5 R T (C) 15 R T (D) 11 R T

3. V–T diagram for a process of a given mass of ideal gas is as shown in the figure. During the process
pressure of gas.

(A) first increases then decreases (B) continuously decreases


(C) continuously increases (D) first decreases then increases.

4. The molecules of an ideal gas have 6 degrees of freedom. The temperature of the gas is T. The average
translational kinetic energy of its molecules is:
3 6 1
(A) kT (B) kT (C) k T (D) kT
2 2 2

5. Four particles have velocities 1, 0, 2, 3 m/s. The root mean square velocity of the particles is: (in m/s)
14
(A) 3.5 (B) 3.5 (C) 1.5 (D)
3

6. In a process the density of a gas remains constant. If the temperature is doubled, then the change in
the pressure will be:
(A) 100 % (B) 200 % (C) 50 % (D) 25 %

7. Which of the following will have maximum total kinetic energy at temperature 300 K.
1 1 1 3
(A) 1 kg, H2 (B) 1 kg , He (C) kg H2 + kg He (D) kg H2 + kg He
2 2 4 4

8. Assume a sample of an ideal gas in a vessel. Where velocity of molecules are between 2 m/sec to
5 m/sec and velocity of molecules (v) and number of molecules (n) are related as n = 7v – v 2 – 10. The most
probable velocity in sample is. Where v is measured in m/sec.
(A) 3.5 m/sec (B) 5 m/sec (C) 10 m/sec (D) 4 m/sec

9. A certain gas is taken to the five states represented by dots in the


graph. The plotted lines are isotherms. Order of the most probable
speed v p of the molecules at these five states is :
(A) VP at 3 > VP at 1 = VP at 2 > VP at 4 = VP at 5
(B) VP at 1 > VP at 2 = VP at 3 > VP at 4 > VP at 5
(C) VP at 3 > VP at 2 = VP at 4 > VP at 1 > VP at 5
(D) Insufficient information to predict the result.

KVPY_ SA STREAM # 328


10. At pressure P and absolute temperature T a mass M of an ideal gas fills a closed container of volume
V. An additional mass 2M of the same gas is added into the container and the volume is then reduced
V T
to and the temperature to . The pressure of the gas will now be:
3 3
P
(A) (B) P (C) 3 P (D) 9 P
3

11. An insulated container is divided into two equal portions. One portion contains one mole of an ideal monoatomic
gas at pressure P and temperature T, while the other portion is a perfect vacuum. If the intermediate partition
is removed and the gas is allowed to expand, the change in the internal energy of gas is

P, V V
T

3 RT
(A) RT (B) PV (C) (D) Zero
2 2

12. Two identical rooms in a perfectly insulated house are connected by an open doorway. The temperature in
the two rooms are maintained at different values. The room which contains more air molecules is :
(A) the one with higher temperature (B) the one with lower temperature
(C) the one with higher pressure (D) neither since both have same volume

13. For an adiabatic process graph between PV and V for a monoatomic gas is :

PV PV PV PV

(A) (B) (C) (D)


V V V V

14. In a cyclic waste power plant for thermal decomposition of waste hot gases are supplied from a "Kentuki
Chamber" where gases are heated under kentuki process P = kVT 3 where P, V and T are pressure, volume
and temperature of gases in chamber and k is positive constant. Find work done by 5 mole of hot gases
when temperature is raised from 500 K to 550 K :
(A) 625 R (B) – 625 R (C) 250 R (D) – 250 R

15. One mole of an ideal gas is kept enclosed under a light piston (area=10–2 m2) connected by a
compressed spring (spring constant 100 N/m). The volume of gas is 0.83 m 3 and its
temperature is 100K. The gas is heated so that it compresses the spring further by 0.1
m. The work done by the gas in the process is : (Take R = 8.3 J/K-mole and suppose
there is no atmosphere).
(A) 3 J (B) 6J (C) 9J (D) 1.5 J

16.* Heat energy absorbed by a system in going through a cyclic process is shown in the figure [ V in litres
and p in kPa ] is:

(A) 107  J (B) 104  J (C) 102  J (D) 107  J

KVPY_ SA STREAM # 329


17. One mole of an ideal gas is taken from state A to state B by three different
processes, (a) ACB (b) ADB (c) AEB as shown in the P  V diagram. The heat
absorbed by the gas is:
(A) greater in process (b) then in (a)
(B) the least in process (b)
(C) the same in (a) and (c)
(D) less in (c) than in (b)

18. In a cyclic process ABCA for an ideal gas. In AB, BC and CA process 50 J, 20 J and 5 J heat is supplied to
an ideal gas. In process AB internal energy of gas increases by 60 J and in process BC work done by gas is
30 J. The increase in internal energy of gas in process CA is :
(A) 50 J (B) – 50 J (C) 75 J (D) 55 J

19. P-T diagram is shown below then choose the corresponding V-T diagram

(A) (B) (C) (D)

20. A thermodynamic process of one mole ideal monoatomic gas is shown in figure. The efficiency of cyclic
process ABCA will be :

100
(A) 25% (B) 12.5% (C) 50% (D) %
13

21. An ideal gas undergoes a thermodynamic cycle as shown in figure.:

Which of the following graphs may represent the same cycle ?

(A) (B) (C) (D)

KVPY_ SA STREAM # 330


22. 1 mole of an ideal gas undergoes an isothermal expansion as energy is added to it as heat Q. Graph
shows the volume V versus Q. The gas temperature is nearly equal to : (use R = 8.31 J/K.mole)

(A) 208.4 K (B) 268.2 K (C) 312.6 K (D) 353.8 K


p(Pa)
23. Curve in the figure shows an adiabatic compression of an ideal gas from
15 m3 to 12 m3, followed by an isothermal compression to a final volume of
3.0 m3. There are 2.0 moles of the gas. Total heat supplied to the gas is
400
equal to : (n2 = 0.693)
(A) 4521 J (B) –4521 J
3
(C) –6653 J (D) –8476 J V(m )
3 12 15

1
24. The correct curve between V/T and   for an ideal gas at constant pressure is :
V

(A) (B) (C) (D)

25. A mixture of ideal gasses N2 and He are taken in the mass ratio of 14 : 1 respectively. Molar heat
capacity of the mixture at constant pressure is.

19R 6R 13 R 6R
(A) (B) (C) (D)
6 19 6 13

26. U = 0 in a noncyclic process of an ideal gas. The process :


(A) may be isothermal (B) must be isothermal (C) may be adiabatic (D) may be isobaric.

27. An ideal monoatomic gas initially at 300 K undergoes an isobaric expansion at a pressure of 2.5 kPa.
If the volume increases from 1m3 to 3m3, then heat added to the gas and its final temperature respectively are :
(A) 12500 J, 450 K (B) 12500 J, 600 K (C) 12500 J, 900 K (D) 25000 J, 1200 K

28. A non–conducting container is divided into two chambers that are separated by a valve. The left chamber
contains one mole of a monatomic ideal gas. The right chamber is evacuated. At some instant, the valve is
opened and the gas rushes freely into the right chamber. Which are of the following statements concerning
this process is true?
(A) Work is done by the gas.
(B) The temperature of the gas decreases. valve

(C) The walls of the containing vessel must get colder.


(D) The change in the internal energy of the gas is zero.

29. An ideal gas with adiabatic exponent ( = 1.5) undergoes a process in which work done by the gas is
same as increase in internal energy of the gas. The molar heat capacity of gas for the process is:
(A) C = 4R (B) C = 0 (C) C= 2R (D) C = R

KVPY_ SA STREAM # 331


30. The ratio of specific heats at constant pressure and constant volume of a gas is 4/3. Then the average
number of degree of freedom of the gas molecules is :
(A) 3 (B) 5 (C) 6 (D) 7
31. A wall is made of two layers A and B of the same thickness but different materials
as shown. The thermal conductivity of A is thrice that of B. In steady state, the
temperature difference across the wall is 360C. The temperature difference across
the layer A is :
(A) 60 C (B) 90 C (C) 180 C (D) 270 C
32. A rod of length  and cross section area A has a variable thermal conductivity given by k =  T, where  is a
positive constant and T is temperature in kelvin. Two ends of the rod are maintained at temperatures T1 and
T2 (T1 > T2). Heat current flowing through the rod will be

A  (T12  T22 ) A  (T12  T22 ) A  (T12  T22 ) A  (T12  T22 )


(A) (B) (C) (D)
  3 2

33. Two identical long, solid cylinders are used to conduct heat from temp T 1 to temp T 2. Originally the
cylinder are connected in series and the rate of heat transfer is H. If the cylinders are connected in
parallel then the rate of heat transfer would be :
(A) H /4 (B) 2H (C) 4H (D) 8H
34. A solid spherical black body of radius r and uniform mass distribution is in free space. It emits power
‘P’ and its rate of colling is R then
1
(A) R P  r2 (B) R P  r (C) R P  1/r2 (D) R P 
r
35. Two bodies A and B have emissivities 0.5 and 0.8 respectively. At some temperatures the two bodies
have maximum spectral emissive powers at wavelength 8000 Å and 4000 Å respectively. The ratio of
their emissive powers at these temperatures is:
5 5
(A) (B) 10 (C) (D) None of these
128 16

KVPY PROBLEMS (PREVIOUS YEARS)


1.* You are riding on your bicycle with inflated tyres. Your friend asks for a lift and sits on the carrier behind you
(A) The air pressure in the tyres increases. [KVPY_2007]
(B) The air pressure in the tyres decreases.
(C) The air pressure in the tyres remains the same.
(D) Nothing in the system changes except the reaction of the ground
2. A molecule of gas in a container hits one wall (1) normally and Wall 1

rebounds back. It suffers no collision and hits the opposite wall (2)
which is at an angle of 30º with wall 1. [KVPY_2008]
Assuming the collisions to be elastic and the small collision time
to be the same for both the walls, the magnitude of average force by
wall 2 (F2) provided to the molecule during collision satisfy : (mag-
nitude of average force by wall 1 is F1) 30º
(A) F1 > F2 Wall 2

(B) F1 < F2
(C) F1 = F2, both non-zero
(D) F1 = F2 = 0

3. On a day during the Monsoon season, the relative humidity at a place is 90% and the temperature is 15°(C)
The saturation vapour pressure of water at this temperature is 0.0169 × 105 Pa. The partial pressure of water
vapour in the air on that day is [KVPY_2008]
(A) 0.9 × 105 Pa (B) 0.0169 × 105 Pa (C) 0.0152 x 105 Pa (D) 1.0 x 105 Pa

KVPY_ SA STREAM # 332


4. We sit in the room with windows open. Then [KVPY_2011]
(A) Air pressure on the floor of the room equals the atmospheric pressure but the air pressure on the ceiling
is negligible
(B) Air pressure is nearly the same on the floor, the walls and ceiling
(C) Air pressure on the floor equals the weight of the air coloumn inside the room (from floor to ceiling) per
unit area
(D) Air pressure on the walls is zero since the weight of air acts downward

5. The number of air molecules in a (5m × 5m × 4m) room at standard temperature and pressure is of the order
of
(A) 6 × 1023 (B) 3 × 1024 (C) 3 × 1027 (D) 6 × 1030 [KVPY_2011]

6. In a thermally isolated system. Two boxes filled with an ideal gas are connected by a valve. When the valve
is in closed position, states of the box 1 and 2. respectively, are (1 atm, V, T) and (0.5 atm, 4V, T). When the
valve is opened, the final pressure of the system is approximately.
(A) 0.5 atm (B) 0.6 atm (C) 0.75 atm (D) 1.0 atm [KVPY_2011]

7. A coolant in a chemical or nuclear plant is a liquid that is used to prevent different parts of a plant from getting
too hot. One important property of coolant is that it [KVPY_2007]
(A) should have low specific heat. (B) should have high specific heat.
(C) high density and low thermal conductivity. (D) low density and high thermal conductivity.

8. A solid cube and a solid sphere of identical material and equal masses are heated to the same temperature
and left to cool in the same surroundings. Then [KVPY_2009]
(A) the cube will cool faster because of its sharp edges.
(B) the cube will cool faster because it has a larger surface area.
(C) the sphere will cool faster because it is smooth.
(D) the sphere will cool faster because it has a larger surface area.

9. In the Arctic region hemispherical houses called Igloos are made of ice. It is possible to maintain inside an
Igloo as high as 20ºC because : [KVPY_2012]
(A) ice has high thermal conductivity (B) ice has low thermal conductivity
(C) ice has high specific heat (D) ice has higher density than water

10. An ideal gas filled in a cylinder occupies volume V. The gas is compressed isothermally to the volume V/3.
Now the cylinder valve is opened and the gas is allowed to leak keeping temperature same. What percentage
of the number of molecules escape to bring the pressure in the cylinder back to its original valuye. [KVPY_2013]
(A) 66% (B) 33% (C) 0.33% (D) 0.66%

11. An ideal gas follows aprocess described by PV2=C from (P1,V1,T1) to (P2,V2,T2)(C is a constant). Then
[KVPY_2014_SB]
(A) if P1 > P2 then T2 > T1 (B) if V2 > V1 then T2 < T1
(C) if V2 > V1 then T2 > T1 (D) if P1 > P2 then V2 > V2

12. One mole of a monoatomic ideal gas is expanded by a process described by PV3 = C where C is a
constant. The heat capacity of the gas during the process is given by (R is the gas constant) [KVPY_2014_SB]

5 3
(A) 2R (B) R (C) R (D) R
2 2

KVPY_ SA STREAM # 333


13. One mole of ideal gas undergoes a linear process as shown in figure below. Its temperature expressed
as a function of volume V is. [KVPY_2015_SA] [1 Mark]

P0

(0, 0) V0

 V    V 2 
P0V0 P0V P0V
1  
P0V 1    
(A) (B) (C) (D) R   V0  
R R R  V0   

14. A closed bottle containing water at 30ºC is open on the surface of the moon. Then
[KVPY_2015_SB] [1 Mark]
(A) the water will boil (B) the water will come out as a spherical ball
(C) the water will freeze (D) the water decompose into hydrogen and oxygen

15. The state of an ideal gas was changed isobarically. The graph depicts three such isobaric lines. Which
of the following is true about the pressures of the gas? [KVPY_2015_SB] [1 Mark]
V
P2
P3

P1

T
(A) P1  P2  P3 (B) P1  P2  P3 (C) P1  P2  P3 (D) P1 /P2  P3 /P1

16. An ideal gas undergoes a circular cycle centered at 4atm, 4 lit as shown in the diagram. The maximum
temperature attained in this process is close to [KVPY_2015_SB] [2 Marks]
P(atm)

2 4 6
V(lit)
(A) 30/R (B) 36/R (C) 24/R (D) 16/R

KVPY_ SA STREAM # 334


17. A gas at initial temperature T undergoes sudden expansion from volume V to 2V. Then
(A) the process is adiabatic [KVPY_2016_SB] [1 Mark]
(B) the process is isothermal
(C) the work done in this process is nRTlne(2) where n is the number of moles of the gas
(D) the entropy in the process does not change.
18. A container is divided into two equal part  and  by a partition with a small hole of diameter d. The two
partitions are filled with same ideal gas, but held at temperatures T = 150 K and T = 300 K by connecting
to heat reservoirs. Let and  be the mean free paths of the gas particles in the two parts such that d >>
 and d >> . Then is close [KVPY_2016_SB] [1 Mark]
(A) 0.25 (B) 0.5 (C) 0.7 (D) 1.0
19. An ideal gas is reversibly around the cycle a-b-c-d-a as shown on the T (temperature) –S (entropy) diagram
[KVPY_2016_SB] [2 Mark]

b c

a d

The most appropriate representation of above cycle on a U (internal energy) – V (volume) diagram is.

U U

c c
b b

(A) (B)
d
d
a a
V V

U c U c

b b

d d
(C) (D)
a a
V V

20. The heat capacity of one mole an mole an ideal is found to be CV = 3R(1 + RT)/2 where a is a constant. The
equation obeyed by this gas during a reversible adiabatic expansion is. [KVPY_2016_SB] [2 Mark]
(A) TV3/2eaRT = constant (B) TV3/2e3aRT/2 = constant
(C) TV3/2 = constant (D) TV3/2e2aRT/3 = constant

KVPY_ SA STREAM # 335


21. A long cylindrical pipe of radius 20 cm is closed at its upper end and has an
airtight piston of negligible mass as shown. When a 50 Kg mass is attached
to the other end of the piston, it moves down. If the air in the enclosure is
cooled from temperature T to T – T, the piston moves back to its original L
position. Then T/T is close to (Assuming air to be an ideal gas, g = 10 m/s2,

atmospheric pressure is 105 Pascal),[KVPY_2017_SA] [1 Mark]
(A) 0.01 (B) 0.02

(C) 0.04 (D) 0.09

22. Two different liquids of same mass are kept in two identical ves-
sel, which are placed in a freezer that extracts heat from them at
the same rate causing each liquid to transform into a solid. The
schematic figure below shows the temperaure T vs time t plot for
the two materials. We denote the specific heat in the liquid states
to be CL1 and CL2 for materials 1 and 2 respectively, and latent
heats of fusion U1 and U2 respectively.[KVPY_2017_SA] [2 Mark]
Choose the correct option.
(A) CL1 > CL2 and U1 < U2 (B) CL1 > CL2 and U1 > U2
(C) CL1 < CL2 and U1 > U2 (D) CL1 < CL2 and U1 < U2
23. A thin piece of thermal conductor of constant thermal conductivity insulated on the lateral sides connects
two reservoirs which are maintained at temperatures T1 and T2 as shown. Assuming tha the system is in
steady state, which of the following plots best represents the dependence of the rate of change of entropy on
the ratio of temperatures T1/T2 [KVPY_2017_SB] [1 Mark]
co nductor

T1 T2
dS/dt

dS/dt

dS/dt

dS/dt

0 0 0 0
1 T1/T2 1 T1/T2 1 T1/T2 1 T1/T2
(A) (B*) (C) (D)

24. Two bottles A and B have radii RA and RB and heights hA and hB respectively with RB = 2RA and hB = 2hA. These
are filled with hot water at 60ºC. Consider that heat loss for the bottles takes place only from side surfaces.
If the time the water takes to cool down to 50ºC is tA and tB for bottles A and B, respectively, then tA and tB are
best related as, [KVPY_2017_SB] [1 Mark]
(A) tA = tB (B) tB = 2tA (C) tB = 4tA (D) tB = tA/2
25. The number of gas molecules striking per second per square meter of the top surface of a table placed in a
room at 20ºC and 1 atmospheric pressure is of the order of (kB = 1.4 × 10–23 J/K, and the average mass of an
air molecule is 5 × 10–27 kg) [KVPY_2017_SB] [1 Mark]
(A) 1027
(B) 1023
(C) 1025
(D) 1029
26. One mole of an ideal monatomic gas undergoes the following four reversible processes :
Step 1 : It is first compressed adiabatically from volume V1 to 1 m2. [KVPY_2017_SB] [2 Mark]
Step 2 : then expanded isothermally to volume 10 m3
Step 3 : then expanded adiabatically to volume V3
Step 4 : then compressed isothermally to volume V1.
If the efficiency of the above cycle is ¾ then V1 is
(A) 2 m3 (B) 4 m3 (C) 6 m3 (D) 8 m3

KVPY_ SA STREAM # 336


27. A gas obeying the equation of state PV = RT undergoes a hypothetical reversible process described by the

 PV 
equation, PV exp     C1 where C1 and E0 are dimensioned constants. Then, for this process, the
5/ 3

 E0 
thermal compressibility at high temperature [KVPY_2017_SB] [2 Mark]
(A) approaches a constant value (B) is proportional to T
(C) is proportional to T1/2 (D) is proportional to T2

EXERCISE - 1
1. (C) 2. (D) 3. (B) 4. (A) 5. (B) 6. (A) 7. (A)
8. (A) 9. (A) 10. (C) 11. (D) 12. (B) 13. (B) 14. (D)
15. (D) 16.* (C) 17. (D) 18. (B) 19. (D) 20. (D) 21. (A)
22. (C) 23. (C) 24. (D) 25. (A) 26. (A) 27. (C) 28. (D)
29. (A) 30. (C) 31. (B) 32. (D) 33. (C) 34. (B) 35. (A)

EXERCISE - 2
1. (A) 2. (A) 3. (C) 4. (B) 5. (C) 6. (B) 7.
8. (B) 9. (B) 10. (A) 11. (B) 12. (D) 13. (C) 14. (A)
15. (B) 16. (A) 17. (A) 18. (C) 19. (A) 20. (A) 21. (C)
22. (C) 23. (B) 24. (B) 25. (A) 26. (D) 27. (A)

1 3
1. mV 2  kT
2 2
1 2 2
  kT
2 2

V 3

 2m

f
2. In an ideas gas internal energy = nRT
2
5 3
U= × 2RT + 4 × RT = 11 RT.
T.
2 2

3. V = KT + C
nRT nRT
P=  P=
V KT  C

dP nRC

dT (KT  C)2
As C < 0 by diagram
dP
 < 0 for all T  P continuously decreases.
dT

KVPY_ SA STREAM # 337


12  0 2  2 2  3 2
5. Vr.m.s. = = 3.5 .
4
PM
6. We have =
RT
P1M P2M P1 P2
= 
RT1 RT2 T1 2T1
P2 = 2P1


7. TKE = U = nRT
2
In case of H2 degree of freedom is greatest and numebr of moles n is highest
So this is the case of maximum kinetic energy.

8. n = 7v – v 2 – 10
dn
for most probable velocity 0  n is maximum at this velocity
dv
7
7–2v = 0  v = = 3.5 m/sec
2

9. Stages 1 and 2 are at same temperature also stages 4 and 5 are at same temperature.
As, VP is more at higher temperature and same at all stages at equal temperature.
 VP3 > VP1 = VP2 > VP4 = VP5
Hence (A).
M
10. If M0 is molecular mass of the gas then for initial condition PV = . RT ...(1)
M0
After 2M mass has been added
V 3M T
P . = M .R. ...(2)
3 0 3
By dividing (2) by (1)
P = 3P

11. As the container is insulated Q = 0


The expansion of gas is in vacuum W = 0
 From first law
Q = W + U
 U = 0

12. [Moderate]
PV = nRT
Since both the rooms have same volume and are connected, so they will have same pressure
PV = nRT = constant
Constan t fu ; r
0 = nRT = constant  nT = = constant
R R
If T is more n has to be less
Hence, lower the temperature, more the number of molecules.
13. PV5/3 = constant
(PV) V2/3 = constant
14. PV = nRT
kVT 3 V = nRT

W=  PdV = nR (T – T ) = – 250 R
i f

KVPY_ SA STREAM # 338


15. Before heating let the pressure of gas be P1 from the equilibrium piston,
PA = kx1
PA  nRT  A 1 8.3  100  10 2
 x1 = =   = = 10–1 = 0.1 m
K  V K 0.83  100
Since during heating process,
The spring is compressed further by 0.1 m
 x2 = 0.2 m
1 1
work done by gas = .100(0.22 – 0.12) = .100.(0.1) (0.3)
2 2
= 1.50 = 1.5 J
 30  10   30  10 
16.* W   × 103 ×   × 10–3 =  × 102
 2   2 

17. Heat absorbed by gas in three processes is given by


Q ACB = U + W ACB
Q ADB = U
Q AEB = U + W AEB
The change in internal energy in all the three cases is same. And W ACB is +ve, VAEB is –ve.
Hence Q ACB > Q ADB > Q AEB

18.

19. [Moderate]
BC is isochoric. VB > VA , VB = VC , VD > VC
1 1
20. W= P V = RT 0 .
2 0 0 2
13
Heat absorbed = Q AB + Q BC = CVT 0 + CP2T 0 = RT 0
2
1
P0 V0
2  13 13 
 Efficiency = 13 × 100  P0 V0  RT0 
P0 V0  2 2 
2
1
= × 100 = 7.7 %
13

21. (A) (C)

22. For isothermal process


v2
Q = nRT n
v1
1800 = 1 × 8.3 T n 2
get T = 312.6 K

KVPY_ SA STREAM # 339


23. There is no heat transfer in adiabatic compression. In isothermal process
V2
Q=W = P1V1 ln
1 V1
= 400 x 12 ln = –6653 J
4
24. PV = nRT
V nR
= = constant.
T P

MN2 14
25.  MN2  14m & MHe  m
MHe 1

n1Cp1  n 2Cp 2
Cp =
n1  n 2

14m 7 m 5
 R  R 7 5 5
28 2 4 2 2 R  R 7R  R
= 14m m = 2 2 = 2  19R
 21 3 6
28 4

26. Let A & B are two points on an isothermal curve. Join A & B by any curve C. Then on this curve c,
U = 0, where c is not an isothermal curve.

27. Q = nCP(T 2 – T 1)
R 
= n (T – T1) = (v – v 1)p
 1 2  1 2

5/3
= × 2.5 × 103 × (3 – 1) = 12500 J.
(5 / 3)  1

28. Q = U + W
W = 0, Q = 0
U = 0
T = 0

29.  PdV =  nC dt v

 dQ= 2dU
 nCdT = 2nCv dT  C = 2Cv
2R
 C= = 4R.
1.5  1

Cp 2 4 2
30. = 1  = 1  f = 6.
Cv f 3 f

31. The two are in series.


KA = 3KB
i.e. thermal resistance of A (say RA) is one third that of B (say RB)
i.e. RB =3 RA
36 9
 Thermal current = 4 R  R
A A
 Temperature difference across 'A'
 9 
=   .R = 9°CAns.
 RA  A

KVPY_ SA STREAM # 340


dT
32. Heat current i = – k A
dx
idx = – kA dT
 T2


i dx = – A 
0
 T dT
T1

(T22  T12 )
 i  = – A 
2

A  (T12  T22 )
 i=
2

R
33. Initially effective resistance = 2R. In parallel effective resistance = . It has reduced by a factor of 1/
2
4 so rate of heat transfer would be increased by a factor of 4, keeping other parameters same.

34. Rate of radiation per unit area is proportional to (T 4)

 P  AT 4

 P  r2.
dT
Also ms  AT
T4
dt
dT 1
 =R
dt r
(because m = (v)  r3 and A  r2)

35. Let the body have temperatures T 1 and T 2 respectively at wavelength 1 = 8000Å and 2 = 4000Å.
 From Wien’s displacement law
 T = constant
 1T1 = 2T2

or 8000 × T 1 = 4000T 2

T1 1
or =
T2 2

Emissive power = e AT 4

 Ratio of emissive powers at these temperature is

e1T14
=
e 2 T2 4

4
0.5  1
= ×  
0.8  2

5
=
128

KVPY_ SA STREAM # 341


CALORI M ET RY AN D T H ERM AL EX PAN SI ON

1. HEAT
The energy that is being transferred between two bodies or between adjacent parts of a body as a
result of temperature difference is called heat. Thus, heat is a form of energy. It is energy in transit
whenever temperature differences exist. Once it is transferred, it becomes the internal energy of the
receiving body. It should be clearly understood that the word "heat" is meaningful only as long as the
energy is being transferred. Thus, expressions like "heat in a body" or "heat of a body" are meaningless.
T1 > T2 T2

A Heat B

When we say that a body is heated it means that its molecules begin to move with greater kinetic
energy.
S.. unit of heat energy is joule (J). Another common unit of heat energy is calorie (cal).
1 calorie = 4.18 joules.
1 calorie : The amount of heat needed to increase the temperature of 1 gm of water from 14.5 to 15.5
ºC at one atmospheric pressure is 1 calorie.

1.1 Mechanical Equivalent of Heat


In early days heat was not recognized as a form of energy. Heat was supposed to be something
needed to raise the temperature of a body or to change its phase. Calorie was defined as the
unit of heat. A number of experiments were performed to show that the temperature may also
be increased by doing mechanical work on the system. These experiments established that
heat is equivalent to mechanical energy and measured how much mechanical energy is
equivalent to a calorie. If mechanical work W produces the same temperature change as heat
H, we write,
W = JH
where J is called mechanical equivalent of heat. J is expressed in joule/calorie. The value
of J gives how many joules of mechanical work is needed to raise the temperature of 1 g of
water by 1°C.

Example 1. What is the change in potential energy (in calories) of a 10 kg mass after 10 m fall ?
Solution : Change in potential energy
U = mgh = 10 × 10 × 10
1000
= 1000 J = cal Ans.
4.186


2. SPECIFIC HEAT
Specific heat of substance is equal to heat gain or released by that substance to raise or fall its temperature
by 1ºC for a unit mass of substance.
When a body is heated, it gains heat. On the other hand, heat is lost when the body is cooled. The gain
or loss of heat is directly proportional to:
(a) the mass of the body Q  m
(b) rise or fall of temperature of the body Q   T
Q  m  T or Q = m s  T
or dQ = m s d T or Q=m  s d T..

KVPY_ SA STREAM # 342


Q
where s is a constant and is known as the specific heat of the body s = . S.. unit of s is joule/
mT
kg-kelvin and C.G.S. unit is cal./gm °C.

Specific heat of water : S = 4200 J/kgºC = 1000 cal/kgºC = 1 Kcal/kgºC = 1 cal/gmºC

Specific heat of steam = half of specific heat of water = specific heat of ice

Example 2. Heat required to increases the temperate of 1 kg water by 20ºC


Solution : heat required = Q = ms
 S = 1 cal/gmºC = 1 Kcal/kgºC
= 1 × 20 = 20 Kcal.


2.1 Heat capacity or Thermal capacity :
Heat capacity of a body is defined as the amount of heat required to raise the temperature of
t hat body by 1°. If 'm ' is t he m ass and 's' t he speci f i c heat of t he body, t hen
Heat capacity = m s.
Units of heat capacity in: CGS system is, cal ºC1; SI unit is, JK1

2.2 Important Points:


Q
(a) We know, s = , if the substance undergoes the change of state which occurs at
mT
constant temperature (T = 0) , then s = Q/0 = . Thus the specific heat of a substance
when it melts or boils at constant temperature is infinite.
(b) If the temperature of the substance changes without the transfer of heat (Q = 0) then
Q
s = = 0. Thus when liquid in the thermos flask is shaken, its temperature
mT
increases without the transfer of heat and hence the specific heat of liquid in the thermos
flask is zero.
(c) To raise the temperature of saturated water vapours, heat (Q) is withdrawn. Hence,
specific heat of saturated water vapours is negative. (This is for your information only
and not in the course)
(d) The slight variation of specific heat of water
with temperature is shown in the graph at 1
atmosphere pressure. Its variation is less than
1% over the interval form 0 to 100ºC.

2.3 Relation between Specific heat and Water equivalent:


It is the amount of water which requires the same amount of heat for the same temperature rise as
that of the object
ms
ms T = mW SW T  mW =
sW
In calorie sW = 1
 mW = ms
mw is also represent by W
so W = ms.

KVPY_ SA STREAM # 343


2.4 Phase change:
Heat required for the change of phase or state,
Q = mL , L = latent heat.
Latent heat (L): The heat supplied to a substance which changes its state at constant
temperature is called latent heat of the body.
Latent heat of Fusion (L f ): The heat supplied to a substance which changes it from solid to
liquid state at its melting point and 1 atm. pressure is called latent heat of fusion. Latent heat
of fusion of ice is 80 kcal/kg
Latent heat of vaporization (L v ): The heat supplied to a substance which changes it from
liquid to vapour state at its boiling point and 1 atm. pressure is called latent heat of vaporization.
Latent heat of vaporization of water is 540 kcal kg1.
Latent heat of ice : L = 80 cal/gm = 80 Kcal/kg = 4200 × 80 J/kg
Latent heat of steam : L = 540 cal/gm = 540 Kcal/kg = 4200 × 540 J/kg
The given figure, represents the change of state
by different lines
OA  solid state , AB  solid + liquid state (Phase change)
BC  liquid state , CD  liquid + vapour state (Phase change)
DE  vapour state
Q = msT
T 1 T 1
slope =  
Q ms Q S
where mass (m) of substance constant slope of T – Q graph is inversely proportional to specific
heat, if in given diagram
(slope) OA > (slope) DE
then (s)OA < (s)DE
when Q = mL
If (length of AB) > (length of CD)
then (latent heat of AB) > (latent heat of CD)

Example 3. Find the amount of heat released if 1 kg steam at 200ºC is converted into –20ºC ice.
Solution : Heat released Q = heat release to convert steam at 200 ºC into 100ºC steam + heat release to
convert 100º C steam into 100ºC water + heat release to convert 100º water into 0ºC water + heat
release to convert 0 ºC water into – 20ºC ice.
1 1
Q = 1 × × 100 + 540 × 1 + 1 × 1 × 100 + 1 × 80 + 1 × × 20
2 2
= 780 Kcal.


3. CALORIMETRY
The branch of thermodynamics which deals with the measurement of heat is called calorimetry.
A simple calorimeter is a vessel generally made of copper with a stirrer of the same material. The
vessel is kept in a wooden box to isolate it thermally from the surrounding. A thermometer is used to
measure the temperature of the contents of the calorimeter. Object at different temperatures are made
to come in contact with each other in the calorimeter. As a result, heat is exchanged between the
object as well as with the calorimeter. Neglecting any heat exchange with the surrounding.
3.1 Law of Mixture:
When two substances at different temperatures are mixed together, then exchange of heat
continues to take place till their temperatures become equal. This temperature is then called
final temperature of mixture. Here, Heat taken by one substance = Heat given by another
substance

KVPY_ SA STREAM # 344


 m 1 s1 (T 1  T m ) = m 2 s2 (T m  T 2)

Example 4. An iron block of mass 2 kg, fall from a height 10 m. After colliding with the ground it loses 25%
energy to surroundings. Then find the temperature rise of the block. (Take sp. heat of iron 470 J/kg
ºC)
1 10  10
Solution : mS = mgh  =
4 4  470


Zeroth law of thermodynamics :
If objects A and B are separately in thermal equilibrium with a third object C , then objects A and B are in
thermal equilibrium with each other.

Example 5. The temperature of equal masses of three different liquids A, B, and C are 10ºC 15ºC and 20ºC
respectively. The temperature when A and B are mixed is 13ºC and when B and C are mixed, it is
16ºC. What will be the temperature when A and C are mixed?

Solution :

when A and B are mixed


mS1 × (13 – 10) = m × S2 × (15 – 13)
3S1 = 2S2 .....(1)
when B and C are mixed
S2 × 1 = S3 × 4 ......(2)
when C and A are mixed
S1( – 10) = S3 × (20 – ) ....(3)
by using equation (1), (2) and (3)
140
we get  = ºC
11
Example 6. If three different liquid of different masses specific heats and temperature are mixed with each other
and then what is the temperature mixture at thermal equilibrium..
m1, s1, T1  specification for liquid
m2, s2, T2  specification for liquid
m3, s3, T3  specification for liquid.
Solution : Total heat lost or gain by all substance is equal to zero
Q = 0
m1s1(T – T1) + m2s2(T – T2) + m3s3 (T – T3) = 0
m1s1T1  m 2 s 2 T2  m 3 s3 T3
so T=
m1s1  m 2 s 2  m 3 s 3

KVPY_ SA STREAM # 345


Example 7. In following equation calculate value of H 1 kg ice at –20ºC = H + 1 Kg water at 100ºC, here H means
heat required to change the state of substance.
Solution : Heat required to convert 1 kg ice at – 20ºC into 1 kg water at 100ºC
= 1 kg ice at – 20ºC to 1 kg ice at 0ºC ice at 0ºC + 1 kg water
at 0ºC + 1 kg water at 0ºC to 1 kg water at 100ºC
1
=1× × 20 + 1 × 80 + 1 × 100 = 190 Kcal. So H = – 190 Kcal
2
Negative sign indicate that 190 Kcal heat is with drawn from 1 kg water at 100ºC to convert it into
1 kg ice at – 20ºC

Example 8. 1 kg ice at –20ºC is mixed with 1 kg steam at 200ºC. Then find equilibrium temperature and mixture
content.
Solution : Let equilibrium temperature is 100 ºC heat required to convert 1 kg ice at –20ºC to 1 kg water at
100ºC is equal to
1
H1 = 1 × × 20 + 1 × 80 + 1 × 1 × 100 = 190 Kcal
2
heat release by steam to convert 1 kg steam at 200ºC to 1 kg water at 100ºC is equal to
1
H2 = 1 × × 100 + 1 × 540 = 590 Kcal
2
1 kg ice at – 20ºC = H1 + 1kg water at 100ºC ......(1)
1 kg steam at 200ºC = H2 + 1kg water at 100ºC .......(2)
by adding equation (1) and (2)
1 kg ice at –20ºC + 1 kg steam at 200ºC = H1 + H2 + 2 kg water at 100ºC.
Here heat required to ice is less than heat supplied by steam so mixture equilibrium temperature is
100ºC then steam is not completely converted into water.
So mixture has water and steam which is possible only at 100ºC
mass of steam which converted into water is equal to
1
190  1  100 7
m= 2 = kg
540 27
so mixture content
7 20 7 34
mass of steam = 1 – = kg mass of water = 1 + = kg
27 27 27 27


4. THERMAL EXPANSION
Most materials expand when their temperature is increased. Rails roads tracks, bridges all have some
means of compensating for thermal expansion. W hen a homogeneous object expands, the distance
between any two points on the object increases. Figure shows a block of metal with a hole in it. The
expanded object is like a photographic enlargement. That in the hole expands in the same
proportion as the metal, it does not get smaller

Potential
energy

r0 r1
r2
r
E2
E1

Thermal expansion arises because the well is


not symmetrical about the equilibrium position r0.
As the temperature rise the energy of the atom increases.
The average position when the energy is E2 is not
the same as that when the energy is E1.

KVPY_ SA STREAM # 346


At the atomic level, thermal expansion may be understood by considering how the potential energy of
the atoms varies with distance. The equilibrium position of an atom will be at the minimum of the
potential energy well if the well is symmetric. At a given temperature each atom vibrates about its
equilibrium position and its average remains at the minimum point. If the shape of the well is not
symmetrical the average position of an atom will not be at the minimum point. When the temperature
is raised the amplitude of the vibrations increases and the average position is located at a greater inter
atomic separation. This increased separation is manifested as expansion of the material.
Almost all solids and liquids expand as their temperature increases. Gases also expand if allowed.
Solids can change in length, area or volume, while liquids change in their volumes.

Example 9. A rectangular plate has a circular cavity as shown in the figure. If


we increase its temperature then which dimension will increase in
following figure.

Solution : Distance between any two point on an object increases with


increase in temperature.
So, all dimension a, b, c and d will increase

Example 10. In the given figure, when temperature is increased then which of the following increases

(A) R1 (B) R2 (C) R2 – R1


Solution : All of the above
- - - - - represents expanded Boundary
--------- represents original Boundary
As the intermolecular distance between atoms increases on heating hence
the inner and outer perimeter increases. Also if the atomic arrangement in
radial direction is observed then we can say that it also increases hence all
A,B,C are true.

5. LINEAR EXPANSION
When the rod is heated, its increase in length L is proportional to its original length L0 and change in
temperature T where T is in ºC or K.

dL = L0 dT  L =  L0  T If  T << 1
L
= where  is called the coefficient of linear expansion whose unit is ºC 1 or K1.
L0  T
L = L0 (1 +  T). Where L is the length after heating the rod.

Variation of a with temperature and distance


(a) If  varies with distance,  = ax + b.

Then total expansion =  (ax + b) T dx.

(b) If  varies with temperature,  = f (T). Then L =  L0 dT

KVPY_ SA STREAM # 347


Note : Actually thermal expansion is always 3-D expansion. When other two dimensions of object are
negligible with respect to one, then observations are significant only in one dimension and it is
known as linear expansion.

Example 11. What is the percentage change in length of 1m iron rod if its temperature changes by 100ºC.  for
iron is 2 × 10–5/ºC.
Solution : percentage change in length due to temperature change

% = × 100 =  × 100

= 2 × 10–5 × 100 × 100
= 0.2% Ans.


5.1 Thermal stress of a material:
If the rod is free to expand then there will be no stress and strain. Stress and strain is produced
only when an object is restricted to expand or contract according to change in temperature.
When the temperature of the rod is decreased or increased under constrained condition ,
compressive or tensile stresses are developed in the rod. These stresses are known as thermal
stresses.

L final length – original length


Strain = =
original length
= T ,
L0

Note : Original and final length should be at same temperature.

//////////////////////////////
//////////////////////////////
Consider a rod of length 0 which is fixed between to rigid end
separated at a distance 0 now if the temperature of the rod is F F
increased by  then the strain produced in the rod will be :
0

length of the rod at new temperature - natural length of the rod at new temperature
strain 
natural length of the rod at new temperature

 0   0 (1  )  0 
= =
 0 (1  )  0 (1  )
  is very small so
strain = –  (negative sign in the answer represents that the length of the rod is less than the
natural length that means is compressed by the ends.)

Example 12. In the given figure a rod is free at one end and other end is fixed. When we change the temperature
of rod by , then strain produced in the rod will be

1
(A)  (B) (C) zero (D) information incomplete
2
Solution : Here rod is free to expand from one side by so by changing temperature no strain will be produced
in the rod.
Hence ans. is (C)

KVPY_ SA STREAM # 348


Example 13. An iron ring measuring 15.00 cm in diameter is to be shrunk on a pulley which is 15.05 cm in
diameter. All measurements refer to the room temperature 20°C. To what minimum temperature
should the ring be heated to make the job possible? Calculate the strain developed in the ring when
it comes to the room temperature. Coefficient of linear expansion of iron = 12 × 10–6/°C.
Solution : The ring should be heated to increase its diameter from 15.00 cm to 15.05 cm.
Using 2 = 1 (1 +  ),
0.05 cm
= = 278°C
15.00 cm  12  10 6 /º C
The temperature = 20°C + 278°C = 298°C.
 2  1
The strain developed = = 3.33 × 10–3 .
1

Example 14. A steel rod of length 1m rests on a smooth horizontal base. If it is heated from 0°C to 100°C, what is
the longitudinal strain developed?
Solution : in absence of external force no strain or stress will be created hear rod is free to move.

Example 15. A steel rod is clamped at its two ends and rests on a fixed horizontal base. The rod is in natural
length at 20°C. Find the longitudinal strain developed in the rod if the temperature rises to 50°C.
Coefficient of linear expansion of steel = 1.2 × 10–5/°C.
Solution : as we known that strain
change in length 
strain = =
original length 0
 Strain = 
= 1.2 × 10–5 × (50 – 20) = 3.6 × 10–4
here strain is compressive strain because final length is smaller than initial length.

Example 16. A steel wire of cross-sectional area 0.5 mm2 is held between two fixed supports. If the wire is just
taut at 20°C, determine the tension when the temperature falls to 0°C. Coefficient of linear expansion
of steel is 1.2 × 10–5/°C and its Young’s modulus is 2.0 × 1011 N/m2.
Solution : here final length is more than original length so that strain is tensile and tensile force is given by
F = AY  t = 0.5 × 10–6 × 2 × 1011 × 1.2 × 10–5 × 20 = 24 N


5.2 Variation of time period of pendulum clocks:
The time represented by the clock hands of a pendulum clock depends on the number of oscillation
performed by pendulum every time it reaches to its extreme position the second hand of the clock
advances by one second that means second hand moves by two seconds when one oscillation in
complete

L0 L
Let T = 2  at temperature 0 and T = 2  at temperature .
g g

T L L 1     1
= = = 1+ 
T L L 2
Therefore change (loss or gain) in time per unit time lapsed is
T´T 1
= 
T 2
gain or loss in time in duration of 't' in
1
t =  t , if T is the correct time then
2
(a)  < 0 , T < T clock becomes fast and gain time
(b)  > 0 , T > T clock becomes slow and loose time

KVPY_ SA STREAM # 349


Example 17. A pendulum clock consists of an iron rod connected to a small, heavy bob. If it is designed to keep
correct time at 20°C, how fast or slow will it go in 24 hours at 40°C? Coefficient of linear expansion of
iron = 1.2 × 10–6 /°C.
Solution : The time difference occurred in 24 hours (86400 seconds) is given by
1
t =   t
2
1
= × 1.2 × 10–6 × 20 × 86400 = 1.04 sec. Ans.
2
This is loss of time as  is greater than 0 . As the temperature increases, the time period also
increases. Thus, the clock goes slow.


5.3 Measurement of length by metallic scale:
Case (i)
When object is expanded only
2 = 1 {1 + 0(2 – 1)
1 = actual length of object at 1ºC = measure length of object at 1ºC.
2 = actual length of object at 2ºC = measure length of object at 2ºC.
0 = linear expansion coefficient of object.
1
1

2
2

0 2 3

Case (ii)
When only measurement instrument is expanded actual length of object will not change but
measured value (MV) decreases.
MV = 1 { 1 – S (2 – 1)} 1
S = linear expansion coefficient of measuring instrument.
at 1 C MV = 3
at 2 C MV = 2.2 1ºC
0 1 2 3 4

Case (iii) 2ºC 0 2


1 3
If both expanded simultaneously
MV = {1 + (0 – s) (2 – 1)
(i) If 0 > s, then measured value is more then the actual value at 1ºC
(ii) If 0 < s, then measured value is less then the actual value at 1ºC
1
1ºC

2
2ºC

1ºC
0 1 2 3 4 5

2ºC
0 1 2 3 4

KVPY_ SA STREAM # 350


at 1ºC MV = 3.4
2ºC MV = 4.1
Measured value = calibrated value × {1 + }
where  = 0 – s
o = coefficient of linear expansion of object material, s = coefficient of linear expansion of scale
material
= C
 = temperature at the time of measurement C = temperature at the time of calibration.
For scale, true measurement = scale reading [1 +  ( 0)]
If  >  0 true measurement > scale reading
 <  0 true measurement < scale reading

Example 18. A bar measured with a Vernier caliper is found to be 180mm long. The temperature during the
measurement is 10ºC. The measurement error will be if the scale of the Vernier caliper has
been graduated at a temperature of 20ºC : ( = 1.1 × 10-5 °C-1. Assume that the length of the
bar does not change.)
(A) 1.98 × 10–1 mm (B*) 1.98 × 10–2 mm (C) 1.98 × 10–3 mm (D) 1.98 × 10–4 mm
Solution : True measurement = scale reading [1 +  (0)]
= 180 [1– 10 × 1.1 × 10–5 ]
error = 180 – 180 [1 – 1.1 × 10–4] = 1.98 × 10–2 mm


6. SUPERFICIAL OR AREAL EXPANSION
When a solid is heated and its area increases, then the thermal expansion is called superficial or areal
expansion. Consider a solid plate of area A 0. When it is heated, the change in area of the plate is
directly proportional to the original area A 0 and the change in temperature T.
dA = A0 dT or A =  A0  T
L0
A size of object
= Unit of  is ºC1 or K1. L0
DL
L at 1 < 2
A0  T
DL size of object
A = A0 (1 +  T) at 2
where A is area of the plate after heating,

Example 19. A plane lamina has area 2m2 at 10ºC then what is its area at 110ºC t’s superficial expansion is
2 × 10–5//C
Solution : A = A0 ( 1 + ) = 2 { 1 + 2 × 105 × (110 – 10) }
= 2 × {1 + 2 × 10–3} Ans.


7. VOLUME OR CUBICAL EXPANSION
When a solid is heated and its volume increases, then the expansion is called volume expansion or
cubical expansion. Let us consider a solid or liquid whose original volume is V 0. When it is heated to
a new volume, then the change V
dV = V0 dT or V =  V0  T
V
= Unit of  is ºC1 or K1.
V0  T
V = V0 (1 +   T) where V is the volume of the body after heating

KVPY_ SA STREAM # 351


Example 20. The volume of glass vessel is 1000 cc at 20°C. What volume of mercury should be poured into
it at this temperature so that the volume of the remaining space does not change with tempera-
ture? Coefficient of cubical expansion of mercury and glass are 1.8 × 10–4/°C and 9.0 × 10–6/°C
respectively.
Solution : Let volume of glass vessel at 20ºC is Vg and volume of mercury at 20ºC is Vm
so volume of remaining space is = Vg – Vm
It is given constant so that
Vg – Vm = Vg’ – V’m
where Vo' and Vm' are final volumes.
Vg – Vm = Vg {1 + g } – Vm{1 + Hg }  Vg g = Vm Hg

100  9  10 6
 Vm =  Vm = 50 cc.
1.8  10 4


8. RELATION BETWEEN  ,  AND 
  
(i) For isotropic solids:  :  :  = 1 : 2 : 3 or = =
1 2 3
(ii) For non-isotropic solid  = 1 + 2 and  = 1 + 2 + 3 . Here 1 , 2 and 3 are coefficient of linear
expansion in X, Y and Z direction.

Example 21. If percentage change in length is 1% with change in temperature of a cuboid object (  × 2 × 3)
then what ispercentage change in its area and volume.
Solution : percentage change in length with change in temperature = % 

×100 = × 100 = 1

change in area
A
% A = × 100 = × 100 2 ( × 100)
A
%A=2% Ans.
change in volume
V
%V= × 100 = V × 100 = 3 (× 100)
V
%V=3% Ans.


9. VARIATION OF DENSITY WITH TEMPERATURE
As we known that mass = volume × density .
Mass of substance does not change with change in temperature so with increase of temperature, volume
increases so density decreases and vice-versa.
d0
d= .
(1  T )
For solids values of  are generally small so we can write d = d0 (1 T) (using binomial expansion).

KVPY_ SA STREAM # 352


Note : (i)  for liquids are in order of 103 .
(ii) Anamolous expansion of water :
For water density increases from 0 ºC to 4 ºC so  is negative
and for 4 ºC to higher temperature  is positive. At 4 ºC density
is maximum. This anamolous behaviour of water is due to
presence of three types of molecules i.e. H 2O, (H2O)2 and
(H2O)3 having different volume/mass at different temperatures.

. This anomalous behaviour of water causes ice to form first at the surface of a lake in cold weather.
As winter approaches , the water temperature decreases initially at the surface. The water there
sinks because of its increase density. Consequently , the surface reaches 00C first and the lake
becomes covered with ice. Aquatic life is able to survive the cold winter as the lake bottom remains
unfrozen at a temperature of about 40C.

Example 22. The densities of wood and benzene at 0°C are 880 kg/m3 and 900 kg/m3 respectively. The coeffi-
cients of volume expansion are 1.2 × 10–3/°C for wood and 1.5 × 10–3/°C for benzene. At what
temperature will a piece of wood just sink in benzene?
Solution : At just sink gravitation force = upthrust force
 mg = FB  V1g = V2g  1 = 2
880 900
 =   = 83º C
1  1.2  10 3  1  1.5  10 3 


10. APPARENT EXPANSION OF A LIQUID IN A CONTAINER
Initially container was full . When temperature change by T,
volume of liquid VL = V0 (1 + L  T)
volume of container VC = V0 (1 + C  T)
So overflow volume of liquid relative to container
V = VL  VC V = V0 ( L   C) T
So, coefficient of apparent expansion of liquid w.r.t.
container
 apparent =  L  C .
In case of expansion of liquid + container system:
if L > C  level of liquid rise
if L < C  level of liquid fall
Increase in height of liquid level in tube when bulb
was initially not completely filled

volume of liquid V0 (1   L T )
h= = A (1  2 T ) = h0 { 1 + ( L – 2S) T}
area of tube 0 S

h = h 0 { 1 + (  L – 2 S) T}
where h0 = original height of liquid in container
S = linear coefficient of expansion of container.

Example 23. A glass vessel of volume 100 cm3 is filled with mercury and is heated from 25°C to 75°C. What
volume of mercury will overflow? Coefficient of linear expansion of glass = 1.8 × 10–6/°C and coeffi-
cient of volume expansion of mercury is 1.8 × 10–4/°C.
Solution : V = V0(L – C) T = 100 × {1.8 × 10–4 – 3 × 1.8 × 10–6 } × 50
V = 0.87 cm3 Ans.

KVPY_ SA STREAM # 353



11. VARIATION OF FORCE OF BUOYANCY WITH TEMPERATURE
If body is submerged completely inside the liquid
For solid, Buoyancy force F B = V 0 dL g
V0 = Volume of the solid inside liquid,
dL = density of liquid
Volume of body after increase its temperature V = V0 [1 + S ] ,
dL
Density of body after increase its temperature dL =
1   L   .
FB 1   S  
Buoyancy force of body after increase its temperature, FB = V dL g , = ,
FB 1   L  
if S < L then FB < F B
(Buoyant force decreases) or apparent weight of body in liquid gets increased
[W  FB > W  F B] .

Example 24. A body is float inside liquid if we increases temperature then what
changes occur in Buoyancy force. (Assume body is always in
floating condition)
Solution : Body is in equilibrium
so mg = B
and gravitational force does not change with change in temperature. So Buoyancy force remains
constant.
By increasing temperature density of liquid decreases so volume of body inside the liquid increases
to kept the Buoyance force constant for equal to gravitational force)
Example 25. In previous question discuss the case when body move downward, upwards and remains at same
position when we increases temperature.
Solution : Let f = fraction of volume of body submerged in liquid.
volume of body submerged in liquid
f=
total volume of body
v1
f1 = v at 1ºC
0
v2
f 2 = v (1  3 ) at 2ºC
0 S
for equilibrium mg = B = v 1d1g = v 2d2g.
v1d1 d1
so v 2 =  d2 = = v 1(1 + L )
d2 1   L 

v1(1   L )
 f2 =
v 0 (1  3 s )
where  = 2 – 1
Case I : Body move downward if f2 > f 1
means L > 3S
Case II : Body move upwards if f2 < f 1
means L < 3S
Case III : Body remains at same position
if f 2 = f 1
means L = 3S

KVPY_ SA STREAM # 354



12. BIMETALLIC STRIP
It two strip of different metals are welded together to form a bimetallic strip, when heated uniformly it bends
in form of an arc, the metal with greater coefficient of linear expansion lies on convex side. The radius of arc
thus formed by bimetal is :
a2
 d 0
0 (1 +1) =  R   t
 2 d

 d d
0 (1 +2) =  R  
 2 a2 > a1
a1
Lower temperature (at 1ºC)
d
1   2  R On 250 C
2
 1    = d
1 R
2

d
 R  (   ) Bimetallic strrip
2 1
R
  = change in temperature
= 2– 1
Higher temperature (at 2ºC) 0
(Off) 30 C

A bimetallic strip, consisting of a strip of brass and a strip of steel welded together, at temperature T0 in figure
(a) and figure (b). The strip bends as shown at temperatures above the reference temperature. Below the
reference temperature the strip bends the other way. Many thermostats operate on this principle, making
and breaking an electrical circuit as the temperature rises and falls.

13. APPLICATIONS OF THERMAL EXPANSION


(a) A small gap is left between two iron rails of the railway.
(b) Iron rings are slipped on the wooden wheels by heating the iron rings
(c) Stopper of a glass bottle jammed in its neck can be taken out by heating the neck.
(d) The pendulum of a clock is made of invar [an alloy of zinc and copper].

14. TEMPERATURE
Temperature may be defined as the degree of hotness or coldness of a body. Heat energy flows from
a body at higher temperature to that at lower temperature until their temperatures become equal. At
this stage, the bodies are said to be in thermal equilibrium.

14.1 Measurement of Temperature


The branch of thermodynamics which deals with the measurement of temperature is called
thermometry. A thermometer is a device used to measure the temperature of a body. The substances
like liquids and gases which are used in the thermometer are called thermometric substances.

14.2 Different Scales of Temperature


A thermometer can be graduated into following scales.
(a) The Centigrade or Celsius scale (ºC)
(b) The Fahrenheit scale (ºF)
(c) The Reaumer scale (ºR)
(d) Kelvin scale of temperature (K)

KVPY_ SA STREAM # 355


14.3 Comparison between Different Temperature Scales
K C F
Water boils 373.15 100 212
body temp. 310.2 37.0 98.6
Room temp. 300 27 80.6
Triple point of water 273.16 0.01
Water freezes 273.15 0 32

Solid CO2 195 - 78 - 109

Hydrogen boils 20.7 -252.5 - 422.5


Absolute zero 0 -273.15 - 489.67
The formula for the conversion between different temperature scales is:
K  273 C F  32 R
= = =
100 100 180 80
General formula for the conversion of temperature from one scale to another:
Temp on one scale(S1 ) - Lower fixed point (S1 )
Upper fixed point (S 2 )  Lower fixed point (S1 )

Temp. on other scale(S 2 ) - Lower fixed point (S 2 )


=
Upper fixed point (S 2 )  Lower fixed point (S2 )
14.4 Thermometers
Thermometers are device that are used to measure temperatures. All thermometers are based on
the principle that some physical property of a system changes as the system temperature changes.
Required properties of good thermometric substance.
(1) Non-sticky (absence of adhesive force)
(2) Low melting point (in comparison with room temperature)
(3) High boiling temperature
(4) Coefficient of volumetric expansion should be high (to increase accuracy in measurement).
(5) Heat capacity should be low.
(6) Conductivity should be high
Mercury (Hg) suitably exhibits above properties.
14.5 Types of Thermometers (not for JEE)
Type of
Thermometric
thermometer and Advantages Disadvantages Particular Uses
property
its range
(i) Every laboratory use where high accuracy
Length of column of (i) Quick and easy to (i) Fragile is not required.
Mercury-in-glass
mercury in capillary (direct reading) (ii) Small size limits (ii) Can be calibrated against constant-
– 39ºC to 450ºC
tube (ii) Easily portable (iii) Limited range volume gas thermometer for more
accurate work
(i) Very accurate (i) Very large volume of bulb (i) Standard against which others calibrated
Constant-volume Pressure of a fixed
(ii) Very sensitive (ii) Slow to use and (ii) He, H2 or N2 used depending on range
gas thermometer – mass of gas at
(iii) Wide range inconvenient (iii) can be corrected to the ideal gas scale
270º to 1500ºC constant volume
(iv) Easily reproducible (iv) Used as standard below-183ºC
(i) Best thermometer for small steady
Platinum Not suitable for varying
Electrical resistance (i) Accurate temperature differences
resistance –180º temperature (i.e., is slow to
of a platinum coil (ii) Wide range (ii) Used as standard between 183ºC and
to 1150ºC respond to changes)
630ºC.
(i) Fast response
Emf produced Accuracy is lost if emf is (i) Best thermometer for small steady
because of low heat
between junctions of measured using a moving- temperature differences
capacity.
Thermocouple dissimilar metals at coil voltmeter (as may be (ii) Can be made direct reading by calibrating
(ii) wide range
–250ºC to 1150ºC different necessary for rapid changes galvanometer
(iii) can be employed for
temperatures for when potentiometer is (iii) Used as standard between 630ºC and
remote readings using
measurement of emfs unsuitable) 1063ºC
long leads.
Radiation Does not come into (i) Cumbersome (i) Only thermometer possible for very high
Colour of radiation
pyrometer above contact when (ii) Not direct reading (needs temperatures
emitted by a hot body
1000ºC temperature is measured a trained observer) (ii) Used as standard above 1063ºC.

Example 26.
The readings of a thermometer at 0ºC and 100ºC are 50 cm and 75 cm of mercury column respectively. Find
the temperature at which its reading is 80 cm of mercury column?
80  50 T 0
Solution : By using formula =  T = 120ºC
75  50 100  0

KVPY_ SA STREAM # 356


1. If specific heat capacity of a substance in solid and liquid state is proportional to temperature of the substance,
and now if heat is supplied to the solid initially at – 20°C (having melting point 0°C) at constant rate. Then the
temperature dependence of solid with time will be best represented by :

(A) (B) (C) (D)

2. An ice block at 0°C is dropped from height ‘h’ above the ground. What should be the value of ‘h’ so that
it just melts completely by the time it reaches the bottom assuming the loss of whole gravitational
potential energy is used as heat by the ice ? [Given : L f = 80 cal/gm]
(A) 33.6 m (B) 33.6 km (C) 8 m (D) 8 km

3. A small ball of mass m is dropped from an aeroplane moving at 50 m height above the ground with a speed
of 25 2 meter/sec. If half of the mechanical energy of ball with respect to ground is lost as a thermal energy
due to air friction. The change in the temperature of the ball as it lands on the ground is. Specific heat
capacity of ball is 56.25 J/kg. (g = 10 m/sec2)
(A) 5°C (B) 10°C (C) 20°C (D) 500/56.25°C

4. Four cubes of ice at – 10°C each one gm is taken out from the refrigerator and are put in 150 gm of water at
20°C. The temperature of water when thermal equilibrium is attained. Assume that no heat is lost to the
outside and water equivalent of container is 46 gm. (Specific heat capacity of water = 1 cal/gm-°C, Specific
heat capacity of ice = 0.5 cal/gm-°C, Latent heat of fusion of ice = 80 cal/gm)
(A) 0°C (B) – 10°C (C) 17.9°C (D) None

5. A current of 2.50 A passing through a heating coil immersed in 180g of paraffin (specific heat capacity 2.00
J g–1 K–1) contained in a 100 g calorimeter (specific heat capacity 0.400J g–1 K–1) raises the temperature from
5ºC below room temperature to 5ºC above room temperature in 100 s. The reading of the voltmeter connected
across the heating coil is
(A) 8.0 V (B) 16.0 V (C) 24.0 V (D) 32.0 V

6. Two solid bodies of equal mass m initially at T = 0°C are heated at a uniform and same rate under
identical conditions.The temperature of the first object with latent heat L 1 and specific heat capacity in
solid state C1 changes according to graph 1 on the diagram. The temperature of the second object with
latent heat L2 and specific heat capacity in solid state C2 changes according to graph 2 on the diagram.
Based on what is shown on the graph, the latent heats L 1 and L2, and the specific heat capacities C 1
and C2 in solid state obey which of the following relationships :

(A) L1 > L2 ; C1 < C2 (B) L1 < L2 ; C1 < C2 (C) L1 > L2 ; C1 > C2 (D) L1 < L2 ; C1 > C2

KVPY_ SA STREAM # 357


7. Water of mass m 2 = 1 kg is contained in a copper calorimeter of mass m 1 = 1 kg. Their common
temperature t = 10°C. Now a piece of ice of mass m 3 = 2 kg and temperature is –11°C dropped into the
calorimeter. Neglecting any heat loss, the final temperature of system is. [specific heat of copper = 0.1
Kcal/ kg°C, specific heat of water = 1 Kcal/kg°C, specific heat of ice = 0.5 Kcal/kg°C, latent heat of
fusion of ice = 78.7 Kcal/kg]
(A) 0°C (B) 4°C (C) – 4°C (D) – 2°C

8. An ice block at 0°C and of mass m is dropped from height 'h' such that the loss in gravitational potential
energy of block is exactly equal to the heat required to just completely melt the ice. Taking latent heat
of fusion of ice = 80 cal/gm, acceleration due to gravity = 10 m/s2 and mechanical equivalent of heat =
4.2 J/ Cal. The value of 'h' is
(A) 8 m (B) 8 km (C) 33.6 m (D) 33.6 km

9. Heat required to vaporize 4g of water by boiling at 373 K is 2160 calories. The specific heat of water in this
condition is :
(A) 0.36 cal/g-K (B) 5.4 cal/g-K (C) zero ' kw
U; (D) infinity

10. The specific heat of many solids at low temperatures varies with absolute temperature T according to
the graph as shown. Then heat energy required to raise the temperature of a unit mass of such a solid
from T = 0 to T = 30 K :

(A) 300 3 J (B) 900 3 J (C) 450 3 J (D) 150 3 J

11. Two ice blocks each of mass M = 6.3 kg are moving towards each other with 20 m/sec. Intial temperature of
the each ice block is 0ºC. The head on collision between them is perfectly inelastic. Assume that heat
generated due to collision is completely used for melting some quantity of ice. Latent heat of ice for fusion is
80 cal/gm and 1 cal = 4.2 Joule. Total mass of water formed due to melting is

(A) 2.5 gm (B) 5 gm (C) 7.5 gm (D) 10 gm

12. A milk chilling device can remove heat from the milk at the rate of 41.87 × 106 J / hr. Milk also absorbs energy
from surrounding at the rate of 4.187 × 106 J / hr. Assuming both the given rate to be uniform, it is observed
that it takes 'X' minutes for cooling of 500 kg milk from 50° to 5°, then 'X' is . (here specific heat at constant

kJ
pressure CP of milk is 4.187 )
kg K

(A) 75 (B) 100 (C) 150 (D) 200

13. An electric kettle has coils A and B, when coil A is switched on, 2 kg water boils in 1 minute, and when coil
B is switched on the 2kg water boils in 2 minute. The time (in minutes) taken by 4kg water to boil if the coils
are connected in series.
(A) 3 (B) 6 (C) 9 (D) 12
14. Ice at 0ºC is added to 200 gm of water initially at 70ºC in a vaccum flask. When 50 gm of ice has been added
and has all melted the temperature of the flask and its contents is 40ºC. When a further 80 gm of ice has
been added and has all melted, the temperature of the whole becomes 10ºC. The latent heat of fusion of ice
in cal/gm is-
(A) 80 (B) 90 (C) 100 (D) 150

KVPY_ SA STREAM # 358


15. The amount of heat supplied to decrease the volume of an ice water mixture by 1 cm 3 without any
change in temperature, is equal to : (ice = 0.9, water = 80 cal/gm)
(A) 360 cal (B) 500 cal (C) 720 cal (D) None
16. A small quantity of mass m of water at a temperature (inºC) is poured on to a large mass M of ice which
is at its melting point. f c is the specific heat capacity of water and L the latent heat of fusion of ice, then the
mass of ice melted is given by :
mc Mc mc
(A) ML (B) (C) (D)
mc ML L L
17. Two large holes are cut in a metal sheet. If this is heated, distances AB and BC, (as shown)

(A) both will increase (B) both will decrease


(C) AB increases, BC decreases (D) AB decreases, BC increases
18. Expansion during heating –
(A) occurs only in a solid (B) increases the density of the material
(C) decreases the density of the material (D) occurs at the same rate for all liquids and solids.
19. A difference of temperature of 25º C is equivalent to a difference of :
(A) 45º F (B) 72º F (C) 32º F (D) 25º F
20. A one litre glass flask contains some mercury. It is found that at different temperatures the volume of air
inside the flask remains the same. What is the volume of mercury in this flask if coefficient of linear
expansion of glass is 9 × 10–6/°C while of volume expansion of mercury is 1.8 × 10–4/°C :
(A) 50 cc (B) 100 cc (C) 150 cc (D) 200 cc
21. A rod of length 20 cm is made of metal. It expands by 0.075 cm when its temperature is raised from 0°C to
100°C. another rod of a different metal B having the same length expands by 0.045 cm for the same change
in temperature. A third rod of the same length is composed of two parts, one of metal A and the other of
metal B. This rod expands by 0.060 cm for the same change in temperature. The portion made of metal A
has the length :
(A) 20 cm (B) 10 cm (C) 15 cm (D) 18 cm
22. Steam is passed into 22 gm of water at 20°C. The mass of water that will be present when the water
acquires a temperature of 90°C (Latent heat of steam is 540 cal/gm) is :
(A) 24.8 gm (B) 24 gm (C) 36.6 gm (D) 30 gm
23. 80 gm of water at 30°C are poured on a large block of ice at 0°C. The mass of ice that melts is :
(A) 30 gm (B) 80 gm (C) 1600 gm (D) 150 gm
24. Compared to a burn due to water at 100°C, a burn due to steam at 100°C is :
(A) More dangerous (B) Less dangerous (C) Equally dangerous (D) None of these
25. The thermal capacity of 40 gm of aluminium (specific heat = 0.2 cal/gm/°C) is :
(A) 40 cal/°C (B) 160 cal/°C (C) 200 cal/°C (D) 8 cal/°C
26. A water fall is 84 metres high. If half of the potential energy of the falling water gets converted to heat, the
rise in temperature of water will be :
(A) 0.098°C (B) 0.98°C (C) 9.8°C (D) 0.0098°C
27. When the pressure on water is increased the boiling temperature of water as compared to 100°C will be :
(A) Lower (B) The same (C) Higher (D) On the critical temperature
28. How many grams of a liquid of specific heat 0.2 at a temperature 40°C must be mixed with 100 gm of a
liquid of specific heat of 0.5 at a temperature 20°C, so that the final temperature of the mixture becomes
32°C :
(A) 175 gm (B) 300 gm (C) 295 gm (D) 375 gm
29. 5g of ice at 0°C is dropped in a beaker containing 20 g of water at 40°C. the final temperature will be :
(A) 32°C (B) 16°C (C) 8°C (D) 24°C
30. 50 gm of ice at 0°C is mixed with 50 gm of water at 80°C, final temperature of mixture will be :
(A) 0°C (B) 40°C (C) 40°C (D) 4°C

KVPY_ SA STREAM # 359


31. 300 gm of water at 25°C is added to 100 gm of ice at 0°C. the final temperature of the mixture is :
5 5
(A)  C (B)  C (C) –5°C (D) 0°C
3 2
32. Three liquids with masses m1, m2, m3 are thoroughly mixed. If their specific heats are c1, c2, c3 and their
temperature T1, T2, T3 respectively, then the temperature of the mixture is :
c1T1  c 2 T2  c 3 T3 m1c1T1  m 2c 2 T2  m 3c 3 T3
(A) (B)
m1c1  m 2c 2  m 3 c 3 m1c 1  m 2 c 2  m 3 c 3

m1c1T1  m 2c 2 T2  m 3c 3 T3 m1T1  m 2 T2  m 3 T3
(C) m1T1  m 2T2  m3 T3
(D) c T  c T  c T
1 1 2 2 3 3

33. A glass flask is filled up to a mark with 50 cc of mercury at 18°C. If the flask and contents are heated to
38°C, how much mercury will be above the mark ? ( for glass is 9 × 10–6/°C and coefficient of real expansion
of mercury is 180 × 10–6/°C
(A) 0.85 cc (B) 0.46 cc (C) 0.153 cc (D) 0.05 cc
34. A glass flask of volume one litre at 0°C is filled, level full of mercury at this temperature. The flask and
mercury are now heated to 100°C. How much mercury will spill out, if coefficient of volume expansion of
mercury is 1.82 × 10–4 °C and linear expansion of glass is 0.1 × 10–4/°C respectively :
(A) 21.2 cc (B) 15.2 cc (C) 1.52 cc (D) 2.12 cc
35. A piece of metal weight 46 gm in air, when it is immersed in the liquid of specific gravity 1.24 at 27°C it
weighs 30 gm. When the temperature of liquid is raised to 42°C the metal piece weight 30.5 gm, specific
gravity of the liquid at 42°C is 1.20, then the linear expansion of the metal will be :
(A) 3.316 × 10–5 /°C (B) 2.316 × 10–5 /°C (C) 4.316 × 10–5 /°C (D) None of these

KVPY PROBLEMS (PREVIOUS YEARS)


1. An Insulated vessel contains 0.4 kg of water at 0ºC. A piece of 0.1 kg ice at –15°C is put into it and steam
at 100ºC is bubbled into it until all ice is melted and finally the contents are liquid water at 40ºC. Assume that
the vessel does not give or take any heat and there is no loss of matter and heat to the surroundings. Specific
heat of ice is 2.2 × 103 J kg-1K-1, heat of fusion of water is 333 × 103 J kg–1 and heat of vaporization of water
is 2260 × 103 J kg–1. The amount of steam that was bubbled into the water is about : [KVPY_2008]
(A) 34.7 gram (B) 236.0 gram (C) 0.023 gram (D) 48.0 gram
2. A thermometer uses ‘density of water’ as thermometric property. The actual
reading in the thermometer is ‘height of water’(h) which is inversely proportional
to density of water (d). In a certain temperature range, density of water varies
with temperature as shown. The graph is symmetric about the maximum
Two identical bodies (of same mass and specific heat) at different temperatures
T1 and T2 show the same reading of height h1 = h2 in the thermometer. The
bodies are brought into contact and allowed to reach thermal equilibrium. The
thermometer reading ‘height of water for final equilibrium state hf satisfies. [KVPY_2008]
h1  h2
(A) hf = = h1  h2 (B) hf > h1 = h2
2
(C) hf < h1 = h2
(D) hf may be greater or less than h1 = h2 , depending on the specific heat of the bodies.

3. We are able to squeeze snow and make balls out of it because of - [KVPY_2009]
(A) anomalous behaviour of water. (B) large latent heat of ice.
(C) large specific heat of water. (D) low melting point of ice.
4. The temperature of a metal coin is increased by 100°C and its diameter increases by 0.15%. Its area
increases by nearly [KVPY_2009]
(A) 0.15% (B) 0.60% (C) 0.30% (D) 0.0225%

KVPY_ SA STREAM # 360


5. Two identical blocks of metal are at 20ºC and 80ºC, respectively. The specific heat of the material of the two
blocks increases with temperature. Which of the following is true about the final temperature Tƒ when the two
blocks are brought into contact (assuming that no heat is lost to the surroundings) ? [KVPY-2010]
(A) Tƒ will be 50ºC.
(B) Tƒ will be more than 50ºC.
(C) Tƒ will be less than 50ºC.
(D) Tƒ can be either more than or less than 50ºC depending on the precise variation of the specific heat with
temperature.
6. A new temperature scale uses X as a unit of temperature, where the numerical value of the temperature tX in
this scale is related to the absolute temperature T by tX = 3T + 300. If the specific heat of a material using this
unit is 1400 J kg–1 X–1 its specific heat in the S.I. system of units is : [KVPY-2010]
(A) 4200 J kg K –1 –1
(B) 1400 J kg K
–1 –1
(C) 466.7 J kg K
–1 –1

(D) impossible to determine from the information provided


7. The following three objects (1) a metal tray (2) a block of wood and (3) a wooden cap are left in a closed room
overnight. Next day the temperature of each is recorded as T1 , T2 and T3 respectively. The likely situation is
[KVPY_2011]
(A) T1 = T2 = T3 (B) T3 > T2 > T1 (C) T3 = T2 > T1 (D) T3 > T2 = T1
8. 150 g of ice is mixed with 100 g of water at temperature 80ºC. The latent heat of ice is 80 cal/g and the
specific heat of water is 1 cal/g-ºC. Assuming no heat loss to the environment, the amount of ice which does
not melt is : [KVPY_2012]
(A) 100 g (B) 0 g (C) 150 g (D) 50 g
9. An aluminum piece of mass 50g initially at 300 °C is dipped quickly and taken out of 1kg of water, initially
at 30 °C. If the teperature of the aluminum piece be 160 °C, what is the temperature of the water then
(Specific heat capacities of aluminum and water are 900 JKf-1K-1 and 4200Jkg-1K-1, respectively)
[KVPY_2014_SA]
(A) 165°C (B) 45 °C (C) 31.5 °C (D) 28.5 °C
10. A thin paper cup filled with water does not catch fire when placed over a flame. This is because [KVPY_2014_SA]
(A) The water cuts off oxygen supply to the paper cup
(B) Water is an excellent conductor of heat
(C) The paper cup does not become appreciably hotter than the water it contain
(D) Paper is a poor conductor of heat
11. Ice is used in a cooler in order to cool its contents. Which of the following will speed up the cooling process
[KVPY_2014_SA]
(A) Wrap the ice in a metal foil (B) Drain the water from the cooler periodically
(C) Put the ice as a single block (D) Crush the ice
12. A solid expands upon heating because : [KVPY_2014_SB]
(A) the potential energy of interaction between atoms in the solid is asymmetric about the equilbrium positions
of atoms
(B) the frequency of vibration of the atoms increases
(C) the heating generates a thermal gradient between opposite sides
(D) a fluid called the caloric flows into the interatomic spacing of the solid during heating thereby expanding
it.
13. Consider two thermometers T1 and T2 of equal length which can be used to measure temperature over the
range 1 to 2. T1 contains mercury as thermometric liquid while T2 contains bromine. The volumes of the two
liquids are the same at the temperature 1.The volumetric coefficients of expansion of mercury and bromine
are 18 × 10-5 K-1 and 108 × 10-5 K-1, respectively. The increase in length of each liquid is the same for the
same increase in temperature. If the diameters of the capillary tubes of the two thermometers are d1 and d2
respectively, then the ratio d1:d2 would be closest to : [KVPY_2014_SB]
(A) 6.0 (B) 2.5 (C) 0.5 (D) 0.4
14. 1 Kg of ice at –20ºC is mixed with 2 Kg of water at 90ºC . Assuming that there is no loss of energy to
the environment, what will be the final temperature of the mixture? (Assume latent heat of
ice = 334.4 KJ/Kg, specific heat of water and ice are 4.18 kJ/(kg.K) and 2.09kJ/(kg.K), respectively.)
[KVPY_2015_SA] [2 Marks]
(A) 30ºC (B) 0ºC (C) 80ºC (D) 45ºC
15. Thermal expansion of a solid is due to the [KVPY_2016_SB] [1 Marks]
(A) symmetric characteristic of the inter atomic potential energy curve of the solid
(B) asymmetric characteristic of the inter atomic potential energy curve of the solid
(C) double well nature of the inter-atomic potential energy curve of the solid.
(D) rotational motion of the atoms of the solid

KVPY_ SA STREAM # 361


16. Ice in a freezer is at –7 °C. 100 g of this ice is mixed with 200 g of water at 15 °C. Take the freezing
temperature of water to be 0 °C, the specific heat of ice equal to 2.2 J/g °C, specific heat of water equal to 4.2
J/g °C, and the latent heat of ice equal to 335 J/g. Assuming no loss of heat to the environment, the mass of
ice in the final mixture is closest to [KVPY_2017_SB] [1 Marks]
(A) 88 g (B) 67 g (C) 54 g (D) 45g

17. A block of wood is floating on water at 0ºC with volume V0 above water. When the temperature of water
increases from 0 to 10ºC, the change in the volume of the block that is above water is best described
schematically by the graph [KVPY-SA_2018 1 Mark]

V0 V0
(A) (B)

0ºC 10 ºC 0 ºC 10 ºC

V0 V0
(C) (D)

0 ºC 10 ºC 0 ºC 10 ºC

18. An earthen pitcher used in summer cools water in it essentially by evaporation of water from its porous
surface. If a pitcher carries 4 kg water and the rate of evaporation is 20 g per hour, temperature of water in it
decreases by T in two hours. The value of T is close to (ratio of latent heat of evaporation to specific heat
of water is 540) [KVPY-SA_2018 1 Mark]
(A) 2.7 °C (B) 4.2 °C (C) 5.4 °C (D) 10.8 °C

19. A certain liquid has a melting point of – 50° C and a boiling point of 150° C. A thermometer is designed with
this liquid and its melting and boiling points are designated as 0° L and 100° L. The melting and boiling points
of water on this scale are [KVPY-SA_2018 1 Mark]
(A) 25° L and 75° L, respectively, (B) 0° L and 100° L, respectively,
(C) 20° L and 70° L, respectively, (D) 30° L and 80° L, respectively,

20. A coffee maker makes coffee by passing steam through a mixture of coffee powder, milk and water. If the
steam is mixed at the rate of 50 g per minute in a mug containing 500 g of mixture, then it takes about t0
seconds to make coffee at 70°C when the initial temperature of the mixture is 25°C. The value of t0 is close
to (ratio of latent heat of evaporation to specific heat of water is 540°C) and specific heat of the mixture can
taken to be the same as that of water) [KVPY-SA_2018 2 Mark]
(A) 30 (B) 45 (C) 60 (D) 90

KVPY_ SA STREAM # 362


EXERCISE - 1
1. (C) 2. (B) 3. (B) 4. (C) 5. (B) 6. (A) 7. (A)
8. (D) 9. (D) 10. (C) 11. (C) 12. (C) 13. (B) 14. (B)
15. (C) 16. (D) 17. (A) 18. (C) 19. (A) 20. (C) 21. (B)
22. (A) 23. (A) 24. (A) 25. (D) 26. (A) 27. (C) 28. (D)
29. (B) 30. (A) 31. (D) 32. (B) 33. (C) 34. (B) 35. (B)
EXERCISE - 2
1. (D) 2. (C) 3. (A) 4. (C) 5. (B) 6. (A) 7. (A)
8. (D) 9. (C) 10. (C) 11. (D) 12. (B) 13. (D) 14. (A)

15. (B) 16. (B) 17. (A) 18. (C) 19. (A) 20. (B)

1. As dQ = msdT
dQ dT
 ms
dt dt
From question : S  T
or S = K1T. (K1 being propertionality constant)
dQ dT dT
Also, = constant = K2 (say)  ms = K2  m(K1T) = K2
dt dt dt

 K1  T2
  m  =t  T t
 K2  2
2. Applying energy conservation :
mgh = mLf

Lf 80cal / gm 80  4.2  1000 J/kg  336  10 3  N  s2


 h= = 2 = =  10

 kg = 33.6 km.
g 10m / s 10 m/s2  

3.
1  M  g  h  1 MV 2 

2  2
 = MST

 
2  2

1 10  50  1  25 2 2 


 = 56.25 × T

500  625
T = = 10°C.
56.25  2
4. Heat gained by ice = Heat lost by water + Heat lost by container
Initial Temperature of container = 20ºC
1
4× × 10 + 4 × 80 + 4 × 1 x(T–0) = 196 × 1 × (20–T)
2
3580
20 + 320 + 4T = 196 × 20 – 196 T 200 T = 196 × 20 – 340 T = 17.9ºC
200

Heat supplied m s   m2 s 2 
5. Power = = 1 1
Time T

(180g  2Jg 1K 1  10K )  (100g  0.4Jg1K 1  10K ) 40


= = 40 W  VI = 40 W  V= = 16.0 V
100 s 2 .5

KVPY_ SA STREAM # 363


6. If heat is supplied at constant rate P, then Q = Pt and as during change of state Q = mL, so, mL = Pt
P  P
i.e., L =   t = (length of line AB)
m  m
Hence L1 > L2
i.e., the ratio of latent heat of fusion of the two substances are in the ratio 3 : 4.
In the portion OA the substance is in solid state and its temperature is changing.
Q = mCT and Q = Pt
T P P  T 
So,  or slope = = as  slope 
t mC mS  t 
Hence C1 < C2

7. Loss in heat from calorimeter + water as temperture changes from 10°C to 0°C
= m1C110 + m2C210 = 1 × 1 × 10 + 1 × 0.1 × 10 = 11 kcal
Gain in heat of ice as its temperature changes from –11°C to 0°C
= m 3C3 × 11 = 2 × 0.5 × 11 = 11 kcal
Hence ice and water will coexist at 0°C without any phase change.

8. From given statement


mgh = mL or gh = L
80  4.2
or 10 × h = or h = 33.6 × 103 meter = 33.6 km
10 3

9. Q = mST
Since in boiling T = 0, S = 

10. Q=  SdT
0

= 
0
3 T.dT

3 2 3
= T = × 900 = 450 3 J.
2 2

11. By momentum conservation, after collision both blocks come to rest.


1 1
Heat generated (H) = Mu2 + Mu2 = Mu2
2 2
Let m kg mass is melt
H = mL  Mu2 = mL
Mu2
m=
L
80  4.2 J
L = 80 cal/gm = 103 Kg

( 6.3 ) ( 20)2  10 3
m= = 7.5 × 10–3 kg
80  4.2

KVPY_ SA STREAM # 364


12. (41.87 – 4.187) 106 t
= 500 × 4.187 × 103 × (50 – 5)
500  45
 t = hrs = 2.5 hrs = 150 minutes
9000

13. Let thermal energy QA generated when coil A is switched on

V2 V2
QA = tA = 1
RA RA
Let thermal energy QB generated when coil B is switched on

V2 V2
QB = t B = 2
RB RB
since QA = QB
V2 V2
we have 1= 2
RA RB
or 2RA = RB
V2 V2
when A & B are connected in series thermal energy QS = R t S = t
eq (R A  R B )
QS = 2QA = 2QB
V 2t S 2V 2  1
 =
R A  RB RA

tS 2
=
R A  RB RA

using RB = 2RA
tS = 6 min

14. Heat capacity of flask = C


C(70 – 40) + 200 × (70 – 40) × 1
= 50L + 50 × 1 × (40 – 0)
C(40 – 10) + 250(40 – 10) × 1
= 80L + 80 × 1 × (10 – 0)
L = 90 cal/gm.

15. x gm ice convert into x gm water


x 0.9
–x=1 x= =9
0.9 0.1
 Q = 9 × 80 = 720 cal

23. If m gm ice melts then


Heat lost = heat gain
80 × 1 × (30 – 0) = m × 80 = 30 gm

24. Steam at 100°C contains extra 540 calorie/gm energy as compare to water at 100°C. So it's more dangerous
to burn with steam then water.

25. Thermal capacity = mc = 40 × 0.2 = 8 cal/°C

26. as W = JQ 

KVPY_ SA STREAM # 365


E L E C T R O S T AT I C S


1. I N TROD UCTI ON
The branch of physics which deals with electric effect of static charge is called electrostatics.

2. ELECTRI C CHARGE
Charge of a material body or particle is the property (acquired or natural) due to which it produces and
experiences electrical and magnetic effects. Some of naturally occurring charged particles are electrons,
protons, -particles etc.
Charge is a derived physical quantity & is measured in Coulomb in S.. unit. In practice we use mC
(10–3C), C (10–6C), nC(10–9C) etc.
C.G.S. unit of charge = electrostatic unit = esu.
1 coulomb = 3 × 109 esu of charge
Dimensional formula of charge = [MºLºT 11]

2 .1 Prope rt ie s of Cha rge


(i) Cha rge is a sca la r qua ntity : It adds algebraically and represents excess or deficiency
of electrons.
( ii) Cha rge is of two types : (i) Positive charge and (ii) Negative charge Charging a
body implies transfer of charge (electrons) from one body to another. Positively charged body
means loss of electrons i.e. deficiency of electrons. Negatively charged body means excess of
electrons. This also shows that mass of a negatively charged body > mass of a positively
charged identical body.
(iii) Cha rge is conserved : In an isolated system, total charge (sum of positive and negative)
remains constant whatever change takes place in that system.

( i v) Cha rge is qua nt ized : Charge on any body always exists in integral multiples of a
fundamental unit of electric charge. This unit is equal to the magnitude of charge on electron
(1e = 1.6 × 10–19 coulomb). So charge on anybody is Q = ± ne, where n is an integer and e is
the charge of the electron. Millikan's oil drop experiment proved the quantization of charge or
atomicity of charge
1 2
Note : Recently, the existence of particles of charge ± e and ± e has been postulated. These particles are
3 3
called quarks but still this is not considered as the quantum of charge because these are unstable
(They have very short span of life).
(v) Like point charges repel each other while unlike point charges attract each other.
(vi) Charge is always associated with mass, i.e., charge can not exist without mass though mass
can exist without charge. The particle such as photon or neutrino which have no (rest) mass
can never have a charge.
(vii) Charge is relativistically invariant: This means that charge is independent of frame of reference
i.e. charge on a body does not change whatever be its speed. This property is worth mentioning
as in contrast to charge, the mass of a body depends on its speed and increases with increase
in speed.
(viii) A charge at rest produces only electric field around itself, a charge having uniform motion
produces electric as well as magnetic field around itself while a charge having accelerated
motion emits electromagnetic radiations.

KVPY_ SA STREAM # 366



3. COULOM B’S LAW (I NVERSE SQUARE LAW )
On the basis of experiments Coulomb established the following law known as Coulomb's law :
The magnitude of electrostatic force between two point charges is directly proportional to the product
of charges and inversely proportional to the square of the distance between them.
1 q1q2 Kq1q2
i.e. F  q1q2 and F  F  F=
r2 r2 r2
Important points regarding Coulomb's law :
(i) It is applicable only for point charges.

1
(ii) The constant of proportionality K in SI units in vacuum is expressed as and in any other
4 0
1
medium expressed as . If charges are dipped in a medium then electrostatic force on one
4

1 q1q2
charge is ; where  0 and  are called permittivity of vacuum and absolute
4 0 r r2

permittivity of the medium respectively. The ratio  /  0 =  r is called relative permittivity of the
medium, which is a dimensionless quantity.
(iii) The value of relative permittivity r is constant for a medium and can have values between 1 to
. For vacuum, by definition it is equal to 1. For air it is nearly equal to 1 and may be taken to
be equal to 1 for calculations. For metals, the value of r is  and for water is 81. The material
in which more charge can induce r will be higher.

1
(iv) The value of 4 = 9 × 109 Nm 2 C–2 & 0 = 8.855 × 10–12 C2/Nm 2.
0

Dimensional formula of  is [M–1 L–3 T 4 A2 ]


(v) The force acting on one point charge due to the other point charge is always along the line
joining these two charges. It is equal in magnitude and opposite in direction on two charges,
irrespective of the medium in which they lie.
(vi) The force is conservative in nature i.e., work done by electrostatic force in moving a point
charge along a closed loop of any shape is zero.
(vii) Since the force is a central force, in the absence of any other external force, angular momentum
of one particle w.r.t. the other particle (in two particle system) is conserved.
(viii) In vector form formula can be given as below.

 1 q1q2  1 q1q2
F = 4  | r |3 r = 4  | r |2 r̂ ; (q1 & q2 are to be substituted with sign.)
0 r 0 r

Here, r is position vector of the test charge (on which force is to be calculated) with respect to
the source charge (due to which force is to be calculated).

Example 1. Find out the electrostatic force between two point charges placed in air (each of +1 C) if they
are separated by 1m .

kq1q2 9  10 9  1 1
Sol. Fe = = = 9×109 N
r2 12

KVPY_ SA STREAM # 367


From the above result, we can say that 1 C charge is too large to realize. In nature, charge is

usually of the order of C


Example 2.
A particle of mass m carrying charge q 1 is revolving around a fixed charge –q 2 in a circular path of
radius r. Calculate the period of revolution and its speed also.

1 q1q2 42mr
Sol. = mr2
= '
4 0 r 2 T2

( 4 0 )r 2 ( 42mr )  0mr


T2 = or T = 4r q1q2
q1q2
and also we can say that

q1q 2 mv 2 q1q2
2 =  V= 4 0mr
4  0r r

Example 3.
A point charge qA = + 100 µc is placed at point A (1, 0, 2) m and another point charge q B = +200µc is
placed at point B (4, 4, 2) m. Find :
(i) Magnitude of electrostatic interaction force acting between them
 
(ii) Find FA (force on A due to B) and FB (force on B due to A) in vector form

Sol. (i)

9 6 6
kq A qB ( 9  10 ) (100  10 ) ( 200  10 )
Value of F : F  = 
2 = 7.2 N
r 2
 ( 4  1)  ( 4  0 )  ( 2  2 ) 
2 2 2
 

 kq A qB 
(9  10 9 ) (100  10 6 ) (200  10 6 )
3

( 4  1) î  ( 4  0) ĵ  (2  2) k̂ 
(ii) Force on B, FB =  3 r =  ( 4  1)2  ( 4  0)2  (2  2)2 
|r|  
 

3 4 
= 7.2  î  ĵ  N
5 5 

  3 4 
Similarly FA = 7.2   î  ĵ  N
 5 5 

 
Action( FA ) and Reaction ( FB ) are equal but in opposite direction.

KVPY_ SA STREAM # 368



4. P RI N CI P LE OF SUP ERP OSI TI ON
The electrostatic force is a two body interaction i.e. electrical force

between two point charges is independent of presence or absence of other charges and so the principle
of superposition is valid i.e. force on charged particle due to number of point charges is the resultant of
forces due to individual point charges. Therefore, force on a point test charge due to many charges is
   
given by F  F1  F2  F3  ........... .

Example 4
Three equal point charges of charge +q each are moving along
a circle of radius R and a point charge –2q is also placed at
the centre of circle (as shown in figure). If charges are revolving
with constant and same speed in the circle then calculate speed
of charges

Sol.

mv 2
F 2 – 2F 1 cos 30º =
R

K ( q) ( 2q) 2(Kq2 ) mv 2
 – cos 30 =
R2 ( 3R)2 R

kq2  1 
 v 2  
Rm  3

Example 5.
Two equally charged identical small metallic spheres A and B repel each other with a force 2 × 10 –5N
when placed in air (neglect gravitational attraction). Another identical uncharged sphere C is touched to
B and then placed at the mid point of line joining A and B. What is the net electrostatic force on C?
Sol. Let, initially the charge on each sphere be q and separation between their centres be r. Then according
to given problem:

1 qq
F= = 2 × 10–5 N
4 0 r 2

When sphere C touches B, the charge of B i.e. q will distribute equally on B and C as sphere are
identical how charges on spheres;
qB = qC = (q/2)
So sphere C will experience a force

KVPY_ SA STREAM # 369


1 q(q / 2)
F CA = 4 2 = 2F along AB due to charge on A.
0 (r / 2)

1 (q / 2)(q / 2)
and, F CB = 4 = F, along BA due to charge on B :
0 (r / 2)2
So the net force F C on C due to charges on A and B,
F C = F CA – F CB = 2F – F = 2 × 10–5 N along AB .

Example 6.
Five point charges, each of value q are placed on five vertices of a regular hexagon of side L. What is
the magnitude of the force on a point charge of value – q coulomb placed at the centre of the hexagon?
Sol. Method : I
If there had been a sixth charge +q at the remaining vertex of hexagon, force due to all the six charges
on –q at O would have been zero (as the forces due to individual charges will balance each other), i.e.,

FR  0
 
Now if f is the force due to sixth charge and F due to remaining five charges.
    L
From F + f = 0 i.e. F =–f
E D
1 qq 1 q2 q
q
or, |F| = |f| = =
4 0 L2 4 0 L2
q O
  1 q2 F q C
FNet = FOD = 4  2 along OD -q
 L

q
Method : II A B

In the diagram, we can see that force due to L


charge A and D are opposite to each other E D
  q
q
FoF + FOC = 0 ....(i)
 
Similarly FOB + FOE = 0 ....(ii) q O
      F q C
So FoF + FOB + FOC + FOD + FOE = FNet -q

  1 q2 q
Using (i) and (ii) FNet = FOD = along OD.
4   L2 A B


5. ELECTROSTATI C EQUI LI BRI UM
The point where the resultant force on a charged particle becomes zero is called equilibrium position.
5 .1 Sta ble Equilibrium : A charge is initially in equilibrium position and is displaced by a small
distance. If the charge tries to return back to the same equilibrium position then this equilibrium is
called position of stable equilibrium.
5 .2 Unsta ble Equilibrium : If charge is displaced by a small distance from its equilibrium position
and the charge has no tendency to return to the same equilibrium position. Instead it goes away from
the equilibrium position.
5 .3 Neutra l Equilibrium : If charge is displaced by a small distance and it is still in equilibrium
condition then it is called neutral equilibrium.

KVPY_ SA STREAM # 370


Example 7.
Two equal positive point charges 'Q' are fixed at points B(a, 0) and A(–a, 0). Another test charge q 0 is
also placed at O(0, 0). Show that the equilibrium at 'O' is
(i) Stable for displacement along X-axis.
(ii) Unstable for displacement along Y-axis.

Q q0 Q
FBO FAO
O B x
Sol. (i) A
(– a, 0) (a, 0)

    KQq0
Initially FAO + FBO = 0  | FAO | = | FBO | =
a2

When charge is slightly shifted towards + x axis


by a small distance x, then.
 
| FAO | < | FBO |

Therefore, the particle will move towards origin (its original position). Hence, the equilibrium is stable.
(ii) When charge is shifted along y axis:

After resolving components, net force will be along y axis So, the particle will not return to its
original position & it is unstable equilibrium. Finally, the charge will move to infinity.
Example 8.
Two point charges of charge q1 and q2 (both of same sign) and each of mass m are placed such that
gravitational attraction between them balances the electrostatic repulsion. Are they in stable equilibrium?
If not then what is the nature of equilibrium?
Sol. In given example :

K q1 q 2 Gm2
2 =
r r2
We can see that irrespective of distance between them charges will remain in equilibrium. If now
distance is increased or decreased then there is no effect in their equilibrium. Therefore it is a neutral
equilibrium.

Example 9. KVPY_ SA STREAM # 371


A particle of mass m and charge q is located midway between two fixed charged particles each having
a charge q and a distance 2 apart. Prove that the motion of the particle will be SHM if it is displaced
slightly along the line connecting them and released. Also find its time period.
Sol. Let the charge q at the mid-point is displaced slightly to the left.
The force on the displaced charge q due to charge q at A,

1 q2
F1 =
4 0 (  x)2
The force on the displaced charge q due to charge at B,
1 q2
F2 =
4 0 (  x )2
Net restoring force on the displaced charge q.

1 q2 1 q2
F = F 2 – F 1 or F = –
4 0 (  x )2 4 0 (  x)2

q2  1 1  q2 4x
or F=  2
 2  =
4 0  (  x ) (  x)  4 0 (  x 2 )2
2

q2 x q2 x
Since  >> x,  F = or F =
 0  4  0  3
Hence we see that F  x and it is opposite to the direction of displacement. Therefore, the motion is
SHM.

m q2 m  0  3
T = 2 , (here k = )  T = 2
k  0  3 q2

Example 10. Q
Find out mass of the charge Q, so that it remains
in equilibrium for the given configuration. q h q

Sol.  4 Fcos = mg 

q
q
KQq 4KQqh 
 4× 3/2 h = mg  m = 3/2
 2   2 
  h2 
 2  g  h2 
 2 
   
Example 11.
Two identical charged spheres are suspended by strings of equal length. Each string makes an angle
 with the vertical. When suspended in a liquid of density  = 0.8 gm/cc, the angle remains the same.
What is the dielectric constant of the liquid? (Density of the material of sphere is  = 1.6 gm/cc.)
Sol. Initially as the forces acting on each ball are tension T,
weight mg and electric force F, for its equilibrium along vertical
T cos  = mg ...(1)
and along horizontal
T sin  = F ...(2)
Dividing Eqn. (2) by (1), we have
F
tan  = ... (3)
mg

KVPY_ SA STREAM # 372


When the balls are suspended in a liquid of density and dielectric constant K, the electric force will
become (1/K) times, i.e., F' = (F/K) while weight
mg' = mg – F B = mg – Vg [as F B = Vg, where  is density of material of sphere]
   m
i.e. mg' = mg 1    as V  
   
So, for equilibrium of ball,
F' F
tan ' = = ... (4)
mg' Kmg[1  ( / )]
According to given information ' = ; so from equations (4) and (3), we have :
 (1.6)
K= = =2 Ans.
(  ) (1.6  0.8)


6. ELECTRIC FI ELD
Electric field is the region around charged particle or charged body in which if another charge is placed,
it experiences electrostatic force.

6 .1 Electric field intensity E : Electric field intensity at a point is equal to the electrostatic force
experienced by a unit positive point charge both in magnitude and direction.

If a test charge q0 is placed at a point in an electric field and experiences a force F due to some
charges (called source charges), the electric field intensity at that point due to source charges is given
F
by E 
q0

If the E is to be determined practically then the test charge q 0 should be small otherwise it will affect
the charge distribution on the source which is producing the electric field and hence modify the quantity
which is measured.

Example 12.
A positively charged ball hangs from a long silk thread. We wish to measure E at a point P in the same
horizontal plane as that of the hanging charge. To do so, we put a positive test charge q 0 at the point
and measure F/q0. Will F/q0 be less than, equal to, or greater than E at the point in question?
Sol. When we try to measure the electric field at point P then after placing the test
charge at P, it repels the source charge (suspended charge) and the measured
F
value of electric field E measured = q will be less than the actual value E act ,that
0
we wanted to measure.



6 .2 Prope rt ie s of e le ct ric f ie ld int e nsit y E :
(i) It is a vector quantity. Its direction is the same as the force experienced by positive charge.

(ii) Direction of electric field due to positive charge is always away from it while due to negative
charge, always towards it.

(iii) Its S.. unit is Newton/Coulomb.

(iv) Its dimensional formula is [MLT –3A–1]

KVPY_ SA STREAM # 373


(v) Electric force on a charge q placed in a region of electric field at a point where the electric field
  
intensity is E is given by F  qE .
Electric force on point charge is in the same direction of electric field on positive charge and in
opposite direction on a negative charge.

(vi) It obeys the superposition principle, that is, the field intensity at a point due to a system of
charges is vector sum of the field intensities due to individual point charges.
   
i.e. E  E1  E2  E 3 + .....
(vii) It is produced by source charges. The electric field will be a fixed value at a point unless we
change the distribution of source charges.

Example 13.
Electrostatic force experienced by –3C charge placed at point 'P' due

to a system 'S' of fixed point charges as shown in figure is F  (21î  9 ĵ) µN.
(i) Find out electric field intensity at point P due to S.
(ii) If now, 2C charge is placed and –3 C is removed at point P
then force experienced by it will be.
    N
Sol. (i) F  qE  (21î  9 ĵ)µN = -3µC (E)  E = – 7 î – 3 ĵ
C
(ii) Since the source charges are not disturbed the electric field intensity at 'P' will remain same.
 
F 2C = +2( E  = 2(–7 î – 3 ĵ ) = (–14 î – 6 ĵ )  N
Example 14.
Calculate the electric field intensity which would be just sufficient to balance the weight of a particle of
charge –10 c and mass 10 mg. (take g = 10 ms2)

Sol. As force on a charge q in an electric field E is
 
F q = qE Fe
So, according to given problem:

[W = weight of particle] q E
A
|F q | | W | i.e., |q|E = mg

mg W
i.e., E = | q | = 10 N/C., in downward direction.

KVPY_ SA STREAM # 374


List of formula for Electric Field Intensity due to various types of charge distribution :

KVPY_ SA STREAM # 375


Example 15.
Find out electric field intensity at point A (0, 1m, 2m) due to a point charge –20C situated at point
B( 2 m, 0, 1m).
KQ  KQ 
Sol. E=  r =  2 r̂  r = P.V. of A – P.V. of B (P.V. = Position vector)
| r |3 |r|

= (- 2 î + ĵ + k̂ ) |r | = ( 2 )2  (1)2  (1)2 = 2

9  10 9  ( 20  10 6 )
E= (– 2 î + ĵ + k̂ ) = – 22.5 × 103 (– 2 î + ĵ + k̂ ) N/C.
8

Example 16.
Two point charges 2c and – 2c are placed at points A and B
as shown in figure. Find out electric field intensity at points C
and D. [All the distances are measured in meter].

Sol. Electric field at point C


(EA, EB are magnitudes only and arrows represent directions)
Electric field due to positive charge is away from it while due
to negative charge, it is towards the charge. It is clear that E B > EA .
 ENet = (EB – EA) towards negative X-axis
K ( 2c ) K ( 2c )
=  towards negative X-axis = 8000 (– î ) N/C
( 2 )2 (3 2 )2

Electric field at point D :


Since magnitude of charges are same and also AD = BD
So, EA = EB
 
Vertical components of E A and EB cancel each other while horizontal
al
components are in the same direction.
2.K (2c )
So, Enet = 2EA cos = cos450
22

K  10 6 9000
= = î N/C.
2 2

Example 17.
Six equal point charges are placed at the corners of a regular hexagon
of side ‘a’. Calculate electric field intensity at the centre of hexagon?

Ans Zero (By symmetry)

KVPY_ SA STREAM # 376


Similarly electric field due to a uniformly charged ring at the centre of ring :

Note : (i) Net charge on a conductor remains only on the outer surface of a conductor. This property
will be discussed in the article of the conductor. (article no.17)
(ii) On the surface of isolated spherical conductor charge is uniformly distributed.


7. ELECTRI C POTENTIAL :
In electrostatic field, the electric potential (due to some source charges) at a point P is defined as the
work done by external agent in taking a unit point positive charge from a reference point (generally
taken at infinity) to that point P without changing its kinetic energy..
7.1 M a t h e m a t ica l re p re se nt a t ion :
If (W  P)ext is the work required in moving a point charge q from infinity to a point P, the electric
potential of the point P is
Wp )ext  (Welc )p
Vp   
q  K 0 q
Note : (i) (W  P)ext can also be called as the work done by external agent against the electric force on a
unit positive charge due to the source charge.
(ii) Write both W and q with proper sign.

7.2 Prop e rt ie s :
(i) Potential is a scalar quantity, its value may be positive, negative or zero.
joule
(ii) S.. Unit of potential is volt = and its dimensional formula is [M 1L2T –3–1].
coulmb
(iii) Electric potential at a point is also equal to the negative of the work done by the electric field
in taking the point charge from reference point (i.e. infinity) to that point.
(iv) Electric potential due to a positive charge is always positive and due to negative charge it is
always negative except at infinity. (taking V  = 0).
(v) Potential decreases in the direction of electric field.
(vi) V = V1 + V2 + V3 + .......

KVPY_ SA STREAM # 377


Ele ct ric Pot e nt ia l due t o va rious cha rge dist ribut ions a re give n in t a ble .

Name / Type Formula Note Graph

Point charge * q is source charge.


Kq * r is the distance of the point
r from the point charge.

* Q is source chage.
Ring (uniform/nonuniform at centre: KQ * x is the distance of the point on
charge distribution) R the axis from centre f ring
at the axis: KQ

R2  x 2

Uniformly charged hollow kQ


V
con ducting/nonconducting r
/solid conducting sphere
kQ
V
R

Uniforml y c harged so li d kQ
V
nonconducting sphere . r
3
KQ(3R 2  r 2 ) 2
2R 3 4

3
6 0

Infinite line charge Not defined

Infinite nonconducting thin Not defined


sheet


VB  VA   rB  rA 
0

Infinite charged conducting Not defined


thin sheet


VB  VA   rB  rA 
0

KVPY_ SA STREAM # 378



7.4 Pot ent ia l due t o a point cha rge :
Derivation of expression for potential due to point charge Q, at a point which is at a distance r from the
point charge.

Q r P
From definition of potential

r
 
Wext (  p ) 
 ( q 0 E)  d r
 r
r
KQ
r ( dr ) cos 180 º = KQ
V=
qo
 
qo 
=  E  dr

 V=–

2
r

Example 18.
Four point charges are placed at the corners of a square of side  Calculate
potential at the centre of square.

Kq
Sol. V = 0 at 'C'. [Use V = ]
r

Example 19.
Two point charges 2C and – 4C are situated at points (–2m, 0m) and
(2 m, 0 m) respectively. Find out potential at point C (4 m, 0 m) and D
(0 m, 5 m).

Sol. Potential at point C

K (2C) K(4C) 9  10 9  2  10 6 9  10 9  4  10 6
VC = Vq1  Vq2 = + = – = –15000 V..
6 2 6 2

K (2C) K (4C) K (2C) K (4C)


Similarly, VD = Vq1  Vq2 = 2 2 + 2 2 = + = – 6000 V..
( 5)  2 ( 5)  2 3 3


12. R ELATI ON BETW EEN ELECTR I C FI ELD I N TEN SI T Y AN D ELECTR I C
P OTEN TI AL :
1 2 .1 For uniform electric field :

(i) Potential difference between two points A and B



VB – VA = – E . AB

KVPY_ SA STREAM # 379


Example 20.
A uniform electric field is along x – axis . The potential difference VA– VB = 10 V is between two points A (2m
, 3m) and B (4m, 8m). Find the electric field intensity.
V 10
Sol. E= = = 5 V / m. (It is along + ve x-axis)
d 2


13. ELECTRIC D I P OLE
1 3 .1 Ele ct ric D i p ol e
If two point charges, equal in magnitude ‘q’ and opposite in sign separated by a distance ‘a’ such that
the distance of field point r>>a, the system is called a dipole. The electric dipole moment is defined as
a vector quantity having magnitude p = (q × a) and direction from negative charge to positive charge.
Note: [In chemistry, the direction of dipole moment is assumed to be from positive to negative charge.] The
C.G.S unit of electric dipole moment is debye which is defined as the dipole moment of two equal and
opposite point charges each having charge 10–10 Franklin and separation of 1 Å, i.e.,
1 debye (D) = 10–10 × 10–8 = 10–18 Fr × cm
C
or 1 D = 10–18 × × 10–2 m = 3.3 × 10–30 C × m.
3  10 9
S.I. Unit is coulomb × metre = C . m

Example 21.
A system has two charges q A = 2.5 × 10 –7 C and q B = – 2.5 × 10 –7 C located at points A :
(0, 0, – 0.15 m) and B ; (0, 0, + 0.15 m) respectively. What is the net charge and electric dipole
moment of the system ?
Sol. Net charge = 2.5 × 10–7 – 2.5 × 10–7 = 0
Electric dipole moment,
P = (Magnitude of charge) × (Separation between charges)
= 2.5 × 10–7 [0.15 + 0.15] C m = 7.5 × 10 –8 C m
The direction of dipole moment is from B to A.

1 3 .2 Electric Field Intensity Due to Dipole :
(i) At the axial point :-

 Kq Kq Kq (2ra)
E  (along the P) = P̂
2 2 2
 a  a  2
r   r   r2  a 
 2  2  4 
 
If r >> a then,

 Kq 2ra 2KP
E = P̂ = ,
r4 r3

As the direction of electric field at axial position is along the dipole moment ( P )

 2K P
So, E axial =
r3
KVPY_ SA STREAM # 380
(ii) Electric field at perpendicular Bisector (Equatorial Position)
Enet = 2 E cos  (along – P̂ )

 
 
  a
 
  Kq  2 Kqa
Enet =2  ( P̂) = ( P̂)
2 2 3/2
  2  a 2   r 2   a   2
 a 
r    
2
 r      
 2   2  
 2   
  

If r >> a then
 KP
Enet = 3 ( P̂)
r
 
As the direction of E at equatorial position is opposite of P so we can write in vector form:

 KP
E eqt = –
r3
(iii) Electric field at general point (r, ) :


For this, let’s resolve the dipole moment P into components.

One component is along radial line (=P cos) and other component is r to the radial line (=Psin)

2 2
 2KP cos    KP sin   KP
From the given figure Enet = 2 2
Er  E t      = 3
1  3 cos 2 
 r3   r3  r

KP sin 
Et r3 tan 
tan  =  =
Er 2KP cos  2
3
r
KP tan 
Enet = 3
1  3 cos 2  ; tan  =
r 2

KVPY_ SA STREAM # 381


Example 22.
The electric field due to a short dipole at a distance r, on the axial line, from its mid point is the same
r
as that of electric field at a distance r', on the equatorial line, from its mid-point. Determine the ratio .

1 2p 1 p 2 1 r3 r
Sol. = or = or =2 or, = 21/3
4 0 r 3 4 0 r '3 r 3
r´ 3
r´ 3 r´

Example 23.
Two charges, each of 5 C but opposite in sign, are placed 4 cm apart. Calculate the electric field
intensity of a point that is at a distance 4 cm from the mid point on the axial line of the dipole.
Sol. We cannot use formula of short dipole here because distance of the point is comparable to the distance
between the two point charges.

q = 5 × 10–6 C, a = 4 ×10–2 m, r = 4 × 10–2 m

K (5C) K (5C) 144


Eres = E+ + E– = – = NC–1 = 108 N C–1
2cm2
6 cm 2
144  10 8

Example 24.
Two charges ± 10 C are placed 5 × 10–3 m apart as shown in figure. Determine the electric field at a
point Q which is 0.15 m away from O, on the equatorial line.
Sol. In the given problem, r >> a

1 p 1 q( a)
 E= 3 = 4
4 0 r 0 r3

10  10 6  5  10 3
or E = 9 × 109 × NC–1
0.15  0.15  0.15
= 1.33 ×105 NC–1


1 3 .3 Electric Potentia l due to a sm all dipole :
(i) Potential at axial position :
Kq K( q)
V=  
a  a
r   r  
 2  2

Kqa
V=
 2
 r 2   a  
  2  

If r >> a then
Kqa
V= ; where, qa = p
r2

KP
 Vaxial =
r2

KVPY_ SA STREAM # 382


(ii) Potential at equatorial position :
Kq K (  q)
V=  =0
2 2
2a 2 a
r   r  
2  2

Veqt = 0

(iii) Potential at general point (r,) :



Lets resolve the dipole moment P into
components: Pcos along radial
line and Psin r to the radial line

For the Pcos component, the point A is an axial point,


K P cos  
So, potential at A due to Pcos =
r2
And for Psin component, the point A is an equatorial point,
So potential at A due to Psin = 0
K P cos  
Vnet =
r2

  
K  P . r 
 V=  
r3

Example 25.
(i) Find potential at point A and B due to the small charge - system fixed near origin.(Distance between the
charges is negligible).
(ii) Find work done to bring a test charge q0 from point A to point B, slowly. All parameters are in S.I. units.

Sol. (i) Dipole moment of the system is



P = (qa) î + (qa) ĵ
Potential at point A due to the dipole
 
(P · r ) K [(qa) î  (qa ) ĵ] · ( 4 î  3 ĵ ) k(qa)
VA = K 3 = 3 = 7
r 5 125

 VB =
 
K (qa ) î  (qa ) ĵ · 3 î – 4 ĵ  = –K(qa)
3 125
(5 )

 K(qa)  K(qa ) (7 ) 
(ii) W A B = UB – UA = q0 (VB – VA) = q0  – – 
 125  125 

–K qq0a
 W A B = (8)
125

KVPY_ SA STREAM # 383



1 3 .4 Dipole in uniform electric field
(i) Dipole is placed along electric field :

In this case, Fnet = 0, net = 0, so it is an equilibrium state. And it is a stable equilibrium position.

(ii) If the dipole is placed at angle  from E : -

In this case Fnet = 0 but


Net torque  = (qEsin) (a)
Here qa = P   = PE sin
  
in vector form :   P E

Example 26.
A dipole is formed by two point charge –q and +q, each of mass m, and both the point charges are connected

by a rod of length  and mass m. This dipole is placed in uniform electric field E . If the dipole is disturbed by
a small angle  from stable equilibrium position, prove that its motion will be almost SHM. Also find its time
period.
Sol.
If the dipole is disturbed by  angle,
net = –PE sin (Here – ve sign indicates that direction
of torque is opposite to )
If  is very small, sin  
 net = –(PE)
net  (–) so motion will be almost SHM & C = PE (where, P = q)


 T = 2
C

2
m 
 T = 2m –  2m 
12 2
P.E
2
m  m
 7m  2 7m
= 2 12 2 =2 = 2
qE 12qE 12qE

7m
T=
3qE

KVPY_ SA STREAM # 384



(iii) Potential energy of a dipole placed in uniform electric field :
  B

UB – UA = –  F . dr (for translational motion)


A

 
B
Here, UB – UA = –   . d (for rotational motion)
A
In the case of dipole, at  = 90° , P.E. is assumed to be zero.

U – U90° = –  ( PE sin ) (d) (As the direction of torque is opposite of )
 90 U
U – 0 = – PE cos 

 = 90° is chosen as reference,


 = 90°
so that the lower limit comes out to be zero.

  =0  = 180°
U = – P  E

From the potential energy curve, we can conclude :


(i) At  = 0, there is minimum of P.E. so it is a stable equilibrium position.
(ii) At = 180° , there is maxima of P.E. so it is a position of unstable equilibrium.

Example 27.
Two point masses of mass m and equal and opposite charge of
magnitude q are attached on the corners of a non-conducting uniform
rod of mass m and the system is released from rest in uniform electric
field E as shown in figure from  = 53°
(i) Find angular acceleration of the rod just after releasing
(ii) What will be angular velocity of the rod when it passes
through stable equilibrium.
(iii) Find work required to rotate the system by 180°.
Sol. (i) net = PE sin53° = I 

4
( q ) E  
5 48qE
 = 2 2

m 2
  35 m
 m   m 
12 2  2
(ii) From energy conservation :

1 2
Ki + Ui = Kf + Uf  0 + (– PE cos 53°) =  + (–PE cos 0°)
2
2 2
m 2   7m 2 1 2
where I = + m  + m  =  2 = PE (1– 3/5) = PE
12 2 2 12 2 5

1 7m 2 2 48 qE
 × ×2 = qE or =
2 12 5 35 m
(iii)  W ext = Uf – Ui
 W ext = (–PE cos(180° + 53°)) – (–PEcos 53°)
3 3 6
or W ext = (q)E  5  + (q)E  5   W ext =  5  qE
     

KVPY_ SA STREAM # 385



14. ELECTRI C LI NES OF FORCE (ELOF)
The line of force in an electric field is an imaginary line, the tangent to which at any point on it repre-
sents the direction of electric field at the given point.
1 4 .1 Prop e rt ie s :
(i) Line of force originates out from a positive charge and terminates on a negative charge. If there
is only one positive charge, then lines starts from positive charge and terminates at . If there
is only one negative charge, then lines starts from  and terminates at negative charge.


(ii) Two lines of force never intersect each other because there cannot be two directions of E at a single
Point

impossilble

(iii) Electric lines of force produced by static charges do not form closed loop.
If lines of force make a closed loop, then work done to move a +q charge along the loop will be non-
zero. So it will not be conservative field. So these type of lines of force are not possible in electrostat-
ics.

(iv) The Number of lines per unit area (line density) represents
the magnitude of electric field.
If lines are dense  E will be more
If Lines are rare  E will be less
and if E = O, no line of force will be found there

(v) Number of lines originating (terminating) at a charge is proportional


to the magnitude of charge
Example 28.
If number of electric lines of force from charge q are 10, then find out number of electric lines of force
from 2q charge.

KVPY_ SA STREAM # 386


Sol. No. of ELOF  charge
10  q
 20  2q
So, number of ELOF will be 20.


(vi) Electric lines of force end or start perpendicularly on the surface of a conductor.
(vii) Electric lines of force never enter into conductors.

Example 29. A
Some electric lines of force are shown in figure. For points A and B
(A) EA > EB (B) EB > EA B
(C) VA > VB (D) VB > VA

Sol.: Lines are more dense at A, so EA > EB In the direction of Electric field, potential decreases so VA > VB
Example 30.
If a charge is released in electric field, will it follow lines of force?
Sol. Case I :
If lines of force are parallel (in uniform electric field) :-


In this type of field, if a charge is released, force on it will be qoE and its direction will be along E .So the
charge will move in a straight line , along the lines of force.
Case II : -
If lines of force are curved (in non-uniform electric field) :-

The charge will not follow lines of force

Example 31.
A charge + Q is fixed at a distance d in front of an infinite metal plate. Draw the lines of force indicating the
directions clearly.
Sol. There will be induced charge on two surfaces of conducting plate, so ELOF
will start from +Q charge and terminate at conductor and then will again
start from other surface of conductor.


17. CON DU CTOR AN D I T 'S PROPERT I ES [FOR ELECT ROSTAT I C CON -
DI TI ON ]
(i) Conductors are materials which contain large number of free electrons which can move freely
inside the conductor.
(ii) n electrostatics conductors are always equipotential surfaces.
(iii) Charge always resides on outer surface of conductor.
(iv) f there is a cavity inside the conductor having no charge then charge will always reside only on
outer surface of conductor.
(v) Electric field is always perpendicular to conducting surface.
(vi) Electric lines of force never enter into conductors.
(vii) Electric field intensity near the conducting surface is given by formula

KVPY_ SA STREAM # 387



E = n̂
0

A  
EA  n̂ ; EB  B n̂ and EC  C n̂
0 0 0
(viii) When a conductor is grounded its potential becomes zero.

(ix) When an isolated conductor is grounded then its charge becomes zero.
(x) When two conductors are connected there will be charge flow till their potentials become
equal.
(xi) Electric pressure : Electric pressure at the surface of a conductor is given by formula
2
P= , where  is the local surface charge density..
2 0

1 7.1 Som e othe r im porta nt re sults for a close d conductor:
(i) f a charge q is kept in the cavity then –q will be induced on the
inner surface and +q will be induced on the outer surface of the
conductor (it can be proved using Gauss theorem)

(ii) If a charge q is kept inside the cavity of a conductor and conductor


is given a charge Q then –q charge will be induced on inner sur-
face and total charge on the outer surface will be q + Q. (it can be
proved using Gauss theorem)

(iii) Resultant field, due to q (which is inside the cavity) and induced
charge on S1, at any point outside S1 (like B,C) is zero. Resultant
field due to q + Q on S2 and any other charge outside S2 , at
any point inside of surface S2 (like A, B) is zero

(iv) Resultant field in a charge free cavity in a closed conductor is


zero. There can be charges outside the conductor and on the
surface also. Then also, this result is true. No charge will be in-
duced on the inner most surface of the conductor.

(v) Charge distribution for different types of cavities in conductors

(A) (B)

KVPY_ SA STREAM # 388


(C) (D)

(E) (F)

(G) (H)


Using the result that Eres in the conducting material should be zero and using result (iii) we can show that

Case A B C D E F G H
S1 Uniform Nonuniform Nonuniform Nonuniform Uniform Nonuniform Nonuniform Nonuniform
S2 Uniform Uniform Uniform Uniform Nonuniform Nonuniform Nonuniform NonUniform
Note : In all cases, charge on inner surface S1 = –q and on outer surface S2 = q. The distribution of charge on ‘S1’
will not change even if some charges are kept outside the conductor (i.e. outside the surface S2). But the
charge distribution on ‘S2’ may change if some charges(s) is/are kept outside the conductor.

Example 32.
An uncharged conductor of inner radius R1 and outer radius R2 contains
a point charge q at the centre as shown in figure

(i) Find E and V at points A,B and C
(ii) If a point charge Q is kept outside the sphere at a distance ‘r’
(>>R2) from centre, then find out resultant force on charge Q and
charge q.
Sol. At point A :
Kq Kq K( q)  Kq
VA = + + , EA = OA
OA R 2 R1 OA 3

Note : Electric field at ‘A’ due to –q of S1 and +q of S2 is zero individually because they are uniformly distributed
Kq K ( q) Kq Kq
At point B : VB = + + R = R , EB = 0
OB OB 2 2

Kq  Kq
At point C : VC = , EC = OC
OC OC3
KVPY_ SA STREAM # 389
(ii) Force on point charge Q :
(Note : Here, force on ‘Q’ will be only due to ‘q’ of S2 (see result (iii) )
 KqQ
FQ = 2 r̂ (r = distance of ‘Q’ from centre ‘O’)
r
Force on point charge q:

Fq = 0 (using result (iii) & charge on S1 uniform)

Example 33.
An uncharged conductor of inner radius R1 and outer radius R2 contains
a point charge q placed at point P (not at the centre) as shown in figure
Find out the following :
(i) VC (ii) VA (iii) VB (iv) EA (v) EB
(vi) Force on charge Q, if it is placed at B.
Kq K( q) Kq
Sol. (i) VC = +
R1
+
R2
CP
K( q)
Note : –q on S1 is non-uniformly distributed. Still it produces potential at ‘C’ because ‘C’ is at distance ‘R1’
R1
from each point of ‘S1’.
Kq Kq
(ii) VA = R (iii) VB = (iv) EA = O (point is inside metallic conductor)
2 CB
Kq ^ KQq ^
(v) EB = CB (vi) FQ = CB
CB 2 CB2

(vi) Sharing of charges :
Two conducting hollow spherical shells of radii R1 and R2 having charges Q1 and Q 2 respectively
and separated by large distance & are joined by a conducting wire

Let final charges on spheres are q1 and q2 respectively.


Potential on both spherical shell becomes equal after joining. Therefore,
Kq1 Kq 2
=
R1 R2
q1 R1
 q2
= R2
......(i)
and, q1 + q2 = Q 1 + Q 2 ......(ii)
from (i) and (ii) :
(Q1  Q 2 )R1 (Q1  Q 2 )R 2
q1 = R1  R 2
q2 =
R1  R 2
ratio of charges :
q1 R1 1 4R12 R1
= R  = R
q2 2  2 4R 22 2

1 R2
 ratio of surface charge densities :  = R
2 1

q1 R1
Ratio of final charges : =
q2 R2

1 R2
Ratio of final surface charge densities : =
2 R1

KVPY_ SA STREAM # 390


Example 34.
The two conducting spherical shells are joined by a conducting wire which is cut after
some time when charge stops flowing.Find out the charge on each sphere after that.
Sol. After cutting the wire, the potential of both the shells is equal
Kx K ( 2Q  x ) k x  2Q 
Thus, potential of inner shell, V in = + =
R 2R 2R
Kx K ( 2Q  x ) KQ
and potential of outer shell, V out = + =
2R 2R R
As, Vout = Vin

 – KQ = K x – 2Q   –2Q = x – 2Q  x=0
R 2R
So, charge on inner spherical shell = 0
and outer spherical shell = – 2Q.

Example 35.
Find charge on each spherical shell after joining the inner most shell and outer
most shell by a conducting wire. Also find charges on each surface.

Sol. Let the charge on the innermost sphere be x.

Finally potential of shell 1 = Potential of shell 3

Kx K ( 2Q) K ( 6Q  x ) Kx K( 2Q) K ( 6Q  x )
 + + =  
R 2R 3R 3R 3R 3R
Q
3x –3Q + 6Q – x = 4Q ; 2x = Q ; x=
2
Q
 Charge on innermost shell =
2
5Q
& Charge on outermost shell =
2
Charge on middle shell = –2Q
 Final charge distribution is as shown in figure.

Example 36.
Two conducting hollow spherical shells of radii R and 2R carry charges – Q and
3Q respectively. How much charge will flow into the earth if inner shell is
grounded ?

Sol. When inner shell is grounded to the Earth then the potential of inner shell will
become zero because potential of the Earth is taken to be zero.
Kx K 3Q
+ =0
R 2R
3Q
or x = , (the charge that has appeared on
2
inner shells after grounding)
3Q Q Q
 – (–Q) = [hence, charge flown into the Earth = ]
2 2 2

KVPY_ SA STREAM # 391


Example 37.
An isolated conducting sphere of charge Q and radius R is connected to a similar uncharged sphere
(kept at a large distance) by using a high resistance wire. After a long time, what is the amount of heat
loss ?
Sol. When two conducting spheres of equal radii are connected, charge is equally distributed on them
(Result VI). So, we can say that heat loss of system :
H = Ui – Uf

 Q2   Q 2 / 4 Q2 / 4  Q2
  0  –    =
 8 R   8 0R 8 0R  16  0R
 0  

Problem 1.
Two equal positive point charges 'Q' each are fixed at points B(a, 0) and A(–a, 0). Another negative point
charge q0 is also placed at O(0, 0) then prove that the equilibrium at 'O' is
(i) Stable for displacement in Y-direction.
(ii) Unstable for displacement in X-direction.
Solution :
(i) When charge is shifted along y-axis:
Let x-y direction as :-

After resolving into components, net force will be along negative y-axis so the particle will
return to its original position. So, it is stable equilibrium
(ii) When negative charge q0 is shifted along x-axis.

    KQq0
Initially, FAO  FBO  0  FAO  FBO 
d2
When charge q0 is slightly shifted towards + x axis by small distance x then
 
FBO  FAO
Also, these forces are attractive forces (due to negative charge)
Therefore, the particle will move towards positive x-axis and will not return to its original position
So, it is unstable equilibrium for negative charge.
Problem 2.
A particle of mass m and charge –q is located midway between two fixed charged particles each having
a charge q and a distance 2 apart. Prove that the motion of the particle will be SHM if it is displaced
slightly along perpendicular bisector and released. Also find its time period.
Solution :
Let x-y direction is taken as :-

Particle is shifted along y-axis by a small displacement x. After resolving component of forces between q
and –q charges: By figure. Fnet in x-axis = 0 [Fnet = net force on –q charge] Net force on – q charge in y di-
rection = –2F cos

KVPY_ SA STREAM # 392


kqq x
= –2. .
2 2
( x   ) ( x   2 )1/ 22

 2Kq2 x
|F| =
( x 2   2 )3 / 2

2Kq 2 x
 ma = (for x << ) (a = acceleration of – q charge)
3

2Kq 2
 a= . x (downwards)
m 3
This is equation of S.H.M. (a = – 2x)
So, time period of this charge (–q) :-

m 3
T = 2 Ans.
2Kq2

Problem 3.
Two charges Q each, are placed at two opposite corners of a square. A charge q is placed at each of the
other two corners.
(a) f the resultant force on Q is zero, how are Q and q related ?
(b) Could q be chosen to make the resultant force on each charge zero ?
Solution :
(a) Let on a square ABCD, charges are placed as shown

 
Now, forces on charge Q (at point A) due to other charge are FQQ , FQq and FQq respectively as shown in
figure.
  
Fnet on Q = FQ.Q  FQq  FQq (at point A)

But Fnet = 0
So, Fx = 0
Fx = – FQQ cos45° – FQq

KQ 2 1 KQq Q
 .  2 =0  q=– Ans.
2
( 2a) 2 a 2 2

KVPY_ SA STREAM # 393


(b) For resultant force on each charge to be zero :
Q
From previous data, force on charge Q is zero when q = – If for this value of charge q, force on
2 2
q is zero, then and only then the value of q exists for which the resultant force on each charge is
zero.
Force on q :-
Forces on charge q (at point D) due to other three charges are
  
FqQ , Fqq and FqQ respectively as shown in figure.

Net force on charge q :-


    
Fnet  Fqq  FqQ  FqQ But Fnet = 0
So, Fx = 0

Kq2 1 KQq
Fx = – . – =  q=– 2 2Q
( 2a) 2
2 (a )2

Q
But from previous condition, q = –
2 2
So, no value of q makes the resultant force on each charge zero.
Problem 4.
An infinitely large non-conducting sheet of thickness t and uniform volume charge density  is given in
which left half of the sheet contains charge density  and right half contains charge density. Find the
electric field at the symmetry plane of this sheet
Solution :
t  t 
We can consider two sheets of thickness   x  and   x 
 2   2 
When a point lies inside the sheet.
Net electric field at point P :
Q1 Q2
E = E1 – E2 = 2A   2A 
0 0
[Q 1 : charge on left sheet; Q 2 = charge of right sheet]

t  t   t
A   x   2A   x   3x  
 2   2   2 
=
2A  0 2 0

At the symmetry plane, x = 0


t
So, E=  Ans.
4 0
Problem 5.
Figure shows a uniformly charged thin non-conducting sphere of total
charge Q and radius R. If point charge q is situated at point ‘A’ which is at a
distance r < R from the centre of the sphere, then find out following:
(i) Force acting on charge q. 
(ii) Electric field at centre of sphere.
(iii) Electric field at point B.
Solution :
(i) Electric field inside a hollow sphere = 0
 Force on charge q.
F = qE = q × 0 = 0

KVPY_ SA STREAM # 394


(ii) Net electric field at centre of sphere
 
Enet = E1 + E 2

E1 = field due to sphere = 0

Kq
E2 = field due to this charge =
r2
Kq
 Enet =
r2
 KQ
(iii) Electric field at B due to charge on sphere , E1 = 2 r̂1
r1
 Kq
and due to charge q at A , E 2 = 2 r̂2
r2

   KQ Kq
So, Enet = E1 + E 2 = r̂1 + r̂
2 2
r12 r2

where r1 = CB and r2 = AB

Problem 6.
Figure shows two concentric spheres of radii R1 and R2 (R2 > R1) which contain uni-
formly distributed charges Q and –Q respectively. Find out electric f ield
intensities at the following positions :
(i) r < R1 (ii) R1  r < R2 (iii) r  R2
Solution :
Net electric field = E1 + E2
E1 = field due to sphere of radius R1
E2 = field due of sphere of radius R2
(i) E1 = 0, E2 = 0
 Enet = 0

KQ  Kq
(ii) E1 = 2 , E2 = 0  E  2 r̂
r r

 Kq  Kq   
(iii) E1 = r̂ 
r 2 E 2 = r 2 (– r̂ ) E net = E1 + E 2 = 0

Problem 7.
Three identical spheres each having a charge q (uniformly distributed) and radius R, are kept in such a way
that each touches the other two. Find the magnitude of the electric force on any one sphere due to other two.
Solution :
Given three identical spheres each having a charge q and
radius R are kept as shown.
For any external point, sphere behaves like a point charge.
So it becomes a triangle having point charges at its corners.
 kq 2
| Fqq | =
4R 2
kq 2 60 kq 2 3 3 kq
2

So, net force (F) = 2. . cos = 2. = . 2 Ans.


4R 2 2 4R 2 2 4 R

KVPY_ SA STREAM # 395


Problem 8.
A uniform electric field of 20 N/C exists in the vertically downward direction. Find the increase in the electric
potential as one goes up through a height of 40cm.
dv  
Solution : E=–  dv = – E . d r
dr
  
for E = constant  V = – E . r
V = – 20 (– ĵ ). (40 × 10–2) ĵ = 8 volts.
Problem 9.
An electric field of 10 N/C exists along the x-axis in space. Calculate the potential difference
VB – VA ,where the points A and B are given by –
(a) A = (0,0) ; B = (8m , 3m)
(b) A = (8m, 3m) ; B = (12m , 6m)
(c) A = (0,0) ; B = (12m , 6m)

Solution : Electric field in x - axis means E = 10 î

 
(a) VAB = E . d = 10 î . 8 î = 80 V  VB – VA = – 80V
 
(b) VBC = E . d = 10 î . 4 î = 40 volt  VC – VB = – 40V
 
(c) VAC = E . d = 10 î . 12 î = 120 volt  VC – VA = – 120 V

Problem 10.
Some equipotential surfaces are shown in figure. What can you say about the magnitude and the direction
of the electric field ?

Solution : We know, that the electric field is always perpendicular to equipotential surface. So, making electric field
lines perpendicular to the surface, we find that these lines are originating from the centre. So, the field is
similar to that due to a point charge placed at the centre. So, comparing the given potentials with that due to
point charge, we have,
KQ
V=  KQ = VA rA = VBrB = VCrC = 6 V-m
r
Hence, electric field at distance r can be given by
KQ 6
E= 2 = V/m
r r2
As the electric field lines are directed towards the decreasing potential. So, electric field is along radially
outward direction.

KVPY_ SA STREAM # 396


Problem 11.
A point charge of charge –q and mass m is released with negligible speed from a distance 3 R on the axis
of a fixed uniformly charged ring of charge Q and radius R. Find out its velocity when it reaches at the centre
of the ring.

Solution :

As potential due to uniformly charged ring at its axis (at x distance) is :

kQ
V= ;
R2  x2
So, potential at point A due to ring

kQ kQ
V1 = 2 2 =
R  3R 2R
So potential energy of charge –q at point A

kQq kQ
P.E. 1 = and potential at point B V2 =
2R R
So, potential energy of charge –q at point B:

kQq
P.E. 2 =
R
Now by energy conservation:
P.E. 1 + K.E. 1 = P.E2 + K.E2

kQq kQq 1 kQq


+0= + mv 2  v2 =
2R R 2 mR

kQq
So velocity of charge – q at point B v= Ans.
mR

KVPY_ SA STREAM # 397


OBJECTIVE QUESTIONS
1. A point charge + Q is placed at the centroid of an equilateral triangle. When a second charge + Q is
placed at a vertex of the triangle, the magnitude of the electrostatic force on the central charge is 8 N.
The magnitude of the net force on the central charge when a third charge + Q is placed at another
vertex of the triangle is:

(A) zero (B) 4 N (C) 4 2 N (D) 8 N

2. The electric field inside a sphere which carries a volume charge density proportional to the distance
from the origin  =  r ( is a constant) is :

 r3  r2  r2
(A) (B) (C) (D) none of these
4 0 4 0 3 0

3. Figure shows three circular arcs, each of radius R and total charge as indicated. The net elecric potential at
the centre of curvature is :

+Q

45°
30°
–2Q •

R
+3Q

Q Q 2Q Q
(A) 2 R (B) 4 R (C)  R (D)  R
0 0 0 0

4. An electric field is given by E x = –2x 3 kN/C. The potential of the point (1, –2), if potential of the point
(2, 4) is taken as zero, is
(A) – 7.5 × 103 V (B) 7.5 × 103 V (C) – 15 × 103 V (D) 15 × 103 V

5. Two concentric uniformly charged spheres of radius 10 cm & 20 cm are arranged as shown in the
figure. Potential difference between the spheres is:

(A) 4.5  1011 V (B) 2.7  1011 V (C) 0 (D) none of these

6. A dipole of dipole moment p is kept at the centre of a ring of radius R and charge Q. The dipole moment
has direction along the axis of the ring. The resultant force on the ring due to the dipole is:
kPQ 2kPQ
(A) zero (B) 3 (C)
R R3
kPQ
(D) only if the charge is uniformly distributed on the ring.
R3

KVPY_ SA STREAM # 398


7. In the figure two conducting concentric spherical shells are shown. If the electric potential at the centre
is 20 V & the electric potential of the outer shell is 5 V, then the potential of the inner shell is:

(A) 5 V (B) 15 V (C) 20 V (D) 25 V

8. The figure shows a charge q placed inside a cavity in an uncharged


conductor. Now if an external electric field is switched on :
(A) only induced charge on outer surface will redistribute.
(B) only induced charge on inner surface will redistribute.
(C) both induced charge on outer and inner surface will redistribute.
(D) force on charge q placed inside the cavity will change.

9. Two infinitely large charged planes having uniform surface charge density + and – are placed along
x-y plane and yz plane respectively as shown in the figure. Then the nature of electric lines of forces in
x-z plane is given by :
z –

+ x

z z z z

(A) x (B) x (C) x (D) x

10. On moving a charge of 20 coulombs by 2 cm, 2J of work is done, then the potential difference between
the points is :
(A) 0.1 V (B) 8 V (C) 2 V (D) 0.5 V

11. A charged particle q is placed at the centre O of cube of length L (ABCDEFGH). Another same charge
q is placed at a distance L from O. Then the electric flux through BCFG is :

(A) q/40L (B) zero (C) q/20L (D) q/30L

KVPY_ SA STREAM # 399


12. If the electric flux entering and leaving an enclosed surface respectively is 1 and 2, the electric charge
inside the surface will be :
(A) (2 – 1)0 (B) (1 + 2)0 (C) (2 – 1)0 (D) (1 + 2)0

13. The number of electrons to be put on spherical conductor of radius 0.1 m, to produce an electric field
of 0.036 N/C just above the surface is :
(A) 2.4  105 (B) 2.5  105 (C) 2.6  105 (D) 2.7  105

14. A charged particle q is shot towards another charged particle Q which is fixed, with a speed v. It
approaches Q upto a closest distance r and then returns. If q was given a speed 2v, the closest
distance of approach would be :

(A) r (B) 2r (C) r/2 (D) r/4

15. Three charges + 4q, -q and +4q are kept on a straight line at position (0, 0, 0), (a, 0, 0) and
(2a, 0, 0) respectively. Considering that they are free to move along the x-axis only
(A) all the charges are in stable equilibrium (B) all the charges are in unstable equilibrium
(C) only the middle charge is in stable equilibrium (D) only middle charge is in unstable equilibrium

16. A simple pendulum of mass m and charge + q is suspended vertically by a massless thread of length
. At the point of suspension, a point charge + q is also fixed. If the pendulum is displaced slightly from
equilibrium position, its time period will be


 
(A) T = 2  kq2 (B) T = 2  (C) T = 2  (D) will be greater than 2 
g g g
m 2

17. The diagram shows the arrangement of three small uniformly charged spheres A, B and C. The arrows
indicate the direction of the electrostatic forces acting between the spheres(for example, the left arrow
on sphere A indicates the electrostatic force on sphere A due to sphere B). At least two of the spheres
are positively charged. Which sphere, if any, could be negatively charged ?

B C

(A) sphere A (B) sphere B (C) sphere C (D) no sphere

18. Figure, shown above, shows three situations involving a charged particle and a uniformly charged spherical
shell. The charges and radii of the shells are indicated in the figure. If F1, F2 and F3 are the magnitudes of the
forces on the particle due to the presence of the shell in situations (I), (II) and (III) respectively then

(A) F3 > F2 > F1 (B) F2 > F2 = F3 (C) F3 = F2 > F1 (D) F1 > F2 > F3

KVPY_ SA STREAM # 400


19. There are three concentric thin spheres of radius a, b, c (a > b > c). The total surface charge densities
on their surfaces are , ,  respectively. The magnitude of electric field at r (distance from centre)
such that a > r > b is :
 
(A) 0 (B) (b2  c2) (C) (a2 + b2) (D) none of these
0 r 2
0 r 2

20. Two semicircular rings lying in same plane, of uniform linear charge density  have radius r and 2r. They are
joined using two straight uniformly charged wires of linear charge density  and length r as shown in figure.
The magnitude of electric field at common centre of semi circular rings is -

 
2r r

1 3 1  1 2 1 
(A) 4  2 r (B) 4  2 r (C) 4  r (D) 4  r
o o o o

21. A mercury drop of water has potential 'V' on its surface. 1000 such drops combine to form a new drop. Find
the potential on the surface of the new drop.
(A) V (B) 10V (C) 100V (D) 1000V

22. An electron is revolving around a proton. The total work done in one revolution by electric force on the electron
will be zero if the trajectory of the electron is
(A) circular only (B) elliptical only (C) any closed curve (D) not possible

23. Total electric force on an electric dipole placed in an electric field of a point charge is:
(A) always zero
(B) never zero
(C) zero when mid point of dipole coincides with the point charge
(D) zero when dipole axis is along any electric line of force.

24. Figure above shows a closed Gaussian surface in the shape of a cube of edge length 3.0 m. There

exists an electric field given by E = [(2.0x + 4.0)i + 8.0 j + 3.0 k] N/C, where x is in metres, in the
region in which it lies. The net charge in coulombs enclosed by the cube is equal to

(A) – 54 0 (B) 6 0 (C) –6 0 (D) 54 0

25. At distance of 5cm and 10cm outwards from the surface of a uniformly charged solid sphere, the
potentials are 100V and 75V respectively. Then charge on the sphere is :
 1 9 2 9 5 9 7 9
(A)    10 C (B)    10 C (C)    10 C (D)    10 C
3 3 3 3

KVPY_ SA STREAM # 401


26. Mark the wrong statement -
(A) Equipotential surface never cross and other
(B) For a uniformly charged nonconducting sphere, the electric potential at the centre of the sphere is
1.5 times that at the surface
(C) If potential in a certain region in non zero constant, then the electric field in that region will also be non
zero constant
(D) Inside a spherical unifromly charged shell, the electric field is zero but the electric potential is the same
as that at the surface.

27. A hollow conducting sphere is placed in an electric field pro-


duced by a point charge placed at P as shown in figure. Let VA,
VB, VC be the potentials at points A, B and C respectively. Then
(A) VC > VB (B) VB > VC
(C) VA > VB (D) VA = VC

28. A point charge is kept at the centre of a metallic insulated spherical shell. Then
(A) Electric field out side the sphere is zero (B) Electric field inside the sphere is zero
(C) Net induced charge on the sphere is zero (D) Electric potenitial inside the sphere is zero
29. A thin straight rod of length  carrying a uniformly distributed charge q is located in vacuum. Find the
magnitude of the electric force on a point charge 'Q' due to rod kept as shown in the figure.

qQ qQ (a   ) qQ
(A) 4 a(a  ) (B) n (C) (D) zero
0 4 0 a 2 0 a(a   )

30. In a uniformly charged hollow sphere of total charge Q and radius R, the electric potiential V is plotted as
function of distance from the centre. The graph which would correspond to the above will be :

(A) (B) (C) (D)

31. The electrostatic potential inside a charged spherical ball is given by  = ar3 + b where r is the distance from
the centre; a,b are constants. Then the charge density inside the ball is :
(A) –24 a0r (B) –9a0r (C) –24 a0 (D) –6 a0
32. Two small balls, each having equal positive charge Q are suspended by two insulating strings of equal
length L from a hook fixed to a stand. If the whole set-up is transferred to a satellite in orbit around the earth,
the tension in equilibrium in each string is equal to

kQ kQ 2 kQ 2
(A) zero (B) (C) (D)
L2 2L2 4L2

KVPY_ SA STREAM # 402


33. Two positive charges of magnitude ‘q’ are placed at the ends of a side (side 1) of a square of side ‘a’. Two
negative charges of the same magnitude are kept at the other corners. Starting from rest, if a charge Q
moves from the middle of side 1 to the centre of square, its kinetic energy at the centre of square is :
1 2qQ  1 
(A) zero (B) 4 1  

a  
0 5

1 4qQ  1  1 2qQ  1 
(C) 4 1   (D) 4 1 – 

a   
a  
0 5 0 5


34. A thin semi-circular ring of radius r has a negative charge q distributed uniformly over it. The net field E at the
centre O is :

q q q q
(A) 2 2
ĵ (B) – 2 2
ĵ (C) – 2 2
ĵ (D) ĵ
4π ε 0r 4π ε 0r 2π ε 0r 2π ε 0r 2
2

35. A charge Q is placed at corner of a square. A charge q is placed at each of the adjucent two corners. If the
net electrical force on Q is

Kq 2 Kq2 2Kq2 2Kq2


(A) (B) (C) (D)
a2 2 a2 a2 a2

36. In the diagram shown P is a point negative charge. It’s weight is balanced by the electric force due to the
fixed very long wire. The equilibrium of the particle is

(A) stable, for vertical displacements


(B) neutral for vertical displacements
(C) stable for horizontal displacements (parallel to the wire)
(D) neutral for horizontal displacements (parallel to the wire)


37. A dipole of dipole moment P = 2 î  3 ĵ  4k̂ is placed at point A (2, –3, 1). The electric potential due to this
dipole at the point B (4 – 1, 0) is equal to (All the parameters specified here are in S.. units.)
(A) 2 ×109 volts (B) – 2 ×109 volts (C) 3 ×109 volts (D) – 3 ×109 volts

38. A charge q is placed at the centre of the cubical vessel (with one face open) as shown in figure. The flux of the
electric field through the surface of the vessel is

q
(A) zero (B) q/0 (C) 5q/60 (D) 4 
0

KVPY_ SA STREAM # 403


39. A large sheet carries uniform surface charge density . A rod of length 2 has a linear charge density 
on one half and - on the second half. The rod is hinged at mid-point O and makes angle  with the
normal to the sheet. The electric force experienced by the rod is

 2  2
(A) 0 (B) sin  (C) sin  (D) None of these
2 0 0

40. The figure shows two large, closely placed, parallel, nonconducting sheets with identical (positive)
uniform surface charge densities, and a sphere with a uniform (positive) volume charge density. Four
points marked as 1, 2, 3 and 4 are shown in the space in between. If E 1, E2, E3 and E4 are magnitude
of net electric fields at these points respectively then :

(A) E1 > E2 > E3 > E4 (B) E1 > E2 > E3 = E4 (C) E3 = E4 > E2 > E1 (D) E1 = E2 = E3 = E4

41. Three charges +4q, Q and q are placed in a straight line of length  at points at distance 0, /2 and
 respectively from one end of line. What should be the value of Q in order to make the net force on q
to be zero?
(A) –q (B) –2q (C) –q/2 (D) 4q
42. Two similar very small conducting spheres having charges 40 C and –20 C are some distance apart. Now
they are touched and kept at same distance. The ratio of the initial to the final force between them is :
(A) 8 : 1 (B) 4 : 1 (C) 1 : 8 (D) 1 : 1
43. Two point charges placed at a distance r in air exert a force F on each other. The value of distance R at
which they experience force 4F when placed in a medium of dielectric constant K = 16 is :
(A) r (B) r/4 (C) r/8 (D) 2r

44. A total charge of 20 C is divided into two parts and placed at some distance apart. If the charges
experience maximum coulombian repulsion, the charges should be :
40 20
(A) 5 C , 15 C (B) 10 C , 10 C (C) 12 C , 8 C (D) C, C
3 3
45. The magnitude of electric force on 2  c charge placed at the centre O of F
two equilateral triangles each of side 10 cm, as shown in figure is P. If
E A
charge A, B, C, D, E & F are 2  c, 2  c, 2  c, -2 c, - 2  c, - 2  c respectively,
then P is: O
(A) 21.6 N (B) 64.8 N D B
(C) 0 (D) 43.2 N
C
46. There is a uniform electric field in X-direction. If the work done by external agent in moving a charge of 0.2
C through a distance of 2 metre slowly along the line making an angle of 60º with X-direction is 4 joule,
then the magnitude of E is:
(A) 3 N/C (B) 4 N/C (C) 5 N/C (D) 20 N/C

KVPY_ SA STREAM # 404


47. Two equal positive charges are kept at points A and B. The electric potential, while moving from A to B
along straight line :
(A) continuously increases (B) remains constant
(C) decreases then increases (D) increases then decreases
48. A semicircular ring of radius 0.5 m is uniformly charged with a total charge of 1.5 × 10 –9 coul. The
electric potential at the centre of this ring is :
(A) 27 V (B) 13.5 V (C) 54 V (D) 45.5 V
49. When a charge of 3 coul is placed in a uniform electric field, it experiences a force of 3000 newton. The
potential difference between two points separated by a distance of 1 cm along field within this field is:
(A) 10 volt (B) 90 volt (C) 1000 volt (D) 3000 volt
50. A 5 coulomb charge experiences a constant force of 2000 N when moved between two points separated
by a distance of 2 cm in a uniform electric field. The potential difference between these two points is:
(A) 8 V (B) 200 V (C) 800 V (D) 20,000 V

KVPY PROBLEMS (PREVIOUS YEARS)


1. A negatively charged particle initially at rest is placed in an electric field that varies from point to point. There
are no other fields. Then : (KVPY/2008)
(A) the particle moves along the electric line of force passing through it.
(B) the particle moves opposite to the electric line of force passing through it.
(C) the direction of acceleration of the particle is tangential to the electric line of force at every instant.
(D) the direction of acceleration of the particle is normal to the electric line of force at every instant.
2. Two charges +q and –q are placed at a distance b apart as shown in the figure
below. (KVPY/2009)
The electric field at a point P on the perpendicular bisector as shown as :
(A) along vector  A
(B) along vector  B

(C) along vector 


C
(D) Zero

3. 5 charges each of magnitude 10–5 C and mass 1 kg are placed (fixed) symmetrically about a movable central
charges of magnitude 5 × 10–5C and mass 0.5 kg as shown. The charges at P1 is removed. The acceleration
of the central charge is : (KVPY/2009)
P1

P2 P5
O

P3 P4
1
[Given OP1 = OP2 = OP3 = OP4 = OP5 1 m ; 4 = 9 × 109 in SI units]
0
(A) 9 m s–2 upwards (B) 9 m s–2 downwards
(C) 4.5 m s–2 upwards (D) 4.5 m s–2 downwards
4. 12 positive charges of magnitude q are placed on a circle of radius R in a manner that they are equally
spaced. A charge +Q is placed at the centre. If one of the charges q is removed, then the force on Q is:
(KVPY 2010/ SA)
(A) zero
qQ
(B) 40R2
away from the position of the removed charge.

11qQ
(C) away from the position of the removed charge.
40R2

qQ
(D) 40R2
towards the position of the removed charge.

KVPY_ SA STREAM # 405


5. Two charges +Q and _2Q are located at points A and B on a horizontal line as shown below :

The electric field is zero at a point which is located at a finite distance : (KVPY 2011/ SA)
(A) On the perpendicular bisector of AB (B) left of A on the line
(C) between A and B on the line (D) right of B on the line

6. A girl brings a positively charged rod near a thin neutral stream of water from a tap. She observes that the
water stream bends towards her. Instead, if she were to bring a negatively charged rod near to the stream, it
will : (KVPY 2012/ SA)
(A) bend in the same direction (B) bend in the opposite direction
(C) not bend at all (D) bend in the opposite direction above and below the rod

7. Consider an initially neutral hollow conducting spherical shell with inner radius r
and outer radius 2r. A point charge +Q is now placed inside the shell at a
distance r/2 from the centre. The shell is then grounded by connecting the outer
surface to the earth. P is an external point at a distance 2r from the point charge
+Q on the line passing through the centre and the point charge +Q as shown in
the figure. [KVPY_2013]
The magnitude of the force on a test charge +q placed at P will be
1 qQ 1 9qQ 1 4qQ
(A) (B) 4 (C) 4 (D) 0
4 0 4r 2 0 100r
2
0 25r
2

8. Two possitively charged spheres of masses m1, and m2, are suspended from a common point at the ceiling
by identical insulating massless strings of length l. Charges on the two spheres are q1 and q2, respectively.
At equilivrium both strings make the same angle  with the vertical. Then
[KVPY_2014_SA]
(A) q1m1 = q2m2 (B) m1 = m2 (C) m1 = m2sin  (D) q2m1 = q1m2.

9. Two equal charges of magnitude Q each are placed at a dictance d apart. Their electrostatic energy is E. A
third charge -Q/2 is brough midway betway these two charges.The electrostatic energy of the system is now
[KVPY_2014_SA]
(A) -2E (B) -E (C) 0 (D) E

10. Two small metal balls of different mass m 1 and m 2 are connected by strings of equal length to a fixed
point. When the balls are given charges, the angles that the two strings make with the vertical are 30º
and 60º, respectively. The ratio m 1/m 2 is close to [KVPY_2015_SB] [2 Marks]
(A) 1.7 (B) 3.0 (C) 0.58 (D) 2.0

11. An electric field due to a positively charged long straight wire at a distance r from it is proportional to r–1 in
magnitude. Two electrons are orbiting such a long straight wire in circular orbits of radii 1Å and 2Å. The ratio
of their respectively time periods is [KVPY_2016_SA] [1 Marks]
(A) 1 : 1 (B) 1 : 2 (C) 2 : 1 (D) 4 : 1

12. A 20 gm bullet whose specific heat is 5000 J(kg–°C) and moving at 2000 m/s plunges into a 1.0 kg block of
wax whose specific heat is 3000 J (kg–°C) . Both bullet and wax are at 25°C and assume that (i) the bullet
comes to rest in the wax and (ii) all its kinetic energy goes into heating the wax. Thermal temperature of the
wax in °C is close to. [KVPY_2016_SA] [2 Marks]
(A) 28.1 (B) 31.5 (C) 37.9 (D) 42.1
2q
13. Positive point charges are placed at the vertices of a star shape as a

shown in the figure. Direction of the electrostatic force on a negative q q


point charge at the centre O of the star is [KVPY_2017_SA] [1 Marks] O.
(A) towards right (B) vertically up
(C) towards left (D) vertically down q 3q

KVPY_ SA STREAM # 406


14. A total solar eclipse is observed from the earth. At the same time an observer on the moon views the earth.
She is most likely to see (E denotes the earth) [KVPY_2017_SA] [1 Marks]
E E

A. C.

E E

B. D.

(A) A (B) B (C) C (D) D


15. A proton of mass m and charge e is projected from a very large distance towards an particle with velocity
v. Intially, particle is at rest, but it is free to move. If gravity is neglected, then the minimum separation along
the straight line of their motion will be [KVPY-SA_2018 2 Mark]
(A) e2/4 mv2 (B) 5e2/4mv2 (C) 2e2/4mv2 (D) 4e2/4mv2

EXERCISE # 1
1. (D) 2. (B) 3. (A) 4. (B) 5. (A) 6. (B) 7. (C)
8. (A) 9. (C) 10. (A) 11. (B) 12. (A) 13. (B) 14. (D)
15. (B) 16. (C) 17. (A) 18. (C) 19. (B) 20. (D) 21. (C)
22. (C) 23. (B) 24. (D) 25. (C) 26. (C) 27. (D) 28. (C)
29. (A) 30. (D) 31. (D) 32. (D) 33. (C) 34. (D) 35. (D)
36. (D) 37. (B) 38. (C) 39. (A) 40. (C) 41. (A) 42. (A)
43. (C) 44. (B) 45. (D) 46. (D) 47. (C) 48. (A) 49. (A)
50. (A)
EXERCISE-2
1. (C) 2. (A) 3. (A) 4. (D) 5. (B) 6. (A) 7. (D)
8. (B) 9. (B) 10. (A) 11. (B) 12. (C) 13. (A) 14. (B)
15. (B)

1. R= 8 2  8 2  2.8 . 8 cos 120 = 8 N

2. We can consider all the charge inside the sphere to be concentrated on the center of sphere
Consider an elementry shell of radius x and thickness dx.
r


K 4x 2 dx (x ) r

E=

K dq
=
0
=
k 4
2  x dx
3
=
r 2
r2 r2 r 0
4 0

KVPY_ SA STREAM # 407


3. V = V1 + V2 + V3
(where V1, V2 & V3 are potentials due to the three parts of ring)
1 Q 1   2Q  1  3Q  1  2Q  Q
= 4  . + 4   +

  = .  =

0 R 0  R  4 0  R  4  0  R  2 0R

 
4. dV = – E. d r = – (2x 3  10 3 î ). (dx î  dy ĵ  dz k̂ ) = 2x 3 ×103 dx
v 1
  dV =–  ( 2x 3) × 103 dx  V = 7.5 × 103 V
0 2

5. Potential difference due to inner 10C charge


1 1
= K 10  .  .  = 9 × 1010 (5) = 45 × 1010 = 4.5 × 10111V
 1 2

 K  20 K  20 
Potential difference due to outer charge =  – =0V
 0. 2 0. 2 
 P.d. = 4.5 × 1011V

kp
6. Electric field at each point on the surface of ring due to dipole is E =
R3
in direction opposite to the dipole moment. (figure below)
kpQ
Hence net force on ring is F = QE =
R3
Alternate solution
Electric field due to ring at point P on its axis at a distance x from centre O of ring is

Qx dE  kQ dE k Qp
E=k. 2 2 3/2 ; dx  =  Force on dipole = =
(x  R )  at x 0 R 3 dx R3

7. The electric field inside the inner shell is zero


So, the potential on inner shell and all the points inside it will be constant.

8. The distribution of charge on the outer surface, depends only on the charges outside, and it distributes itself
such that the net, electric field inside the outer surface due to the charge on outer surface and all the outer
charges is zero. Similarly the distribution of charge on the inner surface, depends only on the charges inside
the inner surface, and it distributes itself such that the net, electric field outside the inner surface due to the
charge on inner surface and all the inner charges is zero.
Also the force on charge inside the cavity is due to the charge on the inner surface. Hence answer is option
9. (Easy) The electric field intensity due to each uniformly charged infinite plane is uniform. The electric
field intensity at points A, B, C and D due to plane 1, plane 2 and both planes are given by E 1, E2 and
E as shown in figure 1. Hence the electric lines of forces are as given in figure 2.
z
E1 – E1 z
E E
B E2 E2 A
1 x x
E2 E2 +
C D
E E
E1 E1
2
(figure 1) (figure 2)
Aliter :
Electric lines of forces originate from positively charged plane and terminate at negatively charged
plane. Hence the correct representation of ELOF is as shown figure 2.

KVPY_ SA STREAM # 408


10. By definition of electrostatic potential energy,
U = qV
U 2
V = = = 0.1 V..
q 20

11. Total flux in = Total flux out


 Net flux through BCFG = 0

qin
12. = 
0

Net  = 2 – 1  qin = (2 – 1) 0 .

13. The electric field at the surface of conductor


1 q
E . 2
4  0 R

Charge q  40 R 2 E
From q = ne
ne  4 0 R 2 E
4 0R 2 E
n
e
1
Here, 4 0  coulom b2/ N-m 2
9  109
R = 0.1 m
E = 0.036 N/C

1 0.12  0.036
e = 1.6 × 10-19 coulomb. So, n   = 2.5 × 105
9  109 1.6  1019
2v
14. Q
q
x
From the given data, using energy conservation
1 KQq
mv 2 =
2 r
When particle is shot with a speed 2V, Let distance of closest apporach = x
1 KQq r
m.4v 2 =  x=
2 x 4

15.

All charges are in unstable equilibrium.


16. Torque of electrostatic force is zero.
17. (Easy)It can be seen from the diagram that only the sphere B and sphere C repel. Hence they both must be
of same type. According to the fact that at least two spheres are positively charged, therefore both spheres
should be positively charged. Since attraction occurs for two remaining pairs it can be concluded that the
sphere A is negatively charged.
18. F1 is zero as charge lies inside the shell (Electric field inside the shell due to its own charge is zero). F2 = F3
as charge over the surface of the shell behaves like centratd at the centre.

KVPY_ SA STREAM # 409


19.

Electric field at a distance r (a > r > b) will be due to charges enclosed in r only, & Since, a sphere acts
as a point charge for points outside its surface,
kQc kQb k 
 E= 2

2 = 2 ( × 4c2 + (– ) 4b2) = (c 2  b 2 )
r r r 2
0r
20. The electric field due to both straight wires shall cancel at common centre
O. The electric field due to larger and smaller semi circular rings at O be E
and E' respectively.
E' E 

1 2 1 2 2r O r
E = 4  2r E' = 4  r
o o

1  2   1 
 Magnitude of electric field at O is = E' – E =    =
4  o  r r 4  o r
21. Let the radius of each mercury drop be r.
If q is charge on each drop
q
The potential of drop V = 4  r or q = 4 0 rV ...(1)
0
Let R be the radius of the new drop formed by combination of 1000 drops of radius r.
4 3 4 3
R = 1000 r  R = 10 r ....(2)
3 3
(1000 q) (100 q)
 Potential of new drop 4  R = 4  r = 100 V
0 0

23. Electric field of a point charge is non-uniform hance net force can never be zero.
 
24.  =  E . ds
Direction of field at x = – 3m is along negative x axis. Area vector is also along same direction.
Q z 2
2
(2(-3)+4)(3)
=6× 9= 3(3)

Q = 54 0 2
8(3)
Components of electric field which are constant, 2
8(3) 2
2(0)+4(3)
do not contribute in net flux in or out. y
qin 2
= 54  qin = 54 0 3(3)
0 x

kQ kQ
25. = 100V & = 75 V
(r  5cm) (r  10cm)
5 kQ kQ
 Q=  10 – 9 C , r = 10 cm  Vsurface = = 150V Esurface = = 1500 V/m
3 2 r2
3 3
Vcentre = Vsurface = × 150 = 225 V
2 2

27. Conducting surface behaves as equipotential surface.

KVPY_ SA STREAM # 410


29. As the charge on the rod is not point charge, therefore, first we have to
find force on charge Q due to charge over a very small part on the
length of the rod. This part, called element of length dy can be considered
as point charge.
q
Charge on element, dq = dy = dy

K.dq.Q K.Q.q.dy
Electric force on 'Q' due to element = 2 =
y y 2 .
All forces are along the same direction,
 F=  dF . This sum can be calculated using integration,

a a
KQqdy KqQ  1  KQ.q  1 1 
therefore, F = 
y a
y 2 =

 
 y a
= 
  a a   
=
KQq
a(a  )

30.

31.  = ar2 + b
d
E=– = – 2ar
dt
 q
 E.dS 
0

q
–2ar . 4r2 = 
0

q = – 8 0ar3

q
=
4 3
r
3
 = –6a0 Ans.
33. Potential at point A,
2Kq 2Kq
VA = 
a a 5
Potential at point B,
VB = 0
 Using work energy theroem,
W AB)electric = Q(VA – VB)
2KqQ  1   1  2Qq  1 
= 1   =   1  
a   4  
5  0  a  5

KVPY_ SA STREAM # 411


  2kλ   λ
34. E  (  ĵ)  E (  ĵ )
 r  2πε 0r

q  q
λ  E (  ĵ)
πr 2π 2 ε 0 r 2


K p. r 9  10 9  (2 î  3 ĵ  4k̂ ).( 2 î  2 ĵ  k̂ )
37. V= =
r3 [ 22  22  12 ]3 / 2

9  10 9  ( 4  6 – 4)
=
27

V = – 2 ×109 volts Ans.

39. Nearby the plate, field is uniform. Equal and opposite forces are experienced by upper half and lower
half

40. Electric field due to both the plates will be cancelled out for all the 1

points. So the net electric field at the points will be governed only by 2 3
+ 4

the sphere. Farther the point from the sphere, lesser the magnitude of 
2Î 0

2Î 0

electric field.
Therefore E3 = E4 > E2 > E1

42. final charge on both spheres = 10 C each.

F1 (q1 q2 ) i 800
 
F2 ( q1 q2 )f 100 = 8 : 1

k q1q2 k q1q2 r
43. F= 2 ....(1) 4F = 16 R2 ...(2)  R=
r 8

44. Let the two charges are q & (20 – q)C

K ( q) ( 20  q)
 Fe =
r2

dFe
Fe will be max, when dq = 0

dFe K
or = 2 (20 – 2q) = 0
dq r

  q = 10 C.

KVPY_ SA STREAM # 412


45.

10
The given figure shows force diagram for charge at O due to all other charges with r = cm
3
 F net = 2F + 4F cos 60º = 4F

4k(2c )(2c ) 4  9  10 9  2  2  10 –12


= 2

 10   1 
   
 300 
 3100 
= 36 × 4 × 300 × 10–3 N = 43.2 N. (Towards E)
46. W = Fr cos 
 4 = 0.2 E 2 cos 60º
 E = 20 N/C.

9  109  1.5  10 –9
48. V= = 27 V..
(.5)

3000
49. E= = 1000 N/C.
3
1000  1
V = Ed = = 10V..
100

KVPY_ SA STREAM # 413


CU RREN T ELECT RI CI T Y

1. ELECTRI C CUR REN T
(a) Time rate of flow of charge through a cross sectional area is called Current.
if q charge flows in time interval t then average current is given by

q
I av = and
t
Instantaneous current

q dq
i =. lim 
t  0 t dt
(b) Direction of current is along the direction of flow of positive charge or opposite to the direction
of flow of negative charge. But the current is a scalar quantity.
i i

q+ velocity q– velocity
SI unit of current is ampere and
1 Ampere = 1 coloumb/sec
1 coloumb/sec = 1A

2. CON D UCTOR
In some materials, the outer electrons of each atom or molecule are only weakly bound to it. These
electrons are almost free to move throughout the body of the material and are called free electrons. They
are also known as conduction electrons. When such a material is placed in an electric field, the free
electrons drift in a direction opposite to the field. Such materials are called conductors.

3. I N SULATOR
Another class of materials is called insulators in which all the electrons are tightly bound to their respec-
tive atoms or molecules. Effectively, there are no free electrons. When such a material is placed in an
electric field, the electrons may slightly shift opposite to the field but they can’t leave their parent atoms
or molecules and hence can’t move through long distances. Such materials are also called dielectrics.

Cu rre n t , ve locit y a n d cu rre n t d e nsit y

n  no. of free charge particles per unit volume


q  charge of each free particle
i  charge flow per unit time
i = nqvA
Current density, a vector, at a point have magnitude equal to current per unit normal area at that
point and direction is along the direction of the current at that point.
 di 
J n
ds
 
so di = J. ds
Current is flux of current density.

KVPY_ SA STREAM # 414


Due to principle of conservation of charge:
Charge entering at one end of a conductor = charge leaving at the other end, so current does not
change with change in cross section and conductor remains uncharged when current flows through it.

4. RELATI ON BETW EEN I & V I N A COND UCTOR
In absence of potential difference across a conductor no net current flows through a cross section.
When a potential difference is applied across a conductor the charge carriers (electrons in case
of metallic conductors) start drifting in a direction opposite to electric field with average drift velocity.
If electrons are moving with velocity v d , A is area of cross section and n is number of free electrons
per unit volume then,
I = nAev d.


vd  ,

 average displacement of electron along the wire between two successive collisions. It is also
called mean free path.
 the time in which the particle does not collide with any other particle and is called as relaxation time.

1  eE  2 2
1 e 2 V
=    = 1 e .E = 
2 m  2 m 2 m 

1 e 2 V 1  nAe 2  
  =  V
i = nAe .  2m 
2 m    
nAe 2 
i = V
2m 
As temperature (T)   

5. ELECTR I CAL RESI STAN CE


The property of a substance by virtue of which it opposes the flow of electric current through it is
termed as electrical resistance. Electrical resistance depends on the size, geometry, temperature
and internal structure of the conductor.
 
R=  V=×
A A
V  
 =   E = J   J= = current density
 A A

2m 1
 is called resistivity (it is also called specific resistance), and = 2 = ,  is called conductivity..
ne  
Therefore current in conductors is proportional to potential difference applied across its ends. This is
Ohm's Law. Units: R  ohm(),   ohm  meter(  m) also called siemens,    1m 1 .

Example 1. The dimensions of a conductor of specific resistance  are shown below. Find the resistance
of the conductor across AB, CD and EF.
A D

c
E F

b
a

C B

KVPY_ SA STREAM # 415


c b a
Answer : RAB = , RCD = , REF =
ab ac bc
Solution : For a condition

 Re sistivity  length
R= = Area of cross sec tion
A

c b a
RAB = , RCD = , REF =
ab ac bc


6. D e pe nde nce of Re sist a nce on va rious f a ct ors

 2m 
R  
A ne 2  A
Therefore R depends as

1 1 1
(1)   (2)  (3)  
A n 
(4) and in metals  decreases as T increases  R also increases.
Results
(a) On stretching a wire (volume constant)

R1  21
If length of wire is taken into account then R  2
2 2

R1 r24
If radius of cross section is taken into account then R  4 , where R1 and R2 are
2 r1

initial and final resistances and  1 ,  2 , are initial and final lengths and r1 and r2 initial
and final radii respectively. (if elasticity of the material is taken into consideration,
the variation of area of cross-section is calculated with the help of Young’s modulus
and Poison’s ratio)
(b) Effect of percentage change in length of wire

2
 x 
 2 1 
R2 100 
  2  where  - original length and x- % increment
R1 

if x is quite small (say < 5%) then % change in R is


 2 
 1  x   1 
R 2  R1  100  
 100      100  2x %
R1  1 
 
 

Example 2. If a wire is stretched to double its length, find the new resistance if original resistance of the
wire was R.

Solution : As we know that R =
A

KVPY_ SA STREAM # 416


 '
in case R =
A'

 ' 2

A '  '  A (volume of the wire remains constant)

A   2 
A'   R = =4 = 4R
2 A/2 A
Example 3. The wire is stretched to increase the length by 1% find the percentage change in the
Resistance.
Solution : As we known that


 R=
A

R   A  A
= + – and = –
R   A  A

R
= O + 1 +1 = 2
R
Hence percentage increase in the Resistance = 2%

N ot e :
 Above method is applicable when % change is very small.
Electric current in resistance
In a resistor current flows from high potential to low potential
High potential is represented by positive (+) sign and low potential
is represented by negative (–) sign.
VA – VB = iR
If V1 > V2
then current will flow from A to B

V1  V2
and i =
R
If V1 < V2

V2  V1
then current will go from B to A and i =
R
Example 4 . Calculate current (i) flowing in part of the circuit shown in figure?

6
Solution : VA – VB = i×R  i= = 3A Ans.
2


7. ELECTRI CAL P OW ER :
Energy liberated per second in a device is called its power. The electrical power P delivered or
consumed by an electrical device is given by P = VI, where V = Potential difference across the
device and
 = Current.
KVPY_ SA STREAM # 417
If the current enters the higher potential point of the device then electric power is consumed by it
(i.e. acts as load). If the current enters the lower potential point then the device supplies power (i.e.
acts as source).

V.dq
Power =
dt
= V 
P = V 
If power is constant then energy = P t

If power is variable then

Energy =  pdt
Power consumed by a resistor

V2
P = 2R = V = .
R
When a current is passed through a resistor energy is wasted in overcoming the resistance of the
wire. This energy is converted into heat.

V2
W = Vt = 2 Rt = t
R
The heat generated (in joules) when a current of  ampere flows through a resistance of R ohm for t
second is given by:
I2 Rt
H = 2 Rt Joule = Calorie
4.2
1 unit of electrical energy = 1 Kilowatt hour = 1 KWh = 3.6 x 10 6 Joule.

Example 5. If bulb rating is 100 watt and 220 V then determine


(a) Resistance of filament
(b) Current through filament
(c) If bulb operate at 110 volt power supply then find power consumed by bulb.
Solution : Bulb rating is 100 W and 220 V bulb means when 220 V potential difference is applied
between the two ends then the power consumed is 100 W
Here V = 220 Volt
P = 100 W

V2
= 100 So R = 484 
R
Since Resistance depends only on material hence it is constant for bulb

V 220 5
=   Amp.
R 22  22 11
power consumed at 110 V

110  110
 power consumed = = 25 W
484

KVPY_ SA STREAM # 418



8. ELECTROM OTI VE FORCE : (E.M .F.)
Definition  : Electromotive force is the capability of the system to make the charge flow.
Definition II : It is the work done by the battery for the flow of 1 coloumb charge from lower potential
terminal to higher potential terminal inside the battery.

1 0 .1 Re pre se nt a t ion f or ba t t e ry :
Idea l cell :
Cell in which there is no heating effect.

N on ide a l ce ll :
Cell in which there is heating effect inside due
to opposition to the current flow internally

Ca se I : Battery acting as a source (or battery is discharging)


VA–VB =  – ir
VA–VB
 it is also called terminal voltage.
The rate at which the chemical energy of the cell is consumed = i

The rate at which heat is generated inside the battery or cell = i 2r


electric power output = i – i 2r
= ( – ir) i
Ca se II : Ba t t e ry a ct ing a s a loa d (or ba t t e ry cha rging) :
VA–VB =  + ir
the rate at which chemical energy stored in the cell = i
thermal power inside the cell = i 2r
electric power input =i + i 2r = (+ir) i = (VA–VB) i
D e f init ion I II :
Electromotive force of a cell is equal to potential difference between its terminals when no current
is passing through the circuit.
Ca se III :
When cell is in open circuit
i = 0 as resistance of open circuit is infinite (  ).
So V =  so open circuit terminal voltage difference is equal to emf of the cell.
Ca se IV :
Short circuiting : Two points in an electric circuit directly connected by a conducting wire are called
short circuited, under such condition both points are at same potential.
When cell is short circuited

i= and V = 0, short circuit current of a cell is maximum.
r

KVPY_ SA STREAM # 419


N ot e :
 The potential at all points of a wire of zero resistance will be same.

* Ea rt hing : If some point of circuit is earthed then its potential is assumed to be zero.
9. RELATIVE P OTEN TIAL
While solving an electric circuit it is convenient to chose a reference point and assigning its voltage
as zero, then all other potentials are measured with respect to this point. This point is also called
the common point.

Example 6. In the given electric circuit find


(a) current
(b) power output
(c) relation between r and R so that the electric power
output (that means power given to R) is maximum.
(d) value of maximum power output.
(e) plot graph between power and resistance of load

(f) From graph we see that for a given power output there exists two values of external
resistance, prove that the product of these resistances equals r 2.
(g) what is the efficiency of the cell when it is used to supply maximum power.
Solution : (a) In the circuit shown if we assume that potential at A is zero then potential at B is
 – ir. Now since the connecting wires are of zero resistance
 VD = VA = 0  VC = VB =  – ir
Now current through CD is also i
( it’s in series with the cell).
VC  VD (  i r )  0 
 i =  Current i =
R R r R

N ot e : After learning the concept of series combination we will be able to


calculate the current directly

2
(b) Power output P=i R=2
.R
r  R 2
2 2 2R 2
(c)
dP
= — = R  r – 2R
dR r  R 2 r  R 3 R  r 3
for maximum power supply
dP
= 0  r + R – 2R = 0  r=R
dR
Here for maximum power output outer resistance should be equal to internal resistance
2
(d) Pmax =
4r
(e) Graph between ‘P’ and R
maximum power output at R = r

2 
Pmax =  i=
4r r R

KVPY_ SA STREAM # 420


(f) Power output

 2R
P =
(r  R )2
P (r2 + 2rR + R2)= 2R

2
R2 + (2r – ) R + r2 = 0
P
above quadratic equation in R has two roots R 1 and R2 for given values of , P and r
such that
 R1R2 =r2 (product of roots)
r2 = R1R2
(g) Power of battery spent

2 2
= .2r =
r  r 2
2r
power (output)
2
   2
=   r =
rr 4r

2
 100
power output 4r 1
Efficiency = = 2
 ×100 = 50%
total power spent by cell  2
2r

Example 7. In the figure given beside


find out the current in the wire BD

Solution : Let at point D potential = 0 and write the potential of other

10
points then current in wire AD = = 5 A from A to D current
2

20
in wire CB = = 4A from C to F
5
 current in wire BD = 1 A from D to B

Example 8. Find the current in each wire

Solution : Let potential at point A is 0 volt then potential of other points is shown in figure.

40  0
current in BG =  40 A from G to B
1

KVPY_ SA STREAM # 421


0  ( 30 )
current in FC =  15 A from C to F
2

0  ( 40 )
current in DE =  20 A from D to E
2
current in wire AH = 40 – 35 = 5 A from A to H


10. KI RCHHOFF'S LAW S
1 2 1 - Kirchhof f’s Curre nt La w (Junct ion la w)
This law is based on law of conservation of charge. It states that " The algebraic sum of the
currents meeting at a point of the circuit is zero " or total currents entering a junction equals
total current leaving the junction.
 in = out.
It is also known as KCL (Kirchhoff's current law).

Example 9. Find relation in between current i 1, i 2, i 3, i 4, i 5 and i 6 .

Solution : i 1 + i 2 – i 3 – i 4+ i 5 + i 6 = 0
Example 10. Find the current in each wire

Solution :

KVPY_ SA STREAM # 422


Let potential at point B = 0. Then potential at other points are mentioned.
Potential at E is not known numerically.
Let potential at E = x
Now applying kirchhoff’s current law at junction E. (This can be applied at any other junction
also).

x  10 x  30 x  14
  0
1 2 2
4x = 36  x = 9

10  9
Current in EF = = 1 A from F to E
1
30  9
Current in BE =  10.5 A from B to E
2
9  ( 14 )
Current in DE =  11 .5 A from E to D
2

Example 11. Find the potential at point A

Solution : Let potential at A = x, applying kirchhoff current law at juction A


x  20  10 x  15  20 x  45 x  30
   0
1 2 2 1
2x  60  x  35  x  45  2x  60
 0
2
 6x + 10 = 0  x = –5/3
5
Potential at A = V
3

11. Kirchhof f ’s Volt a ge La w (Lo op la w) :
“The algebraic sum of all the potential differences along a closed loop is zero.
So R + EMF = 0”.
The closed loop can be traversed in any direction.
While traversing a loop if potential increases, put a
positive sign in expression and if potential decreases
put a negative sign. Assume sign convention)
 V1  V2 + V3  V4 = 0.
Boxes may contain resistor or battery or any other element (linear or nonlinear).
It is also known as KVL

KVPY_ SA STREAM # 423


Example 12. Find current in the circuit

Solution :  all the elements are connected in series


current is all of them will be same
let current = i
Applying kirchhoff voltage law in ABCDA loop
10 + 4i – 20 + i + 15 + 2i – 30 + 3i = 0
10 i = 25
i = 2.5 A

Example 13. Find the current in each wire applying only kirchhoff voltage law

Solution : Applying kirchhoff voltage law in loop ABEFA


i 1 + 30 + 2 (i 1 + i 2) – 10 = 0
3l 1 + 2l 2 + 20 = 0 -------------- (i)
Applying kirchoff voltage law in BEDCB
+ 30 + 2(i 1 + i 2) + 50 + 2i 2 = 0
4i 2 + 2i 1 + 80 = 0
2i 2 + i 1 + 40 = 0 -------------- (ii)
Solving (i) and (ii)
3  40  2i2  + 2i 2 + 20 = 0
– 120 – 4i 2 + 20 = 0
i 2 = –25 A
and i 1 = 10 A
 i 1 + i 2 = – 15 A
current in wire AF = 10 A from A to F
current in wire EB = 15 A from B to E
current in wire DE = 25 A from E to D.

12. COM BI N ATI ON OF RESI STAN CES :
A number of resistances can be connected and all the complicated combinations can be reduced to
two different types, namely series and parallel.

V
The equivalent resistance of a combination is defined as R eq =
i

KVPY_ SA STREAM # 424


1 2 .1 Re sist a nce s in Serie s:
When the resistances (or any type of elements) are connected end to end then they are
said to be in series. The current through each element is same.

Resistances in series carry equal current but reverse may not be true.

Example 14. Which electrical elements are connected in series.

Solution : Here S1, S2, R1, R3 connected in one series and R 4, S3 connected in different series


Equiva le nt of Re sist ors :

The effective resistance appearing across the battery (or between the terminals A and B) is
R = R1 + R2 + R3 +................ + Rn (this means Req is greater then any resistor)
and V = V1 + V2 + V3 +................ + V n .
The potential difference across a resistor is proportional to the resistance. Power in each resistor is
also proportional to the resistance
 V = R and P = 2R
where  is same through any of the resistor.
R1 R2
V1 = V ; V2 = V ; etc
R1  R 2  .........  R n R1  R 2  .........  R n

Example 15. Find the current in the circuit

Solution : Req = 1 + 2 + 3 =  the given circuit is equivalent to

v 30
current i = R  6 = 5 A Ans.
eq

KVPY_ SA STREAM # 425


Example 16. In the figure shown B 1, B2 and B3 are three bulbs rated as (200V, 50 W), (200V, 100W) and
(200 V, 25W) respectively. Find the current through each bulb and which bulb will give more
light?

200 2
Solution : R1 = ;
50

200 2 200 2
R2 = ; R3 =
100 25
the current following through each bulb is

200 200 100 1


= R R R = =  A
 2  1  4  200  7 14
1 2 3
200 2  
 100 

Since R3 > R1 > R2


 Power consumed by bulb = i 2R
 if the resistance is of higher value then it will give more light.
 Here Bulb B3 will give more light.


13 Re si st a n ce s in P a ra l le l :
A parallel circuit of resistors is one in which the same voltage is applied across all
the components in a parallel grouping of resistors R 1, R2, R3,........, Rn.

(a) (b)

In the figure (a) and (b) all the resistors are connected between points A and B so they are
in parallel.
Equ iva le n t re sist a nce :

Applying kirchhoff’s junction law at point P


i0 = i 1 + i2 + i 3
KVPY_ SA STREAM # 426
V V V V
Therefore,  
R eq = R1 R 2 R 3

in general,

Conclusions: (about parallel combination)


(a) Potential difference across each resistor is same.
(b) I = I 1 + I 2 + I 3 +.......... I n .

1 1 1 1 1
(c) Effective resistance (R) then R  R  R  R  ........  R .
1 2 3 n

(R is less than each resistor).


(d) Current in different resistors is inversely proportional to the resistance.

1 1 1 1
I 1: I 2:........... I n = : : : ........ : .
R1 R 2 R 3 Rn

G1 G2
I1 = l, I 2 = l, etc.
G1  G2  .........  Gn G1  G 2  .........  Gn

1
where G = = Conductance of a resistor. [Its unit is 1 or (mho)]
R

Example 17. When two resistors are in parallel combination then determine i 1 and i 2, if the combination
carries a current i ?
Solution :  i 1R1 = i 2R2

i1 R 2
or 
i2 R1

R2 i R1 i
i1 = R  R  i2 = R  R ,
1 2 1 2

1
Note : Remember this law of i  in the resistors connected in parallel. It can be used in problems.
R

Example 18. Find current passing through the battery and each resistor.

Solution : Method () :


It is easy to see that potential difference across each resistor is 30 V.

KVPY_ SA STREAM # 427


30 30
 current is each resistors are = 15 A, 30  10 A and  5A
2 3 6
 Current through battery is = 15 + 10 + 5 =30 A.
Method () :

V 1 1 1 1
By ohm’s law i = R   
eq R eq = 2 3 6 = 1

30
Req = 1   i= = 30 A
1
Now distribute this current in the resistors in their inverse ratio.

Current total in 3  and 6  is 15 A it will be divided as 10 A and 5 A.

N ote : The method () is better. But you will not find such an easy case every where.

Exercise 19. Find current which is passing through battery.

Solution : Here potential difference across each resistor is not 30 V


 battery has internal resistance. Here the concept of combination of resistors is useful.
Req = 1 + 1 = 2 

30
i= = 15 A.
2

Example 20. Find equivalent Resistance

Solution :

Here all the Resistance are connected between the terminals A and B
Modified circuit is

R
So Req =
3

KVPY_ SA STREAM # 428


Example 21. Find the current in Resistance P if voltage supply between A and B is V volts

3R
Solution : Req =
5

5V
I= Modified circuit
3R

5V
R
Current in P = 3R
1.5R  R

2V
=
3R
Example 22. Find the current in 2 resistance

Solution : 2, 1in series = 3

18
3, 6in parallel = =2
9
2, 4in series = 6
6, 3in parallel = 2
Req = 4 + 4 + 2 = 10

120
i= = 12A
10

8
So current in 2 Resistance = A
3


14. W HEATSTON E N ETW ORK : (4 TERM I N ALN ETW ORK)

KVPY_ SA STREAM # 429


The arrangement as shown in figure, is known as Wheat stone bridge
Here there are four terminals in which except two all are connected to each other through resistive
elements.
In this circuit if R1 R3 = R2 R4 then VC = VD and current in R5 = 0 this is called balance point or null point
P R
When current through the galvanometer is zero (null point or balance point) = , then PS = QR 
Q S
Here in this case products of opposite arms are equal. Potential difference between C and D at null
point is zero. The null point is not affected by resistance R 5, E and R. It is not affected even
if the positions of Galvanometer and battery (E) are interchanged.
hence, here the circuit can be assumed to be following,

Q P Q
P

G or

R S R S

Example 23. Find equivalent resistance of the circuit between the terminals A and B.

Solution : Since the given circuit is wheat stone bridge and it is in balance condition.

 10×3 = 30 = 6×5

hence this is equivalent to


16  8 16
Req =  
16  8 3

Example 24.

Find (a) Equivalent resistance (b) and current in each resistance


1
 1 1 1 
Solution : (a) Req =     1 5
 16 8 16 

60
(b) i= = 12 A
4 1
Hence 12 A will flow through the cell.
By using current distribution law.
Current in resistance 10 and 6 = 3A
Current in resistance 5 and 3 = 6A
Current in resistance 20 = 0
Current in resistance 16 = 3A
KVPY_ SA STREAM # 430
Example 25. Find the equivalent resistance between A and B

Solution : This arrangement can be modified as shown in figure


since it is balanced wheat stone bridge

2R  2R
Req = = R
2R  2R
Example 26. Determine the value of R in the circuit shown in figure, when the current is zero in the branch CD.

Solution : The current in the branch CD is zero, if the potential difference across CD is zero.
That means, voltage at point C = voltage at point D.
Since no current is flowing, the branch CD is open circuited. So the same voltage is applied
across ACB and ADB
10 R
V10 = V ×  VR = V ×
15 20  R
10 R
 V10 = VR and V× =V ×
15 20  R
 R = 40  Ans.

15. SYM M ETRI CAL CI RCUI TS :
Some circuits can be modified to have simpler solution by using symmetry if they are solved by
traditional method of KVL and KCL then it would take much time.

Example 27. Find the equivalent Resistance between A and B

Solution :  Method :

KVPY_ SA STREAM # 431


Here no two resistors appear to be in series or parallel no Wheatstone bridge here. This

V
circuit will be solved by using R eq = . The branches AC and AD are symmetrical

 current through them will be same.


The circuit is also similar from left side and right side current distribution while entering
through B and an exiting from A will be same. Using all these facts the currents are as
shown in the figure. It is clear that current in resistor between C and E is 0 and also in ED
is 0. It’s equivalent as shown in figure (b)

2R
Req =
3
 Method
 The potential difference in R between (B, C) and between (B.D.) is same V C = VD
Hence the point C and D are same hence circuit
can be simplified as
this called folding.
Now, it is Balanced Wheatstone bridge

2R  R 2R
Req = 
2R  R 3

N ot e : In II Method it is not necessary to know the currents in CA and DA.

Example 28. Find the equivalent Resistance between A and B

Solution : In this case the circuit has symmetry in the two branches AC and AD at the input
 current in them are same but from input and from exit the circuit is not similar
( on left R and on right 2R)
 on both sides the distribution of current will not be similar.
Here Vc = Vd

KVPY_ SA STREAM # 432


hence C and D are same point
the circuit can be simplified that as shown
Now it is balanced wheat stone bridge
3R
3R 
2
Req =
3R
3R 
2
9
R
2
= = R.
9
2
Example 29. Find the equivalent Resistance between A and B

Solution :

Here VA = VC and VB = VD

Here the circuit can be simplified as

this circuit can be simplified as

4R
2R 
Req = 3
10R
3

4R
= Ans.
5

Example 30. Find the equivalent Resistance between A and B

5 10

A 5 B

10 5

KVPY_ SA STREAM # 433


Solution : It is wheat stone bridge but not balanced. No series parallel connections. But similar values
on input side and output. Here we see that even after using symmetry the circuit does not
reduce to series parallel combination as in previous examples.

 applying kirchoff voltage law


v – 10(i – x) – 5x = 0
v – 10i + 5x = 0 ...(1)
10(i – x) – 5 (2x – i) – 5x = 0
10i – 10x – 10 x + 5i – 5x = 0
15i – 25 x = 0

15
x = i 5x = 3i ....(2)
25
Using (2) and (1)
 v – 10 i + 3i = 0

v
= 7
i
Req = 7 Ans.


16. GROUPIN G OF CELLS
1 6 .1 Cells in Series :

Equivalent EMF
Eeq = E1  E2  .......  En [write EMF's with polarity]
Equivalent internal resistance
req = r1  r2  r3  r4  ....  rn
If n cells each of emf E, arranged in series and if r is internal resistance of each cell, then
total emf = n E so current in the circuit

nE
=
R  nr

nE
If nr << R then  =  Series combination is advantageous.
R

E
If nr >> R then  =  Series combination is not advantageous.
r

KVPY_ SA STREAM # 434


N ot e : If polarity of m cells is reversed, then equivalent emf = (n-2m)E while the equivalent resistance
is still nr+R, so current in R will be
(n  2m)E
i .
nr  R

Example 31. Find the current in the loop.

Solution : The given circuit can be simplified as


35 35
i = =
10  5 15
7 7
= A  = A
3 3
1 6 .2 Cells in Pa ra llel :

1  
 2  ....  n
r1 r2 rn
Eeq  [Use emf’s with polarity]
1  1  .....  1
r1 r2 rn

1 1 1 1
   .... 
req r1 r2 rn

If m cells each of emf E and internal resistance r be connected in parallel and if this
combination is connected to an external resistance then equivalent emf of the circuit = E.
r
Internal resistance of the circuit = .
m

E mE
and = = .
R r
m mR  r

mE
If mR << r ; I=  Parallel combination is advantageous.
r

E
If mR >> r ; I=  Parallel combination is not advantageous.
R
1 6 .3 Cells in M ultiple Arc :
mn = number of identical cells.
n = number of rows
m = number of cells in each row.
The combination of cells is equivalent to single cell of
emf = mE
mr
and internal resistance =
n

KVPY_ SA STREAM # 435


mE
Current I =
R  mr
n

For maximum current nR = mr

mr
or R= = internal resistance of the equivalent battery..
n

nE mE
max =  .
2r 2R

Example 32. Find the emf and internal resistance of a single battery which is equivalent to a combina-
tion of three batteries as shown in figure.

10V 2
6V 1

4V 2

Solution : 10V 2
6V 1
B

A
C 2
4V

Battery (B) and (C) are in parallel combination with opposite polarity. So, their equivalent

10 4

2 2 52
BC = = = 3V  rBC = 1.
1 1 1

2 2

6V 3V
1 1
Now,

ABC = 6 – 3 = 3V
rABC = 2. Ans.


17. GALVAN OM ETER
Galvanometer is represented as follow :

It consists of a pivoted coil placed in the magnetic field of a permanent magnet. Attached to the coil
is a spring. In the equilibrium position, with no current in the coil, the pointer is at zero and spring is
relaxed. W hen there is a current in the coil, the magnetic field exerts a torque on the coil that is
proportional to current. As the coil turns, the spring exerts a restoring torque that is proportional to
the angular displacement. Thus, the angular deflection of the coil and pointer is directly proportional
to the coil current and the device can be calibrated to measure current.
When coil rotates the spring is twisted and it exerts an opposing torque on the coil.
There is a resistive torque also against motion to damp the motion. Finally in equilibrium
 magnetic = spring  BINA sin  = C
But by making the magnetic field radial  = 90º.

KVPY_ SA STREAM # 436


 BINA = C 

here B = magnetic field A = Area of the coil
 Current C = torsional constant
N = Number of turns  = angle rotate by coil.
 Current sensitivity
The ratio of deflection to the current i.e. deflection per unit current is called current sensitivity
 BNA
(C.S.) of the galvanometer CS = =
 C

N ot e :
Shunting a galvanometer decreases its current sensitivity.
A linear scale is obtained. The marking on the galvanometer
are proportionate.
The galvanometer coil has some resistance represented by R g . It is of the
order of few ohms. It also has a maximum capacity to carry a current known
as g . g is also the current required for full scale deflection. This galvanometer
is called moving coil galvanometer.


18. AM M ETER
A shunt (small resistance) is connected in parallel
with galvanometer to convert it into ammeter; An ideal
ammeter has zero resistance
Ammeter is represented as follow -

If maximum value of current to be measured by ammeter is  then


G . RG = ( – G )S

 G .R G
S =
  G

 G  RG
S = when  >> G.

where  = Maximum current that can be measured using the given ammeter.
For measuring the current the ammeter is connected is series.
In calculation it is simply a resistance

Resistance of ammeter

R G .S
RA =
RG  S

for S << RG  RA = S

KVPY_ SA STREAM # 437


Example 33. What is the value of shunt which passes 10% of the main current through a galvanometer of 99
ohm ?
Solution : Rg
 g
As in figure Rgg = ( – g)S G
   
 99 × =    ×S
10  10 
( – g) S
 S = 11 .
Example 34. Find the current in the circuit (a) & (b) and also determine percentage error in measuring the
current through an ammeter.

10
Solution : ln A = = 5A
2
10
ln B = = 4A
2 .5
i  i´
Percentage error is = × 100 = 20% Ans.
i
Here we see that due to ammeter the current has reduced. A good ammeter has very low
resistance as compared with other resistors, so that due to its presence in the circuit the
current is not affected.

Example 35. Find the reading of ammeter ? Is this the current through 6  ?

36
Solution : Req = 1 = 3 
36
Current through battery

18
= =6A
3
So, current through ammeter

6
= 6× = 4A
9
No, it is not the current through the 6  resistor.

N ot e :
 Ideal ammeter is equivalent to zero resistance wire for calculation potential difference across it is
zero.

KVPY_ SA STREAM # 438



19. VOLTM ETER
A high resistance is put in series with galvanometer. It is used to measure potential difference
across a resistor in a circuit.

For maximum potential difference


V
V =  G . R +  G RG R =  – RG
G

V
If RG << R  RS 
G

For measuring the potential difference a voltmeter is connected across that element. (parallel to
that element it measures the potential difference that appears between terminals ‘A’ and ‘B’.)
For calculation it is simply a resistance

Resistance of voltmeter RV = RG + R  R

Vo
g = . R  Ideal voltmeter.
R g R

A good voltmeter has high value of resistance.


Ideal voltmeter  which has high value of resistance.

Note :
 For calculation purposes the current through the ideal voltmeter is zero.
V  V´
 Percentage error in measuring the potential difference by a voltmeter is = × 100
V

Example 36. A galvanometer has a resistance of G ohm and range of V volt. Calculate the resistance to be
used in series with it to extend its range to nV volt.
V
Solution : Full scale current ig =
G
to change its range
V
V1 = (G + Rs)ig  nV = (G + Rs)  Rs = G(n – 1) Ans.
G

Example 37. Find potential difference across the resistance 300  in A and B.

200 

300  V 600 
100 V

(B)
KVPY_ SA STREAM # 439
100
Solution : In (A) : Potential difference =  300 = 60 volt
200  300

100 300  600


In (B) : Potential difference = 
300  600 300  600 = 50 volt
200 
300  600

We see that by connecting voltmeter the voltage which was to be measured has changed.
Such voltmeters are not good. If its resistance had been very large than 300  then it would
not have affected the voltage by much amount.

Current sensitivity
The ratio of deflection to the current i.e. deflection per unit current is called current sensitivity (C.S.) of


the galvanometer CS =

N ot e :
 Shunting a galvanometer decreases its current sensitivity.

Example 38.
A galvanometer with a scale divided into 100 equal divisions, has a current sensitivity of 10
division per mA and voltage sensitivity of 2 division per mV. What adoptions are required to use
it (a) to read 5A full scale and (b) 1 division per volt ?
 100
Solution : Full scale deflection current ig = = mA
cs 10
= 10 mA

Full scale deflection voltage Vg =
vs
100
= mv
2
= 50 mv
Vg 50mV
So galvanometer resistance G = i =
g 10mA
=5
(a) to convert the galvanometer into an ammeter of range 5A, a resistance of value S is
connected in parallel with it such that
( – ig) S = ig G
(5 – 0.01) S = 0.01 × 5
5
S=  0.01  Ans.
499
(b) To convert the galvanometer into a voltmeter which reads 1 division per volt, i.e. of
range 100 V,
V = ig (R + G)
100 = 10 × 10–3 (R + 5)
R = 10000 – 5
R = 9995   9.995 k Ans.

KVPY_ SA STREAM # 440


Problem 1. Current is flowing from a conductor of non-uniform cross
section area if A1 > A2 then find relation between

(a) i 1 and i 2

(b) j 1 and j 2

(c) v 1 and v 2 (drift velocity)


where i is current, j is current density and V is drift velocity.

Answer : i 1 = i 2 , V1 < V2 , J1 < J2

Solution : (a) i = charge flowing through a cross-section per unit time.


 i1 = i2

i
(b) j =
A

as A1 > A2 then j 1 < j 2

(c) j = nev d

j
vd =
ne

as j 1 < j 2 then, v 1 < v 2

Problem 2. Find the equivalent Resistance between A and B

Solution :

Putting A out of the structure in the same plane

2R  2R
Req = = R Ans. : Req = R
2R  2R

KVPY_ SA STREAM # 441


Problem 3. What shunt resistance is required to convert the 1.0 mA, 20 galvanometer into an ammeter
with a range of 0 to 50mA ?
20
Answer : S= = 0.408 
49
Solution : ig Rg = (i – ig)S
ig = 1.0 × 10–3 A , G = 20
i = 50 × 10–3 A
igR g 1 10 3  20
S = i– v = = 0.408 
g 49 10 3

Problem 4. How can we convert a galvanometer with Rg = 20  and ig = 1.0 mA into a voltmeter with a
maximum range of 10 V ?

Answer : A resistance of 9980  is to be connected in series with the galvanometer.


Solution : v = ig RS + ig Rg
10 = 1 × 10–3 × Rs + 1 × 10–3 × 20

10 – 0.02 9.98
RS = = = 9980 
1  10 3
10 3

Problem 5. A Potentiometer wire of 10 m length and having 10 ohm resistance, emf 2 volts and a rheostat. If
the potential gradient is 1 micro volt/mm, the value of resistance in rheostat in ohms will be :
(A) 1.99 (B) 19.9 (C) 199 (D) 1990
Solution : d = 10 m , R = 10 ,
dv
E = 2volts , = 1µ v/mm
d

dv 1 10 6
= v/m = 1 × 10–3 v/m
d 1 10 3
Across wire potential drop ,
dv
×  = 1 × 10–3 × 10 = 0.01 volts
d
0.01 E
i= = 0.001 = (R’ = resistance of rheostat)
10 R  R'
E 2
R’ = R =  10 = 2000 – 10 = 1990  Answer : (D)
0.001 0.001

KVPY_ SA STREAM # 442


OBJECTIVE QUESTIONS
1. In the circuit shown, each resistances is 2 . The potential V 1 as indicated in the circuit, is equal to

(A) 11 V (B) – 11V (C) 9 V (D) – 9 V

2. In the circuit shown, the value of R in ohm that will result in no current through the 30 V battery, is :

R
20

10
(A) 10  (B) 25  (C) 30  (D) 40 

3. The maximum current in a galvanometer can be 10 mA. It’s resistance is 10. To convert it into an
ammeter of 1 Amp. a resistor should be connected in
(A) series, 0.1 (B) parallel, 0.1 (C) series, 100  (D) parallel, 100.

4. When a galvanometer is shunted with a 4 resistance, the deflection is reduced to one - fifth. If the
galvanometer is further shunted with a 2 wire, the further reduction (find the ratio of decrease in
current to the previous current) in the deflection will be (the main current remains the same).
(A) (8/13) of the deflection when shunted with 4 only
(B) (5/13) of the deflection when shunted with 4 only
(C) (3/4) of the deflection when shunted with 4 only
(D) (3/13) of the deflection when shunted with 4 only

5. In the figure shown the current flowing through 2 R is :

(A) from left to right (B) from right to left (C) no current (D) None of these

6. In a practical wheat stone bridge circuit as shown, when one


more resistance of 100  is connected is parallel with unknown
resistance ' x ', then ratio 1/ 2 become ' 2 '. 1 is balance length.
AB is a uniform wire. Then value of ' x ' must be:

(A) 50  (B) 100 

(C) 200  (D) 400 

KVPY_ SA STREAM # 443


7. A battery of internal resistance 2  is connected to a variable resistor whose value can vary from 4  to 10
 . The resistance is initially set at 4  If the resistance is now increased then
(A) power consumed by it will decrease
(B) power consumed by it will increase
(C) power consumed by it may increase or may decrease
(D) power consumed will first increase then decrease.

8. A cell of emf E having an internal resistance ' r ' is connected to an


external resistance R. The potential difference ' V ' across the resistance
R varies with R as shown by the curve:
(A) A (B) B
(C) C (D) D

9. ' n' identical light bulbs, each designed to draw P power from a certain voltage supply are joined in
series and that combination is connected across that supply. The power consumed by one bulb will be-
(A) n P (B) P (C) P/n (D) P/n2
10. To get maximum current through a resistance of 2.5  , one can use ' m ' rows of cells, each row having
' n ' cells. The internal resistance of each cell is 0.5  . What are the values of n & m, if the total number
of cells is 45.
(A) 3, 15 (B) 5, 9 (C) 9, 5 (D) 15, 3
11. 50 V battery is supplying current of 10 amp when connected to a resistor. If the efficiency of battery at
this current is 25%. Then internal resistance of battery is :
(A) 2.5  (B) 3.75  (C) 1.25  (D) 5 
12. A battery is supplying power to a tape-recorder by cable of resistance of 0.02 . If the battery is
generating 50 W power at 5V, then power received by tape-recorder is : (neglect internal resistance of
battery)
(A) 50 W (B) 45 W (C) 30 W (D) 48 W
13. In the shown circuit the resistance R can be varied :

The variation of current through R against R is correctly plotted as :

(A) (B) (C) (D)

14. Specific resistance of a wire depends on its


(A) mass (B) length (C) area of cross–section (D) None of the above

15. The electric current passes through a metallic wire produces heat because of
(A) collisions of conduction electrons with each other
(B) collisions of the atoms of the metal with each other
(C) the energy released in the ionization of the atoms of the metal
(D) collisions of the conduction electrons with the atoms of the metallic wire

KVPY_ SA STREAM # 444


V
16. For a metallic wire, the ratio (V = applied potential difference and i = current flowing) is
i
(A) independent of temperature
(B) increases as the temperature rises
(C) decreases as the temperature rises
(D) increases or decreases as temperature rises depending upon the metal

17. If on applying the potential of 20 V on a conductor its conductance becomes 8 ()–1, then the current flowing
through it will be
(A) 120 A (B) 160 A (C) 90 A (D) 80 A

18. Two wires of same dimension but resistivities 1 and 2 are connected in series. The equivalent resistivity of
the combination is
(A) 1 + 2 (B) 1/2 (1 + 2) (C) 1  2 (D) 2(1 + 2)

19. The dimensions of a block are 1 cm x 1 cm x 100 cm. If the specific resistance of its material is
2  10 7 ohm  metre , then the resistance between the opposite rectangular faces is
(A) 2  10 9  (B) 2  10 7  (C) 2  10 5  (D) 2  10 3 

20. When a resistance wire is passed through a die the cross–section area decreases by 1%, the change in
resistance of the wire is
(A) 1% decrease (B) 1% increase (C) 2% decrease (D) 2% increase

21. When the resistance of copper wire is 0.1  and the radius is 1 mm, then the length of the wire is (specific
resistance of copper is 3.14 × 10–8 ohm x m)
(A) 10 cm (B) 10 m (C) 100 m (D) 100 cm

22. A wire is stretched to n times its length. Then the resistance now will increase by
(A) n times (B) 1/n times (C) n2 times (D) 1/n2 times
23. The specific resistance of a conductor increases with :
(A) Increase in temperature (B) Increases in cross-sectional area
(C) Decreases in length (D) Decrease in cross-sectional area

24. The value of current I in the circuit will be

(A) 1.7 A (B) 2.1 A (C) 3 A (D) zero


25. Kirchoff’s I law and II law of current, proves the
(A) Conservation of charge and energy (B) Conservation of current and energy
(C) Conservation of mass and charge (D) None of these

26. Three wires each have resistance 2  , if we connect 2 in series with one parallel to the combination the
equalent resistance is
(A) 4/3  (B) 3/4  (C) 6  (D) 3 

27. In the shown circuit, what is the potential difference across A and B
20V

A B

(A) 50 V (B) 45 V (C) 30 V (D) 20 V

KVPY_ SA STREAM # 445


28. In the circuit element given here, if the potential at point B, Vn = 0, then the potentials of A and D are given as

(A) VA = – 1.5 V, VD = + 2 V (B) VA = + 1.5 V, VD = + 2 V


(C) VA = + 1.5 V, VD = + 0.5 V (D) VA = + 1.5 V, VD = – 0.5 V

29. A cell has an emf 1.5V. When connected across an external resistance of 2, the terminal potential differ-
ence falls to 1.0V. The internal resistance of the cell is :
(A) 2 (B)1.52 (C) 1.0 (D) 0.5

30. Two bulbs 25W, 220V and 100W, 220V are given. Which has higher resistance ?
(A) 25W bulb (B) 100 W bulb
(C) Both bulbs will have equal resistance (D) Resistance of bulbs cannot be compared

31. In the figure the potential difference across 6 ohm resistor is 48 V. Then the potential difference between A
and B is

(A) 48 V (B) 102 V (C) 204 V (D) Can’t be determined


32. The total power dissipated in watts in the circuit shown here is :-

(A) 4W (B) 16W (C) 40W (D) 54W


33. A student measures the termainal potential diffrerence (V) of a cell (of emf  and internal resistance r) as a
function of the current (I) flowing through it. The slope and intercept of the graph between V and I, then
respectively. equal
(A)  and –r (B) –r and  (C) r and – (D) – and r

34. One filament takes 10 min to heat a kettle and another takes 15 min. If connected in parallel they combindly
take.............. min to heat the same kettle :
(A) 6 (B) 12.5 (C) 25 (D) 7.5

35. The potential difference between points A and B is :

20 40 10
(A) V (B) V (C) V (D) zero
7 7 7

KVPY_ SA STREAM # 446


36. The potential difference between the points A and B in figure will be

2 8 4
(A) V (B) V (C) V (D) 2 V
3 9 3
37. Nine resistors each of resistance R are connected in the circuit as shown in fig. The net resistance between
A and B is

7R 3R 2R
(A) R (B) (C) (D)
6 5 9

38. A wire has a resistance 12 ohms. if it is bent in the form of a equilateral triangle. The resistance between any
two terminals in ohms is:
(A) 8/3 (B) 3/4 (C) 4 (D) 3

39. For the network of resistance shown in the figure the equivalent resistance of the network between the points
A and B is 18 ohm. The value of unknown resistance R is:-

(A) 8 (B) 10 (C) 16 (D) 24

40. When a wire of uniform cross-section a, length  and resistance R is bent into a complete circle, resistance
between two of diametrically opposite points will be :
R R R
(A) (B) (C) 4R (D)
4 8 2

KVPY PROBLEMS (PREVIOUS YEARS)


1. The circuit shown has 3 identical light bulbs A, B, C and 2 identical
batteries E1, E2 . When the switch is open, A and B glow with C
E1 A
equal brightness. When the switch is closed: (KVPY/2007)
(A) A and B will maintain their brightness and C will be dimmer than A and B.
S
(B) A and B will become dimmer and C will be brighter than A and B.
E2 B
(C) A and B will maintain their brightness and C will not glow.
(D) A, B and C will be equally bright.

KVPY_ SA STREAM # 447


2. A battery or 10 V and negligible internal resistance is connected across the diagonally opposite corner of a
cubical network consisting of 12 resistors each of resistance 1 . The total current 1 in the circuit external to
the network is : (KVPY/2007)

10V

(A) 0.83 A (B) 12 A (C) 1 A (D) 4 A

3. We are given n resistors, each of resistance R. The ratio of the maximum to minimum resistance that can be
obtained by combining them is : (KVPY/2008)
(A) nn
(B) n (C) n 2
(D) logn

4. Consider the circuit below. The bulb will light up if : (KVPY/2009)


S1

S2

~ S3

(A) S1 S2 and S3 are all closed. (B) S1 is closed but S2 and S3 are open.
(C) S1 and S3 are closed but S2 is open. (D) none of these
5. Two bulbs, one of 200W and the other of 100W, are connected in series with a 100 V battery which has no
internal resistance. Then, 100V (KVPY/2009)

200W 100W

(A) the current passing through the 200W bulb is more than that through the 100W bulb.
(B) the power dissipation in the 200W bulb is more than that In the 100 W bulb.
(C) the voltage drop across the 200W bulb is more than that across the 100W bulb.
(D) the power dissipation In the 100W bulb is more than that in the 200W bulb.
6. Figure (a) below shows a Wheatstone bridge in which P, Q,
R, S are fixed resistances, G is a galvanometer and B is a
battery. For this particular case the galvanometer shows zero
deflection. Now, only the positions of B and G are
interchanged,. as shown in figure (b). The new deflection of
the galvanometer. (KVPY 2010/ SA)

(A) is to the left. (B) is to the right.


(C) is zero. (D) depends on the values of P, Q, R, S

7. An electric heater consists of a nichrome coil and runs under 220 V, consuming 1 kW power. Part of its coil
burned out and it was reconnected after cutting off the burnt portion. The power it will consume now is :
(KVPY 2010/ SA)
(A) more than 1 kW. (B) less that 1 kW, but not zero.
(C) 1 kW. (D) 0 kW.

KVPY_ SA STREAM # 448


8. Figure below shows a portion of an electric circuit with the currents in ampreres and their directions. The
magnitude and direction of the current in the portion PQ is : (KVPY 2011/ SA)

(A) 0A (B) 3A from P to Q (C) 4A from Q to P (D) 6A from Q to P


9. In the following circuit, the 1 resistor dissipates power P. If the resistor is
replaced by 9. the power dissipated in it is (KVPY 2011/ SA)

(A) P (B) 3P
(C) 9P (D) P/3

10. A cylindrical copper rod has length L and resistance R. If it is melted and formed into another rod of length 2L.
the resistance will be : (KVPY 2011/ SA)
(A) R (B) 2R (C) 4R (D) 8R
11. Following figures show different combinations of identical bulb(s) connected to identical battery(ies). Which
option is correct regarding the total power dissipated in the circuit ? (KVPY 2012/ SA)

(A) P < Q < R < S (B) R < Q < P < S (C) P < Q < R = S (D) P < R < Q < S
12. In the circuit shown, n identical resistors R are connected in parallel (n > 1) and the combination in series to
another resistor R0. In the adjoining circuit n resistors of resistance R are all connected in series along with
R0 . (KVPY 2012/ SA)

The batteries in both circuits are identical and net power dissipated in the n resistors in both circuit is same.
The ratio R0/R is :
(A) 1 (B) n (C) n2 (D) 1/n
13. Which of the following statements is true about the flow of electrons in an electric circuit ?
(A) Electrons always flow from lower to higher potential (KVPY 2012/ SA)
(B) Electrons always flow from higher to lower potential
(C) Electrons flow from lower to higher potential except through power sources
(D) Electrons flow from higher to lower potential, except through power sources
14. An electron with an initial speed of 4.0 × 106 ms–1 is brought to rest by an electric field. The mass and charge
of an electron are 9 × 10–31 kg and 1.6 × 10–19C, respectively. Identify the correct statement
[KVPY_2013] [1 Marks]
(A) The electron moves from a region of lower potential to higher potential through a potential difference of
11.4 V.
(B) The electron moves from a region of higher potential to lower potential through a potential difference of
11.4 V.
(C) The electron moves from a region of lower potential to higher potential through a potential difference of 45 V.
(D) The electron moves from a region of higher potential to lower potential through a potential difference of 45 V

KVPY_ SA STREAM # 449


15. Consider the circuit shown in the figure below : [KVPY_2013][1Marks]

All the resistors are identical. The ratio I/I’ is


(A) 8 (B) 6 (C) 5 (D) 4
16. A current of 0.1 A flows through a 25  resistor represented by the circuit diagram. The current in the 80 
resistor is [KVPY_2013][1Marks]

(A) 0.1 A (B) 0.2 A (C) 0.3 A (D) 0.4 A


17. If the current through a resistor in a circuit increases by 3%, the power dissipated by the resistor
[KVPY_2013][1Marks]
(A) increases approximately by 3% (B) increases approximately by 6%
(C) increases approximately by 9% (D) decreases approximately by 3%
18. Consider the circuit shown below where all resistors are of 1k 
P

If a current of magnitude 1 mA flows through the resistor marked X, what is the potential difference measured
between point P and Q? [KVPY_2014_SA] [2 Marks]
(A) 21V (B) 68V (C) 55V (D) 34V
19. A light bulb of resistance R  16 is attached in series with an infinite resistor network with identical
resistances r as shown below. A 10 V battery derives current in the circuit. What should be the value of r such
that the bulb dissipated about 1 W of power. [KVPY_2015_SA] [1 Mark]

R
r r r

10V r r r

(A) 14.8 (B) 29.6 (C) 7.4 (D) 3.7


20. The primary and the secondary coils of a transformer contain 10 and 100 turns, respectively. The primary
coil is connected to a battery that supplies a constant voltage of 1.5 volts . The voltage across the secondary
coil is [KVPY_2016_SA] [1 Mark]
(A) 1.5V (B) 0.15 V (C) 0.0V (D) 15 V

KVPY_ SA STREAM # 450


21. Water falls down a 500.0 m shaft to reach a turbine which generates electricity. How much water must fall
per second in order to generate 1.00 × 109 Watts of power ? (Assume 50% efficiency of conversion and g =
10m/s2) [KVPY_2016_SA] [1 Mark]
(A) 250m3 (B) 400 m3 (C) 500m3 (D) 200m3
22. In the circuit , wire 1 is of negligible resistance . Then [KVPY_2016_SA] [1 Mark]
R1 R2
wire1
r kj 1

+ – + –
1 2

(A) Current will flow through wire 1 if 1 2


(B) Current will flow through wire 1 if 1 R1 2 /R2
(C) Current will flow through wire 1 if (1 +2) (R1+ R2)  (1 –2) (R1 – R2)
(D) No current will flow through wire 1.
23. In the circuit shown below (on the left) the resistance and the emf source are both variable. The graph of
seven readings of the voltmeter and the ammeter (V and I, respectively) for different settings of resistance
and the emf, taken at equal intervals of time t, are shown (on the right) by the dots connected by the curve
EFGH. Consider the internal resistance of the battery to be negligible and the voltmeter and ammeter to be
ideal devices. Take R0  V0/I0 [KVPY_2017_SA] [2 Mark]

Then the plot of the resistance as a function of time corresponding to the curve EFGH is given by

H R
2R0

G
F
R0 E F
(A) 4t 6t t (B) R0 H
G
E R0/2
0
t
0 2t 0 2t 4t 6t

R R

2R0 H
2R0

E F E F H
R0 R0
(C) R0/2 (D) R /2
G 0
G
0 0
0 2t 4t 6t t 0 2t 4t 6t t

KVPY_ SA STREAM # 451


24. In a particle accelerator, a current of 500 A is carried by a proton beam in which each proton has speed
of 3 × 107m/s. The cross sectional area of the beam is 1.50 mm2. The charge density in this beam in
Coulomb/m3 is close to [KVPY-SA_2018 1 Mark]
(A) 10-8 (B) 10-7 (C) 10-6 (D) 10-5

25. A steady current I is set up in a wire whose cross-sectional area decreases in the direction of the flow of the
current. Then, as we examine the narrowing region [KVPY-SA_2018 1 Mark]
(A) the current density decreases in value.
(B) the magnitude of the electric field increases.
(C) the current density remains constant.
(D) the average speed of the moving charges remains constant.

EXERCISE-1
1. (D) 2. (C) 3. (B) 4. (A) 5. (B) 6. (B) 7. (A)
8. (B) 9. (D) 10. (D) 11. (B) 12. (D) 13. (C) 14. (D)
15. (D) 16. (B) 17. (B) 18. (B) 19. (B) 20. (D) 21. (B)
22. (C) 23. (A) 24. (A) 25. (A) 26. (A) 27. (D) 28. (D)
29. (C) 30. (A) 31. (C) 32. (D) 33. (B) 34. (A) 35. (D)
36. (A) 37. (D) 38. (A) 39. (C) 40. (A)

EXERCISE-2
1. (C) 2. (B) 3. (C) 4. (B) 5. (D) 6. (C) 7. (A)
8. (D) 9. (A) 10. (C) 11. (D) 12. (A) 13. (C) 14. (D)
15. (A) 16. (C) 17. (B) 18. (D) 19. (A) 20. (C) 21. (B)
22. (D) 23. (D) 24. (D) 25. (B)

7V
1. i = 7  = 1 A.

Current flows in anticolockwise direction in the loop.


Therefore 0 – 1 × 2 – 1 × 2 – 5 = V1
V1 = – 9 V.
A C i E
50 50V
2. i=
20  R i
R
Potential drop across R = Potential drop across AB
20
50
 .R = 30  R = 30  B F
20  R 10 D i

3. G = 10 mA
G = 10
S ( – G) = GG where S is shunt is parallel
S = 0.1

4. Case I  /5
G

  
Rg × =    × 4
5  5 
S=4
 Rg = 16 

KVPY_ SA STREAM # 452


Case II 16 = Rg
 
42 G
16 1 = ( – 1)
6
 1 = /13
so decrease in current to previous current 4

 / 5   / 13 8
= =
/5 13 2

5. In figure all resistance are connected in parallel.


R B 2R R B
2R  R / 2 A
So Req = and current in all resistance flow from
2R  R / 2
positive terminal of battery (means A end) to negative terminal
of battery (means B end).

6.  wheat stone bridge is in balanced condition


100 x 100
100 100 x 100
So =
1 2 x

1

2
=2  x = 100 
1 2

7.
R
r=R
Power maximum when r = R.
So, power consumed by it will decrease. for R > r.
Er R  r  r 
8. V = E – ir = – =E  
R r  R r 
ER
V=  V = 0 at R = 0
(R  r )
V = E at R = 
so (B) is correct option.

9. Voltage across each bulb will be


V
V1 = iR = . R = (V/n)
nR

V V V2 V2
so power developed by each bulb = iV 1 = . = 2 &P=
nR n n R R
P
so power consumed by one bulb =
n2
10. For maximum current, net resistance of cells must be equal to 2.5
n (0.5)
i.e. = 2.5 .........(1)
m
& m × n = 45 .........(2)
solving, we get n = 15, m = 3
11. 50 = 10 [R + r]
R+r =5
R R
=  0.25 =
R r R r

KVPY_ SA STREAM # 453


R + r = 4R
r = 3R
5
then R = = 1.25 , and r = 3.75 .
4
12. P = V, 50 = 5 × 
 = 10 A
Power lost in cable = 2 R = 10 × 10 × 0.02 = 2W
Power supplied to T.R. = 50 W – 2 W = 48 W
13. Redrawing the given circuit diagram as shown below :
Using point potential theory,
V E V E V E
  0
r r R
2 1
 (V – E)    = 0
 r R
2 1
As  0 so V–E=0
r R
V E
So, current through R, i = = 0 whatever be the value of R.
R

14. Specific resistance depends only on the material of the wire.

15. The conduction electrons while moving towards the positive end of the conductor (responsible for the
current in conductor) collide with the atoms/ions of the conductor, which is produced as heat.
16. The resistance of metallic wire at temperature t ° C is given by
Rt = R0 (1 + t)
Where  is coefficient of expansion.
V
Hence, resistance of wire increases on increasing thetemperature. Also, from Ohm’s law, ratio of is
i
equal to R ie,
V
=R
i
V
Hence, on increasing the temperature the ratio increases.
i
1
17. Conductane =
Re sis tan ce
1
 Resistance (R) = 
8
 Current flowing through the conductor
V 20  8
I= = = 160 A
R 1

 2  10 7  1 10 2
19. R = = 2×10–7 
A 1 100  10  4

20. Volume = A = constant


A 

A L
 R S  A
R    
A R S  A
R
= 2%
R
KVPY_ SA STREAM # 454
 3.14  10 8 
21. R  0.1    =10m
A   (1 10 3 )2

A
22. A = constant  ’ =nl  A’ =
n

1 2 
R’ = = x
A' A
23. The specific resistance (resistivity) of a metallic conductor nearly increases with increasing temperature as
shown in figure. This is because, with the increases in temperature the ions of the conductor vibrate with
greater amplitude, and the collision between electrons and ions become more frequent, over all small
temperature range (up to 100ºC). The resistivity of a metal can be represented approximately by the equation
24. Accordintg to Kirchhoff’s junction rule,

Sum of incoming currents = Sum of out going currents  2 + 2 = 1 + 1.3 + I


 I = 1.7 A

26.

1 1 1 4
   Rer = 
R er 4 2 3

27. VAB = 20 VVAB = 20 V

28.

29. Internal resistance of the cell is given by


E – V 
r=   R
 V 

 1.5 – 1.0  0.5


Given E = 1.5V, V = 1.0V, R = 2  r=  ×2= × = 1.0
 1. 0  1 .0
30. Power of electric bulb
V2
P=
R
So, resistance of electric bulb
V2
R=
P
Given P1 = 25W P2 = 100W, V1 = V2 = 200V
Therefore, for same potential difference V,
1
R
P
Thus, we observe that for minimum power, resistance will be maximum and vice-versa. Hence, resistance of
25W bulb is maximum and 100W bulb is minimum.

KVPY_ SA STREAM # 455


31.

48
i  12amp
4
VAB = 17×12 = 204 V
32. The resistances of 6 and 3  are in parallel in the given circuit, their equivalent resistance is
1 1 1 1 2 1
R1
= + = =
6 3 6 2
or R1 = 2
Again, R1 is in series with 4resistance, hence
R = R1 + 4 = 2 + 4 = 6 
Thus, the total power dissipated in the circuit
V2
p=
R
Here, V = 18 V, R = 6 
(18)2
Thus, p= = 54 W
6
33. According to ohm’s law
dV
=–r
dI
and V =  if I = 0 [As V + Ir = ]
 Slope of the graph = – r and intercept = 

34. For first filament

V2 V2
H= t1  R1 = t ...(i)
R1 H 1

For second filament

V2 V2
H= t2  R2 = t ...(ii)
R2 H 2

When placed in parallel

V2 V2
H = R t2  Rp = t ...(iii)
p H P

From Eqs. (i),(ii), (iii) we get

1 1 1 H H H 1 1 1
   2
 2  2     tP = 6 min.
R P R1 R 2 V t P V t1 V t 2 t P 10 15

35. From the given figure, current through lower branch of resistance which are joined in series is

10 10
i1 =  A
43 7

Again current through upper branch of resistance which are also connected in series, si

KVPY_ SA STREAM # 456


10 10
i2 =  A
8  6 14

Now, according to the Kirchhoff’s voltage law

10 10
VB – VA = 8 × i2 – 4 × i1 = 8 × – 4 0
14 7

2
36. i1= i2 = amp.
15
2 2 2
VA + ×5 – ×5 – ×5 = VB
15 15 15
10 2
VA – VB = = V
15 3

37.

R
2R / 3.
3 2R
RAB = 2R R =
 9
3 3

1 1 1 3 8
  
38. R eq 8 4 8  Req = 3 

40. When wire is bent to form a complete circle then


R
2r = R  r =
2
R R
Resistance of each semicirle = r = =
2 2

Thus, net resistance in parallel combination of two semicircular resistances

R R R2

R
R  2 2  4 
R R R 4

2 2

KVPY_ SA STREAM # 457


GEOMETRICAL OPTICS
P ROP ERTI ES OF LI GHT

(i) Speed of light in vacuum, denoted by c, is equal to 3 × 10 8 m/s approximately.


(ii) Light is electromagnetic wave (proposed by Maxwell). It
consists of varying electric field and magnetic field.

(iii) Light carries energy and momentum.


(iv) The formula v = f is applicable to light.

(v) When light gets reflected in same medium, it suffers no change in frequency, speed and
wavelength.

(vi) Frequency of light remains unchanged


when it gets reflected or refracted.

REFLECTI ON OF LI GHT
When light rays strike the boundary of two media such as air and glass, a part of light is turned
back into the same medium. This is called Reflection of Light.
(a) Regular Reflection:
When the reflection takes place from a perfect plane surface
it is called Regular Reflection. In this case the reflected
light has large intensity in one direction and negligibly small
intensity in other directions.

(b) Diffused Reflection


When the surface is rough, we do not get a regular behaviour of light. Although at each point
light ray gets reflected irrespective of the overall nature of surface, difference is observed
because even in a narrow beam of light there are many rays which are reflected from different
points of surface and it is quite possible that these rays may move in different directions due
to irregularity of the surface . This process enables us to see an object from any position.
Such a reflection is called as diffused reflection.
For example reflection from a wall, from a news paper etc.
This is why you can not see your face in news paper and in
the wall.

La ws of Reflection
(a) The incident ray, the reflected ray and the normal at the point of incidence lie in the
same plane. This plane is called the plane of incidence (or plane of reflection).
     
This condition can be expressed mathematically as R . (  × N ) = N . (  × R ) =
     
 . ( N × R ) = 0 where  , N and R are vectors of any magnitude along incident
ray, the normal and the reflected ray respectively.

KVPY_ SA STREAM # 458


(b) The angle of incidence (the angle between normal
and the incident ray) and the angle of reflection (the
angle between the reflected ray and the normal) are
equal, i.e.
  i =  r

Specia l Ca ses :
Normal Incidence : In case light is incident normally,
i = r = 0
 = 180º
Note : We say that the ray has retraced its path.

Grazing Incidence : In case light strikes the reflecting surface tangentially,

i = r = 90
deviation, = 0º or 360º

Note : In case of reflection speed (magnitude of velocity) of light remains unchanged but in grazing
incidence velocity remains unchanged.

Example - 1 Show that for a light ray incident at an angle ‘i’ on getting reflected the angle of deviation is
 =   2i or   2i.

Solution

From figure (b) it is clear that light ray bends either by 1 anticlockwise or by 2 (= 2 – 1)
clockwise.
From figure (a) 1 =  – 2i .
 2 =  + 2i .


Obje ct a nd Im a ge
(a) Object (O) : Object is defined as point of intersection of incident rays.

O
O

Let us call the side in which incident rays are present as incident side and the side in which
reflected ( refracted) rays are present, as reflected (refracted) side.

Note : An object is called real if it lies on incident side otherwise it is called virtual.

KVPY_ SA STREAM # 459


(b) Image () : Image is defined as point of intersection of reflected rays (in case of
reflection) or refracted rays (in case of refraction).


Note : An image is called real if it lies on reflected or refracted side otherwise it is called virtual.

PLANE M IRROR
Plane mirror is formed by polishing one surface of a plane thin glass plate .It is also said to be
silvered on one side.

A beam of parallel rays of light, incident on a plane mirror will get reflected as a beam of parallel
reflected rays.

Note : Keeping the mirror fixed if the incident ray is rotated by angle  about the normal then
reflected ray rotates by same angle in the same direction of rotation

Point obje ct
Characteristics of image due to reflection by a plane mirror :
(i) Distance of object from mirror = Distance of
image from the mirror.
(ii) All the incident rays from a point object will
meet at a single point after reflection from a
plane mirror which is called image.
(iii) The line joining a point object and its image is normal to the reflecting surface.
(iv) For a real object the image is virtual and for a virtual object the image is real
(v) The region in which observer's eye must be present in order to view the image is
called field of view.

Example - 2
Figure shows a point object A and a plane mirror MN. Find
the position of image of object A, in mirror MN, by drawing
ray diagram. Indicate the region in which observer’s eye must
be
present in order to view the image. (This region is called field of view).

Solution See figure, consider any two rays


emanating from the object. N1 and
N2 are normals ;
i1 = r1 and i 2 = r2

KVPY_ SA STREAM # 460


The meeting point of reflected rays R 1 and R2 is image A’. Though
only two rays are considered it must be understood that all rays
from A reflect from mirror such that their meeting point is A´. To
obtain the region in which reflected rays are present, join A´ with
the ends of mirror and extend. The following figure shows this
region as shaded. In figure, there are no reflected rays beyond the
rays 1 and 2, therefore the observers P and Q cannot see the
image because they do not receive any reflected ray.

Ext ende d obje ct :
An extended object like AB shown in figure is a combination of
infinite number of point objects from A to B. Image of every point
object will be formed individually and thus infinite images will be
formed. A´ will be image of A, C´ will be image of C, B´ will be
image of B etc. All point images together form extended image.
Thus extended image is formed of an extended object.
Properties of im a ge of a n extended object, form ed by a pla ne m irror :
(1) Size of extended object = Size of extended image.
(2) The image is erect, if the extended object is placed parallel to the mirror.

(3) The image is inverted if the extended object lies perpendicular to the plane mirror.

(4) If an extended horizontal object is placed infront of a mirror inclined 45° with the horizontal,
the image formed will be vertical. See figure.

(5) Minimum size of a plane mirror, required to see the full image of an observer is half the
size of that observer.

KVPY_ SA STREAM # 461



SP H ERI CAL M I RR ORS
Spherical Mirror is formed by polishing one surface of a part of
sphere. Depending upon which part is shining the spherical mirror is
classified as (a) Concave mirror, if the side towards center of
curvature is shining and (b) Convex mirror if the side away from the
center of curvature is shining.

Im port a nt te rm s re la te d with spherica l m irrors :


A
R

P C
B

A spherical shell with the center


of curvature, pole aperture and
radius of curvature identified
(a) Center of Curvature (C) :
The center of the sphere from which the spherical mirror is formed is called the center of
curvature of the mirror. It is represented by C and is indicated in figure.
(b) Pole (P) :
The center of the mirror is called as the Pole. It is represented by the point P on the mirror
APB in figure.
(c) Principal Axis :
The Principal Axis is a line which is perpendicular to the plane of the mirror and passes
through the pole. The Principal Axis can also be defined as the line which joins the Pole to
the Center of Curvature of the mirror.
(d) Aperture (A) :
The aperture is the segment or area of the mirror which is available for reflecting light. In
figure. APB is the aperture of the mirror.
(e) Principle focus (F) :
It is the point of intersection of all the reflected rays for which the incident rays strike the
mirror (with small aperture) parallel to the principal axis. In concave mirror it is real and in
the convex mirror it is virtual. The distance from pole to focus is called focal length.

Concave mirror

Convex mirror

KVPY_ SA STREAM # 462



Ra y tra cing :
Following facts are useful in ray tracing.
(i). If the incident ray is parallel to the principle axis,
the reflected ray passes through the focus. F

(ii). If the incident ray passes through the focus, then the reflected ray is parallel to the
principle axis.
(iii). Incident ray passing through centre of curvature will be reflected back through the
centre of curvature (because it is a normally incident ray).

(iv). It is easy to make the ray tracing


of a ray incident at the pole as
shown in below.

Sign Conve nt ion


We are using co–ordinate sign convention.
(i) Take origin at pole (in case of mirror )or at optical centre (in case of lens)
Take X axis along the Principal Axis ,taking positive direction along the incident light.
u, v, R and f indicate the x coordinate of object, image, centre of curvature and
focus respectively.
(ii) y-coordinates are taken positive above Principle Axis and negative below Principle Axis’
h 1 and h 2 denote the y coordinates of object and image respectively.

Note :
 This sign convention is used for reflection from mirror, reflection through flat or curved
surfaces or lens.

Form ula f or Ref le ction from sphe rica l m irrors :

1 1 2 1
(a) Mirror formula : + = =
v u R f
X-coordinate of centre of curvature and focus of concave mirror are negative and those for
convex mirror are positive. In case of mirrors since light rays reflect back in X-direction,
therefore -ve sign of v indicates real image and +ve sign of v indicates virtual image.

KVPY_ SA STREAM # 463


Example - 3
Figure shows a spherical concave mirror with its
pole at (0, 0) and principal axis along x axis.
There is a point object at (–40 cm, 1cm), find
the position of image.

Solution According to sign convention,


u = –40 cm
h1 = +1 cm
f = – 5 cm.
1 1 1 1 1 1 40 h 2 v
     ;v = cm. ; h  u
v u f v  40  5 7 1

 40 
   1
v  7  1
 h2 = – × h1 = =  cm.
u  40 7

  40 1 
 The position of image is  cm, cm 
 7 7 
Example - 4
Converging rays are incident on a convex spherical mirror so that their extensions intersect
30 cm behind the mirror on the optical axis. The reflected rays form a diverging beam so
that their extensions intersect the optical axis 1.2 m from the mirror . Determine the focal
length of the mirror.
Solution In this case u = + 30
 v = + 120

1 1 1 1 1
 = + = +
f v u 120 30
f = 24 cm
Example - 5
Find the position of final image after three successive
reflections taking first reflection on m 1.

Solution  reflection :
Focus of mirror = – 10 cm  u = – 15 cm
Applying mirror formula :

1 1 1
   v = – 30 cm.
v u f
For  reflection on plane mirror :
u = – 10 cm  v = 10 cm
For  reflection on curved mirror again :
u = – 50 cm
f = – 10 cm
Applying mirror formula :

1 1 1
 
v u f
v = – 12.5 cm.

KVPY_ SA STREAM # 464



h2
(b) Lateral magnification (or transverse magnification) denoted by m is defined as m= and
h1

v
is related as m =  .From the definition of m positive sign of m indicates erect image and
u
negative sign indicates inverted image.

(c) In case of successive reflection from mirrors, the overall lateral magnification is given by
m1 × m2 × m3 ......, where m 1 , m 2 etc. are lateral magnifications produced by individual
mirrors.
h 1 and h 2 denote the y coordinate of object and image respectively.

Note :
 Using 5.3(a) and 5.3(b) the following conclusions can be made (check yourself).

Nature of Object Nature of Image Inverted or erect


Real Real Inverted
Real Virtual Erect
Virtual Real Erect
Virtual Virtual Inverted

f f v
From 5.3(a) and 5.3(b); we get m=  ...............(just a time saving formula)
f u f

Example - 6
An extended object is placed perpendicular to the principal axis of a concave mirror of radius
of curvature 20 cm at a distance of 15 cm from pole. Find the lateral magnification
produced.
Solution u = – 15 cm f = – 10 cm

1 1 1
Using   we get, v = – 30 cm
v u f

v
 m =  = – 2.
u

f 10
Aliter : m = = =–2
f u  10  ( 15)

Example - 7 A person looks into a spherical mirror. The size of image of his face is twice the actual size
of his face. If the face is at a distance 20 cm then find the nature and radius of curvature of
the mirror.
Solution Person will see his face only when the image is virtual. Virtual image of real object is erect.
Hence m = 2

KVPY_ SA STREAM # 465


v
 = 2  v = 40 cm
u
1 1 1
Applying   ; f = – 40 cm or R = –80 cm (concave)  R.O.C. = 80 cm
v u f

f f
Alter : m =  2=
f u f  ( 20 )
 f = – 40 cm or R = –80cm (concave)  R.O.C. = 80 cm
Example - 8
An image of a candle on a screen is found to be double its size. When the candle is shifted
by a distance 5 cm then the image become triple its size. Find the nature and ROC of the
mirror.
Solution Since the images formed on screen it is real. Real object and real image implies concave
mirror.
f f
Applying m = or – 2= .................(1)
f u f  (u)

f
After shifting –3= .................(2)
f  (u  5)
[Why u + 5 ? , why not u – 5 : In a concave mirror, the size of real image will increase, only
when the real object is brought closer to the mirror. In doing so, its x coordinate will
increase]
From (1) & (2) we get,
f = – 30 cm or R = –60 cm (concave) and R.O.C. = 60cm

Example - 9
A point object is placed 60 cm from pole of a concave mirror of focal length 10 cm on the
principle axis. Find
(a) the position of image
(b) If object is shifted 1 mm towards the mirror along principle axis find the shift in
image. Explain the result.
Solution (a) u = – 60 cm
f = – 10cm

fu 10 ( 60) 600


v = = = –12 cm.
uf  60  (10 )  50
1 1 1
(b)  
v u f
2
v2   12  1
Differentiating , we get dv = – du = –  [1 mm ] = – mm
u 2
  60  25

[  du = 1mm; sign of du is + because it is shifted in +ve direction defined by sign


convention.]

(A) –ve sign of dv indicates that the image will shift towards negative direction.

(B) The sign of v is negative. Which implies the image is formed on negative side of pole. (A)
and (B) together imply that the image will shift away from pole.
Note that differentials dv and du denote small changes only.

KVPY_ SA STREAM # 466


(h) Newton's Formula: XY = f 2
X and Y are the distances ( along the principal axis ) of the object and image respectively
from the principal focus. This formula can be used when the distances are mentioned or
asked from the focus.

REFRACTI ON OF LI GHT
When the light changes its medium, some changes occurs in its properties, the phenomenon is
known as refraction.
 If the light is incident at an angle (0º < i < 90º) then it deviates from its actual path. It is due
to change in speed of light as light passes from one medium to another medium.
 If the light is incident normally then it goes to the second medium without bending, but still
it is called refraction.
 Refractive index of a medium is defined as the factor by which speed of light reduces as

c speed of light in vacuum


compared to the speed of light in vacuum.   = .
v speed of light in medium
More (less) refractive index implies less (more) speed of light in that medium, which therefore is
called denser (rarer) medium.

La ws of Refra ction
(a) The incident ray , the normal to any refracting surface at the point of incidence and
the refracted ray all lie in the same plane called the plane of incidence or plane of
refraction.

Sin i
(b) = Constant for any pair of media and for light
Sin r

of a given wave length. This is known as Snell's


Law.

Sin i n2 v 
Also, = = 1 = 1
Sin r n1 v2 2
For applying in problems remember
n 1sini = n 2sinr

n2
= 1n2 = Refractive Index of the second medium with respect to the first medium.
n1
c = speed of light in air (or vacuum) = 3 x 10 8 m/s.

Specia l ca ses :
 Normal incidence : i = 0
from Snell’s law : r = 0

 When light moves from denser to rarer medium it bends away from normal.

KVPY_ SA STREAM # 467


 When light moves from rarer to denser medium it bends towards the normal.

Note :
 Higher the value of R.., denser (optically) is the medium.
 Frequency of light does not change during refraction.

 Refractive index of the medium relative to vacuum =  r r


nvacuum = 1 ; ~ 1
nair =  ; nwater (average value) = 4/3 ; nglass (average value)= 3/2

Devia tion of a Ra y Due to Refra ction

Deviation () of ray incident


at i and refracted at r
is given by  = |i  r| .

Example - 10
A light ray is incident on a glass sphere at an angle of
incidence 600 as shown. Find the angles r, r’,e and the total
deviation after two refractions.

Solution Applying Snell’s law 1sin600 = 3 sinr  r = 300


From symmetry r’ = r = 300 .
Again applying Snell’s law at second surface 1sin e = 3 sinr
 e = 600
Deviation at first surface = i – r = 60 0 – 300 = 300
Deviation at second surface = e – r’ = 60 0 – 300 = 300
Therefore total deviation = 600 .

Example - 11 Find the angle a made by the light ray when it gets
refracted from water to air, as shown in figure.

Solution
Snell’s Law
4 3
W sin W = a sin a    1 sin 
3 5 a

4 4
sin a = a = sin–1
5 5

KVPY_ SA STREAM # 468


c
Example - 12 Find the speed of light in medium ‘a’ if speed of light in medium ‘b’ is
where c = speed of
3
light in vacuum and light refracts from medium ‘a’ to medium ‘b’ making 45º and 60º
respectively with the normal.
Solution : Snell’s Law
a sin a = b sin b

c c
sin a = sin b .
va vb

c c
sin 45º = sin 60º.
va c /3

2c
va =
3 3


Principle of Reversibility of Light Ra ys
(a) A ray travelling along the path of the reflected ray is reflected along the path of the
incident ray.
(b) A refracted ray reversed to travel back along its path will get refracted along the
path of the incident ray. Thus the incident and refracted rays are mutually reversible.

REFRACTI ON THROUGH A PARALLEL SLAB


When light passes through a parallel slab, having same medium on both sides, then
(a) Emergent ray is parallel to the incident ray.

Note :
 Emergent ray will not be parallel to the incident ray if the medium on both the sides of
slab are different.

(b) Light is shifted laterally, given by (students should be able to derive it)

t sin( i  r )
d =
cosr

t = thickness of slab

KVPY_ SA STREAM # 469


Example - 13
Find the lateral shift of light ray while it passes through a parallel glass slab of thickness 10
cm placed in air. The angle of incidence in air is 60º and the angle of refraction in glass is 45º.

t sin (i  r )
Solution d=
cos r

10 sin ( 60  45)


=
cos 45

10 sin 15
=
cos 45

= 10 2 sin 15ºcm


Appa rent Depth a nd shift of Subm erged Object
At near normal incidence (small angle of incidence i) apparent depth (d  ) is given by:

v
d  =
d
and v´ 
nrelative nrelative

ni (R.. of medium of incidence)


where nrelative =
nr (R. . of medium of refraction)

d = distance of object from the interface = real depth


d’ = distance of image from the interface = apparent depth
v = velocity of object perpendicular to interface relative to surface.
v´ = velocity of image perpendicular to interface relative to surface.
This formula can be easily derived using Snell’s law and applying the condition of nearly
normal incidence.... (try it or see in text book).

 1 
Apparent shift = d  1  

 nrel 

Example - 14 An object lies 100 cm inside water .It is viewed from air nearly normally. Find the apparent
depth of the object.

d 100
Solution : d= n = =75 cm
relative 4/3
1
KVPY_ SA STREAM # 470
Example - 15 A concave mirror is placed inside water with its shining surface upwards and principal axis
vertical as shown. Rays are incident parallel to the principal axis of concave mirror. Find the
position of final image.

Solution
The incident rays will pass undeviated through the water
surface and strike the mirror parallel to its principal axis.
Therefore for the mirror, object is at  . Its image A (in
figure) will be formed at focus which is 20 cm from the mirror.
Now for the interface between water and air, d = 10 cm.

d 10
 d’ = = = 7.5 cm.
 nw   4/3 
   
 na   1 

Example - 16
See the figure

(i) Find apparent height of the bird.

(ii) Find apparent depth of fish.

(iii) At what distance will the bird appear to the fish?

(iv) At what distance will the fish appear to the bird?


(v) If the velocity of bird is 12 cm/sec downward and the fish is 12 cm/sec in upward
direction, then find out their relative velocities with respect to each other.
Solution

36 36
(i) d´B =  = 48 cm
1 3/4
 4
 
3

36
(ii) d´F = = 27 cm
4/3
(iii) For fish : dB = 36 + 48 = 84 cm
dB = 36 + 48 = 84 cm
(iv) For bird : dF = 27 + 36 = 63 cm.
dF = 27 + 36 = 63 cm.

KVPY_ SA STREAM # 471


 
 12 
(v) Velocity of fish with respect to bird = 12    = 21 cm/sec.
 4/3
 1/ 1 

 
 12 
Velocity of bird with respect to fish = 12  = 28 cm/sec.
 3 / 4 
 1/ 1 

Example - 17 See the figure.

Find the distance of final image formed by mirror

 1 
Solution Shift = 3 1  
 3/2

 1 
For mirror object is at a distance = 21 – 3 1   = 20 cm
 3/2 
 Object is at the centre of curvature of mirror. Hence the light rays will retrace and image will
be formed on the object itself.


Re fra ction t hrough a com posit e sla b (or re fra ction through a num ber
of pa ra llel m edia , a s seen from a m edium of R. I. n 0 )
Apparent depth (distance of final image from final surface)

t1 t2 t3 tn
= n + n + n +......... + n
1rel 2rel 3rel n rel

Apparent shift

 1     n 
1
= t1  1  + t2  1  +........+ 1  n rel  tn
 n 1rel   n   
n
2 rel

Where ' t ' represents thickness and ' n ' represents the R.I. of the respective media, relative
to the medium of observer. (i.e. n 1rel = n1/n0 , n2 rel = n2/n0 etc.)

KVPY_ SA STREAM # 472


Example - 18

See the figure. Find the apparent


depth of object seen below
surface AB.

Solution

d 20 15
Dapp =  =
 2 
+
 1.5 
= 18 + 18 = 36 cm.
   
 1.8   1.8 


CRITICAL ANGLE AND TOTAL INTERNAL REFLECTION ( T. I. R.)
Critical angle is the angle made in denser medium for which the angle of refraction in rarer medium
is 90º. When angle in denser medium is more than critical angle, then the light ray reflects back in
denser medium following the laws of reflection and the interface behaves like a perfectly reflecting
mirror.
In the figure
O = Object
NN = Normal to the interface
II = Interface
C = Critical angle;
AB = reflected ray due to T. I. R.
When i = C then r = 90o
n
   C = sin  1 r
nd

Conditions of T. I. R.
(a) light is incident on the interface from denser medium.
(b) Angle of incidence should be greater
than the critical angle (i > C). Figure
shows a luminous object placed in
denser medium at a distance h from
an interface separating two media of
refractive indices r and d . Subscript
r &d stand for rarer and denser
medium respectively.
In the figure, ray 1 strikes the surface at an angle less than critical angle C and
gets refracted in rarer medium . Ray 2 strikes the surface at critical angle and
grazes the interface. Ray 3 strikes the surface making an angle more than critical
angle and gets internally reflected. The locus of points where ray strikes at critical
angle is a circle, called circle of illuminance. All light rays striking inside the circle
of illuminance get refracted in rarer medium. If an observer is in rarer medium, he/
she will see light coming out only from within the circle of illuminance. If a circular
opaque plate covers the circle of illuminance , no light will get refracted in rarer
medium and then the object can not be seen from the rarer medium. Radius of
C.O.I can be easily found.

KVPY_ SA STREAM # 473


Example - 19 Find the maximum angle that can be made in glass medium ( = 1.5) if a light ray is
refracted from glass to vacuum.
Solution : 1.5 sin C = 1 sin 90º, where C = critical angle.
sin C = 2/3
C = sin–1 2/3

Example - 20 Find the angle of refraction in a medium ( = 2) if light is incident in vacuum, making angle
equal to twice the critical angle.
Solution : Since the incident light is in rarer medium. Total Internal Reflection can not take place.

1
C = sin–1 = 30º   i = 2C = 60º

Applying Snell’s Law. 1 sin 60º = 2 sin r

 3
3  
sin r =  r = sin–1  4  .
4  

Example - 21
What should be the value of angle so that light entering
normally through the surface AC of a prism (n=3/2)
does not cross the second refracting surface AB?

Solution Light ray will pass the surface AC without bending since it is incident normally. Suppose it
strikes the surface AB at an angle of incidence i.
i = 90-
For the required condition:
90° – > c
or sin (90°) > sin c

1 2 2
or cos  > sin c = = or  < cos-1 .
3/ 2 3 3

Example - 22
What should be the value of refractive index n of a glass rod placed in air, so that the light
entering through the flat surface of the rod does not cross the curved surface of the rod?
Solution It is required that all possible r’ should be more than critical angle. This will be automatically
fulfilled if minimum r’ is more than critical angle ..........(A)
Angle r’ is minimum when r is maximum i.e. C( why ?).Therefore the minimum value of r’is
90-C.
From condition (A) :
90° – C > C or C < 45°

1 1
sin C < sin 45° ; < or n> 2 .
n 2

KVPY_ SA STREAM # 474



CHARACTERI STI CS OF A P RI SM
(a ) A hom oge ne ous solid t ra nspa re nt a nd re f ra ct in g m e dium bound e d by
t w o pla ne surf a ces incline d a t a n a ngle is ca lle d a prism :
3-D view

Refraction through a prism:

View from one side

(b) PQ and PR are refracting surfaces.

(c)  QPR = A is called refracting angle or the angle of prism (also called Apex angle).

(d)  = angle of deviation


(e) For refraction of a monochromatic ( single wave length) ray of light through a prism;
 = (i + e)  (r1 + r2) and r1 + r2 = A
  = i + e  A.

(f) Variation of  versus i (shown in diagram).


For one  (except  min) there are two values of
angle of incidence. If i and e are interchanged then
we get the same value of  because of reversibility
principle of light

Note : (i) For application of above result medium on both sides of prism must be same.
(ii) Based on above graph we can also derive following result, which says that i and e can be
interchanged for a particular deviation in other words there are two angle of incidence for a
given deviation (except minimum deviation).

i r1 r2 e
1 2 3 4 5

4 3 2 1 5

KVPY_ SA STREAM # 475


(g) There is one and only one angle of incidence for which the angle of deviation is minimum.
(h) When  = min , the angle of minimum deviation, then i = e and r1 = r2, the ray passes
symmetrically w.r.t. the refracting surfaces. We can show by simple calculation that
min = 2i min – A
where i min = angle of incidence for minimum deviation, and r = A/2.

 m 
sin
A
 2  nprism
 , where nrel = n
n rel =
sin A
2
 surroundings

Alsomin = (n  1) A (for small values of  A)

(i) For a thin prism ( A 10o) and for small value of i, all values of

nprism
  = ( nrel  1 ) A where n rel =
nsurrounding

Example - 23 Refracting angle of a prism A = 60º and its refractive index is, n = 3/2, what is the angle of
incidence i to get minimum deviation? Also find the minimum deviation. Assume the
surrounding medium to be air (n = 1).
Solution For minimum deviation,

A
r1 = r2 = = 30º.
2
applying Snell’s law at I surface

3 1  3  3
1 × sin i = sin 30º  i = sin    min = 2sin–1   – 60º
2 4 4

Example - 24 See the figure


Find the deviation caused by a prism having

3
refracting angle 4º and refractive index .
2

3
Solution =( – 1) × 40 = 20
2
7
Example - 25 For a prism, A = 60º, n = . Find the minimum possible angle of incidence, so that the
3
light ray is refracted from the second surface. Also find max .
Solution In minimum incidence case the angles will be as shown in figure
Applying snell’s law :

7
1 × sin i min = sin ( A – C)
3

 
7 7  sin 60 1  3  cos 60 3  1
= (sin A cos C – cos A sin C) =  7 7  = 2
3 3  
 i min = 30º  max = i min + 900 – A = 300 + 900 – 600 = 600 .

KVPY_ SA STREAM # 476


Example - 26
Show that if A > A max (= 2 C), then total internal reflection occurs at second refracting
surface PR of the prism for any value of ' i '.
Solution For T.I.R. at second surface
r’ > C  (A – r) > C or A > (C + r)
The above relation will be fulfilled if
or A > C + rmax or A>C+C or A > 2C
(j) On the basis of above example and similar reasoning, it can be shown that (you should try
the following cases (ii) and (iii) yourself.)
(i) If A > 2C, all rays are reflected back from the second surface.
(ii) If A  C , no rays are reflected back from the second surface i.e. all rays are
refracted from second surface.
(iii) If 2C  A  C , some rays are reflected back from the second surface and some
rays are refracted from second surface, depending on the angle of incidence..
(k)  is maximum for two values of i
 imin (corresponding to e = 90º) and i = 90º
(corresponding to emin ).
For i min : nssin i min = np sin(A – C)
If i < imin then T.I.R. takes place at second refracting surface PR.


D I SP ERSI ON OF LI GHT
The angular splitting of a ray of white light into a number of components and spreading in different
directions is called Dispersion of Light. [It is for whole Electro Magnetic Wave in totality]. This
phenomenon is because waves of different wavelength move with same speed in vacuum but with
different speeds in a medium.
Therefore, the refractive index of a medium depends slightly on wavelength also. This variation of
refractive index with wavelength is given by Cauchy’s formula.
b
Cauchy's formula n ( ) = a  where a and b are positive constants of a medium.
2

Note :
 Such phenomenon is not exhibited by sound waves.
Angle between the rays of the extreme colours in the refracted (dispersed) light is called
angle of dispersion.   =  v    r (Fig. (a))
Fig (a) and (c) represents dispersion, whereas in fig. (b) there is no dispersion.

For prism of small ‘A’ and with small ‘i’ :


 = v – r = (nv – nr)A
Deviation of beam (also called mean deviation)
 = y = (ny – 1)A
nv, nr and ny are R. . of material for violet, red and yellow colours respectively.

KVPY_ SA STREAM # 477


Example - 27
The refractive indices of flint glass for red and violet light are 1.613 and 1.632 respectively.
Find the angular dispersion produced by a thin prism of flint glass having refracting angle 5 0.
Solution Deviation of the red light is r = (r – 1)A and deviation of the violet light is v = (v – 1)A.
The dispersion = v – r = (v – r)A = (1.632 – 1.613) × 50 = 0.0950 .
Note :
 Numerical data reveals that if the average value of  is small v – r is also small and if the
average value of  is large v – r is also large. Thus, larger the mean deviation, larger will be
the angular dispersion.
nv  nr
Dispersive power () of the medium of the material of prism is given by: =
ny  1

  is the property of a medium.


For small angled prism ( A 10o ) with light incident at small angle i :

n v  nr  v  r  angular dispersion
ny  1 =  = 
y
= deviation of mean ray ( yellow)
y

n v  nr
[ ny = if ny is not given in the problem ]
2
 n  1 = refractivity of the medium for the corresponding colour.

Example - 28
Refractive index of glass for red and violet colours are 1.50 and 1.60 respectively. Find
(a) the refractive index for yellow colour, approximately
(b) Dispersive power of the medium.
 v  R 1.50  1.60
Solution (a) y ~ = = 1.55
2 2
 v – R 1.60  1.50
(b) = y – 1 = = 0.18.
1.55  1

D ispe rsion w it hout de via tion (Dire ct Vision Com bina tion)
The condition for direct vision combination is :
 n v nr   n v  n r 
n y 1 
A = ny 1  A    1 A    1 A
 2   2 
Two or more prisms can be combined in various ways to get different combination of angular
dispersion and deviation.

or or

D e via t ion w it hout dispersion (Achrom a t ic Com bina tion )


Condition for achromatic combination is: (n v  nr) A = (n v  n r) A

or or

KVPY_ SA STREAM # 478


Example - 29 If two prisms are combined, as shown in figure,
find the total angular dispersion and angle of
deviation suffered by a white ray of light incident
on the combination.
Solution Both prisms will turn the light rays towards their bases and hence in same direction.
Therefore turnings caused by both prisms are additive.
Total angular dispersion
=  + ’ = (V – R) A + (’V – ’R) A’
= (1.5 – 1.4) 4º + (1.7 – 1.5)2º = 0.8°
Total deviation
=  + ´

  V  R   '  ' R   1 . 5  1. 4   1 . 7  1. 5 
=   1 A +  V  1 A’ =   1 0.4º +   1 0.2º
 2   2   2   2 
= (1.45 – 1) 0.4º + (1.6 – 1) 0.2º
= 0.45 × 0.4º + 0.6 × 0.2º
= 1.80 + 1.2 = 3.0º Ans.

Example - 30 Two thin prisms are combined to form an achromatic combination. For I prism A = 4º, R =
1.35, Y = 1.40, V = 1.42. for II prism ’R= 1.7, ’Y = 1.8 and ’V = 1.9 find the prism angle
of II prism and the net mean deviation.
Solution Condition for achromatic combination.
 = ’
(V – R)A = (’V – ’R)A’

(1.42  1.35)4º
 A’ =  1.4
1.9  1.7
Net =  – ’ = (Y – 1)A – (’Y – 1)A’
= (1.40 – 1) 4º – (1.8 – 1) 1.4° = 0.48º.


REFRACTI ON AT SP HERICAL SURFACES
For paraxial rays incident on a spherical surface separating two media:
n2 n n n
 1 = 2 1 .................... (A)
v u R
where light moves from the medium of refractive index n 1 to the medium of refractive index n2.
Transverse magnification (m) (of dimension perpendicular to principal axis) due to refraction at

v R  v / n2 
spherical surface is given by m= =  u / n 
uR  1

Example-31.
Find the position, size and nature of image, for the
situation shown in figure. Draw ray diagram .

KVPY_ SA STREAM # 479


Solution For refraction near point A, u = – 30 ; R = – 20; n 1 = 2 ; n2 = 1.
Applying refraction formula

n2 n n n
 1 = 2 1
v u R

1 2 1 2
– =
v  30  20
v = – 60 cm

h2 n1v 2( 60 )
m = h = n u = = 4  h2 = 4 mm.
1 2 1( 30 )

Spe cia l ca se :
Refra ction at pla ne Surfa ces

n2 n n n
Putting R =  in the formula  1 = 2 1 , we get;
v u R
n2 u
v=
n1
The same sign of v and u implies that the object and the image are always on the same side
of the interface separating the two media. If we write the above formula as
u
v = ,
nrel

it gives the relation between the apparent depth and real depth, as we have seen before.

Example - 32
Using formula of spherical surface or otherwise, find the apparent depth of an object placed
10 cm below the water surface, if seen near normally from air.
Solution Put R =  in the formula of the Refraction at Spherical Surfaces we get,

un 2
v = n
1

u = – 10 cm

4
n1 =
3
n2 = 1

10  1
v =– = – 7.5 cm
4/3
negative sign implies that the image is formed in water.
Alter:

dreal
dapp = 
rel

10 30
= = = 7.5 cm.
4/3 4

KVPY_ SA STREAM # 480



THI N LEN S
A thin lens is called convex if it is thicker at the middle and it is called concave if it is thicker at the
ends.
One surface of a convex lens is always convex . Depending on the other surface a convex lens is
categorized as
(a) biconvex or convexo convex , if the other surface is also convex,
(b) Plano convex if the other surface is plane and
(c) Concavo convex if the other surface is concave.
Similarly concave lens is categorized as
concavo-concave or biconcave, plano-concave
and convexo-concave.

For a spherical, thin lens having the same


medium on both sides:
1 1  1 1 
 = (nrel  1)    .........(a),

v u  R1 R 2 

nlens
where nrel = and R1 and R2 are x coordinates of the centre of curvature of the 1 st surface
nmedium

and 2nd surface respectively.

1  1 1 
= (nrel – 1)  R – R  Lens Maker's formula
 ..............(b)
f  1 2 
From (a) and (b)

1 1 1
 =
f
v u
Lens has two Focii:

1 1 1
If u =  , then    v=f
v  f
 If incident rays are parallel to principal axis then its refracted ray will cut the principal axis at ‘f’.
It is called 2nd focus.
In case of converging lens it is positive and in case of diverging lens it is negative.

1 1 1
If v =  that means    u=–f
 u f
 If incident rays cuts principal axis at – f then its refracted ray will become parallel to the
principal axis. It is called 1 st focus. In case of converging lens it is negative (  f is positive)
and in the case of diverging lens it positive (  f is negative)

use of – f & + f is in drawing the ray diagrams.


KVPY_ SA STREAM # 481
Notice that the point B, its image B and the pole P of the lens are collinear. It is due to
parallel slab nature of the lens at the middle. This ray goes straight. (Remember this)

1  1 1 
From the relation
f
=(nrel  1)    it can be seen that the second focal length depends on
 R1 R2 
two factors.
 1 1 
(A) The factor    is
 R1 R 2 
(i) Positive for all types of convex lenses and
(ii) Negative for all types of concave lenses.

(B) The factor (nrel  1) is


(i) Positive when surrounding medium is rarer than the medium of lens.
(ii) Negative when surrounding medium is denser than the medium of lens.
(C) So a lens is converging if f is positive which happens when both the factors (A) and (B)
are of same sign.
(D) And a lens is diverging if f is negative which happens when the factors (A) and (B) are
of opposite signs.
(E) Focal length of the lens depends on medium of lens as well as surrounding.
(F) It also depends on wavelength of incident light. Incapability of lens to focus light rays of
various wavelengths at single point is known as chromatic aberration.

Example - 33 Find the behaviour of a concave lens placed in a rarer medium.


Solution Factor (A) is negative , because the lens is concave.
Factor (B) is positive , because the lens is placed in a rarer medium.
Therefore the focal length of the lens, which depends on the product of these factors , is
negative and hence the lens will behave as diverging lens.
 1 1 
Example - 34 Show that the factor    (and therefore focal length) does not depend on which
 R1 R 2 
surface of the lens light strike first.
Solution
Consider a convex lens of radii of curvature p and q as shown.

CASE 1: Suppose light is incident from left side and strikes the surface with radius of curvature p, first.

 1 1  1 1 
Then R1 = +p ; R2 = -q and    =   
R
 1 R 2   p  q 

KVPY_ SA STREAM # 482


CASE 2: Suppose light is incident from right side and strikes the surface with radius of curvature q, first.

 1 1  1 1 
Then R1 = +q ; R2 = -p and    =   
 R1 R 2  q p
Though we have shown the result for biconvex lens , it is true for every lens.

Example - 35
Find the focal length of the lens
shown in the figure.

1  1 1 
Solution :  = (nrel – 1)   
f  R1 R 2 

1  1 1  1 1 2
 = (3/2 – 1)     = ×  f = + 10 cm.
f  10 (  10 )  f 2 10

Example - 36
Find the focal length of the
lens shown in figure

1  1 1  3   1 1
Solution : = (nrel – 1)  R  R  =   1   
f  1 2  2    10 10 

f = – 10 cm
Example - 37
Find the focal length of the ROC = 60 cm
lens shown in figure ROC = 20 cm
(a) If the light is incident from left side.
(b) If the light is incident from right side.

1  1 1  3   1 1 
Solution : (a) = (nrel – 1)  R  R  =   1   
f  1 2  2    60  20 

f = 60 cm

1  1 1  3   1 1 
(b) = (nrel – 1)  R  R  =   1   
f  1 2  2   20 60 

f = 60 cm

Example - 38
Point object is placed on the principal axis of a thin lens with parallel curved boundaries
i.e., having same radii of curvature. Discuss about the position of the image formed .

1  1 1 
Solution
f
= (nrel  1)  R  R  = 0 [  R1 = R2]
 1 2 

1 1
 = 0 or v = u i.e. rays pass without appreciable bending.
v u

KVPY_ SA STREAM # 483


Example - 39
Focal length of a thin lens in air, is 10 cm. Now medium on
one side of the lens is replaced by a medium of refractive
index =2. The radius of curvature of surface of lens, in
contact with the medium, is 20 cm. Find the new focal length.

Solution : Let radius of  surface be R1 and refractive index of lens be  . Let parallel rays be incident
on the lens. Applying refraction formula at first surface

 1  1
– = ...(1)
v1  R1

2  2
At  surface – v = ...(2)
v 1  20

Adding (1) and (2)

 1 2   1 2
– + – = +
v1  v v1 R1  20

 1 1   1 2 1 1 2
= ( – 1)  R   20  – – = (in air) + –
 1  20 20 f 20 20

 v = 40 cm  f = 40 cm

Example - 40
Figure shows a point object and a converging lens.
Find the final image formed.

1 1 1
Solution : – =
v u f

1 1 1
– =
v  15 10

1 1 1 1
= – =
v 10 15 30

v = + 30 cm

Example - 41
See the figure
Find the position of final image formed.
Solution For converging lens

fu
u = –15 cm, f = 10 cm v = = 30 cm
f u
For diverging lens
u = 5 cm

fu
f = –10 cm v = = 10 cm
f u

KVPY_ SA STREAM # 484


Example - 42
Figure shows two converging lenses. Incident rays
are parallel to principal axis. What should be the
value of d so that final rays are also parallel.

Solution Final rays should be parallel. For this the  focus of L1


must coincide with  focus of L2.
d = 10 + 20
= 30 cm
Here the diameter of ray beam becomes wider.

Example - 43
See the figure
Find the position of final image formed.

Solution : For lens,

1 1 1
– =
v u f

1 1 1
– =  v = + 30 cm
v  15 10
Hence it is object for mirror
u = – 15 cm

1 1 1
+ =  v = – 30 cm
v  15  10
Now for second time it again passes through lens
u = – 15 cm
v=? ; f = 10 cm

1 1 1
– =  v = + 30
v  15 10
Hence final image will form at a distance 30 cm from the lens towards left.

Example - 44

What should be the value of d so that image is formed


on the object itself.

Solution For lens :

1 1 1
– = v = + 30 cm
v  15 10
Case I :
If d = 30, the object for mirror will
be at pole and its image will be
formed there itself.

KVPY_ SA STREAM # 485


Case II :

If the rays strike the mirror normally, they will retrace


and the image will be formed on the object itself

 d = 30 – 20 = 10 cm


Tra nsve r se m a gnif ica t ion ( m )
Transverse magnification (m) of (of dimension perpendicular to principal axis) is given by

v
m =
u
If the lens is thick or/and the medium on both sides is different, then we have to apply the
formula given for refraction at spherical surfaces step by step.

Example - 45
An extended real object of size 2 cm is placed perpendicular to the principal axis of a
converging lens of focal length 20 cm. The distance between the object and the lens is 30
cm.
(i) Find the lateral magnification produced by the lens.
(ii) Find the height of the image.
(iii) Find the change in lateral magnification, if the object is brought closer to the lens by 1
mm along the principal axis.

1 1 1
Solution Using – =
v u f

v
and m =
u

f
we get m = .........(A)
f u

20 20
 m = = = – 2
 20  ( 30 )  10

–ve sign implies that the image is inverted.

h2
(ii) =m
h1

 h2 = mh1 = (–2) (2) = – 4 cm

(iii) Differentiating (A) we get

f (20 ) 2
dm = 2 du = (0.1) = = –.02
( f  u) ( 10 )2 100

Note that the method of differential is valid only when changes are small.

KVPY_ SA STREAM # 486


Alternate method :
u (after displacing the object)
= –(30 + 0.1) = – 29.9 cm
Applying the formula

f
m =
f u

20
m = = – 2.02
20  (29.9)
 change in ‘m’ = – 0.02.
Since in this method differential is not used, this method can be used for any changes,
small or large.

COM BIN ATI ON OF LEN SES
The equivalent focal length of thin lenses in contact is given by
1 1 1 1
   ...
F f1 f2 f3

where f `1 , f 2 , f 3 are focal lengths of individual lenses.


If two converging lenses are separated by a distance d and the incident light rays are parallel to the
common principal axis ,then the combination behaves like a single lens of focal length given by the
relation
1 1 1 d
  
F f1 f2 f1f2
d F
and the position of equivalent lens is with respect to 2nd lens
f1

Example - 46 Find the lateral magnification produced by the


combination of lenses shown in the figure.

1 1 1 1 1 1
Solution = f + f = – =  f = + 20
f 1 2 10 20 20

1 1 1 1 1 1 1
 – =  = – = = – 20 cm
v  10 20 v 20 10 20

20
 m = = 2
 10
Example - 47 Find the focal length of equivalent system.

KVPY_ SA STREAM # 487


1 3   1 1  1 2 1
Solution
f1
=   1    = × =
 2   10 10  2 10 10

1  6   1 1  1   30  3
=   1    = ×   =
f2 5   10 20  5  10  20  100

1 8   1 1  3
=   1    =
f3 5   20 20  50

1 1 1 1 1 3 3 100
= + + = + + f = Ans.
f f1 f 2 f 3 10 100 50 13


COM BI NATI ON OF LENS AND M IRROR

The combination of lens and mirror behaves like a


mirror of focal length ‘f’ given by

1 1 2
= F – F
f m 

If lenses are more than one, ‘f’ is given by

1 1  1
f
=
Fm
– 2 

f 

 

For the following figure

1 1 1 1
‘f’ is given by = F – 2   
f m  f1 f 2 

Example - 48 Find the position of final image formed.


(The gap shown in figure is of negligible width )

1 1 2 1
Solution feq = 10 –
10
=
10
 f eq = – 10 cm

1 1 1
+ =  v = – 20 cm
v  20  10
Hence image will be formed on the object itself

KVPY_ SA STREAM # 488


1. In the figure shown a person AB of height 170 cm is standing infront of a
plane mirror. His eyes are at height 164 cm. At what distance from P
should a hole be made in the mirror so that he cannot see the top of his
head.
(A) 167 cm (B) 161 cm (C) 163 cm (D) none of these a

2.  is the image of a point object O formed by spherical mirror, then which of the following statement is incorrect
(A) If O and  are on same side of the principal axis, then they have to be on opposite sides of the mirror.
(B) If O and  are on opposite side of the principal axis, then they have to be on same side of the mirror.
(C) If O and  are on opposite side of the principal axis, then they can be on opposite side of the mirror as well.
(D) If O is on principal axis then  has to lie on principal axis only.

3. An object and a plane mirror are shown in figure. Mirror is moved with velocity V as shown. The velocity
of image is :

(A) 2 V sin (B) 2 V (C) 2V cos (D) none of these

4. Two plane mirrors are joined together as shown in the figure. Two point
objects O 1 and O 2 are placed symmetrically such that AO 1 = AO 2. The
image of the two objects is common if :
(A)  = 60° (B)  = 90°
(C)  = 30° (D)  = 45°

5. The following figure represents a wave front AB which passes from air to another transparent medium
and produces a new wave front CD after refraction. The refractive index of the medium is (PQ is the
boundary between air and the medium).

cos 1 cos  4 sin  2


(A) cos  (B) cos  (C) (D) sin 
4 1 3
6. A man starting from point P crosses a 4 km wide lagoon and reaches point Q in the shortest possible
time by the path shown in the figure. If the person swims at a speed of 3 km/hr and walks at a speed of
4 km/hr, then his time of journey is (µ salt water = 4/3) :

Q
Land
6km
3km

LAGOON
LAGOON
(Salt water lake)
(Salt lake)
4km

(A) 4hr, 10 min. (B) 4 hr and 30 min. (C) 3 hr and 50 min (D) 5 hr and 10 min.

KVPY_ SA STREAM # 489


7. A mango tree is at the bank of a river and one of the branch of tree extends over the river. A tortoise lives
in the river . A mango falls just above the tortoise. The acceleration of the mango falling from tree
appearing to the tortoise is (Refractive index of water is 4/3 and the tortoise is stationary)
3g 4g
(A) g (B) (C) (D) None of these
4 3

8. In the figure ABC is the cross section of a right angled prism and ACDE
is the cross section of a glass slab. The value of  so that light incident A E
normally on the face AB does not cross the face AC is

n=6/5
n=3/2
(given sin–1 (3/5) = 37º)
(A)  37º (B) < 37º )q
(C)  53º (D) < 53º B C D

7
9. Refractive index of a prism is and the angle of prism is 60º. The minimum angle of incidence
3
of a ray that will be transmitted through the prism is :
(A) 30º (B) 45º (C) 15º (D) 50º

10. For a prism kept in air it is found that for an angle of incidence 60°, the angle of refraction 'A', angle of
deviation '' and angle of emergence 'e' become equal. Then the refractive index of the prism is
(A) 1.73 (B) 1.15 (C) 1.5 (D) 1.33

11. As shown in the figure The observer 'O' sees the distance AB as infinitely large. If refractive index of
1
liquid is 1 and that of glass is 2, then
 2 is :

(A) 2 (B) 1/2 (C) 4 (D) None of these

12. A transparent cylinder has its right half polished so as to act as a


mirror. A paraxial light ray is incident from left, that is parallel to principal
axis, exits parallel to the incident ray as shown in the figure. The
refractive index n of the material of the cylinder is :
(A) 1.2 (B) 1.5 (C) 1.8 (D) 2.0

13. In the figure shown, the image of a real object O is formed at point . AB is the principal axis of the
mirror. The mirror must be:

(A) concave & placed towards right of  (B) concave & placed towards left of 
(C) convex & placed towards right of  (D) convex & placed towards left of .

14. If a prism having refractive index 2 , has angle of minimum deviation equal to the angle of refraction of
the prism, then the angle of refraction of the prism is:
(A) 30º (B) 45º (C) 60º (D) 90º

KVPY_ SA STREAM # 490


15. A bird is flying up at angle sin 1 (3/5) with the horizontal. A fish in a pond looks at that bird when it is
vertically above the fish. The angle at which the bird appears to fly (to the fish) is: [ n water = 4/3 ]
(A) sin1 (3/5) (B) sin1 (4/5) (C) 45º (D) sin1 (9/16)

16. In the figure shown a point object O is placed in air. A spherical boundary separates two media. AB is
principal axis. The refractive index above AB is 1.6 and below AB is 2.0. The separation between the
images formed due to refraction at spherical surface is :

(A) 12 m (B) 21 m (C) 14 m (D) 10 m

17. Light of wavelength 4000 Å is incident at small angle on a prism of apex angle 4º. The prism has
nv = 1.5 & nr = 1.48. The angle of dispersion produced by the prism in this light is :
(A) 0.2º (B) 0.08º (C) 0.192º (D) none of these

18. A convex lens of focal length f and a plane mirror are y distance apart. An object O is kept on the principal
axis of the lens at a distance x from the lens. The values of x and y for the final image of O to fall exactly
(position & size) on the object ‘O’ :

y x

(A) x = f, y = f (B) x = f, y = 2f (C) x = 2f, y = f (D) x = 2f, y = 2f

19. A square ABCD of side 1mm is kept at distance 15 cm infront of the concave mirror as shown in the figure.
The focal length of the mirror is 10 cm. The length of the perimeter of its image will be :

(A) 8 mm (B) 2 mm (C) 12 mm (D) 6 mm

20. A point object is kept in front of a plane mirror. The plane mirror is doing SHM of amplitude 2 cm. The plane
mirror moves along the x-axis and x- axis is normal to the mirror. The amplitude of the mirror is such that the
object is always infront of the mirror. The amplitude of SHM of the image is
(A) zero (B) 2 cm (C) 4 cm (D) 1 cm

21. The given lens is broken into four parts and rearranged as shown in the figure. If the initial focal length
is f then after rearrangement the equivalent focal length is

12

34

in air

f f
(A) f (B) (C) (D) 4f
2 4

KVPY_ SA STREAM # 491


22. An infinitely long rectangular strip is placed on the principal axis of a concave mirror as shown in the
figure. One end of the strip coincides with centre of curvature as shown. The height of rectangular strip
is very small in comparison to focal length of the mirror. Then the shape of image of strip formed by
concave mirror is

F C

(A) Rectangle (B) Trapezium (C) Triangle (D) Square


23. The distance of an object from the pole of a concave mirror is equal to its radius of curvature. The image
must be:
(A) real (B) inverted (C) same sized (D) erect
24. Two plane mirrors are inclined at 70º. A ray incident on one mirror at angle after reflection falls on the
second mirror and is reflected from there parallel to the first mirror.  is:
(A) 50º (B) 45º (C) 30º (D) 55º
25. AB is an incident beam of light and CD is a reflected beam (the number of
reflections for this may be 1 or more than 1) of light. AB & CD are separated
by some distance (may be large). It is possible by placing what type of
mirror on the right side.
(A) one plane mirror (B) one concave mirror
(C) one convex mirror (D) none of these
26. The relative refractive index of glass with respect to a material is 0.9 and the absolute refractive index of glass
is 1.512. The refractive index of the material is
(A) 1.42 (B) 1.58 (C) 1.68 (D) 1.72
sin i
27. In the figure shown is equal to:
sin r

 22 3  3 1
(A) (B) (C) (D) none of these
 3 1 1  22
28. The critical angle of light going from medium A to medium B is . The speed of light in medium A is v.
The speed of light in medium B is:
v
(A) (B) v sin  (C) v cot  (D) v tan 
sin
29. If lower half of the body of a lens is covered with black paper, the image produced by the lens will
(A) disappear (B) remain almost unaffected
(C) have its size reduced to half (D) have its intensity reduced to half.
30. Two diverging lenses are kept as shown in the figure. The final image formed will be :

(A) virtual for any value of d1 & d2 (B) real for any value of d1 & d2
(C) virtual or real depends on d1 & d2 only
(D) virtual or real depends on d1 & d2 & also on the focal lengths of the lens.

KVPY_ SA STREAM # 492


31. In the figure shown an object O is placed in front of a thin lens of small
aperture. The dotted curves represent the extensions of the respective
curves in the figure. How many images will be formed finally.
(A) 1 (B) 2
(C) 3 (D) 4

32. In the figure (i) a thin lens of focal length 10 cm is shown. The lens is cut into two equal parts, and the
parts are arranged as shown in the figure (ii). An object AB of height 1 cm is placed at distance of 7.5
cm from the arrangement. The height of the final image will be:

(A) 0.5 cm (B) 2 cm (C) 1 cm (D) 4 cm

33. In the figure (a) the light is incident at an angle  (slightly greater than the critical angle). Now keeping
the incident ray fixed a parallel slab of refractive index n 3 is placed on surface AB.(figb)

(A) total internal reflection occurs at AB for n 3 = n2


(B) total internal reflection occurs at AB for n 3 > n1
(C) the ray will return back to the same medium for all values of n3
(D) total reflection occurs at CD for n3 < n1.

34. An equilateral prism deviates a ray through 40º for two angles of incidence differing by 20º. The possible
angles of incidence is
(A) 300 (B) 500 (C) 200 (D) 600

35. A man is sitting in a room at 2 m from a wall W 1 wants to see the full height of the wall W 2 behind him 4 m
high and 6 m away from the facing wall W 1. What is the minimum vertical length of mirror on the facing wall
required for the purpose ?
(A) 4 m (B) 2 m (C) 3 m (D)1 m

36. A flint glass prism and a crown glass prism are to be combined in such a way that the deviation of the mean
ray is zero. The refractive index of flint and crown glasses for the mean ray are 1.6 and 1.9 respectively. If the
refracting angle of the flint prism is 6°, what would be the refracting angle of crown prism?
(A) 4º (B) 2º (C) 3º (D) `1º

37. A plane mirror is moving with velocity 4 î  5 ĵ  8 k̂ . A point object in front of the mirror moves with a

velocity 3 î  4 ĵ  5 k̂ . Here k̂ is along the normal to the plane mirror and facing towards the object.
The velocity of the image is :
(A)  3 î  4 ĵ  5 k̂ (B) 3 î  4 ˆj  11 k̂ (C)  3 î  4 ĵ  11 k̂ (D) 7 î  9 ĵ  11 k̂

KVPY_ SA STREAM # 493


3 4
38. A light wave travels from glass to water. The refractive index for glass and water are and respectively..
2 3
The value of the critical angle will be:
 1 9 8 5
(A) sin–1  2  (B) sin–1  8  (C) sin–1  9  (D) sin–1  7 
       

39. When a beam of light goes from denser medium (d) to rarer medium (r), then it is generally observed that
magnitude of angle of incidence is half that of angle of refraction. Then magnitude of incident angle will be-
(here  = d/r)

  


(A) 2 sin   (C) cos   (D) 2 cos  
-1 -1 -1 -1
(B) 2 cos 
2 2 2
40. The image for the converging beam after refraction through the curved surface is formed at:

40 40 180
(A) x = 40 cm (B) x = cm (C) x =  cm (D) x = cm
3 3 7
41. A thin linear object of size 1 mm is kept along the principal axis of a convex lens of focal length 10 cm.
The object is at 15 cm from the lens. The length of the image is:
(A) 1 mm (B) 4 mm (C) 2 mm (D) 8 mm
42. A convex lens is cut into two parts in different ways that are arranged in four manners, as shown.
Which arrangement will give maximum optical power ?

(A) (B) (C) (D)

KVPY PROBLEMS (PREVIOUS YEARS)


1. The objective lens of a telescope has a diameter of 12.2 cm. The angular resolution of the telescope at the
wavelength of 500 nm is [KVPY_2008] [2 Marks]
(A) 2 × 10 –9 rad (B) 1.22 × 10 –7 rad (C) 4 × 10 –5 rad (D) 5 × 10 –6 rad

2. A plano-convex lens made of material of refractive index  with radius of curvature R is silvered on the curved
side. How far away from the lens-mirror must you place a point object so that the image coincides with the
object ? [KVPY_2009] [2 Marks]
R R
(A) (B) R (C) (D) R
  1
3. A point source of light is placed at the bottom of a vessel which is filled with water of refractive index  to a
height h. If a floating opaque disc has to be placed exactly above it so that the source is invisible from above,
the radius of the disc should be : [KVPY_2010] [1 Marks]
h h h h
(A) (B) (C) (D)
 1 2  1 2  1 2  1

KVPY_ SA STREAM # 494


4. Three transparent media of refractive indices 1,2, 3, respectively, are
stacked as shown. A ray of light follows the path shown. No light enters the
third medium. [KVPY_2010] [1 Marks]

Then :
(A) 1 < 2 < 3 (B) 2 < 1 < 3
(C) 1 < 3 < 2 (D) 3 < 1 < 2

5. A narrow parallel beam of light falls on a glass sphere of radius R and refractive index  at normal incidence.
The distance of the image from the outer edge is given by : [KVPY_2010] [2 Marks]

R(2  ) R(2   ) R(2  ) R(2   )


(A) (B) (C) (D)
2(  1) 2(  1) 2(  1) 2(  1)

6. A material is embedded between two glass plates . Refractive index n of the material varies with thickness as
shown below .The maximum incident angle (in degrees) on the material for which beam will pass through the
material is [KVPY_2011] [2 Marks]

(A) 60.0 (B) 53.1 (C) 43.5 (D) 32.3

7. On a bright sunny day a diver of height h stands at the bottom of a lake of depth H. Looking upward, he can
see objects outside the lake in a circular region of radius R. Beyond this circle he sees the images of objects
lying on the floor of the lake. If refractive index of waler is 4/3, then the value of R is :
[KVPY_2012] [2 Marks]

3(H  h) (H  h) (H  h)
(A) (B) 3h 7 (C) (D)
7 7 5
3 3

8. A ray of light incident on a glass sphere (refractive index 3 ) suffers total internal reflection before emerging
out exactly parallel to the incident ray. The angle of incidence was [KVPY_2013] [1 Marks]
(A) 75° (B) 30° (C) 45° (D) 60°

9. The refractive index of a prism measured using three lines of a mercury vapour lamp. If 1, 2 and 3 are the
measured refractive indices for these green, blue and yellow lines respectively, then [KVPY_2013] [1 Marks]
(A) 2 > 3 > 1 (B) 2 > 1 > 3 (C) 3 > 2 > 1 (D) 1 > 2 > 3

10. A ray of light incident on a transparent sphere at an angle /4 and refracted at an angle r, emerges from the
sphere after suffering one internal reflection. The total angle of deviation of the ray is [KVPY_2013] [1 Marks]

3   5
(A)  4r (B)  4r (C) r (D)  4r
2 2 4 2

KVPY_ SA STREAM # 495


11. A horizontal parallel beam of light passes through a vertical convex lens of focal length 20 cm and is then
reflected by a tilted plane mirror so that it converges to a point I. The distance PI is 10 cm.
[KVPY_2013] [1 Marks]

M is a point at which the axis of the lens intersects the mirror. The distance PM is 10 cm. The angle which
the mirror makes with the horizontal is
(A) 15° (B) 30° (C) 45° (D) 60°

12. In a car a rear view mirror having a radius of curvature 1.50 m forms a virtual image of a bus located 10.0 m
from the mirror. The factor by which the mirror magnifies the size of the bus is close to
[KVPY_2013] [1 Marks]
(A) 0.06 (B) 0.07 (C) 0.08 (D) 0.09

13. The angle of a prism is 60°. When light is incident at an angle of 60° on the prism, the angle of emergence is
40°. The angle of incidence i for which the light ray will deviate the least is such that
[KVPY_2014_SA ] [1 Mark]
(A) i < 40° (B) 40° < i < 50° (C) 50° < i < 60° (D) i > 60°

14. A concave lens made of material of refractive index 1.6 is immersed in a medium of refractive index 2.0. The
two surfaces of the concave lens have the same radius of curvature 0.2 m. The lens will behave as a
[KVPY_2014_SA] [1 Mark]
(A) Divergent lens of focal length 0.4m (B) Divergent lens of focal length 0.5 m.
(C) Convergent lens of focal length 0.4 m. (D) Convergent lens of focal length 0.5 m

15. A ray of light incident paralled to the base PQ of an isosceles right-angled triangular prism PQR suffers two
successive total internal reflections at the faces PQ and QR before emerging reversed in direction as shown
R

Q P

If the refractive index of the material of the prism is  , then [KVPY_2014_SA] [2 Marks]
(A)  > 5 (B) 3 < < 5 (C) 2 < < 5 (D)  < 2

16. An optical device is constructed by fixing three identical convex lenses of focal lengths 10 cm each
inside a hollow tube at equal spacing of 30 cm each. One end of the device is placed 10 cm away from
a point source. How much does the image shift when the device is moved away from the source by
another 10 cm? [KVPY_2015_SA] [1 Mark]
(A) 0 (B) 5 cm (C) 15 cm (D) 45 cm

17. An isosceles glass prism with angles 40º is clamped over a tray of water in a position such that the
base is just dipped in water. A ray of light incident normally on the inclined face suffers total internal
reflection at the base. If the refractive index of water is 1.33 then the condition imposed on the refractive
index  of the glass is [KVPY_2015_SA] [1 Mark]
(A)   2.07 (B)   2.07 (C)   1.74 (D)   1.74

18. A point source of light is moving at a rate of 2 cm-s-1 towards a thin convex lens of focal length 10 cm
along its optical axis. When the source is 15 cm away from the lens the image is moving at
[KVPY_2015_SA] [1 Mark]
(A) 4 cm-s-1 towards the lens (B) 8 cm-s-1 towards the lens
(C) 4 cm-s-1 away from the lens (D) 8 cm-s-1 away from the lens

KVPY_ SA STREAM # 496


19. A girl sees through a circular glass slab(refractive index 1.50 of thickness 20 mm and diameter 60 cm
to the bottom of a swimming pol. Refractive index of water is 1.33. The bottom surface of the slab is in
contact with the water surface. [KVPY_2015_SA] [2 Marks]

The depth of swimming pool is 6m. The area of bottom of swimming pool that can be seen through the
slab is approximately.
(A) 100 m 2 (B) 160 m 2 (C) 190 m 2 (D) 220 m 2

20. A point object is placed 20 cm left of a convex lens of focal length f = 5 cm (see the figure). The lens is
made to oscillate with small amplitude A along the horizontal axis. The image of the object will also
oscillate along the axis with. [KVPY_2015_SA] [2 Marks]

(A) amplitude A/9, out of phase with the oscillations of the lens
(B) amplitude A/3, out of phase with the oscillations of the lens
(C) amplitude A/3, in phase with the oscillations of the lens
(D) amplitude A/9, in phase with the oscillations of the lens

21. A cubical vessel has opaque walls. An observer (dark circle in figure below) is located such that she
can see only the wall CD but not the bottom. Nearly to what height should water be poured so that she
can see an object placed at the bottom at a distance of 10cm from the corner C ? (Refractive index of
water is 1.33. [KVPY_2015_SB] [2 Marks]
A D

B C
(A) 10 cm (B) 16 cm (C) 27 cm (D) 45 cm

KVPY_ SA STREAM # 497


22. Electromagnetic waves emanating from a point A (in air) are incident on a rectangular block of material
M and emerge from the other side as shown. The angles I and r are angles of incidence and refraction
when the wave travels from air to the medium. Such paths for the rays are possible
[KVPY_2015_SB] [2 Marks]

i r

A
r i

air M air
(A) if the material has a refractive index very nearly equal to zero
(B) only with gamma rays with a wavelength smaller than atomic nuclei of the material
(C) if the material has a refractive index less than zero
(D) only if the wave travels in M with a speed faster than the speed of light in vacuum.
23. A hollow lens is made of thin glass and in the shape of a double concave lens. It can be filled with air, water
of refractive index 1.33 or CS2 of refractive index 1.6. It will act as a diverging lens if it is.
[KVPY_2016_SA] [1 Marks]
(A) filled with air and immersed in water (B) filled with water and immersed in CS2
(C) filled with air and immersed CS2 (D) filled with CS2 and immersed in water.
24. Mercury is often used in clinical thermometers. Which one of the following properties of mercury is not a
reason for this ? [KVPY_2016_SA] [1 Marks]
(A) The coefficient of the thermal expansion is large. (B) It is shiny.
(C) It is liquid at room temperature. (D) It has high density.
25. A rectangular block is composed of three different glass prisms ( with refractive indices 1, 2 and 3) as
shown in the figure below. A ray of light incident normal to the left face emerges normal to the right face. Then
the refractive indices are related by. [KVPY_2016_SA] [2 Marks]

1 3
2

45° 45°

(A)  12   22  2 23 (B)  12   22   32 (C)  12   23  2 22 (D)  22   23  212


26. A personlooks at the image of two parallel finite length lines PQ and RS in a convex mirror (see figure)
[KVPY_2017 _SA] [1 Marks]

P Q

R S

Which of the following represents schematically the image correctly ? (Note : letters P, Q, R and S are used
only to denote the endpoints of the lines.)
(Note : letters P, Q, R and S are used only to denote the endpoints of the lines.)

Q P
P Q Q P P Q

R S S R R S
S R

A. B. C. D.
(A) A (B) B (C) C (D) D
27. A point source of light is placed at 2f from a converging lens of focal length f. A flat mirror is placed on the
other side of the lens at distance d such that rays reflected from the mirror are parallel after passing through
the lens again. If f = 30 cm, then d is equal to [KVPY_2017_SA] [1 Marks]
(A) 15 cm (B) 30 cm (C) 45 cm (D) 75 cm

KVPY_ SA STREAM # 498


28. The word “ KVPY” is written on board and viewed through different lenses such that board is at a distance
beyond the focal length of the lens. [KVPY_2017_SA] [1 Marks]

KVPY

(i) (ii)

lgnoring magnification effects, consider the following statements.


(I) Image (i) has been viewed from the planar side of a plano-convex lens and image
(ii) from the convex side of a plano-convex lens.
(II) Image (i) has been viewed from the concave side of a plano-concave lens and image (ii) from the convex
side of a plano-convex lens.
(III) Image (i) has been viewed from the concave side of a plano-concave lens and image (ii) from the planar
side of a plano-convex lens.
(IV) Image (i) has been viewed from the planar side of a plano-concave lens and image (ii) from the convex
side of a plano-convex lens.
Which of the above statements are correct ?
(A) All four (B) Only (III) (C) Only (IV) (D) Only (II), (III) and (IV)

29. A student studying the similarities and differences between a camera and the human eye makes the follow-
ing observations : [KVPY_2017_SA] [1 Marks]
(I) Both the eye and the camera have convex lenses.
(II) In order to focus, the eye lens expands or contracts while the camera lens moves forward or backward.
(III) The camera lens produces upside down real images while the eye lens produces only upright real
images.
(IV) A screen in camera is equivalent to the retina in the eyes.
(V) A camera adjusts the amount of light entering in it by adjusting the aperture of the lens. In the eye the
cornea controls the amount of light.
The correct statements are :
(A) Only (I), (II), (IV) (B) Only (I), (III), (V) (C) Only (I), (II), (IV), (V) (D) All
30. If the image formed by a thin convex lens of power P has magnification m then image distance  is
[KVPY_2017_SA] [1 Marks]
1 m 1 m m 1  2m
(A)   (B)   (C)   (D)  
P P P P
31. A long horizontal mirror is next to a vertical screen (see figure). Parallel light rays are falling on the mirror at
an angle  from the vertical. If a vertical object of height h is kept on the mirror at a distance d > h tan().
The length of the shadow of the object on the screen would be. [KVPY_2017_SA] [2 Marks]

(A) h/2 (B) h tan () (C) 2h (D) 4h

KVPY_ SA STREAM # 499


32. Two plane mirrors are kept on a horizontal table making an angle  with each other as shown schematically
in the figure. The angle  is such that any ray of light reflected after striking both the mirrors returns parallel
to its incident path. For this to happen, the value of  should be [KVPY-SA_2018 1 Mark]
(A) 30° (B) 45° (C) 60° (D) 90°

33. Which of the following is NOT true about the total lunar eclipse? [KVPY-SA_2018 1 Mark]
(A) A lunar eclipse can occur on a new moon and full moon day.
(B) The lunar eclipse would occur roughly every month is the orbits of earth and moon were perfectly copla-
nar.
(C) The moon appears red during the eclipse because the blue light is absorbed in earth’s atmosphere and
red is transmitted.
(D) A lunar eclipse can occur only on a full moon day.

34. Select the correct statement about rainbow: [KVPY-SA_2018 1 Mark]


(A) We can see a rainbow in the western sky in the late afternoon.
(B) The double rainbow has red on the inside and violet on the outside.
(C) A rainbow has an arc shape since the earth is round.
(D) A rainbow on the moon is violet on the inside and red on the outside.

35. Remote sensing satellites move in an orbit that is at an average height of about 500 km from the surface of
the earth. the camera onboard one such satellite has a screen of area A on which the images captured by it
are formed. If the focal length of the camera lens is 50 cm, then the terrestrial area that can be observed from
the satellite is close to [KVPY-SA_2018 1 Mark]
(A) 2 × 10 A
3
(B) 10 A
6
(C) 10 A
12
(D) 4 × 1012 A

36. Let A,B,C and D are written on a cardboard as shown in the picture. [KVPY-SA_2018 1 Mark]
A

D B

The cardboard is kept at a suitable distance behind a transparent empty glass of cylindrical shape. If the
glass is now filled with water, one sees an inverted image of the pattern on the cardboard when looking
through the glass. Ignoring magnification effect, the image would appers as

A A A

B D
(A) (B) (C) (D)
C

37. A glass beaker is filled with water up to 5 cm. It is kept on top of a 2 cm thick glass slab. When a coin at the
bottom of the glass slab is viewed at the normal incidence from above the beaker, its apparent depth from the
water surface is d cm. Value of d is close (the refractive indices of water and glass are 1.33 and 1.50,respectively)
[KVPY-SA_2018 2 Mark]
(A) 2.5 (B) 5.1 (C) 3.7 (D) 6.0

KVPY_ SA STREAM # 500


EXERCISE # 1
1. (A) 2. (C) 3. (A) 4. (B) 5. (C) 6. (A) 7. (C)
8. (B) 9. (A) 10. (A) 11. (A) 12. (D) 13. (D) 14. (D)
15. (C) 16. (A) 17. (D) 18. (D) 19. (C) 20. (C) 21. (B)
22. (C) 23. (A) 24. (A) 25. (B) 26. (C) 27. (B) 28. (A)
29. (D) 30. (A) 31. (B) 32. (B) 33. (C) 34. (D) 35. (D)
36. (A) 37. (B) 38. (C) 39. (C) 40. (A) 41. (B) 42. (A)

EXERCISE # 2
1. (D) 2. (A) 3. (B) 4. (D) 5. (A) 6. (B) 7. (A)
8. (D) 9. (B) 10. (A) 11. (D) 12. (B) 13. (B) 14. (D)
15. (C) 16. (A) 17. (B) 18. (D) 19. (B) 20. (A) 21. (C)
22. (C) 23. (D) 24. (D) 25. (A) 26. (B) 27. (C) 28. (D)
29. (A) 30. (A) 31. (C) 32. (D) 33. (D) 34. (B) 35. (C)
36. (C) 37. (B)

1. 6 cm A 3 cm

167 cm

B P
Figure in self explanatory.
I V
2. =–
O u
I
If O and I are on same sides of PA . will be positive which implies v and u will be of opposite signs.
O
I
Similarly if O and I are on opposite sides, will be -ve which implies v and u will have same sign.
O
 V
If O is on PA, I =    (O)  I will also be on. P.A.
 u
 
3. V m = V0m (normal to plane mirror)
   
 V  Vm = – ( V0  Vm )

V  V sin  = – (0 – V sin) = V = 2V sin

4. As AB is common and O 1B = BI
O,BA and BAI are congruent
By symmetry AI is perpendicular to O 1 to O 2

and  O 1AB =  BAI


 BAI = 45°
and  BAC = 90°

KVPY_ SA STREAM # 501


5. By snell's law :
sin i  2

sin r 1
for i = 1, r = 4 and 1 = 1.
sin 1
2 = sin  .
4

6. As we know that light travels in a path such as to reach from one point to another in shortest possible
time.
Therefore, the man must travel along that path on which light would have travelled in moving from P to
Q.
By Snell's law ;
sin i  2

sin r 1

1
sin r = . sin i
2

3 4 4
sin r = . =
5 3 5
 r = 53º
 AQ = 10 Km.
From P to A :
5
t1 =
3
From A to Q :
10 5
t2 = =
4 2
5 5 25
T = t1 + t2 = + = hr..
3 2 6

 24 1   1 
=    hr =  4hr  hr  = 4hr + 10 minutes
 6 6  6 

x x rel
7.  x rel = x
1 

d2 x rel d2 x

dt 2 dt 2
arel =  g

8. A = 90° – 
r2 = A = 90° – > C A

6/5 4 r2
cos > sinC = = (C is critical angle)
3/2 5

4
  < cos–1 = 37° .
5 B C

KVPY_ SA STREAM # 502


9. r2 < C ; A – r1 < C
r1 > A – C  sinr1 > sin(A – C)
sin i A
 > sin (A – C)
 60º
i
 sini > (sinA cosC – sinC cosA) r1 r2

7  3 3 3 1  1 1
= 1  . = 1 
µ

3 2 7 7 2  2 2
B C
1
 sini >
2
or i > 300.

10. Given i = 60° A =  = e


= i + e–A  =i ( e = A) and = i = e
 A  m 
sin  
 2 
= A Here angle of deviation is minimum ( i = e)
sin
2

 60  60 
sin  
 2 
= = 3 Ans.
sin 60 / 2
11. Using formula of spherical surface taking 'B' as object
2 1  2  1 1
 = (R being the radius of the curved surface)  =2
 ( 2 R ) R 2

12. For spherical surface


n2 n1 n  n1
using – = 2
v u R
n 1 n 1
 – =  n = 2n – 2  n = 2.
2R  R
13. The image is erect hence,mirror must be between object & image. Virtual image of real object is
diminished, hence mirror is convex.

14. Put A = min and  = 2

 A   min 
sin  
 2 
The relation = and solve for A
A
sin 
2
vy(real) v
15. Let y-axis be vertically upwards and x-axis be horizontal.
Vy (real)
Vy (app.) =
 1
 
 = sin
-1(3/5)

Vx (app.) = Vx (real) vx(real)


Vy (app v kHkkl h) 4 4 3
tan  =  tan  =  1
Vx (app v kHkkl h) 3 3 4

16. Use refraction formulae seperatelly that is for air and  = 1.6 and for air and  = 2.0 and find the
positions of the two images.

KVPY_ SA STREAM # 503


17. Dispersion will not occur for a light of single wave length  = 4000 Å.
18. This question should be solved by directly substituting the options
If x = f, y = f then final image will be formed as shown.
For option B and C the position of image will be different.
When x = 2f, y = 2f, the lens makes image  of object O on the surface of
mirror as shown in the figure. Mirror shall make of image of  over  itself.
Hence lens shall make image of  at the position of O. (which is  )

v
19. v = – 30, m = – = – 2  AB = CD = 2 × 1 = 2 mm
u

BC A D v2
Now = = 2 = 4  BC = AD = 4 mm  length = 2 + 2 + 4 + 4 = 12 mm
BC AD u

20.

(i) (ii)
from figure (i) & (ii) it is clear that if the mirror moves distance ‘A’ then the image moves a distance ‘2A’.
21. Cutting a lens in transverse direction doubles their focal length i.e. 2f.
1 1 1 1 1
Using the formula of equivalent focal length    
f f1 f2 f3 f 4
f
We get equivalent focal length as .
2
22. (Moderate) Draw an incident ray along the top side of rectangular strip,which happens to be parallel to
the principal axis. After reflection this ray passes through focus. Hence image of all points (for e.g. O 1,
O 2, O 3, .......) on top side of the strip lie on this reflected ray (at I 1, I 2, I 3, .......) in between focus and
centre of curvature. Thus the image of this strip is a triangle as shown in figure

O1 O2 O3
F
I C
I3 I
2 I1

23. If object is at centre then image forms on the centre, and real and if object is virtual then image forms at
R
V=– (in both the cases image is real)
3

24.

From the digram  = 50º


25. (Tough)The only possibility is by reflection from concave mirror as shown.

nglass
26.  0. 9
nmaterial
1.512
nmaterial = = 1.68
0.9

KVPY_ SA STREAM # 504


27. 1 sin i = 2 sin .............(i)
2 sin = 3 sin r .............(ii)
From (i) and (ii)
1 sin i = 3 sin r
sin i  3

sin r 1

28. nA sin = nB sin 90º


C C C V
V = n = VB = n = VB = n sin   sin 
A B A

29. only half of the total rays will form the image so intensity is reduced so half.
30. for real object concave lens made the virtual image
31. this lens is behaves like as two different lesns one converging and other is diverging.

32. The effective focal length is 5 cm.


v f 5
The height of final images is =  O  ×O = × 1 = 2 cm
u uf  7 .5  5
33. For combination of n1 and n2 critical angle is 
When be places parallel slab of refractive index n3 (n3 < n1)
Then critical angle decreases and T.I.R. takes place.
For any value of n3 the ray will be return back in the medium n2 and reflection takes place at the surface
CD.
34. i + e – A = 40º
i + e = 100º (A = 60º)
difference between i and e is 20º.
So, i and e may be 40º and 60º.
35. Let the min. length of mirror be L
L 2
 =
4 8
 L=1m

36. Since deviation of mean ray is zero.


A, (m1 – 1) = A2 (m2 – 1)
6º (1.6 – 1) = A2 (1.9 – 1)
6 º 0.6
A2 = = 4º
0 .9
 
37. V 0  3 î  4 ĵ  5k̂ V m  4 î  5 ĵ  8k̂

VIZ = 2 Vmz – Voz = 2 × 8 – 5 = 11


VIx = Vox = 3
VIy = Voy = 4

V I  3 î  4 ĵ  11k̂

3 4 8
38. sin C = sin 90  C = sin–1  
2 3 9
39. µd sin i = µr sin 2i
µd = 2 r cos i

 µd 
 = cos–1  
µ
i = cos
–1 
 2µ 
 r  2

KVPY_ SA STREAM # 505


3
1– 1 1 1 1
40. 1 3 2 –  
–  V 40 20 40
V = 40 cm.
V 2  30  20

41.

1 1 1
= =–  v = + 30 cm
10 v ( 15 )
2
v2  30 
for small object dv = 2 du =    1 = 4 mm
u  15 

42. Focal length is minimum in case I, therefore power is maximum.

KVPY_ SA STREAM # 506


M AGN ET I C EFFECT OF CU RREN T AN D ELECT ROM AGN ET I C I N DU CT I ON

EARTH’S MAGNETIC FIELD


The earth behaves as a huge bar magnet with its magnetic field on its surface and upto a height of about 3 × 104 km.
The magnetic field of the earth is attributed to one or more of the following causes:-
(i) Another point of view attributes the earth’s magnetism is the presence of iron and nickel in the core of the
earth. It is assumed that the rotation of the earth about its own axis causes the magnetisation of the iron and
nickel and the earth behaves as a bar magnet.
(ii) Prof. Blackett, studied that the rotation of the earth about its own axis causes earth’s magnetism. For
example, the presence of ions in the upper atmosphere constitutes a current due to the rotation of the earth
about its own axis.
Long ago, it has been known that earth has a magnetic field, which is very similar to the field produced by a
huge magnet which is supposed to be at its centre. Its north pole lies towards the geographic south and south
pole towards the geographic north. The axis of the earth’s magnetic field does not coincide with the geographic
axis, the angle between them being about 150. The positions of the magnetic poles are not well defined on the
globe, they are spread over an area. The magnetic pole in the northern hemisphere is located on a peninsula
in northern Canada. The opposite magnetic pole in the southern hemisphere lies on the Antarctic continent,
south of Australia. The earth’s magnetic field is supposed to arise from electric currents in the molten iron rich
outer core surrounding the solid inner core of the earth.
Figure shows some field lines in the earth’s magnetic field. The field lines usually dip towards or come out of the
earth’s surface at some angle. This angle is called the magnetic inclination or the magnetic dip at that place. At
the magnetic poles, the angle of dip is 90º. The line on the earth’s surface passing through the places having
angle of dip 00 is called the magnetic equator, it passes through Thumba in south India, where the space
research centre is located.

Geographical North N
Magnetic South

Geographical Equator
N

Magnetic North
S Geographical South

ELEMENTS OF EARTH’S MAGNETIC FIELD

The following three quantities are called elements of earth’s magnetic field.
(a) Angle of declination :
(b) Angle of dip (or Inclination)
(c) Horizontal component of earth’s magnetic field
(a) Angle of Declination :
The vertical plane passing through the axis of a freely suspended magnet is called magnetic
meridian. The direction of earth’s magnetic field lies in the magnetic meridian and may not be
horizontal. The vertical plane passing through the true geographical north and south (or geographical axis of
earth) is called geographical meridian. The angle between the magnetic meridian and the geographic meridian
at a place is called angle of declination at that place. The knowledge of declination at a place helps in finding the
true geographical directions at that place. In our country, the angle of declination is zero in Pondicherry.

KVPY_ SA STREAM # 507


Geographic G
meridian Angle of
declination
O

Magnetic
meridian

(b) Angle of Dip (or Angle of Inclination) :


The angle between the axis of a freely suspended magnetic needle (which is free to rotate in the vertical plane)
and the horizontal plane at a place is called the angle of dip (or inclination) at that place.

Magnetic needle

Horizontal plane H

Angle of dip
M

Magnetic axis

(i) Angle of dip at the poles : The magnetic lines of force at the poles of earth are vertical due to which the
magnetic needle becomes vertical. Thus the angle of dip at the magnetic poles of the earth is 900.
0 900 60
60


0
0

30
30


0

Dip
00 00

Circle –
30
0


30
0

60

60
0

900 0

Angle of dip at the equator : The lines of force around the magnetic equator of the earth are perfectly horizontal.
So, the magnetic needle will become horizontal there. Thus, the angle of dip at the magnetic equator of the earth
will be 00 as shown in figure. The line on the earth’s surface passing through the places where the angle of dip is
00 is called magnetic equator.
90º 6
60º 0º

30º
30


º

Dip
0º 0º

Circle – 30–
º
30

º

º 60
60

º 90º

(c) Horizontal Component of Earth’s Magnetic Field :

KVPY_ SA STREAM # 508


Let I be the total intensity of earth’s magnetic field. At a place the total intensity I of earth’s magnetic field can be
resolved into two components i.e. horizontal component H and vertical component V.
H = I cos .......(i)
V = I sin .......(ii)
Where is the angle of dip at a place.
on dividing equation (ii) by (i)

V
tan = .......(iii)
H

from equation (i) and (ii)

I= V 2  H2 .......(iv)

BIOT-SAVART’S LAW (BDUE TO WIRE)

It is an experimental law. A current i flows in a wire (may be straight or curved). Due to d length of the wire the
magnetic field at P is :
dB  id  ....(i)

1
dB  ....(ii)
r2
dB  sin ....(iii)

Combining these equations, we get


i
id  sin  P
dB  2

r d r

  0  id sin 
dB =   2
 4  r
Vector representation

 0  id  r
 
dB =  4  r 3

Here r = position vector of the test point with respect to (w.r.t.) d



= angle between d and r .

Using this fundamental formula we can derive the expression of B due to a long wire. P

 I
(a) B due to a straight wire :
2
A
1

Due to a straight wire PQ carrying a current I, the B at A is given

Q
by the formula : r

0I 
B= (sin1 + sin2)  (i.e., direction of B is inward perpendicular to the plane of page)
4r

KVPY_ SA STREAM # 509


RULE FOR FINDING DIRECTION OF MAGNETIC FIELD
The direction of the magnetic field at a point P due to a straight wire can be found by a slight variation in the right
hand thumb rule. If we stretch the thumb of the right hand along the current and curl our fingers to pass through
the point P, the direction of the fingers at

P gives the direction of the magnetic field there.

ILLUSTRA TIONS

1. A horizontal overhead power line carries a current of 90 A in east to west direction. What is the magnitude and
direction of the magnetic field due to the current 1.5 m below the line ?
Sol. I = 90º A, r = 1.5 m
E W
20 I 4  10 7  90 S
B= = T
4r 2  1.5 N B
180
B=  10 7 T = 1.2 × 10–5T
1.5
Magnetic
 field is towards south.
( b ) B due to circ ular loop :

(i) At centre : Due to each d element of the loop, B at point c is inwards (in this case).
 Bresultant at point c is  . (inwards) I

  0NI 
B =  2R  , c R
 
N = No. of the turns in the loop.

= ,  = length of the loop
2R

 1 1 11 
N can be fraction  , , etc.  or integer..
4 3 3 

Direction of B : The direction of the magnetic field at the centre of a circular wire can be obtained using the
right-hand thumb rule. If the fingers are curled along the current, the stretched thumb will point towards the
magnetic field (see figure ).

Another way to find the direction is to look into the loop along its axis. If the current is in anticlockwise direction,
the magnetic field is towards the viewer. If the current is in clockwise direction, the field is away from the viewer.

KVPY_ SA STREAM # 510


Semicircular and Quarter of a circle :

1
N=
I 4
B=
8R

2. A wire as shown in figure carries a current I ampere. The circular part


has a radius r. The magnetic field at the centre C will be :

I
I
r C

Sol. Magnetic field at the centre due to AC and BC is zero.


I
3
the magnetic field at the centre will only due to the th part of the circular loop. I
4
A r C
r

3 0 I 30 I
Magnetic field at C =  =
4 2r 8r B

MAGNETIC FORCE ON MOVING CHARGE



When a charge q moves with velocity v in a magnetic field B , then the magnetic force experienced by moving
charge is given by following formula :
  
F = q( v  B ), (Put value of q with sign), v and B remain in the same plane

v : Instantaneous velocity

B : Magnetic field at that point.

 NOTE :
   
(i) F  v and also F  B (Here  is symbol for perpendicular)
 
(ii)   F  v

power due to magnetic force on a charged particle is zero.



(use the formula of power P = F.v for its proof).
 
(iii) Since F  B so work done by magnetic force is zero in every part of the motion. The magnetic force cannot
increase or decrease the speed (or kinetic energy) of a charged particle. It can only change the direction of
velocity.

(iv) On a stationary charged particle, magnetic force is zero.


 
(v) If v || B , then also magnetic force on charged particle is zero. It moves along a straight line if only magnetic
field is acting. (Here || is symbol for parallel)

KVPY_ SA STREAM # 511


3. An infinitely long straight conductor ‘XY’ is carrying a current of 5 A. An electron is moving with a speed of 105 m/
s parallel to the conductor in air from point A to B, as shown in figure. The perpendicular distance between the
electron and the conductor ‘XY’ is 20 cm. Calculate the magnitude of the force experienced by the electron. Write
the direction of the force.
electron B
A

20cm

x 5A Y
Sol. Magnetic field at a distance of 20 cm from current carrying conductor XY is

20 I
B= electron
4r A B

2  10 7  5
B= T 20cm
20  10 2
B = 5 × 10–6 T
Force experienced by the electron is x 5A Y
F = evB (  = 90º)
= 1.6 ×10 × 10 × 5 × 10–6 N
–19 5

= 8 × 10–20 N
According to Fleming’s left hand rule direction of force will be upwards.

MAGNETIC FLUX
If we consider a plane perpendicular to a uniform magnetic field, then the product of the magnitude of the field and
the area of the plane is called the magnetic flux () linked with that plane. Figure shows a plane of area A placed

perpendicular to a uniform magnetic field B . The magnetic flux linked with this plane is given by
 = BA

If the magnetic field B , instead of being perpendicular to the plane, makes an angle  with the perpendicular to
the plane as shown in figure, then the magnetic flux linked with the plane will be equal to the product of the
component of the magnetic field perpendicular to the plane and the area of the plane.
Thus,  = (B cos) A = BA cos .......(i)

Case I : If  = 0º, then from equation (i)

 = BA cos0= BA (outgoing flux)

KVPY_ SA STREAM # 512


Case II : If  = 90º, then from equation (i)
A
B

 = BA cos90= 0

, Case III : If  = 180º, then from equation (i)

 = BA cos180= – BA (incoming flux) 


 is positive if the outward normal to the plane is in the same direction as B . It is negative if the outward normal

is opposite to B .

S.I. unit of B and :

The S.I. unit of magnetic flux  is weber (Wb). Since B = /A, the magnetic field is also expressed in
weber/metre2, (Wb-m–2). That is why the magnetic field induction B is also called the magnetic flux density.

Definition of magnetic flux density (B) :


B = , if A = 1 metre2, then B = 
A

In a magnetic field the number of lines of force (flux) passing through per metre2 perpendicular to the field is equal
to the magnetic flux density.

FARADAY’A LAWS OF ELECTROMAGNETIC


INDUCTION
(i) When magnetic flux passing through a loop changes with time or magnetic lines of force are cut by a conducting
wire then an e.m.f. is produced in the loop or in that wire. This e.m.f. is called induced e.m.f.
If the circuit is closed then the current will be called induced current.
(ii) In case of loop, the magnitude of induced e.m.f. is equal to the rate of change of flux w.r.t. time . In case of a
wire it is equal to the rate at which magnetic lines of force are cut by a wire.
d
E= –
dt
(–) sign indicates that the e.m.f. will be induced in such a way that it will oppose the change of flux. S.I. unit of
magnetic flux is weber, so S.I. unit of induced e.m.f. is
Wb
= Volt
Second

KVPY_ SA STREAM # 513


LENZ’S LAW
According to this law, e.m.f. will be induced in such a way that it will oppose the cause which has produced it.
Figure shows a magnet approaching a ring with its north pole towards the ring.

We know that magnetic field lines come out of the north pole and magnetic field intensity increases as we move
towards the magnet.
If we consider the approach of North pole to be the cause of flux change, the lenz’s law suggests that the side of
the coil towards the magnet will behave as North pole and will repel the magnet. We know that a current carrying
coil will behave like North pole if current in it flows anticlockwise, as seen in figure.
If we consider the approach of magnet as the cause of the flux change, Lenz’s law suggest that a force opposite
to the motion of magnet will act on the magnet, whatever be the mechanism. Lenz’s law tells that if the coil is set
free, it will move away from magnet, because in doing so it will oppose the ‘approach’ of magnet
If the magnet is given some initial velocity towards the coil and is released, it will slow down. It can be explained
as the following.
The current induced in the coil will produce heat. From the energy conservation, if heat is produced, there must
be an equal decrease of energy in some other form, here it is the kinetic energy of the moving magnet. Thus the
magnet must slow down. So we can justify that the lenz’s law is conservation of energy principle.

SELF INDUCTANCE
When current flows through a coil or circuit, magnetic field is produced and hence a magnetic flux gets associated
with this coil or circuit. This magnetic flux is directly proportional to the current flowing in the circuit (If other
factors remain constant). If current through the coil is changed, the magnetic field is produced and hence the
magnetic flux associated with it changes and as a result of which, an e.m.f. is induced in the coil or circuit,
According to Lenz’s law the direction of induced e.m.f. is such as that it always opposes change due to which it
is produced. As shown in figure, if current increases in circuit, induced e.m.f. is set up in such a way that it will
decrease the current i.e., induced current produced due to induced e.m.f. flows opposite to the main current.
Similarly, if main current decreases in the circuit, induced e.m.f. will increase it. Now, the induced current due to
induced e.m.f. will flow in the direction of main current.

In this way the phenomenon, in which, on changing the current in a coil or circuit an induced e.m.f. is set up in
that very coil or circuit, is called self induction. This induced e.m.f. is called back e.m.f.
(a) Coefficient of Self Induction or Self Inductance :

When current I flows in a circuit, associated magnetic flux  is proportional to the current flowing i.e.,
 or = L .....(i)

KVPY_ SA STREAM # 514


Here, L is a constant of proportionality, called the Coefficient of self-induction or self-inductance.
Self-inductance of a coil depends on its area, number of turns and the medium inside it (material of core). If  =
1A, then L =  . Hence, self-inductance of a coil is equal to the magnetic flux associated with the coil when unit
current flows through it.
In equation (i), on changing the current , will also change and an induced e.m.f. E is produced in the circuit i.e.,
d() d(L ) d
E=   L
dt dt dt
d
or L = |E| if = 1 A/s
dt
Hence, the coefficient of self induction in a circuit is numerically equal to the induced e.m.f. produced in the
circuit when the rate of change of current is 1 A/s. When flow of current in the circuit starts, induced e.m.f.
opposes this change.
4. An average induced e.m.f. of 0.20 V appears in a coil when the current in it is changed from 5.0 A in one direction
to 5.0 A in the opposite direction in 0.20s. Find the self-inductance of the coil.

di ( 5.0 A )  (5.0 A )
Sol. Average = = – 50 A/s
dt 0.20 s
di
Using E = – L ,
dt
0.2 V = L (50 A/s)
0.2V
or, L =  4.0mH
50 A / s
MUTUAL INDUCTANCE
Consider two arbitrary conducting loops 1 and 2. Suppose that I1 is the instantaneous current flowing in loop 1.
This current generates a magnetic field B1 which links the second circuit, giving rise to a magnetic flux 2 through
the second circuit.

Furthermore, it is obvious that the flux through the second circuit is zero whenever the current flowing around the
first circuit is zero. It follows that the flux 2 through the second circuit is directly proportional to the current I1
flowing around the first circuit. Hence ,we can write 2 = M21I1 where the constant of proportionality M21 is called
the mutual inductance of circuit 2 with respect to circuit 1. Similarly, the flux 1 through the first circuit due to the
instantaneous current I2 flowing around the second circuit is directly proportional to that current, so we can write
1 =M12I2 where M12 is the mutual inductance of circuit 1 with respect to circuit 2. It can be shown that M21 = M12.

Note: M is a purely geometric quantity, depending only on the size, number of turns, relative position and
relative orientation of the two circuits. The S.I. unit of mutual inductance is called Henry (H). One henry is
equal to a volt-second per ampere.
Suppose that the current flowing around circuit 1 changes by an amount I1 in a small time interval  t. The flux
linking circuit 2 changes by an amount 2 = MI1 in the same time interval. According to Faraday’s law, an e.m.f.
2
E2 = – is generated around the second circuit due to the changing magnetic flux linking that circuit, Since,
t

I1
2 = MI1, this e.m.f. can also be written E2 = – M
t

KVPY_ SA STREAM # 515


Thus, the emf generated around the second circuit due to the current flowing around the first circuit is directly
proportional to the rate at which that current changes. Likewise, if the current I2 flowing around the second circuit

changes by an amount I1 in a time interval  t then the e.m.f. generated around the first circuit is E1 = –M I 2 .
t
Note that there is no direct physical connection (coupling) between the two circuits the coupling is due entirely
due to the magnetic field generated by the currents flowing around the circuits.
 NOTE :
(i) M  L1L 2 (Here L is inductance of coil)
(ii) For two coils in series if mutual inductance is considered then
Leq = L1 + L2 ± 2M ( M = mutual inductance )
5. Two conducting circular loops of radii R1 and R2 are placed in the same plane with their centres coinciding. Find
the mutual inductance between them assuming R2 << R1.
Sol. Suppose a current i is established in the outer loop. The magnetic field at the centre will be :
0i
B = 2R
1
As the radius R2 of the inner coil is small compared to R1, the flux of magnetic field through it will be approximately
 0i
= R 22
2R1
So that the mutual inductance is
  o R22
M= 
i 2R1
TRANSFORMER
It is a device which raises or lowers the voltage in AC circuits through mutual induction. It consists of two coils
wound on the same core. The coil which is connected to the source (i.e. to which input is applied) is called
primary coil while the other which is connected to the load (i.e. from which output is taken) is called secondary
coil. The alternating current passing through the primary coil creates a continuously changing flux through the
core. This changing flux induces an alternating e.m.f. in the secondary coil. As magnetic lines of force are closed
curves, the flux per turn of the primary must be equal to the flux per turn of the secondary coil.

P and S are the flux passing through the primary and secondary coils, NP and NS are the number of the turns in
primary and secondary coils respectively,
then,
S 
 P
NS NP
The number of turns in each coil is constant, if dP and dS are the changes in flux in time dt in primary and
secondary coils respectively,
then,
1 d S 1 d P ES NS  d 
 or  as E  dt 
NS dt NP dt EP NP  
And as in an ideal transformer there is no loss of power, so P = EI = constant, therefore
ES IP NS
 
EP IS NP

KVPY_ SA STREAM # 516


TYPES OF TRANSFORMER
Transformer can be divided mainly into two types:
(a) Step up transformer (b) Step down transformer
(a) Step up transformer :
If the secondary coil has a greater number of turns than the primary (Ns > Np), the voltage is stepped up (Vs > Vp).
This type of arrangement is called a step-up transformer. In this arrangement, there is less current in the secondary
than in the primary (Np/Ns < 1 and Is< Ip ).
Eg.: If the primary coil of a transformer has 100 turns an the secondary has 200 turns,
Ns Np 1
Np = 2 and Ns
= . Thus, a 220 V input at 10 A will step-up to 440 V output at 5.0 A.
2
(b) Step down Transformer :
If the secondary coil has less number of turns than the primary (Ns < Np), the voltage is stepped down
(Vs < Vp). This type of arrangement is called a step-down transformer. In this arrangement, current in the
secondary coil is more increased than in the primary (Np/Ns < 1 and Is> Ip ).

Efficiency of Transformer :
Efficiency of transformer,
Power output Po
= % =
Power input Pi ×100
 NOTE :
Regarding a transformer it is worth noting that :
(i) It works on AC only and never on DC
(ii) It can increase or decrease either voltage or current but not both simultaneously (as power = constant).
(iii) Some power is always lost due to flux leakage, hysteresis, eddy currents, humming and heating of coils.
6. A step-down transformer converts a supply line voltage of 2200 volt into 220 volt. The primary coil has 5000
turns. The efficiency and power transmitted by the transformer are 90% and 8 kilowatt respectively. Calculate the
number of turns in the secondary coil.
e s Ns
Sol. 
ep Np

220 N
 s  Ns = 500
2200 Np

7. The primary winding of a transformer has 500 turns whereas its secondary has 5000 turns. The primary is
connected to an AC supply of 20 V, 50 Hz. What will be output of secondary coil ?
Sol. We know that
NS ES 5000 ES
 or =
NP EP 500 20
5000  20
or Es = = 200 V
500
Frequency remains same.
FORCE BETWEEN TWO PARALLEL INFINITELY
LONG CURRENT CARRYING CONDUCTORS
The force per unit length between two long parallel conductors carrying currents I1 and I2 and separated by
distance r is given by
µ0 I 2 I 2
f .
2 r
The force is attractive when the current are in the same direction and repulsive when the currents are in opposite
directions.

KVPY_ SA STREAM # 517


TORQUE ON CURRENT CARRYING COIL IN A
MAGNETIC FIELD


A rectangular coil of area A, carrying current I and capable of rotation about an axis perpendicular to the field B
experiences a torque,
 = NIBA sin = mBsin
where N = number of turns in the coil, m = NIA = magnetic dipole moment,  angle which the normal to the plane

of the coil makes with the field B .



In vector rotation,  = m × B

Torque is minimum when the plane of the coil is perpendicular to the magnetic field ( = 0º). Torque is maximum
when the plane of the coil is parallel to the magnetic field ( = 90).
max = NIBA.

MOVING COIL GALVANOMETER


It is a device used to detect current in a circuit. It is based on the principle that a current carrying coil placed in
a magnetic field experiences a torque, the magnitude of which depends on the strength of current. It consists of
a coil of wire of area. A and N turns carrying current I to be measured. It is suspended in a radial magnetic field

so that its plane always remains parallel to B (sin  =1) by a suspension fibre of torism constant K. In equilibrium
position.

Restoring torque = Deflecting torque

NBA
or k  NIBA or  = .I
k
i.e., deflection of coil  current in the coil.

(i) Figure of merit of a galvanometer : It is the current which produces a deflection of one scale division in the
galvanometer. It is given by
 k
G= =
 NBA

(ii) Current sensitivity of a galvanometer : It is the deflection produced in a galvanometer when a unit current
flows through it.
 NBA
Current sensitivity = =
 k

(iii) Voltage sensitivity of a galvanometer : It is the deflection produced in a galvanometer when a unit
potential difference is applied across its ends.

  NBA
Voltage sensitivity = = =
V R kR

KVPY_ SA STREAM # 518


DIFFERENCE BETWEEN DIAMAGNETIC SUBSTANCE, PARAMANGETIC AND FERROMAGNETIC SUBSTANCES

Property Diamagnetic substance Paramagnetic substances Ferromagnetic substances

1. Effect of They are feebly repelled They are feebly attracted They are strongly attracted
magnets by magnets. by magnets. by magnets.

2. In external Acquire feeble magnetisation Acqurie feeble magnetisa- Acqurie strong magnetisa-
magnetic in the opposite direction tion in the direction of the tion in the direction of the
field magnetising field. magnetising field.

3. In a non- Tend to move slowly from Tend to move slowly from Tend to move quickly from
uniform stronger to weaker parts weaker to stronger parts weaker to stronger parts
magnetic of the field. of the field. of the field.
field
4. In a uniform A freely suspended diamag- A freely suspended param- A freely suspended ferro-
magnetic netic rod aligns itself agnetic rod aligns itself magnetic rod aligns itself
field prependicular to the field parallel to the field. parallel to the field.

5. Susceptibility Susceptibility is very larger


value (xm) and positive xm > 1000

6. Relative Of the order of thousands


permeability µr > 1000
value (µ)

7. Permeability µ < µ0 µ > µ0 µ > > µ0


value (µ)

Effect of Susceptibility is independent Susceptibility is varies Susceptibility decreases


8.
temperature of temperature. inversely as temperature : with temperature in a complex
manner.
1
1
T (T > TC)
T–TC

9. Examples Bi, Cu, Pb,Si N2 (at STP), Al, Na, Ca, O2 (at STP), Fe, Ni, Co, Gd, Fe2O3, Anion
H2O, NaCl CuCl2

HYSTERESIS LOOPS
Figure shows the variation of magnetic induction B with magnetising field intensity H. point O represents the
initial unmagnetised state of a ferromagnetic sample. As the magnetising field intensity H increases, the magnetic
induction B first gradually increases and then attains a constant value. In other words, the magnetic induction B
saturates at a certain value + Hmax.
B

A
Saturation
B
Initial
build up
–Hmax
H
F +Hmax
OB = Rentivity
E OC = Coercivity
D
Saturation

Now if the magnetising field intensity H is gradually decreased to zero, B decreases but along a new path AB. It
is found that the magnetic induction B does not become zero even when the magnetising field H is zero, i.e., the
sample is not demagnetised even when the magnetising field has been removed. The magnetic induction (=OB)
left behind in the sample after the magnetising field has been removed is called residual magnetism or retentivity
or remanence.

KVPY_ SA STREAM # 519


To reduce the magnetism to zero, the field H is gradually increased in the reverse direction, the induction B
decreases and becomes zero at a value of H = OC. The value of reverse magnetising field intensity H required for
the residual magnetism of a sample to become zero is called corecivity of the sample. On further increasing H in
the reverse direction to a value

– Hmax, we reach the saturation point D located symmetrically to point. A now if H is decreased gradually, the point
A reached after going through the path DEFA.
The closed curve ABCDEFA which represents a cycle of magnetisation of a ferromagnetic sample is called its
hysteresis loop. Throughout the cycle, the magnetic field B lags behind the magnetising field intensity H, i.e.,
the value of B when H is decreasing is always more than when H is increasing. The magnetising field is called
hysteresis. In fact , the word hysteresis originates from a Greek word meaning "delayed" .
Significance of the area of hysteresis loop :

B B2
The product BH = B   = , has the dimensions of energy per unit volume. Hence the area within the B-
 µ  µ0µr
H loop represent the energy dissipated per unit volume in the material when it is carried through a cycle of
magnetisation. The source is the source of emf used in magnetising the material and the sink is the hysteretic
heat loss in the magnetic material.
Types of ferromagnetic materials : Ferromagnetic materials can be divided into two categories.

(a) Soft ferromagnetic materials or soft ferromagnets:

These are the ferromagnetic materials in which the magnetisation disappears on the removal of the external
magnetising field. Such materials have narrow hysteresis loop, materials have low retentivity, low coercivity, and
low hysteresis loss. But they have high relative magnetic permeability. They are used as cores of solenoids and
transformers. Example soft iron, mu metal, stalloy, etc.

B B

H H

(a) (b)

(b) Hard ferromagnetic materials or hard ferromagnets :

These are the ferromagnetic materials which retain magnetisation even after the removal of the external magnetising
field, Such materials have wide hysteresis loop, as shown in fig. consequently, they have high retentivity, high
coercivity and large hysteresis loss. They are used for making permanent magnets.
Example : Steel, alnico, iodestone, ticonal etc.

KVPY_ SA STREAM # 520


1. In the figure shown a square loop PQRS of side 'a' and resistance 'r' is
placed near an infinitely long wire carrying a constant current . The sides
PQ and RS are parallel to the wire. The wire and the loop are in the same Q R
plane. The loop is rotated by 180º about an axis parallel to the long wire
a
and passing through the mid points of the side QR and PS. The total
amount of charge which passes through any point of the loop during rotation P S
a
is : a

 0 a  0 a  0 a 2
(A) n2 (B) n2 (C)
2r r 2r
(D) cannot be found because time of rotation not give.

2. A wooden stick of length 3 is rotated about an end with constant angular velocity  in a uniform
magnetic field B perpendicular to the plane of motion. If the upper one third of its length is coated with
copper, the potential difference across the whole length of the stick is

9B 2 4B 2 5B 2 B  2


(A) (B) (C) (D)
2 2 2 2

3. A wire of fixed length is wound on a solenoid of length '' and radius 'r'. Its self inductance is found to be L.
 r
Now if same wire is wound on a solenoid of length and radius , then the self inductance will be :
2 2
(A) 2 L (B) L (C) 4 L (D) 8 L

4. Figure shows three regions of magnetic field, each of area A, and in


X X
each region magnitude of magnetic field decreases at a constant rate X XX
X X
X XX
 X X X X
 

. If E is induced electric field then value of line integral E . d r along
the given loop is equal to
(A) A (B) –A
(C) 3A (D) –3A

5. In an ideal transformer, the voltage and the current in the primary are 200 volt and 2 amp respectively.
If the voltage in the secondary is 2000 volt. Then value of current in the secondary will be :
(A) 0.2 amp (B) 2 amp (C) 10 amp (D) 20 amp

6. A superconducting loop of radius R has self inductance L. A uniform & constant magnetic field B is applied
perpendicular to the plane of the loop. Initially current in this loop is zero. The loop is rotated by 180º. The
current in the loop after rotation is equal to
BR 2 2B R 2 BR 2
(A) zero (B) (C) (D)
L L 2L

KVPY_ SA STREAM # 521


7. In the figure shown the section EDFG is fixed. A rod having resistance ' R '
is moved with constant velocity in a uniform magnetic field B as shown in
the figure. DE & FG are smooth and resistanceless. Initially capacitor is
uncharged. The charge on the capacitor:
(A) remains constant (B) increases with time
(C) increases linearly with time (D) oscillates.

8. A vertical rod of length  is moved with constant velocity v towards East. The vertical component of the
earth's magnetic field is B and the angle of dip is . The induced e.m.f. in the rod is :
(A) B  v cot  (B) B  v sin  (C) B  v tan  (D) B  v cos 

9. Two identical cycle wheels (geometrically) have different number of spokes connected from centre to rim.
One is having 20 spokes and other having only 10 (the rim and the spokes are resistanceless). One resistance
of value R is connected between centre and rim. The current in R will be :
(A) double in first wheel than in the second wheel
(B) four times in first wheel than in the second wheel
(C) will be double in second wheel than that of the first wheel
(D) will be equal in both these wheels.
10. A constant force F is being applied on a rod of length '' kept at rest on two
parallel conducting rails connected at ends by resistance R in uniform
magnetic field B as shown.
(A) the power delivered by force will be constant with time
(B) the power delivered by force will be increasing first and then will decrease
(C) the rate of power delivered by the external force will be increasing continuously
(D) the rate of power delivered by external force will be decreasing continuously.


11. A uniform magnetic field exists in region given by B  3 î  4 ĵ  5 k̂ . A rod of length 5 m is placed along y 
axis is moved along x  axis with constant speed 1 m/sec. Then induced e.m.f. in the rod will be:
(A) zero (B) 25 volt (C) 20 volt (D) 15 volt

12. In a L-R growth circuit, inductance and resistance used are 1 H and 20  respectively. If at t = 50
millisecond, current in the circuit is 3.165 A then applied direct current emf is :
(A) 200 V (B) 100 V
(C) 50 V (D) Data is insufficient to find out the value.

13. Figure shows a square loop of side 1 m and resistance 1 . The magnetic field on left side of line PQ has a
magnitude B = 1.0T. The work done in pulling the loop out of the field uniformly in 1 s is

(A) 1 J (B) 10 J (C) 0.1 J (D) 100 J

14. A and B are two metallic rings placed at opposite sides of an infinitely long straight conducting wire as
shown. If current in the wire is slowly decreased, the direction of induced current will be :

(A) clockwise in A and anticlockwise in B (B) anticlockwise in A and clockwise in B


(C) clockwise in both A and B (D) anticlockwise in both A & B

KVPY_ SA STREAM # 522


15. A vertical conducting ring of radius R falls vertically with a speed V in a horizontal uniform magnetic
field B which is perpendicular to the plane of the ring :

(A) A and B are at same potential (B) C and D are at same potential
(C) current flows in clockwise direction (D) current flows in anticlockwise direction

16. Two identical conducting rings A & B of radius R are in pure rolling over a
horizontal conducting plane with same speed (of center of mass) but in
opposite direction. A constant magnetic field B is present pointing inside
the plane of paper. Then the potential difference between the highest points
of the two rings, is :
(A) zero (B) 2 Bvr (C) 4Bvr (D) none of these

17. An inductor L and a resistor R are connected in series with a direct current source of emf E. The
maximum rate at which energy is stored in the magnetic field is :

E2 E2 4E2 2E2
(A) (B) (C) (D)
4R R R R

18. In the circuit shown in figure, the switch S was initially at position C
R 2
1. After sufficiently long time, the switch S was thrown from S
position 1 to position 2. The voltage drop across the resistor at L 1
that instant is :

E
E
(A) zero (B) E (C) LC (D) none of these
R
19. As shown in the figure, the key K is closed, the direction of the induced current in the coil B will be-

B
+ – k
E
A

(A) clockwise and instantaneous (B) anti-clockwise and instantaneous


(C) clockwise and continuous (D) anti-clockwise and continuous

20. A copper ring is tied to a string and suspended vertically. On bringing a magnet towards the coil, as
shown in the figure -

(A) the ring will move away from magnet (B) the ring will move towards the magnet
(C) the ring will remain stationary (D) none of the above

KVPY_ SA STREAM # 523


21. The graph between the current and the time for an inductance coil is shown below. Which of the
following graph show the voltage-time variation -

I
t

v v v v
(A) t (B) t (C) t (D) t

22. A metallic rod completes its circuit as shown in the figure. The circuit
is normal to a magnetic field of B = 0.15 tesla. If the resistances of the
rod is 3the force required to move the rod with a constant velocity of v=2m/s
50cm
2 m/sec is -

(A) 3.75 × 10–3 N (B) 3.75 × 10–2 N


B=0.15T (into page)
(C) 3.75 × 102 N (D) 3.75 × 10–4 N
23. A small square loop of wire of side  is placed inside a large square loop of wire of side L (L >>
). The loop are coplanar and their centres coincide. The mutual inductance of the system is pro-
portional to -

 2 L L2
(A) (B) (C) (D)
L L  

24. A metallic square loop ABCD is moving in its own plane with velocity v in a uniform magnetic field
perpendicular to its plane as shown in the figure. An electric field is induced -

A B

D C

(A) in AD, but not in BC (B) in BC, but not in AD

(C) neither in AD, nor in BC (D) in both AD and BC

25. P and Q are two circular thin coils of same radius and subjected to the
× × × × B×
same rate of change of flux. If coil P is made up of copper and Q is made
× × × × ×
up of iron, then the wrong statement is - P Q
× × × × ×
(A) emf induced in the two coils is the same
× × × × ×
(B) the induced current in P is more than that in Q
(C) the induced current in P and Q are in the same direction
(D) the induced currents are the same in both the coils

KVPY_ SA STREAM # 524


26. Three identical circular coils A, B and C are placed coaxial (with planes parallel to each other). The
coils A and C carry equal currents in opposite direction as shown . The coils B and C are fixed in
positions and the coil A is moved towards B with uniform motion then -

(A) there is no induced current in B .


(B) current produced by A and C in B will be equal and opposite, therefore net current in B is zero
(C) there is an induced current in B which is in anticlockwise direction
(D) there is an induced current in B which is in clockwise direction

27. Two plane circular coils P and Q have radii r1 and r2, respectively, Q
(r1 << r2) and are coaxial as shown in fig. The number of turns in P and r2 r1
Q are respectively N1 and N2. If current in coil Q is varied steadily at a N1
P
rate x ampere/second then the induced emf in the coil P will be approximately
(A)  0 N1N2 r12 N2

(B)  0 N1N2 r12 x

(C)  0 N1N2 r12 x / 2r2


(D) zero

28. A conducting rod AB of length l = 1 m is moving at a velocity v = 4 m/s making an angle 30º with its length.
A uniform magnetic field B = 2T exists in a direction perpendiuclar to the plane of motion. Then

A 30º B

(A) VA – VB = 8V (B) VA – VB = 4 V (C) VB – VA = 8V (D) VB – VA = 4 V

29. The horizontal component of earth’s magnetic field is 3 × 10–5 Wb/m2. The magnetic flux linked with a
coil of area 1 m2 and having 5 turns, whose plane is normal to the magnetic field, will be -
(angle of dip = 30º)
(A) 3 × 10–5 Wb (B) 5 × 10–5 Wb (C) 15 × 10–5 Wb (D) zero

30. The north pole of a magnet is brought near a coil. The induced current in the coil as seen by an observer
on the side of magnet will be -
(A) in the clockwise direction
(B) in the anticlockwise direction
(C) initially in the clockwise and then anticlockwise direction
(D) initially in the anticlockwise and then clockwise direction

31. The self-inductance of a coil is 2H. The current in the coil changes from 8A to 2.95A in 0.01s. The time
constant of the coil will be-
(A) 2s (B) 0.1s (C) 10 ms (D) 1 ms

32. Area of a coil is 0.16 m2. If the magnetic field through it changes from 0.1 Wb/m2 to 0.5 Wb/m2 in 0.02s,
then the emf induced in the coil will be-
(A) 1.6 V (B) 3.2 V (C) 4.8 V (D) 6.4V

KVPY_ SA STREAM # 525


33. Which of the following quantities is increased in a step-down transformer -
(A) current (B) voltage (C) power (D) frequency

34. If in a coil rate of change of area is 5m2/milli second and current becomes 1amp from 2 amp in 2 × 10–3 sec.
If magnetic field is 1 tesla then self inductance of the coil is -
(A) 2H (B) 5H (C) 20H (D) 10H

35. An electron is moving near to a conducting loop then the induced current in the loop is -

A B

e
(A) clockwise (B) anticlockwise
(C) first anticlockwise and then clockwise (D) no current

36. The magnetic flux linked with the coil varies with time as  = 3t2 + 4t + 9. The magnitude of induced
emf at t = 2 second is -
(A) 4 V (B) 3 V (C) 16 V (D) 9 V

37. A metallic conductor of 1 m length is rotated vertically its one end at an angular velocity of
–4
5 rad/sec. If the horizontal component of earth's field is 0.2 × 10 T, the voltage generated at both
ends of the conductor will be-
–4
(A) 5 mV (B) 5 × 10 V (C) 50 mV (D) 50 µ V

38. The resistance of a coil is 5 ohm and a current of 0.2 A is induced in it due to a varying magnetic field.
The rate of change of magnetic flux in it will be -
(A) 0.5 Wb/s (B) 0.05 Wb/s (C) 1 Wb/s (D) 20 Wb/s

39. If the length and area of cross-section of an inductor remain same but the number of turns is doubled,
its self-inductance will become -
(A) half (B) four times (C) double (D) one-fourth

40. Self-inductance of a solenoid depend on -


(A) the number of turns N of the coil (B) the area of cross-section A and length of the coil.
(C) the permeability of the core of the coil (D) all the above

41. When current flowing in a coil changes from 3A to 2A in one millisecond, 5 volt emf is induced in it.
The self-inductance of the coil will be -
(A) zero (B) 5kH (C) 5H (D) 5 mH
42. In the figure magnetic energy stored in the coil is (in steady state)
2H

10V

(A) 0 (B)  (C) 25 joules (D) none of these

43. The rate of change of current in the inductor initially is

(A) 0 (B) /L (C) /2L (D) 2/L

KVPY_ SA STREAM # 526


44. Both the inductors and the cell are ideal. Find the current (in Amperes) 2H inductance in steady state.

(A) zero (B) 1A (C) 2A (D) 3A

45. A metallic square wire frame ABCD of side a is moving with a constant velocity v in a uniform magnetic field
B as shown. The emf induced between points A and C is :

(A) 0 (B) avB (C) 2 avB (D) a2vB

KVPY PROBLEMS (PREVIOUS YEARS)


1. Two charges of same magnitude move in two circles of radii R1 = R and R2 = 2R in a region of constant

uniform magnetic field B0 . The work W 1 and W 2 done by the magnetic field in the Two cases, respectively
are such that [KVPY_2009_SA]
(A) W 1 = W 2 = 0 (B) W 1 > W 2 (C) W 1 = W 2  0 (D)W 1 < W 2

2. A circular loop of wire is in the same plane as an infinitely long wire carrying a constant current i. Four
possible motions of the loop are marked by N,E,W and S as shown. A clockwise current is induced in the
loop when loop is pulled towards : [KVPY_2012_SA]

(A) N (B) E (C) W (D) S

3. An arangement with a pair of quarter circular coils of radii r and R with a common centre C and carrying a
current I is shown. [KVPY_2013] [1 Marks]

The permeability of free space is 0. The magnetic field at C is


(A) 0 I(1/r – 1/R)/8 into the page (B) 0 I (1/r – 1/R)/8 out of the page
(C) 0 I(1/r + 1/R)/8 out of the page (D) 0 I(1/r + 1/R)/8 into the page

KVPY_ SA STREAM # 527


4. An electron enters a chamber in which a uniform magnetic field is present as shown. Ignore gravity.
[KVPY_2013] [1 Marks]

During its motion inside the chamber


(A) the force on the electron remains constant
(B) the kinetic energy of the electron remains constant
(C) the momentum of the electron remains constant
(D) the speed of the electron increases at a uniform rate

5. An electron enters a chamber in which a uniform magnetic field is present as shown [KVPY_2013] [1 Marks]

An electric field of appropriate magnitude is also applied so that the electron travels undeviated without any
change in its speed thorugh the chamber. We are ignoring gravity. Then, the direction of the electric field is
(A) opposite to the direction of the magnetic field
(B) opposite to the direction of the electron’s motion
(C) normal to the plane of the paper and coming out of the plane of the paper
(D) normal to the plane of the paper and into the plane of the paper

6. A blackbox (BB) which may contain a combination of electrical circuit elements (resistor, capacitor or inductor)
is connected with other external circuit elements as shown below in the figure (a). After the switch (S) is
closed at time t = 0, the current (I) as a function of time (t) is shown in the figure (b). [KVPY_2013] [1 Marks]

From this we can infer that the blackbox contains


(A) A resistor and a capacitor in series (B) A resistor and a capacitor in parallel
(C) A resistor and an inductor in series (D) A resistor and an inductor in parallel

7. The figure shows a bar magnet and a metallic coil. Consider four situations. [KVPY_2013] [1 Marks]
(I) Moving the magnet away from the coil. (II) Moving the coil towards the magnet.
(III) Rotating the coil about the vertical diameter. (IV) Rotating the coil about its axis.

An emf in the coil will be generated for the following situations.


(A) (I) and (II) only (B) (I), (II) and (IV) only (C) (I), (II), and (III) only (D) (I),(II), (III), and (IV)

KVPY_ SA STREAM # 528


8. A charged particle, initially at rest at O, When released follows atrajectory as shown. Such a trajectory is
possible in the presence of [KVPY_2014_SA] [1 Mark]

o
(A) Electric field of constant magnitude and varying direction
(B) Magnetic field of constant magnitude and varying direction
(C) Electric field of constant magnitude and constant direction
(D) Electric and magnetic fields of constant magnitudes and constant directions which are parallel to each
other

9. A bar magnet falls with its north pole pointing down through the axis of a copper ring. When viewed from
above, the currecnt in the ring will be [KVPY_2014_SA] [1 Mark]
(A) Clockwise while the magnet is above the plane of the ring and counter clockwise while below the plane
of the ring
(B) Counter clockwise throughout
(C) Counter clockwise while the magnet is above the plane of the ring, and clockwise while below the plane
of the ring
(D) Clockwise throughout.

10. Two identical bar magnets are held perpendicular to each other with a certain separation, as shown below.
The area around the magnets is divided into four zones [KVPY_2014_SA] [1 Mark]

N S I

N
II S V

Given that there is a neutral point it is located in


(A) Zone I (B) Zone II (C) Zone III (D) Zone IV

11. The diagram below shows two circular loops of wire (A and B) centred on and perpendicular to the
x-axis, and oriented with their planes parallel to each other. The y-axis passes vertically through loop A
(dashed line). There is a current IB in loop B as shown . Possible actions which we might perform on loop A
are : [KVPY_2016_SA] [1 Mark]

IB
–x

B
A

(i) Move A to the right along x-axis closer to B


(ii) Move A to the left along x-axis away from B
(iii) As viewed from above, rotate A clockwise about Y-axis
(iv) As viewed from above, rotate A anticlockwise about y-axis
Which of these actions will induce a current in A only in the direction shown.
(A) only (i) (B) only (ii) (C) only (i) and (iv) (D) only (ii) and (iii)

KVPY_ SA STREAM # 529


EXERCISE-1
1. (B) 2. (C) 3. (A) 4. (B) 5. (A) 6. (C) 7. (B)
8. (A) 9. (D) 10. (D) 11. (B) 12. (B) 13. (A) 14. (B)
15. (B) 16. (C) 17. (A) 18. (B) 19. (B) 20. (A) 21. (C)
22. (A) 23. (B) 24. (D) 25. (D) 26. (C) 27. (C) 28. (B)
29. (B) 30. (B) 31. (C) 32. (B) 33. (A) 34. (D) 35. (C)
36. (C) 37. (D) 38. (C) 39. (B) 40. (D) 41. (D) 42. (C)
43. (C) 44. (D) 45. (B)
EXERCISE-2
1. (A) 2. (B) 3. (B) 4. (B) 5. (D) 6. (C) 7. (C)
8. (A) 9. (C) 10. (B) 11. (A)

1 d  µ0I a
1. q=  I dt   – r dt dt  – r

r
n2 .

2. When the rod rotates, there will be an induced current in the rod. The given situation can be treated as
if a rod 'A' of length '3' rotating in the clockwise direction, while an other say rod 'B' of length '2 '
rotating in the anti clockwise direction with same angular speed ''.
1
As we know that Sa] e = B2
2
For 'A' : A % For 'B' : B
1 1
eA = B (3)2 & eB = B (– ) (2)2
2 2
Resultant induced emf will be :
1 5
e = eA + eB = B2 (9 – 4) e= B2
2 2

 0 N2  r 2
3. L=

length of wire = N 2r = Constant (= C, suppose)
2
 C  r2 1
 L = 0    L
 2r   
 Self inductance will become 2L.

  d
4. E . dr = –
dt
and take the sign of flux according to right hand curl rule.
 
E . d r = – (– (–A) – (–A) + (–A)) = –A

KVPY_ SA STREAM # 530


5. Given :
Voltage in primary VP = 200 volt
Current in primary i p = 2 amp
Voltage in secondary Vs =2000 volt
The relation for the current in the secondary is
Vs ip

Vp is

2000 2 2  200
 or is  = 0.2 amp.
200 is 2000

6. Flux can't change in a superconducting loop.  = 2  R2. B


Initially current was zero. So self flux was zero.

2  R2  B
 Finally L i = 2  R2  B. i=
L

q  dq  q
7. BlV = iR + BlV =  dt  R +
or
C   C
Hence the charge on capacitor increases with time

8. BH = Bv cot  = B cot 
Hence the induced e.m.f. in the rod is B  v cot 

9. Since all the wires are connected between rim and axle so they will generate induced emf in parallel, hence
it is same for any number of spokes.

dP dF.v Fdv
10. = = = Fa
dt dt dt
as 'a' is decreasing continuously hence the rate of power delivered by external force will be decreasing
continuously.

 
11. e = (v  B ). 
e = [ î  ( 3 î  4 ĵ  5k̂ ) ] . 5 ĵ  e = 25 volt

1
12. Time constant = = 50 msec
20

E 3.165  20
so i = 0.633 i max = 0.633  E= = 100 V
R 0.633

13. W = (L)F
= L × LB

L2B 2 V
=L× =1J
R
14. The field at A and B are out of the paper and inside the paper respectively.

KVPY_ SA STREAM # 531


As the current in the straight wire decreases the field also decreases.
For B :


 B


The change in the magnetic field which causes induced current ( B ) is along (+)z direction.

Hence, induced emf and hence current should be such as to oppose this change  B.
Hence, induced emf should be along – z direction which results in a clockwise current in 'B'. Similarly,
there will be anticlockwise current in 'A'. Hence (B).

15. When the ring falls vertically, there will be an induced emf across A & B (e
= Bv (2r)).
Note that there will be a potential difference across any two points on the
ring of line joining them has a projected length in the horizontal plane.
For example, between points 'P' & 'Q' there is a projected length 'x' in the
horizontal plane.
 P.d. across P & Q is :
V = B v x.
But for points C and D : x = 0.
Therefore; P.d. = 0.
Hence (B).
16. Considering a projected length 2R on the ring in vertical plane.
This length will move at a speed v perpendicular to the field. This results in
an induced emf :
e = Bv(2R) in the ring.
e = Bv(2R) in the ring.
In Ring "A" : eA =B(-v)(2R)
In Ring "B" : eB = B(v)(2R)
Therefore, potential differnce between A & B = B(v)(2R) - B(-v)(2R) = 4 BvR.
Note : there wil be no p.d. across a diameter due to rotation.
Alternate – Considering rotation of diameter about lowest point :

B (2 r )2
e= = 2Bvr in A (since pure rotation).
2
and e = – 2Bvr in B.
Hence (C)

17. The graph of current is given by :


di i0
i = i 0 (1 – e–t/ )  = e–t/
dt 
Energy stored in the form of magnetic field energy is :
1 2
UB = Li
2
 Rate of increase of magnetic field energy is :

dUB di Li 20
R= = Li = (1 – e–t/ ) e–t/
dt dt 

KVPY_ SA STREAM # 532


dR
This will be maximum when =0
dt
 e–t/ = 1/2
Substituting :

Li 20 1  1   1  Li 20  L (E / R )2  E2
Rmax =     = =  4 (L / R)  =
  2 2 4   4R
18. When the switch is at position 1 :

1 2 LE 2
UB = Li =
2 0 2R 2
E
Just after the switch is shifted to position 2, current  = is flowing across the resistance. Hence, at
R
that instant P.d. across reisistance will be :
E
V = R = .R = E
R
Hence (B).
di
21. e=–L i = Kt
dt
22. e = Bv
e B v
i= 
R R
F – iB = 0
F = iB= 3.75 × 10–3 N
23.  = Mi

4 0i  1 1  2
      Mi
4(L / 2)  2 2

2 2 0  2
M=
L

2
M
L
24. Magnetic field lines are cut by rod AD and BC.

25. Induced emf will be same but resistance of both loop will be dfferent.

27.  = Mi

Mdi  0N1N2 r12 x


e= = Mx =
dt 2r2

29.   BH A

I 2.95
31.  = 0.368
Imax 8
then t =  = 0.01 s = 10 ms

KVPY_ SA STREAM # 533


(B 2  B1 )A
32. e= = 3.2 V
t

 Li
34. 
t t

A Li
B  L = 10 H
t t
36. q = 3t2 + 4t + 9
d
| v |  = 6t + 4
dt
= 6 × 2 + 4 = 12 + 4 = 16 volt
–4
37.  = 1m, B = 0.2 × 10 T,  = 5 rad/s
1
e= V Bwl 2
2
38. R = 5 , i = 0.2A,
d
V = i × R = 5 × 0.2 = 1volt
dt

1wb
Rate of chage of magnetic flux = 1volt =

39. L  n2

V 5
41.     3  5  10 3 H = 5 mH
i 10
t

1
42. U= × L I2
2
2
1  10 
= × 2 × 
2  2 
= 1 × 52 = 25 J

10
44. In steady state current from battery = = 5A
2
In parallel inductors L1I1 = L2I2 all the time
L2 3
 i1 = L  L i = × 5 = 3A
1 2 32

   
45. Emf = ( v  B).L (Here L  AB  BC  –a î  a ĵ )

= v î  B(k̂ ). a( î )  aĵ


   
= vB j .   a i  a j 
 
= vBa

KVPY_ SA STREAM # 534


N U CLEAR PH YSI CS
THOMSON MODEL OF AN ATOM
J.J. Thomson (1898) tried to explain the arrangement of electrons and protons within the atom. He proposed
that an atom consists of a sphere of positive electricity in which electrons are embedded like plum in pudding
or seeds evenly distributed in red spongy mass in case of a watermelon. The radius of the sphere is of the order
10–8 cm.
+
+
+ Electron
+
+ + +
Positive charge
+
+

(a) Merits :
(i) Thomson’s model could explain the electrical neutrality of an atom.
(ii) Thomson’s model could explain why only negatively charged particles are emitted when a metal is heated
as he considered the positive charge to be immovable by assuming it to be spread over the total volume of the
atom.
(iii) He could explain the formation of ions and ionic compounds.

(b) Demerits :
This model could not satisfy the facts proposed by Rutherford through his alpha particle scattering experiment
and hence was discarded.

RUTHERFORD MODEL OF AN ATOM

(a) Rutherford’s Alpha Particle Scattering Experiment (1909) :


Ernest Rutherford and his coworkers performed numerous experiments in which - particles emitted from a
radioactive element such as polonium were allowed to strike thin sheets of metals such as gold or platinum.
(i) A beam of -particles (He2+) was obtained by placing polonium in a lead box and letting the alpha particles
come out of a pinhole in the lead box. This beam of -rays was directed against a thin gold foil (0.0004 cm). A
circular screen coated with zinc sulphide was placed
on the other side of the foil.
(ii) About 99.0% of the -particles passed undeflected through the gold foil and caused illumination of zinc
sulphide screen.
(iii) Very few -particles underwent small and large deflections after passing through the gold foil.
(iv) A very few (about 1 in 20,000) were deflected backward on their path at an angle of 180º.

Rutherford was able to explain these observations as follows:


(i) Since a large number of -particles pass through the atom undeflected, hence, there must be large empty
space within the atom.

KVPY_ SA STREAM # 535


(ii) As some of the -particles got deflected, therefore, there must be something massive and positively charged
structure present in the atom.
(iii) The number of -particles which get deflected is very small, therefore, the whole positive charge in the
atom is concentrated in a very small space.
(iv) Some of the -particles retracted their path i.e. came almost straight back towards the sources as a result
of their direct collisions with the heavy mass.

 Note :
- particles are made up of two protons and two neutrons and are Helium (He) nuclei.

(b) Rutherford Nuclear Model of Atom (1911) :


Rutherford proposed a new picture of the structure of atom.
Main features of this model are as follows-
(i) The atom of an element consists of a small positively charged “Nucleus” which is situated at the centre of the
atom and which carries almost the entire mass of the atom.
(ii) The electrons are distributed in the empty space of the atom around the nucleus in different concentric
circular paths (orbits).
(iii) The number of electrons in the orbits is equal to the number of positive charges (protons) in the nucleus.
(iv) Volume of nucleus is very small as compared to the volume of atom.
(v) Most of the space in the atom is empty.
The stability of such a system in which negatively charged electrons surrounding a positively charged nucleus
was explained by proposing that the electrons revolve around the nucleus at very high speed in circular orbits.
This arrangement is just like our solar system. The high speed of the moving electrons give them a centrifugal
force acting away from the nucleus.
The centrifugal force balance the electrostatic force of attraction acting between the nucleus and the electrons.
 Note :
Rutherford’s model is also called “Planetary model’.

(c) Defects in Rutherford’s Model :

(i) Rutherford did not specify the number of electrons in each orbit.

(ii) According to electromagnetic theory, if a charged particle (like electron) is accelerated around another
charged particle (like protons in nucleus) then there would be continuous radiation of energy. This loss of
energy would slow down the speed of electron and eventually the electron would fall into the nucleus. But such
a collapse does not occur. Rutherford’s model could not explain this theory.
(iii) If the electron loses energy continuously, the observed spectrum should be continuous but the actual
observed spectrum consists of well defined lines of definite frequencies. Hence the loss of energy is not
continuous in an atom.
BOHR MODEL OF AN ATOM (1913)
To overcome the objections to Rutherford’s model and to explain the hydrogen spectrum, Bohr proposed a
quantum mechanical model of the atom.
The important postulates on which Bohr’s model is based are the following -

KVPY_ SA STREAM # 536


(i) The atom has a nucleus where all the protons are present. The size of the nucleus is very small. It is present
at the centre of the atom.
(ii) Each stationary orbit is associated with a definite amount of energy. The greater is distance of the orbit from
the nucleus, more shall be the energy associated with it. These orbits are also called energy levels and are
numbered as 1,2, 3,4 ------or K, L M, N ---- from nucleus to outwards.
(iii) By the time, the electron remains in any one of the stationary orbits , it does not lose energy. Such a stage
is called ground or normal state.
(iv) The emission or absorption of energy in the form of radiation can only occur when an electron jumps from
one stationary orbit to another.
E = Efinal - Einitial = h
Where h is Planck’s constant h = 6.625 × 10–34 Js Energy is absorbed when the electron jumps from lower
to higher orbit and is emitted when it moves from higher to lower orbit.
When the electron moves from inner to outer orbit by absorbing definite amount of energy, the new state of the
electron is said to be excited state.
(v) Negatively charged electrons are revolving around the nucleus in the same way as the planets are revolving
around the sun. The path of the electron is circular. The force of attraction between the nucleus and the
electron is equal to centrifugal force of the moving electron.
Force of attraction towards nucleus = Centrifugal force
KZe2 mv 2
2 =
r r
(vi) Out of infinite number of possible circular orbits around the nucleus, the electron can revolve only in those
h h
orbits whose angular momentum is an integral multiple of , i.e. mvr = n where m = mass of the electron,
2 2
v = velocity of electron, r = radius of the orbit and n =1,2,3 ---- number of the orbit. The angular momentum can
h 2h 3h
have values such as , , , but it cannot have a fractional value. Thus, the angular momentum is
2 2 2
quantized. The specified circular orbits (quantized) are called stationary orbits.

RADII OF VARIOUS ORBITS


Consider an electron of mass ‘m’ and charge ‘e’ revolving around the nucleus of charge ‘Ze’ (Z = atomic
number). Let ‘v’ be the velocity of the revolving electron and ‘r’ the radius of the orbit. The electrostatic force of
attraction between the nucleus and electron (applying Coulomb’s law)

KZe e KZe2
= =
r2 r2
v
Centrifugal
e force

Coulombic
force

+Ze

1
Here K is a constant. It is equal to ,  being absolute permittivity of medium. In SI units , the numerical
4 0 0
1
value of 4 is equal to 9 × 109 Nm2/c2.
0

KZe2 mv 2 KZe2
So, 2 = or v 2 =
r r mr
1 Ze 2
v 2 = 4 × -------- (i)
0 mr

KVPY_ SA STREAM # 537


According to one of the postulates,
h
Angular momentum = mvr = n
2
nh
or v = ------- (ii)
2mr
Putting the value of ‘v’ in eq. (i),

n 2h2 KZe2 n2h 2


2 2 2 = or = KZe2
4 m r mr 4 2mr

n2h2
or r = -------- (iii)
4 2mKZe 2
 NOTE :
Greater is the value of ‘n’ larger is the size of atom. On the other hand, greater is the value of ‘Z’ smaller is the
size of the atom.
Across the period from left to right, atomic number ‘Z’ increases with constant value of ‘n’ hence atomic radius
decreases towards right. On moving down the group, both ‘Z’ and ‘n’ increase but due to effective nuclear
charge remains same. Hence, on moving downwards, atomic radius increases due to increase in ‘n’.
n2h2
For hydrogen atom, Z = 1; so r =
42mKe2
Now putting the values of h, , m, e and K.
n 2  (6.625  10 –34 )2
r= 4  (3.14)2  (9.1 10 – 31 )  (9  10 9 )  (1.6  10 –19 ) 2

= 0.529 ×n2 × 10–10 m = 0.529 × n2 Å


= 0.529 × 10–8 × n2 cm
Thus, radius of 1st orbit
= 0.529 × 10–8 × 12 = 0.529 × 10–8 cm = 0.529 × 10–10 m = 0.529 Å
Radius of 2nd orbit
= 0.529 × 10–8 × 22 = 2.11 × 10–8 cm = 2.11 × 10–10 m = 2.11 Å
Radius of 3rd orbit
= 0.529 × 10–8 × 32 = 4.76 × 10–8 cm = 4.76 × 10–10 m = 4.76 Å
and so on
 rn = r1 ×n2 for hydrogen atom

n2
and rn = 0.529 × Å for hydrogen like species.
Z
ENERGY OF AN ELECTRON IN BOHR’S ORBIT
Let the total energy of the electron be E. It is the sum of kinetic energy and potential energy.
E = kinetic energy + potential energy
1 KZe2
E= mv 2 –
2 r
Putting the value of mv 2 from eq. (i),
K Ze2 KZe2 KZe2
E – –
2r r 2r
Putting the value of r from eq. (iii).

KVPY_ SA STREAM # 538


K Ze 2 4π 2mKZe 2 2π 2 Z2K 2me 4
E   – ---- (iv)
2 n2h 2 n2h 2
For hydrogen atom, Z = 1
So,

2π 2K 2me 4
E–
n 2h 2
Putting the values of , K, m, e and h.
2  (3.14)2  (9  109 )2  (9.1 10 –31 )  (1.6  10 –19 ) 4
E=–
n 2  ( 6.625  10 – 34 )2

21.79  10 –19
=– J per atom
n2
13.6
= – eV per atom (1 J = 6.2419 × 1018 eV)
n2
313.6
=- kcal/mol (1 eV = 23.06 kcal/mol)
n2
1312
=- kJ/mol
n2
Substituting the values of n = 1,2,3,4, ---- etc. , the energy of electron in various energy shell in hydrogen atom
can be calculated.
E1
En = for hydrogen atom
n2
Z2
and En = E1 × for hydrogen atom like species
n2
where E1 = energy of electron in first orbit of hydrogen.
Since, n can have only integral values, it follows that total energy of the electron is quantised.
 NOTE :
The negative sign indicates that the electron is under attraction towards nucleus, i.e. it is bound to the nucleus.
The electron has minimum energy in the first orbit and its energy increases as n increases, i.e., it becomes less
negative. The electron can have a maximum energy value of zero when n =  . The zero energy means that the
electron is no longer bound to the nucleus , i.e. , it is not under the force of attraction towards nucleus.
VELOCITY OF AN ELECTRON IN BOHR’S ORBIT
We know that centrifugal force on electron = force of attraction between nucleus and electron

mv 2 Ze 2
= (in CGS units)
s) ------------ (i)
r r2
The angular momentum of an electron is given as
mvr = nh/2 ------------ (ii)
From (i) and (ii) , we have
 nh 
v   = Ze2
 2 
Z  2e 
2

v=  
n  h 
Substituting the values of h, , e.
Z 2  3.14 ( 4.8 10 10 )2
v= ×
n 6.625  10 27
Z
v= × 2.188 × 108 cm/sec ----------- (iii)
n

KVPY_ SA STREAM # 539


2.188  10 8
v= cm/sec (For hydrogen , Z = 1)
n
v 1 = 2.188 × 108 cm/sec
1
v2 = × 2.188 × 108 cm/sec
2

1
v3 = × 2.188 × 108 cm/sec
3
Here v 1, v 2 and v 3 are the velocities of electron in first, second and third Bohr orbit in hydrogen.

PARTICLE AND WAVE NATURE OF ELECTRON


In 1924, de Broglie proposed that an electron, like light, behaves both as a material particle and as a wave. This
proposal gave birth to a new theory, known as wave mechanical theory of matter. According to this theory , the
electrons , protons and even atoms, when in motion , posses wave properties.
De Broglie derived an expression for calculating the wave length of the wave associated with the electron.
According to Planck’s equation -
c
E = h = h ----- (i)

The energy of a photon on the basis of Einstein ‘s mass energy relationship is -
E = mc2 ------ (ii)
where c is the velocity of light.
Equating both the equations , we get
c h
h = mc2  =
 mc
None of the particles can move with the speed of light, therefore, replacing ‘c’ by ‘v’, we get -
h h
= = ------ (iii)
mv p
This is the de-Broglie equation.
Thus, momentum of the moving electron is inversely proportional to its wavelength.
Let kinetic energy of the particle of mass ‘m’ is E.
1
E= mv 2
2
(On multiplying both sides with m)
2Em = m2v 2
2Em = mv = p (momentum)
h h
= =
p 2Em
Davisson and Germer made the following modification in de Broglie equation -
Let a charged particle say an electron be accelerated with a potential V; then the kinetic energy may be given
as -
1
mv 2 = eV
2
(On multiplying both sides with m)

m2v 2 = 2eVm
mv = 2eVm = p
Putting the value of p in equation (iii), we get -
h
=
2eVm

KVPY_ SA STREAM # 540


 NOTE :
The wavelength decreases if the value of mass (m) increases, i.e. in the case of heavier particles, the wavelength
is too small to be measured, de Broglie equation is applicable in the case of smaller particles like electron and
has no significance for larger particles.

BOHR’S THEORY VERSUS DE-BROGUE EQUATION

h
One of the postulates of Bohr’s theory is that angular momentum of an electron is an integral multiple of .
2
This postulate can be derived with the help of de - Broglie concept of wave nature of electron.
Consider an electron moving in a circular orbit around nucleus. The wave train would be associated with the
circular orbit as shown in the figure.

If the two ends of the electron wave meet to give a regular series of crests and troughs, the electron wave is
said to be in phase , i.e., the circumference of Bohr’s orbit is equal to whole number multiple of the wavelength
of the electron wave.

So, 2r = n
2r
= -------(i)
n
From de Broglie equation ,
h
= ------ (ii)
mv
h 2r
Thus, =
mv n
h
or mvr = n [v = velocity of electron and r = radii of the orbit]
2
h
i.e., Angular momentum = n . ----- (iii)
2
This proves that the de Broglie and Bohr concepts are in perfect agreement with each other.

ISOTOPES
The isotopes of an element have the same atomic number but different mass number.
 NOTE :
The term isotope was given by Margaret Todd.
The difference in their masses is due to the presence of different number of neutrons.
e.g. Isotopes of hydrogen :

Hydrogen Pr otiu m D e u te r iu m T r itium


isotopes 1 2 3
1H 1H 1H

1. Atom ic number 1 1 1
2. No. of protons 1 1 1
3. No. of electrons 1 1 1
4. No. of neutrons 0 1 2
5. Mass number 1 2 3

KVPY_ SA STREAM # 541


Isotopes of oxygen :

16 17 18
8O 8O 8O
Oxygen isotopes

1. Atomic number 8 8 8
2. No. of protons 8 8 8
3. No. of electrons 8 8 8
4. No. of neutrons 8 9 10
5. Mas s number 16 17 18

 NOTE :
All the isotopes of an element have identical chemical properties.

(a) Characteristics of Isotopes :


(i) The physical properties of the isotopes of an element are different. This is due to the fact that isotopes have
different numbers of neutrons in their nuclei. Hence mass, density and other physical properties of the isotopes
of an element are different.
(ii) All the isotopes of an element contain the same number of electrons. So, they have the same electronic
configuration with the same number of valence electrons. Since the chemical properties of an element are
determined by the number of valence electrons in its atom,all the isotopes of an element have identical chemical
properties.

(b) Reason for the Fractional Atomic Masses of Elements :

The atomic masses of many elements are in fraction and not whole number.The fractional atomic masses of
elements are due to the existence of their isotopes having different masses.
e.g.
35 37
The atomic mass of chlorine is 35.5 u, chlorine has two isotopes 17 Cl and 17 Cl with abundance of 75% and
25% respectively. Thus the average mass of a chlorine atom will be 75% of 35 and 25% of 37, which is 35.5 u.
So,
75 25
Average atomic mass of chlorine = 35 × + 37 ×
100 100
2625 925
= +
100 100
= 26.25 + 9.25
= 35.5 u.
Thus, the average atomic mass of chlorine is 35.5 u.
(c) Applications of Radioactive Isotopes :
(i) In agriculture : Certain elements such as boron, cobalt, copper, manganese, zinc and molybdenum are
necessary in very minute quantities for plant nutrition. By radioactive isotopes we can identify the presence and
requirement of these element in the nutrition of plants.
(ii) In industry : Isotopes are used for coating on the arm of clock to see in dark. To identify the cracks in metal
casting.
(iii) In medicine : Thyroid, bone diseases, brain tumors and cancer are controlled or destroyed with the help
24 131 32
of radioactive isotope 11 Na, 53 Ι and 15P etc.
(iv) Determination of the mechanism of chemical reaction by replacing an atom or molecule by its isotopes.
(v) In carbon dating : Will and Libby (1960) developed the technique of radiocarbon to determine the age of
plant, fossil and archeological sample.
 NOTE :
Isotopes (Like Uranium-238) are used in Nuclear reactor to produce energy and power.

KVPY_ SA STREAM # 542


ISOBARS
The atoms of different elements with different atomic numbers, which have same mass number are called
isobars.
e.g. 146 C and 14
7N are isobars.
40 40
18 Ar and 20 Ca are isobars.

40 40
Isobars
18 Ar 20 Ca
1. Atomic number 18 20
2. Mass number 40 40
3. No. of electrons 18 20
4. No. of protons 18 20
5. No. of neutrons 22 20
6. Electronic configuration 2, 8, 8 2, 8, 8, 2

 NOTE
Isobars contain different number of electrons, protons and neutrons.

ISOTONES
Isotones may be defined as the atoms of different elements containing same number of neutrons.
e.g. 136 C and 14
7N (Both contain 7 neutrons)
30 31 32
14 Si , 15 P and 16 S (All three contain 16 neutrons)

ISOELECTRONIC
Ion or atom or molecule which have the same number of electrons are called as isoelectronic species.
  2
e.g.
17 Cl 18 Ar 19 K 20 Ca
No. of electrons 18 18 18 18

RADIOACTIVITY
The spontaneous and uncontrollable disintegration of the nucleus of certain heavy atom (like uranium, thorium,
radium and polonium) with emission of alpha particles or beta particles or gamma rays is called radioactivity.
For example, a uranium– 238 atom disintegrates on its own and forms a new element thorium– 234 by emitting
an alpha particle and energy in the form of gamma radiations :
238 234 4
92 U 
 90Th  2 He  Energy
  
Uranium– 238 Thorium–234 Alpha particle
This is an example of natural radioactivity. Radioactivity leads to the transformation (or changing) of one element
into another element. In the above example, the uranium element has been converted into thorium element
during radioactivity. The invisible rays emitted by the radioactive elements consists of three different types of
radiations. These are :
(a)-particle :
It is a doubly charged helium nucleus. It contains two protons and two neutrons.
Mass of -particle = Mass of 2He4 atom – 2me = 4 mp
Charge of -particle = + 2 e
(b) -particle :
(i)  – (electron) :
Mass = me ; Charge = –e

KVPY_ SA STREAM # 543


(ii)  + (positron) :
Mass = me ; Charge = +e
positron is an antiparticle of electron.
(c) -particle :
Gamma rays are electromagnetic ray having short wavelength. Gamma rays have no charge and no mass.
Gamma ray is represented by the symbol . Gamma rays have high energy and high speed, so they are most
penetrating of the three radiations. Gamma rays can penetrate into about 15 cm of steel and upto 10 cm in
body tissues. The penetrating power of gamma rays is even greater than that of X-rays because of which they
are very dangerous to human beings.
RADIOACTIVE DECAY (DISPLACEMENT LAW)
A radioactive change in which a nucleus emits an alpha or beta particle and gamma rays, to form a slightly
smaller, more stable nucleus is called radioactive decay.
(i) Effect of alpha emission (-decay) :
When a radioactive nucleus emits an alpha particle, then the new nucleus formed has an atomic number which
is 2 less than the original nucleus, and a mass number, which is 4 less than the original nucleus.

Z
XA   decay
   YA–4 + 2He4 + Q
Z–2

Q value : It is defined as energy released during the decay process


When a uranium– 238 radioisotope (or nuclide) emits an alpha particle,
238  decay 234 4
92U   90Th  2 He

Uranium– 238 Thorium– 234 Alpha particle


(New element)
(ii) Effect of beta particle emission ( -decay) :

When a radioactive nucleus emits a beta particle, then the new nucleus formed has an atomic number which is
1 greater than that of the original nucleus, but the mass number remains the same.
Z XA Z+1 YA + e0 + Q
–1

e0 can also be written as –1 0.


–1

238   decay 238 0


92 U   93 Np  -1e

Uranium– 238 Neptunium– 238 Beta particle

 b+ decay :

Z
XA  Z–1YA + +1e0 +  + Q

In  + decay, inside a nucleus a proton is converted into a neutron, positron and neutrino.
1
1p  0 n1 + e0 + 
1

(iii) Effect of gamma ray emission ( - decay) :

Gamma rays are electromagnetic waves having no mass and no charge, 00. So, there is no change in the
atomic number or mass number during the emission of gamma rays.

ILLUSTRATIONS
234
1. In radioactive decay 234
90 Th 92 U the number of  and particles emitted respectively are -

Sol. No. of particles

Difference in atomic mass of reactants and products


=
4

KVPY_ SA STREAM # 544


234 – 234 0
= = =0
4 4
No. of -particle=( 2 × no. of -particles) – (ZA – ZB)
where , ZA = Atomic number of reactant
ZB = Atomic number of product
= (2 × 0) – (90 – 92)
= 0 – (–2) = 2
number of -particle = 0
number of  -particle = 2
2. In the nuclear reaction -
14
7 N + 24 He  17 8 O + ----------
The missing particle is -

7 N  2 He  17
8 O x A
14 4 y
Sol.
Atomic number and mass number are conserved in a nuclear reaction. So,
sum of atomic number of reactants = sum of atomic number of products
7+2=8+x
x=1
And , sum of mass number of reactants = sum of mass number of products
14 + 4 = 17 + y
y = 1

So, the atomic number of the particle is 1 and mass number of the particle is also 1. Thus, the particle is 11H or
proton.
RADIOACTIVE DECAY : STATISTICAL LAW :
(Given by Rutherford and Soddy)
Rate of radioactive decay  N
where N = number of active nuclei
=N
where = decay constant of the radioactive substance.
Decay constant is different for different radioactive substances, but it does not depend on amount of
substance and time.
S unit of  is s–1
If 1  2 then first substance is more radioactive (less stable) than the second one.
For the case, if A decays to B with decay constant 

A  B
t=0 N0 0 where N0 = number of active nuclei of A at t = 0
t=t N N’ where N = number of active nuclei of A at t = t
dN N
Rate of radioactive decay of A = – = N
dt N0

N t
dN
– 
N0
N 
 dt 
0
N = N0 e–t (it is exponential decay) t

N´ = N0 – N
Number of nuclei decayed (i.e. the number of nuclei of B formed)
N’ = N0 – N N0
= N0 – N0e–t
N´ = N0(1 – e–t) t
Half life (T1/2) :
It is the time in which number of active nuclei becomes half.
N = N0 e–t
N
After one half life, N= 0
2

KVPY_ SA STREAM # 545


N0 ln 2 0.693
= N0 e–t  t =  = t1/2
2  

ln 2 0.693
t1/2 = = (to be remembered)
 
Number of nuclei present after n half lives i.e. after a time t = n t1/2
ln 2
 n
N = N0e–t = N0 e–nt1/2 = N0 e 

N0
= N0 eln 2( n ) = N0 (2)–n = N0(1/2)n =
2n
t
{n = t . It may be a fraction, need not to be an integer}
1/ 2
2 3 n
after 1st N0 2  1 3  1 n  1
or N0    N0    N0   ...............  N0  
half life 2 2
  2
  2

Example 3. A radioactive sample has 6.0 × 1018 active nuclei at a certain instant. How many of these nuclei will
still be in the same active state after two half-lives?
Solution : In one half-life the number of active nuclei reduces to half the original number. Thus, in two half
 1  1
lives the number is reduced to      of the original number. The number of remaining active
2 2
 1  1
nuclei is, therefore, 6.0 × 1018 ×      = 1.5 × 10 18.
2 2

Example 4. The number of 238U atoms in an ancient rock equals the number of 206Pb atoms. The half-life of decay
of 238U is 4.5 × 10 9 y. Estimate the age of the rock assuming that all the 206Pb atoms are formed from
the decay of 238U.
Solution : Since the number of 206Pb atoms equals the number of 238U atoms, half of the original 238U atoms
have decayed. It takes one half-life to decay half of the active nuclei. Thus, the sample is 4.5 × 109 y
old.

Activity :
Activity is defined as rate of radioactive decay of nuclei
It is denoted by A or R A =N
If a radioactive substance changes only due to decay then
dN
A = 
dt
As in that case, N = N0 e–t
A = N = N0e–t
A = A0 e–t
S Unit of activity : becquerel (Bq) which is same as 1 dps (disintegration per second)
The popular unit of activity is curie which is defined as
1 curie = 3.7 × 1010 dps (which is activity of 1 gm Radium)

Example 5. The decay constant for the radioactive nuclide 64Cu is 1.516 × 10–5 s–1. Find the activity of a sample
containing 1 g of 64Cu. Atomic weight of copper = 63.5 g/mole. Neglect the mass difference
between the given radioisotope and normal copper.
Solution : 63.5 g of copper has 6 × 1023 atoms. Thus, the number of atoms in 1 g of Cu is
6  10 23  1g
N= = 9.45 × 1015
63.5 g

KVPY_ SA STREAM # 546


The activity = N
= (1.516 × 10–5 s–1) × (9.45 × 1015) = 1.43 × 1011 disintegrations/s
1.43  1011
= Ci = 3.86 Ci.
3.7  1010
A0
Activity after n half lives :
2n

Example 6. The half-life of a radioactive nuclide is 20 hours. What fraction of original activity will remain after 40
hours?
Solution : 40 hours means 2 half lives.
A0 A0
Thus, A= 2

2 4
A 1
or, A0
= .
4
So one fourth of the original activity will remain after 40 hours.

Specific activity : The activity per unit mass is called specific activity.


Average Life :

 λN
 λt
0e dt . t
sum of ages of all the nuclei 0 1
T avg = = 
N0 N0 λ

Example 7. The half-life of 198Au is 2.7 days. Calculate (a) the decay constant, (b) the average-life and (c) the
activity of 1.00 mg of 198Au. Take atomic weight of 198Au to be 198 g/mol.
Solution : (a) The half-life and the decay constant are related as

ln 2 0.693 0.693 0.693


t1/2 = = or, = t 1/ 2
=
2.7 days
 

0.693
= = 2.9 × 10–6 s–1.
2.7  24  3600 s

1
(b) The average-life is tav = = 3.9 days.

(c) The activity is A = N. Now, 198 g of 198Au has 6 × 1023 atoms.
The number of atoms in 1.00 mg of 198Au is
1.0 mg
N = 6 × 1023 × = 3.03 × 10 18.
198 g
Thus, A = N = (2.9 × 10–6 s–1) (3.03 × 10 18)
= 8.8 × 1012 disintegrations/s

8.8  1012
= Ci = 240 Ci.
3.7  1010

KVPY_ SA STREAM # 547


Example 8. Suppose, the daughter nucleus in a nuclear decay is itself radioactive. Let p and d be the decay
constants of the parent and the daughter nuclei. Also, let Np and Nd be the number of parent and
daughter nuclei at time t. Find the condition for which the number of daughter nuclei becomes
constant.
Solution : The number of parent nuclei decaying in a short time interval t to t + dt is p Npdt. This is also the
number of daughter nuclei decaying during the same time interval is dNddt. The number of the
daughter nuclei will be constant if
pNpdt = dNddt
or, pNp = dNd.
Example 9. A radioactive sample decays with an average-life of 20 ms. A capacitor of capacitance 100 F is
charged to some potential and then the plates are connected through a resistance R. What should
be the value of R so that the ratio of the charge on the capacitor to the activity of the radioactive
sample remains constant in time?
Solution : The activity of the sample at time t is given by
A = A0 e–t
where  is the decay constant and A0 is the activity at time t = 0 when the capacitor plates are
connected. The charge on the capacitor at time t is given by
Q = Q0 e–t/CR
Q Q0 e t / CR
where Q0 is the charge at t = 0 and C = 100 F is the capacitance. Thus,  .
A A 0 e t
1
It is independent of t if =
CR
1 t av 20  10 3 s
or, R= = = = 200 .
C C 100  10 6 F

Example 10. A radioactive nucleus can decay by two different processes. The half-life for the first process is t1 and
that for the second process is t2. Show that the effective half-life t of the nucleus is given by
1 1 1
  .
t t1 t 2
ln 2 ln 2
Solution : The decay constant for the first process is 1 = and for the second process it is 2 = . The
t1 t1
probability that an active nucleus decays by the first process in a time interval dt is 1dt. Similarly,
the probability that it decays by the second process is 2dt. The probability that it either decays by
the first process or by the second process is 1dt + 2dt. If the effective decay constant is , this
probability is also equal to dt. Thus.
dt = 1dt + 2dt

or,  = 1 + 2
1 1 1
or,   . (To be remembered)
t t1 t 2

Example 11. A factory produces a radioactive substance A at a constant rate R which decays with a decay constant
 to form a stable substance. Find (i) the no. of nuclei of A and (ii) Number of nuclei of B, at any time
t assuming the production of A starts at t = 0. (iii) Also find out the maximum number of nuclei of ‘A’
present at any time during its formation.
R 
Solution : Factory  A B
const. rate decay

Let N be the number of nuclei of A at any time t


N t
dN dN

dt
 R  N 
0
R  N =  dt
0

On solving we will get


N = R/(1 - e -t)
(ii) Number of nuclei of B at any time t, NB = R t – NA = Rt – R/(1 - e -t) = R/ ( t – 1 + e -t).
(iii) Maximum number of nuclei of ‘A’ present at any time during its formation = R/.

KVPY_ SA STREAM # 548


Example 12. A radioactive substance “A” having N0 active nuclei at t = 0, decays to another radioactive substance
“B” with decay constant 1. B further decays to a stable substance “C” with decay constant 2.
(a) Find the number of nuclei of A, B and C after time t. (b) What would be the answer of part (a) if 1
>> 2 and 1 << 2.
Solution : (a) The decay scheme is as shown
1 2
A  B  C (stable)
t=0 N0 0 0
t N1 N2 N3
Here N1, N2 and N3 represent the nuclei of A, B and C at any time t.
For A, we can write
N1 = N0 e–1t .... (1)
For B, we can write
dN 2
= 1N1  2N2 .....(2)
dt
dN 2
or, + 2N2 = 1N1
dt
This is a linear differential equation with integrating factor
.F. = e2t
dN 2
e  2t  e  2 t  2N 2 =  N e  2 t
dt 1 1

 d(N e  N e
 2t  2t
2 ) = 1 1 dt


 t  t
N2 e  2 t = 1N0 e 1 e 2 dt ...... using (1)

e( 2 1 ) t
N2 e  2 t = 1N0 +C ......(3)
 2  1
1N0
At t = 0, N2 = 0 0= +C
 2  1
1N0
Hence C =   
1 2
Using C in eqn. (3), we get
1N0
N2 =    e 1t  e  2 t
2 1
 
and N1 + N2 + N3 = N0
 N3 = N0 - (N1 + N2)
1N0  t
(b) For 1 >> 2 N2 = ( e 2 ) = N0 e–2t
 1

1N0
For 1 << 2 N2 = ( e  1t ) =0
2
Alternate solution of (b) part without use of answer of part (a) :
If 1 > 2 that means A will decay very fast to ‘B’ and B will then decay slowly. We can say that practically N1
vanishes in very short time & B has initial no. of atoms as N0.
 Now N2 = N0 e  2t & N1 = N0 e  1t
If 1 << 2 then B is highly unstable and it will soon decay into C.
So, it’s rate of formation its rate of decay.
1N1 N
 1N1  2N2  N2 =     =  1 0 ( e  1t )
2 2

KVPY_ SA STREAM # 549


1. In an -decay the Kinetic energy of  particle is 48 MeV and Q-value of the reaction is 50 MeV. The mass
number of the mother nucleus is:- (Assume that daughter nucleus is in ground state)
(A) 96 (B) 100 (C) 104 (D) none of these

2. A sample of radioactive material decays simultaneously by two processes A and B with half lives
1 1
and hr respectively. For first half hour it decays with the process A, next one hr with the process B and
2 4
for further half an hour with both A and B. If originally there were N0 nuclei, find the number of nuclei after 2 hr
of such decay.
N0 N0 N0 N0
(A) 8 (B) 4 (C) 6 (D)
( 2) ( 2) ( 2) ( 2) 5

3. In which of the following process the number of protons in the nucleus increases .
(A)  decay (B)   decay (C)  +  decay (D) k  capture

4. A radioactive nucleus ' X ' decays to a stable nucleus ' Y '. Then the graph of rate of formation of ' Y' against
time ' t ' will be :
R R R R

(A) (B) (C) (D)


t t t t

5. A heavy nucleus having mass number 200 gets disintegrated into two small fragments of mass number 80
and 120. If binding energy per nucleon for parent atom is 6.5 M eV and for daughter nuclei is 7 MeV and 8
MeV respectively, then the energy released in the decay will be:
(A) 200 MeV (B)  220 MeV (C) 220 MeV (D) 180 MeV

6. An element X decays, first by positron emission and then two -particles are emitted in successive
radioactive decay. If the product nuclei has a mass number 229 and atomic number 89, the mass
number and atomic number of element X are
(A) 237, 93 (B) 237, 94 (C) 221, 84 (D) 237, 92

7. When an electron accelerated by potential difference U is
bombarded on a specific metal, the emitted X-ray spectrum
obtained is shown in adjoining graph. If the potential difference
is reduced to U/3, the correct spectrum is


m m X-ray wavelength 

(A) (B)
 
(A) (B)

 

(C) (D)
 
(C) (D)

8. 90% of a radioactive sample is left undecayed after time t has elapsed. What percentage of the initial
sample will decay in a total time 2t.
(A) 20% (B) 19% (C) 40% (D) 38%

KVPY_ SA STREAM # 550


9. A radioactive element X converts into another stable element Y. Half life of X is 2 hrs. Initially only X is present.
After time t, the ratio of atoms of X and Y is found to be 1 : 4, then t in hours is :
(A) 2 (B) 4 (C) between 4 and 6 (D) 6

10. A radioactive nuclide A decays to nuclide B which further decays to C. Their disintegration constant are 1
and 2 respectively. At t = 0 only nuclei A are present. Number of nuclei A at t = 0 is N0.

 
A 
1 B 

2 C
t=0 N0 0 0
t N1 N2 N3
In the above radioactive decay C is stable nucleus. Then:
(A) rate of decay of A will first increase and then decrease
(B) number of nuclei of B will first increase and then decrease
(C) if 2 > 1, then activity of B will always be higher than activity of A
(D) if 1 >> 2, then number of nucleus of C will always be less than number of nucleus of B.

11. Two radioactive sources A and B initially contain equal number of radioactive atoms. Source A has a half-life
of 1 hour and source B has a half-life of 2 hours. At the end of 2 hours, the ratio of activity of source A to that
of B is :
(A) 1 : 2 (B) 2 : 1 (C) 1 : 1 (D) 1 : 4

12. The radionuclide 238U decays by emitting an alpha particle.


238 U  234Th + 4He
The atomic masses of the three isotopes are.
238U 238.05079 amu
234U 234.04363 amu
4He 4.00260 amu
What is the maximum kinetic energy of the emitted alpha particle? Express your answer in Joule.
(1 amu = 1.67 × 10–27 kg)
(A) 6.8 × 10–14 J (B) 6.8 × 10–13 J (C) 4.3 × 10–14 J (D) 4.3 × 10–13 J

13. Choose the correct statement.


(A) The nuclear force between two nucleons depends upon charge on each nucleon.
(B) The nuclear force is not a central force
(C) The nuclear force between the two nucleons increases rapidly as size of nucleus increases.
(D) Nuclear force is a conservative force.

14. A radioactive nucleus can decay by either emitting an  particle or by emitting a  particle. Probability
of  decay is 75% while that of  decay is 25%. The decayconstant of  decay is 1 and that of 
1
decay is 2 . is
2

1
(A) 3 (B) (C) 1 (D) cannot be said
3

KVPY_ SA STREAM # 551


15. A radioactive nucleus ' X ' decays to a stable nucleus ' Y '. Then the graph of rate of formation of ' Y' against
time ' t ' will be :

(A) (B) (C) (D)

16. A free neutron decays to a proton but a free proton does not decay to a neutron out side nucleus. This is
beacuse
(A) neutron is a composite particle made of a proton and an electron where as proton is fundamental particle
(B) neutron is an uncharged particle whereas proton is a charged particle
(C) neutron has larger rest mass than the proton
(D) weak forces can operate in a neutron but not in a proton.

17. Assuming that about 20 MeV of energy is released per fusion reaction, 1H2 + 1H3  0n1 + 2He4, the
mass of 1H2 consumed per day in a future fusion reactor of power 1 MW would be approximately
(A) 0.1 gm (B) 0.01 gm (C) 1 gm (D) 10 gm

18. Nuclei of radioactive element A are produced at rate ' t 2 ' at any time t. The element A has decay
dN
constant . Let N be the number of nuclei of element A at any time t. At time t = t 0, is minimum.
dt
Then the number of nuclei of element A at time t = t 0 is

2t 0  t 20 t 0  t 20 2t 0  t 20 t 0  t 20
(A) (B) (C) (D)
2 2  

19. A fusion reaction takes place at very high temperature because


(A) atoms get ionized at high temperature
(B) molecules get decomposed at high temperature
(C) nuclei get decomposed at high temperature
(D) due to their high energy nuclei overcome their mutual repulsion and combines

20. The wrong statement about binding energy is-


(A) It is the sum of the rest mass energies of nucleus minus the rest mass energy of the nucleus
(B) It is the energy released when the nucleons combine to form a nucleus
(C) It is the energy required to break a given nucleus into its constituent nucleus
(D) It is the sum of the kinetic energies of all the nucleons in the nucleus

21. Two protons are kept at a separation of 50Å. Fn is the nuclear force and Fe is the electrostatic force between
them, then
(A) Fn >> Fe (B) Fn = Fe (C) Fn << Fe (D) Fn  Fe

22. In a fast breeder atomic reactor


(A) fast neutrons converts natural uranium into fissionable fuel and released energy
(B) thermal neutrons cause fission of enriched uranium and released energy
(C) fast neutrons cause fission of enriched uranium and release more energy
(D) thermal neutron cause fission of natural uranium and produce energy

23. Masses of nucleus, neutron an protons are M, nm and mp respectively. If nucleus has been divided in to
neutrons and protons, then
(A) M = (A – Z) mn + Zmp (B) M = Zmn + (A – Z) mp
(C) M < (A – Z) mn + Zmp (D) M > (A – Z) + Zmp

KVPY_ SA STREAM # 552


24. A positron of 1MeV collides with an electron of 1 MeV and gets annihilated and the reaction produces two
-ray photons. If the effective mass of each photon is 0.0016 amu, then the energy of each -ray photon is
about-
(A) 1.5 MeV (B) 3 MeV (C) 6 MeV (D) 2 MeV
25. If mass of the fissionable material is less than the critical mass, then
(A) fission and chain reactions both are impossible
(B) fission is possible but chain reaction is impossible
(C) fission is impossible but chain reaction is possible
(D) fission and chain reaction both are possible.
26. The value of binding energy for 1H2, 2He4, 26Fe56, 92U235 are 2.22, 28.3, 492 and 1786 units respectively.
Then most stable nucleus is-
(A) 26Fe56 (B) 1H2 (C) 92U235 (D) 2He4
27. The binding energies per nucleon of deutron and -particle are x1 and x2 respectively. Then energy released
in the following reaction will be - 1H2 + 1H2 = 2He4 + Q
(A) (x1 + x2) (B) (x2 – x1) (C) 4(x1 + x2) (D) 4(x2 – x1)
28. If the mass of proton = 1.008 a.m.u. and mass of neutron = 1.009 a.m.u. then binding energy per nucleon
for 4Be9 (mass = 9.012 amu) would be-
(A) 0.065 MeV (B) 60.44 MeV (C) 67.2 MeV (D) 6.72MeV

29. An electron-positron pair (rest mass of each particle = 0.51 MeV) is produced by a -ray photon of energy
2.42 MeV. The K.E. imparted to each of the charged particles shall be-
(A) 0.3 MeV (B) 0.7 MeV (C) 1.4 MeV (D) 1.8 MeV

KVPY PROBLEMS (PREVIOUS YEARS)


238
1. The isotope U92 decays through a chain of  and  emissions ending up at the product Pb206
82 . This chain
involves [KVPY_2007-SA]
(A) 8  emission and 6  emissions (B) 8  emission and 8  emissions
(C) 6  emission and 8  emissions (D) 6  emission and 6  emissions

2. The energy released in a modest size atomic bomb (20 kiloton) is about 1014 J. On a monsoon day in
Mumbai, there was a heavy rainfall of about 100 cm over an area of about 100 km2. The energy released in the
atmosphere on that day is roughly equivalent to that released in [KVPY_2007-SA]
(A) 2000 atomic bombs. (B) 100 atomic bombs
(C) atomic bomb (D) negligible compared to an atomic bomb.
235 207
3. 92 U atom disintegrates to 82 Pb with a half-life of 109 years. In the process it emits 7 alpha particles and
n  particles. Here n is -

[KVPY_2009-SA]
(A) 7 (B) 3 (C) 4 (D) 14

4. A nucleus of lead pb 314


82
emits two electrons followed by an alpha particle. The resulting nucleus will have
[KVPY_2011_SA]
(A) 82 protons and 128 neutrons (B) 80 protons and 130 neutrons
(C) 82 protons and 130 neutrons (D) 78 protons and 134 neutrons

238 214
5. 92 U atom disintegrates to 84 Po with a half life of 4.5×109 years by emitting six alpha particle and
n electrons. Here n is : [KVPY_2012_SA]
(A) 6 (B) 4 (C) 10 (D) 7

KVPY_ SA STREAM # 553


6. Which statement about the Rutherford model of the atom is NOT true ? [KVPY_2012_SA]
(A) There is a positively charged centre in an atom called the nucleus
(B) Nearly all the mass of an atom resides in the nucleus
(C) Size of the nucleus is comparable to the atom
(D) Electrons occupy the space surrounding the nucleus

7. A nuclear decay is possible if the mass of the parent nucleus exceeds the total mass of the decay particles.
If M(A, Z) denotes the mass of a single neutral atom of an element with mass number A and atomic number
Z, then the minimal condition that the  decay [KVPY_2013] [1 Marks]

X ZA  YZA1   –  v e
will occur is (me denotes the mass of the  particle and the neutrino mass mv can be neglected) :
(A) M(A, Z) > M(A, Z + 1 ) + me (B) M(A, Z) > M(A, Z + 1)
(C) M(A, Z) > M(A, Z + 1) + Zme (D) M(A,Z) > M(A, Z + 1) – me

8. Two species of radioactive atoms are mixed in equal number. The disintegration of the first species is  and
of the second is /3. After a long time the mixture will behave as a species with mean life of approximately
[KVPY_2013] [1 Marks]
(A) 0.70/ (B) 2.10/ (C) 1.00/ (D) 0.52/

9. In 1911, the physicist Ernest Rutherford discovered that atoms have a tiny, dense nucleus by shooting
pisitively charged particles at a very thin gold foil. A key physical property which led Rutherford to use gold
that it was [KVPY_2014_SA] [1 Mark]
(A) Electrically conducting (B) Highly malleable
(C) Shiny (D) none-reactive
10. Consider the following statements [KVPY_2014_SA] [1 Mark]
(i) All isotopes of an element have the same number of neutrons
(ii) Only one isotope of an element can be stable and non-radioactive
(iii) All elements have isotops
(iv) All isotopes of Carbon can form chemical compounds with Oxygen-16
The correct option regarding an isotope is
(A) (iii) and (iv) only (B) (ii), (iii) and (iii) only
(C) (i), (ii) and (iii) only (D) (i), (iii) and (iv) only
11. The beta particles of a radioactive metal originate from. [KVPY_2015_SA] [1 Marks]
(A) the free electrons in the metal
(B) the orbiting electrons of the metal atoms
(C) the photons released from the nucleus.
(D) the nucleus of the metal atoms.
12. The radius of a nucleus is given by r0 A1/3 where r0 = 1.3 × 10–15 m and A is the mass number of the nucleus.
The Lead nucleus has A = 206. The electrostatic force between two protons in this nucleus is approximately.
[KVPY_2016_SA] [1 Marks]
(A) 102 N (B) 107N (C) 1012N (D) 1017N
13. Consider the following statements (X and Y stand for two different elements) [KVPY_2017_SA] [1 Marks]
I. 32X65 and 33Y65 are isotopes.
II. 42X86 and 42Y85 are isotopes.
III. 85X174 and 88Y177 have the same number of neutrons.
IV. 92X235 and 94Y235 are isobars.
The correct statements are :
(A) II and IV only. (B) I, II and IV only. (C) II, III and IV only. (D) I, II, III and IV.

KVPY_ SA STREAM # 554


EXERCISE - 1
1. (B) 2. (A) 3. (B) 4. (C) 5. (C) 6. (B) 7. (B)
8. (B) 9. (C) 10. (B) 11. (C) 12. (B) 13. (B) 14. (A)
15. (C) 16. (C) 17. (A) 18. (C) 19. (D) 20. (B) 21. (C)
22. (A) 23. (C) 24. (A) 25. (B) 26. (A) 27. (D) 28. (D)
29. (B)
EXERCISE - 2
1. (A) 2. (A) 3. (C) 4. (A) 5. (B) 6. (C) 7. (A)
8. (B) 9. (B) 10. (A) 11. (D) 12. (A) 13. (C)

my A 4 A 4
1. We have K = m  m .Q  K = .Q  48 = .50  A = 100
y  A A

1
2. After first half hrs N = N0
2
4 5
1 1  1  1   1
for t = to t = 1 N =  N0    = N0  
2 2  2   2  2

1 1 1 1
for t = 1 to t = 2 hrs. [for both A and B = + =2+4=6
2 t 1/ 2 1/ 2 1/ 4
t1/2 = 1/6 hrs.]
  1 5  1 3 8
   1
N = N0  2     = N0  
     2  2

3. For  - decay : xA
y
 x  2B y 4 + 
– y
For  – decay : xA  x  1B y + 1
0

+ y
For  – decay : xA  x  1B y + 1
0

For k - capture : there will be no change in the number of protons.


Hence, only case in which no of protons increases is  - decay

4. N = N0e–t Ny = N0(1 – e–t)


dN
Rate of formation of R= = + N0e–t X  Y
dt
At t = 0 , R = N0
t=,R=0
5. Energy released = (B.E. of product – BE of reactant)
= (80 × 7 + 120 × 8 – 200 × 6.5) = 220 MeV.
6.

Given A – 8 = 224
& Z – 5 = 89  A = 237, Z = 94.

KVPY_ SA STREAM # 555


7. m will increase to 3 m due to decrease in the energy of bombarding electrons. Hence no characteristic
x-rays will be visible, only continous X-ray will be produced.

8.  90% of sample left undecayed after time t


9
N = N0 e–t
10 0
1  10 
= ln   .............(1)
t  9 
After time 2t,
1  10 
 ln   2t
t   9 
N = N0 e–(2t) = N0 e
2
 10  2
ln  
 9 
N = N0 e 9 = N0   .............(2)
 10 
 19% of initial value will decay in time 2t.

9. Let N2 be the number of atoms of X at time t = 0. Then at t = 4 hrs (two half lives)
N0 3N0
Nx = Ny =
4 4
 Nx / Ny = 1/3
at t = 6 hrs (three half lives)
N0 7N0
Nx = and Ny =
8 8
Nx 1
or Ny = 7

1 1 1
The given ratio lies between and .
4 3 7
Therefore, t lies between 4 hrs and 6 hrs.

10. Rate of decay of A keeps on decreasing continuously because concentration of A decreases with time A
is false
Initial rate of production of B is 1N0 and rate of decay is zero. With time, as the number of B atoms increase,
the rate of its production decreases and its rate of decay increases. Thus the number of nuclei of B will first
increase and then decrease  B is the correct choice
The initial activity of B is zero whereas initial activity of A is 1N0  C is false.
As time t : NA = 0, NB = 0 and NC = N0  D is false

n 2
 dN  n2  .2
  . N0 e 1
 dt  A ( N)A 1
11.  =
 dN  ( N)B n2 
n 2
.2
  . N0 e 2
 dt B 2

2  22
= =1
21

12. Mass defect =(238.05079 – 234.04363 – 4.00260) u = 4.56 × 10–3 u


= 4.56 × 10–3 × 1.66 × 10–27 = 7.57 × 10–30 kg
mc = 7.57 × 10–30 × 9 × 1016 = 6.8 × 10–13 J
2

KVPY_ SA STREAM # 556


13. The correct statement in nuclear force is not a central force.

75
1 1N decay rate of  decay probability of  decay 100  3
14. = = = =
2  2N decay rate of  decay probability of  decay 25
100
15. N = N0e–t
Ny = N0(1 – e–t)
dN
Rate of formation of Y = = + N0e–t
dt
X  Y
At t = 0 , R = N0
t = 0 N0 0 t=,R=0 t=tN Ny = N0(1 – e–t)

17. As proton mass is less than neutron mass proton does not decay to neutron outside nucleus.

18. no of moles of 1H2 consumed


1MW  (24  3600 ) sec/ day
= = 0.05
(20 MeV  6.023  10 23 )
 m = 0.1 g

dN
19. = t2 – N
dt

dN d2N d2N dN
for to be minimum; 2 =0  2 = 2t –  dt = 2t –  (t – N) = 0
2
dt dt dt

2t 0  t 20
or N=
2

h
25. E= × c2 = 0.0016 × 931.478 MeV = 1.5 MeV..
c2

28. Binding energy of product – Binding energy of reactent = Q

29. (4 × 1.0089 +5 × 1.009 ) – 9.012 = m


Binding energy = m × c2
Binding energy / Nucleon = (Binding energy) / 9 = 6.72 MeV.

30. Rest mass energy + kinetic energy = energy of gama photon


0.51 × 2 MeV + 2 × Ek = 2.42 MeV.
EK = 0.7 MeV.

KVPY_ SA STREAM # 557


CI RCU LAR M OT I ON

CIRCULAR M OTION :
When a particle moves in a plane such that its distance from a fixed (or moving) point remains constant,
then its motion is known as circular motion with respect to that fixed (or moving) point. The fixed point is
called centre, and the distance of particle from it is called radius.

1. KINEM ATICS OF CIRCULAR M OTION :


1.1 Va ria bles of M ot ion :
(a ) Angula r Posit ion :
To decide the angular position of a point in space we need to specify (i) origin and (ii) reference line.
The angle made by the position vector w.r.t. origin, with the reference line is called angular position.
Clearly angular position depends on the choice of the origin as well as the reference
Y
line.
Circular motion is a two dimensional motion or motion in a plane.
P'
Suppose a particle P is moving in a circle of radius r and centre O.
P
The angular position of the particle P at a given instant may be 
described by the angle  between OP and OX. This angle  is  X
O r
called the angular position of the particle.

(b) Angula r Displa cem ent :


Definition: Angle through which the position vector of the moving particle rotates in a given time
interval is called its angular displacement. Angular displacement depends on origin, but it does not
depends on the reference line. As the particle moves on above circle its angular position  changes.
Suppose the point rotates through an angle  in time t, then  is angular
displacement.

 Im porta nt points :
 Angular displacement is a dimensionless quantity. Its SI unit is radian, some other units are degree
and revolution
2 rad = 360° = 1 rev
 Infinitesimally small angular displacement is a vector quantity, but finite angular displacement is a
scalar, because while the addition of the Infinitesimally small angular displacements is commuta-
tive, addition of finite angular displacement is not.
   
d1  d2  d2  d1 but 1  2  2  1

 Direction of small angular displacement is decided by right hand thumb rule. When the fingers are
directed along the motion of the point then thumb will represents the direction of angular
displacement.
( c) Angula r Velocit y 
(i) Average Angular Velocity
Angular displaceme nt
av =
Total time taken
 2  1 
av  
t 2  t1 t
where  1 and  2 are angular position of the particle at time t 1 and t 2. Since angular
displacement is a scalar, average angular velocity is also a scalar.

KVPY_ SA STREAM # 558


(ii) Instantaneous Angular Velocity
It is the limit of average angular velocity as t approaches zero. i.e.
 
   d
 = lim
t 0 t
=
dt

Since infinitesimally small angular displacement d is a vector quantity, instantaneous angular velocity  is
also a vector, whose direction is given by right hand thumb rule.

 Im porta nt points :
 Angular velocity has dimension of [T-1] and SI unit rad/s.
 For a rigid body, as all points will rotate through same angle in same time, angular velocity is a
characteristic of the body as a whole, e.g., angular velocity of all points of earth about earth’s axis is
(2/24) rad/hr.

 If a body makes ‘n’ rotations in ‘t’ seconds then average angular velocity in radian per second will be
2 n
 av 
t
If T is the period and ‘f’ the frequency of uniform circular motion

2
av = = 2f
T

(d) Angula r Accelera tion  


(i) Average Angular Acceleration :
Let 1 and 2 be the instantaneous angular speeds at times t1 and t2 respectively, then the
average angular acceleration av is defined as
  
   1 
 av  2 
t 2  t1 t
(ii) Instantaneous Angular Acceleration :
It is the limit of average angular acceleration as t approaches zero, i.e.,
 
  d 
  lim 
t 0 t dt
   
 d  d d2   d
since   ,   = = 2 , Also   
dt dt dt d
 Im porta nt points :
 Both average and instantaneous angular acceleration are axial vectors with dimension [T-2] and unit
rad/s2.
 If  = 0, circular motion is said to be uniform.

1 .2 M otion w ith const a nt a ngula r velocity


 = t ,  = 0

1 .3 M otion with consta nt a ngula r a ccele ra tion


0  Initial angular velocity
  Final angular velocity
  Constant angular acceleration
  Angular displacement
Circular motion with constant angular acceleration is analogous to one dimensional translational motion with
constant acceleration. Hence even here equation of motion have same form.
 = 0 + t

KVPY_ SA STREAM # 559


1 2
 = 0t + t
2
2 = 02 + 2 
   0 
=   t
 2 


n th = 0 + ( – n-1)
2 n

2. RELATION BETW EEN SPEED AND ANGULAR VELOCITY :


  
v   r
  
Here, v is velocity of the particle,  is angular velocity about centre of circular motion and ‘ r ’ is position of
particle w.r.t. center of circular motion.
 
Since   r
v =  r for circular motion.

Example 1. A particle is moving with constant speed in a circular path. Find the ratio of average velocity to its


instantaneous velocity when the particle describes an angle  =
2

 R R
Solution : Time taken to describe angle , t = = =
 v 2v

Total displacement 2R 2 2
Average velocity = = = v
Total time R / 2v 
Instantaneous velocity = v

2 2
The ratio of average velocity to its instantaneous velocity = Ans.

3. RELATIVE ANGULAR VELOCITY


Just as velocities are always relative, sim ilarly angular velocity is also always relative. There is no such thing as
absolute angular velocity. Angular velocity is defined with respect to origin, the point from which the position
vector of the moving particle is drawn.
 Consider a particle P moving along a circular path shown in the figure given below
Here angular velocity of the particle P w.r.t. ‘O’ and ‘A’ will be different

d
Angular velocity of a particle P w.r.t. O, PO =
dt

d
Angular velocity of a particle P w.r.t. A, PA =
dt
Definition :
Angular velocity of a particle ‘A’ with respect to the other moving particle ‘B’ is the rate at which position
vector of ‘A’ with respect to ‘B’ rotates at that instant. (or it is simply, angular velocity of A with origin fixed at
B). Angular velocity of A w.r.t. B, AB is mathematically define as

Component of relative velocity of A w.r.t. B, perpendicu lar to line ( VAB )


AB = =
seperation between A and B rAB

KVPY_ SA STREAM # 560


 Important points:
 If two particles are moving on two different concentric circles with different
velocities then angular velocity of B as observed by A will depend on
their positions and velocities. Consider the case when A and B are
closest to each other moving in same direction as shown in figure. In
this situation
 (VAB) = vB  v A
Separation between A and B is rBA = rB – rA
( VAB ) vB  v A
so, AB = =
rAB rB  rA

 If two particles are moving on the same circle or different coplanar concentric circles in same direc-
tion with different uniform angular speed A and B respectively, the rate of change of angle between

OA and OB is
B
B
d  A 
A
=B – A
dt O O Initial line
Initial line

So the time taken by one to complete one revolution around O w.r.t. the other

2 2 TT
T   1 2
rel 2  1 T1  T2

 B – A is rate of change of angle between OA and OB . This is not angular velocity of B w.r.t. A.
(Which is rate at which line AB rotates)

Example 2. Find angular velocity of A with respect to B at the instant shown in the figure.

Solution : Angular velocity of A with respect to B is ;


( v AB )
AB = rAB

v AB = 2 v = (vAB)  rAB = 2d

( v AB ) v 2 v
AB = rAB
= =
d 2 d

KVPY_ SA STREAM # 561


4. RADIAL AND TANGENTIAL ACCELERATION
There are two types of acceleration in circular motion ; Tangential acceleration and centripetal acceleration.
(a) Tangential acceleration :-
Component of acceleration directed along tangent of circle is called tangential acceleration. It is
responsible for changing the speed of the particle. It is defined as,

dv d| v |
at = = = Rate of change of speed.
dt dt
at = r
 IMPORTANT POINT
  
(i) In vector form a t =   r
(ii) If tangential acceleration is directed in direction of velocity then the speed of the particle increases.
(iii) If tangential acceleration is directed opposite to velocity then the speed of the particle decreases.

(b) Ce ntripe t a l a cce le ra t ion :-


It is responsible for change in direction of velocity. In circular motion, there is always a centripetal
acceleration.
Centripetal acceleration is always variable because it changes in direction.
Centripetal acceleration is also called radial acceleration or normal acceleration.

(c) Tota l a cce lera tion :-


Total acceleration is vector sum of centripetal acceleration and tangential acceleration.
  
 dv
a = ar  a t
dt

a= a t2  a r2

ar
tan  = a
t

 IMPORTANT POINT
(i) Differentiation of speed gives tangential acceleration.

(ii) Differentiation of velocity ( v ) gives total acceleration.
  
dv d| v | dv
(iii) & are not same physical quantity.. is the magnitude of rate of change of velocity, i.e.
dt dt dt

d| v |
magnitude of total acceleration and is a rate of change of speed, i.e. tangential acceleration.
dt

4 .1 Ca lcula tion of centripe ta l a cce lera tion :


Consider a particle which moves in a circle with constant speed v as shown in figure.

 change in velocity between the point A and B is ;


  
 v = vB  v A

Magnitude of change in velocity.


  
v  v B  v A  v B2  v 2A  2v A v B cos (   )
(v A = v B = v, since speed is same)

KVPY_ SA STREAM # 562


 
 | v | = 2v sin
2
Distance travelled by particle between A and B = r
r
Hence time taken, t =
v

 v 2v sin  / 2 v 2 2 sin  / 2
Net acceleration , anet =  =
t r / v r 
 
v dv v2
If t 0, then  is small, sin (/2) = /2 lim  
t 0 t dt r

v2 dv
i.e. net acceleration is but speed is constant so that tangential acceleration, at = = 0.
r dt
v2
 anet = ar =
r
** Through we have derived the formula of centripetal acceleration under condition of constant speed, the
same formula is applicable even when speed is variable.
 IMPORTANT POINT
  
In vector form a c    v

Example 3. A block of mass m is kept on rough horizontal turn table at a distance r from centre of table.
Coefficient of friction between turn table and block is µ . Now turn table starts rotating with uniform
angular acceleration .
(i) Find the time after which slipping occurs between block and turn table.
(ii) Find angle made by friction force with velocity at the point of slipping.
Solution : (i) at = r
speed after t time
dv
= r  v = 0 + rt
dt
Centripetal acceleration

v2
ac = = 2rt2
r

Net acceleration anet = a 2t  a 2c =  2r 2   4r 2 t 4


block just start slipping
µmg = manet = m  2r 2   4r 2 t 4

1/ 4 1/ 4
 µ 2 g2   2r 2   µg  2  1  2 
t =  
  t =  2      Ans.
  4r 2    r     

ac  2rt 2
(ii) tan =  tan  =
at r
  = tan–1 (t2) Ans.

KVPY_ SA STREAM # 563


5. DYNAM ICS OF CIRCULAR M OTION :
If there is no force acting on a body it will move in a straight line (with constant speed). Hence if a body is
moving in a circular path or any curved path, there must be some force acting on the body.
If speed of body is constant, the net force acting on the body is along the inside normal to the path of the
body and it is called centripetal force.
mv 2
Centripetal force (Fc) = mac = = m 2 r
r
However if speed of the body varies then, in addition to above centripetal force which acts along inside
normal, there is also a force acting along the tangent of the path of the body which is called tangential force.
dv
Tangential force (Ft) = Mat = M =Mr ; where  is the angular acceleration
dt
 IMPORTANT POINT

mv 2
Remember is not a force itself. It is just the value of the net force acting along the inside normal which
r
is responsible for circular motion. This force may be friction, normal, tension, spring force, gravitational force
or a combination of them.
So to solve any problem in uniform circular motion we identify all the forces acting along the normal (towards

mv 2
center), calculate their resultant and equate it to .
r
If circular motion is non uniform then in addition to above step we also identify all the forces acting along the

mdv md | v |
tangent to the circular path, calculate their resultant and equate it to or .
dt dt

6. CIRCULAR M OTION IN HORIZONTAL PLANE

Example 4. The coefficient of friction between block and table is µ. Find the tension in the string
if the block moves on the horizontal table with speed v in circle of radius R.

mv 2
Solution : The magnitude of centripetal force is .
R

mv 2
(i) If limiting friction is greater than or equal to , then static friction alone provides centripetal
R
force, so tension is equal to zero.
T = 0 Ans.

mv 2
(ii) If limiting friction is less than , then friction as well as tension both combine to provide the
R
necessary centripetal force.

mv 2
T+fr=
R
In this case friction is equal to limiting friction, fr = mg

mv 2
 Tension = T = – mg Ans.
R

KVPY_ SA STREAM # 564


Example 5. A block of mass m is tied to a spring of spring constant k , natural length , and the other end of
spring is fixed at O. If the block moves in a circular path on a smooth horizontal surface with
constant angular velocity  , find tension in the spring.

Solution : Assume extension in the spring is x


Here centripetal force is provided by spring force.
Centripetal force, kx = m2( + x)
m 2 
 x=
k  m2
therefore,
km 2 
Tension = kx = Ans.
k  m2

Example 6. A boy whirls a stone in a horizontal circle of radius 2 m and at height 4.9 m above level ground. The
string breaks, and the stone files off horizontally and strikes the ground at a point which is 10 m
away from the point on the ground directly below the point where the string had broken. What is the
magnitude of the centripetal acceleration of the stone while in circular motion? (g = 9.8 m/s2)

2h 2  4.9
Solution : t=   1s
g 9.8

10
v= = 10 m/s
t

v2
a= = 50 m/s2
R

7. RADIUS OF CURVATURE
Any curved path can be assumed to be made of infinite circular arcs. Radius of curvature at a point is the
radius of the circular arc at a particular point which fits the curve at that point.


If R is radius of the circular arc at a given point P, where velocity is v , then centripetal force at that point is,

mv 2 mv 2
Fc =  R=
R FC
Now centripetal force Fc is simply the component of force perpendicular to velocity (let us say F).

mv 2 v2
 R=  R=
F a
Where, aComponent of acceleration perpendicular to velocity.

KVPY_ SA STREAM # 565


 If a particle moves in a trajectory given by y = f(x) then radius of curvature at any point (x, y) of the trajectory
3/2
  dy  2 
1    
  dx  
is given by  R =
d2 y
dx 2
8. M OTION IN A VERTICAL CIRCLE :
Let us consider the motion of a point mass tied to a string
of length  and whirled in a vertical circle. If at any time
the body is at angular position , as shown in the figure,
the forces acting on it are tension T in the string along the
radius towards the center and the weight of the body mg
acting vertically down wards.
Applying Newton’s law along radial direction
mv 2
T – mg cos  = m.ac =

2
or T = mv + mg cos  .....(1)

The point mass will complete the circle only and only if tension is never zero (except momentarily, if at all) if
tension becomes zero at any point, string will go slack and subsequently, the only force acting on the body
is gravity. Hence its subsequent motion will be similar to that of a projectile.
From equation ...(1), it is evident that tension decreases with increase in  because cos  is a decreasing
function and v decreases with height. Hence tension is minimum at the top most point. i.e. Tmin = Ttopmost.
T > 0 at all points.  Tmin > 0.
However if tension is momentarily zero at highest point the body
would still be able to complete the circle.
Hence condition for completing the circle (or looping the loop) is
Tmin  0 or Ttop  0.
mv 2top
Ttop + mg = ...............(2)

Equation...(2) could also be obtained by putting  =  in equation ..(1).
For looping the loop, Ttop  0.

mv 2top
  mg  v top  g .................(3)

Condition for looping the loop is v top  g .


If speed at the lowest point is u, then from conservation of mechanical energy between lowest point and top
most point.
1 1
mu2 = m v 2top + mg . 2
2 2

using equation ..(3) for v top we get u  5g 

i.e., for looping the loop, velocity at lowest point must be  5g  .

 If velocity at lowest point is just enough for looping the loop, value of various quantities. (True for a point mass
attached to a string or a mass moving on a smooth vertical circular track.)

KVPY_ SA STREAM # 566


P.E. = 0

By energy conservation, v =
2
T–mg =
could also be obtained by

anet = 5 g anet = g 10

by energy conservation
+

A B,D C P( general po int)


1 Velocity 5g 3g g g(3  2 cos )
2 Tension 6mg 3mg 0 3mg(1  cos )
3 Potential Energy 0 mg  2mg  mg (1  cos )
4 Radial accelerati on 5g 3g g g(3  2 cos )
5 Tangential accelerati on 0 g 0 g sin 

Note :- From above table we can see , Tbottom – Ttop = TC – TA = 6 mg , this difference in tension remain same even

if V > 5g

8 .1 . CONDITION FOR OSCILLATION OR LEAVING THE CIRCLE :


In case of non uniform circular motion in a vertical plane if velocity of body
at lowest point is lesser than 5g , the particle will not complete the
circle in vertical plane. In this case, the motion of the point mass which
depend on ‘whether tension becomes zero before speed becomes zero or
vice versa.

KVPY_ SA STREAM # 567


Case  (Speed becomes zero before tension)
In this case the ball never rises above the level of the center O i.e. the body
is confined to move within C and B, ( |  | < 90° ) for this the speed at A,
v< 2g (as proved in above example)
In this case tension cannot be zero, since a component of gravity acts
radially outwards.
Hence the string will not go slack, and the ball will reverse back as soon as
its speed becomes zero.
Its motion will be oscillatory motion.

Case  (Tension becomes zero before speed)


In this case the ball rises above the level of center O i.e. it goes beyond point B ( > 90°) for this v > 2g 
(as proved in above example)
In this case a component of gravity will always act towards center,
hence centripetal acceleration or speed will remain nonzero. Hence
tension becomes zero first.
As soon as, Tension becomes zero at any point, string will go
slack and subsequently, the only force acting on the body is
gravity. Hence its subsequent motion will be similar to that of a
projectile. In this case motion is a combination of circular and
projectile motion.

8 .2 CONDITION FOR LOOPING THE LOOP IN SOM E OTHER CASES


Case 1 : A mass moving on a smooth vertical circular track.

Case 2 : A particle attached to a light rod rotated in vertical circle.


Condition for just looping the loop, velocity v = 0 at highest point (even if tension is zero, rod won’t slack,
and a compressive force can appear in the rod).
By energy conservation,
velocity at lowest point = 4g

Vmin  4g (for completing the circle)

Case 3 : A bead attached to a ring and rotated.


Condition for just looping the loop, velocity v = 0 at highest point (even if normal is zero, the bead will not lose
contact with the track, normal can act radially outward).
By energy conservation,
velocity at lowest point = 4g

Vmin  4g (for completing the circle)

KVPY_ SA STREAM # 568


Case 4 : A block rotated between smooth surfaces of a pipe.
Condition for just looping the loop, velocity v = 0 at highest point (even if normal is zero, the bead will not lose
contact with the track, normal can act radially outward).
By energy conservation,
velocity at lowest point = 4g

Vmin  4g (for completing the circle)

Example 7. Two point mass m are connected the light rod of length  and it is free to rotate in
vertical plane as shown. Calculate the minimum horizontal velocity is given to mass
so that it completes the circular motion in vertical lane.

Solution : Here tension in the rod at the top most point of circle can be zero or negative for
completing the loop. So velocity at the top most point is zero.
From energy conservation

1 1 v2 48 g
mv 2 + m = mg(2) + mg(4) + 0  v= Ans.
2 2 4 5

Example 8. A point mass m connected to one end of inextensible string of length  and
other end of string is fixed at peg. String is free to rotate in vertical plane.
Find the minimum velocity give to the mass in horizontal direction so that
it hits the peg in its subsequent motion.

Solution : Tension in string is zero at point P in its subsequent motion, after this point its motion is projectile.

mv 2
Velocity at point P, T = 0  mgcos =  v = g cos 

Assume its projectile motion start at point P and it passes through point C. So that equation of
trajectory satisfy the co-ordinate of C (sin , – cos)
Equation of trajectory

gx 2
y = xtan –
2v cos 2 
2

g( sin )2


–  cos = sin tan –
2(g cos ) cos 2 

sin 2  1 sin 2 
 – cos = –
cos  2 cos 3 
 – 2 cos4 = 2sin2 cos2 – sin2  sin2 = 2sin2 cos2 + 2cos4
 sin2 = 2cos2 (sin2 + cos2)  tan2 = 2

1 2
 tan  = 2   cos = , sin  =
3 3
From energy conservation between point P and A.
1 1
mu2 = mv 2 + mg (1+cos)
2 2
 u = v + 2g (1 + cos )
2 2
 u2 = 2g + 3gcos
1
 u2 = 2g + 3g
3 
u = 2  3 g   1/ 2
Ans.

KVPY_ SA STREAM # 569


Example 9. A car is moving with uniform speed over a circular bridge of radius R which subtends an angle. of 90°
at its centre. Find the minimum possible speed so that the car can cross the bridge without losing
the contact any where.
Solution : Let the car losses the contact at angle  with the vertical

mv 2
mgcos – N =
R

mv 2
N = mgcos – .......... (1)
R
for losing the contact N = 0 ,
 v= Rg cos  (from (1) )
for minimum speed, cos should be minimum so that  should be maximum.
1
max = 45°  cos 45° =
2

1/ 2
 Rg 
v min =   Ans.
 2
So that if car cannot lose the contact at initial or final point, car cannot be lose the contact anywhere.

9. CIRCULAR TURNING ON ROADS :


When vehicles go through turnings, they travel along a nearly circular arc. There must be some force which
will produce the required centripetal acceleration. If the vehicles travel in a horizontal circular path, this
resultant force is also horizontal. The necessary centripetal force is being provided to the vehicles by following
three ways.
1. By friction only
2. By banking of roads only.
3. By friction and banking of roads both.
In real life the necessary centripetal force is provided by friction and banking of roads both. Now let
us write equations of motion in each of the three cases separately and see what are the constant in
each case.

9 .1 By Frict ion Only


Suppose a car of mass m is moving at a speed v in a horizontal circular arc of radius r. In this case, the
necessary centripetal force to the car will be provided by force of friction f acting towards center
mv 2
Thus, f
r
Further, limiting value of f is N
or fL   N  mg (N = mg)

mv 2
Therefore, for a safe turn without sliding  fL
r
mv 2 v2
or  mg or   or v  rg
r rg
Here, two situations may arise. If  and r are known to us, the speed of the vehicle should not exceed
v2
rg and if v and r are known to us, the coefficient of friction should be greater than .
rg
9 .2 . By Ba nk ing of Roa ds Only
Friction is not always reliable at circular turns if high speeds and sharp turns are involved to avoid depen-
dence on friction, the roads are banked at the turn so that the outer part of the road is some what lifted
compared to the inner part.

KVPY_ SA STREAM # 570


Applying Newton’s second law along the radius and the first law in the vertical direction.
mv 2
N sin  
r
or N cos = mg
from these two equations, we get
v2
tan   or v  rg tan 
rg

9 .3 By Friction a nd Ba nk ing of Roa d Both


If a vehicle is moving on a circular road which is rough and banked also, then three forces may act on the
vehicle, of these the first force, i.e., weight (mg) is fixed both in magnitude and direction.

N f N
 


mg f mg
 
Figure (i) Figure (ii)
The direction of second force, i.e., normal reaction N is also fixed (perpendicular to road) while the direction
of the third force i.e., friction f can be either inwards or outwards while its magnitude can be varied upto a
maximum limit (fL = N). So the magnitude of normal reaction N and directions plus magnitude of friction f are
mv 2
so adjusted that the resultant of the three forces mentioned above is towards the center. Of these m
r
and r are also constant. Therefore, magnitude of N and directions plus magnitude of friction mainly depends
on the speed of the vehicle v. Thus, situation varies from problem to problem. Even though we can see that :
(i) Friction f will be outwards if the vehicle is at rest v = 0. Because in that case the component of
weight mg sin is balanced by f.

(ii) Friction f will be inwards if v  rg tan 


(iii) Friction f will be outwards if v  rg tan  and
(iv) Friction f will be zero if v  rg tan 
(v) For maximum safe speed (figure (ii)
mv 2
N sin + f cos = ........(i)
r
N cos – f sin = mg .......(ii)
As maximum value of friction
f = N

sin    cos  v2 rg(tan    )


 =  v max =
cos    sin  rg (1   tan )

rg(tan   )
Similarly ; v min =
(1   tan )

v2
N ot e :  The expression tan  = also gives the angle of banking for an aircraft, i.e., the angle through
rg
which it should tilt while negotiating a curve, to avoid deviation from the circular path.
v2
 The expression tan  = also gives the angle at which a cyclist should lean inward, when
rg
rounding a corner. In this case,  is the angle which the cyclist must make with the vertical which will
be discussed in chapter rotation.


KVPY_ SA STREAM # 571
10. CENTRIFUGAL FORCE :
When a body is rotating in a circular path and the centripetal force vanishes, the body would leave the
circular path. To an observer A who is not sharing the motion along the circular path, the body appears to fly
off tangentially at the point of release. To another observer B, who is sharing the motion along the circular
path (i.e., the observer B is also rotating with the body which is released, it appears to B, as if it has been
thrown off along the radius away from the centre by some force. This inertial force is called centrifugal force.)

mv 2
Its magnitude is equal to that of the centripetal force. = = m2r
r
Direction of centrifugal force, it is always directed radially outward.

Centrifugal force is a fictitious force which has to be applied as a concept


only in a rotating frame of reference to apply Newton’s law of motion in
that frame. FBD of ball w.r.t. non inertial frame rotating with the ball.
Suppose we are working from a frame of reference that is rotating at a constant, angular velocity  with
respect to an inertial frame. If we analyses the dynamics of a particle of mass m kept at a distance r from the
axis of rotation, we have to assume that a force mr2 react radially outward on the particle. Only then we can
apply Newton’s laws of motion in the rotating frame. This radially outward pseudo force is called the centrifugal
force.

Example 10. A ring which can slide along the rod are kept at
mid point of a smooth rod of length L. The rod is
rotated with constant angular velocity  about
vertical axis passing through its one end. Ring is
released from mid point. Find the velocity of the
ring when it just leave the rod.

Solution : Centrifugal force


m2x = ma

vdv
2x =
dx

L v


L/2
2 x dx =  v dv
0
(integrate both side.)

L v
 x2   v2   L2 L2  2
 2  =    2    = v  v=
3
L.
 2   2  2 8 
 L / 2  0   2 2

Velocity at time of leaving the rod

2
 3  7
v´ = ( L ) 2   L  = L Ans.
 2  2
 

11. EFFECT OF EARTHS ROTATION ON APPARENT W EIGHT :


The earth rotates about its axis at an angular speed of one revolution per 24 hours.
The line joining the north and the south poles is the axis of rotation.
Every point on the earth moves in a circle. A point at equator moves in a circle of
radius equal to the radius of the earth and the centre of the circle is same as the
centre of the earth. For any other point on the earth, the circle of rotation is smaller
than this. Consider a place P on the earth (figure).

KVPY_ SA STREAM # 572


Draw a perpendicular PC from P to the axis SN. The place P rotates in a circle with the centre at C. The
radius of this circle is CP. The angle between the line OM and the radius OP through P is called the latitude
of the place P. We have
CP = OP cos or, r = R cos
where R is the radius of the earth and  is colatitude angle.
If we work from the frame of reference of the earth, we shall have to assume
the existence of pseudo force. In particular, a centrifugal force mw2r has to
be assumed on any particle of mass m placed at P.
If we consider a block of mass m at point P then this block is at rest with
respect to earth. If resolve the forces along and perpendicular the centre of
earth then
N + mr2 cos  = mg  N = mg – mr2 cos   N = mg – mR2 cos2 

Note: At equator ( = 0) W app. is minimum and at pole ( = /2) W app. is maximum.


This apparent weight is not along normal but at some angle  w.r.t. it. At all point except poles and equator
( = 0 at poles and equator)

KVPY_ SA STREAM # 573


KVPY PROBLEMS (PREVIOUS YEARS)
1. The maximum value attained by the tension in the string of a swinging pendulum is four times the minimum
value it attains. There is no slack in the string. The angular amplitude of the pendulum is
[KVPY_2014_SB] [2 Marks]
(A) 90° (B) 60° (C) 45° (D) 30°

2. A car around uniform circular track of radius R at a uniform speed v once in every T second. The magnitude
of the centripetal acceleration is ac. If the car now goes uniformly around a larger circular track of radius 2R
and experiences a centripetal acceleration of magnitude 8ac,then its time period is.
[KVPY_2016_SA] [1 Marks]
(A)2T (B) 3T (C)T/2 (D) 3/2T

3. Two particles of identical mass are moving in circular orbits under a potential given by V(r) = Kr–n, where K is
a constant. If the radii of their or orbits are r1,r2 and their speeds are v 1 ; v 2, respectively, then
[KVPY_2016_SA] [1 Marks]
(A) v12r1n  v 22r2n (B) v12r1n  v 22r2n (C) v12r1  v 22r2 (D) v12r12 – n  v 22r22– n

4. A particle at a distance of 1m from the origin starts moving such that dr/d = r, where (r, ) are polar
coordinates. Then the angle between resultant velocity and tangential velocity component is :
(A) 30 degrees (B) 45 degrees (C) 60 degrees [KVPY_2016_SB] [1 Marks]
(D) dependent on where the particle is
5. Which of the following plots represents schematically the dependence of the time period of a pendulum if
measured and plotted as a function of the amplitude of its oscillations ? Note : amplitude need not be small)
[KVPY_2017_SB] [1 Marks]
Time period T

Time period T

Time period T

Time period T

(A) (B) (C) (D)

Amplitude  Amplitude  Amplitude  Amplitude 

EXERCISE - 2

1. (B) 2. (C) 3. (A) 4. (B) 5. (A)

KVPY_ SA STREAM # 574


WAV E ON A ST RI N G
~~~~~~

W AVES
Wave motion is the phenomenon that can be observed almost everywhere around us, as well it appears
in almost every branch of physics. Surface waves on bodies of matter are commonly observed. Sound
waves and light waves are essential to our perception of the environment. All waves have a similar
mathematical description, which makes the study of one kind of wave useful for the study of other
kinds of waves. In this chapter, we will concentrate on string waves, which are type of a mechanical
waves. Mechanical waves require a medium to travel through. Sound waves, water waves are other
examples of mechanical waves. Light waves are not mechanical waves, these are electromagnetic
waves which do not require medium to propagate.
Mechanical waves originate from a disturbance in the medium (such as a stone dropping in a pond) and
the disturbance propagates through the medium. The forces between the atoms in the medium are
responsible for the propagation of mechanical waves. Each atom exerts a force on the atoms near it,
and through this force the motion of the atom is transmitted to the others. The atoms in the medium do
not, however, experience any net displacement. As the wave passes, the atoms simply move back and
forth. Again for simplicity, we concentrate on the study of harmonic waves (that is those that can be
represented by sine and cosine functions).

TYPES OF M ECHAN I CAL W AVES


Mechanical waves can be classified according to the physical properties of the medium, as well as in
other ways.
1. Direction of particle motion :
Waves can be classified by considering the direction of motion of the particles in the medium as wave
passes. If the disturbance travels in the x direction but the particles move in a direction, perpendicular
to the x axis as the wave passes it is called a transverse wave. If the motion of the particles were
parallel to the x axis then it is called a longitudinal wave. A wave pulse in a plucked guitar string is a
transverse wave. A sound wave is a longitudinal wave.

2. Number of dimensions :
Waves can propagate in one, two, or three dimensions. A wave moving along a taut string is a one
dimensional wave. A water wave created by a stone thrown in a pond is a two dimensional wave. A
sound wave created by a gunshot is a three-dimensional wave

3. Periodicity :
A stone dropped into a pond creates a wave pulse, which travels outward in two dimensions. There may
be more than one ripple created, but there is still only one wave pulse. If similar stones are dropped in
the same place at even time intervals, then a periodic wave is created.
4. Shape of wave fronts : The ripples created by a stone dropped into a pond are circular in shape. A
sound wave propagating outward from a point source has spherical wavefronts. A plane wave is a three
dimensional wave with flat wave fronts.
(Far away from a point source emitting spherical waves, the waves appear to be plane waves.)
A solid can sustain transverse as well as longitudinal wave. A fluid has no well-defined form or structure
to maintain and offer far more resistance to compression than to a shearing force. Consequently, only
longitudinal wave can propagate through a gas or within the body of an ideal (non viscous) liquid.
However, transverse waves can exist on the surface of a liquid. In the case of ripples on a pond, the
force restoring the system to equilibrium is the surface tension of the water, whereas for ocean waves,
it is the force of gravity.
Also, if disturbance is restricted to propagate only in one direction and there is no loss of energy during
propagation, then shape of disturbance remains unchanged.

KVPY_ SA STREAM # 575


D ESCRIBIN G WAVES :
Two kinds of graph may be drawn - displacement-distance and displacement-time.
A displacement - distance graph for a transverse mechanical wave shows the displacement y of the
vibrating particles of the transmitting medium at different distance x from the source at a certain instant
i.e. it is like a photograph showing shape of the wave at that particular instant.
The maximum displacement of each particle from its undisturbed position is the amplitude of the wave.
In the figure 1, it is OA or OB.

One wavelength

Displacement

O
Distance x
B

Crest Trough Crest Trough

The wavelength  of a wave is generally taken as the distance between two successive crests or two
successive trough. To be more specific, it is the distance between two consecutive points on the wave
which have same phase.
A displacement-time graph may also be drawn for a wave motion, showing how the displacement of one
particle at a particular distance from the source varies with time. If this is simple harmonic variation
then the graph is a sine curve.
W AVE LEN GTH, FR EQUEN CY, SP EED
If the source of a wave makes f vibrations per second, so too will the particles of the transmitting
medium. That is, the frequency of the waves equals frequency of the source.
When the source makes one complete vibration , one wave is generated and the disturbance spreads
out a distance  from the source. If the source continues to vibrate with constant frequency f, then f
waves will be produced per second and the wave advances a distance f  in one second. If v is the wave
speed then
v = 
This relationship holds for all wave motions.
Travelling wave :
Imagine a horizontal string stretched in the x direction. Its equilibrium shape is flat and straight. Let y
measure the displacement of any particle of the string from its equilibrium position, perpendicular to
the string. If the string is plucked on the left end, a pulse will travel to the right. The vertical displacement
y of the left end of the string (x = 0) is a function of time.
i.e. y (x = 0, t) = f(t)
If there are no frictional losses, the pulse will travel undiminished, retaining its original shape. If the
pulse travels with a speed v, the ‘position’ of the wave pulse is x = vt. Therefore, the displacement of the
x
particle at point x at time t was originated at the left end at time t – . [y, (x, t) is function of both x and
v
x x
t]. But the displacement of the left end at time t is f(t) thus at time t – , it is f(t – ).
v v
Therefore :
x x
y(x, t) = y (x = 0, t – ) = f (t – )
v v

KVPY_ SA STREAM # 576


This can also be expressed as
f f
 (vt – x)  – (x – vt)
v v
y (x, t) = g(x – vt)
using any fixed value of t (i.e. at any instant), this shows shape of the string.
If the wave is travelling in – x direction, then wave equation is written as
x
y(x, t) = f(t + )
v
The quantity x – vt is called phase of the wave function. As phase of the pulse has fixed value
x – vt = const.
dx
Taking the derivative w.r.t. time =v
dt
where v is the phase velocity although often called wave velocity. It is the velocity at which a particular
phase of the disturbance travels through space.
In order for the function to represent a wave travelling at speed v, the three quantities x, v and t must
appear in the combination (x + vt) or (x – vt). Thus (x – vt) 2 is acceptable but x 2 – v 2 t 2 is not.
TRAVELLING SINE WAVE IN ONE DIMENSION (WAVE ON STRING) :
 x
The wave equation y  f  t   is quite general. If holds for arbitrary wave shapes, and for transverse
 v
as well as for longitudinal waves.
A complete description of the wave requires specification of f(x). The most important case, by far, in
physics and engineering is when f(x) is sinusoidal, that is, when the wave has the shape of a sine or
cosine function. This is possible when the source, that is moving the left end of the string, vibrates the
left end x = 0 in a simple harmonic motion. For this, the source has to continuously do work on the
string and energy is continuously supplied to the string.
The equation of motion of the left end may be written as
f (t) = A sin t
where A is amplitude of the wave, that is maximum displacement of a particle in the medium from its
equilibrium position  is angular frequency, that is 2f where f is frequency of SHM of the source.
The displacement of the particle at x at time t will be
 x  x
y  f t   or y = A sin   t   y = A sin (t – kx)
 v  v
 2 2 1
where k =  is called wave number. T = = is period of the wave, that is the time it takes
v   f
to travel the distance between two adjacent crests or through (it is wavelength ).
The wave equation y = A sin (t – kx) says that at x = 0 and t = 0, y = 0. This is not necessarily the
case, of source. For the same condition, y may not equal to zero. Therefore, the most general expression
would involve a phase constant , which allows for other possibilities,
y = A sin (t – kx + )
A suitable choice of  allows any initial condition to be met. The term kx – wt +  is called the phase of
the wave. Two waves with the same phase (on phase differing by a multiple of 2) are said to be “in
phase”. They execute the same motion at the same time.
The velocity of the particle at position x and at time t is given by
y
 A cos (t – kx + )
t
The wave equation has been partially differentiated keeping x as constant, to specify the particle. Note
dx
that wave velocity is different from particle velocity while waves velocity is constant for a medium
dt
and it along the direction of string, whereas particle velocity is perpendicular to wave velocity and is
dependent upon x and t.

KVPY_ SA STREAM # 577


Example 1. A sinusoidal wave travelling in y(cm)
the positive x direction has an 40
am pl i t ude of 15 cm ,
wav el engt h 40 cm and
15
frequency 8 Hz. The vertical
displacement of the medium at
x(cm)
t = 0 and x = 0 is also 15 cm,
as shown.

(a) Find the angular wav e


number, time period, angular
frequency and speed of the
wave.
(b) Determine the phase constant , and write a general expression for the wave function.
2 2 rad 
Solution : (a) k= = 40 cm = rad/cm
 20

1 1
T= = s  = 2f = 16 s–1
f 8
v = f = 320 cm/s
(b) It is given that A = 15 cm
and also y = 15 cm at x = 0 and t = 0
then using y = A sin (t – kx + )
15 = 15 sin   sin  = 1

or =
rad.
2
Therefore, the wave function is
  1   rad  
y = A sin (t – kx + ) = (15 cm) sin (16 s )t   .x  
2   20 cm  2

Example 2. The wave function for a travelling wave on a string is given as



y (x, t) = (0.350 m) sin (10 t – 3x + )
4
(a) What are the speed and direction of travel of the wave?
(b) What is the vertical displacement of the string at t = 0, x = 0.1 m?
(c) What are wavelength and frequency of the wave?

Solution : Y (x, t) = (0.350m) sin (10t – 3  x + )
4
comparing with equation ;

Y = A sin (wt – kx + ) w = 10  , k = 3, f =
4
w 10
(a) speed = = = 3.33 m/sec and along +ve x axis
k 3

(b) y (0.1, 0) = 0.35 sin (10  x O – 3  (0.1) + )
4
 3 
= 0.35 sin    = – 5.48 cm
 4 10 
2 2
(c) k= = 3  =)  cm = 0.67 cm
 3
 10 / 3
and ¦= = 2 / 3 = 5 Hz.

KVPY_ SA STREAM # 578


THE LIN EAR WAVE EQUATI ON :
By using wave function y = A sin (t – kx + ), we can describe the motion of any point on the string.
Any point on the string moves only vertically, and so its x coordinate remains constant. The transverse
velocity v y of the point and its transverse acceleration ay are therefore

 dy  y
v y =  dt   = A cos (t – kx + ) ....(1)
  x constant t

 dv y  v y  2 y
ay =  dt    2 = –2 A sin (t – kx + ) ....(2)
  x constant t t
and hence v y, max = A
ay, max = 2A
The transverse velocity and transverse acceleration of any point on the string do not reach their maximum
value simultaneously. Infact, the transverse velocity reaches its maximum value (A) when the
displacement y = 0, whereas the transverse acceleration reaches its maximum magnitude (2A) when
y = ±A

 dy  y
further  dx   = –kA cos (t – kx + ) ....(3)
  t constant x

2y
= = – k2 A sin (t – kx + ) ....(4)
x 2

y  y
From (1) and (3) =–  v P = – v w × slope
t k x

i.e. if the slope at any point is negative, particle velocity is positive and vice-versa, for a wave moving
along positive x axis i.e. v w is positive.
y
For example, consider two points A and B on
the y-x curve for a wave, as shown. The wave
is moving along positive x-axis.
A
Slope at A is positive therefore at the given B
moment, its velocity is negative. That means
it is coming downward. Reverse is the situation x
for particle at point B. x1 x2
Now using equation (2) and (4)
2y k 2 2y 2y 1 2 y
  
x 2 2 t 2 x 2 v 2 t 2
This is known as the linear wave equation or differential equation representation of the travelling wave
model. We have developed the linear wave equation from a sinusoidal mechanical wave travelling through
a medium, but it is much more general. The linear wave equation successfully describes waves on
strings, sound waves and also electromagnetic waves.

Example 3. Show that the wave function y = eb(x – vt) is a solution of the linear wave equation.
y y
Solution : Y = eb(x–v t) = beb(x–v t) and = (bv)eb(xv– v t)
x t

 2y  2y
=b e 2 b(x–v t)
and = (bv)2 eb(x – v t)
x 2 t 2

 2y
2
1  y
obviously ; = 2 which is a Linear wave equation.
x 2 v 2 t

KVPY_ SA STREAM # 579


THE SPEED OF TRAN SVERSE WAVES ON STRIN GS
The speed of a wave on a string is given by
T
v

where T is tension in the string (in Newtons) and  is mass per unit length of the string (kg/m).
It should be noted that v is speed of the wave w.r.t. the medium (string).
In case the tension is not uniform in the string or string has nonuniform linear mass density then v is
speed at a given point and T and  are corresponding values at that point.

Example 4. A uniform rope of mass m and length L hangs from a ceilling. (a) Show that the speed of a
transverse wave on the rope is a funciton of y, the distance from the lower end, and is given by
v= gy . (b) Show that the time a transverse wave takes to travel the length of the rope is given

by t = 2 L / g .

Solution :

M, i
Y

(a) As mas per unit length


m
µ=
i

T yg
 V=  Tension at P = yg  V= = yg
 

dy
(b) Now = yg
dt

 t
dy

O y
= 
g dt t = 2 /g
o
POW ER TRANSM ITTED ALONG THE STRING BY A SINE W AVE
When a travelling wave is established on a string, energy is transmitted along the direction of propagation of
the wave, in form of potential energy and kinetic energy
Average Power P = 22 f 2 A2 v
t
Energy Transferred = P
0
av dt

Energy transferred in one time period = P av T


This is also equal to the energy stored in one wavelength.

Intensity : Energy transferred per second per unit cross sectional area is called intensity of the wave.
Power P 1
=   = 2 A2v
Cross sectional area s 2
This is average intensity of the wave.
Energy density : Energy per unit volume of the wave
Pdt 
= 
svdt v

KVPY_ SA STREAM # 580


Example 5. A string with linear mass density m = 5.00 × 10–2 kg/m is under a tension of 80.0 N. How much
power must be supplied to the string to generate sinusoidal waves at a frequency of 60.0 Hz
and an amplitude of 6.00 cm?
Solution : The wave speed on the string is
1/ 2
T  80.0 N 
v= =  2
 = 40.0 m/s

  5.00  10 kg / m 
Because  = 60 Hz, the angular frequency  of the sinusoidal waves on the string has the value
 = 2f = 2(60.0 Hz) = 377 s–1
Using these values in following Equation for the power, with A = 6.00 × 10–2 m,. gives
1
p= 2A2 v
2
1
= (5.00 × 10–2 kg/m) (377s–1)2 × (6.00 × 10–2 m)2 (40.0 m/s)
2
= 512 W.

TH E P RIN CIP LE OF SUPER POSITI ON


When two or more waves simultaneously pass through a point, the disturbance at the point is given by
the sum of the disturbances each wave would produce in absence of the other wave(s).
In general, the principle of superposition is valid for small disturbances only. If the string is stretched
too far, the individual displacements do not add to give the resultant displacement. Such waves are
called nonlinear waves. In this course, we shall only be talking about linear waves which obey the
superposition principle.
To put this rule in a mathematical form, let y 1(x, t) and y2(x, t) be the displacements that any element
of the string would experience if each wave travelled alone. The displacement y(x, t) of an element of
the string when the waves overlap is then given by
y(x, t) = y1(x, t) + y2(x, t)
The principal of superposition can also be expressed by stating that overlapping waves algebraically
add to produce a resultant wave. The principle implies that the overlapping waves do not in any way
alter the travel of each other.
If we have two or more waves moving in the medium the resultant waveform is the sum of wave functions
of individual waves.
Fig: a sequence of pictures showing two pulses
travelling in opposite directions along a stretched string.
W hen the two disturbances overlap they giv e a
(a)
complicated pattern as shown in (b). In (c), they have
passed each other and proceed unchanged.

An Illustrative examples of this principle is phenomena (b)


of interference and reflection of waves.

(c)

Example 6. Two waves passing through a region are represented by


y = (1.0 m) sin [(3.14 cm–1) x – (157 s–1) t]
and y = (1.5 cm) sin [(1.57 cm–1)x – (314 s–1) t].
Find the displacement of the particle at x = 4.5 cm at time t = 5.0 ms.

KVPY_ SA STREAM # 581


Solution : According to the principle of superposition, each wave produces its disturbance independent of the
other and the resultant disturbance is equal to the vector sum of the individual disturbance. The
displacements of the particle at x = 4.5 cm at time t = 5.0 ms due to the two waves are,
y1 = (1.0 cm) sin [(3.14 cm–1) (4.5 cm)– (157 s–1) (5.0 × 10–3 s)]
    1.0 cm
= (1.0 cm) sin 4.5  4  = (1.0 cm) sin  4  4  =
    2
and y2 = (1.5 cm) sin [(1.57 cm–1)(4.5 cm) – (314 s–1) (5.0 × 10–3 s)]
   
= (1.5 cm) sin 2.25   2  = (1.5 cm ) sin 2  4 
   

 1.5 cm
= (1.5 cm) sin =–
4 2

0.5 cm
The net displacement is : y = y1 + y2 = = – 0.35 cm.
2

IN TERFEREN CE OF WAVES GOIN G IN SAM E D IRECTION


Suppose two identical sources send sinusoidal waves of same angular frequency  in positive x-
direction. Also, the wave velocity and hence, the wave number k is same for the two waves. One source
may be situated at different points. The two waves arriving at a point then differ in phase. Let the
amplitudes of the two waves be A 1 and A2 and the two waves differ in phase by an angle . Their
equations may be written as
y1 = A1 sin (kx – t)
and y2 = A2 sin (kx – t + ).
According to the principle of superposition, the resultant wave is represented by
y = y1 + y2 = A1 sin (kx – t) + A2 sin (kx – t + ).
we get y = A sin (kx – t + )

where, A= A 12  A 22  2A 1A 2 cos  (A is amplitude of the resultant wave)

A 2 sin 
Also, tan  = ( is phase difference of the resultant wave with the first wave)
A1  A 2 cos 
Constructive and Destructive Interference
Constructive Interference :
When resultant amplitude A is maximum
A = A1 + A2 when cos  = + 1 or  = 2n
where n is an integer.
Destructive interference :
When resultant amplitude A is minimum
or A = |A1 – A2|
When cos  = – 1 or  = (2n + 1)
where n is an integer.

REFLECTION AN D TRAN SM ISSION OF W AVES


A travelling wave, at a rigid or denser boundary, is reflected with a phase reversal but the reflection at an
open boundary (rarer medium) takes place without any phase change. The transmitted wave is never
inverted, but propagation constant k is changed.

INCIDENT WAVE INCIDENT WAVE


yi = Ai sin(t – k1x) yi = Ai sin(t – k1 x)
TRANSMITTED WAVE TRANSMITTED WAVE
y t = At sin(t – k2x) y t = At sin( t – k2 x)

REFLECTED WAVE REFLECTED WAVE


y r = Ar sin( t + k1x + ) y r = Ar sin( t + k 1x)

Fig. : Reflection at denser boundary Fig. : Reflection at rarer boundary

KVPY_ SA STREAM # 582


Amplitude of reflected and transmitted waves :
v 1 and v 2 are speeds of the incident wave and reflected wave in mediums respectively then
v 2  v1 2v 2
Ar = Ai At = .Ai
v1  v 2 v1  v 2

Ar is positive if v 2 > v 1, i.e., wave is reflected from a rarer medium.

Example 7. Two sinusoidal waves of the same frequency are to be sent in the same direction along a taut string.
One wave has an amplitude of 5.0 mm, the other 8.0 mm. (a) What phase difference 1 between the
two waves results in the smallest amplitude of the resultant wave? (b) What is that smallest amplitude?
(c) What phase difference 2 results in the largest amplitude of the resultant wave? (d) What is that
largest amplitude? (e) What is the resultant amplitude if the phase angle is (1 – 2)/2?
Solution : (a) For smallest amplitude ;
AR = | A1 – A2 | and that is possible when 1 =  between A1 and A2
(b) AR = | A1 – A2 | = 3 mm
(c) for largest amplitude ;
AR = | A1 + A2 | and that is possible when  2 = 0 between A1 and A2
(d) AR = | A1 + A2 | = 13 mm

1   2 o 
(e) when  = = =
2 2 2
 1/2
 AR = [A12 + A22 + 2A1 A2 cos ]
2
= 9.4 mm
Ans. (a)  rad; (b) 3.0 mm; (c) 0 rad; (d) 13 mm; (e) 9.4 mm

Example 8. A harmonic wave is travelling on string 1. At a junction with string 2 it is partly reflected and
partly transmitted. The linear mass density of the second string is four times that of the first
string, and that the boundary between the two strings is at x = 0. If the expression for the
incident wave is, y i = Ai cos (k1 x – 1t)
What are the expressions for the transmitted and the reflected waves in terms of A i, k1 and 1?
Solution : Since v = T/ , T 2 = T 1 and 2 = 41
v1
we have, v2 = ... (i)
2
The frequency does not change, that is,
1 = 2 ....(ii)
Also, because k = /v, the wave numbers of the harmonic waves in the two strings are related by,
2 1 1
k2 = v = v / 2 = 2 v = 2k1 ....(iii)
2 1 1

The amplitudes are,


 2v 2   2( v 1 / 2)  2
At =  v  v  Au =  v  ( v / 2)  Ai = A ....(iv)
 1 2   1 1  3 i

 v 2  v1   ( v 1 / 2)  v 1  Ai
and Ar =  v  v  Au =  v  ( v / 2)  Ai = ....(v)
 1 2   1 1  3
Now with equation (ii), (iii) and (iv), the transmitted wave can be written as,
2
A cos (2k1 x – 1t)
yt = Ans.
3 i
Similarly the reflected wave can be expressed as,
Ai
= cos (k1x + 1t + ) Ans.
3

KVPY_ SA STREAM # 583


STAN D IN G WAVES :
Suppose two sine waves of equal amplitude and frequency propagate on a long string in opposite
directions. The equations of the two waves are given by
y1 = A sin (t – kx) and y2 = A sin (t + kx + ).
These waves interfere to produce what we call standing waves. To understand these waves, let us
discuss the special case when  = 0.
The resultant displacements of the particles of the string are given by the principle of superposition as
y = y1 + y2
= A [sin (t – kx) + sin(t + kx)]
= 2A sin t cos kx
or, y = (2A cos kx) sin t. ....

This is the required result and from this it is clear that :


1. As this equation satisfies the wave equation,

2y 1 2 y
2

x v2  t2

it represents a wave. However, as it is not of the form f(ax ± bt), the wave is not travelling and
so is called standing or stationary wave.

2. The amplitude of the wave


As = 2A cos kx
is not constant but varies periodically with position (and not with time as in beats).
3. The points for which amplitude is minimum are called nodes and for these

 3 5
cos kx = 0, i.e., kx = , ,
2 2 2

 3 5  2 
i.e., x= , , , .... as k   
4 4 4  

i.e., in a stationary wave, nodes are equally spaced.

4. The points for which amplitude is maximum are called antinodes and for these,
cos kx = ± 1, i.e., kx = 0, , 2, 3, ......

 2 3  2 
i.e., x = 0, , , ,.... as k   
2 2 2  

i.e., like nodes, antinodes are also equally spaced with spacing (/2) and Amax = ± 2A.
Furthermore, nodes and antinodes are alternate with spacing (/4).

5. The nodes divide the medium into segments (or loops). All the particles in a segment vibrate in
same phase, but in opposite phase with the particles in the adjacent segment. Twice in one
period all the particles pass through their mean position simultaneously with maximum velocity
(As), the direction of motion being reversed after each half cycle.

KVPY_ SA STREAM # 584


Node

t=0

t = T/8

t = T/4
t = 3T/8

t = T/2

t = 5T/8

t = 3T/4

t = 7T/8

t=T

Antinode
(a)

6. Standing waves can be transverse or longitudinal, e.g., in strings (under tension) if reflected
wave exists, the waves are transverse-stationary, while in organ pipes waves are longitudinal-
stationary.
7. As in stationary waves nodes are permanently at rest, so no energy can be transmitted across
them, i.e., energy of one region (segment) is confined in that region. However, this energy
oscillates between elastic potential energy and kinetic energy of the particles of the medium.
When all the particles are at their extreme positions KE is minimum while elastic PE is maximum
(as shown in figure A), and when all the particles (simultaneously) pass through their mean
position KE will be maximum while elastic PE minimum (Figure B). The total energy confined
in a segment (elastic PE + KE), always remains the same.

v=0 v = max. = As string


string
v = min = 0
y y

x x

Elastic PE = max = E Elastic PE = min = 0


Kinetic energy = min = 0 Kinetic energy = max = E
(A) (B)

Example 9. Two travelling waves of equal amplitudes and equal frequencies move in opposite direction along a
string. They interfere to produce a standing wave having the equation.
y = A cos kx sin t
in which A = 1.0 mm, k = 1.57 cm–1 and  = 78.5 s–1. (a) Find the velocity and amplitude of the
component travelling waves. (b) Find the node closest to the origin in the region x > 0. (c) Find the
antinode closest to the origin in the region x > 0. (d) Find the amplitude of the particle at x = 2.33 cm.
Solution : (a) The standing wave is formed by the superposition of the waves
A A
y1 = sin (t – kx) and y2 = sin (t + kx).
2 2
The wave velocity (magnitude) of either of the waves is

KVPY_ SA STREAM # 585


 78.5 s 1
v= = = 50 cm/s; Amplitude = 0.5 mm.
k 1.57 cm 1
(b) For a node, cos kx = 0.
The smallest positive x satisfying this relation is given by

 3.14
kx = /2or, x= = = 1 cm
2k 2  1.57 cm 1
(c) For an antinode, |cos kx| = 1.
The smallest positive x satisfying this relation is given by

kx =  or, x= = 2 cm
k
(d) The amplitude of vibration of the particle at x is given by | A cos kx |. For the given point,
7 
kx = (1.57 cm–1) (2.33 cm) = =+ .
6 6
Thus, the amplitude will be
3
(1.0 mm) | cos ( + /6) | = mm = 0.86 mm.
3

VIBRATION OF STRING :
(a) Fixed at both ends :
Suppose a string of length L is kept fixed at the ends x = 0 and x = L. In such a system suppose we send a
continuous sinusoidal wave of a certain frequency, say, toward the right. When the wave reaches the right
end. It gets reflected and begins to travel back. The left-going wave then overlaps the wave, which is still
travelling to the right. When the left-going wave reaches the left end, it gets reflected again and the newly
reflected wave begins to travel to the right. overlapping the left-going wave. This process will continue and,
therefore, very soon we have many overlapping waves, which interfere with one another. In such a system, at
any point x and at any time t, there are always two waves, one moving to the left and another to the right. We,
therefore, have
y1(x, t) = ym sin (kx – t) (wave travelling in the positive direction of x-axis)
and y2(x, t) = ym sin (kx + t) (wave travelling in the negative direction of x-axis).
The principle of superposition gives, for the combined wave
y’(x, t) = y1(x, t) + y2(x, t)
= ym sin (kx – wt) + ym sin (kx + t)
= (2ym sin kx) cos t
It is seen that the points of maximum or minimum amplitude stay at one position.
Nodes : The amplitude is zero for values of kx that give sin kx = 0 i.e. for,
kx = n, for n = 0, 1, 2, 3,.....
Substituting k = 2 in this equation, we get

x=n , for n = 0, 1, 2, 3,.....
2

The positions of zero amplitude are called the nodes. Note that a distance of or half a wavelength
2
separates two consecutive nodes.
Antinodes :
The amplitude has a maximum value of 2ym, which occurs for the values of kx that give |sin kx| = 1. Those
values are
kx = (n + 1/2) for n = 0, 1, 2, 3,....
Substituting k = 2 in this equation, we get.

x = (n + 1/2) for n = 0, 1, 2, 3,....
2
KVPY_ SA STREAM # 586
as the positions of maximum amplitude. These are called the antinodes. The antinodes are separated by /
2 and are located half way between pairs of nodes.
For a stretched string of length L, fixed at both ends, the two ends of the ends is chosen as position x = 0,
then the other end is x = L. In order that this end is a node; the length L must satisfy the condition

L=n , for n = 1, 2, 3,....
2
This condition shows that standing waves on a string of length L have restricted wavelength given by
2L
= , for n = 1, 2, 3,.....
n
The frequencies corresponding to these wavelengths follow from Eq. as
v
f=n , for n = 1, 2, 3,.....
2L
where v is the speed of travelling waves on the string. The set of frequencies given by equation are called the
natural frequencies or modes of oscillation of the system. This equation tells us that the natural frequencies
v
of a string are integral multiples of the lowest frequency f = , which corresponds to n = 1. The oscillation
2L
mode with that lowest frequency is called the fundamental mode or the first harmonic. The second harmonic
or first overtone is the oscillation mode with n = 2. The third harmonic and second overtone corresponds to
n = 3 and so on. The frequencies associated with these modes are often labeled as 1, 2, 3 and so on. The
collection of all possible modes is called the harmonic series and n is called the harmonic number.
Some of the harmonic of a stretched string fixed at both the ends are shown in figure.
A
(a)
Fundamental
or first harmonic
f0 = v/2L
A

A A (b)
second harmonic
or first overtone =
N f1 = 2f0 = 2v/2L
A A
(c)
A A A third harmonic
or 2nd
N N overtone =
A A A f2 = 3f0, = 3v/2L
(d)
A A A A
fourth harmonic
or 3rd
N N N overtone =
A A A A
f3 = 4f0, = 4v/2L
(b) Fixed at one end :
Standing waves can be produced on a string which is fixed at one end and whose other end is free to move
in a transverse direction. Such a free end can be nearly achieved by connecting the string to a very light
thread.
If the vibrations are produced by a source of “correct” frequency, standing waves are produced. If the end x =
0 is fixed and x = L is free, the equation is again given by
y = 2A sin kx cos t
with the boundary condition that x = L is an antinode. The boundary condition that x = 0 is a node is
automatically satisfied by the above equation. For x = L to be an antinode,
sin kL = ± 1

 1 2L  1
or, kL =  n    or, = n   
 2   2 

KVPY_ SA STREAM # 587


1
1 v n
2Lf 1  2
or, =n+ or, f = n   = T /  .....
v 2  2  2L 2L
These are the normal frequencies of vibration. The fundamental frequency is obtained when n = 0, i.e.,

(a) Fundamental
f 0 = v/4L N
A
The overtone frequencies are
3v First
(b) Overtone
f1 = = 3f0 N N
4L A A

5v Second
f2 = = 5f0 (c) Overtone
4L A N A
N
A

We see that all the harmonic of the fundamental are not the allowed frequencies for the standing waves. Only
the odd harmonics are the overtones. Figure shows shapes of the string for some of the normal modes.

Example 10. A middle C string on a piano has a fundamental freuquency of 262 Hz, and the A note has fndametal
frquency of 440 Hz. (a) Calculate the frequencies of the next two harmonics of the C string. (b) If the
strings for the A and C notes are assumed to have the same mass per unit length and the same
length, determine the ratio of tensions in the two strings.
Solution :. (a) Because f1 = 262 Hz for the C string, we can use Equation to find the frequencies f2 and f3;
f 2 = 2f1 = 524 Hz
f 3 = 3f1 = 786 Hz
Using Equation for the two strings vibrating at their fundamental frequencies gives

1 TA 1 TC
f 1A =  f 1C =
2L  2L 

2 2
f1A TA TA  f1A   440 Hz 
 =  =   =   = 2.82.
f1C TC TC  Ans.
 f1C   262 Hz 

LAW S OF TRANSVERSE VIBRATIONS OF A STRING - SONOM ETER W IRE

1 1 2 f L
(a) Law of length f so f  L ; if T &  are constant
L 2 1

1 1 f T
(b) Law of tension f  T so f  T ; L &  are constant
2 2

1 1 2f 
(c) Law of mass f 
 so f2 1 ; T & L are constant

KVPY_ SA STREAM # 588


SOU N D WAV ES

1. PROPAGATION OF SOUND WAVES :
Sound is a mechanical three dimensional and longitudinal wave that is created by a vibrating source such as
a guitar string, the human vocal cords, the prongs of a tuning fork or the diaphragm of a loudspeaker. Being
a mechanical wave, sound needs a medium having properties of inertia and elasticity for its propagation.
Sound waves propagate in any medium through a series of periodic compressions and rarefactions of pres-
sure, which is produced by the vibrating source.
Consider a tuning fork producing sound waves.

When Prong B moves outward towards right it compresses the air in front of it, causing the pressure to rise
slightly. The region of increased pressure is called a compression pulse and it travels away from the prong
with the speed of sound.

A B normal
v atmospheric
 pressure
compression
pulse

After producing the compression pulse, the prong B reverses its motion and moves inward. This drags away
some air from the region in front of it, causing the pressure to dip slightly below the normal pressure. This
region of decreased pressure is called a rarefaction pulse. Following immediately behind the compression
pulse, the rarefaction pulse also travels away from the prong with the speed of sound.

A B
. . .
. . . v
. . .
}

rarefaction compression pulse


pulse

If the prongs vibrate in SHM, the pressure variations in the layer close to the prong also varies simple
harmonically and hence increase in pressure above normal value can be written as
P = P0 sin t
where P0 is the maximum increase in pressure above normal value.
As this disturbance travel towards right with wave velocity v, the excess pressure at any position x at time t
will be given by

P = P0 sin (t – x/v) (1.1)


Using p = P, p0 = P0 , the above equation of sound wave can b written as :
p = p0 sin (t – x/v) (1.2)

Example 1.
The equation of a sound wave in air is given by
p = (0.02 ) sin [(3000 ) t – (9.0 ) x ], where all variables are in S.. units.
(a) Find the frequency, wavelength and the speed of sound wave in air.
(b) If the equilibrium pressure of air is 1.0 × 105 N/m2, what are the maximum and minimum pressures
at a point as the wave passes through that point?

KVPY_ SA STREAM # 589


Solution :
(a) Comparing with the standard form of a travelling wave
p = p0 sin [ (t – x/v)]
we see that  = 3000 s–1. The frequency is
 3000
f= = Hz
2 2
Also from the same comparison, /v = 9.0 m–1.
 3000 s 1 1000
or, v= = m/s–1
9.0 m 1 9.0 m1 3

v 1000 / 3m / s 2
The wavelength is  = = 3000 / 2 Hz  9 m
f
(b) The pressure amplitude is p0 = 0.02 N/m2. Hence, the maximum and minimum pressures at a point
in the wave motion will be (1.01 × 105 ± 0.02) N/m2.

2. FREQUENCY AND PITCH OF SOUND WAVES


FREQUENCY :
Each cycle of a sound wave includes one compression and one rarefaction, and frequency is the number of
cycles per second that passes by a given location. This is normally equal to the frequency of vibration of the
(tuning fork) source producing sound. If the source, vibrates in SHM of a single frequency, sound produced
has a single frequency and it is called a pure tone..
However a sound source may not always vibrate in SHM (this is the case with most of the common sound
sources e.g. guitar string, human vocal card, surface of drum etc.) and hence the pulse generated by it may
not have the shape of a sine wave. But even such a pulse may be considered to be obtained by superposition
of a large number of sine waves of different frequency and amplitudes. We say that the pulse contain all
these frequencies.

AUDIBLE FREQUENCY RANGE FOR HUMAN :


A normal person hears all frequencies between 20 & 20 KHz. This is a subjective range (obtained experimen-
tally) which may vary slightly from person to person. The ability to hear the high frequencies decreases with
age and a middle-age person can hear only upto 12 to 14 KHz.

INFRASONIC SOUND :
Sound can be generated with frequency below 20 Hz called infrasonic sound.

ULTRASONIC SOUND :
Sound can be generated with frequency above 20 kHz called ultrasonic sound.

Even through humans cannot hear these frequencies, other animals may. For instance Rhinos communicate
through infrasonic frequencies as low as 5Hz, and bats use ultrasonic frequencies as high as 100 KHz for
navigating.

PITCH :
Frequency as we have discussed till now is an objective property measured its units is Hz and which can be
assigned a unique value. However a person’s perception of frequency is subjective. The brain interprets fre-
quency primarily in terms of a subjective quality called Pitch. A pure note of high frequency is interpreted as
high-pitched sound and a pure note of low frequency as low-pitched sound

3. PRESSURE WAVE AND DISPLACEM ENT WAVE :


We can describe sound waves either in terms of excess pressure (equation 1.1) or in terms of the longitudi-
nal displacement suffered by the particles of the medium w.r.t. mean position.
If s = s0 sin (t – x/v) represents a sound wave where,
s = displacement of medium particle from its mean position at x,

KVPY_ SA STREAM # 590


s = s0 sin (t – kx) ...(3.1)
Change in volume = V = (x + s)A – xA = sA
V sA s
 
V xA x
BV
P  
V
Bs
P  
x

Bds
dp  
dx
dp = –B (–k s0) cos (t – kx)
dp = Bks0 cos (t – kx)
dp = (dp)max cos (t – kx)
p = p0 sin (t – kx + /2) .... (3.2)
where p = dp = variation in pressure at position x and
p0 = Bks0 = maximum pressure variation
Equation 3.2 represents that same sound wave where, P is excess pressure at position x, over and above
the average atmospheric pressure
and pressure amplitude p0 is given by P0 = BKs0 .....(3.3)
(B = Bulk modulus of the medium, K = angular wave number)
 Note from equation (3.1) and (3.2) that the displacement of a medium particle and excess pressure

at any position are out of phase by . Hence a displacement maxima corresponds to a pressure
2
minima and vice-versa.

4. SPEED OF SOUND WAVES


4.1 Velocity of sound waves in a linear solid medium is given by

Y
v ....(4.1)

where Y = young’s modulus of elasticity and  = density.

4.2. Velocity of sound waves in a fluid medium (liquid or gas) is given by

B
v ....(4.2)

where,  = density of the medium and B = Bulk modulus of the medium given by,
dP
B=–V .....(4.3)
dV

Newton’s formula : Newton assumed propagation of sound through a gaseous medium to be an isothermal
process.
PV = constant
dP P
 
dV V
and hence B = P using equ. ...(4.3)
and thus he obtained for velocity of sound in a gas,

P RT
v  where M = molar mass
 M

KVPY_ SA STREAM # 591


the density of air at 0º and pressure 76 cm of Hg column is  = 1.293 kg/m3. This temperature and pressure
is called standard temperature and pressure at STP. Speed of sound in air is 280 m/s. This value is some
what less than measured speed of sound in air 332 m/s then Laplace suggested the correction.
Laplace’s correction : Later Laplace established that propagation of sound in a gas is not an isothermal
but an adiabatic process and hence PV  = constant
dP P
  
dV V

dP
where, B = – V = P
dV
and hence speed of sound in a gas,

P RT
v=  .... (4.4)
 M

4.3 Factors affecting speed of sound in atmosphere.


(a) Effect of temperature : as temperature (T) increases velocity (v) increases.
v T
For small change in temperature above room temperature v increases linearly by 0.6 m/s for every
1°C rise in temp.

R
v  T1 / 2
M

v 1 T

v 2 T

1 v
v    T
2 T

v  (0.6) T

(b) Effect of pressure :

P RT
The speed of sound in a gas is given by v = 
 M
(c) Effect of humidity : With increase in humidity density decreases. This is because the molar mass
of water vapour is less than the molar mass of air.

So at constant temperature, if P changes then  also changes in such a way that P/ remains constant.
Hence pressure does not have any effect on velocity of sound as long as temperature is constant.

5. INTENSITY OF SOUND WAVES :


Like any other progressive wave, sound waves also carry energy from one point of space to the other. This
energy can be used to do work, for example, forcing the eardrums to vibrate or in the extreme case of a sonic
boom created by a supersonic jet, can even cause glass panes of windows to crack.
The amount of energy carried per unit time by a wave is called its power and power per unit area held
perpendicular to the direction of energy flow is called intensity.
For a sound wave travelling along positive x-axis described by the equation.
s = s0 sin (t – kx + )
P = p0 cos (t – kx + )
s
= s0 cos (t – kx + )
t

KVPY_ SA STREAM # 592


s
Instantaneous power P = F.v = pA
t
P = p0 cos (t – kx + ) As0 cos (t – kx + )
Paverage = <P>
= p0 As0 <cos2 (t – kx + )>

p 0 s 0 A B
=  v=
2 
B = v 2  p0 = Bks0 = = v 2 ks0

1  p  p 02 A pAv2 s 20
Paverage = p0 A 02  = =
2  v k  2 v 2
 

p 02 A
maximum power = Pmax = = (pA) v v p2, max = pAv2s02
v

Av2 s 20
Total energy transfer = Pav × t = ×t
2
Average intensity = average power / area
the average intensity at position x is given by

1 2 s02B P2v
<> = = 0 .... (5.1)
2 v 2B
Substituting B = v 2, intensity can also be expressed as

P02
= .... (5.2)
2 v

Note :
1 1 a
 If the source is a point source then I  2
and s0  and s = sin (t – kr + )
r r r

1 1
 If a sound source is a line source then I  and s0  and
r r
a
s= sin (t – kr + )
r

Example 2.
A microphone of cross-sectional area 0.40 cm2 is placed in front of a small speaker emitting  W of sound
output. If the distance between the microphone and the speaker is 2.0 m, how much energy falls on the
microphone in 5.0 s ?
Solution :
The energy emitted by the speaker in one second is  J. Let us consider a sphere of radius 2.0 m centered
at the speaker. The energy J falls normally on the total surface of this sphere in one second. The energy
falling on the area 0.4 cm2 of the microphone in one second

0.4 cm2
= ×  J = 2.5 × 10–6J.
4  (2.0 m)2
The energy falling on the microphone in 5.0 is
2.5 × 10–6 J × 5 = 12.5 µJ.

KVPY_ SA STREAM # 593


Example 3.
Find the amplitude of vibration of the particles of air through which a sound wave of intensity
8.0 × 10–6 W/m2 and frequency 5.0 kHz is passing. Density of air = 1.2 kg/m3 and speed of sound in air = 330
m/s.
Solution :
The relation between the intensity of sound and the displacement amplitude is
2 s20B
= , where B = v 2 and  = 2
2v

  = 22 s0220v or, s02 = 2 2
2  0 v

8.0  10 6 W / m 2
= or s0 = 6.4 nm.
22  (25.0  10 6 s 2 )  (1.2 kg / m3 )  (330 m / s)

6. LOUDNESS :
Audible intensity range for humans :
The ability of human to perceive intensity at different frequency is different. The perception of intensity is
maximum at 1000 Hz and perception of intensity decreases as the frequency decreases or increases from
1000 Hz.

 For a 1000 Hz tone, the smallest sound intensity that a human ear can detect is 10–12 watt./m2. On the other
hand, continuous exposure to intensities above 1W/m2 can result in permanent hearing loss.

 The overall perception of intensity of sound to human ear is called loudness.

 Human ear do not perceives loudness on a linear intensity scale rather it perceives loudness on logarithmic
intensity scale.
For example ;
If intensity is increased 10 times human ear does not perceive 10 times increase in loudness. It roughly
perceived that loudness is doubled where intensity increased by 10 times. Hence it is prudent to define a
logarithmic scale for intensity.
DECIBEL SCALE :
The logarithmic scale which is used for comparing two sound intensity is called decibel scale.

The intensity level  described in terms of decibels is defined as


  
 =10 log    (dB)
 0
Here 0 is the threshold intensity of hearing for human ear
i.e.  = 10–12 watt/m2.
 In terms of decibel threshold of human hearing is 1 dB
 Note that intensity level  is a dimensionless quantity and is not same as intensity expressed in W/m2.

Example 4.
The sound level at a point is increased by 40 dB. By what factor is the pressure amplitude increased ?
Solution :
The sound level in dB is
  
 = 10 log10   .
 0 
If 1 and 2 are the sound levels and 1 and 2 are the intensities in the two cases,

KVPY_ SA STREAM # 594


  2   1 
2 – 1 = 10 log10     log10   
  0  0 
 2 
or, 40 = 10 log10  
 1 
2
or, = 104.
1
As the intensity is proportional to the square of the pressure amplitude,
p 02 2
we have
p 01
= 1 = 10000  100.

7. INTERFERENCE OF SOUND WAVES :


If p1 = pm1 sin (t – kx1 + 1)
and p2 = pm2 sin (t – kx2 + 2)
resultant excess pressure at point O is
p = p1 + p2
 p = p0 sin (t – kx + )

where, p0 = 2
pm 1
2
 pm 2
 2pm1 p m2 cos  ,  = | k (x1 – x2) + (2 – 1) | ...(6.1)
(i) For constructive interference
 = 2n  p0 = pm1 + pm2

(ii) For destructive interference


 = (2n+ 1)  p0 = | pm1 – pm2 |
2
If  is only due to path difference, then  = x, and

Condition for constructive interference : x = n, n = 0, ± 1, ± 2

Condition for destructive interference : x = (2n + 1)  n = 0, ± 1, ± 2
2
from equation (6.1)
P02  Pm21  Pm2 2  2Pm1Pm2 cos 
Since intensity,  (Pressure amplitude)2,
we have, for resultant intensity,  = 1 + 2 + 2  1 2 cos  ....(6.2)
If 1 = 2 = 0

 = 20 (1 + cos )   = 40 cos2 ....(6.3)
2
Hence in this case,
for constructive interference :  = 0, 2, 4.... and max = 40
and for destructive interference : = , 3 ... and min = 0

7.1 Coherence : Two sources are said to be coherent if the phase difference between them does not
change with time. In this case their resultant intensity at any point in space remains constant with
time. Two independent sources of sound are generally incoherent in nature, i.e. phase difference
between them changes with time and hence the resultant intensity due to them at any point in
space changes with time.

KVPY_ SA STREAM # 595


Example 5.
Figure shows a tube structure in which a sound signal is sent from one end and is received at the other end. The
semicircular part has a radius of 10.0 cm. The frequency of the sound source can be varied from 1 to 10 kHz.
Find the frequencies at which the ear perceives maximum intensity. The speed of sound in air = 342 m/s.

Solution :
The sound wave bifurcates at the junction of the straight and the semicircular parts. The wave through the
straight part travels a distance p1 = 2 × 10 cm and the wave through the curved part travels a distance p2 = 
10 cm = 31.4 cm before they meet again and travel to the receiver. The path difference between the two
waves received is, therefore.
p = p2 – p1 = 31.4 cm – 20 cm = 11.4 cm
v 330 m / s
The wavelength of either wave is = . For constructive interference, p= n, where n is an integer..
 
v n.v
or, p = n.  =
 p

n.342
 = 3000 n
(0.114)
Thus, the frequencies within the specified range which cause maximum of intensity are
3000 × 1, 3000 × 2
and 3000 × 3 Hz

Example 6.
A source emitting sound of frequency 165 Hz is placed in front of a wall at a distance of 2 m from it. A detector
is also placed in front of the wall at the same distance from it. Find the minimum distance between the source
and the detector for which the detector detects a maximum of sound. Speed of sound in air = 330 m/s.
Solution :
The situation is shown in figure. Suppose the detector is placed at
a distance of x meter from the source. The direct wave received
from the source travels a distance of x meter. The wave reaching the
detector after reflection from the wall has travelled a distance of
2[(2)2 + x2/4]1/2 metre. The path difference between the two waves is

  4
1/ 2 
 2 x 
= 2 ( 2)    x  metre.
  4  
 
Constructive interference will take place when  = , 2, ...... The minimum distance x for a maximum
corresponds to
= ..........(i)

 330 m / s
The wavelength is  = = = 2 m.
v 165 s 1

1/ 2
 2 x2 
Thus, by (i) 2 ( 2)   –x=2
 4 

1/ 2
 x2 
4  
x x2 x2
or, =1+ or, 4+ =1+ +x
 4  2 4 4
or, 3 = x.
Thus, the detector should be placed at a distance of 3 m from the source. Note that there is no abrupt phase
change.

KVPY_ SA STREAM # 596


8. REFLECTION OF SOUND W AVES :
Reflection of sound waves from a rigid boundary (e.g. closed end of an organ pipe) is analogous to reflection
of a string wave from rigid boundary; reflection accompanied by an inversion i.e. an abrupt phase change of
. This is consistent with the requirement of displacement amplitude to remain zero at the rigid end, since a
medium particle at the rigid end can not vibrate. As the excess pressure and displacement corresponding to
the same sound wave vary by /2 in term of phase, a displacement minima at the rigid end will be a point of
pressure maxima. This implies that the reflected pressure wave from the rigid boundary will have same phase
as the incident wave, i.e., a compression pulse is reflected as a compression pulse and a rarefaction pulse
is reflected as a rarefaction pulse.

On the other hand, reflection of sound wave from a low pressure region (like open end of an organ pipe) is
analogous to reflection of string wave from a free end. This point corresponds to a displacement maxima, so
that the incident & reflected displacement wave at this point must be in phase. This would imply that this
point would be a minima for pressure wave (i.e. pressure at this point remains at its average value), and
hence the reflected pressure wave would be out of phase by  with respect to the incident wave. i.e. a
compression pulse is reflected as a rarefaction pulse and vice-versa.

9. LONGITUDINAL STANDING WAVES :


Two longitudinal waves of same frequency and amplitude travelling in opposite directions interfere to produce
a standing wave.
If the two interfering waves are given by
p1 = p0 sin (t – kx)
and p2 = p0 sin (t + kx + )
then the equation. of the resultant standing wave would be given by
 
p = p1 + p2 = 2p0 cos (kx + ) sin (t + )
2 2

 p = p´0 sin (t + ) .... (9.1)
2
This is equation of SHM in which the amplitude p´0 depends on position as

p’0 = 2p0 cos (kx + ) ....(9.2)
2
Points where pressure remains permanently at its average value; i.e. pressure amplitude is zero is called a
pressure node, and the condition for a pressure node would be given by
p´0 = 0

i.e. cos(kx + ) = 0
2
 
i.e. kx + = 2n ± , n = 0, 1, 2,..... ..... (9.3)
2 2
Similarly points where pressure amplitude is maximum is called a pressure antinode and condition for a
pressure antinode would be given by
p´0 = ± 2p0

i.e. cos(kx + ) = ± 1
2

or (kx + ) = n n = 0, 1, 2,..... ....(9.4)
2
 Note that a pressure node in a standing wave would correspond to a displacement antinode; and a pressure
anti-node would correspond to a displacement node.
 (when we label eqn (8.1) as SHM, what we mean is that excess pressure at any point varies simple-
harmonically. If the sound waves were represented in terms of displacement waves, then the equation of
standing wave corresponding to (8.1) would be
 
s = s´0 cos(t + ) where s´0 = 2s0 sin (kx + )
2 2
This can be easily observed to be an equation of SHM. It represents the medium particles moving simple
harmonically about their mean position at x.)

KVPY_ SA STREAM # 597


Example 7.
The range audible frequency for humans is 20 Hz to 20,000 Hz. If speed of sound in air is 336 m/s. What can
be the maximum and minimum length of a musical instrument, based on resonance pipe.
Solution :
v
For an open pipe, f = n
2
v
 = .n
2f
Similarly for a closed pipe,
v
= (2n + 1)
4f
v 336
min = (2n + 1)min = = 4.2 mm
4fmax 4  20000

v v
max = 2f nmax = n = 8.4 (m) × nmax
min 2  20 max
clearly there is no upper limit on the length of such an musical instrument.

10. VIBRATION OF AIR COLUM NS :


Standing waves can be set up in air-columns trapped inside cylindrical tubes if frequency of the tuning fork
sounding the air column matches one of the natural frequency of air columns. In such a case the sound of the
tuning fork becomes markedly louder, and we say there is resonance between the tuning fork and air-
column. To determine the natural frequency of the air-column, notice that there is a displacement node
(pressure antinode) at each closed end of the tube as air molecules there are not free to move, and a
displacement antinode (pressure-node) at each open end of the air-column.
In reality antinodes do not occurs exactly at the open end but a little distance outside. However if diameter
of tube is small compared to its length, this end correction can be neglected.

10.1 Closed organ pipe


(In the diagram, Ap = Pressure antinode, As = displacement antinode, Np = pressure node,
Ns = displacement node.)

p s

AP NS AS
NP

Fundamental mode :
The smallest frequency (largest wavelength) that satisfies the boundary condition for resonance (i.e.
displacement node at left end and antinode at right end is 0 = 4, where  = length of closed pipe the
corresponding frequency.
v v
0 = = is called the fundamental frequency. ..(9.1)
 4L

p s

Ap Np Ap Np Ns As Ns As

KVPY_ SA STREAM # 598


First Overtone : Here there is one node and one antinode apart from the nodes and antinodes at the ends.
4 0
1 = =
3 3
and corresponding frequency,
v
1 =  = 30
1

This frequency is 3 times the fundamental frequency and hence is called the 3rd harmonic.

nth overtone :
In general, the nth overtone will have n nodes and n antinodes between the two ends. The corresponding
wavelength is
4 0
n = = and nn  ( 2n + 1)0 ....(9.2)
2n  1 2n  1
This corresponds to the (2n + 1)th harmonic. Clearly only odd harmonic are allowed in a closed pipe.

10.2 Open organ pipe :


Np Ap Np
P

Fundamental mode :
The smallest frequency (largest wave length) that satisfies the boundary condition for resonance
(i.e. displacement antinodes at both ends) is,
0 = 2
corresponding frequency, is called the fundamental frequency
v
0 = ....(10.3)
2

As Ns As Ns As p
s

Np Ap Np Ap Np

1st Overtone : Here there is one displacement antinode between the two antinodes at the ends.
2 0
1 =   =
2 2
and, corresponding frequency
v
1 =  = 20
1
This frequency is 2 times the fundamental frequency and is called the 2nd harmonic.
nth overtone : The nth overtone has n displacement antinodes between the two antinode at the ends.
2 0
n = = and n = (n + 1) 0 ....(10.4)
n1 n1
This correspond to (n + 1)th harmonic: clearly both even and odd harmonics are allowed in an open pipe.

KVPY_ SA STREAM # 599


10.3 End correction : As mentioned earlier the displacement antinode at an open end of an organ pipe
lies slightly outside the open end. The distance of the antinode from the open end is called end
correction and its value is given by
Np Ap Np
P
2r
e = 0.6 r

e=0.6r
where r = radius of the organ pipe.
with end correction, the fundamental frequency of a closed pipe (fc) and an open organ pipe (f0) will be
given by
v v
fc = and f0 = ....(10.5)
4(  0.6 r ) 2(  1.2 r )

Example 8.
A closed organ pipe has length ‘ ‘. The air in it is vibrating in 3rd overtone with maximum amplitude ‘ a‘.
Find the amplitude at a distance of  /7 from closed end of the pipe.
Solution :
The figure shows variation of displacement of particles in a closed organ pipe for 3 rd overtone.

7 4  
For third overtone  = or  = or =
4 7 4 7


Hence the amplitude at P at a distance from closed end is ‘a’ because there is an antinode at that
7
point

11. INTERFERENCE IN TIM E : BEATS


When two sound waves of same amplitude and different frequency superimpose, then intensity at any point
in space varies periodically with time. This effect is called beats.
If the equation of the two interfering sound waves emitted by s1 and s2 at point O are,
p1 = p0 sin (2f 1 t – k1x1 + 1)
p2 = p0 sin (2f 2 t – k2x2 + 2) s1
Let –k1x1 + 1 = 1 and – k2x2 + 2 = 2 x1

By principle of superposition O
   – s2 x2
= 2p0 sin ((f 1 + f 2)t + 1 2 ) cos ((f 1 – f 2) t + 1 2 )
2 2
 f1  f2 
i.e., the resultant sound at point O has frequency   while pressure amplitude p´ (t) varies with time as
 2  0

 1   2 
p´0 (t) = 2p0 cos (f1  f2 )t  2 

 f1  f2 
Hence pressure amplitude at point O varies with time with a frequency of  .
 2 
Hence sound intensity will vary with a frequency f1 – f2.
This frequency is called beat frequency (fB) and the time interval between two successive intensity maxima
(or minima) is called beat time period (TB)
fB = f1 – f2
1
TB = (10.1)
f1  f2

KVPY_ SA STREAM # 600


IMPORTANT POINTS :
(i) The frequency | f 1 – f2 | should be less than 16 Hz, for it to be audible.
(ii) Beat phenomenon can be used for determining an unknown frequency by sounding it together with
a source of known frequency.
(iii) If the arm of a tuning fork is waxed or loaded, then its frequency decreases.
(iv) If arm of tuning fork is filed, then its frequency increases.

Example 9.
A string 25 cm long fixed at both ends and having a mass of 2.5 g is under tension. A pipe closed from one
end is 40 cm long. When the string is set vibrating in its first overtone and the air in the pipe in its fundamental
frequency, 8 beats per second are heard. It is observed that decreasing the tension in the string decreases
the beat frequency. If the speed of sound in air is 320 m/s. Find tension in the string.
Solution :
2.5
 = 0.1 g / cm = 10–2 Kg/m
25
Ist overtone

1 T
 s = 25 cm = 0.25 m  fs =
s 
pipe in fundamental freq

V
 p = 160 cm = 1.6 m  fp = 
p

 by decreasing the tension , beat freq is decreased


 fs > fp  f s –f p = 8

1 T 320
  = 8  T = 27.04 N
0.25 10 2 1. 6

12. DOPPLER’S EFFECT


When there is relative motion between the source of a sound/light wave & an observer along the line joining
them, the actual frequency observed is different from the frequency of the source. This phenomenon is called
Doppler’s Effect. If the observer and source are moving towards each other, the observed frequency is greater
than the frequency of the source. If the observer and source move away from each other, the observed
frequency is less than the frequency of source.
(v = velocity of sound wrt. ground. , c = velocity of sound with respect to medium, v m = velocity of medium,
v O = velocity of observer, vs = velocity of source.)

KVPY_ SA STREAM # 601


(a) Sound source is moving and observer is stationary :
If the source emitting a sound of frequency f is travelling with velocity v s along the line joining the
source and observer,

 v 
observed frequency, f =  v  v f
 ...(11.1)
 s 

 v  vs 
and Apparent wavelength  =    ...(11.2)
 v 
 In the above expression, the positive direction is taken along the velocity of sound, i.e. from source to
observer. Hence v s is positive if source is moving towards the observer, and negative if source is moving away
from the observer.
(b) Sound source is stationary and observer is moving with velocity v0 along the line joining
them :
The source (at rest) is emitting a sound of frequency ‘f’ travelling with velocity ‘v’ so that
wavelength is
 = v/f, i.e. there is no change in wavelength. How ever since the observer is moving with a
velocity v 0 along the line joining the source and observer, the observed frequency is
 v  v0 
f = f  v  ...(11.3)
 
 In the above expression, the positive direction is taken along the velocity of sound, i.e. from source to
observer. Hence v O is positive if observer is moving away from the source, and negative if observer is moving
towards the source.

(c) The source and observer both are moving with velocities vs and v0 along the line joining
them :
vv 
f = f  v  v 
0
The observed frequency, ...(11.4)
 s 

 v  vs 
and Apparent wavelength  =    ...(11.5)
 v 
 In the above expression also, the positive direction is taken along the velocity of sound, i.e. from source to
observer.
 In all of the above expression from equation 11.1 to 11.5, v stands for velocity of sound with respect to ground.
If velocity of sound with respect to medium is c and the medium is moving in the direction of sound from
source to observer with speed v m, v = c + v m , and if the medium is moving opposite to the direction
of sound from observer to source with speed v m, v = c – v m

Example 10.
A whistle of frequency 540 Hz is moving in a circle of radius 2 ft at a constant angular speed of 15 rad/s. What
are the lowest and height frequencies heard by a listener standing at rest, a long distance away from the
centre of the circle? (velocity of sound in air is 1100 ft/sec.)
Solution :
The whistle is moving along a circular path with constant angular velocity . The linear velocity of the whistle
is given by A
v S = R
O P
where, R is radius of the circle.
B

At points A and B, the velocity v S of whistle is parallel to line OP; i.e., with respect to observer at P, whistle
has maximum velocity v s away from P at point A, and towards P at point B. (Since distance OP is large
compared to radius R, whistle may be assumed to be moving along line OP).

KVPY_ SA STREAM # 602


Observer, therefore, listens maximum frequency when source is at B moving towards observer:
v
f max = f v  v
s
where, v is speed of sound in air. Similarly, observer listens minimum frequency when source is at A, moving
away from observer:
f
fmin 
v  vs
For f = 540 Hz, v s = 2 ft × 15 rad/s = 30 ft/s, and v = 1100 ft/s,
we get (approx.)
f max = 555 Hz
and, f min = 525 Hz.

Example 11.
The equation of a sound wave in air is given by
p = (0.02 ) sin [(3000 ) t – (9.0 ) x ], where all variables are in S.. units.
(a) Find the frequency, wavelength and the speed of sound wave in air.
(b) If the equilibrium pressure of air is 1.0 × 105 N/m2, what are the maximum and minimum pressures
at a point as the wave passes through that point?
Solution :
(a) Comparing with the standard form of a travelling wave
p = p0 sin [ (t – x/v)]
we see that  = 3000 s–1. The frequency is
 3000
f= = Hz
2 2
Also from the same comparison, /v = 9.0 m–1.

 3000 s 1 1000
or, v= 1 = 1 m/s–1
9 .0 m 9.0 m 3

v 1000 / 3m / s 2
The wavelength is  = =  m
f 3000 / 2 Hz 9
(b) The pressure amplitude is p0 = 0.02 N/m2. Hence, the maximum and minimum pressures at a point
in the wave motion will be (1.01 × 105 ± 0.02) N/m2.

Example 12.
A microphone of cross-sectional area 0.40 cm2 is placed in front of a small speaker emitting  W of sound
output. If the distance between the microphone and the speaker is 2.0 m, how much energy falls on the
microphone in 5.0 s ?
Solution :
The energy emitted by the speaker in one second is  J. Let us consider a sphere of radius 2.0 m centered
at the speaker. The energy J falls normally on the total surface of this sphere in one second. The energy
falling on the area 0.4 cm2 of the microphone in one second

0.4 cm2
= ×  J = 2.5 × 10–6J.
4  (2.0 m)2
The energy falling on the microphone in 5.0 is
2.5 × 10–6 J × 5 = 12.5 µJ.

KVPY_ SA STREAM # 603


Example 13.
Find the amplitude of vibration of the particles of air through which a sound wave of intensity
8.0 × 10–6 W/m2 and frequency 5.0 kHz is passing. Density of air = 1.2 kg/m3 and speed of sound in air = 330
m/s.
Solution :
The relation between the intensity of sound and the displacement amplitude is
2 s20B
= , where B = v 2 and  = 2
2v

  = 22 s0220v or, s02 = 2 2
2  0 v

8.0  10 6 W / m 2
= or, s0 = 6.4 nm.
22  (25.0  10 6 s 2 )  (1.2 kg / m3 )  (330 m / s)

Example 14.
The sound level at a point is increased by 40 dB. By what factor is the pressure amplitude increased ?
Solution :
The sound level in dB is
  
 = 10 log10   .
 
 0
If 1 and 2 are the sound levels and 1 and 2 are the intensities in the two cases,
  2   1 
2 – 1 = 10 log10     log10   
  0  0 

 2  2
or, 40 = 10 log10   or, 1
= 104.

 1
As the intensity is proportional to the square of the pressure amplitude,
p 02 2
we have
p 01
= 1 = 10000  100.

Example 15.
Figure shows a tube structure in which a sound signal is sent from one end and is received at the other end. The
semicircular part has a radius of 10.0 cm. The frequency of the sound source can be varied from 1 to 10 kHz.
Find the frequencies at which the ear perceives maximum intensity. The speed of sound in air = 342 m/s.

Solution :
The sound wave bifurcates at the junction of the straight and the semicircular parts. The wave through the
straight part travels a distance p1 = 2 × 10 cm and the wave through the curved part travels a distance p2 = 
10 cm = 31.4 cm before they meet again and travel to the receiver. The path difference between the two
waves received is, therefore.
p = p2 – p1 = 31.4 cm – 20 cm = 11.4 cm
v 330 m / s
The wavelength of either wave is = . For constructive interference, p= n, where n is an integer..
 
v n.v
or, p = n.  =
 p

n.342
 = 3000 n
(0.114)
Thus, the frequencies within the specified range which cause maximum of intensity are
3000 × 1, 3000 × 2 and 3000 × 3 Hz

KVPY_ SA STREAM # 604


Example 16.
A source emitting sound of frequency 165 Hz is placed in front of a wall at a distance of 2 m from it. A detector
is also placed in front of the wall at the same distance from it. Find the minimum distance between the source
and the detector for which the detector detects a maximum of sound. Speed of sound in air = 330 m/s.
Solution:
The situation is shown in figure. Suppose the detector is placed at
a distance of x meter from the source. The direct wave received
from the source travels a distance of x meter. The wave reaching the
detector after reflection from the wall has travelled a distance of
2[(2)2 + x2/4]1/2 metre. The path difference between the two waves is

  4
1/ 2 
 2 x 
 = 2 (2)    x  metre.
  4  
 
Constructive interference will take place when  = , 2, ...... The minimum distance x for a maximum
corresponds to
= ..........(i)
1/ 2
 330 m / s  2 x2 
The wavelength is  = = = 2 m. Thus, by (i) 2 ( 2)   –x=2
v 165 s 1  4 

1/ 2
 x2 
4  
x x2 x2
or, =1+ or, 4+ =1+ +x or, 3 = x.
 4  2 4 4
Thus, the detector should be placed at a distance of 3 m from the source. Note that there is no abrupt phase
change.

Example 17.
The range audible frequency for humans is 20 Hz to 20,000 Hz. If speed of sound in air is 336 m/s. What can
be the maximum and minimum length of a musical instrument, based on resonance pipe.
Solution :
v v
For an open pipe, f = n  = .n
2 2f
v
Similarly for a closed pipe, = (2n + 1)
4f
v 336 v v
min = (2n + 1)min = = 4.2 mm ; max = n = n = 8.4 (m) × nmax
4fmax 4  20000 2fmin max 2  20 max
clearly there is no upper limit on the length of such an musical instrument.

Example 18.
A closed organ pipe has length ‘ ‘. The air in it is vibrating in 3rd overtone with maximum amplitude ‘ a‘.
Find the amplitude at a distance of  /7 from closed end of the pipe.
Solution :
The figure shows variation of displacement of particles in a closed organ pipe for 3 rd overtone.

7 4  
For third overtone  = or  = or =
4 7 4 7


Hence the amplitude at P at a distance from closed end is ‘a’ because there is an antinode at that
7
point

KVPY_ SA STREAM # 605


Example 19.
A string 25 cm long fixed at both ends and having a mass of 2.5 g is under tension. A pipe closed from one
end is 40 cm long. When the string is set vibrating in its first overtone and the air in the pipe in its fundamental
frequency, 8 beats per second are heard. It is observed that decreasing the tension in the string decreases
the beat frequency. If the speed of sound in air is 320 m/s. Find tension in the string.
Solution :
2.5
 = 0.1 g / cm = 10–2 Kg/m
25
Ist overtone

1 T
 s = 25 cm = 0.25 m  fs =
s 
pipe in fundamental freq

V
 p = 160 cm = 1.6 m  fp = 
p

 by decreasing the tension , beat freq is decreased


1 T 320
 fs > fp  f s –f p = 8  2
 = 8  T = 27.04 N
0.25 10 1. 6
Example 20.
A whistle of frequency 540 Hz is moving in a circle of radius 2 ft at a constant angular speed of 15 rad/s. What
are the lowest and height frequencies heard by a listener standing at rest, a long distance away from the
centre of the circle? (velocity of sound in air is 1100 ft/sec.)
Solution :
The whistle is moving along a circular path with constant angular velocity . The linear velocity of the whistle
is given by A
v S = R
O P
where, R is radius of the circle.
B
At points A and B, the velocity v S of whistle is parallel to line OP; i.e., with respect to observer at P, whistle
has maximum velocity v s away from P at point A, and towards P at point B. (Since distance OP is large
compared to radius R, whistle may be assumed to be moving along line OP).
Observer, therefore, listens maximum frequency when source is at B moving towards observer:
v
f max = f v  v
s
where, v is speed of sound in air. Similarly, observer listens minimum frequency when source is at A, moving
away from observer:
f
fmin 
v  vs
For f = 540 Hz, v s = 2 ft × 15 rad/s = 30 ft/s, and v = 1100 ft/s,
we get (approx.)
f max = 555 Hz
and, f min = 525 Hz.

KVPY_ SA STREAM # 606


KVPY PROBLEMS (PREVIOUS YEARS)
1. In the figure shown a source of sound of frequency 510 Hz moves with constant velocity v s = 20 m/s in
the direction shown. The wind is blowing at a constant velocity v w = 20 m/s towards an observer who is
at rest at point B. The frequency detected by the observer corresponding to the sound emitted by the
source at initial position A, will be (speed of sound relative to air = 330 m/s)
[KVPY_2014_SB] [1 Mark]

(A) 485 Hz (B) 500 Hz (C) 512 Hz (D) 525 Hz

2. A whistle emitting a loud sound of frequency 540 Hz is whirled in a horizontal circle of radius 2m and at a
constant angular speed of 15rad/s. The speed of sound is 330 m/s. The ratio of the highest to the lowest
frequency heard by a listener standing at rest at a large distance from the center of the circle is :
[KVPY_2014_SB] [1 Mark]
(A) 1.0 (B) 1.1 (C) 1.2 (D) 1.4

3. A boy is standing on top of a tower of height 85m and throws a ball in the vertically upward direction
with a certain speed. If 5.25 seconds later he hears the ball hitting the ground, then the speed with
which the boy threw the ball is (take g = 10 m/s2 , speed of sound in air = 340 m/s)
[KVPY_2015_SB] [1 Mark]
[A] 6 m/s [B] 8 m/s [C] 10 m/s [D] 12 m/s

4. A hearing test is conducted on an aged person. It is found that her threshold of hearing is 20 decibels at 1
kHz and it rises linearly with frequency to 60 decibels at 9 kHz. The minimum intensity of sound that the
person can hear at 5 kHz is : [KVPY_2016_SB] [1 Mark]
(A) 10 times than that at 1 kHz (B) 100 times than that at 1 kHz
(C) 0.5 times than that at 9 kHz (D) 0.05 times than that at 9 kHz

5. A person in front of a mountain is beating a drum at the rate of 40 per minute and hears no distinct echo. If the
person moves 90 m closer to the mountain, he has to beat the drum at 60 per minute to not hear any distinct
echo. The speed of sound is [KVPY-SA_2018 2 Mark]
(A) 320 ms1 (B) 340 ms1 (C) 360 ms1 (D) 380 ms1

EXERCISE
1. (D) 2. (C) 3. (B) 4. (B) 5. (C)

KVPY_ SA STREAM # 607


ELAST I CI T Y & V I SCOSI T Y
ELASTICITY AN D PLASTICITY
The property of a material body by virtue of which it regains its original configuration (i.e. shape and size)
when the external deforming force is removed is called elasticity. The property of the material body by virtue
of which it does not regain its original configuration when the external force is removed is called plasticity.
Deforming force : An external force applied to a body which changes its size or shape or both is called
deforming force.
Perfectly Elastic body : A body is said to be perfectly elastic if it completely regains its original form when
the deforming force is removed. Since no material can regain completely its original form so the concept of
perfectly elastic body is only an ideal concept. A quartz fiber is the nearest approach to the perfectly elastic
body.
Perfectly Plastic body : A body is said to be perfectly plastic if it does not regain its original form even
slightly when the deforming force is removed. Since every material partially regain its original form on the
removal of deforming force, so the concept of perfectly plastic body is also only an ideal concept. Paraffin
wax, wet clay are the nearest approach to a perfectly plastic bodies.
Cause of Elasticity : In a solid, atoms and molecules are arranged in such a way that each molecule is
acted upon by the forces due to the neighboring molecules. These forces are known as intermolecular
forces. When no deforming force is applied on the body, each molecule of the solid (i.e. body) is in its
equilibrium position and the inter molecular forces between the molecules of the solid are minimum.
On applying the deforming force on the body, the molecules either come closer or go far apart from each
other. As a result of this, the molecules are displaced from their equilibrium position. In other words,
intermolecular forces get changed and restoring forces are developed on the molecules. When the deforming
force is removed, these restoring forces bring the molecules of the solid to their respective equilibrium
positions and hence the solid (or the body) regains its original form.

STRESS
When deforming force is applied on the body then the equal restoring force in opposite direction is developed
inside the body. The restoring forces per unit area of the body is called stress.
restoring force F
stress = Area of the body  A

The unit of stress is N/m2. There are three types of stress

1. Longitudinal or Normal stress


When object is one dimensional then force acting per unit area is called longitudinal stress.
It is of two types : (a) compressive stress (b) tensile stress

Examples :
(i) Consider a block of solid as shown in figure. Let a force F be applied to the face which has

area A. Resolve F into two components :
Fn = F sin  called normal force and Ft = F cos  called tangential force.

Fn F sin 
 Normal (tensile) stress = =
A A

KVPY_ SA STREAM # 608


2. Tangential or shear stress
It is defined as the restoring force acting per unit area tangential to the surface of the body. Refer to shown in
figure above.
Ft F cos 
Tangential (shear) stress = =
A A
The effect of stress is to produce distortion or a change in size, volume and shape
(i.e. configuration of the body).

3. Bulk stress : When force is acting all along the surface normal to the area, then force acting per unit area is
known as pressure. The effect of pressure is to produce volume change. The shape of the body may or may
not change depending upon the homogeneity of body.

STR AIN
The ratio of the change in configuration (i.e. shape, length or volume) to the original configuration of the body
is called strain,
change in configuration
i.e. Strain,  =
original configuration
It has no unit
Types of strain : There are three types of strain
(i) Longitudinal strain : This type of strain is produced when the deforming force causes a change in length
of the body. It is defined as the ratio of the change in length to the original length of the body.
Consider a wire of length L : When the wire is stretched by a force F, then let the change in length of the wire
is L.
change in length L
 Longitudinal strain ,  = or Longitudinal strain =
original length L
(ii) Volume strain : This type of strain is produced when the deforming force
produces a change in volume of the body as shown in the figure. It is defined as the
ratio of the change in volume to the original volume of the body.

If V = change in volume V = original volume


V
v = volume strain =
V
(iii) Shear Strain : This type of strain is produced when the deforming force causes a change in the shape
of the body. It is defined as the angle  through which a face originally perpendicular to the fixed face is turned
as shown in the figure.

x
tan  or  =

HOOKE’S LAW AND M OD ULUS OF ELASTICITY


According to this law, within the elastic limit, stress is proportional to the strain.
i.e. stress  strain
stress
or stress = constant × strain or = Modulus of Elasticity..
strain
This constant is called modulus of elasticity.
Thus, modulus of elasticity is defined as the ratio of the stress to the strain.
Modulus of elasticity depends on the nature of the material of the body and is independent of its dimensions
(i.e. length, volume etc.).
Unit : The Sl unit of modulus of elasticity is Nm–2 or Pascal (Pa).

KVPY_ SA STREAM # 609


TYPES OF M ODULUS OF ELASTICITY
Corresponding to the three types of strain there are three types of modulus of elasticity.
1. Young's modulus of elasticity (Y) 2. Bulk modulus of elasticity (K)
3. Modulus of rigidity ().

1. Young's modulus of elasticity


It is defined as the ratio of the normal stress to the longitudinal strain.
Longitudin al stress
i.e. Young's modulus (Y) =
Longitudin al strain
Normal stress = F/A,
Longitudinal strain = L/L
F/ A FL
Y= 
L / L AL

ELONGATION OF ROD UNDER IT’S SELF W EIGHT


Let rod is having self weight ‘W’, area of cross-section ‘A” and length ‘L’. Considering on element at a
distance ‘x’ from bottom.
W
then T  x
L
Tdx
elongation in ‘dx’ element =
AY
L L
Tdx Wx dx WL
Total elongation s  
0
AY
 
0
LAY

2AY

Note : One can do directly by considering total weight at C.M. and using effective length /2.

Example 1.
Find out the elongation in block. If mass, area of cross-section and young
modulus of block are m, A and y respectively.

Sol.

F m
Acceleration, a= then T = ma where  m = x
m 
m F Fx
T= x =
 m 
Tdx
Elongation in element ‘dx’ = Ay

 
Tdx Fxdx F
total elongation,  = 
o
Ay d =  A y
o
= 2Ay

Note : - Try this problem, if friction is given between block and surface (µ = friction coefficient), and
Case : () F < µmg () F > µmg
F
Ans. In both cases answer will be
2Ay

KVPY_ SA STREAM # 610


2. Bulk modulus :
It is defined as the ratio of the normal stress to the volume strain
Pr essure
i.e. B=
Volume strain
The stress being the normal force applied per unit area and is equal to the pressure applied (p).
p pV
B= 
 V V
V
Negative sign shows that increase in pressure (p) causes decrease in volume (V).
Compressibility : The reciprocal of bulk modulus of elasticity is called compressibility. Unit of compressibility
in Sl is N-1 m2 or pascal-1 (Pa-1).
Bulk modulus of solids is about fifty times that of liquids, and for gases it is 10–8 times of solids.
Bsolids > Bliquids > Bgases
Isothermal bulk modulus of elasticity of gas B = P (pressure of gas)
Cp
Adiabatic bulk modulus of elasticity of gas B =  × P where  = .
Cv

Example 2.
Find the depth of lake at which density of water is 1% greater than at the surface. Given compressibily k =
50 × 10–6 /atm.
p V p
Sol. B= =–
V V B

V
We know p = patm + hg
or m = V = const.
d. v + dv.  = 0
d dV
d V + dV .  = 0  =–
 V
 p  1
i.e. =  =
 B  100
1 hg
= [assuming  = const.]
100 B
B 1 1 1 10 5
hg= = 100 k  hg=
100 100  50  10 6
5
10 100  10 3
h= 6 = = 2km Ans.
5000  10  1000  10 50

3. Modulus of Rigidity :
It is defined as the ratio of the tangential stress to the shear strain. Let us
consider a cube whose lower face is fixed and a tangential force F acts on the
upper face whose area is A.
 Tangential stress = F/A.
Let the vertical sides of the cube shifts through an angle , called shear strain
 Modulus of rigidity is given by
Tangential stress F/ A F
 = or =  = A
Shear strain

KVPY_ SA STREAM # 611


Example 3.
A rubber cube of side 5 cm has one side fixed while a tangential force equal to 1800 N is applied to opposite
face find the shearing strain and the lateral displacement of the strained face. Modulus of rigidity for rubber is
2.4 × 106 N/m2.
F x
Sol. L = 5 × 10–2 m  
A L
F 1800 180 3
strain  = = =  = 0.3 radian
A 25  10 4
 2.4  10 6 25  24 10

x
= 0.3  x = 0.3 × 5 × 10–2 = 1.5 × 10–2 m = 1.5 mm Ans.
L
VARIATION OF STRAIN W ITH STRESS
When a wire is stretched by a load, it is seen that for small value of load, the extension produced in the wire
is proportional to the load. On removing the load, the wire returns to its original length. The wire regains its
original dimensions only when load applied is less or equal to a certain limit. This limit is called elastic limit.
Thus, elastic limit is the maximum stress on whose removal, the bodies regain their original dimensions. In
shown figure, this type of behavior is represented by OB portion of the graph. Till A the stress is proportional
to strain and from A to B if deforming forces are removed then the wire comes to its original length but here
stress is not proportional to strain.

C D
B E OA  Limit of Proportionality
OB  Elastic limit
A
C  Yield Point
Strees

CD  Plastic behaviour
D  Ultimate point
O Strain DE  Fracture
As we go beyond the point B, then even for a very small increase in stress, the strain produced is very large.
This type of behavior is observed around point C and at this stage the wire begins to flow like a viscous fluid.
The point C is called yield point. If the stress is further increased, then the wire breaks off at a point D called
the breaking point. The stress corresponding to this point is called breaking stress or tensile strength of the
material of the wire. A material for which the plastic range CD is relatively high is called ductile material.
These materials get permanently deformed before breaking. The materials for which plastic range is relatively
small are called brittle materials. These materials break as soon as elastic limit is crossed.
Important points
 Breaking stress = Breaking force/area of cross section.
 Breaking stress is constant for a material.
 Breaking force depends upon the area of the section of the wire of a given material.
 The working stress is always kept lower than that of a breaking stress so that safety factor =
breaking stress/working stress may have a large value.
 Breaking strain = elongation or compression/original dimension.
 Breaking strain is constant for a material.
Elastic after effect
We know that some material bodies take some time to regain their original configuration when the deforming
force is removed. The delay in regaining the original configuration by the bodies on the removal of deforming
force is called elastic after effect. The elastic after effect is negligibly small for quartz fiber and phosphor
bronze. For this reason, the suspensions made from quartz and phosphor-bronze are used in galvanometers
and electrometers.
For glass fiber elastic after effect is very large. It takes hours for glass fiber to return to its original state on
removal of deforming force.

KVPY_ SA STREAM # 612


Elastic Fatigue
The, the loss of strength of the material due to repeated strains on the material is called elastic fatigue. That
is why bridges are declared unsafe after a longtime of their use.
Analogy of Rod as a spring
stress F
Y=  Y=
strain A 
AY
or F= 

AY
= constant, depends on type of material and geometry of rod. F = k

AY
where k = = equivalent spring constant.

for the system of rods shown in figure (a), the replaced spring system is shown in figure (b) two spring in
series]. Figure (c) represents equivalent spring system.
Figure (d) represents another combination of rods and their replaced spring system.

ELASTIC POTENTIAL ENERGY STORED IN A STRETCHED W IRE OR IN A ROD

Strain energy stored in equivalent spring


1 2
U= kx
2
F AY 1 AY F 2  2 1 F2
where x= , k= U= = .
AY  2  A 2Y2 2 AY
equation can be re-arranged
1 F2 A
U= × [A = volume of rod, F/A = stress]
2 A2 Y

1 (stress )2
U= × volume
2 Y
1 F F F 1
again, U= × ×A [ Strain = ] U= stress × strain × volume
2 A AY AY 2
1 F2 1
again, U= AY  U= Y (strain)2 × volume
2 A 2Y2 2

strain energy 1 (stress )2 1 1


strain energy density = =  Y(strain)2 = stress × strain
volume 2 Y 2 2
KVPY_ SA STREAM # 613
OTHERWAY BY S.H.M .

k eq
= v =  a2  y2
m keq

k eq 2mgh m 2 g2
2gh = a –y 2 2   2 =a
m k eq k eq mg 2gh
y =k
eq
equilibrium
position
mg m 2 g2 2mgh a
maxm extension =a+y= k + 
eq k eq k eq
THERM AL STRESS :

If temp of rod is increased by T, then change in length


 =  T

strain =   T

But due to rigid support, there is no strain. Supports provide force on stresses to keep the length of rod same

stress
Y= If T = positive
strain
thermal stress = Y strain = Y T

If T = negative

F
 YT F = AY  T
A

Example 4.
When composite rod is free, then composite length increases to 2.002 m
for temp 20ºC to 120ºC. When composite rod is fixed between the support,
there is no change in component length find y and  of steel,
if ycu = 1.5 × 1013 N/m2 cu = 1.6 × 10–5/ºC.
Sol.  = s s T + c cT
.002 = [1.5 s + 0.5 × 1.6 ×10–5] × 100
1.2  10 5
s = = 8 × 10–6/ºC
1 .5
there is no change in component length
For steel
F s
x = s s T – AY  0
s

F
  s T ....(A)
AYs
for copper
F c
x = Ay – c c T = 0
c

KVPY_ SA STREAM # 614


F
Ay c
=  T ...(B)

ys c
B/A  yc
= 
s

c 1.5  1013  16  10 5
ys = y c s
= = 3×1013 N/m2
8  10 6

APPLICATIONS OF ELASTICITY
Some of the important applications of the elasticity of the materials are discussed as follows :
1. The material used in bridges lose its elastic strength with time bridges are declared unsafe after long use.

2. To estimate the maximum height of a mountain :


The pressure at the base of the mountain = hg = stress. The elastic limit of a typical rock is
3 × 108 N m–2
The stress must be less than the elastic limits, otherwise the rock begins to flow.
3  10 8
h<  h < 104 m ( = 3 × 103 kg m–3 ; g = 10 ms–2) or h = 10 km
g
It may be noted that the height of Mount Everest is nearly 9 km.
TORSION CONSTANT OF A W IRE
r 4
C= Where  is modulus of rigidity r and  is radius and length of wire respectively..
2
(a) Toque required for twisting by an angle  = C
1
(b) Work done in twisting by an angle , W = C2.
2
VI SCOSI TY
When a solid body slides over another solid body, a frictional-force begins to act between them. This force
opposes the relative motion of the bodies. Similarly, when a layer of a liquid slides over another layer of the
same liquid, a frictional-force acts between them which opposes the relative motion between the layers. This
force is called 'internal frictional-force'.
Suppose a liquid is flowing in streamlined motion on a fixed horizontal surface AB (Fig.). The layer of the
liquid which is in contact with the surface is at rest, while the velocity of other layers increases with distance
from the fixed surface. In the Fig., the lengths of the arrows represent the increasing velocity of the layers.
Thus there is a relative motion between adjacent layers of the liquid. Let us consider three parallel layers a,
b and c. Their velocities are in the increasing order. The layer a tends to retard the layer b, while b tends to
retard c. Thus each layer tends to decrease the velocity of the layer above it. Similarly, each layer tends to
increase the velocity of the layer below it. This means that in between any two layers of the liquid, internal
tangential forces act which try to destroy the relative motion between the layers. These forces are called
'viscous forces'. If the flow of the liquid is to be maintained, an external force must be applied to overcome the
dragging viscous forces. In the absence of the external force, the viscous forces would soon bring the liquid
to rest. The property of the liquid by virtue of which it opposes the relative motion between its
adjacent layers is known as 'viscosity’.
The property of viscosity is seen in the following examples :

(i) A stirred liquid, when left, comes to rest on account of viscosity. Thicker liquids like honey, coaltar,
glycerin, etc. have a larger viscosity than thinner ones like water. If we pour coaltar and water on a
table, the coaltar will stop soon while the water will flow upto quite a large distance.

KVPY_ SA STREAM # 615


(ii) If we pour water and honey in separate funnels, water comes out readily from the hole in the funnel
while honey takes enough time to do so. This is because honey is much more viscous than water.
As honey tends to flow down under gravity, the relative motion between its layers is opposed strongly.
(iii) We can walk fast in air, but not in water. The reason is again viscosity which is very small for air but
comparatively much larger for water.
(iv) The cloud particles fall down very slowly because of the viscosity of air and hence appear floating in
the sky.
Viscosity comes into play only when there is a relative motion between the layers of the same
material. This is why it does not act in solids.

FLOW OF LIQUID IN A TUBE: CRITICAL VELOCITY


When a liquid flows in a tube, the viscous forces oppose the flow of the liquid, Hence a pressure difference is
applied between the ends of the tube which maintains the flow of the liquid. If all particles of the liquid passing
through a particular point in the tube move along the same path, the flow of the liquid is called 'stream-lined
flow'. This occurs only when the velocity of flow of the liquid is below a certain limiting value called 'critical
velocity'. When the velocity of flow exceeds the critical velocity, the flow is no longer stream-lined but be-
comes turbulent. In this type of flow, the motion of the liquid becomes zig-zag and eddy-currents are devel-
oped in it.

Reynold’s proved that the critical velocity for a liquid flowing in a tube is v c = k/r. where  is density and 
is viscosity of the liquid, r is radius of the tube and k is 'Reynold's number' (whose value for a narrow tube and
for water is about 1000). When the velocity of flow of the liquid is less than the critical velocity, then the flow
of the liquid is controlled by the viscosity, the density having no effect on it. But when the velocity of flow is
larger than the critical velocity, then the flow is mainly governed by the density, the effect of viscosity becom-
ing less important. It is because of this reason that when a volcano erupts, then the lava coming out of it flows
speedily inspite of being very thick (of large viscosity).

VELOCITY GRAD IEN T AN D COEFFICIENT OF VISCOSITY


The property of a liquid by virtue of which an opposing force (internal friction) comes into play when ever there
is a relative motion between the different layers of the liquid is called viscosity. Consider a flow of a liquid over
the horizontal solid surface as shown in fig. Let us consider two layers AB and CD moving with velocities
  
v and v + d v at a distance x and (x + dx) respectively from the fixed solid surface.
According to Newton, the viscous drag or back ward force (F) between these layers depends.
(i) directly proportional to the area (A) of the layer and (ii) directly proportional to the velocity gradient

 dv 
  between the layers.
 dx 

dv dv
i.e. F A or F = – A ..........(1)
dx dx

 is called Coefficient of viscosity. Negative sign shows


that the direction of viscous drag (F) is just opposite to
the direction of the motion of the liquid.

KVPY_ SA STREAM # 616


SIM ILARITIES AND DIFFERENCES BETW EEN VISCOSITY AND SOLID FRICTION
Sim ila rit ie s
Viscosity and solid friction are similar as
1. Both oppose relative motion. Whereas viscosity opposes the relative motion between two adjacent liquid
layers, solid friction opposes the relative motion between two solid layers.
2. Both come into play, whenever there is relative motion between layers of liquid or solid surfaces as the case
may be.
3. Both are due to molecular attractions.
Differences between them 
–––––––––––––––––––––––––––––––––––––––––––––––––––––––––––––––––––––––––––––––––––––––––––––––
Viscosity Solid Friction
–––––––––––––––––––––––––––––––––––––––––––––––––––––––––––––––––––––––––––––––––––––––––––––––
(i) Viscosity (or viscous drag) between layers of (i) Friction between two solids is independent of
liquid is directly proportional to the area of the area of solid surfaces in contact.
the liquid layers.
(ii) Viscous drag is proportional to the relative (ii) Friction is independent of the relative
velocity between two layers of liquid. velocity between two surfaces.
(iii) Viscous drag is independent of normal (iii)
Friction is directly proportional to the
reaction between two layers of liquid. normal reaction between two surfaces in
contact.
–––––––––––––––––––––––––––––––––––––––––––––––––––––––––––––––––––––––––––––––––––––––––––––––
SOM E APPLICATION S OF VISCOSITY
Knowledge of viscosity of various liquids and gases have been put to use in daily life. Some applications of
its knowledge are discussed as under 
1. As the viscosity of liquids vary with temperature, proper choice of lubricant is made depending upon season.

2. Liquids of high viscosity are used in shock absorbers and buffers at railway stations.
3. The phenomenon of viscosity of air and liquid is used to damp the motion of some instruments.

4. The knowledge of the coefficient of viscosity of organic liquids is used in determining the molecular weight
and shape of the organic molecules.

5. It finds an important use in the circulation of blood through arteries and veins of human body.

UNITS OF COEFFICIENT OF VISCOSITY


F
From the above formula, we have  
A( v x / z )
[MLT 2 ] [MLT 2 ]
 dimensions of  =   [ML1T 1 ]
[L2 ][LT 1 / L] [L2 T 1 ]
Its unit is kg/(meter-second)*
In C.G.S. system, the unit of coefficient of viscosity is dyne s cm –2 and is called poise. In SI the unit of
coefficient of viscosity is N sm–2 and is called decapoise.
1 decapoise = 1 N sm–2 = (105 dyne) × s × (102 cm)–2 = 10 dyne s cm–2 = 10 poise

Example 5.
A cubical block (of side 2m) of mass20 kg slides on inclined plane lubricated with
the oil of viscosity  = 10–1 poise with constant velocity of 10 m/sec. (g = 10 m/
sec2)
find out the thickness of layer of liquid.
dv dv v
Sol. F = F =  A = mg sin  =
dz dz h

KVPY_ SA STREAM # 617


10
20 × 10 × sin 30° =  × 4 ×
h

40  10 2
h= – [ = 10–1 poise = 10–2 N-sec-m–2 ]
100
= 4 × 10–3 m = 4 mm

EFFECT OF TEM PERATURE ON THE VISCOSITY


The viscosity of liquids decrease with increase in temperature and increase with the decrease in temperature.

1
That is,  . On the other hand, the value of viscosity of gases increases with the increase in temperature
T

and vice-versa. That is,  T .

STOKE’S LAW
Stokes proved that the viscous drag (F) on a spherical body of radius r moving with velocity v in a fluid of
viscosity  is given by F = 6 r v. This is called Stokes’ law.

TERM I N AL VELOCITY
When a body is dropped in a viscous fluid, it is first accelerated and then its acceleration becomes zero and
it attains a constant velocity called terminal velocity.

Calculation of Terminal Velocity


Let us consider a small ball, whose radius is r and density is , falling freely in a liquid (or gas), whose
density is  and coefficient of viscosity  . When it attains a terminal velocity v. It is subjected to two forces
: 6rv
(i) effective force acting downward
v
4 3
= V (–) g = r ( – )g,
3
4/3r3 (–) g

(ii) viscous force acting upward


= 6   rv..
Since the ball is moving with a constant velocity v i.e., there is no acceleration in it, the net force acting on
it must be zero. That is

4
6  rv = p r3 ( – ) g
3

2 r 2 (  )g
or v=
9 
Thus, terminal velocity of the ball is directly proportional to the square of its radius
Important point
Air bubble in water always goes up. It is because density of air () is less than the density of water (). So
the terminal velocity for air bubble is Negative, which implies that the air bubble will go up. Positive terminal
velocity means the body will fall down.

KVPY_ SA STREAM # 618


Example 6.
A metallic sphere of radius 1.0 × 10–3 m and density 1.0 × 104 kg/m3 enters a tank of water, after a free fall
through a distance of h in the earth’s gravitational field. If its velocity remains unchanged after entering water,
determine the value of h. Given : coefficient of viscosity of water = 1.0 × 10–3 N-s/m2, g = 10 m/s2 and density
of water = 1.0 × 103 kg/m3.
Sol. The velocity attained by the sphere in falling freely from a height h is
= 2 gh ....(i)
This is the terminal velocity of the sphere in water. Hence by Stoke’s law, we have
2 r 2 (   ) g
=
9 
where r is the radius of the sphere,  is the density of the material of the sphere
 (= 1.0 × 103 kg/m3) is the density of water and  is coefficient of viscosity of water.
2  (1.0  10 3 )2 (1.0  10 4  1.0  10 3 )  10
 = = 20 m/s
9  1.0  10 3
from equation (i), we have
 2 20  20
h=  = 20 m
2g 2  10

Applications of Stokes' Formula


(i) In determining the Electronic Charge by Millikan's Experiment : Stokes' formula is used in
Millikan's method for determining the electronic charge. In this method the formula is applied for
finding out the radii of small oil-drops by measuring their terminal velocity in air.

(ii) Velocity of Rain Drops : Rain drops are formed by the condensation of water vapour on dust
particles. When they fall under gravity, their motion is opposed by the viscous drag in air. As the
velocity of their fall increases, the viscous drag also increases and finally becomes equal to the
effective force of gravity. The drops then attain a (constant) terminal velocity which is directly propor-
tional to the square of the radius of the drops. In the beginning the raindrops are very small in size
and so they fall with such a small velocity that they appear floating in the sky as cloud. As they grow
in size by further condensation, then they reach the earth with appreciable velocity,

(iii) Parachute : When a soldier with a parachute jumps from a flying


aeroplane, he descends very slowly in air.

In the beginning the soldier falls with gravity acceleration g, but soon the acceleration goes on decreasing
rapidly until in parachute is fully opened. Therefore, in the beginning the speed of the falling soldier increases
somewhat rapidly but then very slowly. Due to the viscosity of air the acceleration of the soldier becomes
ultimately zero and the soldier then falls with a constant terminal speed. In Fig graph is shown between the
speed of the falling soldier and time.

KVPY_ SA STREAM # 619


KVPY PROBLEMS (PREVIOUS YEARS)
1. One end of a slack wire (Young’s modulus Y, length L and cross-sectional area A) is clamped to a rigid
wall and the other end to a block (mass m) which rests on a smooth horizontal plane. The block is set
in motion with a speed v. What is the maximum distance the block will travel after the wire becomes
taut? [KVPY 2015 SB] [1 Mark]

mL 2mL mL mv
(A) v (B) v (C) v (D) L
AY AY 2 AY AY

2. A horizontal steel railroad track has a length of 100 m when the temperature is 25ºC. The track is
constrained from expanding or bending. The stress on the track on a hot summer day, when the
temperature is 40ºC, is (Note: The linear coefficient of thermal expansion for steel is 1.1 × 10 -5/ºC and
the Young’s modulus of steel is 2 × 10 11 Pa) [KVPY 2015 SB] [2 Mark]
(A) 6.6  10 7 Pa (B) 8.8  10 Pa (C) 3.3  107 Pa (D) 5.5 10 7 Pa
7

3. Electrons accelerated from rest by an electrostatic potential are collimated and sent through a Young’s
double slit setup. The fringe width is w. If the accelerating is doubled then the width is now close to :
[KVPY_2016_SB] [1 Mark]
(A) 0.5 w (B) 0.7 w (C) 1.0 w (D) 2.0 w

1. (A) 2. (C) 3. (B)

KVPY_ SA STREAM # 620


SU RFACE T EN SI ON

EXPLANATION OF SOME OBSERVED PHENOMENA
1. Lead balls are spherical in shape.
2. Rain drops and a globule of mercury placed on glass plate are spherical.
3. Hair of a shaving brush/painting brush, when dipped in water spread out, but as soon as it is taken out.
Its hair stick together.
4. A greased needle placed gently on the free surface of water in a beaker does not sink.
5. Similarly, insects can walk on the free surface of water without drowning.
6. Bits of Camphor gum move irregularly when placed on water surface.

SURFACE TENSION
Surface Tension is a property of liquid at rest by virtue of which a liquid surface gets contracted to a
minimum area and behaves like a stretched membrane.
Surface Tension of a liquid is measured by force per unit length on either
side of any imaginary line drawn tangentially over the liquid surface, force
being normal to the imaginary line as shown in the figure.
i.e. Surface tension

Total force on either of the imginary line (F)


(T) =
Length of the line ()
Units of Surface Tension.
In C.G.S. system the unit of surface tension is dyne/cm (dyne cm -1) and Sl system its units is Nm -1

Ex. 1 A ring is cut form a platinum tube of 8.5 cm internal and 8.7 cm external diameter. It is supported horizontally
from a pan of a balance so that it comes in contact with the water in a glass vessel. What is the surface
tension of water if an extra 3.97 g weight is required to pull it away from water? (g = 980 cm/s2).
Sol. F F

T T
Cross
section

The ring is in contact with water along its inner and outer circumference; so when pulled out the total force on
it due to surface tension will be
F = T (2 r1 + 2 r2)
mg 3.97  980
So, T = 2(r  r ) [ F = mg] i.e., T= = 72.13 dyne/cm
1 2 3.14  (8.5  8.7)

KVPY_ SA STREAM # 621


EXCESS PRESSURE INSIDE A LIQUID DROP AND A BUBBLE
1. Inside a bubble : Consider a soap bubble of radius r.
Let p be the pressure inside the bubble and pa outside.
The excess pressure = p – pa. Imagine the bubble
broken into two halves, and consider one half of it as
shown in Fig. Since there are two surfaces, inner and
outer, so the force due to surface tension is
F = surface tension x length = T x 2 (circumference of the bubble) = T x 2 (2 T r) ... (1)
The excess pressure (p - pa) acts on a cross-sectional area  r2, so the force due to excess
pressure is  F = (p – pa) r2 .......... (2)
The surface tension force given by equation (1) must balance the force due to excess pressure
given by equation (2) to maintain the equilibrium. i.e. (p – pa) r2 = T × 2 (2 r)
4T
or (p – pa) == pexcess
r
above expression can also be obtained by equation of excess pressure of curve surface by
putting R1 = R2.
2. Inside the drop : In a drop, there is only one surface and hence excess pressure can be
written as
2T
(p – pa) = = pexcess
r
2T
3. Inside air bubble in a liquid : (p – pa) = = pexcess
r

4. A charged bubble : If bubble is charged, it's radius increases. Bubble has pressure excess
due to charge too. Initially pressure inside the bubble
4T
= pa + r
1

4T 2
For charge bubble, pressure inside = pa + r – , where  surface is surface charge density..
2 2 0
Taking temperature remains constant, then from Boyle's law

 4T  4  4T 2  4
 pa   p   
r1  3 r1 =  r2 2 0  3 r2
3 a 3

From above expression the radius of charged drop may be calculated. It can conclude that radius of
charged bubble increases, i.e. r 2 > r1

KVPY_ SA STREAM # 622


THE FORCE OF COHESION
The force of attraction between the molecules of the same substance is called cohesion.
In case of solids, the force of cohesion is very large and due to this solids have definite shape and size.
On the other hand, the force of cohesion in case of liquids is weaker than that of solids. Hence liquids
do not have definite shape but have definite volume. The force of cohesion is negligible in case of
gases. Because of this fact, gases have neither fixed shape nor volume.
Examples.
(i) Two drops of a liquid coalesce into one when brought in mutual contact because of the cohesive
force.
(ii) It is difficult to separate two sticky plates of glass wetted with water because a large force has
to be applied against the cohesive force between the molecules of water.
(iii) It is very difficult to break a drop of mercury into small droplets because of large cohesive force
between mercury molecules.

FORCE OF ADHESION
The force of attraction between molecules of different substances is called adhesion.
Examples.
(i) Adhesive force enables us to write on the black board with a chalk.
(ii) Adhesive force helps us to write on the paper with ink.
(iii) Large force of adhesion between cement and bricks helps us in construction work.
(iv) Due to force of adhesive, water wets the glass plate.
(v) Fevicol and gum are used in gluing two surfaces together because of adhesive force.

ANGLE OF CONTACT
The angle which the tangent to the liquid surface at the point of contact makes with the solid surface
inside the liquid is called angle of contact. Those liquids which wet the walls of the container (say in
case of water and glass) have meniscus concave upwards and their value of angle of contact is less
than 90° (also called acute angle). However, those
liquids which don't wet the walls of the container
(say in case of mercury and glass) have meniscus
convex upwards and their value of angle of contact
is greater than 90° (also called obtuse angle). The
angle of contact of mercury with glass about 140°,
whereas the angle of contact of water with glass is
about 8°. But, for pure water, the angle of contact
 with glass is taken as 0°.

SHAPE OF LIQUID MENISCUS


When a capillary tube or a tube is dipped in a liquid, the liquid surface becomes curved near the point
of contact. This curved surface is due to the two forces i.e.
(i) due to the force of cohesion and
(ii) due to the force of adhesion. The curved surface of the liquid is called meniscus of the liquid.
Various forces acting on molecule A are:
(iii) Force F 1 due to force of adhesion which acts outwards at right angle to the wall of the tube.
This force is represented by AB.
(iv) Force F 2 due to force of cohesion which acts at an angle of 45° to the vertical. This force is
represented by AD.
(v) The weight of the molecule A which acts vertically downward along the wall of the tube.

KVPY_ SA STREAM # 623


Since the weight of the molecule is negligible as compared to F 1 and F 2 and hence can be
neglected. Thus, there are only two forces (F 1 and F 2) acting on the liquid molecules. These forces are
inclined at an angle of 135°.

(a) (b) (c)


The resultant force represented by AC will depend upon the values of F 1 and F 2. Let the resultant force
makes an angle  with F 1.
F2 sin135 º F2 / 2 F2
According to parallelogram law of vectors tan  = F  F cos135 º = =
1 2 F1  F2 / 2 2 F1  F2
Special cases :
(i) If F 2 = 2 F 1, then tan  =  90°
Then the resultant force will act vertically downward and hence the meniscus will be plane or
horizontal shown in figure (a). Example; pure water contained in silver capillary tube.
(ii) If F 2 < 2 F 1, then tan  is positive   is acute angle
Thus, the resultant will be directed outside the liquid and hence the meniscus will be concave
upward shown in figure (b). This is possible in case of liquids which wet the walls of the
capillary tube. Example ; water in glass capillary tube.
(iii) If F 2 > 2 F 1, then tan  is negative   is obtuse angle.
Thus, the resultant will be directed inside the liquid and hence the meniscus will be convex
upward shown in figure (c). This is possible in case of liquids which do not wet the walls of the
capillary tube. Example ; mercury in glass capillary tube.

RELATION BETWEEN SURFACE TENSION, RADII OF CURVATURE AND EXCESS


PRESSURE ON A CURVED SURFACE.

Let us consider a small element ABCD (fig.) of a curved


liquid surface which is convex on the upper side. R 1
and R2 are the maximum and minimum radii of curvature
respectively, They are called the 'principal radii of
curvature' of the surface. Let p be the excess pressure
on the concave side.
 1 1 
then p = T  R  R  . If instead of a liquid surface,
 1 2

we have a liquid film, the above expression will be


 1 1 
p = 2T  R  R  , because a film has two surface.
 1 2

EXCESS OF PRESSURE INSIDE A CURVED SURFACE


1. Plane Surface : If the surface of the liquid is plane [as shown in Fig.(a)], the molecule on the
liquid surface is attracted equally in all directions. The resultant force due to surface tension is
zero. The pressure, therefore, on the liquid surface is normal.

2. Concave Surface : If the surface is concave upwards [as shown in Fig.(b)], there will be
upward resultant force due to surface tension acting on the molecule. Since the molecule on
the surface is in equilibrium, there must be an excess of pressure on the concave side in the
2T
downward direction to balance the resultant force of surface tension p A – pB = .
r
KVPY_ SA STREAM # 624
3. Convex Surface : If the surface is convex [as shown in Fig.(c)], the resultant force due to
surface tension acts in the downward direction. Since the molecule on the surface are in equilibrium,
there must be an excess of pressure on the concave side of the surface acting in the upward
direction to balance the downward resultant force of surface tension, Hence there is always an
excess of pressure on concave side of a curved surface over that on the convex side.
2T
pB – pA =
r
CAPILLARITY
A glass tube of very fine bore throughout the length of the tube is called capillary tube. If the capillary
tube is dipped in water, the water wets the inner side of the tube and rises in it [shown in figure (a)]. If
the same capillary tube is dipped in the mercury, then the mercury is depressed [shown in figure (b)].
The phenomenon of rise or fall of liquids in a capillary tube is called capillarity.

PRACTICAL APPLICATIONS OF CAPILLARITY


1. The oil in a lamp rises in the wick by capillary action.
2. The tip of nib of a pen is split up, to make a narrow capillary so that the ink rises upto the tin or nib
continuously.
3. Sap and water rise upto the top of the leaves of the tree by capillary action.
4. If one end of the towel dips into a bucket of water and the Other end hangs over the bucket the towel
soon becomes wet throughout due to capillary action.
5. Ink is absorbed by the blotter due to capillary action.
6. Sandy soil is more dry than clay. It is because the capillaries between sand particles are not so fine as
to draw the water up by capillaries.
7. The moisture rises in the capillaries of soil to the surface, where it evaporates. To preserve the moisture
m the soil, capillaries must be. broken up. This is done by ploughing and leveling the fields
8. Bricks are porous and behave like capillaries.

CAPILLARY RISE (HEIGHT OF A LIQUID IN A CAPILLARY TUBE)


ASCENT FORMULA
Consider the liquid which wets the walls of the tube, forms a concave meniscus
shown in figure. Consider a capillary tube of radius r dipped in a liquid of surface
tension T and density p. Let h be the height through which the liquid rises in
the tube. Let p be the pressure on the concave side of the meniscus and pa be
the pressure on the convex side of the meniscus. The excess pressure
2T
(p – pa) is given by (p – pa) =
R

KVPY_ SA STREAM # 625


Where R is the radius of the meniscus. Due to this excess pressure,
the liquid will rise in the capillary tube till it becomes equal to the
hydrostatic pressure hpg. Thus in equilibrium state.
2T
Excess pressure = Hydrostatic pressure or = hpg
R
Let  be the angle of contact and r be the radius of the capillary tube shown in the fig.

OC r 2T cos 
From OAC, = cos  or R =  h =
OA cos  rg
This expression is called Ascent formula.
Discussion.
(i) For liquids which wet the glass tube or capillary tube, angle of contact  < 90°. Hence
cos  = positive.  h = positive. It means that these liquids rise in the capillary tube.
Hence, the liquids which wet capillary tube rise in the capillary tube. For example,
water, milk, kerosene oil, patrol etc.

(ii) For liquids which do not wet the glass tube or capillary tube, angle of contact  > 90°.
Hence cos  = negative h = negative. Hence, the liquids which do not wet capillary tube are
depressed in the capillary tube. For example, mercury.

1
(iii) T, ,  and g are constant and hence h  . Thus, the liquid rises more in a narrow tube and
r
less in a wider tube. This is called Jurin's Law.

(iv) If two parallel plates with the spacing 'd' are placed in water reservoir, then height of rise

 2T  = hdg
2T
h = dg

(v) If two concentric tubes of radius 'r 1' and 'r2' (inner one is solid) are placed in water reservoir,
then height of rise

 T [2r1 + 2r2] = [r22 h – r12h] g


2T
h = (r  r )g
2 1

KVPY_ SA STREAM # 626


(vi) If weight of the liquid in the meniscus is to be consider :
1 2
T cos × 2r = [r2h + r × r] g
3

 r 2T cos 
h  3  = rg
 

(vii) When capillary tube (radius, 'r') is in vertical position, the upper meniscus is concave and
pressure due to surface tension is directed vertically upward and is given by p 1 = 2T/R1 where
R1 = radius of curvature of upper meniscus.

The hydrostatic pressure p2 = h g is always directed downwards.


If p1 > p2 i.e. resulting pressure is directed upward. For equilibrium, the pressure due to lower
meniscus should be downward. This makes lower meniscus concave downward (fig.a). The
2T
radius of lower meniscus R2 can be given by R = (p1 – p2).
2
If p1 < p2, i.e. resulting pressure is directed downward for equilibrium, the pressure due to lower
meniscus should be upward. This makes lower meniscus convex upward (fig. b).
2T
The radius of lower meniscus can be given by R = p2 – p1.
2

2T
If p1 = p2, then is no resulting pressure. then, p1 – p2 = R = 0 or, R2 =  i. e. lower surface will
2
be FLAT. (fig.c).

(viii) Liquid between two Plates - When a small drop of water is placed between two glass
plates put face to face, it forms a thin film which is concave outward along its boundary. Let 'R'
and 'r' be the radii of curvature of the enclosed film in two perpendicular directions.

Hence the pressure inside the film is less than the atmospheric pressure outside it by an
1 1  T
amount p given by p = T    and we have. p = .
r  r
If d be the distance between the two plates and  the angle of contact for water and glass, then,
1
d
2 1 2 cos 
from the figure, cos  = or = .
r r d
1 2T
Substituting for in , we get p = cos .
r d

KVPY_ SA STREAM # 627


 can be taken zero for water and glass, i.e. cos  = 1. Thus the upper plate is pressed
2T
downward by the atmospheric pressure minus . Hence the resultant downward pressure
d
2T
acting on the upper plate is . If A be the area of the plate wetted by the film, the resultant
d
2TA
force F pressing the upper plate downward is given by F = resultant pressure × area = .
d
For very nearly plane surface, d will be very small and hence the pressing force F very large.
Therefore it will be difficult to separate the two plates normally.

Ex. 1 A liquid of specific gravity 1.5 is observed to rise 3.0 cm in a capillary tube of diameter 0.50 mm and the liquid
wets the surface of the tube. Calculate the excess pressure inside a spherical bubble of 1.0 cm diameter
blown from the same liquid. Angle of contact = 0º.
Sol. The surface tension of the liquid is
rhg
T=
2
(0.025 cm) (3.0 cm) (1.5 gm / cm3 ) (980 cm / sec 2 )
= = 55 dyne/cm.
2
Hence excess pressure inside a spherical bubble
4T 4  55 dyne / cm
p= = = 440 dyne/cm2 .
R (0.5 cm)

Ex. 2 A glass tube of circular cross-section is closed at one end. This end is weighted and the tube floats vertically
in water, heavy end down. How far below the water surface is the end of the tube? Given : Outer radius of the
tube 0.14 cm, mass of weighted tube 0.2 gm, surface tension of water 73 dyne/cm and
g = 980cm/sec2.

Sol. Let  be the length of the tube inside water. The forces acting on the tube are :

(i) Upthrust of water acting upward


22
= r2 × 1 × 980 = × (0.14)2  × 980 = 60.368  dyne.
7
(ii) Weight of the system acting downward
= mg = 0.2 × 980 = 196 dyne.
(iii) Force of surface tension acting downward
= 2rT
22
= 2× × 0.14 × 73
7
= 64.24 dyne.
Since the tube is in equilibrium, the upward force is balanced by the downward forces. That is,
60.368  = 196 + 64.24 = 260.24.
260.24
 = = 4.31 cm.
60.368

KVPY_ SA STREAM # 628


CAPILLARY RISE IN A TUBE OF INSUFFICIENT LENGTH

We know, the height through which a liquid rises in the capillary tube of radius r is given by
2T 2T
 h = Rg or h R = g = constant

When the capillary tube is cut and its length is less then h (i.e. h'),
then the liquid rises upto the top of the tube and spreads in such a way
that the radius (R') of the liquid meniscus increases and it becomes
more flat so that hR = h'R' = Constant. Hence the liquid does not
overflow.

If h' < h then R' > R


r r
or >
cos ' cos 
 cos ' < cos 
 ' > 

Ex. 3 If a 5 cm long capillary tube with 0.1 mm internal diameter open at both ends is slightly dipped in water having
surface tension 75 dyne cm–1, state whether (i) water will rise half way in the capillary. (ii) Water will rise up
to the upper end of capillary (iii) Water will overflow out of the upper end of capillary/ Explain your answer.
Sol. Given that surface tension of water, T = 75 dyne/cm
0.1
Radius r = mm = 0.05 mm = 0.005 cm,
2
density  = 1 gm/cm3, angle of contact,  = 0º.
Let h be the height to which water rise in the capillary tube. Then
2T cos  2  75  cos 0º
h= rg
= cm = 30.58 cm.
0.005  1 981
But length of capillary tube, h’ = 5 cm
h'
(i) Because h > therefore the first possibility does not exist.
2

(ii) Because the tube is of insufficient length therefore the water will rise upto the upper end of the tube.

(iii) The water will not overflow out of the upper end of the capillary. It will rise only upto the upper end of
the capillary.
The liquid meniscus will adjust its radius of curvature R’ in such a way that
 2T 
R’h’ = Rh  hR   constant 
 g 
where R is the radius of curvature that the liquid meniscus would possess if the capillary tube were
of sufficient length
Rh rh  r r 
 R’ = =  R   r
h' h'  cos  cos 0 º 
0.005  30 .58
= = 0.0306 cm
5

KVPY_ SA STREAM # 629


APPLICATIONS OF SURFACE TENSION

(i) The wetting property is made use of in detergents and waterproofing. When the detergent
materials are added to liquids, the angle of contact decreases and hence the wettability
increases. On the other hand, when water proofing material is added to a fabric, it increases
the angle of contact, making the fabric water-repellant.

(ii) The antiseptics have very low value of surface tension. The low value of surface tension prevents
the formation of drops that may otherwise block the entrance to skin or a wound. Due to low
surface tension the antiseptics spreads properly over the wound. The lubricating oils and paints
also have low surface tension. So they can spread properly.
(iii) Surface tension of all lubricating oils and paints is kept low so that they spread over a large area.

(iv) Oil spreads over the surface of water because the surface tension of oil is less than the surface
tension of cold water.

(v) A rough sea can be calmed by pouring oil on its surface.

EFFECT OF TEMPERATURE AND IMPURITIES ON SURFACE TENSION


The surface tension of a liquid decreases with the rise in temperature and vice versa. According to
n
 
Ferguson, T = T 0 1   where T 0 is surface tension at 0ºC,  is absolute temperature of the liquid,
 c 
c is the critical temperature and n is a constant varies slightly from liquid and has mean value 1.21.
This formula shows that the surface tension becomes zero at the critical temperature, where the
interface between the liquid and its vapour disappears. It is for this reason that hot soup tastes better
while machinery parts get jammed in winter.
The surface tension of a liquid changes appreciably with addition of impurities. For example, surface
tension of water increases with addition of highly soluble substances like NaCI, ZnSO 4 etc. On the
other hand surface tension of water gets reduced with addition of sparingly soluble substances like
phenol, soap etc.

SURFACE ENERGY

We know that the molecules on the liquid surface experience net downward force. So to bring a molecule
from the interior of the liquid to the free surface, some work is required to be done against the
intermolecular force of attraction, which will be stored as potential energy of the molecule on the
surface. The potential energy of surface molecules per unit area of the surface is called surface energy.
Unit of surface energy is erg cm –2 in C.G.S. system and Jm –2 in Sl system. Dimensional formula of
surface energy is [ML°T –2 ] Surface energy depends on number of surfaces e.g. a liquid drop is having
one liquid air surface while bubble is having two liquid air surface.

RELATION BETWEEN SURFACE TENSION AND SURFACE ENERGY

Consider a rectangular frame PQRS of wire, whose arm RS can slide on the arms PR and QS. If this
frame is dipped in a soap solution, then a soap film is produced in the frame PQRS in fig. Due to
surface tension (T), the film exerts a force on the frame (towards the interior of the film). Let  be the
length of the arm RS, then the force acting on the arm RS towards the film is. F = T × 2  [Since soap
film has two surfaces, that is why the length is taken twice].

KVPY_ SA STREAM # 630


Let the arm RS be displaced to a new position R'S' through a distance x

 work done, W = Fx = 2T x

Increase in potential energy of the soap film.

= EA = 2Ex = work done in increasing the area (W)

where E = surface energy of the soap film per unit area.


According the law of conservation of energy, the work done must be equal to the increase in the
potential energy
W
 2T  x = 2Ex or T = E =
A

Thus, surface tension is numerically equal to surface energy or work done per unit increase surface
area.

Ex. 4 A film of water is formed between two straight parallel wires each 10 cm long and at a separation 0.5 cm.
Calculate the work required to increase 1 mm distance between them.
Surface tension of water = 72 × 10–3 N/m.

10cm

0.5cm
Sol.
0.1cm

Here the increase in area is shown by shaded portion in the figure.


Since this is a water film, it has two surfaces, therefore
increase in area,  S = 2 × 10 × 0.1 = 2 cm2
 Work required to be done,
W = S × T
= 2 × 10–4 × 72 × 10–3
= 144 × 10–7 joule
= 1.44 × 10–5 joule

KVPY_ SA STREAM # 631


U N I T & DI M EN SI ON

PHYSICAL QUANTITIES :
The quantities which can be measured by an instrument and by means of which we can describe the
laws of physics are called physical quantities. Till class X we have studied many physical quantities
eg. length, velocity, acceleration, force, time, pressure, mass, density etc.

Physical quantities are of three types

Fundamental Derived Supplementary


or Quantities Quantities
Basic quantities

1. Fun d a m e nt a l ( Ba sic) Qu a nt it ie s :
 These are the elementary quantities which covers the entire span of physics.
 Any other quantities can be derived from these.
 All the basic quantities are chosen such that they should be different, that means indepen-
dent of each other. (i.e., distance (d) , time (t) and velocity (v) cannot be chosen as basic

d
quantities (because they are related as V = ). An International Organization named CGPM
t
: General Conference on weight and Measures, chose seven physical quantities as basic or
fundamental.

Length Time Mass Temperature Electrical Luminous Amount


(L) (T) (M) (K) current Intensity of
(A) (Cd) Substance
(mol)
These are the elementary quantities (in our planet) that’s why chosen as basic quantities.
In fact any set of independent quantities can be chosen as basic quantities by which all
other physical quantities can be derived.

i.e.,

Can be chosen as basic quantities (on some other planet, these might also be used as
basic quantities)

But (L) (A) (V)


Length Area Velocity

cannot be used as basic quantities as


Area = (Length) 2 so they are not independent.

KVPY_ SA STREAM # 632


2. D e rive d Qu a nt it ie s :
Physical quantities which can be expressed in terms of basic quantities (M,L,T....) are called derived
quantities.
i.e., Momentum P = mv
displaceme nt ML
= (m) = = M 1 L1 T – 1
time T
Here [ M1 L1 T – 1 ] is called dimensional formula of momentum , and we can say that momentum has
1 Dimension in M (mass)
1 Dimension in L (length)
and –1 Dimension in T (time)
The representation of any quantity in terms of basic quantities (M,L,T....) is called dimensional for-
mula and in the representation, the powers of the basic quantities are called dimensions.
3. Su pp le m e n t a ry qu a n t it ie s :
Besides seven fundamental quantities two supplementary quantities
are also defined. They are
 Plane angle (The angle between two lines) 
 Solid angle

FI N DI N G D I M EN SI ON S OF VARI OUS P HYSI CAL QUAN TI TIES :


 Height, width, radius, displacement etc. are a kind of length. So we can say that their dimension
is [L]
[Height]
[Width]
[radius] [L]
[displacement]
here [Height] can be read as “Dimension of Height”
 Area = Length × Width
So, dimension of area is [Area] = [Length] × [Width]
= [L] × [L]
= [L2]
For circle
Area = r2
[Area] = [] [r2]
= [1] [L2]
= [L2]
Here  is not a kind of length or mass or time so  shouldn’t affect the dimension of Area.
Hence its dimension should be 1 (M 0L0T 0) and we can say that it is dimensionless. From
similar logic we can say that all the numbers are dimensionless.

[200]
[-1]
0 0 0
[3] [M L T ] = 1
Dimensionless
 1
 
 2
 [Volume] = [Length] × [Width] × [Height]
= L × L× L = [L3]
For sphere

4 3 4 
Volume = r [Volume] =   [r3] = (1) [L3] = [L3]
3 3 
So dimension of volume will be always [L 3] whether it is volume of a cuboid or volume of
sphere.

KVPY_ SA STREAM # 633


Dimension of a physical quantity will be same, it doesn’t depend on which formula we
are using for that quantity.

mass
 Density =
volume

[mass] M
[Density] = = = [M1L– 3]
[ volume] L3

displaceme nt
 Velocity (v) =
time
[Displacement ] L
[v] = = = [M0L1T –1]
[ time] T

dv
 Acceleration (a) =
dt

LT 1
[a] = =  LT 2
T
 Momentum (P) = mv
[P] = [M] [v]
= [M] [LT –1]
= [M1L1T –1]

 Force (F) = ma
[F] = [m] [a]
= [M] [LT –2]
= [M1L1T –2] (You should remember the dimensions of force because it is used several times)

 Work or Energy = force × displacement


[Work] = [force] [displacement]
= [M1L1T –2] [L]
= [M1L2T –2]
work
 Power =
time

[ work ] M1L2 T 2
[Power] = = = [M1L2T – 3]
[ time] T
Force
 Pressure =
Area
[Force] M1L1T 2
[Pressure] = = = M 1L– 1T – 2
[ Area] L2

1. D im e n sions of a ngula r q ua n t it ie s :
 Angle ()
Arc
(Angular displacement)  =
radius

[ Arc ] L
[] = = = [M0L0T 0] (Dimensionless)
[radius] L

KVPY_ SA STREAM # 634


 [] 1
 Angular velocity () = [] = = = [M0L0T –1]
t [ t] T

d [ d] M0L0 T 1
 Angular acceleration () = [] = = = [M0L0T – 2]
dt [ dt ] T
 Torque = Force × Arm length
[Torque] = [force] × [arm length]
= [M1L1T –2] × [L] = [M1L2T –2]

2. D im e nsion s of Physica l Const a nt s :

 Gra vit a t iona l Con st a n t :

m1 r m2
Fg Fg

If two bodies of mass m 1 and m 2 are placed at r distance, both feel gravitational attraction
force, whose value is,
Gm1m 2
Gravitational force F g =
r2
where G is a constant called Gravitational constant.
[G][m1][m 2 ]
[F g ] =
[r 2 ]

[G][M][M]
[M1L1T – 2] =
[L2 ]
[G] = M– 1 L3 T – 2
 Spe cif ic he a t ca pa cit y :
To increase the temperature of a body by T, Heat required is Q = ms T
Here s is called specific heat capacity.
[Q] = [m] [s] [T]
Here Q is heat : A kind of energy so [Q] = M 1L2T –2
[M1L2T –2] = [M] [s] [K]
[s] = [M0L2T –2K–1]
 Ga s consta nt (R) :
For an ideal gas, relation between pressure (P)
Value (V) , Temperature (T) and moles of gas (n) is
PV = nRT where R is a constant, called gas constant.
[P] [V] = [n] [R] [T] ............ (1)

[Force]
here [P] [V] = [Area × Length]
[ Area]

= [Force] × [Length]
= [M1L1T –2] [L1] = M1L2T –2
From equation (1)
[P] [V] = [n] [R] [T]
 [M1L2T –2] = [mol] [R] [K]
 [R] = [M1L2T –2 mol –1 K–1]

KVPY_ SA STREAM # 635


 Coe f ficie nt of viscosit y :
If any spherical ball of radius r moves with velocity v in a viscous fv
liquid, then viscous force acting on it is given by
r
F v = 6rv
Here  is coefficient of viscosity r
[F v] = [6] [] [r] [v] V
M1L1T – 2 = (1) [] [L] [LT – 1]
[] = M1L– 1T – 1

 P la nck ’s const a n t :
If light of frequency  is falling , energy of a photon is given by
E = h Here h = Planck’s constant
[E] = [h] []
1 1  1
 = frequency =  [] = =  
Time Period [Time Period] T 
so M1L2 T – 2 = [h] [T –1]
[h] = M1L2T – 1
3. Som e spe cia l f e a t ure s of dim e nsions :
 Suppose in any formula, (L + ) term is coming (where L is length). As length can be added
only with a length, so  should also be a kind of length.
So [] = [L]
 Similarly consider a term (F – ) where F is force. A force can be added/subtracted with a
force only and give rises to a third force. So  should be a kind of force and its result (F –)
should also be a kind of force.

F–
a third force should be a kind of
and its dimension force  [ = M L T
1 1 –2
1 1 –2
will also be M L T

Rule N o. 1 : One quantity can be added / subtracted with a similar quantity only and give rise to
the similar quantity.

Example 1. For n moles of gas, Vander waal’s equation is

 a 
 P  2  (V – b) = nRT
 V 

Find the dimensions of a and b, where P is gas pressure, V = volume of gas T = tempera-
ture of gas
Solution :

KVPY_ SA STREAM # 636


[a]
So = M 1L– 1T – 2 So [b] = L3
[V 2 ]

[a]
= M–1 L–1 T – 2
[L ] 2

 [a] = M1 L5 T –2

Rule N o. 2 : Consider a term sin()

 Perpendicu lar 
Here  is dimensionless and sin   is also dimensionless.
 Hypoteneou s 
 Whatever comes in sin(......) is dimensionless and entire [sin (.......)] is also dimen-
sionless.

sin(- - -)
 dimensionless
dimensionless

Sim ila rly :

cos(- - -)
dimensionless
dimensionless

tan(- - -)
dimensionless
dimensionless

(- - -)
e

dimensionless dimensionless

loge(- - -)
dimensionless
dimensionless

KVPY_ SA STREAM # 637


Fv 2  2 
Example 2. = 2 loge  2  where F = force , v = velocity
  v 

Find the dimensions of  and .


Solution :

[2][]
 =1
[v 2 ]

[1][]
 =1
L2 T 2
 [] = L2T – 2

[F][ v 2 ] [M1L1T 2 ][L2 T 2 ]


as [] =  [] =  [] = M1L – 1 T0
[ 2 ] [L2 T 2 ] 2

4. USES OF DIMENSIONS :

 To che ck t he corre ct ne ss of the form ula :


If the dimensions of the L.H.S and R.H.S are same, then we can say that this eqn. is at
least dimensionally correct. So this equation may be correct.
But if dimensions of L.H.S and R.H.S is not same then the equation is not even dimension-
ally correct.
So it cannot be correct.

mv 2
e.g. A formula is given centrifugal force F e =
r
(where m = mass , v = velocity , r = radius)
we have to check whether it is correct or not.
Dimension of L.H.S is
[F] = [M1L1T -2]
Dimension of R.H.S is

[m] [ v 2 ] [M] [LT -1 ] 2


= = [M1L1T – 2]
[r ] [L]
So this eqn. is at least dimensionally correct.

thus we can say that this equation may be correct.

KVPY_ SA STREAM # 638


Example 3. Check whether this equation may be correct or not.

3 Fv 2
Solution : Pressure Pr = (where Pr = Pressure , F = force ,
2t 2 x
v = velocity , t = time , x = distance)
Dimension of L.H.S = [Pr] = M1L– 1T – 2

[3] [F] [v 2 ] [M1L1T -2 ] [L2 T -2 ]


Dimension of R.H.S = = = M1L2T – 6
[ ] [t 2 ] [x] [T 2 ] [L]
Dimension of L.H.S and R.H.S are not same. So the relation cannot be correct.
Sometimes a question is asked which is beyond our syllabus, then certainly it must be the
question of dimensional analyses.

Example 4. A Boomerang has mass m surface Area A, radius of curvature of


lower surface = r and it is moving with velocity v in air of den
sity . The resistive force on it should be –

2vA  m  2v 2 A  A 
(A) log   (B) log  
r 2
 Ar  r  m 

 Ar  2v 2 A  Ar 


(C*) 2v 2A log   (D) log  
 m  r 2  m 
Solution : Only C is dimensionally correct.

 W e ca n de rive a new form ula roughly :


If a quantity depends on many parameters, we can estimate, to what extent, the quantity
depends on the given parameters !

Example 5.

Time period of a simple pendulum can depend on

So we can say that expression of T should be in this form


T = (Some Number) (m) a ()b (g)c
Equating the dimensions of LHS and RHS,
M0L0T1 = (1) [M1]a [L1]b [L1T –2]c
M0L0T 1 = Ma Lb+c T – 2c
Comparing the powers of M,L and T,
get a = 0 , b + c = 0, – 2c = 1

KVPY_ SA STREAM # 639


1 1
so a= 0, b= , c = –
2 2
so T = (some Number) M 0 L1/2 g– 1/2

T = (Some Number) g

The quantity “Some number” can be found experimentally. Measure the length of a pendulum
and oscillate it, find its time period by stopwatch.
Suppose for  = 1m, we get T = 2 sec. so
1
2 = (Some Number) 9. 8
 “Some number” = 6.28  2.

Example 6.
Natural frequency (f) of a closed pipe

So we can say that f = (some Number) ( ) a ()b (P)c

 1
 T  = (1) [L] a [ML–3]b [M1L–1T–2]c
 
MºLºT –1 = Mb + c La – 3b – c T –2c
comparing powers of M, L, T
0 = b + c
0 = a – 3b – c
–1 = –2c
get a = – 1 , b = - 1/2 , c = 1/2

1 P
So f = (some number)
 

 W e ca n e xpre ss a ny qua nt it y in t erm s of t he give n ba sic qua nt itie s.

Example 7. If velocity (V), force (F) and time (T) are chosen as fundamental quantities , express (i) mass
and (ii) energy in terms of V,F and T
Solution :
Let M = (some Number) (V) a (F)b (T)c
Equating dimensions of both the sides

KVPY_ SA STREAM # 640


M1L0T0 = (1) [L1T–1]a [M1L1T - 2]b [T1]c
M1L0T 0 = Mb La + b T – a – 2b + c
get a = – 1, b = 1, c = 1
M = (Some Number) (V –1 F 1 T 1)  [M] = [V–1 F 1 T 1]
Similarly we can also express energy in terms of V , F , T
Let [E] = [some Number] [V] a [F] b [T] c
 [ML2T–2] = [MºLºTº] [LT –1]a [MLT –2] b [T] c

 [M1L2T –2] = [Mb La + b


T –a – 2b + c]

 1 = b; 2 = a + b ; – 2 = –a – 2b + c

get a =1 ; b = 1 ; c = 1

 E = (some Number) V 1F 1T 1 or [E] = [V1][F 1][T 1].

 To f ind out unit of a physica l qua nt it y :


Suppose we want to find the unit of force. We have studied that the dimension of force is
[Force] = [M1L1T–2]
As unit of M is kilogram (kg) , unit of L is meter (m) and unit of T is second (s) so unit of force can
be written as (kg) 1 (m)1 (s)– 2 = kg m/s2 in MKS system. In CGS system, unit of force can be written
as (g)1 (cm)1 (s)–2 = g cm/s2.
LIMITATIONS OF DIMENSIONAL ANALYSIS :

From Dimensional analysis we get T = (Some Number) g
so the expression of T can be

 
T =2 g T= g sin (.....)
or or

 
T = 50 g T= g log (......)
or or

 
T = 2 g T= g + (t 0)

 Dimensional analysis doesn’t give information about the “some Number” :The dimensional con-
stant.
 This method is useful only when a physical quantity depends on other quantities by multipli-
cation and power relations.
(i.e., f = x a yb zc)
It fails if a physical quantity depends on sun or difference of two quantities
(i.e.f = x + y – z)
i.e., we cannot get the relation
1 2
S = ut + at from dimensional analysis.
2
 This method will not work if a quantity depends on another quantity as sine or cosine
,logarithmic or exponential relation. The method works only if the dependence is by power
functions.

KVPY_ SA STREAM # 641


 We equate the powers of M,L and T hence we get only three equations. So we can have only
three variable (only three dependent quantities)
So dimensional analysis will work only if the quantity depends only on three parameters, not
more than that.

DIMENSIONS BY SOME STANDARD FORMULAE :-


In many cases, dimensions of some standard expression are asked
e.g. find the dimension of (µ 00)
for this, we can find dimensions of µ 0 and 0, and multiply them, but it will be very lengthy process.
Instead of this, we should just search a formula, where this term (µ 00 ) comes.
1 1
It comes in c = µ00 (where c = speed of light)  µ 0 0 =
c2

1 1
[µ 00] = = = L–2 T 2
c 2
(L / T )2

Example 8. Find the dimensions of

(i) 0E2 (0 = permittivity in vacuum , E = electric field)

B2
(ii) (B = Magnetic field , µ 0 = magnetic permeability)
µ0

1
(iii) (L = Inductance , C = Capacitance)
LC

(iv) RC (R = Resistance , C = Capacitance)

L
(v) (R = Resistance , L = Inductance)
R

E
(vi) (E = Electric field , B = Magnetic field)
B
(vii) G0 (G = Universal Gravitational constant , 0 = permittivity in vacuum )

e
(viii)
m
(e = Electrical flux ; m = Magnetic flux)

Solution :

1
(i) Energy density =  E2
2 0
[Energy density] = [0E2]

1
 E 2 [energy ] M1L2 T 2
2 0 =
 [ volume] = = M1L-1T –2
  L3

KVPY_ SA STREAM # 642


1 B2
(ii) = Magnetic energy density
2 0

1 B2 
  = [Magnetic Energy density]
 2  0 

 B2  [energy ] M1L2 T 2
 = = = M1L-1T –2
  0  [ volume] L3

1
(iii) = angular frequency of L – C oscillation
LC

 1  1
  = [] = = T –1
 LC  T

(iv) RC = Time constant of RC circuit = a kind of time


[RC] = [time] = T 1

L
(v) = Time constant of L – R circuit
R

L 
 R  = [time] = T
1
 

(vi) magnetic force F m = qvB , electric force F e = qE

 [F m] = [F e]
  [qvB] = [qE]

E 
 B  = [v] = LT
T –1
 

Gm2 1 q2
(vii) Gravitational force F g = , Electrostatic force F e =
r2 4 0 r 2

 Gm2   1 q2 
 2  =  2 
 r   4 0 r 

 q2   (it)2 
[G0] =  2  =  2  = A2T 2M–2
 m   m 

 e   ES   E 
(viii)  =  BS  =   = [v] (from part (vi)) = LT
T –1
 m    B 

KVPY_ SA STREAM # 643


Dimensions of quantities related to Electromagnetic and Heat (only for XII and XIII students)
(i) Charge (q) :
dq a small ch arg e flow
We know that electrical current i = =
dt small time interval

[dq]
[i]=
[ dt ]

[q]
[A] =  [q] = [ A1 T1 ]
t

(ii) Permittivity in Vacuum (0) :


k q1q2 1 q1q2
Electrostatic force between two charges Fe = 2 =
r 4 0 r 2

1 [q1][q2 ]
[Fe] = [ 4] [  ]
0 [r ] 2

1 [ AT ][ AT ]
M1 L1 T–2 = [0 ] = M–1 L–3 T4 A2
(1)[ 0 ] [L] 2

F
(iii) Electric Field (E) :  Electrical force per unit charge E =
q

[F] [M1L1T –2 ]
[E] = = = M1L1T–3A–1
[q] [ A 1 T1 ]

U
(iv) Electrical Potential (V) :Electrical potential energy per unit charge V =
q

[U] [M1L2 T –2 ]
[V] = = = M1L2 T–3A–1
[ q] [ A 1T1 ]

(v) Resistance (R) :


From Ohm’s law V = i R
[V] = [i] [R]
[M1L2T–3A–1 ] = [A1] [R]
[R] = M1 L2 T–3A–2
(vi) Capacitance (C) :

q [q] [ A 1T1]
C=  [C] = = [C] = M–1 L–2 T4 A2
V [ V ] [M1L2 T – 3 A 1 ]

(vii) Magnetic field (B) :


magnetic force on a current carrying wire Fm = i B [Fm] = [i] [] [B]
[M1L1T–2] = [A1] [L1] [B]
[B] = M1LOT–2A–1

(viii) Magnetic permeability in vacuum (µ0) :


F  o i1i2
Force /length between two wires =
 4 r 2

M1L1T –2 [ O ] [ A ][ A ]
=  [0] = M1L2T–2 A–2
L1 [ 4] [L] 2

KVPY_ SA STREAM # 644


(ix) Inductance (L) :
Magnetic potential energy stored in an inductor U =1/2 L i2
[U] = [1/2] [L] [i]2
[M1 L2 T–2] = (1) [L] (A)2
[L] = M1L2T–2 A–2

(x) Thermal Conductivity : 

dQ  dT 
Rate of heat flow through a conductor = A 
dt  dx 

[dQ] [ dT ] [M1L2 T –2 ] [K ]
= [k ] [A] = [ ] [L2] 1 [ ] = M1 L–1 T–3 K–1
[dt ] [ dx ] [T] [L ]

(xi) Stefan’s Constant () : 

dE
If a black body has temperature (T), then Rate of radiation energy emitted =  A T4
dt

[ dE] [M1L2 T –2 ]
= [] [A] [T4] = [] [L2] [K4] [] = [M1 Lo T–3 K–4]
[dt ] [T]

(xii) Wien’s Constant : 


b
Wavelength corresponding to max. spectral intensity . m = (where T = temp. of the black body)
T

[b ] [b]
[m] = [L] = [b] = [L1K1]
[T] [K]

UN I T :

 Un it :
Measurement of any physical quantity is expressed in terms of an internationally accepted
certain basic standard called unit.

 SI Unit s :
In 1971 , an international Organization “CGPM” : (General Conference on weight and Measure)
decided the standard units, which are internationally accepted. These units are called SI units
(International system of units)

KVPY_ SA STREAM # 645


1. SI Unit s of Ba sic Qua nt it ies :

S Units
Base Quantity
Name Symbol Definition

The metre is the length of the path traveled by light in


Length metre m vacuum during a time interval of 1/299, 792, 458 of a second
(1983)

The kilogram is equal to the mass of the international


prototype of the kilogram (a platinum-iridium alloy cylinder)
Mass kilogram kg
kept at International Bureau of Weights and Measures, at
Sevres, near Paris, France. (1889)
The second is the duration of 9, 192, 631, 770 periods of the
radiation corresponding to the transition between the two
Time second s
hyperfine levels of the ground state of the cesium-133 atom
(1967)

The ampere is that constant current which, if maintained in


two straight parallel conductors of infinite length, of negligible
Electric Current ampere A circular cross-section, and placed 1 metre apart in vacuum,
will produce between these conductors a force equal to 2 x
10-7 Newton per metre of length. (1948)

Thermodynamic The kelvin, is the fraction 1/273.16 of the thermodynamic


kelvin K
Temperature temperature of the triple point of water. (1967)

The mole is the amount of substance of a system, which


Amount of
mole mol contains as many elementary entities as there are atoms in
Substance
0.012 kilogram of carbon-12. (1971)

The candela is the luminous intensity, in a given direction, of


Luminous a source that emits monochromatic radiation of frequency
candela cd
Intensity 540 x 1012 hertz and that has a radiant intensity in that
direction of 1/683 watt per steradian (1979).

2. Tw o su p p le m e n t a r y u n it s w e re a lso d e f in e d :
 Plane angle – Unit = radian (rad)
 Solid angle – Unit = Steradian (sr)

3. Ot h e r cl a ssi f i ca t io n :
If a quantity involves only length, mass and
time (quantities in mechanics), then its unit
can be written in MKS, CGS or FPS system.

 For M KS system :
In this system Length, mass and time are expressed in meter, kg and second. respectively.
It comes under SI system.
 For CGS syst em :
In this system ,Length, mass and time are expressed in cm, gram and second. respectively.
 For FPS system :
In this system, length, mass and time are measured in foot, pound and second. respectively.

KVPY_ SA STREAM # 646


4. SI unit s of de rive d Qua nt it ie s :

 Velocity =
So unit of velocity will be m/s

change in velocity m/ s m
 Acceleration = = = 2
time s s
 Momentum = mv
so unit of momentum will be = (kg) (m/s) = kg m/s
 Force = ma
Unit will be = (kg) × (m/s2) = kg m/s2 called newton (N)
 Work = FS
unit = (N) × (m) = N m called joule (J)

work
 Power =
time
Unit = J / s called watt (W)

5. Un i t s of som e p h ysica l Co n st a n t s :
 Unit of “Universal Gravitational Constant” (G)
G(m1 )(m 2 ) kg  m G(kg)(kg)
F=  =
r 2
s2 m2

m3
so unit of G =
kg s2

 Unit of specific heat capacity (s) :

Q = ms T J = (kg) (S) (K) Unit of s = J / kg K

 Unit of  0 :

F 0 i1 i2
force per unit length between two long parallel wires is: =
 4 r2

N 0 ( A ) (A) N.m
= Unit of 0 =
m (1) (m 2 ) A2

6. SI Pre fix :
Suppose distance between kota to Jaipur is 3000 m. so
d = 3000 m = 3 × 1000 m
kilo(k)
= 3 km (here ‘k’ is the prefix used for 1000 (10 3))
Suppose thickness of a wire is 0.05 m
d = 0.05 m = 5 × 10-2 m

centi(c)
= 5 cm (here ‘c’ is the prefix used for (10 –2))

Similarly, the magnitude of physical quantities vary over a wide range. So in order to express the
very large magnitude as well as very small magnitude more compactly, “CGPM” recommended some
standard prefixes for certain power of 10.

KVPY_ SA STREAM # 647


Power of 10 Prefix Symbol Power of 10 Prefix Symbol

1018 exa E 101 deci d


15 peta P 2 centi c
10 10
12 tera T 3 milli m
10 10
6
10 9 giga G 10 micro 
6 mega M 9 nano n
10 10
3 kilo K 12 pico p
10 10
2 hecto h 15 femto f
10 10
1 deca da 18 atto a
10 10

m3
Example 9. G = 6.67 × 10 – 11
convert it into CGS system.
kg s2

m3
Solution : G = 6.67 × 10 – 11
kg s2

(100 cm)3 cm3


= (6.67×10 )
–11
= 6.67 × 10 – 8
(1000 g) s 2 g s2

P OI N T TO REM EM BER
5
To convert km/hour into m/sec, multiply by .
18
Some SI units of derived quantities are named after the scientist, who has contributed in that field a
lot.

KVPY_ SA STREAM # 648


8. SI D e rive d unit s, na m e d a f t e r t he scient ist :

SI Units
S.N Physical Quantity Symbol of the Expression in terms Expression in terms
Unit name
unit of other units of base units

Frequency
Oscillatio n -1
1. 1 hertz Hz s
(f = ) s
T

Force 2
2. newton N ----- Kg m / s
(F = ma)

Energy, Work, Heat 2 2


3. joule J Nm Kg m / s
(W = Fs)

Pressure, stress
2 2
4. F pascal Pa N/m Kg / m s
(P = )
A

Power,
2 3
5. W watt W J/s Kg m / s
(Power = )
t

Electric charge
6. coulomb C ------ As
(q = it)

Electric Potential
Emf. 2 3
7. U volt V J/C Kg m / s A
(V = )
q

Capacitance
2 4 2
8. q farad F C/V A s / kgm
(C = )
v

Electrical Resistance 2 3 2
9. ohm  V/A kg m / s A
(V = i R)

Electrical
Conductance siemens 3 2 2
10. S, A/V s A / kg m
1 i (mho)
(C = = )
R V

11. 2 2 1
Magnetic field tesla T Wb / m Kg / s A

2 2
kg m / s
12. Magnetic flux weber Wb V s or J/A 1
A
2 2
kg m / s
13. Inductance henry H Wb / A 2
A

Activity of Di sin tegration -1


14. becquerel Bq s
radioactive material sec ond

KVPY_ SA STREAM # 649


9. Som e SI units expressed in term s of the specia l na m es a nd a lso in term s of
b a se u nit s:
SI Units
Physical Quantity In terms of special
In terms of base units
names

2 2
Torque (  = Fr) Nm Kg m / s

Dynamic Viscosity
dv Poiseiulle (P  ) or Pa s Kg / m s
(Fv =  A )
dr

Impulse (J = F  t) Ns Kg m / s

Modulus of elasticity
2 2
stress N/m Kg / m s
(Y = )
strain

Surface Tension Constant (T)


2 2
F N/m or J/m Kg / s
(T = )

J/kg K
Specific Heat capacity (s) cal 2 -2 -1
(old unit s ) m s K
(Q = ms  T)
g. º C

Thermal conductivity (K)


-3 -1
dQ dT W/mK m kg s K
( = KA )
dt dr

F -3 -1
Electric field Intensity E = V/m or N/C m kg s A
q

Gas constant (R) (PV = nRT) or


molar Heat Capacity 2 -2 -1 -1
J / K mol m kg s K mol
Q
(C = )
MT

CHAN GE OF NUM ERICAL VALUE W I TH THE CHAN GE OF UN IT :


Suppose we have
If we convert 7
 = 7 cm  = m
it into metres, we get 100
we can say that if the unit is increased to 100 times (cm  m),

1  7 
the numerical value became times  7  
100  100 
So we can say

1
Numerical value 
unit
We can also tell it in a formal way like the following :–

KVPY_ SA STREAM # 650


Magnitude of a physical quantity = (Its Numerical value) (unit)
= (n) (u)

Magnitude of a physical quantity always remains constant ,it


will not change if we express it in some other unit.

So

1
numerical value 
unit

Example 10. Force acting on a particle is 5N.If unit of length and time are doubled and unit of mass is
halved than the numerical value of the force in the new unit will be.
Solution :
kg  m
Force = 5
sec 2
If unit of length and time are doubled and the unit of mass is halved.

1 
 2
Then the unit of force will be 
2  = 1 times
2
 (2)  4
 
 

1
Hence the numerical value of the force will be 4 times. (as numerical value  )
unit

KVPY_ SA STREAM # 651


KVPY PROBLEMS (PREVIOUS YEARS)
1. The dimensions of stefan-Boltzmann constant  can be written in terms of Planck’s constanth’ Boltzmann
constant KB and the speed of light c as  = h KB C. Here [KVPY_2014_SB] [2 Marks]

(A)  = 3,  = 4 and  = -3 (B)  = 3,  = -4 and  = 2


(C)  = -3,  = 4 and  = -2 (D)  = 2,  = -3 and  = -1

2. In an experiment, mass of an object is measured by applying a known force on it, and then measuring
its acceleration. IF, in the experiment, the measured values of applied force and the measured acceleration
are F  10.0  0.2 N and a  1.00  0.01 m/s2 , respectively, the mass of the object is
[KVPY_2015_SA] [1 Marks]
(A) 10.0 Kg (B) 10.0  0.1 Kg (C) 10.0  0.3 Kg (D) 10.0  0.4Kg

3. Stoke’s law states that the viscous drag force F experience by a sphere of radius a, moving with a
speed v through a fluid with coefficient of viscosity  , is given by F  6av .
If this fluid is flowing through a cylindrical pipe of radius r, length l and a pressure difference of P across its
two ends, then the volume of water V which flows through the pipe in time t can be written as
a
v  p
 k   br c
t l 
Where k is a dimensionless constant. Correct values of a,b and c are [KVPY_2015_SA] [2 Marks]
(A) a = 1, b = – 1, c = 4 (B) a = – 1, b = 1, c = 4
(C) a = 2, b = – 1, c = 3 (D) a = 1, b = – 2, c = – 4

4. The dimensions of the area A of a black hole can be written in terms of the universal constant G, its
mass M and the speed of light c as A  G  M  c  . Here [KVPY_2015_SB] [1 Mark]

(A)   2,   2, and   4 (B)   2,   2, and   4


(C)   3,   3, and   2 (D)   3,   3, and   2

5. The length and width of a rectangular room are measured to be 3.95  0.05 m and 3.05 0.05 m, respec-
tively. The area of the floor is [KVPY_2016_SA] [1 Mark]
(A) 12.05  0.01 m2. (B) 12.05  0.005m2
(C) 12.05  0.34 m2. (D) 12.05  0.40 m2.

6. Stoke’s law states that the viscous drag force F experienced by a sphere of radius a, moving with a speed
 through a fluis with coefficient of viscosity  , is given by F= 6a . If the fluid is flowing through a
cylindrical pipe of radius r, length l, and the pressure different of P across its two ends, then the volume of
a
 p b c
water V which flows through the pipe in time t can be written as  k    r
t l
Where k is a dimensionless constant. Correct values of a, b and c are [KVPY_2017_SA] [2 Mark]
(A) a = 1, b = –1, c = 4 (B) a = –1, b = 1, c = 4 (C) a= 2, b = –1, c = 3 (D) a= 1, b=–2, c=–4
   
7. Force F applied on a body is written as F  (nˆ .Fˆ ) nˆ  G , where n̂ is a unit vector. The vector G is equal to
[KVPY_2017_SB] [1 Mark]
     
(A) n̂  F (B) nˆ  (nˆ  F ) (C) ( nˆ  F )  F / | F | (D) ( nˆ  F )  nˆ

KVPY_ SA STREAM # 652


8. A neutron star with magnetic moment of magnitude m is spinning with angular velocity  about its magnetic
axis. The electromagnetic power P radiated by its is given by  0x m y  z c u , where 0 and c are the permeability
and speed of light in free space, respectively. Then [KVPY_2017_SB] [2 Mark]

(A) x = 1, y = 2, z = 4 and u = –3 (B) x = 1, y = 2, z = 4 and u = 3

(C) x = –1, y = 2, z = 4 and u = –3 (D) x = –1, y = 2, z = 4 and u = 3

EXERCISE
1. (C) 2. (C) 3. (A) 4. (B) 5. (C) 6. (A) 7. (D)

8. (A)

KVPY_ SA STREAM # 653

You might also like